You are on page 1of 374

F R O M T H E E X P E RT S I N E N D O C R I N O L O G Y

MEET-THE-PROFESSOR

2016
2016 MEET-THE-PROFESSOR ENDOCRINE CASE MANAGEMENT
2016 Meet-the-Professor Endocrine Case Management is your source for the
latest updates in the diagnosis and management of a wide range of endocrine
disorders. This valuable resource allows you to evaluate your endocrine
ENDOCRINE
CASE MANAGEMENT
knowledge and gain insight into the strategies used by clinical experts.

Features of Meet-the-Professor Endocrine Case Management 2016 include:

Historical Overview C E L E B R A T I N G A C E N T U R Y O F


Significance of the Clinical Problem
Key Learning Objectives
E N D O C R I N O L O G Y
Strategies for Diagnosis and Management
Clinical Pearls and Pertinent References
Cases and Questions

Endocrine Society
2055 L Street NW, Suite 600
Washington, DC 20036
endocrine.org

The Endocrine Society. Downloaded from press.endocrine.org by [${individualUser.displayName}] on 12 January 2017. at 12:19 For personal use only. No other uses without permission. . All rights reserved.

MTP 2016_COVER.Final.R1_2-18-16.indd 1 2/29/16 3:15 PM


The Endocrine Society. Downloaded from press.endocrine.org by [${individualUser.displayName}] on 12 January 2017. at 12:19 For personal use only. No other uses without permission. . All rights reserved.
2055 L Street, NW, Suite 600
Washington, DC 20036
www.endocrine.org

Other Publications:
press.endocrine.org

The Endocrine Society is the worlds largest, oldest, and most active organization working to advance the clinical practice of
endocrinology and hormone research. Founded in 1916, the Society now has more than 18,000 global members across a range
of disciplines.

The Society has earned an international reputation for excellence in the quality of its peer-reviewed journals, educational
resources, meetings, and programs that improve public health through the practice and science of endocrinology.

Clinical Science Chair, ENDO 2016


Gary D. Hammer, MD, PhD

Physician-In-Practice Chair, ENDO 2016


Michael T. McDermott, MD

The statements and opinions expressed in this publication are those of the individual authors and do not necessarily reflect the
views of the Endocrine Society. The Endocrine Society is not responsible or liable in any way for the currency of the
information, for any errors, omissions or inaccuracies, or for any consequences arising therefrom. With respect to any drugs
mentioned, the reader is advised to refer to the appropriate medical literature and the product information currently provided by
the manufacturer to verify appropriate dosage, method and duration of administration, and other relevant information. In all
instances, it is the responsibility of the treating physician or other health care professional, relying on independent experience
and expertise, as well as knowledge of the patient, to determine the best treatment for the patient.

Copyright 2016 by the Endocrine Society, 2055 L Street, NW, Suite 600, Washington, DC 20036. All rights reserved. No
part of this publication may be reproduced, stored in a retrieval system, posted on the internet, or transmitted in any form, by
any means, electronic, mechanical, photocopying, recording, or otherwise, without written permission of the publisher.

Requests for permission for reproduction should be directed to the Endocrine Society Publications Department: http://www.endo-
society.org/journals/rights.cfm, or send an email to permissions@endo-society.org. For more information or to purchase copies, please
contact Society Services by telephone at 202-971-3646, fax at 202-736-9704, or email at societyservices@endocrine.org or visit the
online store: www.endocrine.org/store.

ISBN: 978-1-943550-02-9
eISBN: 978-1-943550-04-3
Library of Congress Control Number: 2016932453

The Endocrine Society. Downloaded from press.endocrine.org by [${individualUser.displayName}] on 12 January 2017. at 12:19 For personal use only. No other uses without permission. . All rights reserved.
ENDO 2016 CONTENTS iii

ENDO 2016
CONTENTS
DISCLOSURE INDEX ......................................................................................... iv

FOREWORD......................................................................................................... v

TOPIC INDEX ..................................................................................................... vi

SPEAKER HANDOUT INDEX ............................................................................... xi

ADRENAL HPA AXIS .......................................................................................... 1

BONE, CALCIOTROPIC HORMONES, AND VITAMIN D ...................................... 47

DIABETES AND GLUCOSE METABOLISM ......................................................... 83

GENERAL ENDOCRINOLOGY .......................................................................... 143

NEUROENDOCRINOLOGY AND PITUITARY ...................................................... 177

OBESITY AND LIPIDS...................................................................................... 217

PEDIATRIC ENDOCRINOLOGY.......................................................................... 249

REPRODUCTIVE ENDOCRINOLOGY .................................................................. 295

THYROID/HPT AXIS ....................................................................................... 329

The Endocrine Society. Downloaded from press.endocrine.org by [${individualUser.displayName}] on 12 January 2017. at 12:19 For personal use only. No other uses without permission. . All rights reserved.
iv ENDO 2016 DISCLOSURE INDEX

ENDO 2016
DISCLOSURE INDEX
The faculty, committee members, and staff who are in position to control Wang, C: Advisory Board Member and Consultant, Novo Nordisk (spouse);
the content of this activity are required to disclose to the Endocrine Society Advisory Board Member, CSL Behring, Biogen, Baxter (spouse); Medical
and to learners any relevant financial relationship(s) of the individual or Safety Officer-CPC Clinical Research/University of Colorado, Astra
spouse/partner that have occurred within the last 12 months with any Zeneca, Osiris; Speaker, Medical Education Resources. Matsumoto, A:
commercial interest(s) whose products or services are related to the CME Principal Investigator, Abbott Laboratories, GlaxoSmithKline; Advisory
content. Financial relationships are defined by remuneration in any amount Group Member, Abbott Laboratories, Clarus; Ad Hoc Consultant, Endo
from the commercial interest(s) in the form of grants; research support; Pharmaceuticals, Lilly USA, LLC. McCall, A: Clinical Researcher for
consulting fees; salary; ownership interest (e.g., stocks, stock options, or ELIXA Trial (monies paid to university); Clinical Researcher, Lilly USA,
ownership interest excluding diversified mutual funds); honoraria or other LLC; Consultant, Pfizer, Inc. McIver, B: Consultant, Veracyte, Inc.,
payments for participation in speakers bureaus, advisory boards, or boards Asuragen, CBL Path. Nieman, L: Investigator, HRA Pharma. Pinkerton, J:
of directors; or other financial benefits. The intent of this disclosure is not to Consultant, Pfizer, Inc., Noven, Inc., Therapeutic; Investigator,
prevent CME planners with relevant financial relationships from planning or TherapeuticMD. Riddell, M: Speaker, Eli Lilly & Company, Sanofi,
delivery of content, but rather to provide learners with information that Medtronic Minimed. Seaquist, E: Principal Investigator, Amgen, Lilly
allows them to make their own judgments of whether these financial USA, LLC, Merck & Co.; Investigator, Eli Lilly & Company, locemia;
Consultant, locemia; Speaker, Sanofi, Novo Nordisk. Shane, E: Principal
relationships may have influenced the educational activity with regard to
Investigator, Amgen, Lilly USA, LLC, Merck & Co. Trainer, P: Principal
exposition or conclusion.
Investigator, Ipsen, Antisense Therapeutics, Chiasma; Advisory Board
Member, Ipsen, Chiasma. Vella, A: Investigator, GI Dynamics. Verbalis, J:
The Endocrine Society has reviewed all disclosures and resolved or man-
Ad Hoc Consultant, Ferring Pharmaceuticals, Otsuka. Vinik, A: Consultant,
aged all identified conflicts of interest, as applicable.
Merck & Co., Pfizer, Inc., Neurometrix, Pamlab, Principal Investigator,
Tercica, Intarcia, ViroMed, Impeto Medical, Novo Nordisk VeroScience.
The following faculty reported relevant financial relationship, as identi-
Vogiatzi, M: Advisory Group Member, Novo Nordisk. Watts, N: Advisory
fied below: Aronne, L: Grantee, Eisai, Aspire Bariatric; Scientific Board
Group Member, Amgen, Merck & Co.; Scientific Board Member, Abbvie;
Member, Pfizer, Inc. Novo Nordisk, GI Dynamics, Jovia Health, Zafgen,
Speaker Bureau Member, Amgen; Investigator, NPS; Ad Hoc Consultant,
Gelesis; Board Member, Jamieson Labs, MYOS Corp. Auchus, R: Consul-
Sanofi. Young, W: Consultant, Nihon Medi-Physics.
tant, Bluebird Bio. Blonde, L: Principal Investigator, Eli Lilly & Company;
Investigator, Novo Nordisk, Sanofi; Speaker, Astra Zeneca, Jansen Pharma- The following faculty reported no relevant financial relationships:
ceuticals; Consultant, Merck & Co. GlaxoSmithKline, Intarcia. Davis, S:
Lecturer, Abbott Laboratories; Investigator, Trimel Pharmaceuticals, Adler, G; Alter, C; Arafah, B; Bachrach, L; Bantle, J; Barbour, L; Becker,
Lawley Pharmaceuticals. Feingold, K: Speaker, Merck & Co., Amgen, C; Boh, B; Bollerslev, J; Brent, G; Burch, H; Busui, R; Cappola, A;
Sanofi, Regeneron. Feldt-Rasmussen, U: Principal Investigator, Ad Hoc Chamberlain, A; Cornier, M-A; Donahoo, W; Edelman, S; Else, T; Farooki,
Consultant, Novo Nordisk; Advisory Group Member, Pfizer, Inc. Francis, G: A; Farwell, A; Findling, J; Fishbein, L; Habra, MA; Hall, J; Hennessey, J;
Stockholder, Genentech, Inc., Merck & Co.; Principal Investigator, Glaxo- Ho, K; Hodak, S; Holick, M; Jonklaas, J; Kiseljak-Vassiliades, K; Krone,
SmithKline, Novo Nordisk. Gafni, R: Investigator, Shire. Gagel, R: Princi- N; Lopes-Virella, M; McDonnell, M; McMahon, G; Molitch, M; Neggers,
pal Investigator, AstraZeneca. Haugen, B: Investigator, Speaker, Genzyme S; ODorisio, T; ystese, K; Pattison, D; Pearce, E; Pettifor, J; Radovick, S;
Corporation. Husebye, E: Speaker, Shire. Inzucchi, S: Advisory Group Rothman, M; Sabra, M; Safer, J; Sarapura, V; Saunders, K; Schauer, I;
Member, Merck & Co., Jansen Pharmaceuticals; Committee Member, Novo Sheffield-Moore, M; Shukla, A; Silverberg, S; Sparks, L; Sperling, M; Stan,
Nordisk; Speaker, Astra Zeneca, Coinvestigator, Takeda. Kerr, J: Re- M; Urban, R; Vaidya, A; Vincent, A; Wierman, M.
searcher, Novartis Pharmaceuticals. Lane, W: Advisory Group Member,
Novo Nordisk; Investigator, Eli Lilly & Company, Novo Nordisk. Low Endocrine Society staff report no relevant financial relationships.

The Endocrine Society. Downloaded from press.endocrine.org by [${individualUser.displayName}] on 12 January 2017. at 12:19 For personal use only. No other uses without permission. . All rights reserved.
ENDO 2016 FOREWORD v

ENDO 2016
FOREWORD
For 100 years the Endocrine Society has been devoted to educating its
members. According to Albert Einstein, Intellectual growth should com-
mence at birth and cease only at death. The field of endocrinology is
constantly evolving, and as physicians, we want to provide the best
possible care to our patients. To do this, we must engage in lifelong
learning. The Meet-The-Professor Endocrine Case Management book is
designed to provide the clinician with an efficient and high-quality review
of over 70 common and rare endocrine disorders. The expert contributors
have been selected for their deep understanding of the topic, their clinical
experience, and their facility in clearly communicating complex information.

Physicians learn best by active participation in case-based clinical


discussions. When physicians are actively engaged the learning is most
enjoyable and enduring. To this end, we have asked each expert contribu-
tor to provide a historical perspective about the topic, learning objectives,
a concise up-to-date review of their topic, and a summary of cases
followed by a brief discussion of the cases. This format allows the reader
to learn actively by testing their knowledge of clinical endocrinology.

We are deeply grateful to the many experts who contributed to these


sessions and made this book such a valuable learning experience. We are
also thankful to the Endocrine Society staff whose efforts are instrumental
to the successful publication of this book.

Gary D. Hammer, MD, PhD


Clinical Science Chair, ENDO 2016
University of Michigan
Ann Arbor, Michigan 48109

Michael T. McDermott, MD
Physician-In-Practice Chair, ENDO 2016
University of Colorado
Aurora, Colorado 80045

The Endocrine Society. Downloaded from press.endocrine.org by [${individualUser.displayName}] on 12 January 2017. at 12:19 For personal use only. No other uses without permission. . All rights reserved.
vi ENDO 2016 TOPIC INDEX

ENDO 2016
TOPIC INDEX
ADRENAL/HPA AXIS
CMF4 Adrenal Insufficiency, Subclinical and Adrenal Fatigue Page 2
Richard J. Auchus, MD, PhD and James W. Findling, MD

CMF10 Evaluation and Management of the Adrenal Mass Page 10


Anand Vaidya, MD, MMSc and Mouhammed Amir Habra, MD

M01 Management of Classical CAH: From Birth to Adulthood Page 17


Maria G. Vogiatzi, MD

M11 Adrenal Insufficiency: Individualized Management Page 21


Eystein S. Husebye, MD, PhD

M12 Pheochromocytomas and Paragangliomas Page 27


Lauren Fishbein, MD, PhD

M33 Adrenal Insufficiency in Intensive Care Patients Page 32


Baha M. Arafah, MD, FACP

M46 Primary Aldosteronism Page 36


William F. Young, Jr, MD, MSc

M53 Primary Aldosteronism and Cardiometabolic Risk: Approach to Medical Management Page 42
Gail K. Adler, MD, PhD

BONE, CALCIOTROPIC HORMONES, AND VITAMIN D


M02 Applying DXA and Other Imaging to Clinical Conundrums Page 48
Micol S. Rothman, MD

M13 Cancer Treatment and Bone Health Page 51


Azeez Farooki, MD

M23 Hyperparathyroidism Management after Unsuccessful Parathyroid Surgery Page 55


Shonni J. Silverberg, MD

M34 Rickets Page 59


John M. Pettifor, MD, PhD

M35 Osteoporosis in Premenopausal Women Page 64


Elizabeth Shane, MD

M47 Osteoporosis: Managing Patients Who Fracture on Osteoporosis Treatment Page 70


Carolyn B. Becker, MD

The Endocrine Society. Downloaded from press.endocrine.org by [${individualUser.displayName}] on 12 January 2017. at 12:19 For personal use only. No other uses without permission. . All rights reserved.
ENDO 2016 TOPIC INDEX vii

M51 Osteoporosis Drug Holidays: Data and Opinions Page 75


Nelson B. Watts, MD

M55 Vitamin D Replacement in Patients with Malabsorption Disorders Page 80


Michael F. Holick, MD, PhD

DIABETES AND GLUCOSE METABOLISM


CMF7 ADA and AACE Guidelines for Individualized Diabetes Management n/a
Lawrence Blonde, MD, FACP, FACE and Silvio E. Inzucchi, MD

M03 Using CGM For Day-To-Day Insulin-Dosing Decisions Page 84


Steven Edelman, MD

M10 When and How to Use U500 (or Other Concentrated) Insulin Page 89
Wendy Lane, MD

M14 Exercise Resistance in Type 2 Diabetes Page 96


Lauren M. Sparks, PhD

M15 Diabetic Neuropathies Page 101


Rodica Pop-Busui, MD, PhD

M21 Inpatient Management of Hyperglycemia Page 105


Cecilia C. Low Wang, MD

M22 Management of Hypoglycemia and Impaired Awareness of Hypoglycemia


in Diabetes Page 110
Elizabeth R. Seaquist, MD

M24 Exercise Prescriptions for Patients With DM2 Page 113


Irene Schauer, MD, PhD

M36 Controversies and Consequences of Gestational Diabetes for Mother and Child:
More than Glucose Page 118
Linda A Barbour, MD, MSPH

M37 Management of DM1 in Athletes Page 125


Michael C. Riddell, PhD

M54 Individualizing Management with Insulin Pumps Page 129


Anthony L. McCall, MD, PhD

M56 Diabetes in the Older Patient Page 138


Graham T. McMahon, MD, MMSc

GENERAL ENDOCRINOLOGY
CMF1 Endocrine Tumor Genetics: Challenging Issues Page 144
Tobias Else, MD and Mrta Korbonits, MD, PhD

The Endocrine Society. Downloaded from press.endocrine.org by [${individualUser.displayName}] on 12 January 2017. at 12:19 For personal use only. No other uses without permission. . All rights reserved.
viii ENDO 2016 TOPIC INDEX

M04 Endocrine Consequences of Opiate Therapy Page 153


Ken Ho, MD, FRACP

M16 Neurohumoral Syndromes Page 157


Aaron Vinik, MD, PhD, FCP, MACP, FACE

M31 Hypoglycemic Disorders Page 167


Adrian Vella, MD

M32 Whats New in the Management of Multiple Endocrine Neoplasia Type 2? Page 172
Robert F. Gagel, MD

NEUROENDOCRINOLOGY AND PITUITARY


CMF6 Acromegaly: Navigating the Difficult Cases Page 178
Katja Kiseljak-Vassiliades, DO and Peter Trainer, MD, FRCP

CMF8 Molecular Imaging and Radionuclide Therapy of Functional Neuroendocrine


Tumors (NETS) Page 184
David Pattison, MBBS, MPH, FRACP, FAANMS and Thomas ODorisio, MD

CMF11 ACTH-Dependent Cushings Syndrome: Challenging Cases Page 191


Lynnette K. Nieman, MD and Janice M. Kerr, MD

M06 Diabetes Insipidus: Principles of Diagnosis and Treatment Page 195


Joseph G. Verbalis, MD

M26 Non-Functioning Pituitary Adenomas and Incidentalomas Page 202


Jens Bollerslev, MD, DMSc and Kristin Astrid Berland ystese, MD

M44 Growth Hormone DeficiencyControversies in the Clinical Management


in Adults Page 206
Ulla Feldt-Rasmussen, MD, DMSc

M48 Traumatic Brain Injury and Fatigue Page 210


Albert Chamberlain, MD, Randall J. Urban, MD, and Melinda Sheffield-Moore, PhD

M57 Prolactinomas Page 214


Mark E. Molitch, MD

OBESITY AND LIPIDS


M05 Bariatric Surgery Nutritional Management Page 218
William Troy Donahoo, MD

M17 Diet and Exercise Recommendations in Metabolic Disease: Implementing


Effective Behavior Change Page 222
Marie E. McDonnell, MD

M18 Post-Bariatric Surgery Hypoglycemia Page 227


John P. Bantle, MD

The Endocrine Society. Downloaded from press.endocrine.org by [${individualUser.displayName}] on 12 January 2017. at 12:19 For personal use only. No other uses without permission. . All rights reserved.
ENDO 2016 TOPIC INDEX ix

M25 Advanced Lipoprotein Analysis n/a


Kenneth Feingold, MD

M38 Hypertriglyceridemia and Low HDL Page 232


Maria F. Lopes-Virella, MD, PhD

M39 Obesity Medications Page 239


Katherine H. Saunders, MD, Alpana P. Shukla, MD, MRCP, and Louis J. Aronne, MD, DABOM, FTOS

M52 Management of Severe Hypercholesterolemia Page 244


Marc-Andre Cornier, MD

PEDIATRIC ENDOCRINOLOGY
M09 Thyroid Nodules and Cancer Guidelines in Children Page 250
Gary L. Francis, MD, PhD

M08 Hypoparathyroidism in Children Page 257


Rachel I. Gafni, MD

M42 MODY and Other Monogenic Forms of Diabetes Page 262


Mark A. Sperling, MD

M45 Disorders of Sexual Differentiation in Newborns, Infants, and Children Page 270
Nils P. Krone, MD, FRCPCH

M60 Endocrine Effects of Cancer Treatment Page 277


Sebastian Neggers, MD, PhD

M30 Puberty Disorders in Girls Page 280


Sally Radovick, MD

M29 Bone Fragility in Children: When to Worry and What to Do Page 287
Laura K. Bachrach, MD

M50 Diabetes Insipidus in Children Page 292


Craig A. Alter, MD

REPRODUCTIVE ENDOCRINOLOGY
CMF5 State-of-the-Art: Use of Hormones in Transgender Individuals Page 296
Benjamin Boh, DO, MS and Joshua D. Safer, MD, FACP

M07 Female Sexual Dysfunction: Do Hormones Help? Page 301


Susan R. Davis, MBBS, FRACP, PhD

M19 Polycystic Ovarian Syndrome Page 306


Margaret E. Wierman, MD

M27 Bioidentical Hormone Replacement Page 310


JoAnn V. Pinkerton, MD

The Endocrine Society. Downloaded from press.endocrine.org by [${individualUser.displayName}] on 12 January 2017. at 12:19 For personal use only. No other uses without permission. . All rights reserved.
x ENDO 2016 TOPIC INDEX

M40 Testosterone Replacement Therapy in Men Page 315


Alvin M. Matsumoto, MD

M49 Menopause: Weighing the Options n/a


Janet E Hall, MD, MSc

M58 Care of the Adult Woman with Turner Syndrome Page 322
Amanda Vincent, MBBS, B Med Sci, FRACP, PhD

THYROID/HPT AXIS
CMF2 Thyroid Nodules: What Molecular Markers are Most Useful? n/a
Steven Paul Hodak, MD and Bryan McIver, MB, ChB, PhD

CMF3 Thyroid Function Tests That Do Not Make Sense Page 330
Virginia D. Sarapura, MD and Jacqueline Jonklaas, MD, PhD

CMF9 Thyroid Cancer: Advanced Cases Page 334


Mona M. Sabra, MD and Bryan Haugen, MD

CMF12 Nonthyroidal Illness Syndrome: To Treat or Not? Page 338


Gregory A. Brent, MD and Alan P. Farwell, MD, FACE

M20 Thyroid Disease in Pregnancy Page 343


Elizabeth N. Pearce, MD, MSc

M28 Graves Orbitopathy Page 348


Marius N. Stan, MD

M41 Thyrotoxicosis: When Antithyroid Drugs Fail Page 353


Henry B. Burch, MD

M43 Hypothyroidism in the Elderly Page 355


Anne R. Cappola, MD, ScM

M59 Levothyroxine Therapy: When Outcomes Are Less Than Optimal - Optimizing
Patient Care Page 359
James V. Hennessey, MD

The Endocrine Society. Downloaded from press.endocrine.org by [${individualUser.displayName}] on 12 January 2017. at 12:19 For personal use only. No other uses without permission. . All rights reserved.
ENDO 2016 SPEAKER HANDOUT INDEX xi

ENDO 2016
SPEAKER HANDOUT INDEX
Adler, Gail K. .......................................................... 42 Lane, Wendy S. ........................................................ 89
Alter, Craig A. ........................................................ 292 Lopes-Virella, Maria F. ............................................. 232
Arafah, Baha M. ....................................................... 32 Low Wang, Cecilia C. .............................................. 105
Aronne, Louis J. ...................................................... 239 Matsumoto, Alvin M. ............................................... 315
Auchus, Richard J. ...................................................... 2 McCall, Anthony L. ................................................. 129
Bachrach, Laura K. .................................................. 287 McDonnell, Marie E. ................................................ 222
Bantle, John P. ........................................................ 227 McIver, Bryan ......................................................... n/a
Barbour, Linda A. .................................................... 118 McMahon, Graham T. .............................................. 138
Becker, Carolyn B. .................................................... 70 Molitch, Mark E. ..................................................... 214
Blonde, Lawrence ..................................................... n/a Neggers, Sebastian J.C.M.M. ...................................... 277
Boh, Benjamin ........................................................ 296 Nieman, Lynnette K. ................................................ 191
Bollerslev, Jens ....................................................... 202 ODorisio, Thomas M. .............................................. 184
Brent, Gregory A. .................................................... 338 Aystese, KAB ......................................................... 202
Burch, Henry B. ...................................................... 353 Pattison, David A. ................................................... 184
Busui Pop Rodica .................................................... 101 Pearce, Elizabeth N. ................................................. 343
Cappola, Anne R. .................................................... 355 Pettifor, John M. ....................................................... 59
Chamberlain, Albert ................................................. 210 Pinkerton, JoAnn V. ................................................. 310
Cornier Marc-Andre ................................................. 244 Radovick, Sally ....................................................... 280
Davis, Susan R. ....................................................... 301 Riddell, Michael C. .................................................. 125
Donahoo, William T. ................................................ 218 Rothman, Micol S. .................................................... 48
Edelman, Steven V. ................................................... 84 Sabra, Mona M. ...................................................... 334
Else, Tobias ........................................................... 144 Safer, Joshua D. ...................................................... 296
Farooki, Azeez ......................................................... 51 Sarapura, Virginia D. ............................................... 330
Farwell, Alan P. ...................................................... 338 Saunders, Katherine ................................................. 239
Feingold, Kenneth .................................................... n/a Schauer, Irene E. ..................................................... 113
Feldt-Rasmussen, Ulla .............................................. 206 Seaquist, Elizabeth R. ............................................... 110
Findling, James W. ..................................................... 2 Shane, Elizabeth ....................................................... 64
Fishbein, Lauren M. .................................................. 27 Sheffield-Moore, Melinda .......................................... 210
Francis, Gary L. ...................................................... 250 Shukla, Alpana ........................................................ 239
Gafni, Rachel I. ...................................................... 257 Silverberg, Shonni J. .................................................. 55
Gagel, Robert F. ...................................................... 172 Sparks, Lauren M. ..................................................... 96
Habra, Mouhammed A. .............................................. 10 Sperling, Mark A. .................................................... 262
Hall, Janet E. .......................................................... n/a Stan, Marius N. ....................................................... 348
Haugen, Bryan R. .................................................... 334 Trainer, Peter J. ...................................................... 178
Hennessey, James V. ................................................ 359 Urban, Randall J. ..................................................... 210
Ho, Ken ................................................................ 153 Vaidya, Anand ......................................................... 10
Hodak, Steven P. ...................................................... n/a Vella, Adrian .......................................................... 167
Holick, Michael F. .................................................... 80 Verbalis, Joseph G. .................................................. 195
Husebye, Eystein S. ................................................... 21 Vincent, Amanda ..................................................... 322
Inzucchi, Silvio E. .................................................... n/a Vinik, Aaron I. ....................................................... 157
Jonklaas, Jacqueline ................................................. 330 Vogiatzi, Maria G. .................................................... 17
Kerr, Janice M. ....................................................... 191 Watts, Nelson B. ....................................................... 75
Kiseljak-Vassiliades, Katja ........................................ 178 Wierman, Margaret E. .............................................. 306
Korbonits, Mrta ..................................................... 144 Young, William F. .................................................... 36
Krone, Nils P. ......................................................... 270 n/a Handout not available

The Endocrine Society. Downloaded from press.endocrine.org by [${individualUser.displayName}] on 12 January 2017. at 12:19 For personal use only. No other uses without permission. . All rights reserved.
ADRENAL/HPA AXIS

The Endocrine Society. Downloaded from press.endocrine.org by [${individualUser.displayName}] on 12 January 2017. at 12:19 For personal use only. No other uses without permission. . All rights reserved.
2 ENDO 2016 MEET-THE-PROFESSOR CLINICAL CASE MANAGEMENT

Adrenal Insufficiency, Subclinical and Adrenal Fatigue

CMF4 BARRIERS TO OPTIMAL PRACTICE


Presented, April 1 4, 2016 The concept of adrenal fatigue has been promoted by integra-
tive medicine and naturopathic medicine for several years, and
endocrinologists are often asked for consultation on patients
Richard J. Auchus, MD, PhD; James W. Findling, with presumed subclinical adrenal insufficiency. Many of
MD. Department of Pharmacology & Internal Medicine, these patients are provided with adrenal support and some
Division of Metabolism, Endocrinology & Diabetes, are actually given glucocorticoid therapy based on a salivary
University of Michigan Medical Center, Ann Arbor, cortisol day curve. Few, if any, of these patients have any
Michigan 48109, E-mail: rauchus@med.umich.edu; and biochemical evidence of either ACTH deficiency or cortisol
Medical College of Wisconsin, Milwaukee, Wisconsin deficiency.
Although clinical or relative adrenal insufficiency has
53051, E-mail: jfindling@mcw.edu
been suggested as a possible problem in patients who are either
acutely ill or have undergone chronic stress, there is currently
INTRODUCTION no biochemical criteria that establishes such a problem and
Historical Overview very little clinical evidence of its existence. Nonetheless, there
In 1855, Dr Thomas Addison published his classic work, On are many circumstances in which there is a modest decrease in
the Constitutional and Local Effects of Disease of the Suprare- cortisol secretion and the optimal corticosteroid replacement
nal Capsules, in which he described adrenocortical insuffi- therapy remains controversial.
ciency (1). Thomas Addison was born in April 1793 near New
Castle-upon-Tyne. He studied medicine at the University of LEARNING OBJECTIVES
Edinburgh and was awarded an MD degree in 1815. He even- As a result of participating in this session, learners should be
tually joined the staff at Guys Hospital in London and helped able to:
to elevate the reputation of the medical school there. In addi- Appropriately use and interpret the results of basal and
tion to describing Addisons disease, he also made important dynamic tests of adrenal function.
observations on pneumonia, pulmonary tuberculosis, and fatty Distinguish the clinical and laboratory findings of
liver. Dr Addison also provided the first description of appen- primary and secondary adrenal insufficiency.
dicitis in his and Richard Brights, The Elements of the Prac- Recognize the myth of adrenal fatigue propagated by
tice of Medicine, published in 1839 (2). Twenty-five years after entrepreneurial naturopathic practitioners.
Dr Addisons death, Dr Edward Kendall was born in South List the drugs that interfere with the hypothalamic-
Norwalk, Connecticut in 1886. He received his training at pituitary-adrenal (HPA) axis and cortisol metabolism.
Columbia University and was not only responsible for the Appreciate the degree of cortisol deficiency required for
isolation of T4 while working at Parke-Davis but also discov- clinically meaningful manifestations.
ered and isolated Compound E (cortisone) during a long re-
search career at the Mayo Clinic. The latter discovery earned STRATEGIES FOR DIAGNOSIS, THERAPY,
him the Nobel Prize in 1950 with Phillip Hench and Taddeus AND/OR MANAGEMENT
Reichstein. This remarkable story is beautifully told in The Clinical Presentations of Adrenal Insufficiency
Quest for Cortisone by Thom Rooke from the Mayo Clinic (3). Table 1 summarizes the possible clinical presentations of adre-
nal insufficiency. Constitutional and nonspecific complaints
Dr Kendall was president of the Endocrine Society in
such as fatigue, malaise, nausea, anorexia, and weight loss are
1930 1931.
appreciated but none of them provide very good sensitivity or
Adrenal insufficiency remains a well-appreciated endocri-
specificity. Nonetheless, the diagnosis should be considered in
nopathy whose causes have changed dramatically since the
any patient with unexplained weight loss or hypotension.
mid-19th century. Most cases of adrenal insufficiency are now
An important clinical point is that, in the absence of con-
due to deficiencies of ACTH secretion from the pituitary, comitant conditions such as renal, heart, or liver failure, weight
usually due to iatrogenic factors. Although less common, pri- gain essentially excludes the diagnosis of adrenal insufficiency.
mary adrenal insufficiency (the term for which Addisons dis- The most common clues for the diagnosis are usually from
ease is usually reserved) remains a very important clinic disor- routine laboratory studies. Hyponatremia is a common clinical
der with protean presentations. And the discovery of cortisone manifestation of both primary and secondary adrenal insuffi-
by Dr Kendall provided life-saving therapy for patients with all ciency. Glucocorticoids exert negative feedback on vasopressin
causes of adrenal gland insufficiency. secretion and deficiencies of cortisol may result in nonosmotic

The Endocrine Society. Downloaded from press.endocrine.org by [${individualUser.displayName}] on 12 January 2017. at 12:19 For personal use only. No other uses without permission. . All rights reserved.
ENDO 2016 ADRENAL/HPA AXIS 3

TABLE 1. Clinical Presentations of Adrenal Insufficiency The most common causes of cortisol deficiency are iatro-
Signs/symptoms genic. All exogenously administered glucocorticoids, regard-
Fatigue, malaise, nausea/vomiting, unexplained weight loss less of dose and route of administration, may suppress the HPA
Hypotension, increased skin pigmentation axis. Given that corticosteroids are metabolized by CYP3A4,
Routine laboratory abnormalities drugs that inhibit this hepatic enzyme may reduce metabolic
Hyponatremia, hyperkalemia, hypercalcemia clearance of exogenous steroids and amplify their clinical ef-
Lymphocytosis, eosinophilia fect. It has been shown that endogenous cortisol deficiency will
Drug-induced occur in 50% of patients who receive either intra-articular or
Exogenous corticosteroids (any route of administration), oral glucocorticoids and may even be present in 510% of
medroxyprogesterone acetate patients who receive inhaled or topical corticosteroids. In ad-
Opioids dition, it is less-well appreciated that opioids and some psycho-
CTLA-4 monoclonal antibody-induced hypophysitis (eg, tropic medications may attenuate the pituitary-adrenal axis
ipilimumab) function (4 7). Finally, the diagnosis of adrenal insufficiency
Psychotropic medication may also need to be considered in patients with a genetic
Benzodiazepines (alprazolam), atypical antipsychotics predisposition for adrenal insufficiency including various forms
(onlanzapine, quetiapine)
of congenital adrenal hyperplasia, adrenoleukodystrophy, fa-
Adrenostatic/adrenolytic (etomidate, ketoconazole, metyrapone,
mitotane, osilodrostat)
milial glucocorticoid deficiency (ACTH resistance syndromes),
GR antagonist (mifepristone)
and autoimmune polyglandular syndromes (21).
Imaging abnormalities
Primary: bilateral adrenal enlargement/masses vs atrophic Diagnostic Approach
adrenals History and Physical
Secondary: pituitary mass With the exception of iatrogenic HPA axis suppression by
Hypothalamic-pituitary disease exogenous glucocorticoids, adrenal insufficiency is rare. The
Pituitary tumors, granulomatous diseases, hypophysitis, probability of adrenal insufficiency is extremely low in the
iatrogenic, genetic conditions absence of specific clinical features such as hyperpigmentation,
Genetic disorders hypotension, hypoglycemia, vitiligo, known pituitary disease,
Congenital adrenal hyperplasias weight loss, and anorexia.
Adrenoleukodystrophy
Autoimmune polyglandular syndromes
Basal Testing
ACTH insensitivity (familial glucocorticoid deficiency),
Basal Serum Cortisol and Plasma ACTH Values. Basal se-
triple-A syndrome
rum cortisol and plasma ACTH values alone can be used to
Adrenal hypoplasia congenital
exclude all forms of adrenal insufficiency in most patients.
Other transcription factor defects
Because of the strong diurnal rhythm for these hormones, the
Abbreviation: CTLA-4, cytotoxic T-lymphocyte antigen 4. sample is most useful when drawn before 0900 hours; how-
ever, adequate values at any time of day exclude adrenal
insufficiency. A serum cortisol of greater than 14 g/dL (400
nmol/L) is 99% specific for predicting a cortisol increase
greater than 18 g/dL (500 nmol/L) during insulin-induced
stimulation of vasopressin and hyponatremia. Accordingly, the
hypoglycemia (insulin tolerance test [ITT]) (26). Although the
diagnosis of adrenal insufficiency should be considered in any
ACTH value itself is not used to exclude adequate cortisol
patient with unexplained hyponatremia. Hyperkalemia as a
production, when cortisol is low (5 g/dL, 140 nmol/L),
result of mineralocorticoid deficiency may occur in some pa-
the ACTH is used to distinguish primary (100 pg/mL, 20
tients with significant primary adrenal insufficiency. When
pmol/L) from secondary (20 pg/mL, 4.4 pmol/L) adrenal
observed in patients with hyponatremia, the exclusion of the
insufficiency. Basal cortisol values 5-14 g/dL are inconclu-
diagnosis is mandatory.
sive and require additional testing.
Bilateral adrenal enlargement, particularly when the enlarge-
ment coincides with the normal contour of the adrenal glands,
may also be the initial clue to the diagnosis of adrenocortical Random Serum DHEAS. A random serum dehydroepiandrosterone
insufficiency. The presence of adrenal insufficiency in this (DHEA) sulfate (DHEAS) value greater than 60 g/dL (1500
setting is one of the few indications to perform percutaneous nmol/L) also rules out adrenal insufficiency unless the patient
computed tomography (CT) guided biopsy of the adrenal is taking DHEA supplements (24). Unlike cortisol and ACTH,
glands (after pheochromocytoma has been excluded). Of DHEAS is so heavily protein bound that its diurnal rhythm is
course, adrenal insufficiency needs to always be excluded in very slight, permitting use of values obtained any time of day.
patients with hypothalamic-pituitary disease. Because the data supporting its performance vs ITT are sparse,

The Endocrine Society. Downloaded from press.endocrine.org by [${individualUser.displayName}] on 12 January 2017. at 12:19 For personal use only. No other uses without permission. . All rights reserved.
4 ENDO 2016 MEET-THE-PROFESSOR CLINICAL CASE MANAGEMENT

DHEAS is usually used to adjudicate basal cortisol values decrease to less than 5 g/dL to document adequate
slightly less than 14 g/dL (27), particularly when the cortisol 11-hydroxylase inhibition. The criterion for a normal test is a
is drawn after 0900 hours, and caution should be used when serum 11-deoxycortisol greater than 7 g/dL. Note that for
interpreting values close to 60 g/dL. Limitations to DHEAS many commercial cortisol immunoassays, 11-deoxycortisol
include its facile and sustained suppression by exogenous cross-reacts more than 50%; consequently, both cortisol and
glucocorticoids and its normal decline with age, which limits 11-deoxycortisol are best measured by tandem mass spectrom-
its utility in the elderly (age 65 y). etry during this test (23). Conceptually, this test asks whether
the sum of cortisol and cortisol precursors increases to greater
Serum Aldosterone and Plasma Renin. Serum aldosterone and than 12 g/dL (330 nmol/L) when ACTH is elevated by a
plasma renin (activity or mass) are sometimes useful in distin- physiologic stimulus. Based on this logic, the test is unneces-
guishing primary from secondary adrenal insufficiency, but sary when the basal cortisol is itself greater than 12 g/dL. The
aldosterone values are only interpretable when renin is current suppliers of metyrapone for treatment and dynamic
midnormal to high (2 ng/mL/h). testing vary with country, and metyrapone can be expensive
and difficult to obtain.

Dynamic Testing
Standard 250-g Cosyntropin (Short Synachten) ITT Test. The ITT is the gold standard test for all forms of
Stimulation Test. The standard 250 g cosyntropin (short adrenal insufficiency, but it should be restricted to patients with
synachten) stimulation test (CST, SST) is used to definitively suspected secondary adrenal insufficiency and equivocal basal
exclude primary adrenal insufficiency or longstanding second- testing. The ITT has been criticized as nonphysiologic, but
ary adrenal insufficiency. The agent may be administered as an other stimuli that stimulate the HPA axis such as exercise are
iv bolus with sampling as early as 30 minutes or im, when the difficult to standardize and to administer. A bolus of 0.1 0.2
peak is delayed until 45 60 minutes. A peak cortisol value at U/kg of regular insulin is administered by iv bolus in the early
30 60 minutes of greater than 18 g/dL (500 nmol/L) is morning after an overnight fast. Blood samples are obtained for
normal; the change in serum cortisol is highly dependent on the serum cortisol every 15 minutes for 75 minutes. The glucose
basal value, which varies by time of day and clinical status, and nadir should be 40 mg/dL (2.2 mmol/L) to interpret an
should never be used as a diagnostic criterion. Aldosterone abnormal test, and this nadir typically occurs between 30 and
normally doubles in response to cosyntropin, which is help- 45 minutes after the bolus. A cortisol increase to greater than 18
ful in confirming normal adrenal function when cortisol g/dL is normal, and measurement of ACTH does not provide
testing is confounded. additional diagnostic information (28). The test is contraindicated
in patients with seizure disorders, significant cardiovascular dis-
ease, and inability to verbalize symptoms of hypoglycemia.
Low-Dose 1-g CST. The low-dose 1 g CST is popular in
some centers excluding secondary adrenal insufficiency but re-
quires precise timing of samples (30 min) and bolus administra- Special Conditions
tion directly into an iv port with minimal tubing. The exact value Corticosteroid-Binding Deficiency. Corticosteroid-binding
for confidently excluding secondary adrenal insufficiency is prob- globulin (CBG) deficiency is very rare. These patients have low
ably also 18 g/dL, but conflicting numbers are found in the serum cortisol, approximately 0.52 g/dL (15 nmol/L), lack fea-
literature. We have found that this test provides little more infor- tures of adrenal insufficiency, and do not require treatment (20).
mation beyond early morning ACTH, cortisol, and DHEAS mea-
surements, but it can be helpful when a diagnosis is urgently Patients With Critical Illness. Criteria for patients with criti-
needed. In patients with partial central adrenal insufficiency, the cal illness are no different than for outpatients; however, see
DHEA response to low-dose cosyntropin is lost before the cortisol comments about hypoproteinemia and free cortisol below. In
response, so DHEA measurement can be helpful (25). addition, neutrophil elastase cleaves liganded CBG, which re-
leases bound cortisol at sites of inflammation and increases
Overnight Metyrapone Test. The overnight metyrapone test cortisol delivery.
is used to drive the entire HPA axis and to test for secondary
adrenal insufficiency. Metyrapone inhibits 11-hydroxylase Hypoproteinemia. Hypoproteinemia reduces plasma cortisol
(CYP11B1, P450 11B1, P450c11), which acutely lowers cor- binding, elevating the free cortisol fraction and lowering the
tisol with a resultant increase in ACTH and cortisol precursors. total cortisol concentration needed to attenuate ACTH secre-
A bolus dose of 30 mg/kg up to 3 g is given at 2300 hours with tion. In critically ill patients with a serum albumin less than 2.5
food, and a blood sample is obtained at 0800 hours the follow- mg/dL, cosyntropin stimulated total cortisol values as low as 7
ing morning, similar to the overnight dexamethasone suppres- g/dL (200 nmol/L) can be normal, as adjudicated with plasma
sion test. For the test to be valid, the serum cortisol must free cortisol values (1215).

The Endocrine Society. Downloaded from press.endocrine.org by [${individualUser.displayName}] on 12 January 2017. at 12:19 For personal use only. No other uses without permission. . All rights reserved.
ENDO 2016 ADRENAL/HPA AXIS 5

Glucocorticoid Insensitivity Syndrome. Glucocorticoid in- hours achieve serum cortisol concentrations 40 120 g/dL,
sensitivity syndrome is always partial and is likewise very rare. well above those typically achieved by the normal adrenal
These patients have high cortisol and adrenal-derived andro- during critical illness. We tend to give smaller doses more
gens and mineralocorticoids but reach a new set point with frequently, such as 50-mg bolus, then 25 mg every 6 hours,
normal glucocorticoid physiology. Mifepristone is a glucocor- rather than the traditional 100-mg-every-8-hours regimen.
ticoid receptor (GR, NR3C1) antagonist, and mifepristone ad- What is important is that the dosing is continued until the
ministration yields a pharmacologic, partial, and reversible patient is well and can take their usual oral regimen. At these
glucocorticoid insensitivity condition with high serum cortisol doses greater than 40 mg per day, the mineralocorticoid activ-
if the HPA axis is intact. ity of hydrocortisone is sufficient to obviate the need for
fludrocortisone acetate. Alternatively, methylprednisolone (Solu-
Plasma Free Cortisol During CST. The use of plasma free medrol) as a 510-mg bolus followed by 5 mg every 6 8 hours
cortisol during CST has recently shown equivalent if not supe- is plenty, or dexamethasone, 2 mg every 8 12 hours. With
rior performance to total serum cortisol (17), and this type of these alterative glucocorticoids, mineralocorticoid replacement
testing might be useful in certain circumstances, such as might be necessary, and therefore hydrocortisone is preferable.
patients with hypoproteinemia, particularly in critical ill-
ness, or oral contraceptive drug therapy with high CBG (8). Chronic Corticosteroid Replacement
Saliva is an ultrafiltrate of plasma, and thus saliva cortisol Hydrocortisone. Hydrocortisone remains the preferred gluco-
concentrations typically parallel those of plasma free corti- corticoid replacement therapy for adults with all forms of
sol (18, 19). The utility of late-night saliva cortisol in the adrenal insufficiency, because it is the most physiologic of the
diagnosis of Cushings syndrome is well documented, but available treatments and the least likely to cause iatrogenic
fewer studies have addressed its performance and developed Cushing syndrome. Ideally, 1520 mg per day given in three
normative values for diagnosing adrenal insufficiency. divided doses to mimic the normal diurnal rhythm provides the
most physiologic regimen, with the first dose taken on arising
How Much Cortisol Does One Really Need? or at the bedside 30 minutes prior to arising and subsequent
To put the issue in perspective, consider that circulating corti- doses at 1200 1400 hours and a third before 1800 hours if
sol is approximately 10% free hormone and that the affinity of used. Many adults have difficulty remembering all three doses
cortisol for GR is roughly 10 nmol/L. Thus, a serum cortisol of (10-5-5 mg), and two daily doses are usually feasible, such as
7 g/dL (200 nmol/L, free 20 nmol/L) is nearly enough to or 15-5 mg. Rarely, patients with primary adrenal insufficiency
saturate GR. Consequently, even low amounts of cortisol are feel well on a single 20-mg dose on arising, whereas patients
better than none during physiologic stress. Case 2 illustrates with partial central adrenal insufficiency often only need a
this point. single 10- or 15-mg morning dose. Alternatively, a morning
dose of prednisolone or methylprednisolone, 4 8 mg is a
Do Not Forget Etiology and Implications reasonable strategy for patients who cannot take medication
Primary adrenal insufficiency is rare. If a diagnosis is made, the during the day, but synthetic glucocorticoids are more likely to
etiology should be pursued because this information will have cause iatrogenic Cushing syndrome. Prednisone should be
additional consequences for the patient. Young men with primary avoided, given that it is not the active drug and requires
adrenal insufficiency should be screened for adrenoleukodystrophy metabolism to prednisolone in the liver, which is unpredictable
by measuring very-long-chain fatty acids, and all children at the small doses used for adrenal insufficiency. Dexametha-
should be assessed for genetic causes. sone causes even more adverse effects and should be avoided
Finally, the diagnosis of adrenal insufficiency carries pro- for chronic therapy, although its temporary use in moderate
found implications: life-long replacement therapy, medical illness can be beneficial. Sustained-release hydrocortisone
alert identification, increased risk of death (22), limitations for preparations have been developed, and one form (Plenadren)
employment and insurance, etc. is in use in Europe. Glucocorticoid dosing to manage the
Do not label someone with this diagnosis unless you are androgen excess and/or hypertension of congenital adrenal
absolutely certain. hyperplasia is a special situation that is beyond the scope of
this handout.
Management
Acute Management
In the acutely ill patient with adrenal insufficiency, aggressive Fludrocortisone Acetate. Fludrocortisone acetate is the stan-
volume replacement with normal saline is essential. Hydrocor- dard mineralocorticoid replacement therapy, and this is one of
tisone hemisuccinate (Solu-cortef) is given as an iv bolus as the rare drugs for which the dose is similar in infants and
soon as possible, or im if venous access is difficult. The actual adults, 0.1 0.4 mg per day and occasionally more. We find that
amount is not so critical, but at least 20 mg will raise serum most adults with adrenal insufficiency feel better when ad-
cortisol concentrations dramatically. Doses of 50 mg every 6 equately volume replaced with salt and fludrocortisone.

The Endocrine Society. Downloaded from press.endocrine.org by [${individualUser.displayName}] on 12 January 2017. at 12:19 For personal use only. No other uses without permission. . All rights reserved.
6 ENDO 2016 MEET-THE-PROFESSOR CLINICAL CASE MANAGEMENT

Fludrocortisone acts chronically rather than acutely and has a glucocorticoids. In fact, low-normal or subnormal ACTH lev-
long half-life, so it may be taken any time of day or on els in patients with primary adrenal insufficiency suggest ex-
alternate days when using very low doses. cessive glucocorticoid replacement.

Sickness and surgery Mineralocorticoid titration. Orthostasis and unexplained fa-


Our sick day rules are very simple: tigue that do not resolve after a hydrocortisone dose suggest
If you are losing fluid faster than you can replace it insufficient fludrocortisone. The goal for fludrocortisone dosing
meaning high fever, diarrhea or vomitingthen double are normal sitting and standing blood pressure (BP) without
(or triple) your glucocorticoid dose until you are orthostatic tachycardia, normal serum potassium, and plasma
completely well for 1 day and then resume your usual renin activity in the normal range or slightly lower. Thus, a
dose without tapering. typical laboratory assessment would be electrolytes, glucose,
If you cannot take your medication orally, take your creatinine, and plasma renin.
emergency hydrocortisone hemisuccinate (100 mg im)
and immediately go to the nearest emergency department
CASES WITH QUESTIONS
or call an ambulance.
Case 1
For planned surgeries, a 50-mg hydrocortisone You are asked to provide an opinion regarding the diagnosis of
hemisuccinate bolus iv or im prior to induction of adrenal fatigue in a 19-year-old man complaining of anxiety
anesthesia will cover at least 4 hours of surgery well. and panic attacks accompanied by palpitations and weakness.
Depending on the extent of surgery, complications, and He had sought care from a naturopath who made the diagnosis
blood loss, subsequent 25-mg doses every 6 hours is of adrenal fatigue based on salivary cortisol profile, which
usually sufficient until the patient can take medication showed that all his cortisol measurements were below the
orally. Many times, the routine outpatient dose may be reference range. His mother accompanies him at the visit and
resumed the following day, or higher doses are insisted that he see an endocrinologist.
continued when necessary. Fludrocortisone may be His examination showed a healthy-seeming man with nor-
suspended for a few days, particularly when the mal skin pigmentation, BP of 124/74 mm Hg; pulse, 60
hydrocortisone dose is high. For dental procedures, beats/min; and a body mass index of 21.1 kg/m2. His entire
routine colonoscopy, and minor surgeries lasting less examination was completely normal. An early-morning serum
than 2 hours, doubling the morning oral dose before the cortisol level was 1.2 g/dL (33.1 nmol/L).
procedure is usually sufficient, followed by resumption 1. Which of the following tests would you obtain next?
of the usual dosing. A. Plasma ACTH
B. A rapid CST
Glucocorticoid titration C. Pituitary magnetic resonance imaging (MRI)
Unlike T4 replacement in primary hypothyroidism, the titration D. DHEAS
of corticosteroid replacement is mainly based on clinical evalu- A rapid ACTH stimulation test (250 g cosyntropin) yields a
ation rather than laboratory parameters. First and foremost, peak cortisol response of 11.6 g/dL (320 nmol/L) and a basal
evaluation for physical stigmata of iatrogenic Cushing syn- ACTH is less than 5 pg/mL. His total testosterone, free T4, and
drome should be conducted at every visit. Findings such as IGF-I levels are normal.
supraclavicular fat pads, easy bruising and dermal atrophy, 2. Which of the following studies would you secure next?
facial plethora, and worsening glucose tolerance suggest that A. Pituitary MRI
the total glucocorticoid dose is too high. Weight loss, asthenia, B. Insulin-induced hypoglycemia
anorexia, and weakness suggest underdosing. If a patient taking C. Metyrapone stimulation test
two doses of hydrocortisone feels well all morning after the D. CRH stimulation test
first dose but has fatigue and nausea later in the day, consider- A pituitary MRI is normal.
ation should be given to three doses, a larger second dose, or 3. Which of the following studies would you secure in this
switching to a longer-acting glucocorticoid. gentleman?
A serum cortisol measurement after a hydrocortisone dose is A. A measurement of long-chain fatty acids
occasionally useful to document good absorption, but a single B. 21-hydroxylase antibodies
value has limited utility for assessing drug exposure. Some C. A synthetic glucocorticoid screen
clinicians measure day curves with frequent cortisol sam- D. 17-hydroxyprogesterone
pling on the outpatient regimen, but this procedure is generally A synthetic glucocorticoid screen was negative. Exogenous
not feasible in the United States. ACTH measurements are not glucocorticoid therapy is the most common cause of cortisol
helpful in patients with adrenal insufficiency to guide therapy, deficiency. Many patients are not aware of which compounds
as ACTH remains high despite appropriate doses of are being injected in their joints and epidural space. Given that

The Endocrine Society. Downloaded from press.endocrine.org by [${individualUser.displayName}] on 12 January 2017. at 12:19 For personal use only. No other uses without permission. . All rights reserved.
ENDO 2016 ADRENAL/HPA AXIS 7

the effect of such injections may suppress the HPA axis for many insufficiency reported. The most likely explanation is that super-
months, the patients may have even forgotten about the steroid imposed acute stressful events seem to probably easily override
treatment. In addition, there are many arthritis remedies from Asia the opioid-induced HPA axis suppression. Moreover, as the opioid
and Latin America (available over the Internet) that may have effect dissipates, there seems to be a very prompt hyper-response
unrecognized and high doses of various synthetic corticosteroids. of the HPA axis function. The accompanying volatility of HPA
A synthetic glucocorticoid screen (Mayo Clinic with tandem mass axis function in patients on chronic opioids makes the assessment
spectroscopy) is commercially available and provides assessment of pituitary adrenal function in these patients very challenging. It
of many common synthetic glucocorticoid products. is not clear whether and how opioid-induced decreases in HPA
4. Two weeks after the initial visit, the patients mother calls axis function should be treated.
you and says she found which of the following in his
dresser drawer? Case 2
A. Marijuana A 64-year old man was found to have bilateral adrenal enlarge-
B. Cocaine ment with a discrete 6.9-cm left adrenal mass on a positron
C. DHEA emission tomography computed tomography (PET/CT) scan
D. Buprenorphine performed for suspected right tonsillar malignancy. Adrenocor-
tical carcinoma (ACC) was suspected, and laparoscopic left
Discussion adrenalectomy was performed. Pathology was interpreted as
The concept of adrenal fatigue is imaginary. There is no scien- having features of low-grade ACC, and he was treated with
tific evidence to support its existence. Adrenal fatigue has been mitotane. Follow-up PET/CT showed increased metabolic ac-
promoted by integrative medicine and naturopathic medicine tivity in the right testicle, and he underwent right orchiectomy.
for many years based on salivary cortisol day curves. These Pathology showed two small yet distinct masses, which were
particular practitioners employ intentionally arbitrary and very interpreted as consistent with metastatic ACC. Upon referral to
narrow reference ranges for these salivary day curves, so that our VA hospital, physical examination showed normal BP, short
most patients will have at least one measurement outside of this stature, normal secondary sexual characteristics, and diffusely
so-called normal range. The idea that chronic stress physi- tanned skin with hyperpigmentation in the surgical scars. Basal
cal or psychological may somehow down regulate the HPA testing at 0800 hours showed plasma ACTH, 1400 pg/mL (300
axis is not supported by any good clinical science. In fact, the pmol/L) and serum cortisol, 2.3 g/dL (30 nmol/L).
contrary is true. Patients with chronic fatigue syndrome have 5. Which of the following tests would you do next?
not been shown to have any consistent dysregulation of A. Stop mitotane and give etoposide, daunorubicin, and
pituitary-adrenal function, and well-designed randomized con- cisplatin (EDP).
trolled trials assessing the use of hydrocortisone in these pa- B. Take a family history and measure serum 17-hydroxypro-
tients have shown only short-term benefit. In addition, patients gesterone.
with post-traumatic stress disorder may have basal cortisol C. Measure DHEAS and corticosteroid-binding globulin.
levels slightly lower than healthy subjects, but there is clearly a D. Perform 250-g CST.
hyper-responsiveness of the HPA axis during periods of stress,
and this heightened sympatho-adrenal response may contribute
to dysphoria and rage seen in some of these patients. Discussion
Although suppression of the hypothalamic-pituitary-gonadal The correct answer is B. This man has four siblings, three with
axis related to opioid use is well known, the potential suppres- short stature who were tall as children, including a sister born
sion of the HPA axis in patients receiving narcotics is much with ambiguous genitalia. Review of the adrenal and testicular
less well appreciated. The interactions of opioids with HPA pathology revealed identical adrenocortical masses with focal
axis are complex and poorly understood. Early studies in pigmentation. The masses lacked the usual features associated
methadone addicts found lower plasma cortisol and ACTH with malignant behavior, such as invasion, high mitotic activ-
responses to naloxone. Approximately 70% of heroin addicts ity, and necrosis. Foci of myelolipoma and tight collections of
have been shown to have an impaired cortisol response to the lymphocytes were also present in the left adrenal mass, and the
CST. The findings for most studies support the concept that testicular masses were testicular adrenal rest tumors. His
opioids act on neurotransmitters that regulate the secretion of 17-hydroxyprogesterone (17-OHP) was 4500 ng/dL (1500
corticotroph-releasing hormone (CRH) thereby suppressing nmol/L), which with the elevated plasma ACTH and serum corti-
ACTH and cortisol. The most likely cause of adrenal insuffi- sol less than 5 g/dL established the diagnosis of classic
ciency in this patient is the surreptitious use of buprenorphine. 21-hydroxylase deficiency (21-OHD). He has classic, not nonclas-
Buprenorphine is a mixed agonist-antagonist effect on the sic, 21-OHD, given that he meets criteria for primary adrenal
and opioid receptors. Given the widespread use of opioids insufficiency, and his 17-OHP is well above 1000 ng/dL.
for chronic pain management, it seems surprising that there Wait a minute! How can he have classic 21-OHD? He
have only been a few cases of opioid-induced clinical adrenal served in the military, survived at least two surgeries, and did

The Endocrine Society. Downloaded from press.endocrine.org by [${individualUser.displayName}] on 12 January 2017. at 12:19 For personal use only. No other uses without permission. . All rights reserved.
8 ENDO 2016 MEET-THE-PROFESSOR CLINICAL CASE MANAGEMENT

not present to endocrine attention until in his 60s. All true. But serious illness is actually accompanied by decreases in plasma
remember, a serum cortisol of 2-3 g/dL will still activate GR ACTH but maintenance of high levels of serum cortisol. This
fairly well and maintain day-to-day physiology in well patients. disassociation of ACTH and cortisol seems to be related, in
Even surgery, if a major body cavity is not entered and blood large part, to decreases in peripheral cortisol metabolism me-
loss is low, is not an extreme stress. Children undergoing minor diated by impaired activity of cortisol-metabolizing enzymes
urologic procedures have a serum cortisol of 3-5 g/dL during in the liver and kidneys (9). Another possible explanation
the procedure with a slight increase upon reversal of anesthesia for this may be increased adrenocortical sensitivity to
(29). He likely has the intron 2 splice mutation or possibly I172N, ACTH or possible changes in post-translational processing
and both of these alleles yield a little 21-hydroxylase activity. It of pro-opiomelanocortin with liberation of fragments of
would be unlikely, however, that he would sustain systemic sepsis ACTH that are biologically active but poorly measured in
or a major myocardial infarction without experiencing an adrenal immunoassays. In addition, serious illness is often accompa-
crisis, unless treated with glucocorticoids. The point of our pre- nied by decreases in protein synthesis of the major binding
senting this case is to emphasize that routine physiology in well protein for cortisol, CBG. This decrease results in high frac-
patients does not require much cortisol. In addition, this case tional concentrations of free circulating cortisol (as might be
should remind you that occasional cases of classic 21-OHD born reflected in salivary cortisol measurements). In sites of inflam-
before newborn screening are still diagnosed late into adulthood mation during critical illness, CBG may actually be an impor-
with incidental adrenal masses. tant carrier protein for cortisol to these sites, where CBG is
cleaved by neutrophil elastase. The real meaning of any corti-
Case 3 sol level during critical illness should probably be assessed by
You are consulted on a 73-year-old man with type 2 diabetes measurements of glucocorticoid-mediated gene expression and
mellitus who represented with lobar pneumonia 6 days prior to signaling pathways, which is not something that can be per-
your consultation. During the past 36 hours he has developed formed clinically. Adrenal androgen and mineralocorticoid
fever, hypotension, and decreased urine output mandating ad- production seem to decrease during prolonged serious illness.
mission to the Intensive Care Unit. Respiratory failure required There is a disassociation between renin and aldosterone during
intubation and mechanical ventilation and sedation. He was critical illness that is not well understood (16).
receiving aggressive iv isotonic fluid replacement, broad- The American College of Critical Care Medicine has pro-
spectrum antibiotics, and a continuous insulin infusion. posed the term critical illness-related corticosteroid insuffi-
His electrolyte composition was normal. He has a total ciency. They have suggested that any random total cortisol
serum calcium of 7.9 mg/dL and a serum albumin of 1.9 less than 10 g/dL (276 nmol/L) during critical illness estab-
mg/dL. Glucose was well controlled with the insulin infusion. lishes the diagnosis of corticosteroid insufficiency. Despite this
The intensivist performed an assessment of adrenal function. A policy, there is not any good evidence that slightly low serum
basal serum cortisol was 11 g/dL (351 nmol/L) with a peak total cortisol is a significant contributing factor to poor out-
response of 17.5 g/dL (407 nmol/L). A serum aldosterone comes or that high-dose hydrocortisone therapy provides any
was 3.5 ng/dL and a plasma renin activity was 12 ng/mL/h. benefit in patients with sepsis. A systematic review of six
Basal plasma ACTH was 17 pg/mL. high-quality randomized trials has not shown any decrease in
6. Which of the following tests would you do next? mortality with hydrocortisone therapy (10), including the CORTICUS
A. Administer hydrocortisone with fludrocortisone trial of 499 randomly assigned patients (11). There is an ongo-
B. Initiate high-dose dexamethasone ing randomized multicenter trial (NCT01448109; ClinicalTrials.gov)
C. Measure plasma free cortisol and/or salivary cortisol that is planning to investigate the effect on 90-day mortality
D. Obtain a pituitary MRI from high-dose hydrocortisone therapy for 1 week in more than
3000 patients with the systemic inflammatory response
Discussion syndrome/sepsis.
Mechanisms to cope with critical illness are mediated by com- In summary, the concept of relative adrenal insufficiency
plex endocrine responses. The HPA axis is an important com- during critical illness is controversial. Free cortisol concentra-
ponent because increased exposure to cortisol is important to tions seem to be quite normal in most these patients, particu-
provide energy and support cardiovascular function as well as larly those with serum albumin less than 2.5 mg/dL, and there
reduce excessive inflammation in seriously ill patients. Failure is very little evidence that exogenous glucocorticoid therapy
of this stress response may have contributed to poor outcomes provides any measurable benefit. Certainly, there are many
in patients, and there has always been concern and controversy things done to seriously ill patients that may attenuate HPA
about the possible presence of relative adrenal insufficiency function including the administration of high-dose corticosteroids as
during critical illness. Acute stress is accompanied by increases well as many psychotropic medications. Nonetheless, it is cer-
in circulating ACTH, and stimulation of steroidogenic enzyme tainly prudent to evaluate the HPA axis in any seriously ill
expression to increase cortisol production. However, protracted hypotensive/febrile patient. Measurement of free cortisol

The Endocrine Society. Downloaded from press.endocrine.org by [${individualUser.displayName}] on 12 January 2017. at 12:19 For personal use only. No other uses without permission. . All rights reserved.
ENDO 2016 ADRENAL/HPA AXIS 9

(plasma or saliva) may be necessary to establish an accurate 16. Findling JW, Waters VO, Raff H. The dissociation of renin and aldoste-
rone during critical illness. J Clin Endocrinol Metab. 1987;64:592-595.
assessment of adrenocortical function.
17. Bancos I, Erickson D, Bryant S, et al. Performance of free versus total
cortisol following cosyntropin stimulation testing in an outpatient setting.
REFERENCES Endocr Pract. 2015;21:1353-63.
18. Lewis JG, Bagley CJ, Elder PA, Bachmann AW, Torpy DJ. Plasma free
1. Addison T. On the constitutional and local effects of disease of the
cortisol fraction reflects levels of functioning corticosteroid-binding globu-
suprarenal capsules. London: Samuel Highley, 1855.
lin. Clin Chim Acta. 2005;359:189-194.
2. Bright R, Addison T. The elements of the practice of medicine. London:
19. Aardal-Eriksson E, Karlberg BE, Holm AC. Salivary cortisolan alterna-
Longman, Orme, Brown, Green, and Longmans, 1839.
tive to serum cortisol determinations in dynamic function tests. Clin Chem
3. Rooke T. The quest for cortisone. East Lansing: Michigan State University
Lab Med. 1998;36:215-222.
Press, 2012.
20. Torpy DJ, Bachmann AW, Grice JE, et al. Familial corticosteroid-binding
4. Mussig K, Knaus-Dittmann D, Schmidt H, et al. Secondary adrenal failure
globulin deficiency due to a novel null mutation: association with fatigue
and secondary amenorrhoea following hydromorphone treatment. Clin
and relative hypotension. J Clin Endocrinol Metab. 2001;86:3692-3700.
Endocrinol (Oxf). 2007;66:604-605.
21. Brett EM, Auchus RJ. Genetic forms of adrenal insufficiency. Endocr
5. Oltmanns KM, Fehm HL, Peters A. Chronic fentanyl application induces Pract. 2015;21:395-399.
adrenocortical insufficiency. J Intern Med. 2005;257:478-480. 22. Hahner S, Spinnler C, Fassnacht M, et al. High incidence of adrenal crisis
6. Schimke KE, Greminger P, Brandle M. Secondary adrenal insufficiency in educated patients with chronic adrenal insufficiency: A prospective
due to opiate therapy - another differential diagnosis worth consideration. study. J Clin Endocrinol Metab. 2015;100:407-416.
Exp Clin Endocrinol Diabetes. 2009;117:649-651. 23. Monaghan PJ, Owen LJ, Trainer PJ, Brabant G, Keevil BG, Darby D.
7. Facchinetti F, Volpe A, Farci G, et al. Hypothalamus-pituitary-adrenal axis Comparison of serum cortisol measurement by immunoassay and liquid
of heroin addicts. Drug Alcohol Depend. 1985;15:361-366. chromatography-tandem mass spectrometry in patients receiving the
8. Qureshi AC, Bahri A, Breen LA, et al. The influence of the route of 11-hydroxylase inhibitor metyrapone. Ann Clin Biochem. 2011;48:441-446.
oestrogen administration on serum levels of cortisol-binding globulin and 24. Nasrallah MP, Arafah BM. The value of dehydroepiandrosterone sulfate
total cortisol. Clin Endocrinol (Oxf). 2007;66:632-635. measurements in the assessment of adrenal function. J Clin Endocrinol
9. Boonen E, Vervenne H, Meersseman P, et al. Reduced cortisol metabolism Metab. 2003;88:5293-5298.
during critical illness. N Engl J Med. 2013;368:1477-1488. 25. Sayyed Kassem L, El Sibai K, Chaiban J, Abdelmannan D, Arafah BM.
10. Marik PE. Critical illness-related corticosteroid insufficiency. Chest. Measurements of serum DHEA and DHEA sulphate levels improve the
2009;135:181-182. accuracy of the low-dose cosyntropin test in the diagnosis of central
11. Sprung CL, Annane D, Keh D, et al. Hydrocortisone therapy for patients adrenal insufficiency. J Clin Endocrinol Metab. 2012;97:3655-3662.
with septic shock. N Engl J Med. 2008;358:111-124. 26. Stewart PM, Corrie J, Seckl JR, Edwards CR, Padfield PL. A rational
12. Hamrahian AH, Oseni TS, Arafah BM. Measurements of serum free approach for assessing the hypothalamo-pituitary-adrenal axis. Lancet.
cortisol in critically ill patients. N Engl J Med. 2004;350:1629-1638. 1988;1(8596):1208-1210.
13. Raff H, Brock S, Findling JW. Cosyntropin-stimulated salivary cortisol in 27. Al-Aridi R, Abdelmannan D, Arafah BM. Biochemical diagnosis of adre-
hospitalized patients with hypoproteinemia. Endocrine. 2008;34:68-74. nal insufficiency: the added value of dehydroepiandrosterone sulfate mea-
14. Arafah BM, Nishiyama FJ, Tlaygeh H, Hejal R. Measurement of salivary surements. Endocr Pract. 2011;17:261-270.
cortisol concentration in the assessment of adrenal function in critically ill 28. Auchus RJ, Shewbridge RK, Shepherd MD. Which patients benefit from
subjects: a surrogate marker of the circulating free cortisol. J Clin provocative adrenal testing after transsphenoidal pituitary surgery? Clin
Endocrinol Metab. 2007;92:2965-2971. Endocrinol (Oxf). 1997;46:21-27.
15. Arafah BM. Hypothalamic pituitary adrenal function during critical ill- 29. Taylor LK, Auchus RJ, Baskin LS, Miller WL. Cortisol response to
ness: limitations of current assessment methods. J Clin Endocrinol Metab. operative stress with anesthesia in healthy children. J Clin Endocrinol
2006;91:3725-3745. Metab. 2013;98:3687-3693.

The Endocrine Society. Downloaded from press.endocrine.org by [${individualUser.displayName}] on 12 January 2017. at 12:19 For personal use only. No other uses without permission. . All rights reserved.
10 ENDO 2016 MEET-THE-PROFESSOR CLINICAL CASE MANAGEMENT

Evaluation and Management of the Adrenal Mass

CMF10 these components of the adrenal gland produce vital steroid


Presented, April 1 4, 2016 hormones such as cortisol, aldosterone, and sex hormones, as
well as catecholamines such as epinephrine, norepinephrine,
and dopamine.
Anand Vaidya, MD, MMSc; Mouhammed Amir The rapid proliferation of noninvasive imaging has given
Habra, MD. Division of Endocrinology, Diabetes and rise to unexpected and incidentally discovered findings. An
Hypertension, Brigham and Womens Hospital/Harvard incidentally discovered adrenal mass is generally defined as an
Medical School, Boston, Massachusetts 02115, E-mail: adrenal tumor, usually 1 cm or more in diameter, that is
anandvaidya@bwh.harvard.edu; and The University of observed in the absence of signs or symptoms suggestive of
Texas M D Anderson Cancer Center, Department of adrenal disorders by imaging studies ordered for nonadrenal
Endocrine Neoplasia and Hormonal Disorders, Houston, indications. The precise incidence and prevalence of adrenal
Texas 77030, E-mail: mahabra@mdanderson.org tumors is unknown; however, prevalence estimates between 1
and 7% are accepted as a general approximation based on
INTRODUCTION autopsy and imaging series (5 8). Importantly, the prevalence
Historical Overview of adrenal masses is significantly higher with older age. Adre-
The first written description of the adrenal glands as suprarenal nal masses are rare in individuals less than 30 years of age
organs is often attributed to Galen, dating back nearly 2000 (1%), but may be seen in up to 7% of individuals over the
years (1). More detailed anatomical and physiological insights age of 70 years (9). Therefore, in locations where abdominal
into the location and function of the adrenal glands were imaging is frequently used, clinicians should anticipate a sig-
produced over the subsequent centuries, with notable contribu- nificant number of incidentally discovered adrenal masses.
tions by Batholemeus Eustachius in the 1500s, Thomas The assessment of patients with adrenal tumors should in-
Wharton in the 1600s, and Thomas Addison in the 1800s (1, clude evaluation for both the malignant potential of the adrenal
2); however, the first description of an adrenal tumor is attrib- mass as well as the potential for adrenal hormone overproduc-
uted to Felix Frankel in 1886 (3). Dr Frankel described the case tion. Adrenal hormone excess may result in subtle or overt
of Minna Roll, a young woman who died suddenly following clinical manifestations associated with long-term cardiovascu-
repeated attacks of paroxysmal anxiety, in whom bilateral ad- lar, musculoskeletal, and/or metabolic complications. Most ad-
renal tumors were discovered on autopsy. In 2007, a study of renal masses represent benign and nonfunctional entities; how-
Minna Rolls extended family and descendents confirmed ever, the determination of benign or malignant status requires
germline mutations in RET as well as a family history of careful consideration of clinical and radiographic evidence.
medullary thyroid cancer and pheochromocytoma; therefore, Rarely, adrenal masses are malignant. This session will
implicating Minna Roll as the first case of pheochromocytoma review the approach to an incidentally discovered adrenal
described (3, 4). mass, with emphasis on the clinical elucidation of whether a
The advent and proliferation of rapid cross-sectional ab- mass is benign or malignant, and hormonally functional or
dominal imaging in the 20th and 21st centuries has resulted in nonfunctional.
the frequent incidental discovery of adrenal masses, often
termed adrenal incidentalomas. Given that adrenal masses BARRIERS TO OPTIMAL PRACTICE
have the potential to be benign or malignant, as well as hor- Evidence-based guidelines on the management of adrenal
monally active or inactive, practicing clinicians in 2016 are masses rely on small retrospective studies and expert consen-
faced with the challenge of characterizing adrenal masses as sus. There is a lack of longitudinal prospective and intervention
benign and nonfunctional, benign and functional, or malignant. studies to inform the most efficient and cost-effective method
Given the increasing prevalence of incidentally discovered ad- to use imaging in the surveillance of adrenal masses, when to
renal masses, efficient and evidence-based diagnostic and man- employ surgical resection, and regarding the duration and fre-
agement approaches are needed. quency of testing for hormone excess.

SIGNIFICANCE OF THE CLINICAL PROBLEM LEARNING OBJECTIVES


Although considered a single organ, the adrenal glands are As a result of participating in this session, learners should be
composed of two distinct areas: the cortex and the medulla. familiar with:
The adrenal cortex serves as a factory for adrenal steroids The differential diagnosis of an adrenal mass.
whereas the medulla originates from neural crest cells and Initiating the biochemical evaluation of an incidentally
produces catecholamines as a neuroendocrine organ. Together, discovered adrenal mass.

The Endocrine Society. Downloaded from press.endocrine.org by [${individualUser.displayName}] on 12 January 2017. at 12:19 For personal use only. No other uses without permission. . All rights reserved.
ENDO 2016 ADRENAL/HPA AXIS 11

Evaluating the radiographic phenotype of an adrenal thought to be nonfunctional may actually produce very small
mass. amounts (or unmeasured forms) of adrenal hormones that con-
The indications for biopsy of an adrenal mass. tribute to subclinical disease (10, 11, 15).
Current evidence regarding the surveillance and Therefore, the diagnostic approach should involve evalua-
management of adrenal masses. tion of three crucial parameters:
Although this session will review the evaluation for The clinical presentation of the patient.
particular states of adrenal hormone excess, this session The biochemical phenotype of the patient to assess for
will not cover the detailed management of disorders of hormone excess.
overt adrenal hormone excess (such as primary The radiographic characteristics to assess for malignant
aldosteronism, Cushings syndrome, or potential.
pheochromocytoma).
Clinical Presentation
STRATEGIES FOR DIAGNOSIS, THERAPY, In patients with adrenal masses, detailed medical history is
AND/OR MANAGEMENT essential to guide subsequent testing and management. In par-
Differential Diagnosis and Diagnostic Approach ticular, prior personal history of malignancy (especially breast
The diagnostic approach to an incidentally discovered adrenal cancer, lung cancer, melanoma, colorectal cancer, prostate can-
mass is focused on investigating whether the mass is benign or cer, and renal cell carcinoma) significantly increases the possi-
malignant, and whether it is functional (produces excessive bility of metastasis to the adrenal gland and may warrant
adrenal hormone such as aldosterone, cortisol, or sex steroids) imaging-guided biopsy after excluding pheochromocytoma.
or nonfunctional (Table 1). Excessive cortisol production may Family history is equally important given that adrenal masses
result in the Cushings syndrome, but chronic subclinical originating from the cortex can be seen in multiple hereditary
hypercortisolism is associated with increased risk for cardio- syndromes (multiple endocrine neoplasia type 1, Lynch syn-
vascular and metabolic diseases and death (10 12). Similarly, drome, familial adenomatous polyposis, Li-Fraumeni syn-
excessive aldosterone production in primary aldosteronism can drome, congenital adrenal hyperplasia, and Carneys com-
cause the classic clinical picture of resistant hypertension and plex) whereas tumors originating from the adrenal medulla
hypokalemia, but chronic subclinical exposure to inappropri- (pheochromoctyoma) can be seen in the context of familial
ately high aldosterone may also increase the risk of cardiovas- pheochromocytoma-paraganglioma syndromes associated with
cular and metabolic diseases by activating the mineralocorti- succinate-dehydrogenase mutations, Von-Hipple Lindau syn-
coid receptor (13, 14). Excessive catecholamine production drome, neurofibromatosis type 1, and multiple endocrine neo-
associated with pheochromocytomas can contribute to or worsen
plasia type 2. The clinical assessment for adrenal hormone
cardiovascular and psychiatric diseases. In recent observational
excess should include assessment for features of aldosterone
studies, even adrenal masses with no detectable hormone produc-
excess (resistant or difficult to control hypertension, hypokale-
tion are associated with poorer cardiometabolic profiles and po-
mia), cortisol excess (weight gain, difficulty to control blood
tentially higher cardiovascular risk, suggesting that tumors
pressure [BP], hyperglycemia, ostopenia or osteoporosis, easy
bruising, proximal muscle myopathy, and others), and cate-
TABLE 1. The Differential Diagnosis for an Incidentally cholamine excess (paroxysmal hypertension, anxiety, sweating,
Discovered Adrenal Mass palpitations). Isolated sex hormone excess is extremely rare;
Nonfunctional Functional however, androgen and estrogen excess is not an uncommon
feature of adrenocortical carcinoma (ACC), which may also
Benign Adenoma Adenoma
(aldosterone or present with cortisol excess in more than 50% of cases. Fur-
Myelolipoma
cortisol producing) ther, a general assessment for features suggestive of malig-
Ganglioneuroma
nancy should also be performed (anorexia, weight loss, night
Cyst/pseudocyst Micro- or macronodular
disease (aldosterone sweats, anemia).
Hemorrhage
Infection (fungal, or cortisol
mycobacterial, producing)
Biochemical Phenotype
hydatid cyst) All adrenal masses should be evaluated for the presence of
Hemangioma Pheochromocytoma cortisol excess even in the absence of clinical signs or symp-
Malignant Adrenocortical carcinoma Adrenocortical toms of Cushings syndrome given the cardiometabolic risks
Neuroblastoma carcinoma
associated with chronic subclinical hypercortisolism (10 12).
Sarcoma This is most commonly assessed using a 1-mg dexamethasone
Primary adrenal lymphoma suppression test (Table 2); however, other methods of assess-
Metastatic cancer from a Pheochromocytoma ing hypercortisolism can also be used, especially when factors
non-adrenal primary
that alter dexamethasone metabolism are present. Most adrenal

The Endocrine Society. Downloaded from press.endocrine.org by [${individualUser.displayName}] on 12 January 2017. at 12:19 For personal use only. No other uses without permission. . All rights reserved.
12 ENDO 2016 MEET-THE-PROFESSOR CLINICAL CASE MANAGEMENT

TABLE 2. The Suggested Biochemical Screening Tests for Incidentally Discovered Adrenal Masses
Condition Patients Test Interpretation
Cortisol excess All 1-mg dexamethasone 1.8 mcg/dL: excludes cortisol exess
suppression test 1.85.0 mcg/dL: may suggest sub-clinical cortisol excess
5.0 mcg/dL: suggestive of cortisol excess
Catecholamine ALL, except those adrenal masses Plasma fractionated ULN: normal
excess with 10 HU attenuation on metanephrines 12.5 ULN: May represent false positive secondary
non-contrast CT and no or to medications that inhibit catecholamine re-uptake
suggestive clinical signs or 24 h urinary mechanisms or increased sympathoadrenergic tone.
symptoms fractionated May be suggestive of pheochromocytoma in the setting
metanephrines of strongly supportive clinical and radiographic features.
Higher than 4 ULN: strongly suggestive of
pheochromocytoma
Aldosterone Hypertension Serum aldosterone ARR 2025 in the setting of a suppressed plasma renin
excess Hypokalemia to plasma renin activity strongly suggestive of hyperaldosteronism
activity ratio (ARR)
Adrenal androgen Hirsutism or virilization DHEAS Higher than ULN
excess Total testosterone

Abbreviation: ARR, aldosterone renin ratio; ULN, upper limit of normal reference range.

masses should also be screened for catecholamine excess given the mass. In general, the more lipid-rich an adrenal mass is, the
the cardiovascular risks associated with subclinical or clinically more likely it represents a benign entity. An unenhanced CT
apparent pheochromocytoma, and the fact that many patients attenuation of less than 10 HU is strongly supportive of a
with pheochromocytoma present without hypertension or other lipid-rich mass and almost always represents a benign entity
classical symptoms associated with catecholamine excess (16). (19, 20). Very low densities, such as 30 to 50 HU, are
The simplest and most sensitive method to screen for catechol- strongly suggestive of myelolipomas, which are generally be-
amine excess is measurement of plasma-fractionated metanephrines; nign tumors. When unenhanced CT attenuations of greater than
however, 24-hour urinary fractionated metanephrines also pro- 10 HU are encountered, the differential diagnosis includes a
vide highly sensitive and fairly specific results (17). In cases in lipid-poor benign adrenal adenoma, or a more vascular tumor
which the adrenal mass displays a noncontrast computed to- such as a pheochromocytoma, or a malignant entity (either
mography (CT) attenuation of less than 10 Hounsfield units primary adrenal or extra-adrenal metastases) or infiltrative in-
(HU), the probability of a pheochromocytoma is almost zero, fection. In these situations, using an iv contrast adrenal wash-
and in these cases testing for catecholamine excess may be out protocol may be helpful, where postcontrast imaging at 1
deemed unnecessary unless there are other clinical features and 15 minutes to evaluate the contrast avidity of the adrenal
suggestive of pheochromocytoma (18, 19). Screening for hy-
peraldosteronism can be performed with a serum aldosterone-
to-plasma renin ratio and should be particularly considered in TABLE 3. Radiographic Characteristics of Adrenal
patients with an adrenal mass associated with hypertension or Masses to Determine Benign or Malignant Potential
hypokalemia. Testing for adrenal androgens or estrogen should be Likely Potentially
considered when clinical signs of virilization (hirsutism, acne, oily Characteristic Benign Malignant
skin, deepening of the voice) are concerning, and in these situa- Size, cm 4 4-6
tions the concern for ACC should be increased. Attenuation on unenhanced CT, HU 10 10
Contrast washout on CT protocol at 50-60 50
15 min, %
Radiographic Characteristics
MRI chemical shift suggestive of Yes No
The radiographic features of an incidentally discovered adrenal
lipid-rich content
mass provide useful information to determine its etiology and
FDG avidity on PET No Yes
particularly whether it may be benign or malignant. Features
Irregular borders, heterogeneous content, No Yes
such as large size, hetereogeneity, calcifications, necrosis, in- calcifications, necrosis
creased vascularity, and rapid rate of growth are all concerning Rate of growth, cm/y 1 1
for a malignant adrenal tumor or metastases (Table 3).
The most common and well-described imaging modality is Abbreviation: MRI, magnetic resonance imaging.
CT. CT provides excellent information on size, homogeneity, Adapted from Miller and Doherty (22).
calcifications, and valuable information on the lipid content of

The Endocrine Society. Downloaded from press.endocrine.org by [${individualUser.displayName}] on 12 January 2017. at 12:19 For personal use only. No other uses without permission. . All rights reserved.
ENDO 2016 ADRENAL/HPA AXIS 13

mass is calculated. Masses that retain less than 50% of contrast of malignant features such as growth of the tumor. There is no
material at 15 minutes (washout 50 60%) are very likely to defined consensus on how long patients should be monitored.
be benign (20, 21). A larger size of the adrenal mass may Expert consensus generally recommends annual evaluation of
increase the risk for malignancy; however, size should be the clinical and biochemical phenotypes for up to 4 years,
evaluated in the context of other features that may support whereas for a stable radiographic phenotype for 2 years may be
malignancy. For example, the risk for a malignant entity in- sufficient (7, 8); however, shorter followup may be considered
creases substantially when an adrenal mass is greater than 4 in those deemed to be very low risk (ie, elderly patients and
cm, but if the mass exhibits an unenhanced CT attenuation those with prior imaging suggesting a chronic process), and
of less than 10 HU and no other malignant radiographic longer followup may be considered in those deemed to be at
characteristics, size by itself may not provide sufficient higher risk (ie, younger patients or irregular radiographic find-
support for a malignant etiology (19, 20). ings that are suspicious but not definitive for malignancy).
Other imaging modalities, such as magnetic resonance im- Longitudinal prospective studies are needed to better define the
aging and positron emission tomography, can also provide optimal surveillance frequency and duration.
valuable information when assessing adrenal masses (22)
(Table 3).
Complications and Prognosis
The health complications of adrenal masses are directly related
Management to their malignant potential or biochemical function. Adrenal
The suggested clinical management and approach to an inci- masses that are assessed to be benign and nonfunctional are
dentally discovered adrenal mass is outlined in Figure 1. Sur- currently considered to not pose any health risks. However, a
gery should be considered when hormone excess is confirmed growing field of research has increasingly observed that some
or when radiographic features suggestive of malignancy are
adrenal masses deemed to be nonfunctional by current stan-
detected. The role of biopsy is typically restricted to scenarios
dards may still produce subclinical levels or adrenal hormones
when the differential diagnosis involves an extra-adrenal me-
that evade detection (such as mineralocorticoid and/or glucocorti-
tastases to the adrenal for which systemic medical therapy
coid agonists), and that this mild hormonal excess may result in
would be the most appropriate next step, there is suspicion for
cardiometabolic diseases over time (10 12, 15). In the rare sce-
adrenal lymphoma or sarcoma, or when an infiltrative infection
nario that an adrenal mass represents an ACC, the complications
(often fungal or mycobacterial infections) is suspected. It is not
can be severe and the prognosis generally very poor.
advised that adrenal mass biopsy be conducted in other situa-
tions given the known difficulties in differentiating primary
benign vs malignant adrenal tumors, the risk of tumor spread in MAIN CONCLUSIONS
primary ACC, and the risk of catecholamine crises in potential With the increasing use of cross-sectional abdominal imaging,
pheochromcytomas. When neither hormone excess nor malig- the prevalence and incidence of incidentally discovered adrenal
nancy are identified, patients should be monitored prospec- masses has grown. A systematic approach for evaluating
tively for incident hormonal functionality and the development whether an adrenal mass is benign or malignant, and hormon-

Adrenal Mass

Clinical Presentation

Biochemical Phenotype (-)


(+)
Confirm Hormone
Excess
Radiographic Characteristics
Radiographic Phenotype
and Localization Benign Appearing Suspicious
<10HU, non-contrast avid, >10HU, >4-6cm, contrast avid,
homogeneous heterogeneous

Consider surgery
If Unilateral: Consider surgery

Concern for extra-adrenal


If <4cm: Biochemical and
Growth>1cm/year metastases or infection: Biopsy
Suspicious radiographic features
Radiographic Surveillance
Hormonal excess If >4-6cm: Consider surgery

FIGURE 1. Algorithm for Management of Adrenal Masses.

The Endocrine Society. Downloaded from press.endocrine.org by [${individualUser.displayName}] on 12 January 2017. at 12:19 For personal use only. No other uses without permission. . All rights reserved.
14 ENDO 2016 MEET-THE-PROFESSOR CLINICAL CASE MANAGEMENT

ally active or inactive, is necessary to efficiently manage pa- headache, or any other significant complaints. Abdominal CT
tient care. We propose evaluation of the clinical presentation revealed a heterogeneous 4-cm left kidney mass and a 3-cm
and biochemical phenotype in each patient to determine the heterogeneous left adrenal mass (precontrast density of 20
risk of hormone excess. We recommend reliance on the radio- HU).
graphic characteristics to determine the malignant potential of This adrenal mass is most likely to be a(n):
the mass. Most adrenal masses will represent benign entities. A. Adrenocortical adenoma
There is a need for more longitudinal and prospective studies to B. Adrenocortical carcinoma
determine the thresholds for subclinical adrenal hormone C. Conns tumor (primary aldosteronism)
excess that should trigger clinical action, and the optimal sur- D. Metastatic breast cancer to the adrenal gland
veillance strategies to follow adrenal masses over time. E. Myelolipoma
F. Pheochromocytoma
CASES
Case 1 Case 4
A 50-year-old normotensive woman was incidentally found to A 42-year-old woman is found to have a left adrenal mass after
have a left adrenal mass after undergoing an unenhanced ab- undergoing an unenhanced CT scan of her abdomen. The CT
dominal CT scan for pain. The mass is described as being 1.2 scan was performed after she developed a headache, abdominal
cm in size, round, with a homogenous consistency and attenu- pain, and had a syncopal event. The mass is described as being
ation of 4 HU. The right adrenal gland is normal. Her serum 2.2 cm, round, and with an attenuation of 4 HU. One year prior
potassium is 4.2 mmol/L and she takes no antihypertensive to this incident, she was diagnosed with hypertension and
medications. She has no personal history of cancer. treated with lisinopril. Six months prior to this incident she was
This adrenal mass is most likely to be a(n): advised to augment her antihypertensive regimen to include
A. Adrenocortical adenoma amlodipine. During this incident, her BP was found to be
B. Adrenocortical carcinoma 200/100 mm Hg and her serum potassium was 3.2 mmol/L.
C. Conns tumor (primary aldosteronism) This adrenal mass is most likely to be a(n):
D. Metastatic cancer to the adrenal gland A. Adrenocortical adenoma
E. Myelolipoma B. Adrenocortical carcinoma
F. Pheochromocytoma C. Conns tumor (primary aldosteronism)
D. Metastatic cancer to the adrenal gland
Case 2 E. Myelolipoma
A 71-year-old woman was found to have a new left adrenal F. Pheochromocytoma
mass after undergoing an unenhanced abdominal CT scan to
evaluate for pain. The mass is described as being 3.8 2.9
Case 5
1.9 cm, poorly circumscribed, and with an attenuation of 20
A 30-year-old woman presented with new onset of moderate
HU. On close examination of the right adrenal gland, there is a
hirsutism and acne. Laboratory evaluation showed elevations in
suggestion that it may be thickened. The patient has no history
DHEA-sulfate and total testosterone. CT scan of the abdomen
of hypertension, hypokalemia, or hyperandrogenism, but does
revealed a 7.6-cm right adrenal mass with precontrast density
have a remote history of breast cancer (estrogen receptorposi-
of 45 HU and enhancement washout of 69%. Interestingly, she
tive, progesterone receptorpositive, HER2/neu-negative) dat-
reported a history of abdominal imaging 3 years prior to this
ing back more than 10 years prior this presentation. She was
presentation where an incidental 3-cm right adrenal mass was
treated with lumpectomy, chemotherapy, and 10 years of
seen.
aromatase inhibitor therapy and considered to be in complete
This adrenal mass is most likely to be a(n):
remission. Her adrenal glands had been normal on repeated
A. Adrenocortical adenoma
staging imaging during the course of the last 10 years.
B. Adrenocortical carcinoma
This adrenal mass is most likely to be a(n):
C. Conns tumor (primary aldosteronism)
A. Adrenocortical adenoma
D. Metastatic cancer to the adrenal gland
B. Adrenocortical carcinoma
C. Conns tumor (primary aldosteronism) E. Myelolipoma
D. Metastatic cancer to the adrenal gland F. Pheochromocytoma
E. Myelolipoma
F. Pheochromocytoma DISCUSSION OF CASES AND ANSWERS
Case 1
Case 3 The correct answer is an adrenocortical adenoma. The radio-
A 68-year-old man presented with abdominal pain. He denied graphic characteristics of a small, round, and homogenous mass
any prior history of hypertension, weight changes, palpitations, with a lipid-rich attenuation on unenhanced CT are all supportive

The Endocrine Society. Downloaded from press.endocrine.org by [${individualUser.displayName}] on 12 January 2017. at 12:19 For personal use only. No other uses without permission. . All rights reserved.
ENDO 2016 ADRENAL/HPA AXIS 15

of a benign entity. Adrenocortical adenomas are the most common Case 5


benign cause of an adrenal mass. All adenomas should still be The combination of hyperandrogenism and large adrenal mass
evaluated for hormone excess; therefore, this patient should un- (6 cm) is highly suspicious for ACC. Interestingly, some
dergo testing for hypercortisolism, ideally with a 1-mg dexameth- patients with ACC report having prior imaging for other rea-
asone suppression test. Testing for hyperaldosteronism is unlikely sons and this can help documenting the growth rate of the
to be informative given her normal BP and serum potassium, and adrenal mass (23, 24). Adrenal masses that grow more than 1
testing for pheochromocytoma is also unlikely to be informative cm per year raise the suspicion of malignancy and require
given that pheochromocytomas are extremely unlikely when the surgical resection. The diagnosis of ACC is often based on nine
attenuation of a mass is less than 10 HU. pathological features (Weiss criteria) with tumors having
Weiss score of 3 being classified as ACC whereas tumors
Case 2 with a Weiss score of 12 being classified as atypical adeno-
The correct answer is metastatic breast cancer to the adrenal mas (or adrenal neoplasms of undetermined malignant poten-
gland. The radiographic characteristics of a relatively large, heter- tial) such as this case.
ogeneous, and nonlipid-rich mass should raise concern for a ma-
lignant entity. The fact that this mass was not apparent on prior REFERENCES
serial images should raise further concern of a relatively rapid- 1. Leoutsakos B, Leoutsakos A. The adrenal glands: A brief historical per-
spective. Hormones (Athens). 2008;7:334-336.
growing entity. The patients personal history of breast cancer, in 2. Shifrin A. The history of adrenal gland tumors. 2012. Available at:
combination with the fact that the contralateral adrenal gland may http://www.adrenaltumors.org/history.
also be affected, raises the suspicion of metastatic disease to the 3. Neumann HP, Vortmeyer A, Schmidt D, et al. Evidence of MEN-2 in the
original description of classic pheochromocytoma. N Engl J Med.
adrenal glands; however, infiltrative infections should also be 2007;357:1311-1315.
considered in this situation. An adrenal biopsy may be useful in 4. Frankel F. Ein Fall von doppelseitigem, vollig latent verlaufenen
this scenario given that the treatment of stage 4 breast cancer, or Nebennierentumor und gleichzeitiger Nephritis mit Veranderungen am
Circulationsapparat und Retinitis. Arch Pathol Anat Physiol Klin Med.
an infiltrative infection, could alter the subsequent decision to 1886;103:244-263.
pursue medical vs surgical treatments. 5. Barzon L, Sonino N, Fallo F, Palu G, Boscaro M. Prevalence and natural
history of adrenal incidentalomas. Eur J Endocrinol. 2003;149:273-285.
6. Bovio S, Cataldi A, Reimondo G, et al. Prevalence of adrenal
Case 3 incidentaloma in a contemporary computerized tomography series. J
In patients with a history of malignancy known to metastasize Endocrinol Invest. 2006;29:298-302.
to the adrenal gland, the presence of a heterogeneous adrenal 7. Nieman LK. Approach to the patient with an adrenal incidentaloma. J Clin
Endocrinol Metab. 2010;95:4106-4113.
mass is most likely to represent metastasis. The presence of
8. Young WF Jr. Clinical practice. The incidentally discovered adrenal mass.
both a heterogeneous and dense renal mass and adrenal mass N Engl J Med. 2007;356:601-610.
should also raise concern for pheochromocytoma and an inher- 9. Young WF Jr. Management approaches to adrenal incidentalomas. A view
from Rochester, Minnesota. Endocrinol Metab Clin North Am. 2000;29:
itable pheochromocytoma-paraganglioma tumor syndrome. It
159-185, x.
is now estimated that up to 40% of all pheochromocytomas and 10. Di Dalmazi G, Vicennati V, Garelli S, et al. Cardiovascular events and
paraganglioms are attributed to a germline mutation in one of mortality in patients with adrenal incidentalomas that are either non-
15 genes. A significant number of these gene mutations (VHL, secreting or associated with intermediate phenotype or subclinical
Cushings syndrome: A 15-year retrospective study. Lancet Diabetes
SDHA, SDHB, SDHC, SDHC, TMEM127, FH) also involve a Endocrinol. 2014;2:396-405.
lifetime risk of developing a renal cell carcinoma. Pheochro- 11. Androulakis, II, Kaltsas G, Kollias GE, et al. Patients with apparently
mocytoma can be hard to distinguish radiologically from other non-functioning adrenal incidentalomas may be at increased cardiovascu-
lar risk due to excessive cortisol secretion. J Clin Endocrinol Metab.
heterogeneous adrenal tumors and must be ruled out prior to 2014:jc20134064.
surgery or imaging guided biopsy. 12. Morelli V, Reimondo G, Giordano R, et al. Long-term follow-up in
adrenal incidentalomas: an italian multicenter study. J Clin Endocrinol
Metab. 2014;99:827-834.
Case 4 13. Brown JM, Underwood PC, Ferri C, et al. Aldosterone dysregulation with
The correct answer is a Conns tumor. The radiographic char- aging predicts renal vascular function and cardiovascular risk. Hyperten-
acteristics of a small, round, and lipid-rich mass are supportive sion. 2014;63:1205-1211.
14. Vaidya A, Underwood PC, Hopkins PN, et al. Abnormal aldosterone
of a benign entity. All benign adrenal masses should still physiology and cardiometabolic risk factors. Hypertension. 2013;61:886-
undergo biochemical evaluation for subclinical or overt adrenal 893.
hormone excess. In this case, the history of severe hypertension 15. Tuna MM, Imga NN, Dogan BA, et al. Non-functioning adrenal
incidentalomas are associated with higher hypertension prevalence and
and hypokalemia in this young patient should raise particular
higher risk of atherosclerosis. J Endocrinol Invest. 2014;37(8):765-768.
concern for primary aldosteronism and to a lesser degree 16. Mannelli M, Lenders JW, Pacak K, Parenti G, Eisenhofer G. Subclinical
hypercortisolism or catecholamine excess. The evaluation phaeochromocytoma. Best Pract Res Clin Endocrinol Metab. 2012;26:507-
should include serum aldosterone and plasma renin activity, as 515.
17. Lenders JW, Duh QY, Eisenhofer G, et al. Pheochromocytoma and para-
well as a 1-mg dexamethasone suppression test and plasma- ganglioma: an endocrine society clinical practice guideline. J Clin
fractionated metanephrines. Endocrinol Metab. 2014;99:1915-1942.

The Endocrine Society. Downloaded from press.endocrine.org by [${individualUser.displayName}] on 12 January 2017. at 12:19 For personal use only. No other uses without permission. . All rights reserved.
16 ENDO 2016 MEET-THE-PROFESSOR CLINICAL CASE MANAGEMENT

18. Motta-Ramirez GA, Remer EM, Herts BR, Gill IS, Hamrahian AH. Com- 21. Caoili EM, Korobkin M, Francis IR, et al. Adrenal masses: Characteriza-
parison of CT findings in symptomatic and incidentally discovered pheo- tion with combined unenhanced and delayed enhanced CT. Radiology.
chromocytomas. AJR Am J Roentgenol. 2005;185:684-688. 2002;222:629-633.
19. Hamrahian AH, Ioachimescu AG, Remer EM, et al. Clinical utility of 22. Miller BS, Doherty GM. Surgical management of adrenocortical tumours.
Nat Rev Endocrinol. 2014;10:282-292.
noncontrast computed tomography attenuation value (hounsfield units) to
23. Nogueira TM, Lirov R, Caoili EM, et al. Radiographic characteristics of
differentiate adrenal adenomas/hyperplasias from nonadenomas: Cleveland
adrenal masses preceding the diagnosis of adrenocortical cancer. Horm
Clinic experience. J Clin Endocrinol Metab. 2005;90:871-877. Cancer. 2015;6:176-181.
20. Korobkin M, Brodeur FJ, Yutzy GG, et al. Differentiation of adrenal 24. Ozsari L, Kutahyalioglu M, Elsayes KM, et al. Preexisting adrenal masses
adenomas from nonadenomas using CT attenuation values. AJR Am J in patients with adrenocortical carcinoma: clinical and radiological factors
Roentgenol. 1996;166:531-536. contributing to delayed diagnosis. Endocrine. 2015. [Epub ahead of print].

The Endocrine Society. Downloaded from press.endocrine.org by [${individualUser.displayName}] on 12 January 2017. at 12:19 For personal use only. No other uses without permission. . All rights reserved.
ENDO 2016 ADRENAL/HPA AXIS 17

Management of Classical CAH: From Birth to Adulthood

M01 complications such as short stature, hypertension (HTN), obe-


Presented, April 1 4, 2016 sity, insulin resistance, and infertility. Furthermore, glucocorti-
coid therapy is far from perfect, resulting in either overtreat-
ment or suboptimal suppression of adrenal androgen secretion.
Maria G. Vogiatzi, MD. Department of Endocrinology Optimizing care and addressing all the medical needs of these
and Diabetes, Childrens Hospital of Philadelphia, patients becomes, therefore, very important.
Philadelphia, Pennsylvania 19104, E-mail: vogiatzim@email.
chop.edu
BARRIERS TO OPTIMAL PRACTICE
Children and adults with CAH are at an increased risk for
INTRODUCTION various complications such as short stature, obesity, insulin
Historical Overview resistance, infertility and a reduced quality of life (QOL). These
Congenital adrenal hyperplasia (CAH) was first documented by are related to either excessive treatment with glucocorticoids or ad-
Luigi DeCrecchio, an Italian anatomist, in 1865. Upon autopsy, renal androgen excess.
DeCrecchio found a female patient to have enlarged adrenal The goal of therapy in CAH is to provide optimal control of
glands, male seeming genitals but no testes, and an internal adrenal androgen secretion while avoiding overtreatment with
female reproductive system. There was no effective medical or glucocorticoids. However, recent studies suggest that hormonal
surgical treatment for CAH until 1950 when the discovery was control is frequently not optimal and that many patients are
made independently by Wilkins et al (18) at John Hopkins either overtreated or undertreated.
Hospital and Bartter et al (19) at Massachusetts General
Hospital that cortisone suppressed the elevated urinary
17-ketosteroids. This provided the first clue that the true basis LEARNING OBJECTIVES:
As a result of participating in this session, learners should be
of the disease was inadequate corticoid production. In 1962,
able to:
the 21-hydroxylase deficiency form of CAH was discovered to
Detail the more frequent complications of untreated
be transmitted as a genetic trait that affects both males and
CAH in both children and adults including short stature,
females. A better understanding of the genetic transmission of
HTN, obesity, insulin resistance, infertility, and an
CAH was gained in 1978, when the gene for 21-hydroxylase
impaired QOL.
deficiency was discovered to be located on the short arm of the
sixth chromosome. At an International Newborn Screening Understand the relationship between genotype and
Meeting held in Tokyo in 1982, CAH was recommended as a glucocorticoid therapy with regard to health,
disease that meets the criteria to be included in newborn psychosexual adjustment, and QOL in individuals with
screening systems. classical CAH.

SIGNIFICANCE OF THE CLINICAL PROBLEM STRATEGIES FOR DIAGNOSIS, THERAPY,


CAH refers to a group of inherited autosomal-recessive disor- AND/OR MANAGEMENT
ders that lead to defective steroidogenesis. The most common Beyond optimizing adrenal control, individuals with CAH have
form of CAH, 21-hydroxylase deficiency (21OHD), accounts multiple medical needs that may vary across life span (Figure
for approximately 95% of all cases. In Its severe form, named 1). Herein, we present various cases to review some frequent or
classical CAH, 21OHD is associated with cortisol and/or newly appreciated complications that occur in patients with
mineralocorticoid deficiency (salt wasting or simple viriliz- classical CAH.
ing types respectively). In this presentation, the term CAH
refers to 21OHD. CASES AND DISCUSSION
Classical CAH is a potentially fatal disorder. With the intro- Case 1
duction of glucocorticoid therapy in the 1950s and the applica- A 20-month-old girl with salt-wasting CAH is found to have a
tion of newborn screening, patients with CAH are living lon- blood pressure (BP) of 133/105 mm Hg during a viral illness.
ger. As the number of affected adults is increasing, CAH has Medications: Hydrocortisone, 5 mg twice daily or 18
become a lifelong chronic disease. A number of recent studies mg/m2/d
have shed light into the health and quality of life (QOL) Fludrocortisone, 0.15 mg daily
outcomes of these patients. The results suggest that many live a Na supplements, 500 mg in the morning and 1000 mg in
healthy and productive life and are well adjusted psychoso- the evening
cially. However, a significant number suffer from unwanted Genotype: R356W/Intron 2G

The Endocrine Society. Downloaded from press.endocrine.org by [${individualUser.displayName}] on 12 January 2017. at 12:19 For personal use only. No other uses without permission. . All rights reserved.
18 ENDO 2016 MEET-THE-PROFESSOR CLINICAL CASE MANAGEMENT

Figure 1. Medical needs and frequent health concerns in classical CAH from birth to adulthood.

Laboratory results: Na, 142 mEq/L; K, 3.0 mEq/L; 17 is between 9 and 10 years, and height prediction by BP is
hydroxyprogesterone (17 OH Prog), 230 ng/dL; practically unchanged at 168 cm (close to 66).
androstendione, 16 ng/dL; T 3 ng/dL; renin, 0.1 ng/mL/h 4. What is the typical height deficit in CAH and what are
Heart echocardiogram: moderate left ventricular the factors that influence height outcomes?
hypertrophy 5. Is there a role for GH therapy in such cases?
Disposition: fludrocortisone was decreased to 0.05 mg 6. Should he be monitored for central precocious puberty?
daily
Na supplementation was discontinued
She was started on nifedipine and propranolone Discussion
Outcome: HTN resolved after 23 years A recent meta-analysis by Muthusamy et al (1) suggests that
final height in CAH is at 1.38 SDS (1.56 to 1.20; I2
1. What are the rates of HTN in individuals with CAH?
90.2%), whereas final height SDS midparental height SDS is
2. What factors contribute to the development of HTN in
1.03 (1.20 to 0.86; I2 63%). Although there is variability
CAH?
3. Do rates of HTN increase with age? among studies, final height SDS was not associated with gen-
der, age of onset of puberty, type or dose of steroids. GH,
frequently used in combination with pubertal suppression, im-
Discussion
proves final height in nonrandomized studies. Most of the
Hypertension is common, particularly in children. Studies
document a relationship to body mass index and children with CAH experience normal puberty, but there is a
fludrocortisone dose. Mineralocorticoid replacement therapy subgroup with sexual precocity. Menarche usually occurs at a
should avoid suppressing plasma levels of renin below the normal age.
normal range, and blood pressure (BP) should be monitored
regularly in both children and adults. Case 3
A 20-year-old noncompliant men with classical CAH com-
Case 2 plains of right testicular pain.
A 6-year-old boy with classical CAH is found to have a bone Sonogram: right testis, 4-mL mass; left testis, 1-mL
age between 8 and 9 years during routine endocrine followup. mass
Physical Examination: Height, 75%; weight, 50th Laboratory results: 17 OH Prog, 4260 mg/dL;
percentile; evidence of adrenarche (Pubic hair Tanner II; androstendione, 440 ng/dL; T, 420 mg/dL; LH, 2.1 IU/L,
testis, 2 mL)
FSH, 3.1 IU/L
Family history: midparental height, 50; height prediction
Sperm Count: moderate oligospermia
by BP, 167 cm (close to 66)
Meds: Hydrocortisone, 18 mg/m2/d; fluodrocortisone, 0.1
Medications: hydrocortisone, 15 mg/m2/d;
fludrocortisone, 0.1 mg daily mg daily
Laboratory results: 17 OH Prog, 1200 ng/dL; 7. What are the rates of Testicular Adrenal Rest Tumor
androstendione, 64 ng/dL; T, 10 ng/dL (TART) and the most appropriate management?
A year later at age 7 years, and after improving adrenal 8. What are the fertility rates in males with classical CAH
control, his growth rate has slowed down, the repeat bone age and the factors leading to reduced fertility?

The Endocrine Society. Downloaded from press.endocrine.org by [${individualUser.displayName}] on 12 January 2017. at 12:19 For personal use only. No other uses without permission. . All rights reserved.
ENDO 2016 ADRENAL/HPA AXIS 19

TABLE 1. Hydrocortisone (HC) Pharmacokinetics in 48 Individuals With Classical CAH (6-20 yr)
Hindmarsh PC Clin Endocrinology 2015
Oral HC (Cmax) (mcg/dl) 28 2.2
(tmax) (min) 66.7 5.9 (range: 20118)
Time to 3.6 mcg/dl (min) 289 15 (range 140540)
IV HC (15 mg/m2) Half-life (min) 76.5 5.2 (range: 40225 min)

Discussion Case 5
Fertility rates are decreased in men with classical CAH; testicular A 4-year-old boy has diagnosed with classical CAH by new-
adrenal rest tumors are a common cause of infertility, require surveil- born screening. He has been treated with glucocorticoids and
lance with repeated ultrasonography, and can respond to therapy with mineralocorticoids since 10 days of life. The family is contem-
glucocorticoids. In addition, dysregulation of hypothalamic-pituitary- plating a second pregnancy. The genotype of the patient was
gonadal axis and Leydig and Sertoli cell dysfunction also contribute performed and revealed a deletion in both CYP21A2 alleles.
to the decreased fertility rates in males. 13. What are the main concerns about prenatal therapy in
CAH?
14. What is the role of free fetal DNA determination in
Case 4
prenatal diagnosis and therapy?
A 17-year-old girl was diagnosed with classical CAH at birth
when she presented with ambiguous genitalia. She has been
treated with glucocorticoids and mineralocorticoids since 7 Discussion
days of life and underwent genital surgery at first year of life. Dexamethasone is effective in reducing or eliminating viriliza-
tion of affected female fetal genitalia. However, it is a category
The family is very compliant, and except for a body mass index
B drug, whose safety in pregnancy is not established. Systemic
greater than the 95th percentile, the child is doing well and has
review and meta-analysis of publications that examined preg-
never experienced salt wasting. She is now ready to go to
nancy outcomes suggest no statistically significant effect on
college. Parents question the need for fludrocortisone therapy
birth weight and no increased teratogenicity. A small but rig-
and are worried about the long-term effect of surgery and
orous study showed worse verbal working memory, lower
steroids on sexuality and QOL. Genotype of the patient was self-perception of scholastic competency, and increased self-
performed and revealed an Int2 mutation in one CYP21A2 rated anxiety among exposed, unaffected children. It is advised
allele and a deletion in the other. that prenatal therapy to be considered only under internal
9. Is this patient a salt waster? What are the phenotype- review boardapproved protocols. Lately, early sex determina-
genotype correlations in CAH? tion using free fetal DNA in maternal circulation around 5-6
10. Does she need a urological/gyn evaluation? weeks of gestation has been shown to be effective in prevent-
11. Is there an effect of genotype on gender behavior, ing exposure of male fetuses to prenatal dexamethasone. Suc-
sexuality, and family planning? cessful prenatal diagnosis using free fetal DNA has been re-
12. What are the factors that may affect QOL in CAH? ported by Dr News group (15).

Discussion Case 6
A 25-year-old woman with salt-wasting CAH is contemplating
In general, the genotype correlates well with the severity of
pregnancy.
cortisone and aldosterone deficiencies, especially in salt-
14. What is true in her case?
wasting and nonclassical (ie, mild) forms. Gender-atypical be-
A. She is at increased risk for miscarriage and other
havior in women with CAH also correlates with genotype.
gestational complications such as diabetes and pre-
However, gender identity disorder is rare in these patients. The
eclampsia
results on QOL in adults are inconsistent. Delayed start to B. She will most likely require cesarean section
sexual activity and fewer relationships have been reported in C. Adrenal androgens increase during pregnancy and
both women and men. Psychosexual difficulties in women have she is likely to have multiple adjustments of her
been linked to genotype and the type of corrective surgery. glucocorticoid doses
However, these results reflect older surgical techniques and D. Babies born to mothers with CAH have low birth
data with most recent techniques are sparse. Finally, certain weight
studies have linked obesity with prednisone or dexamethasone E. Baby girls born to mothers with CAH can be viril-
therapy and impaired QOL. ized if maternal adrenal control is poor

The Endocrine Society. Downloaded from press.endocrine.org by [${individualUser.displayName}] on 12 January 2017. at 12:19 For personal use only. No other uses without permission. . All rights reserved.
20 ENDO 2016 MEET-THE-PROFESSOR CLINICAL CASE MANAGEMENT

Discussion born with classic 21-hydroxylase deficiency. A French national survey.


Spontaneous miscarriages have been reported at higher rates J Clin Endocrinol Metab. 2015;100(6):2303-2313.
7. Han TS, Conway GS, Willis DS, et al. Relationship between final height
among glucorticoid-untreated women compared with the gen- and health outcomes in adults with congenital adrenal hyperplasia: United
eral population. Cesarean section is usually performed in indi- Kingdom congenital adrenal hyperplasia adult study executive (CaHASE).
J Clin Endocrinol Metab. 2014;99(8):E1547-E1555.
viduals with prior genital reconstructive surgery. There is no
8. Arlt W, Willis DS, Wild SH, et al. Health status of adults with congenital
consensus or established guidelines for the management of adrenal hyperplasia: A cohort study of 203 patients. J Clin Endocrinol
glucocorticoid or mineralocorticoid doses during pregnancy. Metab. 2010;95(11):5110-5121.
9. Volkl TM, Ohl L, Rauh M, Schofl C, Dorr HG. Adrenarche and puberty in
Fetal outcomes are thus far reassuring with lack of fetal
children with classic congenital adrenal hyperplasia due to 21-hydroxylase
masculinization. deficiency. Horm Res Paediatr. 2011;76(6):400-410.
10. Bonfig W, Roehl FW, Riedl S, et al. Blood pressure in a large cohort of
children and adolescents with classic adrenal hyperplasia (CAH) due to
MAIN CONCLUSIONS 21-hydroxylase deficiency. Am J Hypertens. 2015;hpv087. [Epub ahead of
Recent studies suggest that a significant number of children print]
11. Claahsen-van der Grinten HL, Hermus AR, Otten BJ. Testicular adrenal
and adults with classical CAH experience adverse complica- rest tumours in congenital adrenal hyperplasia. Int J Pediatr Endocrinol.
tions related either to the disease itself or the therapy. Therapy 2009:624823.
with glucocorticoids can frequently be challenging and some of 12. Smeets EE, Span PN, van Herwaarden AE, et al. Molecular characteriza-
tion of testicular adrenal rest tumors in congenital adrenal hyperplasia:
these difficulties and variability in outcomes may be related to Lesions with both adrenocortical and Leydig cell features. J Clin
differences in absorption and clearance of administered ste- Endocrinol Metab. 2015100(3):E524-E530.
roids among patients (Table 1). 13. New MI, Abraham M, Gonzalez B, et al. Genotype-phenotype correlation in
1,507 families with congenital adrenal hyperplasia owing to 21-hydroxylase
deficiency. Proc Natl Acad Sci U S A. 2013;110(7):2611-2616.
REFERENCES 14. Tardy-Guidollet V, Menassa R, Costa JM, et al. New management strategy
1. Muthusamy K, Elamin MB, Smushkin G, et al. Clinical review: Adult of pregnancies at risk of congenital adrenal hyperplasia using fetal sex
determination in maternal serum: French cohort of 258 cases (2002-2011).
height in patients with congenital adrenal hyperplasia: A systematic review
J Clin Endocrinol Metab. 2014;99(4):1180-1188.
and metaanalysis. J Clin Endocrinol Metab. 2010;95(9):4161-4172.
15. New MI, Tong YK, Yuen T, et al. Noninvasive prenatal diagnosis of
2. Lin-Su K, Harbison MD, Lekarev O, Vogiatzi MG, New MI. Final adult
congenital adrenal hyperplasia using cell-free fetal DNA in maternal
height in children with congenital adrenal hyperplasia treated with growth
plasma. J Clin Endocrinol Metab. 2014;99(6):E1022-E1030.
hormone. J Clin Endocrinol Metab. 2011;96(6):1710-1717.
16. Lekarev O, Lin-Su K, Vogiatzi MG. Infertility and reproductive function
3. Lin-Su K, Vogiatzi MG, Marshall I, et al. Treatment with growth hormone in patients with congenital adrenal hyperplasia: Pathophysiology, advances
and luteinizing hormone releasing hormone analog improves final adult in management, and recent outcomes. Endocrinol Metab Clin N Am. 2015
height in children with congenital adrenal hyperplasia. J Clin Endocrinol 2015;44(4):705-722.
Metab. 2005;90(6):3318-3325. 17. Hindmarsh PC, Charmandari E. Variation in absorption and half-life of
4. Bonfig W, Schwarz HP. Overestimation of final height prediction in hydrocortisone influence plasma cortisol concentrations. Clin Endocrinol
patients with classical congenital adrenal hyperplasia using the Bayley and (Oxf). 2015;82(4):557-561.
Pinneau method. J Pediatr Endocrinol Metab. 2012;25(7-8):645-649. 18. Wilkins L, Lewis RA, Klein R, Rosenberg E. Suppression of androgen
5. Finkielstain GP, Kim MS, Sinaii N, et al. Clinical characteristics of a secretion in congenital adrenal hyperplasia. AMA Am J Dis Child.
cohort of 244 patients with congenital adrenal hyperplasia. J Clin 1950;80(5):883-884.
Endocrinol Metab. 2012;97(12):4429-4438. 19. Bartter FC, Forbes AP, Leaf A. Congenital adrenal hyperplasia associated
6. Bouvattier C, Esterle L, Renoult-Pierre P, et al. Clinical outcome, hor- with the adrenogenital syndrome: an attempt to correct its disordered
monal status, gonadotrope axis, and testicular function in 219 adult men hormonal pattern. J Clin Invest. 1950;29(6):797.

The Endocrine Society. Downloaded from press.endocrine.org by [${individualUser.displayName}] on 12 January 2017. at 12:19 For personal use only. No other uses without permission. . All rights reserved.
ENDO 2016 ADRENAL/HPA AXIS 21

Adrenal Insufficiency: Individualized Management

M11 adrenal crises (4, 5). Increased awareness and improved quality
Presented, April 1 4, 2016 of diagnosis, treatment, and management are needed to reduce
mortality.
Despite state-of-the-art treatment with corticosteroids, quality
Eystein S. Husebye, MD, PhD. Haukeland University of life and working ability is reduced in a substantial number of
Hospital, University of Bergen, N-5053 Bergen, Norway, patients. Many suffer from other autoimmune conditions (3), add-
E-mail: eystein.husebye@med.uib.no ing to the burden of disease. Overtreatment with glucocorticoids
and mineralocorticoids has potential adverse metabolic conse-
INTRODUCTION quences such as overweight, hypertension, cardiovascular disease,
Historical Overview and osteoporosis, although evidence is still scarce.
Tomas Addison of Guys Hospital in London was the first to Most physicians and even specialists in endocrinology rarely
link the typical symptoms of adrenal insufficiency (AI) to manage more than a few patients. Thus, it is reasonable to believe
disease in the adrenal capsules or glands, first in a paper that procedures for diagnosis, treatment, and followup vary
to the South London Medical Society in 1849, followed by greatly. Endocrine Society Guidelines aimed to improve these
his famous monograph of 1855 in which he captures the shortfalls are about to be published (8).
typical clinical features including hyperpigmentation that is
so typical for primary AI. In Addisons own words, The BARRIERS TO OPTIMAL PRACTICE
leading and characteristic features of the morbid state to Suboptimal treatment of adrenal crisis, including lack of
which I would direct attention are, anemia, general languor adequate education of patients to handle acute crises.
and debility, remarkable feebleness of the hearts action, Unavailability of physiological replacement therapy.
irritability of the stomach, and a peculiar change of color in Lack of parameters to monitor glucocorticoid therapy.
the skin, occurring in connection with a diseased condition
of the supra-renal capsules (1). In his first case of 11
LEARNING OBJECTIVES
patients, tuberculosis was a major cause; others were infil-
As a result of participating in this session, learners should be
trative tumor growth and probably autoimmunity. Only the
able to:
year after, Trousseau named the disease Addisons disease.
Diagnose AI and generate an appropriate differential
Shortly after, Charles Edouard Brown-Sequard adre-
diagnosis.
nectomized rabbits, dogs and cats, claiming that they all
Recognize other autoimmune conditions that may be
died in adrenal failure and that the adrenal glands must
coincident with adrenal failure.
contain a life-preserving material. However, it took 40 years
Develop a long-term treatment plan that will prevent
before a pressor substance was found in the adrenal medulla.
adrenal crises and minimize the untoward effects of
After Swingle and Pfiffner treated adrenalectomized cats
glucocorticoid and mineralocorticoid replacement.
with adrenal extracts and made them survive in 1930 (2),
research was intensified, leading to the isolation of
STRATEGIES FOR DIAGNOSIS, THERAPY, AND/
corticosterone by Kendall (1936), deoxycorticosterone by
OR MANAGEMENT
Reichstein (1938) and the introduction of cortisone in the
treatment of rheumatoid arthritis by Hench in 1949 (3). At
Diagnosis
the same time, cortisone became available for Addison pa-
How to Diagnose AI
tients; later, fludrocortisone was added to the treatment, and
Primary AI can be diagnosed in several ways. The short stan-
the combination of hydrocortisone (HC) or cortisone acetate
dard 250-g Synacthen test is considered the gold standard for
(CA) with fludrocortisone is still the standard treatment for
diagnosis. Given either im or iv, S-cortisol at 30 or 60 minutes
AI. On the diagnostic side, a major breakthrough was the identi-
should reach at least 500 nmol/L (18 g/dL). In most cases of
fication of 21-hydroxylase as the major autoantigen in autoim-
primary AI a paired cortisol and ACTH test will suffice to make
mune Addisons disease (2). It is now a standard part of the
the diagnosis. A random S-cortisol level less than 138 nmol/L (5
workup of Addisons disease to test for these autoantibodies.
g/dL) in the combination of an P-ACTH greater than two times
the upper reference limit is most likely diagnostic, but should be
SIGNIFICANCE OF THE CLINICAL PROBLEM confirmed by a Synacthen test.
AI (Addisons disease) is in many cases diagnosed too late (3). Other typical endocrine test results are elevated plasma
Patients die before the diagnosis is made, and even with known renin activity/renin concentration, low aldosterone, and
adrenal insufficiency, untimely deaths are seen as a result of dehydroepiandrosterone sulfate (DHEA-S). TSH levels can

The Endocrine Society. Downloaded from press.endocrine.org by [${individualUser.displayName}] on 12 January 2017. at 12:19 For personal use only. No other uses without permission. . All rights reserved.
22 ENDO 2016 MEET-THE-PROFESSOR CLINICAL CASE MANAGEMENT

be slightly increased due to lack of cortisol-mediated inhi- In autoantibody-negative individuals genetic causes should
bition of pituitary TSH production. Typical routine labo- be considered in children, particularly adrenoleukodystrophy
ratory findings in untreated AI are hyponatremia, hyperka- (ABCD1 mutations) and adrenal hypoplasia congenital, with
lemia, hypercalcemia, normochromic anemia, sometimes eo- hypogonadotropic hypogonadism (DAX1 mutations). Other causes
sinophilia, and elevated liver transaminases. should be sought guided by medical history, symptoms, and signs
(Causes of adrenal insufficiency, Table 1). The antibody negatives
How to Determine the Cause of AI are more frequent among children and the elderly (age 60 y). A
All patients diagnosed with AI should have its cause determined. computer tomography of the adrenals should be performed. It can
Given that autoimmunity is the overwhelmingly most common reveal atrophy, tumors, calcifications as a sign of tuberculous
cause in Europe and North America it is reasonable to start by adrenalitis, or signs of bleeding. Assay for very long-chain fatty
testing for 21-hydroxylase autoantibodies. Studies from different acids should be performed in all autoantibody-negative males to
European countries have shown a cross-sectional prevalence of identify the X-linked disease adrenoleukodystrophy. Sequencing
80 85% in Addison cohorts (3), and in the newly diagnosed the is available for a number of genetic causes of AI.
percentage is even higher. The typical patient is a female between A commercial assay for autoantibodies against 21-hyd-
20 and 50 years of age, but autoimmune Addisons disease can roxylase is available. In addition, a number of laboratories run
occur at almost any age. Thus, all patients should be tested, in-house assays with comparable performance (sensitivities ap-
possibly with the exception of infants and children less than 3 proximately 90% and specificities close to 100%. Immunofluores-
years of age. Among autoimmune Addisons patients, other cence techniques using adrenal tissue is less sensitive. Autoanti-
organ-specific autoimmune diseases are very common (50%), bodies against interferon alpha and omega are almost always
such as autoimmune thyroid disease, type 1 diabetes, autoimmune positive in APS-1, but are not widely available.
gastritis/pernicious anemia, celiac disease, vitiligo, alopecia, and
ovarian insufficiency (3). These combinations are collectively Therapy
named autoimmune polyendocrine syndrome type 2 (APS-2). Di- How to Administer Corticosteroids in Chronic
agnosis in children and adolescents should raise the question of Replacement Therapy
whether the patient has the monogenic APS type-1 with mutations Patients with Addisons disease require lifelong glucocorticoid
in the autoimmune regulator (AIRE) gene. and mineralocorticoid treatment. Normal cortisol production rates

TABLE 1. Causes of Adrenal Insufficiency


Etiology Pathogenesis Diagnosis/Characteristics
Autoimmunity T and B cell autoimmunity against adrenocortical 21-hydroxylase autoantibodies, most common cause.
cells In children, consider autoimmune polyendocrine
syndrome type 1
Infection Mycobacteria Quantiferon test and PCR
Bacteria (e.g. meningococcus; Haemopholus Adrenal computer tomography
influenzae)
Fungus (e.g. Pneumocystis carinii) Culture
Virus (e.g. HIV, herpes simplex, cytomegalovirus)
Bleeding Anticoagulant therapy Evidence of bleeding on adrenal computer
tomography
Anti-phospholipid syndrome
Disseminated intravascular coagulation
Genetic Congenital adrenal hyperplasia, Steroid profile, sequencing of steroidogenic genes
(e.g. CYP21B)
Adrenoleukodystrophy Measure very long-chain fatty acids (X-linked)
Adrenal hypoplasia, congenital with Sequencing of NR0B1 (DAX1, X-linked)
hypogonadotrophic hypogonadism,
Familiar glucocorticoid deficiency (ACTH
resistance syndrome), Smith-Lemli-Opitz
syndrome, Kearns-Sayre syndrome
Surgery Adrenalectomy (e.g. for Cushings syndrome)
Infiltrative disease Bilateral adrenal metastasis or lymphoma, Adrenal computer tomography, investigations
amyloidosis, hemochromatosis xantho- guided by history and clinical findings
granulomatous
Drugs Ketoconazole, etidomate, mitotane metyrapone

The Endocrine Society. Downloaded from press.endocrine.org by [${individualUser.displayName}] on 12 January 2017. at 12:19 For personal use only. No other uses without permission. . All rights reserved.
ENDO 2016 ADRENAL/HPA AXIS 23

are between 5 and 10 mg/m2 body surface equivalent to an oral coid and mineralocorticoid replacement a 3-month trial of
replacement dose of 1525 mg per day of HC and 20 30 mg of DHEA replacement could be tried.
CA. CA has a slightly delayed peak but a longer half-life com-
pared with HC. CA is converted to HC by 11-hydroxysteroid Treatment in Special Situations
dehydrogenase type 1. Current treatment is largely empirical; only The Addisons Patient with Hypertension
a few underpowered and often unblinded studies have been per- Essential hypertension in Addisons disease is not uncommon.
formed. Most physicians administer HC and CA in two or three First, mineralocorticoid and glucocorticoid replacement should be
(sometimes four) divided doses, with the first dose upon awaken- carefully evaluated and fine tuned; do not stop mineralocorticoid
ing (even before awaking may work best) and the last dose replacement. If hypertension persists, angiotensin II receptor
approximately 4 6 hours before bedtime. Plenadren, a dual re- blockers or angiotensin-converting enzyme inhibitors can be used;
lease once-daily HC is available in 5- and 20-mg tablets (6). In a likewise, calcium channel blockers. Diuretics should be avoided;
randomized open study, the patients in the Plenadren arm had aldosterone receptor blockers are contraindicated (11).
lower blood pressure (BP) and lower glycated hemoglobin
(HbA1c) both in persons with diabetes and nondiabetics. Dexa- The exercising Addisons patient
methasone has no place in replacement therapy for Addisons There is no evidence supporting that extra HC/CA increase
disease because of the risk of Cushingoid adverse effects. Subcu- performance or lessens postexercise fatigue in short-term
taneous infusion of HC by an insulin pump is safe and can strenuous exercise (12). However, it is many patients experi-
be an alternative in patients with insufficient effects of ence that an extra 510 mg of HC is beneficial, at least for
peroral therapy (7). prolonged strenuous exercise and stress.
Monitoring of glucocorticoid therapy relies on clinical as-
sessment with a keen eye to symptoms and signs of over-
The pregnant Addisons patient
replacement (weight gain, insomnia, recurrent infection, pe-
Free cortisol levels increase during the third trimester (13),
ripheral edema) and under-replacement (lethargy, nausea, poor
resulting in an increased requirement for HC (by 2.5 or 10 mg
appetite, weight loss, hyperpigmentation, joint pain). Detailed
daily). Serum progesterone has antimineralocorticoid effects and
questioning about the patients daily pattern and routines can
hence the fludrocortisone dose may sometimes be increased dur-
help fine-tune doses and times of medication. Some patients
ing late pregnancy. During delivery, a bolus dose of 100 mg
respond better to four (8) or even five small doses of HC or
parenteral HC should be given, and repeated if necessary. The oral
CA. Serum or salivary cortisol day curves can be useful to
dose should be doubled for 24 48 hours postpartum.
monitor treatment. Weight-adjusted dosing makes it easier to
keep serum cortisol within reference ranges (9).
Addisons disease in surgery
The steroid doses during surgery and medical procedures needs
Mineralocorticoid Replacement to be increased according to the degree of stress induced (14).
Mineralocorticoids are vital for maintaining water and ele-
ctrolyte homeostasis. The synthetic mineralocorticoid, 9-
Adrenal Crisis
fludrocortisone is used for replacement. Patients should be
Acute adrenal crisis is a life-threatening emergency that re-
advised to eat salt, and salty foods ad libitum. A once-daily
quires immediate diagnosis and treatment that is not uncom-
dose of 0.05 0.2 mg (most use 0.1 mg) taken in the morning
mon. Six to eight episodes per 100 patient years has been
is usually sufficient for most patients. Mineralocorticoid
reported (15). Especially vomiting and/or diarrhea are com-
replacement is evaluated clinically by asking about salt
monly implicated as a precipitating cause. The symptoms
cravings, orthostatism, and edema. Measuring BP in the
are malaise, nausea, fatigue, vomiting, severe abdominal pain,
supine or sitting and standing positions can help unmask muscle pain, or cramps. Dehydration and hypotension with
under-replacement by revealing BP decreases. Serum so- progression to circulatory shock is not uncommon. Hyperkale-
dium, potassium, and plasma renin activity/renin should be mia can be life threatening.
measured, aiming at a renin in the upper-normal or slightly Diagnosis of AI should never delay treatment if an adre-
elevated range. nal crisis is suspected. A blood sample for serum cortisol,
ACTH (if the diagnosis is not known), sodium, potassium,
Is There an Indication for DHEA Treatment? creatinine, urea, glucose, and other tests for precipitating
Female patients with AI are androgen deficient, and androgen causes (bacterial or viral infections) should be drawn and
levels can be restored by giving oral DHEA. Daily doses of therapy initiated immediately by giving an iv bolus of 100
10 25 mg are usually sufficient to bring T and androstenedione mg HC and saline infusion. Glucose infusion is needed if
back into the reference range. There is limited objective evi- serum glucose is below the reference range. Any treatable
dence of clinical benefits (10). In female patients with lack of triggering condition should be addressed as well. Parenteral
libido and/or low energy levels despite optimized glucocorti- glucocorticoids should be continued at 200 mg/day (either

The Endocrine Society. Downloaded from press.endocrine.org by [${individualUser.displayName}] on 12 January 2017. at 12:19 For personal use only. No other uses without permission. . All rights reserved.
24 ENDO 2016 MEET-THE-PROFESSOR CLINICAL CASE MANAGEMENT

continuous infusion or intermittent treatment) and tapered serum sodium and potassium, and creatinine. Plasma renin
over 13 days if the underlying condition permits. Overdos- activity can be useful to evaluate fludrocortisone replacement
ing may lead to hypokalemia. (Table 2). Other autoimmune comorbidities should also be
The patient should be equipped with a steroid card and a looked for at longer intervals. Women with autoimmune Addi-
glucocorticoid injection kit and educated on the use of stress sons disease have an increased risk of premature ovarian
doses of glucocorticoids. A bilingual European steroid card failure and it is reasonable to inform them that it is not ideal to
was recently endorsed by European Society for Endocrinology postpone planned pregnancies.
and should be suitable for countries outside Europe (16).

MAIN CONCLUSIONS
Management
The gold standard diagnostic test is the conventional
The annual follow-up (Table 2) should focus on complaints
possibly related to Addisons disease and its treatment. Fur- 250-g Synacthen test, although a paired cortisol and
thermore, it is recommended to focus on subjective health ACTH sample often is sufficient.
perception, weight and appetite, professional activities, and The main cause is autoimmunity revealed by
social life. Questions about episodes of adrenal crisis, medica- autoantibodies against 21-hydroxylase, often as part of
tion, particular extra doses, compliance, timing, and how to and autoimmune polyendocrine syndrome; non-
deal with stress and acute disease are important. All patients autoimmune causes are more frequent in the very young
should be equipped with a steroid card and an injection kit. and old.
Physical examination should include weight, BP, and degree Replacement therapy includes HC or CA in two or three
of pigmentation, keeping in mind the possibilities of autoim- (or four) divided doses, and fludrocortisone without
mune comorbidities. Annual screening for autoimmune thyroid restriction of salt intake. Doses should be individualized
disease, diabetes mellitus, and vitamin B12 deficiency is rec- in terms of numbers and amount. An alternative to
ommended. Routine laboratory analyses include hemogram, regular HC or CA is a duel-release once-daily

Table 2. Suggested Follow-Up Routines for Patients With Adrenal Insufficiency


Action Point Intervention
History History focused on well-being, capacities in work and social life; sexuality, fertility,
adrenal crises; how much and when medication is taken; symptoms and signs of over
and under replacement
Physical examination Weight (height)
Blood pressure sitting/supine and standing
Look for pigmentation changes, alopecia, vitiligo, goiter, and Cushingoid side effects
Recommended annual tests Hemogram
Na, K, creatinine
ferritin and cobalamine
TSH, FT4, anti-TPO
HbA1c
Renin/renin activity
Other tests for consideration Serum or saliva cortisol day curve to check bioavailability
Suspicion of vitamin B 12 deficiency: cobalamine, methylmalonic acid, parietal cell and
intrinsic factor antibodies
Suspicion of celiac disease: transglutaminase antibodies and total IgA (once)
Osteoporosis: bone scan at start of follow-up, around menopause depending on clinical
situation
Other tests dictated by history and findings
Patient (parent/partner) education Steroid emergency card: available and up to date
Self-injection kit: available with vials of hydrocortisone, syringes (alternative is
suppositories)
Sick day rule 1: Need to double the routine oral glucocorticoid dose when the patient
experiences fever, illness requiring bed rest or when requiring antibiotics for an
infection
Sick day rule 2: Need to inject a glucocorticoid preparation intra-muscularly or
intravenously in case of severe illness, trauma, persistent vomiting, when fasting for a
procedure (colonoscopy!) or during surgical intervention

The Endocrine Society. Downloaded from press.endocrine.org by [${individualUser.displayName}] on 12 January 2017. at 12:19 For personal use only. No other uses without permission. . All rights reserved.
ENDO 2016 ADRENAL/HPA AXIS 25

formulation of HC; pump treatment is an option for pigmentation. Blood tests reveal a low-normal morning cortisol
those not managing well on tablets. of 167 nmol/L and ACTH of 267 pmol/L.
Annual follow-up visits should focus on steroid 5. Which of the following studies would best guide your
replacement, measures in acute situations, complications, decision on what to do next?
and autoimmune comorbidities. A. Synacthen testing
Patients should be equipped with a steroid card and B. Plasma renin
injectable HC. C. 21-hydroxylase autoantibodies
D. None of the above
CASES 6. How would you handle the case when you received the
Case 1 test result you ordered?
A 62-year-old women was diagnosed with autoimmune Ad- A. Observe, but inform of risk of overt Addisons
disons disease at 27 years of age. She was put on standard disease
replacement with CA and fludrocortisone but experienced B. Observe, but prescribe HC and tell her to start if her
a general lack of energy, fatigue, and joint pain. She had condition worsens
tried various doses and combinations of glucocorticoids C. Start replacement with glucocorticoid
(CA and prednisolone) and when first evaluated by us took D. Start replacement with glucocorticoid and
50 mg CA, 10 mg prednisolone, and 0.1 mg fludrocortisone. mineralocorticoid
Clinically, she was without Cushingoid symptoms and
signs.
DISCUSSION OF CASES AND ANSWERS
1. Which of the following studies would best identify
Case 1
whether this patient is properly treated? The 62-year-old woman was followed in our out-patient clinic.
A. Hold CA for 48 hours then check 8 am cortisol and The CA dose was reduced from 50 to 37.5 mg daily divided
ACTH in three doses and predinisolone was discontinued. Clinically
B. Synacthen testing the situation was unchanged. She continued with 0.1 mg
C. Cortisol day curve fludrocortisone. She was offered to participate in a clinical
D. Check for other autoimmune manifestations study testing sc HC pump treatment. She was titrated to a daily
2. You elect to alter her adrenal replacement regimen. dose of 28 mg HC. Immediately she felt that her general
Which of the following approaches would you select at condition improved and she felt energized without the need to
this time? take extra doses of HC. After 3 months she was reverted to her
A. Switch from CA to HC and stop prednisolone previous treatment and her feeling of fatigue returned. After the
B. Slow-release HC trial she was offered pump treatment once again and she has
C. HC sc pump treatment now been treated with continuously sc infusion for 4 years.
D. Switch to dexamethasone
Answers
Case 2 1. Answers: D (C). She was checked for autoimmune
A 43-year-old women was diagnosed with autoimmune hypo- comorbidities such as hypothyroidism, celiac disease,
thyroidism at age 21 years, followed by Addisons disease at and vitamin B12 deficiency, but tests were normal. A
25, vitiligo at 37, and celiac disease at 43 years of age. She also cortisol day curve could be useful to check levels and
had vitamin B12 deficiency. Her daughter had vitiligo. compliance. A Syncthen test is not motivated and to
hold CA for 48 could be dangerous.
3. What kind of autoimmune polyendocrine syndrome
does the patient have? 2. Answers: AC could all be correct. In this case we
A. APS-1 tried sc pump treatment with success. One reasonable
B. APS-2 approach could be to first try HC, then slow-release
C. APS-3 HC, and as a third option, pump treatment.
D. The kind of polyendocrine syndrome is irrelevant Dexamethasone is contraindicated due to the high risk
of Cushingoid adverse effects.
4. Should family members be screened for autoimmunity?
A. Yes Case 2
B. No The patient developed several organ-specific autoimmune dis-
eases, of which some were present in family members. Her daugh-
Case 3 ter with vitiligo developed depigmentation in her teens. Her
A 23-year old woman is referred because of fatigue and ac- mother had vitamin B12 deficiency and an aunt had died at 18
cording to her mother who has Addisons disease, increased years of age with an undiagnosed condition that might have been

The Endocrine Society. Downloaded from press.endocrine.org by [${individualUser.displayName}] on 12 January 2017. at 12:19 For personal use only. No other uses without permission. . All rights reserved.
26 ENDO 2016 MEET-THE-PROFESSOR CLINICAL CASE MANAGEMENT

Addisons disease. The patient was clearly 21-hydroxylase REFERENCES


autoantibodypositive. Because of familial clustering of organ- 1. Addison T. On the constitutional and local effects of disease of the
specific autoimmunity the AIRE gene was sequenced revealing a suprarenal capsules. In a collection of the published writings of the late
Thomas Addison MD, Physician to Guys Hospital. New Sydenham Soci-
heterozygous mutation in codon 326 (p.P326L) located in the ety (1868) London, Reprinted in Medical Classics 1939.2(244-293).
PHD1 domain. With this information in hand we analyzed a 2. Winqvist O, Karlsson FA, and Kampe O. 21-Hydroxylase, a major
broader spectrum of autoantibodies including interferon omega autoantigen in idiopathic Addisons disease. Lancet. 1992;339(8809):
autoantibodies, but they all turned out negative. Her daughter had 1559-1562.
3. Erichsen MM, Lvs K, Skinningsrud B, et al. Clinical, immunological,
the same mutation and during followup her 7-year-old son also and genetic features of autoimmune primary adrenal insufficiency: Obs-
developed vitiligo. He also had p.P326L. ervations from a Norwegian registry. J Clin Endocrinol Metab. 2009;
94(12):4882-4890.
4. Hahner S, Spinnler C, Fassnacht M, et al. High incidence of adrenal crisis
Answers in educated patients with chronic adrenal insufficiency: A prospective
3. Answer: A. This and similar causes suggest that mono- study. J Clin Endocrinol Metab. 2015;100(2):407-416.
allelic mutations in AIRE might lead to APS-1-like 5. Erichsen MM, Lvs K, Fougner KJ, et al. Normal overall mortality rate in
clinical picture with dominant inheritance, which can be Addisons disease, but young patients are at risk of premature death. Eur J
named Nonclassical APS-1. In vitro transfection of Endocrinol. 2009;160(2):233-237.
6. Johannsson G, Nilsson AG, Bergthorsdottir R, et al. Improved cortisol
mutated and wild-type AIRE reveal that p.P326L have a
exposure-time profile and outcome in patients with adrenal insufficiency:
dominant negative effect on wild-type AIRE-regulated A prospective randomized trial of a novel hydrocortisone dual-release
transcription. formulation. J Clin Endocrinol Metab. 2012;97(2):473-481.
7. ksnes M, Bjornsdottir S, Isaksson M, et al. Continuous subcutaneous
4. Answer: A. Organ-specific autoimmunity often runs in
hydrocortisone infusion versus oral hydrocortisone replacement for treat-
families. There is an approximately 10% risk for a ment of Addisons disease: A randomized clinical trial. J Clin Endocrinol
patient with autoimmune Addisons disease to have Metab. 2014;99(5):1665-1674.
first-degree relative with the same disease. There is also 8. Ekman B, Bachrach-Lindstrom M, Lindstrom T, Wahlberg J, Blomgren J,
increased risk of other organ-specific autoimmune Arnqvist HJ. A randomized, double-blind, crossover study comparing two-
diseases. Thus, the patient should be informed about and four-dose hydrocortisone regimen with regard to quality of life, corti-
this increased risk, and if there is clinical suspicion, sol and ACTH profiles in patients with primary adrenal insufficiency. Clin
Endocrinol (Oxf). 77(1):18-25.
testing can be performed. In this particular case with
9. Mah PM, Jenkins RC, Rostami-Hodjegan A, et al. Weight-related dosing,
dominant inheritance I would opt for familial screening. timing and monitoring hydrocortisone replacement therapy in patients with
adrenal insufficiency. Clin Endocrinol (Oxf). 2004;61(3):367-375.
Case 3 10. Alkatib AA, Cosma M, Elamin MB, et al. A systematic review and
The initial clinical and biochemical evaluation suggested mild meta-analysis of randomized placebo-controlled trials of DHEA treatment
effects on quality of life in women with adrenal insufficiency. J Clin
AI with fatigue and increased pigmentation (due to elevated Endocrinol Metab. 2009;94(10):3676-3681.
ACTH). 11. Inder WJ, Meyer C, Hunt PJ. Management of hypertension and heart
failure in patients with Addisons disease. Clin Endocrinol (Oxf).
2015;82(6):789-792.
Answers 12. Simunkova K, Jovanovic N, Rostrup E, et al. Effect of a pre-exercise
5. Answers: AC. Cortisol is barely in the normal range. hydrocortisone dose on short term physical performance in female patients
Together with a high ACTH level, this strongly with primary adrenal failure. Eur J Endocrinol. 2016;174(1):97-105.
suggests AI. Ideally, a Synacthen test should be 13. Lebbe M, Arlt W. What is the best diagnostic and therapeutic management
strategy for an Addison patient during pregnancy? Clin Endocrinol (Oxf).
performed and the aldosternone reserve determined by
2013;78(4):497-502.
measuring plasma renin. The cause of AI should always 14. Husebye ES, Allolio B, Arlt W, et al. Consensus statement on the diagno-
be determined by measuring 21-hydroxylase sis, treatment and follow-up of patients with primary adrenal insufficiency.
autoantibodies. The risk of other autoimmune J Intern Med. 2014;275(2):104-115.
comorbidities is high. As a minimum, thyroid function 15. Reisch N, Willige M, Kohn D, et al. Frequency and causes of adrenal
and glucose levels should be determined. crises over lifetime in patients with 21-hydroxylase deficiency. Eur J
Endocrinol. 2012;167(1):35-42.
6. Answer: D. She was about to go on her honeymoon 16. Dahlqvist P, Bensing S, Ekwall O, Wahlberg J, Bergthorsdottir R, Hulting
and it was decided to start full replacement therapy AL. 2011. [A national medical emergency card for adrenal insufficiency. A
with CA and fludrocortisone. The patient was instructed new warning card for better management and patient safety]. [Article in
about acute procedures in relation to adrenal crisis and Swedish] Lakartidningen. 2011;108(44):2226-2227.
17. Pearce SH, Mitchell AL, Bennett S, et al. Adrenal steroidogenesis after B
equipped with a steroid card and a HC injection kit.
lymphocyte depletion therapy in new-onset Addisons disease. J Clin
The Synacthen test was postponed due to her travel Endocrinol Metab. 2012;97(10):E1927-E1932.
plans. Two months later it revealed unmeasurable levels 18. Gan EH, MacArthur K, Mitchell AL, et al. Residual adrenal function
of cortisol at baseline and after 250 g Synacthen (20 in autoimmune Addisons disease: Improvement after tetracosactide
nmol/L in all three samples at 0, 30, and 60 min). (ACTH124) treatment. J Clin Endocrinol Metab. 2014;99(1):111-118.

The Endocrine Society. Downloaded from press.endocrine.org by [${individualUser.displayName}] on 12 January 2017. at 12:19 For personal use only. No other uses without permission. . All rights reserved.
ENDO 2016 ADRENAL/HPA AXIS 27

Pheochromocytomas and Paragangliomas

M12 vent morbidity and mortality associated with surgical removal.


Presented, April 1 4, 2016 In addition, all patients with PCC/PGL should be referred for
clinical genetic testing because knowledge of an germline mu-
tation in PCC/PGL susceptibility gene is important for optimal
Lauren Fishbein, MD, PhD. Department of Medicine, surveillance of the patient and his/her family members as most
Division of Endocrinology, Metabolism and Diabetes, gene mutations are associated with risk of multiple primary
University of Colorado School of Medicine, Aurora, PCC/PGL as well as presence of other tumor types. Cost is
Colorado 80045, E-mail: lauren.fishbein@ucdenver.edu thought to be prohibitive to genetic testing in some cases.

HISTORICAL OVERVIEW LEARNING OBJECTIVES


The first description of the two distinct layers of the adrenal As a result of participation in this session, learners should be
gland (the cortex and medulla), dates back to the mid 1800s. able to:
Shortly thereafter, the first adrenal medulla tumors were de- Understand how to diagnosis PCC/PGL.
scribed and termed pheochromocytomas (PCCs). Parallel to Understand how to manage perioperative blockade.
this discovery, tumors in ganglia throughout the body were Understand the importance of clinical genetic testing for
termed paragangliomas (PGLs). Until just a few decades ago, patients with PCC/PGL.
the mortality rate for secreting PCCs and PGLs (PCC/PGL)
was quite high, around 30 45% (1). With medical and surgical
STRATEGIES FOR DIAGNOSIS, THERAPY,
advances, particularly with regard to perioperative blockade
AND/OR MANAGEMENT
regimens, the morbidity has decreased to 0-2.9% (1). The old
Patients with PCC/PGL often present with the classic triad of
rule of tens associated with PCCs is no longer accurate.
headaches, palpitations, and diaphoresis, but many other symp-
Approximately 25% of PCC/PGL are malignant, approxi-
toms and signs can be clues to the diagnosis (Table 1). Screen-
mately 25% are extra-adrenal, and up to 40% are hereditary. In
ing for PCC/PGL also should occur as part of a workup for
the pediatric population, up to 80% of PCC/PGLs may be
patients with secondary hypertension, with an adrenal
hereditary (2).
incidentaloma (with or without hypertension) (4) or with a
known susceptibility gene mutation (5) (Table 2).
SIGNIFICANCE OF THE CLINICAL PROBLEM Both 24-hour urine fractionated and plasma-free met-
PCC/PGLs are neuroendocrine tumors of chromaffin tissue anephrines have over 90% sensitivity for PCC/PGL, but the
which, even when benign, are highly morbid tumors and diag- plasma tests have slightly higher specificity (79 98% vs
nosis can be difficult in some cases. A quarter of tumors are 69 95%) (6). It is recommended that the plasma-free
malignant defined by the World Health Organization as having metanephrines be the first-line screening test. Numerous medi-
distant metastases in nonchromaffin tissue and can occur even cations can interfere with screening and lead to false-positive
up to 20 years after initial diagnosis. There are no curative results including acetaminophen, several classes of antidepres-
therapies for widely metastatic disease, and unfortunately, pa- sants, attention deficit hyperactivity disorder medications
tients with metastatic disease have only a 50% 5-year survival (stimulants) and certain - and - adrenergic blockers (6).
rate (3). In addition, there are no strong predictors of malig- These medications should be held prior to testing if possible. If
nancy; therefore, all patients need life-long screening. Despite the medications cannot be stopped, such as some of the psy-
the high rate of hereditary mutations in patients with chotropic medications, and the plasma metanephrine screen is
PCC/PGL, many patients are not referred for clinical genetic
testing, which can negatively affect the screening and surveil-
TABLE 1. Common Symptoms and Signs Associated
lance of the patient and his/her family members.
With PCC/PGL
Symptoms Signs
BARRIERS TO OPTIMAL PRACTICE
Headache Tachycardia
PCC/PGLs are rare tumors, and unless practicing at a referral
Diaphoresis Hypertension
center, many clinicians may see only 1-2 patients with this
Palpitations Hyperglycemia
disease in their career. Often, symptoms of catecholamine
Syncope or presyncope
hypersecretion can go unrecognized as they can mimic many
Anxiety
other conditions. Therefore, including PCC/PGL in the differ-
Weight changes
ential diagnosis is critical. Once diagnosed, clinicians must be
No symptoms
familiar with proper perioperative blockade regimens to pre-

The Endocrine Society. Downloaded from press.endocrine.org by [${individualUser.displayName}] on 12 January 2017. at 12:19 For personal use only. No other uses without permission. . All rights reserved.
28 ENDO 2016 MEET-THE-PROFESSOR CLINICAL CASE MANAGEMENT

TABLE 2. PCC and PGL Susceptibility Genes


Gene Syndrome Protein (Function) Tumor Location Malignancy Rate
NF1 Neurofibromatosis Type 1 Neurofibromin (GTPase which Adrenal (bilateral) 12%
inactivates RAS)
RET Multiple Endocrine Neoplasia RET (transmembrane tyrosine Adrenal (bilateral) 5%
Type 2 kinase)
VHL von Hippel Lindau pVHL (ubiquitin ligase activity) Adrenal (bilateral) 5%
SDHx genes Familial paraganglioma Succinate dehydrogenase complex Any location Low
SDHA syndrome (complex II of the mitochondrial Any location, primarily 23%
SDHB respiratory chain and converts extra adrenal Low
SDHC succinate to fumarate) Head and neck, can be 5%
SDHD SDH subunit A (catalytic subunit) thoracic Low
SDHAF2 (SHD5) SDH subunit B (catalytic subunit) Any location, primarily
SDH subunit C (anchoring subunit) head and neck
SDH subunit D (anchoring subunit) Head and neck (multifocal)
SDH cofactor AF2 (cofactor)
TMEM127 Transmembrane protein 127 Any location, primarily Low
(transmembrane protein) adrenal
MAX MYC-associated protein X Adrenal (bilateral) Intermediate
(transcription factor)
EPAS1 Polycythemia paraganglioma Hypoxia inducible factor 2a Any location Not known
syndrome (transcription factor)
FH Hereditary leiomyomatosis and Fumarate hydratase Any location Possibly high
renal cell carcinoma (converts fumarate to malate)
MDH2 Malate dehydrogenase Any location Not known
(converts malate to oxaloacetate)

positive, it is appropriate to move forward with imaging stud- most appropriate. The Endocrine Society guidelines recommend
ies. Although most PCC/PGLs are secretory, some are not, phenoxybenzamine, a nonselective noncompetitive blocker, as
especially those derived from parasympathetic ganglia such as the first line of treatment and doxazocin or another competitive
in the head and neck. Some nonsecretory PGLs are actually selective -1 blocker, with or without a calcium channel blocker
dopamine-only secreting, which is not picked up with standard as second-line treatment (6) (Table 3). The largest retrospective
biochemical screening. study comparing phenoxybenzamine to competitive selective -1
Cross-sectional imaging with computed tomography (CT) or blockers showed that pheonxybenzamine achieved better preop-
magnetic resonance imaging (MRI) should follow a positive erative and intra-operative blood pressure (BP) control but was
screen for PCC/PGL. The vast majority of tumors are in the associated with more transient postoperative hypotension (8). Full
adrenal gland so abdominal imaging is appropriate to start. Keep -blockade often induces tachycardia and orthostatic hypotension,
in mind that approximately 25% of tumors are located outside the which should be treated with -blockade and hydration with high
adrenal gland, so if there is strong clinical suspicion and no salt intake. These symptoms should not necessarily lead to a dose
adrenal mass, imaging of other locations should be performed, reduction as they imply full -blockade. Remember, -blockers
especially for known susceptibility gene mutation carriers (Table can induce a theoretical unopposed -adrenergic stimulation lead-
2). Functional imaging with 123I-metaiodobenzylguanidine ing to a hypertensive crisis and should not be used until the patient
(MIBG) is not recommended as first line because up to 50% of is fully -blocked. Postoperatively, patients should be screened
normal adrenal glands have increased physiologic uptake, which with plasma metanephrines 4-8 weeks after surgery to ensure
can lead to false-positive results (7), and many PGLs are not complete resection and then annually for life given the potential
MIBG avid. 123I-MIBG imaging can be useful to determine avid- for additional primary tumors, recurrence, and the long latency of
ity in metastatic disease in preparation for possible 131I-MIBG metastatic disease.
treatment. 18F-FDG-PET/CT scanning is recommended for diag- Metastatic PCC/PGL occurs in approximately 10% of PCCs
nosis of metastatic disease, especially for those patients with an and 20% of PGLs (3) and is defined by the presence of distant
inherited Succinate Dehydrogenase Subunit B (SDHB) mutation metastases in nonchromaffin tissue. Metastatic disease can oc-
given that the sensitivity of positron emission tomography imag- cur even up to 20 years after the initial diagnosis and once
ing is 74 100% in this population (6). present, patients have a 50% 5-year survival (3). Predicting
Perioperative blockade is critical to reduce morbidity and mor- metastatic disease is difficult. The risk is increased with the
tality with surgery in patients with PCC/PGL. There are no pro- presence of a germline SDHB mutation, but only half of pa-
spective randomized controlled trials to suggest which regimen is tients with metastatic disease have a mutation in this gene (9).

The Endocrine Society. Downloaded from press.endocrine.org by [${individualUser.displayName}] on 12 January 2017. at 12:19 For personal use only. No other uses without permission. . All rights reserved.
ENDO 2016 ADRENAL/HPA AXIS 29

Table 3. Perioperative Blockade Regimens


Category Drug Dosing Common Adverse Effects
First-line nonselective blocker Phenoxybenzamine 10 mg, 2-3/d Orthostatic hypotension, tachycardia, nasal congestion
(up to 60 mg/d)
Second-line selective -1 blocker Doxazosin 2-4 mg, 2-3 times/d Orthostatic hypotension, tachycardia
Prazosin 1-2 mg, 2 times/d
Terazosin 1-4 mg, once daily
Second-line calcium channel blocker Nicardipine 30 mg 2 times/d Edema, headache
Amlodipine 5-10 mg once daily
-blocker only after full -blockade Metoprolol 25-100 mg, 2 times/d Fatigue, dizziness, asthma exacerbation

Other predictors are tumor size (4 5 cm) or secretion of most common reason for a false-positive screening test is the pres-
methyoxytyramine (not clinically available in most centers) (3, ence of an interfering medication or drug. Blood pressure manage-
10). Histopathologic scoring systems have not been reliable ment with phenoxybenzamine or other blockade is critical
given wide inter- and intra-observer variability (11, 12). Given perioperatively and surrounding treatments for those with metastatic
the difficulty in predicting who will develop metastatic disease, disease. Up to 40% of patients with PCC/PGL have a germline
experts recommend lifelong annual screening for any patient mutation in a known susceptibility gene; therefore, all patients with
who had a PCC/PGL. Many studies are focused on identifying PCC/PGL, regardless of family history, should be referred for clinical
biomarkers for metastatic disease, and new histopathologic genetic testing given the implications for patient and his/her family
scoring systems are being tested (13). Treatments for meta- members.
static disease are not curative but can offer disease control.
These include surgical debulking, chemotherapy with cyclo-
CASES WITH QUESTIONS
phosphamide, vincristine, dacarbazine (CVD), external beam
Case 1
radiation therapy, or 131I-MIBG treatment) [reviewed in A 54-year-old woman with a past medical history of hepatitis
Fishbein (14)]. Tyrosine kinase inhibitors and other small mol- C, sarcoidosis, hypertension on amlodipine, and hypothyroid-
ecules are being tested in clinical trials. ism status post thyroidectomy for multinodular goiter on
There are more than 13 known susceptibility genes associ- levothyroxine presents for evaluation of palpitations and dia-
ated with an increased risk of PCC/PGL (Table 2) (15). The phoresis. Last menstrual period was 8 years ago. Palpitations
three classic tumor-suppressor genes which, when mutated, occur approximately once a day, last for a few minutes, and are
increase risk of PCC/PGL are NF1, VHL, and RET leading to associated with diaphoresis but no chest pain or shortness of
Neurofibromatosis Type 1, von Hippel Lindau disease, and breath. No headaches. She had a recent normal cardiac stress
Multiple Endocrine Neoplasia Type 2, respectively. More than test. She takes acetaminophen daily and smokes marijuana a
15 years ago, the first Succinate Dehydrogenase (SDH) subunit few times a week to help her chronic pain in her back and
gene was found to be associated with familial PGL syndrome. knees. No known relevant family history. On examination, BP,
We now know that mutations in any of the SDH subunits, A, 142/97 mm Hg; heart rate, 77 bpm; body mass index, 33.4
B, C, and D, and the cofactor AF2 are associated with in- kg/m2. Aside from obesity, the remainder of the physical ex-
creased risk of PCC/PGL. SDHx mutations also increase risk of amination is not significant. Laboratory tests show normal TSH
gastrointestinal stromal tumors and renal cell carcinomas; there of 1.68 mIU/mL (normal range, 0.4-4.5 mIU/mL) and elevated
are reports of associations with some SDH subunit mutations plasma normetanephrines (plasma normetanephrines, 1.15
and pituitary adenomas. In the last few years several other nmol/L [normal range, 0-0.89 nmol/L], plasma metanephrines,
genes have been associated with PCC/PGL although they occur 0.12 nmol/L [normal range, 0-0.49 nmol/L]).
in 2% or less of cases including TMEM127, MAX, EPAS1, FH, Does this woman have a PCC/PGL?
and MDH2 (Table 2). Most of the susceptibility gene mutations
are inherited in an autosomal-dominant pattern meaning off-
Case 2
spring have a 50% chance of inheriting the mutation. SDHD A 43-year-old woman with a past medical history of schizo-
and SDHAF2 mutations also have paternal inheritance; there- phrenia on clozapine and mirtazapine, hypothyroidism well
fore, family history may be misleading. Of the more commonly controlled on levothyroxine and diabetes mellitus type 2 well
mutated genes, SDHB is the only one that carries a significantly controlled on metformin presents for evaluation of an adrenal
increased risk of malignancy. incidentaloma seen on a chest CT. She has rare palpitations.
She denies headaches or diaphoresis. She has no hypertension.
MAIN CONCLUSIONS On examination, her BP is 119/76 mm Hg with heart rate, 70
PCC/PGL can have high morbidity and mortality even when benign bpm. The rest of the physical examination is unremarkable
due to the hypersecretion of catecholamines and metanephrines. The except for a flat affect. On noncontrast CT imaging, the adrenal

The Endocrine Society. Downloaded from press.endocrine.org by [${individualUser.displayName}] on 12 January 2017. at 12:19 For personal use only. No other uses without permission. . All rights reserved.
30 ENDO 2016 MEET-THE-PROFESSOR CLINICAL CASE MANAGEMENT

nodule is 3.5 cm with Hounsfield units (HU) of 8, stable in size DISCUSSION OF CASES AND ANSWERS
over 1 year. Biochemical workup shows normal 24 hour uri- Case 1
nary free cortisol on two occasions and an appropriately sup- Does this woman have a PCC/PGL?
pressed cortisol after a 1 mg overnight dexamethasone suppres- This patient has some of the classic symptoms and signs of
sion test. She has elevated plasma metanephrines (62 pg/mL; PCC/PGL including palpitations, diaphoresis, and hyperten-
normal range 57 pg/mL) and plasma normetanephrine (339 sion, making it appropriate to test for the diagnosis. The plasma
pg/mL; normal range 148 pg/mL). Subsequent 24-hour urine normetanephrine levels returned elevated and could suggest
biochemistries show a mildly elevated urine normetanephrine possible disease. However, she is taking several drugs that
(898 mcg/d; normal range, 82-500 mcg/d) with the rest of the could cause a false-positive elevation of these levels. First,
urine biochemistries within the normal range including urine acetaminophen is a common over-the-counter medication that
metanephrine 174 mcg/d (normal range, 45-290 mcg/d); urine interferes with the assay. Patients should be instructed to re-
epinephrine, 13 mcg/d (normal range, 0-20, mcg/d); urine nor- frain from taking any acetaminophen containing products for
epinephrine, 110 mcg/d (normal range, 0-135 mcg/d); and 5-7 days prior to testing. Secondly, marijuana can cause false-
urine dopamine, 218 mcg/d (normal range, 10-510 mcg/d). positive elevations as well. This patient agreed to retest when
Does this woman have a PCC/PGL? refraining from both drugs for at least a week. Her subsequent
levels of plasma metanephrines returned normal.
Case 3
A 42-year-old man with no known past medical history presented Case 2
Does this woman have a PCC?
with feeling poorly. Through PCP workup, he had a CT abdo-
It can be difficult to determine a diagnosis PCC in patients
men that showed a 5.5-cm retroperitoneal mass thought to be a
with known adrenal nodules and mildly elevated meta-
possible sarcoma. A biopsy of this lesion was aborted due to a
nephrine/normetanephrine levels. First and foremost, drug in-
hypertensive crisis during the procedure. Subsequently, 24-hour
teractions should be noted. In this case, the patient is taking
urinary free metanephrine and catecholamines were checked and
psychiatric medications for schizophrenia which, after discus-
were elevated confirming a diagnosis of a PGL. In retrospect, he
sion with her psychiatrist should not be discontinued for fear of
noted recent intermittent hypertension not on medication and dia-
the patients safety. Therefore, we must rely on other informa-
phoresis.
tion to make an informed decision. HU on CT scan can be
What perioperative blockade regimen is best for patients
helpful with low HU under 10 suggesting a benign cortical
with PCC or PGL?
adenoma. Low contrast uptake and rapid washout (50%), can
What are the next best steps in following this patient after
be suggestive of benign cortical adenomas. MRI imaging can
surgery?
show lipid-rich signaling suggestive of benign cortical adeno-
mas as well. (PCCs typically have high HU 10, delayed
Case 4 washout 50%, and lipid-poor signal on MRI.) In an experi-
A 16-year-old male with no known past medical history pres- enced center, 123I-MIBG may be helpful, although not all
ents with palpitations, headaches, and syncope while working PCC/PGL are MIBG avid.
out. Cardiac stress test was performed and the patient devel- In this case, the patient was normotensive and had no symp-
oped supraventricular tachycardia during the test. There was a toms, and the CT and 123I-MIBG imaging was suggestive a
plan for an ablation procedure. Family history was significant benign nonfunctional cortical adenoma. The elevated biochem-
for his father recently being diagnosed with a head and neck istry results were attributed to the psychiatric medications. She
PGL and found to have an SDHD mutation. During a conver- continued to be followed over several years with no change in
sation the father was having with his genetic counselor about symptoms or signs, no growth of the adrenal nodule and no
having his children tested for the inherited mutation, he men- change in the laboratory values.
tioned his sons medical issues above. The astute genetic coun-
selor noted that before any procedures were performed on his Case 3
son, that he should be tested for PCC/PGL with plasma What perioperative blockade regimen is best for patients with
metanephrines. The patient was tested and had elevated plasma PCC or PGL?
normetanephrines (13.96 nmol/L; normal range, 0-0.89 Appropriate perioperative blockade is critical before surgery
nmol/L) and normal plasma metanephrines (0.36 nmol/L; nor- or any procedure in patients with PCC/PGL or with metastatic
mal range, 0-0.49 nmol/L). Imaging of the abdomen showed a PCC/PGL. The Endocrine Society guidelines (6) suggest first
peri-aortic mass of 3.9 cm. line-therapy with phenoxybenzamine along with appropriate
Before this patient had symptoms, what was the likelihood hydration and salt loading. -blockade should be used to treat
he had an SDHD mutation and would develop a PCC/PGL? If any tachycardia after full -blockade is achieved.
the patients mother had the SDHD mutation, would his risk be What are the next best steps in following this patient after
the same? surgery?

The Endocrine Society. Downloaded from press.endocrine.org by [${individualUser.displayName}] on 12 January 2017. at 12:19 For personal use only. No other uses without permission. . All rights reserved.
ENDO 2016 ADRENAL/HPA AXIS 31

All patients with secreting PCC and PGL should have bio- paternally inherited (with extremely rare exception). When the
chemical testing (plasma or urine metanephrines) checked SDHD mutation is passed along from the mother, the patient
within 4-8 weeks postoperatively to ensure complete resection remains unaffected. Of note, if he inherited the mutation from his
and then annually for life to screen for recurrence, metastatic mother, although he would be unaffected, any of his children who
disease, or additional primary tumors. Given that up to 40% of inherited the mutation would be at risk of developing PCC/PGL.
patients with PCC/PGL have a mutation in a known germline
susceptibility gene, all patients should be referred for clinical
REFERENCES
genetic testing. Knowing the presence of a germline mutation 1. Bruynzeel H, Feelders RA, Groenland TH, et al. Risk factors for hemody-
affects the screening and surveillance for the patient and any namic instability during surgery for pheochromocytoma. J Clin Endocrinol
family member with the inherited mutation. Mutation carriers Metab. 2010;95(2):678-685.
are at higher risk for additional primary PCC/PGL and other 2. Bausch B, Wellner U, Bausch D, et al. Long-term prognosis of patients with
pediatric pheochromocytoma. Endocr Relat Cancer. 2014;21(1):17-25.
associated tumors depending on the syndrome (5). In addition,
3. Ayala-Ramirez M, Feng L, Johnson MM, et al. Clinical risk factors for
patients with an SDHB mutation are at increased risk of devel- malignancy and overall survival in patients with pheochromocytomas
oping malignancy. and sympathetic paragangliomas: Primary tumor size and primary tu-
This patient had an extra-adrenal PGL that was 5.5 cm. mor location as prognostic indicators. J Clin Endocrinol Metab.
Clinical genetic testing found that he carried an SDHB 2011;96(3):717-725.
4. Zeiger MA, Thompson GB, Duh QY, et al. The American Association of
mutation as did his 15-year-old daughter. There are no
Clinical Endocrinologists and American Association of Endocrine Sur-
formal guidelines for following SDHx mutation carriers, but geons medical guidelines for the management of adrenal incidentalomas.
most experts recommend at least annual biochemical test- Endocr Pract. 2009;15 Suppl 1:1-20.
ing and cross-sectional imaging studies of the neck/ 5. Fishbein L, Nathanson KL. Pheochromocytoma and paraganglioma: Un-
chest/abdomen/pelvis every 2 years (5). His daughter was derstanding the complexities of the genetic background. Cancer Genet.
2012;205(1-2):1-11.
screened annually with plasma metanephrines and cat- 6. Lenders JW, Duh QY, Eisenhofer G, et al. Pheochromocytoma and para-
echolamines and full-body MRI every 2 years. The patient ganglioma: An endocrine society clinical practice guideline. J Clin
had biochemical testing 2 months postoperatively, 6 months Endocrinol Metab. 2014;99(6):1915-1942.
postoperatively for the first year, and then annually after that 7. Mozley PD, Kim CK, Mohsin J, Jatlow A, Gosfield E 3rd, Alavi A. The
efficacy of iodine-123-MIBG as a screening test for pheochromocytoma.
with full-body imaging planned every 2 years. Unfortu-
J Nucl Med. 1994;35(7):1138-1144.
nately, 2 years postoperatively he developed back pain and 8. Weingarten TN, Cata JP, OHara JF, et al. Comparison of two preoperative
was found to have a T12 spinal metastasis. This was treated medical management strategies for laparoscopic resection of pheochromo-
with external beam radiation. Within the next year he devel- cytoma. Urology. 2010;76(2):508 e6 e11.
oped several distant metastases in the bone and liver, and he 9. Fishbein L, Merrill S, Fraker DL, Cohen DL, Nathanson KL. Inherited
mutations in pheochromocytoma and paraganglioma: Why all patients
was treated with CVD chemotherapy (cyclophosphamide,
should be offered genetic testing. Ann Surg Oncol. 2013;20(5):1444-1450.
dacarbazine, vincristine) as his disease was not MIBG avid. 10. Eisenhofer G, Lenders JW, Siegert G, et al. Plasma methoxytyramine: A
novel biomarker of metastatic pheochromocytoma and paraganglioma in
Case 4 relation to established risk factors of tumour size, location, and SDHB
mutation status. Eur J Cancer. 2012;48(11):1739-1749.
Before this patient had symptoms, what was the likelihood he
11. Thompson LD. Pheochromocytoma of the Adrenal gland Scaled Score
had an SDHD mutation and would develop a PCC/PGL? (PASS) to separate benign from malignant neoplasms: A clinicopatho-
SDHD mutations are autosomal dominant with a paternal logic and immunophenotypic study of 100 cases. Am J Surg Pathol.
inheritance. The sons risk of being a mutation carrier is 50% 2002;26(5):551-566.
because one of his parents has the mutation, and if he carries 12. Wu D, Tischler AS, Lloyd RV, et al. Observer variation in the application
of the Pheochromocytoma of the Adrenal Gland Scaled Score. Am J Surg
the mutation, he is at high risk of developing PCC/PGL be-
Pathol. 2009;33(4):599-608.
cause of the high penetrance with paternal inheritance. 13. Kimura N, Takayanagi R, Takizawa N, Itagaki E, et al. Pathological
If this patients mother had the SDHD mutation, would his risk grading for predicting metastasis in phaeochromocytoma and paragan-
be the same? glioma. Endocr Relat Cancer. 2014;21(3):405-414.
If this patients mother carried the known familial SDHD mu- 14. Fishbein L. Pheochromocytoma and paraganglioma: Genetics, diagnosis
and treatment. Hematol Oncol Clin North Am. 2016;30(1):135-150.
tation, the patient would still have a 50% risk of carrying the 15. Favier J, Amar L, Gimenez-Roqueplo AP. Paraganglioma and phae-
mutation; however, he would not be at risk to develop PCC/PGL ochromocytoma: From genetics to personalized medicine. Nat Rev
as the SDHD mutations are associated with PCC/PGL only when Endocrinol. 2015;11(2):101-111.

The Endocrine Society. Downloaded from press.endocrine.org by [${individualUser.displayName}] on 12 January 2017. at 12:19 For personal use only. No other uses without permission. . All rights reserved.
32 ENDO 2016 MEET-THE-PROFESSOR CLINICAL CASE MANAGEMENT

Adrenal Insufficiency in Intensive Care Patients

M33 tients with Cushings syndrome as it is characterized by


Presented, April 1 4, 2016 710-fold increase in serum free cortisol levels, loss of diurnal
variation, poor suppressibility with dexamethasone, and en-
hanced responsiveness to Cosyntropin. Although increased cor-
Baha M. Arafah, MD, FACP. Case Western tisol secretion is evident, an important factor contributing to the
Reserve University, Cleveland, Ohio 44106, E-mail: high serum cortisol levels in critically ill patients is a decrease
baha.arafah@case.edu in its clearance that is likely due to inhibition of the 11-
hydroxy steroid dehydrogenase-2 enzyme activity.
HISTORICAL PROSPECTIVE AND OVERVIEW
OF THE CLINICAL PROBLEM Limitations of Current Assessment Approaches of HPA
Although the essential role of the adrenal glands for survival Function in the Critically Ill
had been recognized for centuries, the work of Dr Thomas An important limitation of most published data are the consis-
Addison published in 1855 as a monograph provided the most tently overlooked effect of hypoproteinemia on measured se-
detailed and still accurate description of patients with primary rum cortisol levels. Current thinking suggests that the free
adrenal insufficiency. The biochemical studies conducted in the fraction of the hormone is the biologically active form. Avail-
1930s led to the identification of several adrenocortical steroids able assays for serum cortisol measurements determine the
and culminated in awarding the Noble Prize in Medicine or total (ie, free and bound fractions) concentration. With more
Physiology to three prominent scientists: Edward Kendall, than 90% of cortisol in the circulation being protein bound, it
Tadeus Reichenstein, and Philip Hench. That discovery led to would easy to appreciate the significant effect of alterations in
the synthesis of several steroids including cortisone and hydro- binding proteins (transcortin and albumin) on measured serum
cortisone and subsequently the recognition that their therapeu- total cortisol concentrations. Thus, patients with hypoproteine-
tic potential goes far beyond treating patients with adrenal mia may exhibit lower-than-expected serum total cortisol lev-
insufficiency but also others with a variety of medical disorders els even though they have normal HPA function and normal
such as allergic, autoimmune, and lymphoproliferative dis- free hormone concentrations. The reverse is also true about
eases, to name a few. Over time, physicians not only became patients with increased binding proteins such as those on estro-
aware of glucocorticoid benefits but also of their serious ad- gen therapy and others with hepatitis. Given that critical illness
verse effects. In the early 1990s, the high morbidity and mor- is frequently associated with variable degrees of hypoproteine-
tality in critically ill patients prompted several studies that mia, it would be easy to appreciate its effect in underestimating
examined the presence of adrenal dysfunction in such pa- glucocorticoid secretion in this setting. This invariably leads to
tients and whether treatment with glucocorticoids would be the inaccurate diagnosis of adrenal insufficiency and unneces-
beneficial. Many such studies have serious limitations and sary use of glucocorticoids. In such patients free cortisol levels
provided conflicting results. At this time and despite the large are appropriately elevated.
volume of published data on adrenal function during critical A common and serious concern reported frequently is the
illness, several controversies continue to be debated such as the use of post-Cosyntropin increment in serum cortisol levels of
definition of normal adrenal response, the concept of rela- less than 9 ug/dL as a diagnostic criterion for impaired adrenal
tive adrenal insufficiency and the indications and the doses of function irrespective of the baseline level (the so-called relative
glucocorticoids needed in critically ill patients with impaired adrenal insufficiency). Although that increment might have a
HPA function. prognostic value, it is of no physiologic significance and should
never be used to diagnose adrenal dysfunction or to determine
LEARNING OBJECTIVES: need for glucocorticoids. Most patients who had a post-
After completing this session, the participant will be able to: Cosyntropin increment of less that 9 ug/dL have hypoproteine-
Understand HPA function in critically ill patients. mia and in fact, up to 20% of normal healthy subjects exhibit
Recognize limitations of methods used in assessing HPA the same biochemical feature.
function.
Determine whether glucocorticoid management is Diagnosis of Adrenal Insufficiency During Critical
indicated in a critically ill patient with suspected adrenal Illness
insufficiency. When adrenal insufficiency is partial, biochemical confirmation
of the diagnosis can at times be difficult in ambulatory patients
Normal HPA During Critical Illness and even in those who are critically ill. The variability in
Critically ill patients with normal HPA function have persistent response to stressors and the many confounding factors influ-
hypercortisolemia that is reminiscent of that observed in pa- encing serum cortisol measurements make the biochemical

The Endocrine Society. Downloaded from press.endocrine.org by [${individualUser.displayName}] on 12 January 2017. at 12:19 For personal use only. No other uses without permission. . All rights reserved.
ENDO 2016 ADRENAL/HPA AXIS 33

diagnosis of adrenal insufficiency more difficult during critical Management of Adrenal Insufficiency During Critical
illness. One should seriously consider the diagnosis in patients Illness
with symptoms especially those at risk for having adrenal Traditionally, patients with adrenal insufficiency have been
insufficiency. Risk factors include a prior history of using overtreated, particularly during critical illness. Despite the lack
glucocorticoids or drugs that have glucocorticoid activity (eg, of evidence, many textbooks or reviews continue to recom-
mend the use of large dose of hydrocortisone (the so called
Megace) and others that can potentially influence their secre-
stress dose). The latter total dose can reach 300 400 mg daily.
tion such as etomidate. The latter drug is an anesthetic agent
Such doses are clearly associated with adverse events such as
frequently used in critically ill patients to help with intubation
hyperglycemia, hypokalemia, and secondary infections, to
and is known to inhibit the 11- hydroxylase enzyme. Thus,
name a few. Recently however, there is an increasing trend to
patients receiving that drug will have adrenal insufficiency that treat adrenally insufficient subjects with lower doses of hydro-
may persist for up to 24 hours after its administration and cortisone in the ambulatory setting and hopefully during criti-
would therefore need hydrocortisone coverage during that time cal illness as well. We have previously demonstrated that
period. Additional risk factors include a personal or family adrenally insufficient subjects given 50 mg of hydrocortisone
history of autoimmune illnesses, know hypothalamic/pituitary every 6 hours achieve serum cortisol levels that are several-
disease, or brain irradiation to name a few. fold higher than those observed in most critically ill patients. A
When serum-binding proteins (albumin, transcortin) are near decrease in cortisol clearance, which is recently shown to be a
normal (albumin 2.5 g/dL), measurements of total serum feature of critical illness contributes to the high serum levels.
cortisol provide reliable assessment of adrenal function in criti- Currently, there are no data to offer specific guidelines on
cally ill patients, in whom a random serum total cortisol would appropriate glucocorticoid doses during critical illness nor are
be expected to 15 ug/dL in most patients. In hypoproteinemic there any data on the normal HPA function/response to specific
(albumin 2.5 g/dL) critically ill subjects, a random serum illnesses.
total cortisol level is expected to be 11 ug/dL in most pa- Based on our own data, we believe that unless the patient is
in shock, the use of 50 mg of hydrocortisone every 6 hours
tients. There is no need to perform a Cosyntropin stimulation
during any critical illness is excessive and unnecessary. Our
test in the setting of critical illness as this will not provide any
approach is to use lower doses of hydrocortisone during critical
additional meaningful information. However, if the test is per-
illness. Thus, if the critically ill, adrenally insufficient patient is
formed, the postCosyntropin serum total and free cortisol lev-
not in shock, we use a dose of 25 mg every 6 hours and taper
els should be interpreted with the understanding that responses down as clinically indicated. Patients with primary adrenal
in critically ill subjects are higher than those of healthy ambu- insufficiency will not need mineralocorticoid therapy as long as
latory volunteers. The Cosyntropin-induced increment in serum the total dose of hydrocortisone used is greater than 50 mg per
total cortisol should not be used as a criterion for defining day. Critically ill patients in shock have a significant inflam-
normal or abnormal adrenal function, especially in critically ill matory response and might therefore need the higher hydrocor-
patients. We have recently demonstrated that measurements of tisone doses. In that setting, the hydrocortisone is more of an
serum dehydroepiandrosterone sulfate (DHEA-S) are helpful in anti-inflammatory agent than a glucocorticoid replacement.
establishing the diagnosis of adrenal insufficiency in the ambu-
latory setting and to a large degree in the critically ill. Use of Glucocorticoids in the Critically Ill Without
Theoretically, serum free cortisol measurement is the most Adrenal Insufficiency
accurate method to assess adrenal function in critically ill The routine use of glucocorticoids during critical illness is not
hypoproteinemic patients. A random serum free cortisol level justified except in patients where adrenal insufficiency was
is expected to be 1.8 ug/dL in most critically ill patients, properly diagnosed, or in others who are hypotensive, septic,
irrespective of the serum-binding proteins. Given that the free and unresponsive to standard therapy (fluids, antibiotics, and
2 pressors). In the latter setting, one would not be treating an
cortisol assay is not currently available for routine clinical use,
episode of adrenal insufficiency but rather a severe inflamma-
alternative approaches to estimate serum free cortisol can be
tory process resulting from sepsis. The results of the recently
used. These include calculated free cortisol (Coolens method)
published CORTICUS study are supportive of that. Among
and determining the free cortisol index (ratio of serum
available glucocorticoids, hydrocortisone should be the drug of
cortisol/transcortin concentrations). The latter method (index) choice and should be given at the lowest dose and for the
is unreliable and is poorly conceived as it does not have any shortest duration possible. The hydrocortisone dose (50 mg
physiological basis. Recent data suggest that salivary cortisol every 6 h) that is inappropriately labeled as low-dose hydro-
measurements could be a reliable alternative approach to esti- cortisone leads to excessive elevation in serum cortisol levels
mating the free cortisol in the circulation. From a practical and are often associated with significant adverse effects. The
point of view, one would need to rely on total cortisol mea- latter discussion should call into question the current practice
surements and be aware of its limitations as discussed earlier. by many of using such doses of hydrocortisone in any patient

The Endocrine Society. Downloaded from press.endocrine.org by [${individualUser.displayName}] on 12 January 2017. at 12:19 For personal use only. No other uses without permission. . All rights reserved.
34 ENDO 2016 MEET-THE-PROFESSOR CLINICAL CASE MANAGEMENT

even those with true adrenal insufficiency. Hydrocortisone After a few hours in the ICU, she was begun on two pressors
therapy should be tapered and discontinued as clinically indi- and had a stable BP (systolic, 110-115 mm Hg) for several
cated in those who receive the drug without definitive diagno- hours. The ICU team obtained a random and a 30-minute
sis given that prolonged use of the drug can lead to iatrogenic post-Cosyntropin serum cortisol levels that were 19 and 23
HPA suppression. ug/dL, respectively. Her Na was 145 meq/L; the K was 4.4
meq/L, whereas the albumin was 1.7 g/dL.
1. Based on the available data, do you believe that this
CASES
patient has adrenal insufficiency?
Case 1
A. Yes
You evaluate a 77-year-old man with persistent postoperative
B. No
hypotension and hypothermia (core temp of 34C) following
C. Cannot determine
triple vessel coronary bypass surgery. His wife stated that he
2. The post-Cosyntropin increment in serum cortisol noted
had surgery to remove a pituitary tumor 10 years earlier, which
in this patient can be:
was followed a year later by gamma knife therapy. He had
A. Observed in ambulatory healthy subjects
complained of fatigue and muscle cramps for several months
B. Always indicative of adrenal dysfunction
and that his primary medical doctor measured a TSH that was
C. Predictive of a potential benefit from hydrocortisone
within the normal range. At the time of your assessment, the
therapy
patient is on iv fluids and two pressors with a blood pressure
3. Which of the following is the best interpretation of the
(BP) of 100/65 mm Hg and a heart rate of 75/min in normal
Cosyntropin stimulation test?
sinus rhythm. He looks pale, with diminished body hair, and
A. Normal
exhibits delayed relaxation of his reflexes. Available laboratory
B. Primary adrenal insufficiency
data shows a Na of 133 meq/L, K of 4.1 meq/L, albumin of
C. Secondary adrenal insufficiency
2.8 g/dL, a random cortisol (1230 h) of 6.8 ug/dL, a free T4 of
4. Which of the following is the best next test in this
0.6 ng/dL, and a TSH of 2.9 IU/L. The intensive care unit
patients evaluation?
(ICU) team orders a Cosyntropin test (250 ug iv), which shows
A. Magnetic resonance imaging of pituitary
cortisol levels of 15 and 19 ug/dL at 30 and 60 minutes, B. Computed tomography of adrenal
respectively. C. Random cortisol paired with ACTH level
1. How do you interpret the random cortisol? D. Serum aldosterone paired with plasma renin activity
A. Normal E. No further endocrine testing
B. Suggestive of adrenal insufficiency
C. As expected for a critically ill patient
Case 3
2. The Cosyntropin-stimulated cortisol levels rule out the A 55-year-old man with a history of metastatic small-cell lung
diagnosis of adrenal insufficiency in this patient. cancer was admitted to the ICU with fever, respiratory distress,
A. Yes and hypotension. He has been getting chemotherapy along with
B. No Megace because of anorexia. He was recently diagnosed to
3. Which of the following endocrine disorders were present have hepatitis C but treatment was delayed in light of his
before surgery? progressive and unresponsive lung cancer. In the ICU, he was
A. Adrenal insufficiency started on antibiotics, iv fluids, and two pressors but despite
B. Hypothyroidism that, he remained hypotensive. A random cortisol performed at
C. Hypogonadism 1400 hours was 17 ug/dL. You are asked to interpret the data
D. Diabetes insipidus and offer your opinion on using hydrocortisone in this patient.
4. Which of the following is the best first hormonal treat- Your examination did not reveal any additional finding. Other
ment to administer? laboratory data showed the following: Na, 142 meq/L; K,
A. Hydrocortisone 4.3 meq/L; a total protein of 6.5 g/dL, and an albumin of
B. Thyroid hormone 2.5 g/dL.
C. Testosterone 1. Based on the available data, do you believe that this
D. Flucortisone patient has adrenal insufficiency?
A. Yes
Case 2 B. No
You evaluate a 65-year-old woman who was transferred from C. Maybe
the medical ward to the ICU because of hypotension. She was 2. Which of the following factors should be considered in
admitted a day earlier for suspected pyelonephritis sepsis interpreting cortisol level in this patient?
and was treated with iv fluids and appropriate antibiotics. Her A. Critical illness
blood pressure (BP) decreased to a nadir of 82/60 mm Hg. B. History of lung cancer

The Endocrine Society. Downloaded from press.endocrine.org by [${individualUser.displayName}] on 12 January 2017. at 12:19 For personal use only. No other uses without permission. . All rights reserved.
ENDO 2016 ADRENAL/HPA AXIS 35

C. History of hepatitis C. Laboratory error


D. Laboratory error D. Drug effect
E. Drug effect E. Malignant melanoma
3. Which of the following is the best next test in this 3. Which of the following is the most likely cause of the
patients evaluation? polyuria and polydipsia?
A. Magnetic resonance imaging of pituitary A. Central diabetes insipidus
B. Cosyntropin stimulation test B. Nephrogenic diabetes insipidus
C. Measurement of free cortisol, transcortin, and ACTH A. Medication effect
D. Serum aldosterone paired with plasma renin activity D. Osmotic diuresis
4. Which of the following would you recommend to the
ICU team? REFERENCES
A. Await the results of tests that are ordered 1. Hamrahian AH, Oseni TS, Arafah BM. Measurements of serum free
B. Start hydrocortisone therapy cortisol in critically ill patients. N Engl J Med. 2004;350:1629-1638.
C. Start therapy for hepatitis C 2. Arafah BM. Hypothalamic Pituitary Adrenal Function during Critical
Illness: Limitations of Current Assessment Methods. J Clin Endocrinol
D. Start fludrocortisone
Metab. 2006;91:3725-3745.
3. Arafah BM, Nishiyama FJ, Tlaygeh H, Hejal R. Measurement of salivary
Case 4 cortisol concentration in the assessment of adrenal function in critically ill
subjects: A surrogate marker of the circulating free cortisol. J Clin
A 58-year-old woman with malignant melanoma was admitted
Endocrinol Metab. 2007;92:2965-2971.
to the ICU with fever and hypotension. Two weeks earlier she 4. Sprung CL, Annane D, Keh D, et al. CORTICUS Study Group. Hydrocor-
had started complaining of polyuria, polydipsia, headache, tisone therapy for patients with septic shock. N Eng J Med. 2008;358:111-124.
blurry vision, and intermittent diplopia. Ipilimumab was re- 5. Zimmerman JJ, Barker RM, Jack R. Initial observations regarding free
cortisol quantification logistics among critically ill children. Intensive Care
cently initiated to treat metastatic melanoma. A computed to-
Med. 2010;36:1914-1922.
mography scan of the abdomen/chest showed widespread 6. Albert SG, Ariyan S, Rather A. The effect of etomidate on adrenal
metastatic disease and bowel perforation. Her examination function in critical illness: a systematic review. Intensive Care Med.
showed a BP of 80/50 mm Hg, a heart rate of 110/min, with 2011;37:901-910.
7. Al-Aridi R, Abdelmannan D, Arafah BM. Biochemical diagnosis of adre-
poor skin turgor, anisocoria, and a left ptosis. Laboratory data nal insufficiency: the added value of dehydroepiandrosterone sulfate mea-
showed a blood urea nitrogen of 32 mg/L, a creatinine of 1.6 surements. Endocr Pract. 2011;17:261-270.
mg/dL a Na of 148 meq/L, a K of 4.5 meq/L, a random 8. Boonen, E, Vervenne H, Meersseman P, et al. Reduced cortisol metabo-
serum cortisol of 11.1 ug/dL and negative urinalysis. She lism during critical illness. N Engl J Med. 2013;368:1477-1488.
9. Al-Aridi R, El Sibai K, Fu P, Khan M, Selman WR, Arafah BM. Clinical
started on iv saline and antibiotics. and biochemical characteristic features of metastatic cancer to the sella
1. Based on the available data, do you believe that this turcica: An analytical review. Pituitary. 2014;17:575-587.
patient has adrenal insufficiency? 10. Heckmann M, dUscio CH, Steckel H, et al. Reduction in cortisol inacti-
vation is part of the adrenal response to cardiac and noocardiac pediatric
A. Yes
surgery: A prospective study using gas chromatography-mass spectrometry
B. No analysis. Horm Metab Res. 2014;46:677-684.
C. Maybe 11. Boonen E, Van den Berghe G. Endocrine responses to critical illness:
2. Which of the following is the most likely cause of her novel insights and therapeutic implications. J Clin Endocrinol Metab.
2014;99:1569-1582.
clinical picture?
12. Faje AT, Sullivan R, Lawrence D, et al. Ipilimumab-induced hypophysitis:
A. Primary adrenal insufficiency A detailed longitudinal analysis in a large cohort of patients with meta-
B. Secondary adrenal insufficiency static melanoma. J Clin Endocrinol Metab. 2014;99:4078-4085.

The Endocrine Society. Downloaded from press.endocrine.org by [${individualUser.displayName}] on 12 January 2017. at 12:19 For personal use only. No other uses without permission. . All rights reserved.
36 ENDO 2016 MEET-THE-PROFESSOR CLINICAL CASE MANAGEMENT

Primary Aldosteronism

M46 the immense delight of Conn and those in the operating room,
Presented, April 1 4, 2016 the surgeon, Dr William Baum, encountered a right 13-g adre-
nal tumor, which was removed while leaving the contralateral
gland intact. The patients postoperative studies showed an
William F. Young, Jr, MD, MSc. Division of almost total reversal of the preoperative metabolic and clini-
Endocrinology, Diabetes, Metabolism, and Nutrition, cal abnormalities. Conn had achieved irrefutable proof of
Mayo Clinic, Rochester, Minnesota 55905, E-mail: the validity of his investigative conclusions and established
young.william@mayo.edu for the first time the relationship among adrenal aldosterone-
producing tumors, hypertension, and hypokalemia. A new
INTRODUCTION era had arrived in the study of hypertension and adrenal
Historical Overview mineralocorticoids.
In his presidential address at the Annual Meeting of the Central
Society for Clinical Research in Chicago, Illinois, October 29, SIGNIFICANCE OF THE CLINICAL PROBLEM
1954, Dr Jerome W. Conn stated, I have prepared no compre- Following Conns prismatic case, it became clear that there
hensive review of my personal philosophy of clinical investi- were two major forms of primary aldosteronism (PA): bilateral
gation. Instead, I plan to make a scientific report to you about a idiopathic hyperaldosteronism and aldosterone-producing ad-
clinical syndrome, the investigation of which has been most enoma (APA). Although initially thought to be rare, prevalence
exciting to me since I initiated it in April of this year (1). studies from every continent have now documented that PA is
Conn, a professor of medicine at the University of Michigan, the most common form of secondary hypertension, affecting
had been active in government-funded research on the mecha- 510% of all people with hypertension.
nisms of human acclimatization to humid heat. He established Knowledge gaps among clinicians are prevalent with regard
that the bodys acclimatization response was to rapidly dimin- to the diagnosis and management of patients with PA. For
ish renal salt and water loss and abruptly curtail the salt content example, many clinicians are not aware of the effect of time of
of body sweat and saliva, and he suggested that it was due to day for venipuncture on case detection testing. This is just one
increased adrenocortical function with elaboration of a salt- of more than 20 knowledge gaps that I hope to cover (close)
retaining steroid termed electrocortin, later named aldoste- during this session.
rone. He also showed that im administration of deoxycortico-
sterone acetate produced similar changes in the electrolyte
composition of urine, sweat, and saliva. BARRIERS TO OPTIMAL PRACTICE
The term barriers is continuing professional development
In April 1954, he was asked to see M.W., a 34-year-old
jargon in the United States for identifying issues that prevent or
woman with a 7-year history of muscle spasms, temporary
inhibit clinicians from optimal diagnosis and treatment of a
paralysis, tetany, weakness, and a 4-year history of hyperten-
medical condition. For example, understanding the potential
sion. She had a blood pressure (BP) of 176/104 mm Hg, severe
barriers posed by clinical guidelines for the diagnosis and
hypokalemia (potassium, 1.6-2.5 mEq/L), mild hypernatremia
treatment of PA: How do you confirm PA in a patient on four
(sodium, 146-151 mEq/L), and alkalosis (serum pH, 7.62).
antihypertensive drugs and poor BP control? The clinical
Because there were no signs or symptoms of glucocorticoid or
guidelines suggest that most antihypertensive drugs should be
androgen excess, Conn suspected, on the basis of his past
discontinued in all patients for this confirmatory step. How-
research, that M.W.s clinical presentation could result from
ever, in the clinical setting listed, discontinuing antihyperten-
excess secretion of the adrenal salt-retaining corticoid. Conn
sive medications will put the patient at risk. This issue and
studied M.W. in the Metabolism Research Unit for 227 days.
many others will be discussed during this MTP session.
Using Streetens bioassay technique to measure sodium reten-
tion in adrenalectomized rats after ip injection of human urine,
M.W. averaged 1333 mcg deoxycorticosterone equivalent per LEARNING OBJECTIVES
day compared with normotensive controls at 61.4 mcg per day. As a result of participating in this session, learners should be
In his presidential address, Conn stated, It is believed that able to:
these studies delineate a new clinical syndrome which is des- Recognize when to test for PA
ignated temporarily as primary aldosteronism (1) (The word Implement the key steps to successful case detection
temporarily was used because aldosterone was yet to be testing for PA
measured in any human bodily fluid.) Conn planned for a Discuss the options for confirmatory testing in a patient
bilateral adrenalectomy on December 10, 1954. In the 1995 with PA and select the most appropriate study for a
retelling of the surgical scene, Gittler and Fajans (2) wrote, To particular individual.

The Endocrine Society. Downloaded from press.endocrine.org by [${individualUser.displayName}] on 12 January 2017. at 12:19 For personal use only. No other uses without permission. . All rights reserved.
ENDO 2016 ADRENAL/HPA AXIS 37

Assess the role for subtype testing for PA Step 2: How to Perform Case-Detection Testing
Counsel patients on the treatment options for PA In patients with suspected PA, case-detection testing can be
accomplished by measuring a morning (preferably between
Session Format 0800 and 1000 h) ambulatory paired random plasma aldoste-
We will use a computer-based clinicopathologic conference rone concentration (PAC) and plasma renin activity (PRA).
approach to discuss a patient with suspected PA. The tests This test may be performed while the patient is taking antihy-
performed and the order in which they are obtained will be pertensive medications (with some exceptions [see below]) and
up to the clinicians in the audience. Many potential without posture stimulation. Hypokalemia reduces the secre-
tests/procedures are listed on the test menu (Table 1); however, tion of aldosterone, and it is optimal (but not necessary in most
just because they are listed, it does not mean that they were cases) to restore the serum level of potassium to normal before
actually performed. performing diagnostic studies. Mineralocorticoid receptor
We will then turn to two clinical vignettes (found at the end (MR) antagonists (eg, spironolactone and eplerenone), renin
of this syllabus section on PA) that lead to multiple-choice inhibitors, and high-dose (5 mg/d) amiloride are the only
questions. medications that absolutely interfere with interpretation of the
Finally, if time allows, we will turn to common e-mail ratio in patients with PA and should be discontinued at least 6
questions that I have received with regard to the diagnosis and weeks before testing if clinically feasible. ACE inhibitors,
treatment of PA. angiotensin receptor antagonists (ARBs), and diuretics have the
potential to falsely elevate PRA. Therefore, in a patient
treated with an ACE inhibitor, ARB, or diuretic, the finding of
STRATEGIES FOR DIAGNOSIS, THERAPY,
a detectable PRA level or a low PAC/PRA ratio does not
AND/OR MANAGEMENT
exclude the diagnosis of PA. However, a very useful clinical
Step 1: When to Test for PA
Patients with hypertension and hypokalemia (regardless of pre- point is that when a PRA level is undetectably low in a patient
sumed cause), treatment-resistant hypertension (three antihy- taking an ACE inhibitor, ARB, or a diuretic, PA should
pertensive drugs and poor control), marked hypertension be highly suspect. Thus, ACE inhibitors, ARBs, and
(150 mm Hg systolic or 100 mm Hg diastolic), hyperten- nonpotassium-sparing diuretics do not need to be discontinued.
sion and an incidental adrenal mass, and onset of hypertension A second important clinical point is that the PRA is suppressed
at a young age should undergo screening for PA. In addition, (1.0 ng/mL/h) in almost all patients with PA. Adrenergic
the diagnosis of PA should be considered whenever performing inhibitors (eg, -adrenergic blockers and central 2 agonists)
a secondary hypertension evaluation (eg, when testing for reno- suppress renin secretion, but also in turn suppress aldosterone
vascular disease or pheochromocytoma). secretion (although to a lesser degree than renin) in normal
individuals; thus, although the PAC/PRA may increase in hy-
pertensive patients without PA treated with adrenergic inhibi-
tors, the PAC remains less than 15 ng/dL (416 pmol/L) and the
TABLE 1. Hyperaldosteronism Test Menu
case detection test is not significantly affected.
Test Menu The PAC/PRA ratio is based on the concept of paired
u Clinical assessment hormone measurements. For example, in a hypertensive hy-
u PAC/PRA ratio pokalemic patient:
u 24-h urinary potassium Secondary hyperaldosteronism should be considered
u Captopril suppression test when both PRA and PAC are increased and the PAC/
u Saline suppression test PRA ratio is less than 10 (277 in SI units) (eg,
u 24-h urine aldosterone on a high-salt diet renovascular disease).
u Fludrocortisone suppression test An alternate source of MR agonism (eg,
u Renal angiogram hypercortisolism) should be considered when both PRA
u Posture test and PAC are suppressed.
u 18-OH B PA should be suspected when PRA is suppressed (1.0
u NP-59 scan ng/mL/h) and PAC is increased.
u CT adrenals/abdomen
u MRI adrenals It is important to understand that the lower limit of detection
u Adrenal vein sampling varies among different PRA assays and can have a dramatic
u GRA test effect on the PAC/PRA ratio. Thus, the cutoff for a high
u Treatment menu PAC/PRA ratio is laboratory dependent and, more specifically,
PRA-assay dependent. In a retrospective study, the combina-
Abbreviation: MRI, magnetic resonance imaging. tion of a PAC/PRA ratio more than 30 (832 in SI units) and
PAC more than 20 ng/dL (555 pmol/L) had a sensitivity of

The Endocrine Society. Downloaded from press.endocrine.org by [${individualUser.displayName}] on 12 January 2017. at 12:19 For personal use only. No other uses without permission. . All rights reserved.
38 ENDO 2016 MEET-THE-PROFESSOR CLINICAL CASE MANAGEMENT

90% and a specificity of 91% for APA. At Mayo Clinic, a PAC testing. Aldosterone suppression testing can be performed with
(in pmol/L)/PRA (in ng/mL/h) ratio of 20 (555 in SI units) or orally administered sodium chloride and measurement of uri-
more and PAC of at least 15 ng/dL (416 pmol/L) are found in nary aldosterone or with iv sodium chloride loading and mea-
more than 90% of patients with surgically confirmed APA. surement of PAC.
Thus, there are patients with PA (especially IHA) who have
PAC levels 15 ng/dL. However, most patients with surgically
Oral Sodium Loading Test
correctable forms of PA will have PACs greater than this After hypertension and hypokalemia are controlled, patients
cutoff. should receive a high-sodium diet (supplemented with sodium
A PAC of 15 ng/dL (416 pmol/L) is in the high-normal chloride tablets if needed) for 3 days, with a goal sodium intake
range (normal range, 121 ng/dL; 28 583 pmol/L). In patients of 218 mmol of sodium (equivalent to 12.8 g sodium chloride).
without PA, most of the variation in PAC/PRA ratios occurs The risk of increasing dietary sodium in patients with severe
within the normal range. A high PAC/PRA ratio is a positive hypertension must be assessed in each case. Because the high-
case detection test result, a finding that warrants further testing. salt diet can increase kaliuresis and hypokalemia, vigorous
It is important for the clinician to recognize that the replacement of potassium chloride may be needed and the
PAC/PRA ratio is only a case-detection tool, and all positive serum level of potassium should be monitored daily. On the
results should be followed by a confirmatory aldosterone sup- third day of the high sodium diet, a 24-hour urine specimen is
pression test to verify autonomous aldosterone production be- collected for measurement of aldosterone, sodium, and creati-
fore treatment is initiated. In a study of 118 subjects with nine. To document adequate sodium repletion, the 24-hour
essential hypertension, neither antihypertensive medications urinary sodium excretion should exceed 200 mmol. Urinary
nor acute variation of dietary sodium affected the accuracy of aldosterone excretion more than 12 mcg/day (33 nmol/d) in
the PAC/PRA ratio adversely, with a sensitivity on and off this setting is consistent with autonomous aldosterone secre-
therapy of 73 and 87%, respectively, and a specificity of 74 and tion. The sensitivity and specificity of the oral sodium loading
75%, respectively (3). test are 96 and 93%, respectively.
The measurement of PRA is time consuming, shows poor
interlaboratory variability, and requires special preanalytical
prerequisites. To overcome these disadvantages, a monoclonal Intravenous Saline Infusion Test
antibody against active renin is being used by several reference The iv saline infusion test has also been used widely for the
laboratories to measure plasma renin concentration (PRC) in- diagnosis of PA. Normal subjects show suppression of PAC
stead of PRA. However, few studies have focused on compar- after volume expansion with isotonic saline; subjects with PA
ing the different methods in the testing for PA and these studies do not show this suppression. The test is done after an over-
lack confirmatory testing. Until such studies are completed it night fast. Two liters of 0.9% sodium chloride solution are
would be reasonable to consider a positive PAC/PRC test when infused iv with an infusion pump over 4 hours into the recum-
the PAC is more than 15 ng/dL (416 pmol/L) and the PRC is bent patient. However, results from a recent study suggest that
below the lower limit of detection for the assay. performing the saline infusion test in the seated position may
improve the accuracy of this test (4). Blood pressure and heart
rate are monitored during the infusion. At the completion of the
Step 3: Confirmatory Testing
An increased PAC/PRA ratio is not diagnostic by itself (sensi- infusion, blood is drawn for measurement of PAC. PAC levels
tivity and specificity are 75%), and PA must be confirmed by in normal subjects decrease to less than 5 ng/dL (139
demonstrating inappropriate aldosterone secretion. The list of pmol/L); most patients with PA do not suppress to less than 10
drugs and hormones capable of affecting the renin-angiotensin- ng/dL (277 pmol/L); postsaline infusion PAC values between
aldosterone axis is extensive, and frequently in patients with 5 and 10 ng/dL (139 and 277 pmol/L) are indeterminate and
severe hypertension, a medication-contaminated evaluation can be seen in patients with IHA.
is unavoidable. Certain calcium channel blockers (eg, vera-
pamil) and 1-adrenergic receptor blockers do not affect the Captopril Stimulation Test and Fludrocortisone
diagnostic accuracy in most cases. It is impossible to interpret Suppression Test
data obtained from patients receiving treatment with MR an- These tests are discussed in more detail elsewhere (5).
tagonists (eg, spironolactone, eplerenone), direct renin inhibi-
tors, or high-dose amiloride when PRA is not suppressed. Step 4: Subtype Testing
Therefore, treatment with a MR antagonist should not be initi- Unilateral adrenalectomy in patients with APA or unilateral
ated until the evaluation has been completed and the final adrenal hyperplasia results in normalization of hypokalemia in
decisions regarding treatment have been made. If PA is sus- all; hypertension is improved in all and is cured in approxi-
pected in a patient receiving treatment with a MR antagonist or mately 30 60% of these patients. In IHA unilateral or bilateral
high-dose amiloride, if clinically feasible, the treatment should adrenalectomy seldom corrects the hypertension. IHA and glu-
be discontinued for at least 6 weeks before further diagnostic cocorticoid remediable aldosteronism (GRA) should be treated

The Endocrine Society. Downloaded from press.endocrine.org by [${individualUser.displayName}] on 12 January 2017. at 12:19 For personal use only. No other uses without permission. . All rights reserved.
ENDO 2016 ADRENAL/HPA AXIS 39

medically. Therefore, for those patients who want to pursue a Normalization of BP should not be the only goal in managing
surgical cure, the accurate distinction between the subtypes of the patient with PA. Excess aldosterone is associated with cardio-
PA is a critical step. vascular toxicity; it induces myocardial fibrosis by either stimula-
The subtype evaluation may require one or more tests, the tion of cardiac fibroblasts and/or vascular fibrinoid necrosis.
first of which is imaging the adrenal glands with computed Unilateral laparoscopic adrenalectomy is an excellent treat-
tomography (CT) (Figure 1). When a solitary, hypodense, and ment option for patients with APA or unilateral hyperplasia.
unilateral macroadenoma (1 cm and 2 cm) and normal Although BP control improves in nearly 100% of patients
contralateral adrenal morphology are found on CT in a young postoperatively, average long-term cure rates of hypertension
patient (eg, 35 y) with marked PA (eg, spontaneous hypoka- after unilateral adrenalectomy for APA range from 30 to 60%
lemia and PAC 30 ng/dL), adrenal venous sampling (AVS) (16). Persistent hypertension following adrenalectomy is corre-
may not be needed and consideration should be given to pro- lated directly with having more than one first-degree relative
ceed directly to unilateral laparoscopic adrenalectomy (14). with hypertension, use of more than two antihypertensive
However, in most cases, CT may show normal-seeming agents preoperatively, older age, increased serum creatinine,
adrenals, minimal unilateral adrenal limb thickening, unilateral and duration of hypertension and is most likely due to coexis-
microadenomas (1 cm), or bilateral macroadenomas. In these tent primary hypertension.
cases, if the patient wants to pursue the surgical option, addi- IHA and GRA should be treated medically. In addition,
tional testing is required to determine the source of excess APA patients may be treated medically if the medical treatment
aldosterone secretion (6, 15). includes MR blockade. There have been no placebo-controlled
Although, in general, patients with APA have more severe randomized trials evaluating the relative efficacy of drugs in
PA than patients with IHA, there is no reliable noninvasive the treatment of PA. Spironolactone has been the drug of
clinical predictor profile to make this distinction (7). choice to treat PA for more than three decades. However, it is
For those patients seeking a surgical cure, AVS is an essen- not selective for the MR. For example, antagonism at the T
tial diagnostic step in most patients with PA to distinguish receptor may result in painful gynecomastia, impotence, and
between unilateral and bilateral adrenal aldosterone hyperse- agonism at the progesterone receptor may cause menstrual
cretion (15). We highlighted the keys to successful AVS in a irregularity. Treatment goals are normotension and normokalemia
2009 article (8). without potassium supplementation.
Eplerenone is a competitive and selective MR antagonist
Step 5: Treatment that was approved by the U.S. Food and Drug Administration
The treatment goal is to prevent the morbidity and mortality- for the treatment of uncomplicated essential hypertension and
associated with hypertension, hypokalemia, and cardiovascular congestive heart failure in late 2003.
and renal damage (9). The cause of PA helps to determine the Because of the deleterious cardiovascular and renal effects
appropriate treatment. of excess aldosterone, normalization of circulating aldosterone

Figure 1. Subtype testing and treatment algorithm for primary aldosteronism.

The Endocrine Society. Downloaded from press.endocrine.org by [${individualUser.displayName}] on 12 January 2017. at 12:19 For personal use only. No other uses without permission. . All rights reserved.
40 ENDO 2016 MEET-THE-PROFESSOR CLINICAL CASE MANAGEMENT

or MR blockade should be part of the management plan for all tabs, 4 tabs thrice daily (240 mEq/d). Physical examination
patients with PA (17). Unilateral laparoscopic adrenalectomy is showed a normal-seeming young woman with a BP of 146/82
an excellent treatment option for patients with APA or unilat- mm Hg, heart rate 72 bpm, and body mass index of 24.4 kg/m2.
eral hyperplasia. IHA and GRA should be treated medically. In Laboratory studies completed on current medications:
addition, patients with APA may be treated medically if the Sodium 143 mEq/L (mmol/L)
medical treatment includes MR blockade. Potassium 3.2 mEq/L (mmol/L)
Creatinine 1.0 mg/dL (80 mol/L)
Guidelines PAC 51 ng/dL (1415 pmol/L)
The Endocrine Society clinical practice guideline on PA was Plasma renin activity 0.6 ng/mL per hour
published in 2008 (5) and an updated and revised version will
Abdominal CT scan shows a 1.5-cm solitary right adrenal
be published in early 2016.
cortical nodule (precontrast density 5 HU) and the left
adrenal seems normal on all cuts.
Recent Advances 2. The best next step in the management of this patients
The major recent advance in PA has been the detection of
hypertension and hypokalemia is:
somatic mutations in several genes (eg, KCNJ5 potassium
A. Perform a seated saline suppression test
channel mutations, CACNA1D calcium channel mutations,
B. Adrenal venous sampling
ATPase genes ATP1A1 and ATP2B3), which are responsible
C. [6-131I]iodomethyl-19-norcholesterol [NP-59] scin-
for more than 50% of APAs (10-13). However, thus far, the
tigraphy
presence of a somatic mutation in the APA has not provided the
D. Posture stimulation test
clinician with any novel preoperative tests or new treatments.
E. No further tests are needed, proceed to laparoscopic
right adrenalectomy
CASES WITH QUESTIONS
Case 1
DISCUSSION
A 28-year-old woman presented to her local physician with
Case 1
new-onset hypertension. There was no family history of hyper-
Correct answer: C. Renin is not suppressed. Therefore, this is
tension. The patient does not use tobacco or alcohol. She was
not PA; rather, this is secondary aldosteronism and in the
taking no medications. Physical examination showed a normal-
clinical setting of a young woman the clinician should suspect
seeming woman with a body mass index of 21 kg/m2. Her BP
fibromuscular dysplasia of one or both renal arteries. Thus, an
was 160/106 mm Hg.
imaging study of the renal arteries is indicated.
The following laboratory studies were obtained at 1030
hours:
Sodium 140 mEq/L (mmol/L) Case 2
Correct answer: E. This patient has marked PA and spontaneous
Potassium 3.5 mEq/L (mmol/L)
hypokalemia. Thus, laboratory confirmatory testing is not needed.
Creatinine 1.0 mg/dL (80 mol/L)
This patient can only have one disorder: PA. In addition, in young
PAC 34 ng/dL (943 pmol/L)
people (35 y) with marked PA who have a unilateral
Plasma renin activity 2.6 ng/mL per hour
macroadenoma on adrenal CT, AVS is usually not needed,
The patient is referred to you for further assessment of her whereas in older patients with PA, because of the age-related
hypertension and hypokalemia. development of adrenal nodularity, the findings on CT cannot be
1. The best next step in her evaluation would be to: trusted and AVS is a key and necessary diagnostic step.
A. Perform a seated saline suppression test
B. Repeat the laboratory tests at 0800 hours in the REFERENCES
seated ambulatory patient 1. Conn JW. Presidential address. I. Painting background. II. Primary aldo-
C. Order a magnetic resonance angiogram of the renal steronism, a new clinical syndrome. J Lab Clin Med. 1955;45:3-17.
arteries 2. Gittler RD, Fajans SS. Primary aldosteronism (Conns syndrome). J Clin
Endocrinol Metab. 1995;80:3438-3441.
D. Order a CT scan of the adrenal glands with 2-mm 3. Schwartz GL, Turner ST. Screening for primary aldosteronism in essential
contiguous cuts hypertension: Diagnostic accuracy of the ratio of plasma aldosterone
concentration to plasma renin activity. Clinical Chemistry. 2005;51:386-
394.
Case 2 4. Ahmed AH, Cowley D, Wolley M, Gordon RD, Xu S, Taylor PJ, Stowas-
A 17-year-old woman presented with hypertension of 3 ser M. Seated saline suppression testing for the diagnosis of primary
months duration. Her menstrual cycles were regular and she aldosteronism: A preliminary study. J Clin Endocrinol Metab. 2014;99(8):
2745-2753.
had no hirsutism or acne. Her body weight had been stable. She
5. Funder JW, Carey RM, Fardella C, et al. Case Detection, diagnosis, and
is a senior in high school and very active and athletic. treatment of patients with primary aldosteronism: An Endocrine Society
Her medications include amlodipine, 5 mg/d; KCL, 20 mEq Clinical Practice Guideline. J Clin Endocrinol Metab. 2008;93:3266-3281.

The Endocrine Society. Downloaded from press.endocrine.org by [${individualUser.displayName}] on 12 January 2017. at 12:19 For personal use only. No other uses without permission. . All rights reserved.
ENDO 2016 ADRENAL/HPA AXIS 41

6. Rossi GP, Auchus RJ, Brown M, et al. An expert consensus statement on ATP1A1 and ATP2B3 lead to aldosterone-producing adenomas and sec-
use of adrenal vein sampling for the subtyping of primary aldosteronism. ondary hypertension. Nat Genet. 2013;45(4):440-444, 444e1 e2.
Hypertension. 2014;63(1):151-160. 12. Scholl UI, Goh G, Stolting G, et al. Somatic and germline CACNA1D
7. Sze WC, Soh LM, Lau JH, et al. Diagnosing unilateral primary calcium channel mutations in aldosterone-producing adenomas and pri-
aldosteronismComparison of a clinical prediction score, computed to- mary aldosteronism. Nat Genet. 2013;45(9):1050-1054.
mography and adrenal venous sampling. Clin Endocrinol (Oxf). 2014;81(1): 13. Williams TA, Monticone S, Schack VR, et al. Somatic ATP1A1, ATP2B3,
25-30. and KCNJ5 mutations in aldosterone-producing adenomas. Hypertension.
8. Young WF, Stanson AW. What are the keys to successful adrenal venous 2014;63(1):188-195.
sampling (AVS) in patients with primary aldosteronism? Clin Endocrinol 14. Lim V, Guo Q, Grant CS, et al. Accuracy of adrenal imaging and adrenal
(Oxf). 2009;70:14-17. venous sampling in predicting surgical cure of primary aldosteronism.
9. Iwakura Y, Morimoto R, Kudo M, et al. Predictors of decreasing glomer- J Clin Endocrinol Metab. 2014;99(8):2712-2719.
ular filtration rate and prevalence of chronic kidney disease after treatment 15. Young WF Jr, Stanson AW, Thompson GB, et al. Role for adrenal venous
of primary aldosteronism: Renal outcome of 213 cases. J Clin Endocrinol sampling in primary aldosteronism. Surgery. 2004;136:1227-1235.
Metab. 2014;99(5):1593-1598. 16. Sawka AM, Young WF Jr., Thompson GB, et al. Primary aldosteronism:
10. Choi M, Scholl UI, Yue P, et al. K channel mutations in adrenal Factors associated with normalization of blood pressure after surgery. Ann
aldosterone-producing adenomas and hereditary hypertension. Science. Intern Med. 2001;135:258-261.
2011;331(6018):768-772. 17. Lim PO, Young WF, MacDonald TM. A review of the medical treatment
11. Beuschlein F, Boulkroun S, Osswald A, et al. Somatic mutations in of primary aldosteronism. J Hypertens. 2001;19:353-361.

The Endocrine Society. Downloaded from press.endocrine.org by [${individualUser.displayName}] on 12 January 2017. at 12:19 For personal use only. No other uses without permission. . All rights reserved.
42 ENDO 2016 MEET-THE-PROFESSOR CLINICAL CASE MANAGEMENT

Primary Aldosteronism and Cardiometabolic Risk:


Approach to Medical Management

M53 electrolytes (6, 7). Ensuring that patients with PA are treated to
Presented, April 1 4, 2016 minimize all the adverse cardiometabolic effects of aldosterone
is challenging.

Gail K. Adler, MD, PhD. Division of Endocrinology,


BARRIERS TO OPTIMAL PRACTICE
Diabetes and Hypertension, Brigham and Womens
To date, we lack prospective studies to identify the best treat-
Hospital, Harvard Medical School, Boston, Massachusetts
ment strategies for the medical management of PA, so medical
02115, E-mail: gadler@partners.org
management of PA is based on expert opinion. Also, an under
appreciation of the full extent of aldosterones adverse
INTRODUCTION cardiometabolic effects hinders care.
Historical Overview
Aldosterone was first isolated and shown to promote renal
sodium retention and potassium loss by Simpson, Tait, and
LEARNING OBJECTIVES
As a result of participating in this session, learners should be
Bush in 1952 (1). These and subsequent studies established
able to:
aldosterone as a key regulator of blood pressure (BP) and
Define the consequences of excess aldosterone on the
electrolyte balance. The receptor for aldosteronethe miner-
bodys cardiovascular, cerebrovascular, renovascular and
alocorticoid receptor (MR)was isolated and characterized by
metabolic systems.
Rousseau and colleagues in 1972 (2). Jerome Conn (3) de-
Identify the drugs used to treat excess aldosterone.
scribed the first case of primary aldosteronism (PA) due to an
Develop an approach for the medical management of
adrenal cortical tumor in the early 1950s. For decades, PA was
patients with PA.
thought to be rare, affecting approximately 1% of individuals
with hypertension. During the past 15 years, it has become For the diagnosis of PA and determination of whether surgical
clear that PAs contribution to hypertension was grossly therapy is appropriate please refer to the Meet the Professor
underappreciated; it is now thought that PA occurs in approxi- sessions and the 2008 Endocrine Society Clinical Practice
mately 10% of all hypertensives (4). Recently, investigators Guideline: Case Detection, Diagnosis, and Treatment of Pa-
have defined the genetic changes leading to the development of tients with Primary Aldosteronism (9).
over 50% of aldosterone-producing adenomas (5). During the
past 15-20 years it has also become clear that the MR has
STRATEGIES FOR MANAGEMENT OF
multiple cardiometabolic effects that are independent of its
PRIMARY ALDOSTERONISM
traditional effects on BP and electrolyte homeostasis (6, 7).
Clinical Features
In PA, adrenal overproduction of aldosterone leads to excess
CLINICAL SIGNIFICANCE MR activation. In the renal collecting duct, this leads to epithe-
Hypertension affects 70 million people in the United States (8). lial sodium channel (ENaC) activation and stimulation of so-
PA is a prevalent cause of hypertension, affecting up to 10% of dium retention and potassium excretion (Figure 1). The sodium
all patients with hypertension and 20% of those with resistant retention leads to volume expansion and hypertension, whereas
hypertension (4). the potassium excretion leads to hypokalemia. Thus, patients with
Approximately 50% of patients with PA have bilateral dis- PA have hypertension, elevated aldosterone, increased ratio of
ease on adrenal vein sampling and thus are treated medically, aldosterone to plasma renin activity, and often hypokalemia.
not by unilateral adrenalectomy. In addition, some patients Multiple preclinical studies have shown that excess MR
with PA are not surgical candidates or choose to avoid surgical activation leads to proteinuria, podocyte damage, glomerular
therapy. Given the large numbers of individuals with PA who damage, inflammation, vascular dysfunction, vascular damage,
need medical management, it is important for physicians to stroke, myocardial inflammation, and cardiac fibrosis, through
know how to treat this disease. mechanisms that are independent of BP and potassium homeo-
In recent years, basic and clinical studies have shown that stasis (6, 7). Further, in obese diabetic animal models, MR
aldosterones adverse effects are not limited to its traditionally blockade improves proteinuria, reduces histopathological evi-
known effects of causing hypertension and hypokalemia. dence of renal damage, restores the proinflammatory phenotype
Rather, excess exposure to aldosterone has adverse effects on of visceral adiposity toward that of a lean animal, improves
the vasculature, heart, kidney and metabolism through mecha- glucose metabolism, reduces hepatic steatosis, and reduces
nisms that are independent of aldosterones effects on BP and liver inflammation (7).

The Endocrine Society. Downloaded from press.endocrine.org by [${individualUser.displayName}] on 12 January 2017. at 12:19 For personal use only. No other uses without permission. . All rights reserved.
ENDO 2016 ADRENAL/HPA AXIS 43

Figure 1. Therapeutic interventions to reduce the adverse cardiometabolic and renal effects of excess aldosterone.

Excess MR activation leads to similar cardiometabolic in- with essential hypertension for a median of 12 years after
jury in humans with PA. Retrospective studies have examined diagnosis and treatment, revealed an increased rate of stroke,
the prevalence of metabolic disturbances and cardiovascular arrhythmias, heart failure and type 2 diabetes in PA (12). The
events in patients with PA compared with patients with essen- concept that excess MR activity contributes to cardiovascular
tial hypertension matched for the degree and duration of hyper- disease is supported by landmark clinical trials of patients with
tension (10-13). Patients with PA have an increased prevalence mild, moderate, and severe heart failure, which showed that
of atrial fibrillation (AF), nonfatal myocardial infarction, coro- adding a low dose MR blockade to standard therapy has mini-
nary artery disease, heart failure, stroke, metabolic syndrome, mal effects on BP, but markedly reduces cardiovascular mor-
and abnormal glucose metabolism (Table 1). Treatment of PA bidity and mortality (6). Excess MR activity is also implicated
by resection of an aldosterone-producing adenoma or by ad- in pathophysiology of pulmonary arterial hypertension in hu-
ministration of a MR antagonist reduces the excessive left mans. Increases in aldosterone are associated with insulin re-
ventricular hypertrophy seen in PA (14) and reduces sistance and cardiometabolic risk factors in non-PA patients,
microalbuminuria (15). However, one retrospective study, whereas treatment of PA improves insulin sensitivity [reviewed
which followed individuals with PA and matched individuals in Garg et al (16)]. Finally, recent studies in humans suggest an

TABLE 1: Cardiometabolic Complications in PA


PA, n EH, n PA, % EH, % OR (95% CI) P
Cardio- and cerebro- vascular events
Atrial fibrillation (11) 459 1289 3.9% 1.1% 5.0 (2.0-12.5)b .001
CAD (11) 459 1289 5.7% 2.8% 1.9 (1.1-3.5)b .03
Heart failure (11) 459 1289 4.1% 1.2% 2.9 (1.4-6.0)b .003
Nonfatal MI (11) 459 1289 4.4% 1.7% 2.6 (1.3-5.4)b .01
Stroke (12) 270 810 7.4% 3.5% 2.2 (1.2-4.0) .006
Metabolic disturbances
Metabolic syndrome (13) 85 381 41.1% 29.6% .05
Abnormal glucose metabolism (10) 1283a 2621a 22.4% 16.8% 1.55 (1.01-2.36)

Abbreviations: CAD, coronary artery disease; CI, confidence interval; EH, essential hypertension; MI, myocardial infarction; OR,
odds ratio.
n, total number of participants in a group; %, percentage of individuals experiencing the disease entity; OR, indicates the odds ratio of
experiencing the disease entity; P-value compares EH vs PA.
a
Meta analysis.
b
Adjusted for hypertension duration.

The Endocrine Society. Downloaded from press.endocrine.org by [${individualUser.displayName}] on 12 January 2017. at 12:19 For personal use only. No other uses without permission. . All rights reserved.
44 ENDO 2016 MEET-THE-PROFESSOR CLINICAL CASE MANAGEMENT

interaction between the renin-angiotensin-aldosterone system spironolactone and eplerenone are steroid-based drugs, finerenone
and PTH with MR blockade reducing PTH, raising the possi- is derived from dihydropyridine.
bility of alterations in bone health in PA (17). These findings The amount of spironolactone or eplerenone needed to treat
suggest that patients with PA are at increased risk of develop- PA varies from one patient to the next. The goal is to use
ing a wide range of cardiometabolic diseases in addition to sufficient MR blockade to raise serum potassium into the nor-
hypertension. mal range and reduce BP into the normal range without adverse
effects. Doses up to 400 mg spironolactone have been used in
Treatment Goals clinical studies of PA (18). However, many patients with PA
It is reasonable to propose that the three key goals for medical have had longstanding hypertension so that even with complete
treatment of PA are to: MR blockade, patients will often require the addition of other
1. Maintain BP in the normal range antihypertensive drugs to control BP. Calcium channel block-
2. Maintain normal serum potassium ers are a reasonable choice. Some physicians recommend a low
3. Reduce the development of target organ damage and dose of hydrochlorothiazide given that PA is a volume-overloaded
adverse metabolic consequences of increased state, but this medication can lower serum potassium. PA is
aldosterone. associated with a low plasma renin activity and low angiotensin II
so angiotensin-converting enzyme inhibitors and angiotensin II
Management: Dietary Sodium Restriction receptor blockers are less likely to reduce BP.
Patients with PA should follow a low-sodium diet. If dietary
sodium is low, there is less sodium to be retained in the Aldosterone Synthase Inhibitors
collecting ducts and therefore less sodium/water retention and Aldosterone synthase inhibitors are under development (18)
less potassium excretion. Thus, dietary sodium restriction low- and have been shown to reduce aldosterone levels in PA.
ers BP and increases serum potassium levels despite continued However, these inhibitors also tend to decrease cortisol. In the
elevations in aldosterone levels. Also, preclinical studies sug- future, we may find that adding a selective aldosterone syn-
gest that the adverse cardiovascular and cerebrovascular effects thase inhibitor to MR blockade is a useful approach to reduce
of excess aldosterone are minimized with a low-sodium diet. MR activation.

Management: Drugs Targeting Excess MR Activation Management: Drugs Targeting Renal Effects of Excess
MR Blockade Aldosterone: ENaC Inhibitors
The mainstay of medical treatment for PA is MR blockade Amiloride and triamterene block the actions of ENaC (18).
[reviewed in Deinum et al (18)]. Spironolactone is the first line Some physicians recommend adding amiloride, if tolerable
of treatment. It is a relatively inexpensive competitive MR doses of MR blockade are insufficient to control BP and hypo-
antagonist. Spironolactone is rapidly metabolized in the liver to kalemia. However, this treatment only blocks the effects of
generate several active metabolites, such as canrenoate. The ENaC leaving individuals exposed to the other tissue effects of
half-life of spironolactone is short (1.4 h), whereas the metabo- excess MR activation.
lites have longer half-lives (16.5 h for canrenoate) allowing for
once-daily dosing. Consuming spironolactone with food im- Management: Treatment of the Cardiometabolic
proves its absorption. Spironolactone also binds to the andro- Abnormalities Associated With PA
gen receptor and the progesterone receptor, although at much PA treatments that reduce aldosterone levels or block the MR
lower affinity than to MR. However, the interaction with these will reduce MR activity in all tissues, whereas those therapies
receptors leads to spironolactones common adverse effects of that block MRs actions in the kidney will leave the rest of the
gynecomastia, breast tenderness, decreased libido, and men- body exposed to excess MR activity. For this reason, I favor
strual irregularities. increasing the doses of spironolactone or eplerenone to achieve
Eplerenone is a newer, more selective competitive inhibitor full MR blockade as tolerated and I am less enthusiastic about
of MR with fewer adverse effects than spironolactone. It is adding ENaC inhibitors to control potassium. I have high hopes
metabolized in the liver to inactive metabolites. Due to its short for aldosterone synthase inhibitors. The adverse cardiovascular
half-life of 4-6 hours, eplerenone should be dosed twice daily. effects of aldosterone seem to be minimized when dietary
Patients unable to tolerate the adverse effects of spironolac- sodium intake is low (7), so I encourage all patients to follow a
tone can be switched to eplerenone. The BP-lowering effect low-sodium diet. Further, I think that physicians should be
of 50 mg spironolactone is roughly equivalent to 75-100 mg proactive in encouraging patients with PA to maintain a
eplerenone. Unfortunately, eplerenone is more costly than healthy lifestyle (healthy weight, good exercise and sleep, and
spironolactone. healthy diet) to reduce the risk of cardiometabolic disease. It is
There are a number of new drugs on the horizon. Finerenone possible that physicians should have a lower threshold for
is a highly potent, highly selective, newly developed MR an- initiating cardiovascular protective therapies (eg, statin or aspi-
tagonist that is not yet available in clinical practice. Although rin) in PA, although there are no studies addressing this ques-

The Endocrine Society. Downloaded from press.endocrine.org by [${individualUser.displayName}] on 12 January 2017. at 12:19 For personal use only. No other uses without permission. . All rights reserved.
ENDO 2016 ADRENAL/HPA AXIS 45

tion. It is difficult to know whether the patient has achieved Case 3


sufficient MR blockade. Serum potassium is one readout. A A 60-year-old patient with PA has a BP of 154/92 mm Hg, K
detectable plasma renin activity would be consistent with good of 3.5 mmol/L, an estimated glomerular filtration rate (eGFR)
MR blockade, but again there are no studies specifically ad- of 65 mL/min/1.73 m2, and a urinary albumin to creatinine
dressing this issue. ratio of 60 mg/g prior to the initiation of MR blockade. After
initiation of spironolactone, 50 mg daily, BP improves to
131/83 mm Hg, K increases to 4.1 mmol/L, and the urinary
MAIN CONCLUSIONS albumin to creatinine ratio improves to 10 mg/g, but the eGFR
MR blockade is the key medical treatment for PA. The goals decreases to 55 mL/min/1.73 m2. Patients primary care physi-
are to control BP and serum potassium and to prevent the cian calls you because she is concerned that renal function has
development of aldosterone-mediated cardiovascular and met- deteriorated with initiation of the spironolactone. What should
abolic complications. Patients with PA due to bilateral adrenal you do?
production of aldosterone may be treated with MR blockade A. Stop spironolactone and start amiloride
for decades so it is important to maintain adequate blockade B. Stop spironolactone and start eplerenone
and institute preventative strategies to minimize the likelihood C. Continue spironolactone at the current dose
of developing cardiometabolic complications. D. Stop spironolactone and start a calcium channel
blockade
CASES WITH QUESTIONS
Case 1 DISCUSSION OF CASES AND ANSWERS
A 50-year-old patient with new-onset hypertension is diag- Case 1
nosed with PA and refuses to consider surgery. His BP is The answer to Case 1 is C, Stop spironolactone and start
150/100 mm Hg and his K is 3.1 mmol/L off of all medica- eplerenone, 50 mg bid. Spironolactone was effective at reduc-
tions. You initiate spironolactone, 25 mg daily and up titrate ing the patients BP and raising serum potassium. However,
the dose to 50 mg daily. The patient returns complaining of given that spironolactone can block the androgen receptor,
breast pain and decreased libido. On examination his BP is decreased libido and breast pain are relatively common adverse
137/86 mm Hg and his K is 3.7 mmol/L. effects. Switching to eplerenone, 50 mg bid is roughly equiva-
You would make which of the following medication lent to 50 mg spironolactone in terms of MR blockade, but the
changes? adverse effects of decreased libido and breast pain should
A. Stop spironolactone and start amiloride 5 mg twice a resolve as eplerenones antiandrogen effects are much less than
day (bid) those of spironolactone. A is incorrect given that stopping
B. Stop spironolactone and start eplerenone, 25 mg bid spironolactone and switching to amiloride will only block
ENaC and not protect the rest of the body from excess MR
C. Stop spironolactone and start eplerenone, 50 mg bid
activation. B is incorrect given that 50 mg spironolactone is
D. Decrease spironolactone to 25 mg daily and add KCL,
roughly equivalent to 75-100 mg eplerenone, not 50 mg
20 mEq bid
eplerenone. D is incorrect as 25 mg spironolactone is not likely
E. Stop spironolactone and start calcium channel blockade
to provide sufficient MR blockade to protect the whole body
plus KCL, 20 mEq bid
from excess MR activation. E is incorrect because again cal-
cium channel blockade and KCL will not protect the whole
Case 2 body from excess MR activation.
A 75-year-old man with longstanding PA who has been treated
with MR blockade for several decades, initially spironolactone
Case 2
and now eplerenone, 100 mg bid, sees you as a new patient. He The answer is E, All of the above. Patients with PA com-
has type 2 diabetes, proteinuria, and AF. He also had a stroke several pared with individuals with essential hypertension have an
years ago. He has always been compliant with his medications. He increased risk for stroke, AF, proteinuria and type 2 diabetes,
asks you whether his medical problems are due to his PA. and coronary artery disease. The increase risk for cerebrovas-
Which of the following do you tell him has been associated cular and cardiovascular disease is not due to increased lipids
with PA? so physicians cannot rely on lipid levels to identify individuals
A. Stroke at risk.
B. Atrial fibrillation
C. Proteinuria Case 3
D. Type 2 diabetes mellitus The answer is C, Continue spironolactone at the current
E. All of the above dose. The patient is well controlled on spironolactone with a
F. C and D good BP and serum K without adverse effects associated with

The Endocrine Society. Downloaded from press.endocrine.org by [${individualUser.displayName}] on 12 January 2017. at 12:19 For personal use only. No other uses without permission. . All rights reserved.
46 ENDO 2016 MEET-THE-PROFESSOR CLINICAL CASE MANAGEMENT

androgen receptor blockade. Microalbuminuria has decreased, 7. Garg R, Adler GK. Aldosterone and the mineralocorticoid receptor: Risk
factors for cardiometabolic disorders. Curr Hypertens Rep. 2015;17(7):52.
which is a known benefit to MR blockade in PA. Hyperaldo-
8. Center for Disease Control and Prevention. High Blood Pressure. Accessed
steronism leads to glomerular hyperfiltration. With MR block- from: http://www.cdc.gov/bloodpressure/facts.htm.
ade, the hyperfiltration resolves and the underlying renal injury 9. Funder JW, Carey RM, Fardella C, et al. Case detection, diagnosis, and
is apparent. With long-term followup of patients with PA on treatment of patients with primary aldosteronism: an endocrine society clinical
practice guideline. J Clin Endocrinol Metab. 2008;93(9):3266-3281.
MR blockade, eGFR is stable compared with matched patients 10. Chen W, Li F, He C, Zhu Y, Tan W. Elevated prevalence of abnormal
with essential hypertension (15). There is no reason to switch glucose metabolism in patients with primary aldosteronism: A meta-
to eplerenone as all MR antagonists as well as surgical resec- analysis. Ir J Med Sci. 2014;183(2):283-291.
11. Savard S, Amar L, Plouin PF, Steichen O. Cardiovascular complications
tion of an aldosterone-producing adenoma have similar effects associated with primary aldosteronism: A controlled cross-sectional study.
on eGFR. Hypertension. 2013;62(2):331-336.
12. Mulatero P, Monticone S, Bertello C, et al. Long-term cardio- and cerebro-
vascular events in patients with primary aldosteronism. J Clin Endocrinol
REFERENCES Metab. 2013;98(12):4826-4833.
1. Simpson SA, Tait JF, Bush IE. Secretion of a salt-retaining hormone by 13. Fallo F, Veglio F, Bertello C, et al. Prevalence and characteristics of the
the mammalian adrenal cortex. Lancet. 1952;2(6727):226-228. metabolic syndrome in primary aldosteronism. J Clin Endocrinol Metab.
2. Rousseau G, Baxter JD, Funder JW, Edelman IS, Tomkins GM. Glucocor- 2006;91(2):454-459.
ticoid and mineralocorticoid receptors for aldosterone. J Steroid Biochem. 14. Rossi GP, Cesari M, Cuspidi C, et al. Long-term control of arterial
1972;3(2):219-227. hypertension and regression of left ventricular hypertrophy with treatment
3. Conn JW, Louis LH. Primary aldosteronism: A new clinical entity. Trans of primary aldosteronism. Hypertension. 2013;62(1):62-69.
Assoc Am Physicians. 1955;68:215-231; discussion, 231-213. 15. Fourkiotis V, Vonend O, Diederich S, et al. Effectiveness of eplerenone or
4. Piaditis G, Markou A, Papanastasiou L, Androulakis, II, Kaltsas G. Progress spironolactone treatment in preserving renal function in primary aldoste-
in aldosteronism: A review of the prevalence of primary aldosteronism in ronism. Eur J Endocrinol. 2013;168(1):75-81.
pre-hypertension and hypertension. Eur J Endocrinol. 2015;172(5):R191 16. Garg R, Adler GK. Role of mineralocorticoid receptor in insulin resis-
R203. tance. Curr Opin Endocrinol Diabetes Obes. 2012;19(3):168-175.
5. Zennaro MC, Boulkroun S, Fernandes-Rosa F. An update on novel mecha- 17. Brown JM, Williams JS, Luther JM, et al. Human interventions to charac-
nisms of primary aldosteronism. J Endocrinol. 2015;224(2):R63R77. terize novel relationships between the renin-angiotensin-aldosterone sys-
6. Parviz Y, Iqbal J, Pitt B, Adlam D, Al-Mohammad A, Zannad F. Emerging tem and parathyroid hormone. Hypertension. 2014;63(2):273-280.
cardiovascular indications of mineralocorticoid receptor antagonists. 18. Deinum J, Riksen NP, Lenders JW. Pharmacological treatment of aldoste-
Trends Endocrinol Metab. 2015;26(4):201-211. rone excess. Pharmacol Ther. 2015;154:120-133.

The Endocrine Society. Downloaded from press.endocrine.org by [${individualUser.displayName}] on 12 January 2017. at 12:19 For personal use only. No other uses without permission. . All rights reserved.
BONE, CALCIOTROPIC
HORMONES AND
VITAMIN D

The Endocrine Society. Downloaded from press.endocrine.org by [${individualUser.displayName}] on 12 January 2017. at 12:19 For personal use only. No other uses without permission. . All rights reserved.
48 ENDO 2016 MEET-THE-PROFESSOR CLINICAL CASE MANAGEMENT

Applying DXA and Other Imaging to Clinical Conundrums

M02 be used to calculate fracture risk when BMD is not available as


Presented, April 1 4, 2016 well. Bone biopsies give the most direct information about
bone microarchitecture but are invasive procedures. More re-
cently, trabecular bone score (TBS), a gray-level textural mea-
Micol S. Rothman, MD. Division of Endocrinology, surement, has become available and is being used both in
Metabolism, and Diabetes, University of Colorado School research and clinical practice (4). TBS can use lumbar spine
of Medicine, Aurora, Colorado 80045, E-mail: micol. data from standard DXA testing to noninvasively look at bone
rothman@ucdenver.edu microarchitecture. It projects a 3D structure onto a 2D plane
and uses gray-level variation in images of the lumbar spine to
INTRODUCTION calculate a score. This score has been shown to predict fracture
Historical Overview in primary osteoporosis and, more recently, in secondary osteo-
Bone mineral density (BMD) testing by dual-energy x-ray porosis. Other modalities such as quantitative computed to-
absorptiometry (DXA) has been a part of clinical practice since mography can measure volumetric BMD at the spine and hip.
the 1980s. It remains the gold standard for measuring bone Although quantitative computed tomography is thought to be a
density at the spine and hip. BMD is the bone mineral content good measure of bone quality, due to cost and radiation expo-
in g/2D projected area of bone, reported as grams/cm2. In 1994 sure, it is generally only used for research at this time.
the World Health Organization defined osteoporosis on the
basis of BMD testing by DXA and T-score. The T-score BARRIERS TO OPTIMAL PRACTICE
(patients BMD-young normal mean)/SD of young normal. In Although BMD testing is widely used, measurements are fre-
postmenopausal women and men over 50 years of age, a quently not reported in concordance with International Society
T-score of 1.0 and greater is considered normal BMD, less for Clinical Densitometry (ISCD) guidelines. Many clinicians
than 1.0 to 2.5 is defined as low bone mass or osteopenia, are not familiar with how to interpret DXA images in the face
and less or equal to than 2.5 is considered osteoporosis (1). of artifacts and other limitations. In addition, the use of T-
Osteoporosis can also be diagnosed as a low-trauma fracture scores vs Z-scores is often misunderstood. The ISCD suggests
(fall from a standing height.) In younger patients, the Z-score is that only the lowest-site T-score be used for a diagnosis (1).
used to compare patients BMD to others their age. A Z-score That is, one cannot have a hip with osteoporosis but a spine
of less than 2.0 indicates an abnormal BMD for age (1, 2). with osteopenia. Furthermore, many DXA centers do not have
a measurement of least significant change (LSC), which can
SIGNIFICANCE OF THE CLINICAL PROBLEM make it difficult to interpret change in BMD over time.
Osteoporosis is a common bone disease. Fractures affect mor- There are also ongoing controversies about the frequency of
bidity and mortality and cost the health care system billions of DXA for screening and monitoring. Recent publications have
dollars every year. There are more than 2 million fractures in shown conflicting data about when to screen and how often to
the United States each year and the risk of having a broken repeat DXA. Medicare coverage for screening in some groups
bone after age 50 years is 1 in 2 for women and 1 in 4 for men. differs from what the ISCD and National Osteoporosis Foun-
Twenty percent of women will die in the first year after hip dation (NOF) recommend. Many women who are eligible for
fracture and 60% of patients never regain their prefracture level screening DXA still do not receive it. In a study of Medicare
of independence. Osteoporosis in men is often underdiagnosed beneficiaries from 2002-2008, 48% of women had no testing
and undertreated, but a man is more likely than a woman to die performed at all. Less than 4% received four or more DXA
after hip fracture (3). studies, and thus, undertesting may be more of a problem than
Although DXA is still the most widely used clinical tool to overtesting (5).
screen for osteoporosis, it has limitations. Additional tools can
be used to evaluate fracture risk, as many patients who fracture LEARNING OBJECTIVES
have bone density scores in the osteopenic or even normal As a result of participating in this session, learners should be
range. Thus, DXA in combination with Fracture Risk Assess- able to:
ment Tool (FRAX; https://www.shef.ac.uk/FRAX/) is used as a Interpret BMD testing for men and women of all ages
fracture prediction tool for untreated patients with a diagnosis Use DXA to help guide secondary workup of low BMD
of osteopenia. FRAX takes into account BMD independent or bone loss
risks for fracture including: age, glucocorticoid treatment, cur- Be familiar with the guidelines (and controversies)
rent tobacco use, parental history of hip fracture, rheumatoid regarding DXA screening and follow-up intervals
arthritis, and alcohol use, greater than three per day. FRAX can Be familiar with the use of TBS

The Endocrine Society. Downloaded from press.endocrine.org by [${individualUser.displayName}] on 12 January 2017. at 12:19 For personal use only. No other uses without permission. . All rights reserved.
ENDO 2016 BONE, CALCIOTROPIC HORMONES, AND VITAMIN D 49

STRATEGIES FOR DIAGNOSIS, THERAPY, Low Z-scores at any age many suggest the need for a
AND/OR MANAGEMENT secondary workup of low bone density.
The NOF Clinicians Guide for 2014 recommends osteoporosis BMD testing should be repeated at a time interval when
screening for all women over age 65 years and men over 70 a change is likely to be significant and/or lead to a
years. They also advise screening for postmenopausal women treatment change. This may vary by patient factors, the
and men above 50 years with other risk factors. In addition, a treatments used and other clinical indicators. For those at
DXA is suggested after a fracture to define the extent of low highest risk, that interval may be 1-2 years. For those at
BMD. The NOF also suggests that DXA be performed at lower risk, it can be longer.
facilities using accepted quality assurance measures. The U.S. Trabecular bone score is a new tool that can help predict
Preventive Services Task Force recommends bone density fracture risk in many groups. It may be of particular use
screening for all women less than 65 years of age (Grade B) in populations where BMD has not historically been
and women less than 65 years of age whose 10-year risk of low.
fracture as calculated by FRAX is greater than a 65-year-old
Caucasian woman without risk factors (9.3%) (Grade B) (6). CASES
The use of additional vertebral imaging for those with low Case 1
BMD is also suggested in the following groups: women greater A 65-year-old woman whose femoral neck T-score lowest site
than 70 years of age and men greater than 80 years of age with is 1.2. How would you decide when to repeat her BMD?
T-scores less than 1.0, women 65-69 years of age and men A. She does not need another BMD ever.
70-79 years of age with T-scores less than 1.5, and men and B. She should have another in 2 years.
women over 50 years of age with adult low-trauma fractures, C. It might be appropriate to extend the screening interval
height loss, and glucocorticoid treatment (3). The NOF advises for this patient, but it depends on her clinical risk
follow-up testing for those on treatment in 1-2 years and every factors.
2 years thereafter, but note that clinical situations may warrant
Answer: C. In 2012, an article by Gourlay et al (7) looked at
more- or less- frequent followup.
the use of screening DXA for osteoporosis. They examined a
Least significant change (LSC) and precision should be
subgroup of women from the Study on Osteoporotic Fractures
calculated by each technician at a site. The technician measures
and looked at the time it took women with normal BMD or
a patient multiple times (15 patients 3 or 30 patients 2) to
osteopenia to transition to osteoporosis. This time varied with
determine precision and, thus, what change in BMD can be
changes in age, estrogen use and body mass index. Less than
interpreted as a true change. There is a formula on the ISCD
5% of the women with mild osteopenia (which would be the
website (ISCD.org) where patients data can be entered and
patient in this case) made the transition to osteoporosis over the
precision for each technician can be calculated. LSC is reported
15-year period. This article does not support retesting in 2
in g/cm2 for hip and spine. T-scores are not used because they
years (answer B) but nor would never testing (answer A) be
can change with alterations in the database reference popula-
appropriate without more knowledge of the clinical scenario.
tion. If a patients change in BMD does not exceed the LSC, it
Patient age and other clinical risks can help determine the
is not considered significant. However, even if changes do
interval. In addition, we should know the LCS for the techni-
exceed LSC, this does not mean they are clinically significant.
cian and machine.
A loss of 12% per year can be seen with normal aging and
We will use this case vignette to discuss the intervals for
BMD testing performed at long intervals may demonstrate loss
screening and also monitoring. We will also talk about LSC
that exceeds LSC, but is not considered pathologic.
and how it is derived.
In the setting of a low T-score (or low Z-score in a young
patient) a workup to rule out secondary causes is advised. This
Case 2
includes looking for common causes of low BMD: renal dis-
A postmenopausal patient has type 2 diabetes. She was previously
ease, liver disease, Vitamin D deficiency, hyperthyroidism,
on thiazolidinedione and currently on sodium/glucose cotran-
hyperparathyroidism, hypogonadism, or hypercalciuria, as well
sporter 2 (SGLT-2)-inhibitor therapy. You decide to obtain a TBS.
as less common causes as clinical suspicion dictates: Cushings
Which of the following is true about/approximately TBS?
syndrome, multiple myeloma, celiac disease, mastocytosis, or
A. TBS requires additional images and radiation exposure
osteogenesis imperfecta.
for the patient.
B. TBS can be used to monitor treatment with
MAIN CONCLUSIONS bisphosphonates.
BMD should be interpreted using T-scores for men 50 C. TBS is associated with an increased risk of
and postmenopausal women. Younger groups should osteoporotic fracture in postmenopausal women with
have Z-scores reported. type 2 diabetes.

The Endocrine Society. Downloaded from press.endocrine.org by [${individualUser.displayName}] on 12 January 2017. at 12:19 For personal use only. No other uses without permission. . All rights reserved.
50 ENDO 2016 MEET-THE-PROFESSOR CLINICAL CASE MANAGEMENT

D. A low TBS score is correlated with reduced fracture (thus, answer D is incorrect). In addition, teriparatide is con-
risk. traindicated in young patients (Answer A) as there can still be
concerns for open epiphyses until age 25 years. With a Vitamin
Answer: C. Trabecular bone score is a gray-level textural
D level of 35 ng/ml, further supplementation is unlikely to help
measurement that uses images already obtained by 2D images
his bone health (thus, C is incorrect). Adolescents with chronic
(4, 8). (A is wrong) A high score is indicative of better
illness may have delayed bone age and it is important to keep
microarchitecture (thus, D is wrong) In a study by Leslie et al
this in mind when thinking about DXA Z-scores. Inquiry about
in 2013 (9) postmenopausal women with diabetes had higher
pubertal status with testicular examination and measurement of
BMD but lower TBS than those without diabetes. TBS pre-
T is appropriate. This patient did have pubertal delay. With
dicted fracture in this group and may be particularly useful in
time, good nutrition (weight gain), and initiation of T, his bone
other groups where BMD may be high but fracture risk is
density markedly improved.
increased. At this time, however, changes in TBS have not
been validated to use for monitoring efficacy of therapy (thus, We will use this case vignette to talk about the use of
B is wrong). T-scores and Z-scores in young people and when intervention
This case will serve as a discussion point for talking about would be considered.
the uses and limitations of TBS.
REFERENCES
1. ISCD Official Positions. 2015. Accessed from http://iscd.org.
Case 3 2. Lewiecki EM. Bone density measurement and assessment of fracture risk.
A 20-year-old man with a history of severe Crohns disease Clin Obstet Gynecol. 2013;56(4):667-676.
and very low Z-scores: (4.1 in the lumbar spine.) He has not 3. National Osteoporosis Foundation Clinicians Guide. 2014. Accessed
fractured. He is not currently on glucocorticoids. He was from: http://nof.org.
4. Silva BC, Leslie WD, Resch H, et al. Trabecular bone score: A noninva-
known to have Vitamin D deficiency in the past, but recent 25 sive analytical method based upon the DXA image. J Bone Miner Res.
Vitamin D level was 35 ng/mL. He does supplement calcium. 2014;29(3):518-530.
What do you suggest? 5. King AB, Fiorentino DM. Medicare payment cuts for osteoporosis testing
reduced use despite tests benefit in reducing fractures. Health Aff
A. Begin teriparatide. (Millwood). 2011;30(12):2362-2370.
B. Inquire about pubertal status and check T. 6. Golob AL, Laya MB. Osteoporosis: Screening prevention and manage-
C. Add 50 000 IU of Vitamin D once a week. ment. Med Clin N Am. 2015;(99):587-606.
D. Begin alendronate based on low T-score. 7. Gourlay ML, Fine JP, Preisser JS, et al. Bone-density testing interval and
transition to osteoporosis in older women. N Engl J Med. 2012;366(3):225-
Answer: B. This question brings up the larger issue of how to 233.
8. Ulivieri FM, Silva BC, Sardanelli F, Hans D, Bilezikian JP, Caudarella R.
approach low BMD in younger patients. Often, young patients Utility of the trabecular bone score (TBS) in secondary osteoporosis.
with low BMD have not reached peak bone mass and their Endocrine. 2014;47(2):435-448.
bone density must be interpreted with caution. The use of 9. Leslie WD, Aubry-Rozier B, Lamy O, Hans D. TBS (trabecular bone
score) and diabetes-related fracture risk. J Clin Endocrinol Metab.
T-scores in men younger than 50 years of age and premeno-
2013;98(2):602-609.
pausal women is not appropriate and in young patients who are 10. Abraham A, Cohen A, Shane E. Premenopausal bone health: Osteoporosis
not fracturing we try to avoid pharmacologic therapy (10) in premenopausal women. Clin Obstet Gynecol. 2013;56(4):722-729.

The Endocrine Society. Downloaded from press.endocrine.org by [${individualUser.displayName}] on 12 January 2017. at 12:19 For personal use only. No other uses without permission. . All rights reserved.
ENDO 2016 BONE, CALCIOTROPIC HORMONES, AND VITAMIN D 51

Cancer Treatment and Bone Health

M13 retical mechanism to explain this benefit is a favorable altera-


Presented, April 1 4, 2016 tion of the bone microenvironment by bisphosphonates.

SIGNIFICANCE OF THE CLINICAL PROBLEM


Azeez Farooki, MD. Memorial Sloan Kettering Cancer
Numerous cancer therapies can induce bone loss and increase
Center, Department of Endocrinology, New York, New
fracture risk. Aromatase inhibitors are first-line adjuvant
York 10065, E-mail: farookia@mskcc.org
therapy for women with breast cancer and may be continued
for 10 years or more in some patients; this represents a huge
INTRODUCTION population of patients. Recent data from Austria has shown that
Historical Overview the increase in fracture risk resulting from these agents may
The concept of a seed and soil was first proposed by Stephen
have been underestimated. At the present time, physicians
Paget in 1889. This hypothesis proposed that the organ-
(particularly oncologists) are in need of guidance on: 1) how to
preference patterns of tumor metastasis are the product of
manage such patients (which patients to treat and for how long)
favorable interactions between metastatic tumor cells (the
and 2) how to communicate information about risks of
seed) and the microenvironment of a given organ (the
antiresorptive therapies. Endocrinologists should be aware of
soil). In 1980 Ian Hart and Isaiah Fidler (1) grafted kidney,
recent landmark studies specifically in patients with breast
ovary, and lung tissue under the skin or into the muscles of
cancer to give optimal consultative advice. Which FDA-
mice. Injected melanoma cells showed a distinct preference to
approved osteoporosis therapies to use and when to use them
develop in the grafted lung and ovary tissue but not in the renal
are important topics.
tissue, thus providing evidence of a preference for a given
Patients with thyroid cancer with metastatic bone disease are
tissue that was independent of blood supply. Bone matrix
possesses a large amount of potentially growth-stimulating usually managed in part by endocrinologists. It is important to
factors. A vicious cycle was initially proposed by the late Dr implement approved antiresorptive therapy at effective dosing
Greg Mundy in 1997 (2) in which tumor cells activate oste- schedules in such patients to reduce the risk of skeletal com-
oclastic bone resorption (via osteoblast/osteoclast-activating plications.
factors such as PTHrp), which liberates bone-derived growth
factors (IGF-I for example), which in turn promotes tumor-cell BARRIERS TO OPTIMAL PRACTICE
proliferation. Lack of familiarity with data on negative skeletal effects
The iv bisphosphonate pamidronate received approval for of adjuvant cancer therapies (such as aromatase
hypercalcemia of malignancy in 1991, for multiple myeloma in inhibitors) and important considerations when choosing
1995, and for osteolytic bone metastases from breast cancer in an agent to protect bone health.
1996. Zoledronic acid was approved for hypercalcemia of ma- Knowledge gap concerning skeletal morbidity due to
lignancy in August 2001 and for bone metastasis in solid thyroid cancer metastatic to bone.
tumors (a broad indication) in February 2002. In nonmetastatic Lack of familiarity with the clinical data supporting use
patients with cancer, the United States Food and Drug Admin- of iv bisphosphonates and denosumab in patients with
istration (FDA) approved denosumab (Prolia) in 2011 in the thyroid cancer metastatic to bone.
context of adjuvant endocrine therapy to increase bone mass
in patients who are at high risk of fracture from: 1) receiving
androgen deprivation therapy for nonmetastatic prostate cancer LEARNING OBJECTIVES
or, 2) adjuvant aromatase inhibitor (AI) therapy for breast As a result of participating in this session, learners should be
cancer. able to:
An exciting question that has been the subject of numerous Understand how adjuvant aromatase inhibitors induce
randomized trials over the years in patients with breast cancer bone loss and increase fracture risk and choose
is whether bisphosphonates can prevent seeding and/or appropriate agents to treat this problem.
propagation in bone of nascent micrometastases; is there an Appreciate new data showing a potential adjuvant
adjuvant effect of bisphosphonates? In breast cancer, this story benefit to bisphosphonate therapy in postmenopausal
has culminated with the 2015 publication of a meta-analysis of patients with breast cancer.
these trials in the Lancet (3) demonstrating, in postmenopausal Appreciate that skeletal-related events (SREs), a
patients only, a reduction in bone recurrence and a highly composite endpoint of skeletal morbidity, are common
significant improvement in cancer-specific survival. One theo- and repetitive in thyroid cancer metastatic to bone.

The Endocrine Society. Downloaded from press.endocrine.org by [${individualUser.displayName}] on 12 January 2017. at 12:19 For personal use only. No other uses without permission. . All rights reserved.
52 ENDO 2016 MEET-THE-PROFESSOR CLINICAL CASE MANAGEMENT

Treat patients with metastatic thyroid cancer to bone CASES WITH QUESTIONS
with the appropriate dosing schedule of antiresorptive Case 1
therapy to prevent SREs. A 51-year-old Caucasian woman was diagnosed with invasive
breast carcinoma, ER PR HER2-negative, and underwent
chemotherapy 1 year prior. She was not treated with radiation
STRATEGIES FOR DIAGNOSIS, THERAPY,
therapy. Her menses have not returned since chemotherapy and
AND/OR MANAGEMENT
she is now going to be treated with an adjuvant aromatase
Patients treated with adjuvant aromatase inhibitors in the setting of
inhibitor to reduce her risk of recurrence. Her T-scores are
breast cancer are at risk for rapid bone loss and increased fracture
2.0 at the lumbar spine, femoral neck, and total hip sites.
risk. Over 5 years, The Arimidex, Tamoxifen, Alone or in Com-
FRAX risk calculates at below country-specific treatment
bination (ATAC) trial showed a reduction in bone mineral density
thresholds.
(BMD) at the lumbar spine equal to 6.1% and a reduction at the
1. What is her risk for bone loss? Risk for fracture?
total hip equal to 7.2% (4). Oral and iv bisphosphonates have been
2. For what duration of therapy should this patient be
shown to protect BMD in these patients; denosumab has not only
treated?
been shown to protect BMD but also to reduce fracture risk in
3. Choice of agent?
aromatase inhibitortreated patients (5). In conclusion, clinicians
4. How would you answer the patient if she asks, How
should have a lower threshold to treat patients receiving adjuvant
will this affect my cancer?
aromatase inhibitors with antiresorptive therapy (bisphosphonates
or denosumab) with the goals of preventing bone loss and reduc-
ing fracture risk. Case 2
A meta-analysis has shown that bisphosphonates may re- A 62-year-old female with metastatic poorly differentiated thy-
duce the risk of bone recurrence and breast cancer-specific roid cancer was diagnosed 2.5 years ago. Current imaging
mortality (3). These data are often very useful when discussing reveals multiple widespread pulmonary nodules ( 1 cm) and
treatment decisions with patients. an unchanged lytic metastasis in the left T12 pedicle. Sorafenib
Thyroid cancer patients with metastatic bone disease are at was considered but not given due to Hepatitis B and increased
high risk for skeletal morbidity (captured in a composite end- liver function tests; subsequently, the hepatitis was treated and
point known as SREs) and should benefit from antiresorptive sorafenib is being considered.
therapy to protect them from SREs (pathologic fracture, the Laboratory tests: TSH 0.23 mcU/mL, Tg 916 ng/mL,
need for radiation, or surgery to bone, spinal cord compression) 25(OH)D 35 ng/mL, glomerular filtration rate is normal,
and, to potentially reduce pain. Although no randomized con- calcium and total alkaline phosphatase are normal.
trolled trials exist in thyroid cancer specifically, data both with
zoledronic acid and denosumab has confirmed the clinical utility Chronology:
of these potent antiresorptives in various solid tumors metastatic June 2013: Lytic lesion in R arm found on x-ray after she was
to bone (6, 7) in reducing he risk of SREs. Data in these patients bumped on a bus and had severe pain.
employed much more intensive dosing schedules compared with 08/2013: MRI showed a large lesion in the right proximal
that employed in osteoporosis patients (Figure 1). humerus that did not extend into the joint.

ZOMETA XGEVA RECLAST PROLIA


4 mg 120 mg 5 mg 60 mg
zoledronic zoledronic
denosumab denosumab
acid acid
Dosing interval monthly* monthly yearly q6 months
CANCER DIAGNOSES
Bone Metastases YES YES - -
Hypercalcemia of Malignancy YES YES - -
Mulple Myeloma YES - - -
Endocrine-therapy induced osteoporosis/penia** YES - - YES
Giant Cell Tumor of the Bone - YES - -
OSTEOPOROSIS
Post-Menopausal Osteoporosis in Women - - YES YES#
Prevenon of Post-Menopausal Osteoporosis in
- - YES -
Women
Senile Osteoporosis in Men - - YES -
Glucocorcoid-induced Osteoporosis - - YES -
PAGET'S DISEASE - - YES -
* New randomized data (not yet peer reviewed) demonstrates non-inferiority to every 3 month dosing
** Every 6 month dosing was studied in this context
# In paents with 1) high-risk of fracture, 2) documented failure of bisphosphonate, or 3) a CrCl < 35 mL/min

Figure 1. FDA-approved therapies and schedules for benign and malignant bone disease.

The Endocrine Society. Downloaded from press.endocrine.org by [${individualUser.displayName}] on 12 January 2017. at 12:19 For personal use only. No other uses without permission. . All rights reserved.
ENDO 2016 BONE, CALCIOTROPIC HORMONES, AND VITAMIN D 53

11/2013: Computed tomography guided biopsy consistent absorptiometry scan after taking the AI for 1 year. A large loss
with metastatic thyroid carcinoma, most likely follicular. of BMD would prompt initiation of antiresorptive therapy.
2/27/2014: Resection of the right humeral metastasis and a Among the several FDA-approved options for osteoporosis
right shoulder hemiarthroplasty. therapy, bisphosphonates or denosumab are preferable for this
5/15/2014: Total thyroidectomy for a 5.0-cm poorly differ- patient. This patient should not be treated with raloxifene
entiated thyroid carcinoma, extensive capsular invasion into therapy due to findings from the ATAC study, which demon-
fibroadipose tissue, extensive blood vessel invasion with more strated that combining a SERM (tamoxifen) with an AI was
than 8 foci of vascular invasion, including invasion of large inferior (from a cancer standpoint) to using an AI alone.
extrathyroid vessels. One benign lymph node was resected. Teriparatide use is also an uncertain area due to theoretical
7/2014: 290 mCi radioiodine. Post-therapy uptake in lesions of concerns of stimulating occult micrometastases (8) and should
frontal brain, T12 and L4, in a lytic lesion in the right mandibular be avoided at this early point in the patients cancer course.
condyle, and diffusely in lungs and in small lung nodules. Bisphosphonate and denosumab are both valid options for this
7/2014: Magnetic resonance (MR) image of spine. Diffuse patient. Although the patient is not osteoporotic, denosumab,
infiltrative lesion in anterior half of L4, and in the pedicle, 60 mg every 6 months would be an FDA-approved option
facet, and posterior vertebral body of T12. Both of these under the indication for endocrine therapyinduced bone
lesions were raidioactive iodine (RAI) avid in the post-therapy loss. Also, zoledronic acid may be given at 3-4 mg every 6
scan. A sclerotic lesion in the alar of S2S3 that was not RAI months in this context (9, 10) although 5 mg yearly may also
avid was also noted. be sufficient. The duration of treatment is debatable and in
7/2014: MR brain. Dural-based lesion measuring 2.5 part depends on the duration of aromatase inhibitor therapy;
1.8 1.9 cm above the right anterior cranial fossa adjacent to a 3-year treatment duration followed by reassessment is
right orbit. This lesion was RAI avid on the post-treatment reasonable.
scan. The patient can be educated on the recent meta-analysis of
8/2014: Hyperfractionated radiotherapy to T12 and L4. randomized trials of bisphosphonate use in nonmetastatic
9/2014: Craniotomy for removal of frontal dural-based breast cancer (18 766 subjects), which demonstrated (only for
metastatic lesion. postmenopausal patients) a significantly decreased risk of
breast cancer bone recurrence and a significant improvement in
10/2014: Stereotactic radiosurgery to right frontal resection bed.
cancer-specific mortality (3). Although a meta-analysis not
1. Is this patient at risk for malignant bone disease
definitive, it is suggestive that adjuvant bisphosphonate
causing fractures and other types of skeletal morbidity
therapy (iv or oral) may improve cancer-specific mortality to a
(SREs)?
magnitude similar to that of adjuvant chemotherapy. The pa-
2. Does this patient have an indication to be treated with
tient will have to interpret this information in light of their own
iv bisphosphonate or denosumab therapy?
beliefs and preferences about medication. Of note, a large
3. If yes, at what dose and schedule? For how long?
randomized trial with adjuvant denosumab examining breast
4. If the patient had a severe reaction to iv zoledronic
cancer recurrence is currently pending.
acid, what would you offer next?

Case 2
DISCUSSION In the oncologic literature, a composite endpoint known as
Case 1 SRE is used to capture the morbidity associated with metastatic
This patient is at high risk for bone loss due to her recent bone disease. This endpoint consists of: pathologic fractures,
chemotherapy-induced menopause and also the initiation of the need for radiation therapy to bone, surgery due to impend-
aromatase inhibitor (AI) therapy. The slope of the bone loss ing fracture, and spinal cord compression. This patient has
related to AI therapy seems to be greatest in the first 2-3 years already suffered an SRE (radiation to bone) and is at risk for
of use. The patients 10-year fracture risk via a FRAX calcu- future SREs. A retrospective analysis of 245 patients with
lation is not high enough to treat although this calculation does differentiated thyroid carcinoma metastatic to bone has shown
not account for the effect of the aromatase inhibitor (given that that 78% of the patients either presented with an SRE or
the secondary cause button in FRAX adds nothing if one developed an SRE over a median followup of 3.4 years (11).
inputs the BMD). The recent paper by Gnant et al (5) demon- Of the patients who sustained an initial SRE, 65% went on to
strated a higher-than-expected risk of fracture in women initi- sustain a second SRE. One small nonrandomized trial of iv
ating adjuvant AIs; surprisingly, on subgroup analysis, fracture pamidronate in differentiated thyroid carcinoma has shown a
risk in the placebo group seemed similar across different ages significant decrease in bone pain and a trend toward improve-
and, for normal or osteopenic BMDs, respectively. ment in performance status (12), and another nonrandomized
The two options here are to watch and wait or treat now. If study has suggested benefit from zoledronic acid in reducing
the patients strong preference was to watch and wait, a rea- SREs (13). Both zoledronic acid, 4 mg monthly and
sonable option would be to repeat the duel-energy x-ray denosumab, 120 mg monthly have been approved in solid

The Endocrine Society. Downloaded from press.endocrine.org by [${individualUser.displayName}] on 12 January 2017. at 12:19 For personal use only. No other uses without permission. . All rights reserved.
54 ENDO 2016 MEET-THE-PROFESSOR CLINICAL CASE MANAGEMENT

tumors metastatic to bone (a small number of thyroid cancer mineral density: 5-year results from the anastrozole, tamoxifen, alone or in
combination trial. J Clin Oncol. 2008;26(7):1051-1057.
patients were enrolled onto these trials) based on a delay in
5. Gnant M, Pfeiler G, Dubsky PC, et al. Adjuvant denosumab in breast
time to first SRE and decreased incidence of SREs. Recent cancer (ABCSG-18): A multicentre, randomised, double-blind, placebo-
randomized data with zoledronic acid in breast, prostate, and controlled trial. Lancet. 2015;386(9992):433-443.
myeloma patients has evaluated a less intensive dosing 6. Rosen LS, Gordon D, Tchekmedyian S, et al. Zoledronic acid versus
placebo in the treatment of skeletal metastases in patients with lung cancer
schedule of zoledronic acid, 4 mg every 3 months vs the and other solid tumors: A phase III, double-blind, randomized trialThe
standard of care, 4 mg monthly. These data have been pre- Zoledronic Acid Lung Cancer and Other Solid Tumors Study Group.
sented in abstract form and showed no difference in SREs J Clin Oncol. 2003;21(16):3150-3157.
7. Henry DH, Costa L, Goldwasser F, et al. Randomized, double-blind study
between once-monthly and once-every-3-month dosing of of denosumab versus zoledronic acid in the treatment of bone metastases
zoledronic acid. There was also no difference in bone turnover in patients with advanced cancer (excluding breast and prostate cancer) or
markers at the end of the study (14). The case patient should be multiple myeloma. J Clin Oncol. 2011;20;29(9):1125-1132.
8. Farooki A, Fornier M, Girotra M. Anabolic therapies for osteoporosis.
offered either of the FDA-approved options in solid tumors: N Engl J Med. 2007;357(23):2410-2411.
zoledronic acid, 4 mg or denosumab, 120 mg (an 9. Brufsky A, Harker WG, Beck JT, et al. Zoledronic acid inhibits adjuvant
every-3-month dosing interval seems to be noninferior to letrozole-induced bone loss in postmenopausal women with early breast
cancer. J Clin Oncol. 2007;25:829-836.
monthly dosing for zoledronic acid but there is no such data 10. Bundred NJ, Campbell ID, Davidson N, et al. Effective inhibition of
with denosumab). The duration of therapy in this setting is an aromatase inhibitor-associated bone loss by zoledronic acid in post-
open question and probably should depend on the activity of menopausal women with early breast cancer receiving adjuvant
letrozole: ZO-FAST Study results. Cancer. 2008;112:1001-1010.
the bone lesions. With long-term use, there is some risk of
11. Farooki A, Leung V, Tala H, Tuttle RM. Skeletal-related events due to
osteonecrosis of the jaw and atypical femur fracture (15). In the bone metastases from differentiated thyroid cancer. J Clin Endocrinol
setting of thyroid cancer, if there is any significant degree of Metab 2012;97(7):2433-2439.
hypoparathyroidism, these potent antiresorptive agents should 12. Vitale G, Fonderico F, Martignetti A, et al. Pamidronate improves the
quality of life and induces clinical remission of bone metastases in patients
not be used (or be used with great caution) due to risk of severe with thyroid cancer. Br J Cancer. 2001;84(12):1586.
hypocalcemia. 13. Orita Y, Sugitani I, Toda K, Manabe J, Fujimoto Y. Zoledronic acid in the
treatment of bone metastases from differentiated thyroid carcinoma. Thy-
roid. 2011;21:31-35.
REFERENCES 14. Himelstein AL, Qin R, Novotny PJ, et al. CALGB 70604 (Alliance): A
1. Hart IR, Fidler IJ. Role of organ selectivity in the determination of randomized phase III study of standard dosing vs. longer interval dosing of
metastatic patterns of B16 melanoma. Cancer Res. 1980;40:2281-2287. zoledronic acid in metastatic cancer. J Clin Oncol. 2015;33(suppl; abstr
2. Mundy GR. Mechanisms of bone metastasis. Cancer. 1997;80:1546-1556. 9501).
3. Coleman R, Powles T, Paterson A, et al. Adjuvant bisphosphonate treat- 15. Puhaindran ME, Farooki A, Steensma MR, Hameed M, Healey JH, Boland
ment in early breast cancer: Meta-analyses of individual patient data from PJ. Atypical subtrochanteric femoral fractures in patients with skeletal
randomised trials. Lancet. 2015;3;386(10001):1353-1361. malignant involvement treated with intravenous bisphosphonates. J Bone
4. Eastell R, Adams JE, Coleman RE, et al. Effect of anastrozole on bone Joint Surg Am. 2011;93:1235-1242.

The Endocrine Society. Downloaded from press.endocrine.org by [${individualUser.displayName}] on 12 January 2017. at 12:19 For personal use only. No other uses without permission. . All rights reserved.
ENDO 2016 BONE, CALCIOTROPIC HORMONES, AND VITAMIN D 55

Hyperparathyroidism Management after Unsuccessful


Parathyroid Surgery

M23 LEARNING OBJECTIVES


Presented, April 1 4, 2016 As a result of participating in this session, learners should be
able to:
Understand the definition of terms used in this clinical
Shonni J. Silverberg, MD. Division of Endocrinology, context (failed PTX, persistent PHPT, recurrent PHPT,
Department of Medicine, Columbia University College of post-PTX hyperparathyroidism).
Physicians and Surgeons, New York City, New York Use a stepwise approach to determining management.
10032, E-mail: sjs5@cumc.columbia.edu
STRATEGIES FOR DIAGNOSIS, THERAPY,
INTRODUCTION AND/OR MANAGEMENT
Historical Overview Some Definitions
Primary hyperparathyroidism (PHPT) has evolved in its pre- It is first necessary to decide whether surgery has in fact been
sentation from a highly symptomatic disease to one that pres- unsuccessful. Beyond reviewing the surgical pathology (if no
ents with asymptomatic hypercalcemia in a majority of patients parathyroid tissue was removed, it is unlikely that the patient
in the United States. Parathyroidectomy (PTX) remains the was cured), it is necessary to obtain postoperative laboratory
only option for cure of PHPT. All patients with the classical values. Persistent PHPT is said to exist when the postopera-
PHPT described by Fuller Albright as a disease of bones tive serum calcium never normalizes (7). This differs from
(osteitis fibrosa cystica), stones, and psychic groans were con- Recurrent PHPT, which describes a patient in whom serum
sidered for surgery, and symptomatic patients today are as calcium normalizes after PTX, but who becomes hypercalce-
well. Today, a more nuanced approach is taken to the decision mic again greater than 6 months after surgery. PTH levels may
about surgery in asymptomatic patients, with four sets (1990, or may not normalize in the early-postoperative period, and a
2002, 2009, 2014) of guidelines for surgery in the asymptom- normal PTH is not required in a cured patient under all circum-
atic patient published as knowledge of the disorder has in- stances. In some studies, PTH remains elevated in as many as
creased (1 4). The problem of managing the patient after 40% of patients after successful PTX (8, 9). If the calcium is
unsuccessful parathyroid surgery is an old one. The most fa- normal and PTH remains elevated, a search for secondary
mous patient with PHPT, Dr Albrights patient, Captain causes of hyperparathyroidism should be undertaken. Any
Martell, underwent seven operations before his ectopic para- identified cause(s) must be addressed before the PTH will
thyroid was found. The patients presenting for surgery are
normalize. The exception to this is seen in patients with
much changed, as the 17 patients described in Albrights study
normocalcemic PHPT. If these patients undergo parathyroidec-
had average serum calcium of 13.9 mg/dL and average gland
tomy, they cannot be considered cured until and unless their
weight of 11 g (5).
PTH levels are normal, as their calcium level was not abnormal
to begin with. Generally speaking, final determination regard-
SIGNIFICANCE OF THE CLINICAL PROBLEM ing cure vs persistent PHPT is made at the 6-month follow up
Today, improved preoperative imaging and surgical advances visit, although the diagnosis may be obvious earlier (10)
have led to a success rate for parathyroidectomy that is over
95% when surgery is done by an experienced parathyroid Confirm the Diagnosis of PHPT
surgeon. However, there are compelling data that less experi- It is important to review the original diagnosis of PHPT and
enced surgeons have a higher failure rate (6). Even in the best confirm hypercalcemia with elevated or nonsuppressed PTH
hands, cure is not always achieved, often because of unrecog- levels. Alternative causes of hypercalcemia should be consid-
nized multiglandular disease where imaging is notoriously un- ered. Familial hypocalciuric hypercalcemia should also be con-
reliable. sidered given that surgery does not cure this syndrome.

BARRIERS TO OPTIMAL PRACTICE Review the Preoperative Imaging


Physicians must fully understand the implications of Was appropriate preoperative imaging performed if a mini-
postsurgical laboratory values in patients who have mally invasive approach was used? There are real limitations to
undergone parathyroid surgery. preoperative imaging, and the surgeon must be prepared to
There are no clear guidelines for which patients should consider a bilateral approach if imaging is negative, or to look
be followed expectantly after unsuccessful surgery, and further if an abnormal gland is not found at the site identified
which should be sent for reoperation. on imaging in a minimally invasive operation.

The Endocrine Society. Downloaded from press.endocrine.org by [${individualUser.displayName}] on 12 January 2017. at 12:19 For personal use only. No other uses without permission. . All rights reserved.
56 ENDO 2016 MEET-THE-PROFESSOR CLINICAL CASE MANAGEMENT

Review the Surgery Itself Localize


If a minimally invasive approach was used, was intraoperative Not only an imaging issue. Localize the best parathyroid sur-
PTH (ioPTH) measurement available? If so, what was the geon possible. Data clearly show that failure and complication
observed PTH decline? The dictum that a 50% decline in PTH rates are inversely associated with the number of parathyroid
indicates cure is problematic in cases where the baseline PTH procedures the surgeon does (6, 14).
is very elevated, and a 50% decline is nowhere close to the
normal range. Even if an enlarged gland is removed, a second Management of Those Who Are Not Surgical
involved gland or four-gland disease may exist. Many surgeons Candidates or Who Choose Not to Have Surgery
are now using dual protocols, which call for further ioPTH Review observation vs pharmacologic intervention. Specifi-
testing if the PTH does not decrease into the normal range (11). cally, review International Consensus Conference Guidelines
When PTH does not decrease appropriately, did the surgeon for following nonoperative patients (3). Pharmacologic agents
convert to look for diseased glands in other usual locations? are not curative in PHPT. Rather, medication can address some
Ultimately, if the abnormal parathyroid gland was not found in individual sequelae of the hyperparathyroid process. Options
the usual location, was an ectopic parathyroid looked for? All for the latter include oral bisphosphonates (not U.S. Food and
this will allow the endocrinologist to make an assessment
Drug Administration [FDA] approved specifically for PHPT)
regarding the adequacy of the initial surgery, and perhaps point
to treat low bone mineral density (BMD) (15, 16). Also, the
to the potential difficulty or ease of a repeat surgical procedure.
calcimimetic, cinacalcet, is FDA approved to treat patients with
moderately severe hypercalcemia in PHPT. However, cinacalcet
Does the Patient Have Any Classical Symptoms of
does not improve BMD, nor does it lower urinary calcium excre-
PHPT?
tion in PHPT patients (17).
If so, reoperation should be considered in all patients who are
operative candidates. There is good observational data that
patients with overt symptoms (such as nephrolithiasis) have Normocalcemic PHPT
clear evidence of progressive disease when followed without It is important to remember that there are no evidence-based
surgery (12, 13). guidelines for surgery in this phenotype of the disease (3, 18).
Threshold higher for reoperation?
Does the Patient have Asymptomatic PHPT?
If so, review the latest International Consensus Conference Parathyroid cancer
Guidelines for Parathyroidectomy to determine whether the Parathyroid cancer is a special circumstance (19). If there is
patient meets surgical guidelines (3). evidence for parathyroid cancer, reoperation should be considered.

Reoperation MAIN CONCLUSIONS


If the patient is symptomatic or if the patient meets surgical In a patient whose PTH levels are elevated after surgery, it is
guidelines or if the patient and his/her physician are interested important to determine the correct diagnosis to allow appropri-
in pursuing a cure of PHPT, reoperation can be considered. In ate management.
asymptomatic patients, a higher threshold is often used before a Reoperation is an appropriate approach to the
decision to operate is made. This is appropriate given higher management of some but not all patients whose
failure rates and higher complication rates in repeat vs initial parathyroid disease is not cured by initial surgery,
parathyroid surgery. Review timing: there is rarely a reason to
depending on severity of the disease and likelihood of
consider reoperation emergently or even hastily.
cure based upon preoperative localization studies.
Watchful waiting or medical management of some
Preoperative Localization manifestations of PHPT (hypercalcemia or osteoporosis)
Although preoperative localization testing is desirable before
can be an alternative to reoperation in some patients.
an initial parathyroid surgery if a minimally invasive approach
is being considered, it is a necessity prior to a reoperation.
Noninvasive imaging should be performed first but invasive CASES AND DISCUSSION
imaging should be considered if surgery is contemplated and Case 1
noninvasive studies are unrevealing. Success of preoperative A 64-year-old man was diagnosed with PHPT (calcium, 10.4
imaging varies from center to center with greatest success at mg/dL; PTH, 142 pg/mL; 25(OH)D, 22 ng/mL) and underwent
high-volume centers, so a referral to such a center is an option parathyroidectomy. Instructions from the surgeon were to take
if testing is unrevealing. Most surgeons will require at least one calcium, 600 mg 4 times daily for a week after surgery. Labs at
positive localization study before a repeat PTX, and some his 3-week postoperative visit: calcium, 8.9 mg/dL; PTH, 112
require two concordant studies. pg/mL.

The Endocrine Society. Downloaded from press.endocrine.org by [${individualUser.displayName}] on 12 January 2017. at 12:19 For personal use only. No other uses without permission. . All rights reserved.
ENDO 2016 BONE, CALCIOTROPIC HORMONES, AND VITAMIN D 57

Questions Questions
1. What is the likely diagnosis at the postoperative visit? 3. What is her diagnosis in 2011? Persistent or recurrent
A. Persistent PHPT PHPT?
B. Recurrent PHPT 4. Which of the following diagnoses would be correct in
C. Normocalcemic PHPT 2014?
D. Secondary hyperparathyroidism A. Persistent PHPT
2. What is the best approach to management of this patient? B. Recurrent PHPT
A. Repeat labs 3 months postoperatively to see whether C. Normocalcemic PHPT
PTH normalizes. D. Primary plus secondary hyperparathyroidism
B. Resume calcium supplementation to 1 g/d between 5. What options would you consider for her after the 2015
diet and supplements and repeat labs at 3 months to studies? Repeat neck computed tomography (CT)? Mini-
see if PTH normalizes. mally invasive parathyroidectomy (MIP)? Bilateral neck
C. Treat vitamin D deficiency and repeat labs at 3
exploration? Medical approach to osteoporosis?
months to see whether PTH normalizes.
D. Treat vitamin D deficiency and resume calcium
supplementation to 1 g/d between diet and supple- Answers
ments. Repeat labs at 3 months to see whther PTH Question 3. Recurrent
normalizes.
Question 4. Recurrent PHPT and primary plus secondary hy-
Answers perparathyroidism. She is vitamin D deficient, which both
Question 1. Answer D is correct. This was not an unsuccess- further raises her PTH levels and lowers her calcium into the
ful parathyroidectomy. Calcium level is now in the lower end normal range.
of the normal range. The persistently elevated PTH level is
likely due to his vitamin D deficiency.
Question 5. Although repeat CT and MIP are incorrect an-
swers, either of the other two approaches could be considered.
Question 2. Answer D is correct. PTH will not completely She clearly meets surgical guidelines. However, in the absence
normalize in the face of significant vitamin D deficiency. It is of positive localizing studies, treating her osteoporosis medi-
important to assure calcium sufficiency after parathyroidec- cally is a safe and acceptable choice.
tomy as well.
Case 3
Case 2 A 60-year-old woman was diagnosed with PHPT (calcium,
A 55-year-old woman was diagnosed with PHPT in 2006. She 10.5 mg/dL; PTH, 64 pg/mL; 25(OH)D, 35 ng/mL) and osteo-
had PTX in early 2007 and 132 mg right upper parathyroid porosis. Sestamibi suggested a left upper-gland adenoma.
adenoma removed. Postoperative calcium and PTH were nor- Left-sided MIP performed with removal of a 400-mg
mal. In 2008, she had an anterior wall myocardial infarction. In adenoma. Normal left lower gland left in place.
2009 she had a metatarsal fracture with minimal trauma and
Intraoperative PTH: baseline, 188 pg/ml; 5 minutes, 147;
BMD showed osteoporosis. The patient was treated with
10 minutes, 133.
zoledronic acid once. Laboratory values on followup are show
Converted to bilateral PTX and right upper (70 mg) and
in Table 1.
right lower (190 mg) glands removed.
Intraoperative PTH: baseline, 242 pg/ml; 5 minutes, 146;
10 minutes, 106; 30 minutes, 68.
Table 1. Case 2 Three-week postoperative visit: calcium, 10.5 mg/dL;
Calcium, PTH, 25(OH)D, PTH, 58 pg/mL; 25(OH)D, 56 ng/mL.
Year mg/dL pg/mL ng/mL Studies
2010 10.0 53
Questions
2011 10.5 65 Spine T-score, 2.5
6. What is her diagnosis at the postoperative visit? Persis-
2014 9.5 95 22 Sestamibi negative
CT: Borderline tent or recurrent PHPT?
enlarged right lower 7. How do you interpret her intraoperative PTH levels?
gland, nl left upper A. Left side: insufficient response; right side: sufficient
gland response.
2015 10.5 68 37 Bilat venous sampling B. Left side: sufficient response; right side: sufficient
negative
response.

The Endocrine Society. Downloaded from press.endocrine.org by [${individualUser.displayName}] on 12 January 2017. at 12:19 For personal use only. No other uses without permission. . All rights reserved.
58 ENDO 2016 MEET-THE-PROFESSOR CLINICAL CASE MANAGEMENT

C. Left side: insufficient response; right side: sufficient 4. Potts JT Jr, Fradkin JE, Aurbach GD, Bilezikian JP, Raisz LG. Proceed-
ings of the NIH consensus development conference on diagnosis and
response.
management of asymptomatic primary hyperparathyroidism. J Bone Miner
D. Left side: insufficient response; right side: insuffi- Res. 1991;6:Suppl 2.
cient response. 5. Albright F, Aub JC, Bauer W. Hyperparathyroidism: A common and
E. Left side: insufficient response; right side: equivocal polymorphic condition as illustrated by seventeen proved cases from one
clinic. JAMA. 1934;102:1276-1287.
response.
6. Stavrakis AI, Ituarte PH, Ko CY, Yeh MW. Surgeon volume as a predictor
8. After allowing her to heal from surgery, what options of outcomes in inpatient and outpatient endocrine surgery. Surgery.
would you consider for her management? Repeat neck 2007;142(6):887-899; discussion 887-899.
CT? MIP? Bilateral neck exploration? Medical approach 7. Udelsman R. Approach to the patient with persistent or recurrent primary
hyperparathyroidism. J Clin Endocrinol Metab. 2011;96(10):2950-2958.
to osteoporosis?
8. Duh QY, Arnaud CD, Levin KE, Clark OH. Parathyroid hormone: Before
and after parathyroidectomy. Surgery. 1986;100(6):1021-1031.
9. Oltmann SC, Maalouf NM, Holt S. Significance of elevated parathyroid
Answers
hormone after parathyroidectomy for primary hyperparathyroidism.
Question 6. Persistent PHPT. She was never cured. Endocr Pract. 2011;17 Suppl 1:57-62.
10. Witteveen JE, Kievit J, Morreau H, Romijn JA, Hamdy NA. No recurrence
of sporadic primary hyperparathyroidism when cure is established 6
Question 7. Answer E is correct. Although the PTH level months after parathyroidectomy. Eur J Endocrinol. 2010;162(2):399-406.
decreased by well over 50% (from 242 to 68 pg/mL) and the 11. Richards ML, Thompson GB, Farley DR, Grant CS. An optimal algorithm
patient may be cured, the PTH did not completely normalize. for intraoperative parathyroid hormone monitoring. Arch Surg.
2011;146(3):280-285.
12. Silverberg SJ, Shane E, Jacobs TP, Siris E, Bilezikian JP. A 10-year
Question 8. Although there are no real guidelines for consid- prospective study of primary hyperparathyroidism with or without para-
ering repeat surgery, and this, like Question 3 in Case 1, is thyroid surgery. N Engl J Med. 1999;341(17):1249-1255.
13. Rubin MR, Bilezikian JP, McMahon DJ, et al. The natural history of
therefore a judgment call. The patient meets surgical guidelines primary hyperparathyroidism with or without parathyroid surgery after 15
(osteoporotic). However, she only has one small parathyroid years. J Clin Endocrinol Metab. 2008;93(9):3462-3470.
left. I would therefore favor medical treatment of her osteopo- 14. Abdulla AG, Ituarte PH, Harari A, Wu JX, Yeh MW. Trends in the
rosis at this time. Surgery can always be considered if her frequency and quality of parathyroid surgery: Analysis of 17,082 cases
over 10 years. Ann Surg. 2015;261(4):746-750.
clinical situation changed. 15. Chow CC, Chan WB, Li JK, et al. Oral alendronate increases bone mineral
density in postmenopausal women with primary hyperparathyroidism.
REFERENCES J Clin Endocrinol Metab. 2003;88(2):581-587.
1. Bilezikian JP, Brandi ML, Eastell R, et al. Guidelines for the management 16. Khan AA, Bilezikian JP, Kung AW, et al. Alendronate in primary hyper-
of asymptomatic primary hyperparathyroidism: Summary statement from parathyroidism: A double-blind, randomized, placebo-controlled trial.
the Fourth International Workshop. J Clin Endocrinol Metab. J Clin Endocrinol Metab. 2004;89(7):3319-3325.
2014;99(10):3561-3569. 17. Peacock M, Bolognese MA, Borofsky M, et al. Cinacalcet treatment of primary
2. Bilezikian JP, Khan AA, Potts JT Jr. Guidelines for the management of hyperparathyroidism: Biochemical and bone densitometric outcomes in a five-year
asymptomatic primary hyperparathyroidism: Summary statement from the study. J Clin Endocrinol Metab. 2009;94(12):4860-4867.
third international workshop. J Clin Endocrinol Metab. 2009;94(2):335-339. 18. Cusano NE, Silverberg SJ, Bilezikian JP. Normocalcemic primary hyper-
3. Bilezikian JP, Potts JT Jr, Fuleihan GH, et al. Summary statement from a parathyroidism. J Clin Densitom. 2013;16(1):33-39.
workshop on asymptomatic primary hyperparathyroidism: A perspective 19. Cetani F, Marcocci C. Parathyroid cancer. In: Bilezikian JP, ed. The
for the 21st century. J Bone Miner Res. 2002;17 Suppl 2:N2N11. Parathyroids, 3rd edition. Cambridge, MA: Academic Press, 2015.

The Endocrine Society. Downloaded from press.endocrine.org by [${individualUser.displayName}] on 12 January 2017. at 12:19 For personal use only. No other uses without permission. . All rights reserved.
ENDO 2016 BONE, CALCIOTROPIC HORMONES, AND VITAMIN D 59

Rickets

M34 erties. Following the clear evidence that vitamin D could be


Presented, April 3rd, 2016 used not only to prevent but also to treat rickets, public health
programs, which included food fortification and supplementa-
tion, dramatically reduced the prevalence of the disease to such
John M. Pettifor, MD, PhD. Medical Research Council/ an extent that it was suggested that rickets had been all but
Wits Developmental Pathways for Health Research Unit, eradicated from the United States.
Faculty of Health Sciences, University of the The almost complete eradication of vitamin D deficiency
Witwatersrand, Johannesburg 2193, South Africa, E-mail: rickets from the United States and a number of European
john.pettifor@wits.ac.za countries brought cases of so called vitamin D-resistant rick-
ets to the fore. Since Albrights classic description of X-linked
hypophosphatemic rickets some 15 years after McCollums
HISTORICAL PERSPECTIVE
discovery of vitamin D, a much clearer understanding of the
Rickets has been a public health problem for centuries among
pathogenesis of the various forms of rickets has developed (4).
children in the temperate zones of the world. Although it has
In particular, the central role that fibroblast growth factor 23
been reported that rickets may have occurred in a Neolithic
and other paracrine hormones play in a number of the
female, whose skeleton was discovered on a Hebridean island
phosphopenic forms of the disease has been elucidated.
off Scotland, and has been described in manuscripts from the
Greco-Roman empires around the Mediterranean Sea during
SIGNIFICANCE OF THE CLINICAL PROBLEM
the first and second centuries AD, the first detailed reports from
The prevalence of rickets in children varies from country to
Europe were described in several theses and books in the
country, because the prevalence of vitamin D deficiency de-
middle of the 17th century. At that time, the disease appeared
pends on a number of factors such as the extent and duration of
to have a predilection for children of well-to-do families, spar-
the childs skin exposure to UV radiation, the amount of UV
ing the middle classes, but also affecting the poor. It was not
radiation reaching the earth, the season of the year, the latitude
until the industrial revolution several centuries later, that large
of the country, the amount of food fortification in the normal
numbers of children of working class families were affected
diet, and the use of vitamin D supplements. Furthermore, in
(1). In Europe, the disease was so widespread that in many less affluent countries, it appears that low dietary calcium
European cities at the end of the first World War most children intakes either by themselves or in combination with poor vita-
were affected with the disease that began within the first 6 min D status may result in the development of active rickets
months of life. and bony deformities (5).
In the North-Eastern United States, the pattern of rickets was There are relatively few countries that have accurate infor-
similar to that seen in Europe, with the disease being particu- mation on the prevalence of rickets within different communi-
larly noted in children of European immigrants (2). It was ties, because the diagnosis of active rickets is dependent on
estimated that at the beginning of the 20th century up to 80% obtaining radiographic evidence, which is often not available
of children in Boston had rickets. during community surveys. There is also uncertainty as to how
The pathogenesis of the endemic form of rickets (nutritional best to screen for active rickets in communities (6). In the few
rickets) had a number of fanciful explanations until the discov- studies that have compared the prevalence of suspected rickets
ery of vitamin D in the third decade of the 20th century. It was using clinical findings, biochemical abnormalities, and radio-
considered to be due to bad air, to dampness and cold, or to graphic features, the figures have varied markedly with clinical
infection, among many other explanations. Despite a lack of features generally greatly overestimating the prevalence that is
understanding of the causes of rickets, cod liver and shark liver found when using a radiographic diagnosis.
oil had been suggested as effective treatments in the 18th There is a need to be aware of the possibility of rickets in
century, and in the next century, the benefits of sunlight were infants and children living in at-risk communities, because
expounded. At the end of the first World War, the British rickets not only can lead to hospital admission for hypocal-
Medical Research Council sent Harriet Chick to Vienna to caemic symptoms such as convulsions and apnoeic episodes,
establish the cause of the high prevalence of rickets in children but also to poor growth, bony deformities, and impaired
in that city. Using controlled trials, she was able to establish immunity resulting in increased risk of respiratory infec-
that rickets was a seasonal disease and that cod liver oil and tions, in particular.
sunlight were effective in not only preventing the disease but
also healing it (3). In 1922, McCollum named the factor in cod BARRIERS TO OPTIMAL PRACTICE
liver oil that healed rickets vitamin D after having differenti- 1. The lack of consensus among scientists as to what
ated it from vitamin A through the formers heat stable prop- constitutes vitamin D deficiency in infants and children,

The Endocrine Society. Downloaded from press.endocrine.org by [${individualUser.displayName}] on 12 January 2017. at 12:19 For personal use only. No other uses without permission. . All rights reserved.
60 ENDO 2016 MEET-THE-PROFESSOR CLINICAL CASE MANAGEMENT

although there is more consensus around 25- Dietary calcium intakes to prevent rickets:
hydroxyvitamin D (25OHD) levels associated with
For infants between 0 6 months and 6 12 months an
active vitamin D deficiency rickets.
adequate calcium intake is 200 and 260 mg/d, respec-
2. The lack of uniform policy guidelines in many
tively
countries to address the high prevalence of vitamin D
For children more than 12 months of age, a dietary
deficiency in women of child bearing age and to
calcium intake of less than 300 mg/d increases the risk of
prevent vitamin D deficiency during infancy.
rickets independent of serum 25OHD levels
3. The lack of availability of cheap and accessible forms
For children 12 months, the panel recommends the
of calcium to supplement diets of young children in
following classification of dietary calcium intake:
less affluent countries where low dietary calcium
f Sufficiency more than 500 mg/d
intakes are the norm rather than the exception.
f Insufficiency 300 500 mg/d

f Deficiency 300 mg/d


LEARNING OBJECTIVES In regard to fractures and rickets, the panel concluded
As a result of participating in this session, learners should be that children with radiographically confirmed rickets
able to: have an increased risk of fractures, but children with
Understand the recommendations made by a global subclinical vitamin D deficiency do not.
pediatric consensus group on the prevention, diagnosis,
and management of nutritional rickets.
Appreciate the difficulties associated with the screening 2. Prevention and treatment of nutrition rickets
of at-risk children in a community to determine the The panel recommended for the prevention of rickets
that all infants within the first year of life should receive
prevalence of rickets in a community.
vitamin D supplements of 400 IU/d, irrespective of the
method of feeding, and that infants and children beyond
Diagnosis, Prevention, and Management of Nutritional 12 months of age should receive 600 IU/d, made up of
Rickets dietary intake and/or sunlight exposure.
Despite effective means of preventing nutritional rickets, it In healthy children, routine 25OHD screening is not
remains a public health problem in many countries, where it recommended. Children in high-risk groups can be
has a major impact on the health of those affected, through the identified based on clinical profile and should be
consequences of hypocalcemia, bony deformities, failure to supplemented irrespective of the 25OHD levels.
thrive, motor delay, altered immune status, and dilated cardio- For the treatment of nutritional rickets, the minimum
myopathy. Furthermore, the consequences may be felt many recommended dose of vitamin D2 or D3 is 2000 IU/d.
years after the active rickets has healed, through its effects on Calcium intake should be ensured at 500 mg/d through
limb and pelvic deformities. diet and/or supplements.
The following consensus was developed by a group of With regard to the route of administration and duration
pediatric endocrinologists and nutritionists from a number of of therapy, the panel recommends that for vitamin D the
societies and groups globally to obtain an international guide- oral rather than im route should be used, that when
line on the diagnosis, prevention, and management of nutri- administered daily vitamins D2 and D3 are equally
effective in treating the disease, and that treatment
tional rickets (7). The publication contains the evidence from
should be given for a minimum of 12 weeks.
which the guidelines were derived.

3. Prevention of nutritional rickets: identification of risk


1. Defining nutritional rickets and the interplay between
factors
vitamin D status and calcium intake
Maternal vitamin D deficiency should be avoided by
Nutritional rickets is a disorder of defective chondrocyte
ensuring that all pregnant women meet the intake of 600
differentiation and mineralization of the growth plate and
IU/d.
is caused by inadequate 25OHD concentrations and/or
Complementary foods introduced no later than 26 weeks
low dietary calcium intakes in children.
of life should include sources rich in calcium.
Although the diagnosis of nutritional rickets can be
Restricting exposure to sunlight increases the risk of
suspected on the basis of history, physical examination
vitamin D deficiency and nutritional rickets.
and biochemical testing, its confirmation requires the use
Environmental factors such as latitude, season, time of
of radiographs.
day, cloud cover, and pollution affect the availability of
The panel recommends the following classification of
vitamin D status based on serum 25OHD levels: UVB, whereas personal factors such as the time spent
outdoors, skin pigmentation, skin coverage, body
Sufficiency 25OHD more than 50 nmol/L (20 ng/mL) composition, and genetics affect the dose response to
Insufficiency 25OHD 20 50 nmol/L (1220 ng/mL) UVB and circulating 25OHD. It is important to reiterate
Deficiency 25OHD less than 30 nmol/L (12 ng/mL) that there is no safe threshold of UV exposure that

The Endocrine Society. Downloaded from press.endocrine.org by [${individualUser.displayName}] on 12 January 2017. at 12:19 For personal use only. No other uses without permission. . All rights reserved.
ENDO 2016 BONE, CALCIOTROPIC HORMONES, AND VITAMIN D 61

allows for sufficient vitamin D synthesis without CONCLUSIONS


increasing skin cancer risk. Vitamin D deficiency should be considered a major global
public health priority. Nutritional rickets can have severe con-
4. Prevention of osteomalacia during pregnancy and sequences, including death. Clinical rickets represents the tip
lactation and congenital rickets of the iceberg, and its resurgence indicates widespread vita-
Maternal vitamin D deficiency increases the risk of min D deficiency and/or low dietary calcium intakes. The
elevated cord blood alkaline phosphatase, increased disease is fully preventable, thus the panel recommends the
neonatal anterior fontanelle size, neonatal hypocalcemia, eradication of rickets through implementation of international
and impaired dental enamel formation, thus maternal vitamin D supplementation and food fortification programs.
vitamin D deficiency should be prevented during
pregnancy. There is little evidence to suggest that Case 1
maternal supplementation increases birth anthropometry A 4-year-old boy is brought by his mother to a primary health
or improves short or long-term bone mass in the care clinic in rural Mpumalanga (latitude 27S), South Africa,
offspring. because she is worried by the progressive deformities develop-
There is no evidence that increased calcium intake
ing in the lower limbs for the past year. She has also noticed
(through diet and/or supplementation) in pregnancy
that the child is less active and complains of pain in the legs
above recommended nonpregnant intakes is beneficial
when walking. Examination revealed a short, well-nourished
for neonatal bone.
looking child (height for age z-score 2.1, weight for height
Vitamin D intake during lactation influences breastmilk
z-score 0.5), who had typical knock knees with an intermal-
vitamin D concentrations, thus breastfeeding infants of
mothers who receive vitamin D 2000 IU/d during leolar distance of 15 cm.
lactation have serum 25OHD levels similar to infants Question 1: Which other clinical features suggestive of rick-
who are directly supplemented with 400 IU/d. However, ets would you look for in this age group child, that would
maternal dietary calcium or vitamin D intakes do not strengthen the likelihood of the child having active rickets?
influence breastmilk calcium concentration. Question 2: Which 2 questions would you ask the mother to
Congential rickets is uncommon and is usually try and establish a cause for the rickets?
associated with mothers who have severe osteomalacia. Question 3: Is it possible to grade the severity of rickets
It is effectively prevented by ensuring maternal vitamin based on radiographs of the wrists and knees?
D sufficiency during pregnancy. Question 4: Which biochemical tests would you consider to
be essential to help in establishing the pathogenesis of the bone
5. Assessing the burden of nutritional rickets and public disease?
health strategies for prevention Question 5: How would you treat the child?
The prevalence of rickets in a community should be
determined by population-based samples, case reports Case 2
from sentinel centers, or by mandatory reporting. An 18-month-old girl, who lives with her single parent in a
Screening for rickets should be based on clinical high-rise block of apartments in the densely residential area of
features, followed by radiographic confirmation of downtown Johannesburg (latitude 26S), is not walking yet.
suspected cases. The panel made the point that screening Her mother emigrated from Zimbabwe several years ago in the
with serum 25OHD, alkaline phosphatase, or radiographs hope of getting employment in South Africa. She breastfeeds
is not indicated. her infant, who has for the past year been cared for by a
Rickets prevention involves universal vitamin D childminder in the same apartment block during the day while
supplementation of all infants between birth and 12
the mother works. Examination reveals a hypotonic toddler
months of age, and the supplementation of children at
with a relatively large skull (circumference 48.8 cm) and an
high risk over 12 months of age. Vitamin D supplements
open anterior fontanelle. She has enlarged wrists and readily
should be incorporated into primary health care
palpable costochondral junctions. Radiographs of the wrists
programs. These programs should be accompanied by
appropriate monitoring, advocacy, and publicity, reveal severe rickets. On more general enquiry, the mother
particularly in communities at high risk of vitamin D comments that there are a number of small children in the
deficiency or low dietary calcium intakes. community who have bracket legs.
Consideration should be given to the fortification of Question 1: Which factors predisposed this child to develop
appropriate foods based on dietary patterns of the target rickets?
populations. Food fortification can prevent rickets and Question 2: From a public health perspective, how would
improve vitamin D status. It should be accompanied by you prevent vitamin D deficiency rickets among the children in
relevant legislation and monitoring. this community?

The Endocrine Society. Downloaded from press.endocrine.org by [${individualUser.displayName}] on 12 January 2017. at 12:19 For personal use only. No other uses without permission. . All rights reserved.
62 ENDO 2016 MEET-THE-PROFESSOR CLINICAL CASE MANAGEMENT

Answers to case 1 likely cause is nutritional rickets (due to vitamin D or dietary


Question 1: There have been very few studies on the reliability calcium deficiency), and treatment consists of both vitamin D
of various clinical signs in predicting the presence of active and calcium supplements. Only if the child is unresponsive to
radiographic rickets in children outside the infant/toddler age therapy for nutritional rickets, should the child be investigated
range. Thacher et al (8) used data obtained from a study of for the rarer causes of calciopenic rickets.
children who had suspected rachitic leg deformities in Nigeria Question 5: The general consensus is that nutritional rickets
to determine the usefulness of various signs in predicting the should be treated with a combination of vitamin D and calcium
presence of active rickets in children older than 18 months of supplements, thus differentiating between vitamin D deficiency
age. Wrist and costochondral junction enlargement had the and dietary calcium deficiency through measuring 25OHD and
greatest sensitivity and specificity. Age less than 5 years, 1,25(OH)2D levels is unwarranted in most situations. The ac-
height for age z-score less than 2, leg pain on walking, wrist tual doses of vitamin D and/or calcium to be given are influ-
enlargement, and costochondral enlargement were each inde- enced by the age of the child and the likely compliance with
pendently predictive of active rickets. Any 3 of these signs daily therapy (11). It is generally believed that daily therapy is
accurately identified 87% of children with active rickets. better than intermittent therapy, but both are effective in heal-
Question 2: Because only a very few foods in South Africa ing rickets. Similarly, although there might be theoretical rea-
are vitamin D fortified (as is the case in many countries), sons why D3 is better than D2, both have been shown to be
children are dependent on sunlight (UVB radiation) to maintain equally effective in healing rickets in the clinical situation.
their vitamin D status. One should also consider that low
dietary calcium intakes may be in part or wholly responsible Answers to case 2
for the bone deformities. Thus, the 2 important questions that Question 1: The factors probably predisposing the child to
should be asked to help establish the pathogenesis are 1) how rickets include:
much time does the child spend playing outside the house, and Lack of exposure to UVB radiation due to living in an
how much skin is exposed to UVB radiation (short-sleeve apartment block with no access to a yard, being looked
shirts and legs and face exposed)? and 2) does the child drink after by a childminder in the same building and thus not
milk or eat dairy products at home on a regular basis? Dietary getting outside.
calcium intakes in many less affluent societies average approxi- Minimal dietary vitamin D intake, as mother continues
mately 300 mg/d, but children who are suspected of having to breastfeed, although not during the day. Few foods,
dietary calcium deficiency rickets uniformly have calcium in- besides infant formulas, are vitamin D fortified in South
takes of 200 mg/d or less (5). Africa.
Question 3: The possibility of being able to grade the sever-
ity of radiological rickets has important implications, because it The mother is Zimbabwean and thus darkly pigmented. She
provides the ability to assess the response of patients with probably has relatively poor vitamin D status due to limited
rickets to treatment over time. A similar longitudinal assess- access to sun exposure, thus limited amounts of vitamin D
ment could be made using the changes in serum alkaline cross in her breastmilk. Darker pigmented individuals are gen-
phosphatase. Thacher et al have developed a 10 point scoring erally more at risk of vitamin D deficiency, as highlighted by
system based on the radiographic severity of rickets at the the fact that it is mainly darker pigmented immigrant children
distal radius and ulna and around the knee (9). A maximum of who are at risk of vitamin D deficiency rickets in temperate
4 points is allocated to the wrist and 6 points to the knee. The climates. It is possible that the child has a relatively low-
average radiographic score correlated reasonably well with calcium intake as breastfeeding is infrequent. Usually on wean-
serum alkaline phosphatase levels (r 0.58). ing, cows milk or milk formulas are not continued, thus
Question 4: In any child with a diagnosis of rickets, the first dietary calcium intakes are low, particularly as the family pot is
steps are to establish that the child has active rickets and then corn based. Children who do not consume dairy products may
to determine whether the child has calciopenic or phosphopenic have calcium intakes of less than 300 mg/d. The panel recom-
rickets (10). Thus, once a radiograph establishes the presence mends a calcium intake for infants 0 6 months of age of 200
of active rickets, the type of rickets can be determined by mg/d, for infants 6 12 months of age of 260 mg/d, and for
measuring serum calcium, phosphorus, and PTH concentra- children older than 12 months of more than 500 mg/d.
tions. Calciopenic rickets is associated with an inability to Question 2: To reduce the prevalence of rickets in the
maintain serum calcium concentrations and thus is typically residential community in the inner city apartments, several
associated with low or low-normal calcium values and elevated approaches can be combined:
PTH levels. Serum phosphorus levels are typically low, but Vitamin D supplementation should be routine for all
these are secondary to high PTH levels. In phosphopenic rick- infants less than 12 months of age. In this community,
ets, serum calcium and PTH values are typically normal but the children are at high risk of developing rickets, and
phosphorus values low. Once calciopenic rickets is diagnosed, thus vitamin D supplementation should be continued
there is generally no need for further investigations as the most after 12 months of age.

The Endocrine Society. Downloaded from press.endocrine.org by [${individualUser.displayName}] on 12 January 2017. at 12:19 For personal use only. No other uses without permission. . All rights reserved.
ENDO 2016 BONE, CALCIOTROPIC HORMONES, AND VITAMIN D 63

As part of the mothers antenatal and postnatal care, she 3. Chick DH. Study of rickets in Vienna 1919-1922. Med Hist.
1976;20(1):41-51.
should have received vitamin D supplements, because
4. Levine BS, Kleeman CR, Felsenfeld AJ. The journey from vitamin D-
she is at high risk of having a poor vitamin D status. resistant rickets to the regulation of renal phosphate transport. Clin J Am
Because the neighborhood is a high-risk community, an Soc Nephrol. 2009;4(11):1866-1877.
education program highlighting the risks of vitamin D 5. Pettifor JM. Calcium and vitamin D metabolism in children in developing
deficiency and emphasizing the importance of using countries. Ann Nutr Metab. 2014;64(suppl 2):15-22.
6. Pettifor JM. Screening for nutritional rickets in a community. J Steroid
parks for the children during the day and the need for Biochem Mol Biol. 2016. In press.
vitamin D supplementation. 7. Munns CF, Shaw N, Kiely M, et al. Global consensus recommendations on
The importance of an adequate calcium intake should prevention and managmeent of nutritional rickets. J Clin Endocrinol
also be emphasized, because it is likely that poor Metab. 2016. In press.
8. Thacher TD, Fischer PR, Pettifor JM. The usefulness of clinical features to
vitamin D status and low dietary calcium intakes identify active rickets. Ann Trop Paediatr. 2002;22(3):229-237.
combine to exacerbate the prevalence of rickets. 9. Thacher TD, Fischer PR, Pettifor JM, Lawson JO, Manaster BJ, Reading
JC. Radiographic scoring method for the assessment of the severity of
REFERENCES nutritional rickets. J Trop Pediatr. 2000;46(3):132-139.
1. ORiordan JL, Bijvoet OL. Rickets before the discovery of vitamin D. 10. Rajah J, Thandrayen K, Pettifor JM. Clinical practice: diagnostic approach
Bonekey Rep. 2014;3:478. to the reachitic child. Eur J Pediatr. 2011;170(9):1089-1096.
2. Weick MT. A history of rickets in the United States. Am J Clin Nutr. 11. Shaw NJ, Mughal MZ. Vitamin D and child health part 1 (skeletal
1967;20(11):1234-1241. aspects). Arch Dis Child. 2013;98(5):363-367.

The Endocrine Society. Downloaded from press.endocrine.org by [${individualUser.displayName}] on 12 January 2017. at 12:19 For personal use only. No other uses without permission. . All rights reserved.
64 ENDO 2016 MEET-THE-PROFESSOR CLINICAL CASE MANAGEMENT

Osteoporosis in Premenopausal Women

M35 Lack of awareness of the clinical significance of low


Presented, April 1 4, 2016 trauma fractures in premenopausal women
Lack of awareness of diseases and medications that can
result in low peak bone mass or accelerated or premature
Elizabeth Shane, MD. Department of Medicine, bone loss in premenopausal women
Columbia University College of Physicians and Surgeons, Lack of clinical trial data on treatment of osteoporosis in
New York City, New York 10032, E-mail: premenopausal women, particularly clinical trials that
es54@columbia.edu include low trauma fracture as an outcome

INTRODUCTION LEARNING OBJECTIVES


Historical Overview As a result of participating in this session, learners should be
Osteoporosis is a common clinical problem in postmenopausal able to:
women and older men, and data from many large observational Understand the clinical significance of low BMD and
and randomized controlled clinical trials are available to guide fractures in premenopausal women
physician management of this common disease of aging. In Understand the causes of low BMD and fractures in
contrast, it is distinctly uncommon for premenopausal women premenopausal women
to present with low bone mass or fractures, and few data exist Understand the principles of management of osteoporosis
to guide physicians in the diagnosis and management of osteo- in premenopausal women with secondary osteoporosis
porosis in this age group. One of the earliest epidemiological and idiopathic osteoporosis.
studies of fractures and bone mineral density (BMD) by dual
x-ray absorptiometry (DXA) published in 1988 demonstrated
that fracture incidence is much lower in young women than
STRATEGIES FOR DIAGNOSIS, THERAPY,
older women, even when BMD is comparably low (1). Another
AND/OR MANAGEMENT
Diagnosis
study published in 1994 demonstrated that 90% of people
Postmenopausal Women
age 20-44 years who presented with low BMD and fractures
In postmenopausal women, osteoporosis is defined as a BMD
have a secondary cause of osteoporosis (2). In the 1990s,
of the spine, hip, or forearm more than 2.5 SD below the
several studies delineated features of unexplained osteoporosis
young-adult mean (T-score 2.5), with or without the pres-
in young men, namely low bone formation that was directly
ence of a major low trauma fracture (forearm, humerus, spine,
associated with low serum IGF-I. However, it was not until
hip, femur, pelvis); Low bone mass (osteopenia) refers to T
after 2000 that data on bone quality (microarchitecture, remod-
scores between 1.0 and 2.5.
eling, collagen properties, marrow adipocytes) and response to
osteoanabolic therapy began to emerge in young women with
osteoporosis (3). Premenopausal Women
In premenopausal women, the International Society for Clinical
Densitometry (ISCD) recommends that T scores not be used to
SIGNIFICANCE OF THE CLINICAL PROBLEM categorize BMD. z scores compare a young womans BMD to
The diagnosis of osteoporosis is difficult in premenopausal the mean of an age- and sex-matched reference population. The
women because the relationship between bone density and ISCD recommendations are that z scores should be used in-
incident fractures is unclear. The treatment of osteoporosis in stead of T scores to categorize BMD measurements and that a
this age group is also difficult because none of the interventional z score that is less than or equal to 2.0 should be designated
studies have a fracture outcome. Most premenopausal women low bone density or below expected for age (4). In con-
with osteoporosis (90%) have an underlying disease or medi- trast, the International Osteoporosis Foundation (IOF) recom-
cation exposure that causes bone loss or skeletal fragility. mends that the use of z scores be confined to women below age
Unexplained (idiopathic) osteoporosis (IOP) in premenopausal 30 years, who may not have reached peak bone mass, and that
women is unusual, but such patients present even greater chal- T scores should be used in women over age 30 years, in whom
lenges in diagnosis and management. peak bone mass has been reached (5).
The z score is a statistical definition that encompasses 95%
BARRIERS TO OPTIMAL PRACTICE of the normal population. Thus, by definition, the ISCD recom-
Lack of data on clinical significance of low BMD mendations mean that 2.5% of premenopausal women will
measurements in premenopausal women, particularly as have a z score less than 2.0, and thus considered to have low
they relate to short-term (510 y) fracture risk bone density or bone density that is below expected for age.

The Endocrine Society. Downloaded from press.endocrine.org by [${individualUser.displayName}] on 12 January 2017. at 12:19 For personal use only. No other uses without permission. . All rights reserved.
ENDO 2016 BONE, CALCIOTROPIC HORMONES, AND VITAMIN D 65

Use of the T-score would restrict the definition of low bone Lactation is associated with losses of 310% at the spine and
density to 0.5% of the general population, which is probably hip over the first 3 6 months with recovery after weaning over
wise because the clinical significance of isolated low BMD the next 1218 months (11). Therefore, BMD measurements
measurements is uncertain, and the extent to which isolated should not performed for at least a year after pregnancy or
low BMD measurements predict short-term (510 y) fracture lactation, during this period of transient loss and recovery.
incidence in young women is unknown. Both the ISCD and the Routine BMD screening of premenopausal or perimenopausal
IOF concur that the term osteoporosis should be avoided in women is not recommended unless there is a history of fragility
premenopausal women with isolated low BMD measurements fracture(s), or a medical conditions or medication exposure
and the term osteopenia should not be used at all. associated with low bone mass or bone loss (estrogen defi-
ciency, glucocorticoids, etc.) (Table 2).
Etiology of Low BMD in Premenopausal Women Premenopausal women with a low trauma fracture or z score
Some premenopausal women with small skeletons may seem to below 2.0 should have a thorough history, physical examination,
have low BMD, because DXA scanners measure areal BMD and laboratory evaluation to identify potential secondary causes of
(g/cm2) rather than true volumetric BMD (g/cm3). DXA cannot bone loss such as renal or liver disease, hyperthyroidism, hyper-
distinguish between small bones and less dense bones. Pre- parathyroidism, Cushings syndrome, early menopause, or other
menopausal women may have low BMD because: disorders associated with estrogen deficiency, such as anorexia,
They have genetically determined low peak bone mass celiac disease and other forms of malabsorption, idiopathic
They are thin and their skeletons are adapted to carrying hypercalciuria, or connective tissue disorders (Table 1).
lower loads
They have a syndrome known as constitutional leanness, Idiopathic Osteoporosis in Premenopausal Women
a nonpathological state of low body weight with normal Premenopausal women with no identifiable etiology after ex-
menses that is often familial (6, 7). tensive evaluation for secondary causes are said to have IOP.
IOP primarily affects Caucasians, men and women equally.
Although it is generally assumed that such women have normal
Fractures may be multiple, occurring over a 5-10-year period.
bone quality (normal trabecular and cortical volumetric BMD,
Women may present with acute vertebral compression frac-
microarchitecture, and strength), there are few data to support
tures during pregnancy or lactation, but may also present with
this. In fact, the opposite may be true. For example, in pre-
fractures temporally unrelated to pregnancy. The mean age at
menopausal women with constitutional leanness and low
diagnosis is in the mid thirties. Abnormalities of osteoblast
areal BMD by DXA, advanced imaging by high-resolution
function and decreased IGF-I have been found in most studies
peripheral quantitative computed tomography (HR-QCT) has
of men with IOP. In a recent bone biopsy study of women with
shown that volumetric BMD is also low, with disrupted
IOP, both those with fractures and those with z scores less than
microarchitecture (8). Moreover, in our study of premeno-
or equal to 2.0 but no fractures had evidence of low volu-
pausal idiopathic osteoporosis (IOP), young women with low
metric BMD of the hip and spine (by central QCT) (9), distal
BMD by DXA and no history of adult low trauma fractures, the
radius, and tibia (by HR-p QCT) (10) and iliac crest bone
positive predictive value of a low areal BMD by DXA for low
biopsies (by microCT) (12). In addition, both those with frac-
volumetric BMD by central QCT was 95% at the lumbar spine,
tures and those with low BMD had comparable
90% at the total hip, and 86% at the femoral neck (9). They also
microarchitectural disruption and reduced estimated strength
had abnormal cortical and trabecular microarchitecture at the dis-
(by finite element analysis). In addition, both groups of af-
tal radius and tibia (10). Thus, low BMD by DXA may accurately
fected women had increased marrow fat (independent of bone
reflect skeletal integrity (volumetric BMD, microarchitecture, and
volume fraction) (13), reduced bone mineralization density
strength) in premenopausal women.
distribution (by quantitative backscattered electron imaging),
Other reasons for low BMD in premenopausal women in-
and abnormal bone matrix (by Fourier transform infrared spec-
clude low peak bone mass due to reduced bone acquisition
troscopy) (14). Bone turnover was heterogeneous, but those in
during adolescence secondary to:
the lowest tertile of bone turnover had the most marked deficits
Lifestyle choices (eg, excessive alcohol, tobacco
in volumetric BMD, microarchitecture, and strength (12). Se-
exposure, low calcium intake, physical inactivity)
rum IGF-I was higher in the women in the lowest tertile of
Underlying illnesses (eg, eating disorders, estrogen
bone formation rate, suggesting that they may have IGF-I
deficiency, celiac disease, inflammatory bowel disease)
resistance at the osteoblast level (12). There were virtually no
Exposure to certain medications (eg, glucocorticoids,
differences between premenopausal women with IOP who had
Depo-Provera) may interfere with peak bone mass
fractures and those with only low BMD. However, whether
acquisition.
these women with low BMD reflect the larger population of
Such secondary causes of osteoporosis (Table 1) may also premenopausal women with low BMD is unknown. It is pos-
cause excessive bone loss after adolescence, and may be asso- sible that the lack of detectable differences may represent
ciated with abnormal bone quality, although there are few data. ascertainment bias, as women with a family history of osteo-

The Endocrine Society. Downloaded from press.endocrine.org by [${individualUser.displayName}] on 12 January 2017. at 12:19 For personal use only. No other uses without permission. . All rights reserved.
66 ENDO 2016 MEET-THE-PROFESSOR CLINICAL CASE MANAGEMENT

porosis or some other reason to suspect poor bone health may Teriparatide has been shown to prevent bone loss in
have been more likely to participate. premenopausal women on GnRH agonists for
endometriosis (16), to increase BMD in premenopausal
Management women with glucocorticoid-induced osteoporosis, and with
There are no official guidelines for management of premeno- IOP (17), and with pregnancy- and lactation-associated
pausal women with low bone mass or osteoporosis. All recom- osteoporosis (18).
mendations are thus based upon expert opinion (3, 5).
Aggressive therapy with antiosteoporosis agents may be
Lifestyle modifications should be encouraged for all women
necessary for women with glucocorticoid-induced osteoporo-
with low bone mass given that peak bone mass may improve
sis. However, the 2010 American College of Rheumatology
into the fourth decade. The following should be encouraged:
guidelines do not recommend pharmacologic therapy for pre-
Adequate calcium intake (1000 1200 mg elemental
vention and treatment of glucocorticoid-induced osteoporosis
calcium daily), preferably from dietary sources
patients under age 50 years unless they have a history of spine
Adequate vitamin D intake (400-800 IU vitamin D3
or hip fracture and have taken or will be taking at least 7.5 mg
daily) or sufficient to maintain serum 25-OHD levels
of prednisone or equivalent daily for at least 90 days.
above 20-30 ng/mL
Premenopausal women receiving chemotherapy for breast
Regular physical activity, particularly weight-bearing
cancer represent another group at risk for rapid bone loss,
exercise
primarily related to induction of premature menopause. Pro-
Cessation of smoking
spective studies demonstrate bone loss at 1 year of 4 8% in
Avoidance of excessive dieting
the spine and 2 4% at the hip in premenopausal women who
Maintenance of normal body weight
become menopausal after receiving adjuvant chemotherapy.
Avoidance of excess alcohol, caffeine and phosphorus
Intravenous bisphosphonates prevent bone loss in premeno-
containing drinks.
pausal women with chemotherapy-induced amenorrhea.
A study of 16 premenopausal women with IOP treated only Premenopausal women with osteogenesis imperfecta can be
with increased dietary calcium and physical activity revealed treated with either oral alendronate or iv pamidronate.
small but significant increases in lumbar spine and femoral In our recent pilot study of teriparatide in 21 premenopausal
neck BMD after 2 or 3 years and no new fractures (15). women with IOP, there were large and highly significant increases
When a secondary cause of osteoporosis is detected in pre- (10%) in lumbar spine BMD, with smaller but also significant
menopausal women, treatment should be targeted to that dis- increases at the femoral neck and total hip, and no change at the
ease or abnormality. Examples of specific approaches that have radius (17). Teriparatide was also associated with marked im-
been shown to lead to increases in BMD include: provements in trabecular volumetric BMD and microarchitecture
Institution of a gluten-free diet in celiac disease and cortical thickness on iliac crest bone biopsies (17). Approxi-
Parathyroidectomy in patients with primary mately 20% of the women, however, did not respond (no change
hyperparathyroidism in BMD at any site). The nonresponsive women had markedly
Discontinuation of medroxyprogesterone acetate lower bone turnover at baseline based on serum bone turnover
Oral contraceptives for women with oligo- or markers and lower bone formation rate on iliac crest bone biop-
amenorrhea, on GnRH therapy with perimenopausal sies. They also had significantly smaller and delayed increases in
bone loss serum P1NP and C-telopeptide during teriparatide therapy, and
significantly higher serum IGF-I levels, more evidence for IGF-I
Pharmacologic therapy should be avoided unless the patient
resistance at the osteoblast level (17). Teriparatide has the advan-
is losing bone or fracturing. Ferrari et al (5) recently summa-
tage of not being retained in the skeleton. However, a subset
rized effects of bisphosphonates and teriparatide in premeno-
of women in the pilot study, re-evaluated approximately 2
pausal women with osteoporosis.
years after completing teriparatide, showed partial loss of
Selective estrogen receptor modulators (SERMs) such as
bone mass at the spine, but stable BMD at the hip. Those who
raloxifene should not be used in premenopausal women;
lost significant bone mass were older (46 vs 38 y; P .05)
they block estrogen action on bone, leading to further
(19). Thus, antiresorptive therapy may be required after stop-
losses.
ping teriparatide in premenopausal women, just as it is in
Bisphosphonates carry a category C rating for safety in
postmenopausal women and men.
pregnancy as they cross the placenta and accumulate in
fetal bones in an experimental rat model. Although they
are probably safe, their long half-life in bone makes MAIN CONCLUSIONS
their use in reproductive-age women a concern. In Although most premenopausal women with osteoporosis have
premenopausal women without fractures or known a secondary disorder that negatively affect bone health, a sig-
secondary causes for fractures, bisphosphonates are nificant proportion of those presenting to tertiary care institu-
generally not indicated. tions have IOP.

The Endocrine Society. Downloaded from press.endocrine.org by [${individualUser.displayName}] on 12 January 2017. at 12:19 For personal use only. No other uses without permission. . All rights reserved.
ENDO 2016 BONE, CALCIOTROPIC HORMONES, AND VITAMIN D 67

IOP is likely to be a disorder of heterogeneous etiology, TABLE 1. Common Secondary Causes of Osteoporosis in
with some women having very low bone formation rates and Premenopausal Women
others having normal or high bone formation rates. The etiol- Cause
ogy of bone loss may vary according to bone turnover status. In Anorexia nervosa
our study, those with low bone formation have slightly GI malabsorption (eg. celiac disease, postoperative states)
HIGHER serum IGF-I concentrations and may manifest a form Vitamin D and/or calcium deficiency
of IGF-I resistance at the osteoblast level. Those with high Hyperthyroidism
bone formation rates may have a mild form of idiopathic Hyperparathyroidism
hypercalciuria. Bone biopsy is necessary to determine whether Cushings syndrome
bone formation is high or low as serum bone turnover markers Hypogonadism
were not predictive in our studies. We found, albeit in a small Hypercalciuria
sample, that women with unexplained low BMD had just as Rheumatoid arthritis and other inflammatory conditions
bad bone quality as those with low trauma fractures. Alcoholism
In general, conservative therapy is best for young women. Renal disease
Management of osteoporosis in premenopausal women with sec- Liver disease
ondary osteoporosis should focus on diagnosis and specific tar- Osteogenesis imperfecta
geted therapy of the secondary cause. Pharmacologic therapy should Marfans syndrome
be reserved for the most severely affected women, who have Homocystinuria
very low BMD (z scores 2.5), declining BMD on conser- Medications
vative therapy or major fractures. Management of IOP could Glucocorticoids
include antiresorptive therapy if appropriate to the patients age Immunosuppressants (cyclosporine)
and bone remodeling status or osteoanabolic therapy. Antiseizure medications (particularly phenobarbital and
phenytoin)
GnRH agonists (when used to suppress ovulation)
CASES
Heparin
Case 1
Cancer chemotherapy
A 31-year-old woman is referred with a history of multiple
SSRIs
vertebral fractures. At age 29 years she was involved in a
Depot medroxyprogesterone acetate
motor vehicle accident. Four months later she developed severe
Excess thyroid hormone
back pain and was found to have compression fractures of T8,
T10, L1, and L4. One year later back pain recurred and new
Abbreviation: SSRIs, selective serotonin reuptake
fractures of T11, L2, and L3 were documented, along with inhibitors.
multiple rib fractures and 2 inches of height loss.
1. What additional history would you seek?
2. What physical examination findings would you look for?
3. Which laboratory tests would you order? Question 3
4. She is anxious to have a child. What would you advise Certainly a DXA scan, CBC, chemistry panel including total
her regarding breast feeding? alkaline phosphatase, serum 25-OHD and 1,25(OH)2D, PTH,
5. Would you recommend therapy? If so, which therapy 24-hour urine calcium and free cortisol, celiac screen and other
and why? tests as appropriate.

Discussion Question 4
Question 1 In general, I suggest such women avoid breast feeding, as the
You should focus your questions on any prior history of frac- rapid decrease in BMD could exacerbate their problems and
tures in her past, any diseases (eg, celiac disease, cystic fibro- perhaps precipitate more fractures. That being said, many pa-
sis, anorexia nervosa) or medication exposures (eg, tients insist on breast feeding and in my clinical experience,
glucocorticoids, anticonvulsants) during childhood or adoles- many do not go on to have more fractures.
cence that could have negatively affected bone health. A family
history of fractures and nephrolithiasis is important. A detailed
Question 5
menstrual/reproductive history and alcohol history is key.
Management depends on whether there is any secondary cause;
if so, it should be treated directly if possible. Conservative
Question 2 management with adequate calcium, vitamin D, and weight-
Signs of Cushings Syndrome, osteogenesis imperfecta, kypho- bearing exercise. SERMs should be avoided. If she is menstru-
sis, mastocytosis. ating normally, there is probably no point in oral contraceptive

The Endocrine Society. Downloaded from press.endocrine.org by [${individualUser.displayName}] on 12 January 2017. at 12:19 For personal use only. No other uses without permission. . All rights reserved.
68 ENDO 2016 MEET-THE-PROFESSOR CLINICAL CASE MANAGEMENT

TABLE 2. Guidelines for BMD Testing in Premenopausal serum 25-OHD of 22 ng/mL (normal range, 20 50
Women mg/dL), an intact PTH of 59 pg/mL (normal range,
History of fragility fracture 14 64 mg/dL) and a 24-hour urinary calcium of 70
Diseases or conditions associated with low bone mass or bone loss mg on an adequate collection (normal range, 100 250
Premenopausal estrogen deficiency (eg, hyperprolactinemia, mg). How do you interpret these results? Would you
athletic triad, prolonged amenorrhea) measure bone turnover markers? What would you do
Eating disorders next?
Chronic obstructive pulmonary disease
Cystic fibrosis Discussion
Hyperparathyroidism Question 6
Rheumatoid arthritis You should not recommend alendronate in a premenopausal
Inflammatory bowel disease woman with low bone mass contemplating pregnancy in the near
Celiac disease future. Bisphosphonates carry a category C rating for safety in
Medications that cause bone loss pregnancy because they cross the placenta and accumulate in fetal
Glucocorticoids bones in an experimental rat model. Although they are probably
Depot progesterone safe, their long half-life in bone makes their use in reproductive-
GnRH agonists age women a concern. In premenopausal women without fractures
Aromatase inhibitors or known secondary causes for fractures, bisphosphonates are gen-
Antiepileptic drugs (phenobarbital, phenytoin, carbamazepine, erally not indicated.
valproate)
If pharmacologic therapy of osteoporosis is being considered
Question 7
Being monitored for effectiveness of pharmacologic therapy for
osteoporosis A DXA scan is not an appropriate test for evaluation of back
pain and should not have been ordered for that purpose. The
ISCD does not recommend the use of T-scores or the term
osteopenia in premenopausal women. They recommend that
pills. Bisphosphonates should be avoided at this age and if z scores be used to categorize BMD measurements and that z
imminent childbearing is being considered. Teriparatide may score that is less than or equal to 2.0 should be designated
be of some help, but there are few data on how long the effect low bone density or below expected for age. Practically
of the drug lasts, and antiresorptive therapy may be required speaking, it does not matter very much which is used, because
after completion of teriparatide to consolidate gains in BMD. at age 32 years, T- and z scores will be very concordant. This
There are no data on whether treatment of osteoporosis in patient is likely to have z scores that are the same as her T
premenopausal women reduces the risk of future fractures. scores, which means that her BMD is within the normal range
for her age.
Case 2
A 32-year-old woman is referred for evaluation and manage-
Question 8
ment of osteopenia. A DXA scan, ordered because she com-
Although her BMD is within the normal range for her age and
plained of low back pain, revealed that she had T scores of 1.9
we do not recommend bone density screening for young
at the lumbar spine, 1.8 at the femoral neck and total hip, and
women, her BMD is at the very low end of the normal range
1.1 at the 13 radius. She was told she had severe osteopenia
for her age. Given that you know this, it is appropriate to
and prescribed alendronate. However, she was planning to
evaluate her with a history and physical examination, your goal
become pregnant and was concerned that alendronate might
being to determine whether there is any condition that could be
affect the baby.
adversely affecting her skeleton. You may elect to obtain a few
6. Do you agree with the recommendation of
basic laboratory tests, as suggested by the history and physical
alendronate?
examination. The history should address detailed menstrual/
7. How would you counsel her about the results of her
reproductive history, history or family history of fractures, any
DXA scan?
diseases (eg, celiac disease, cystic fibrosis, anorexia nervosa),
8. Would you recommend any evaluation?
lifestyle choices (dieting, avoidance of calcium containing foods,
9. Her history is unremarkable excepting for the
tobacco and alcohol exposure) or medication exposures (eg,
occurrence of a femoral neck stress fracture while
glucocorticoids, anticonvulsants) during childhood or adolescence
running in a half-marathon at age 25 years. Does this
that could have negatively affected bone health.
change your thinking?
10. A biochemical evaluation reveals a serum calcium of
8.7 mg/dL (normal range, 8.6 10.2 mg/dL), serum Question 9
PO4 of 2.5 mg/dL (normal range, 2.5 4.5 mg/dL), a A femoral stress fracture is distinctly unusual and warrants a

The Endocrine Society. Downloaded from press.endocrine.org by [${individualUser.displayName}] on 12 January 2017. at 12:19 For personal use only. No other uses without permission. . All rights reserved.
ENDO 2016 BONE, CALCIOTROPIC HORMONES, AND VITAMIN D 69

more careful evaluation of her bone health. Stress fractures subjects and anorexia nervosa. Am J Physiol Endocrinol Metab. 2007;292:
E132-E137.
may be due to either repetitive overuse of a normal bone or
7. Fernandez-Garca D, Rodrguez M, Garca Aleman J, et al. Thin healthy
normal use of a weak (osteoporotic) bone. A history of a stress women have a similar low bone mass to women with anorexia nervosa.
fracture may suggest that although her BMD is within the Br J Nutr. 2009;102:709-714.
normal range for age, that her bone strength is not normal, 8. Galusca B, Zouch M, Germain N, et al. Constitutional thinness: Unusual
human phenotype of low bone quality. J Clin Endocrinol Metab. 2008;93:
especially if the amount of running does not seem excessive 110-117.
(granted, this may be a subjective assessment). 9. Cohen A, Lang TF, McMahon DJ, et al. Central QCT reveals lower
volumetric BMD and stiffness in premenopausal women with idiopathic
osteoporosis, regardless of fracture history. J Clin Endocrinol Metab.
Question 10 2012;97:4244-4252.
Although the serum results are all within the normal range, the 10. Cohen A, Liu XS, Stein EM, et al. Bone microarchitecture and stiffness in
premenopausal women with idiopathic osteoporosis. J Clin Endocrinol
urinary calcium excretion is low. The pattern (low-normal
Metab. 2009;94:4351-4360.
serum calcium, PO4, serum 25-OHD; high-normal PTH) is 11. Kovacs CS. Osteoporosis presenting in pregnancy, puerperium, and lacta-
consistent with mild, compensated secondary hyperparathy- tion. Curr Opin Endocrinol Diabetes Obes. 2014;21:468-475.
roidism. Together with a low urinary calcium, this suggests 12. Cohen A, Dempster DW, Recker RR, et al. Abnormal bone microarchitecture and
evidence of osteoblast dysfunction in premenopausal women with idio-
intestinal malabsorption. You should query the patient about pathic osteoporosis. J Clin Endocrinol Metab. 2011;96:3095-3105.
gastrointestinal (GI) symptoms, weight loss, and family history 13. Cohen A, Dempster DW, Stein EM, et al. Increased marrow adiposity in
of GI diseases such as celiac disease, and obtain celiac serolo- premenopausal women with idiopathic osteoporosis. J Clin Endocrinol
gies. Metab 2012;97:2782-2791.
14. Misof BM, Gamsjaeger S, Cohen A, et al. Bone material properties in
premenopausal women with idiopathic osteoporosis. J Bone Miner Res.
REFERENCES 2012;27:2551-2561.
1. Hui SL, Slemenda CW, Johnston CC, Jr. Age and bone mass as predictors 15. Peris P, Monegal A, Martnez MA, Moll C, Pons F, Guanabens N. Bone
of fracture in a prospective study. J Clin Invest. 1988;81:1804-1809. mineral density evolution in young premenopausal women with idiopathic
2. Khosla S, Lufkin EG, Hodgson SF, Fitzpatrick LA, Melton LJ 3rd. Epi- osteoporosis. Clin Rheumatol. 2007;26:958-961.
demiology and clinical features of osteoporosis in young individuals. 16. Finkelstein JS, Klibanski A, Arnold AL, Toth TL, Hornstein MD, Neer
Bone. 1994;15:551-555. RM. Prevention of estrogen deficiency-related bone loss with human
3. Cohen A, Shane E. Evaluation and management of the premenopausal parathyroid hormone-(1-34): A randomized controlled trial. JAMA. 1998;
woman with low BMD. Curr Osteoporos Rep. 2013;11:276-285. 280:1067-1073.
4. Schousboe JT, Shepherd JA, Bilezikian JP, Baim S. Executive summary of 17. Cohen A, Stein EM, Recker RR, et al. Teriparatide for idiopathic osteo-
the 2013 International Society for Clinical Densitometry Position Devel- porosis in premenopausal women: A pilot study. J Clin Endocrinol Metab.
opment Conference on bone densitometry. J Clin Densitom. 2013;16:455- 2013;98:1971-1981.
466. 18. Choe EY, Song JE, Park KH, et al. Effect of teriparatide on pregnancy and
5. Ferrari S, Bianchi ML, Eisman JA, et al. Osteoporosis in young adults: lactation-associated osteoporosis with multiple vertebral fractures. J Bone
Pathophysiology, diagnosis, and management. Osteoporos Int. 2012;23: Miner Metab. 2012;30:596-601.
2735-2748. 19. Cohen A, Kamanda-Kosseh M, Recker R, et al. Bone density after
6. Bossu C, Galusca B, Normand S, et al. Energy expenditure adjusted for teriparatide discontinuation in premenopausal idiopathic osteoporosis.
body composition differentiates constitutional thinness from both normal J Clin Endocrinol Metab. 2015;100:4208-4214.

The Endocrine Society. Downloaded from press.endocrine.org by [${individualUser.displayName}] on 12 January 2017. at 12:19 For personal use only. No other uses without permission. . All rights reserved.
70 ENDO 2016 MEET-THE-PROFESSOR CLINICAL CASE MANAGEMENT

Osteoporosis: Managing Patients Who Fracture


on Osteoporosis Treatment

M47 nonvertebral fractures, and approximately 1.3% for hip frac-


Presented, April 1 4, 2016 tures after 3-6 years of treatment (5, 6). Similarly, patients
receiving denosumab (DMAB) in the pivotal and extension
trials had incident vertebral fracture rates of approximately
Carolyn B. Becker, MD. Brigham and Womens Hospital, 3.5% and nonvertebral fracture rates of approximately 6.5 and
Department of Endocrinology, Diabetes and Hypertension, 3.8% at 3 and 6 years, respectively (7, 8).
Boston, Massachusetts 02115, E-mail: cbbecker@partners.org In the real world of patient care, fractures that occur while
on therapy may be much higher. For example, in one observa-
INTRODUCTION tional study, nearly 19% of postmenopausal women on oral OT
Historical Overview sustained one or more fractures over 3 years of therapy. All of
Twenty years ago, alendronate (ALN) was the first the fractures were morphometric vertebral fractures picked up
bisphosphonate shown to reduce osteoporotic fractures in on imaging, not clinical fractures (9). In another observational
postmenopausal women (1, 2). This revolutionized the era of study, women with severe osteoporosis on oral OT for an
osteoporosis research and led to rapid development of new average of 26 months had a subsequent risk of incident fracture
therapies. Since then, bisphosphonates have truly become the of 9.5% per year on continued therapy (10). This led the
backbone of osteoporosis therapy (OT). The historical devel- authors to surmise that effectiveness of oral OT may wane
opment of bisphosphonates is a fascinating example of bench over time. In a national patient registry of 38 000 patients
to bedside research and fruitful collaborations between indus- beginning ALN therapy and maintaining greater than 80%
try, academia, and clinicians. The story began with the search adherence, 9.4% sustained a major osteoporotic fracture 6
for agents that could prevent dental caries and work as better months or more after starting the therapy (11). In contrast, a
detergents (by chelating calcium and magnesium). Scientists at longitudinal survey of 5500 women with osteoporosis found
Procter and Gamble in Cincinnati noted the strong affinity of that only 6.5% sustained a single fracture and 1.3% sustained
bisphosphonates for hydroxyapatite crystal, a property that two or more new fractures while taking oral OT continuously
would come to have broad implications for skeletal metabo- for 3 years (12). These were self-reported fractures and not
lism. Collaborative work among scientists and clinicians on confirmed with radiographic imaging.
both sides of the Atlantic ocean resulted in development of
etidronate, the first bisphosphonate used in clinical medicine. BARRIERS TO OPTIMAL PRACTICE
In 1968, etidronate saved the life of a 16-month-old girl with The reality is that no currently available treatment for osteopo-
myositis ossificans progressiva, a disease causing widespread rosis can completely eliminate the risk of fracture.
muscle calcification. Today, bisphosphonates are used all over Reasons for this include low adherence and persistence with
the world as radionuclide bone scanning agents as well as medications, inadequate absorption (in the case of oral
treatments for osteoporosis, Pagets disease, hypercalcemia of bisphosphonates), failure to diagnose and treat secondary causes
malignancy, and multiple myeloma (3). of osteoporosis, intervention that is too little or too late to reverse
existing defects in bone mass and/or bone quality, and the inability
SIGNIFICANCE OF THE CLINICAL PROBLEM of our treatments to address nonskeletal risk factors such as frailty
We have made great progress in diagnosing, preventing, and and falls (13, 14). Additional barriers to optimal treatment include
treating osteoporosis, yet fragility fractures remain a major the high cost of many pharmaceutical agents as well as tier-based,
clinical and public health issue. At best, our current interven- sequential reimbursement policies that discourage individualized
tions reduce the relative risk of fragility fractures by 30 70% approaches to care. In fairness, very few clinical trials comparing
at the spine, 1525% at nonvertebral sites, and 40% at the hip. active therapies or combinations of therapies exist. This means
OT reduces the risk of multiple fractures by 80 90% compared that sequential treatment failures rather than clinical trial evi-
with placebo (4). dence may direct much of our decision making.
Poor compliance with self-administered medications (see Given that fragility fractures occur among a significant pro-
Barriers to Optimal Practice) can greatly lower the effective- portion of patients being treated for osteoporosis, it is surpris-
ness of OT. But even in trials in which drugs are administered ing how few guidelines exist to help clinicians manage these
by healthcare personnel, fractures still occur. For example, in patients. In 2012, a working group of the International Osteo-
the pivotal and extension trials for iv zoledronic acid (ZA), porosis Foundation tried to fill this gap by publishing prag-
new fracture rates in the active treatment arms were approxi- matic advice for clinicians managing patients with osteoporosis
mately 3% for morphometric vertebral fractures, 8% for treatment failure. Alhough based largely on limited data and

The Endocrine Society. Downloaded from press.endocrine.org by [${individualUser.displayName}] on 12 January 2017. at 12:19 For personal use only. No other uses without permission. . All rights reserved.
ENDO 2016 BONE, CALCIOTROPIC HORMONES, AND VITAMIN D 71

expert opinion, this position paper represents a starting point If the therapy is self administered, is the patient actually
for the clinical care of patients who fracture on OT (4). taking the drug?
Is she taking it properly (particularly critical with oral
bisphosphonates)?
LEARNING OBJECTIVES
Is she missing 20% of the doses (the threshold for
As a result of participating in this session, learners should be able to:
adequate adherence)?
Decide when a new fragility fracture truly represents
Is she getting sufficient calcium and vitamin D?
treatment failure.
Decide which workup needs to be performed. Studies have shown that persistence with OT is often dismal.
Decide rationally which therapeutic changes, if any, Among 451 000 new oral bisphosphonate users, only 63%
should be made. persisted with therapy at 1 year, 46% at 2 years, and less than
25% at 5 years (16). Other studies have shown that up to half
of patients starting on oral bisphosphonates stop therapy within
STRATEGIES FOR MANAGEMENT OF
3-6 months (17). A recent pharmacy review of 127 000 patients
PATIENTS WHO FRACTURE ON OT
starting on oral OT or teriparatide (TPT) found that persistence
Definitions
Fragility Fracture with medication at 1 year was only 40% for monthly
A fragility fracture occurs spontaneously or after falling from a risedronate (RIS), 39.9% for weekly ALN, 22.7% for
standing height. Certain fracture sites are excluded from this raloxifene, and 34% for TPT (18).
definition because they are not typically associated with osteo- Once compliance issues have been addressed, the second
porosis and/or are less responsive to OT. These include frac- step is to determine whether the patient has primary or
tures of the hands, skull, digits, feet, and ankles (4). secondary osteoporosis. This involves a careful history and
Treatment failure for patients on OT is defined as (4): physical examination as well as additional laboratory testing.
Two or more incident fragility fractures. Listed in Table 1 are diseases to be considered in the differen-
tial diagnosis when a patient fails to respond to OT.
One incident fracture and an elevated baseline serum collagen
Type 1 C-telopeptide (CTX) that does not decrease by 25%
while on antiresorptive therapy (or, a serum N-terminal Testing for Patients Fracturing on OT
propeptide of Type 1 collagen [P1NP] that does not increase Complete blood count, serum calcium, phosphorus,
creatinine, 25(OH)D, alkaline phosphatase, celiac
by 25% while on anabolic therapy).
antibodies, TSH, liver enzymes, and 24-hour urine for
One incident fracture and a clinically significant decrease in
calcium, sodium, creatinine, and free cortisol.
bone mineral density (BMD) at the spine (5%), or at the hip Additional testing could include intact PTH, serum and
(4%). urine protein electrophoresis, and sex steroid levels.
Persistently elevated bone turnover markers (BTM) Serum or urine markers of bone turnover (eg, serum
accompanied by a clinically significant decrease in BMD. CTX or P1NP for antiresorptive and anabolic therapy,
respectively) may be helpful, particularly when
compared with baseline values prior to therapy. Note
Compliance
that BTMs must be collected under proper conditions
Compliance is the degree to which a patient correctly follows
and using the same assay to be fully comparable (13).
medical advice; for medications, this includes timing, dosage, Finally, a repeat measurement of BMD using dual-energy
and frequency. x-ray absorptiometry (DXA) can be supportive evidence
showing inadequate response to medication therapy. A
Adherence clinically significant decline in BMD coupled with a new
Adherence and compliance are often used interchangeably fracture strongly supports the need to rule out compliance
issues and secondary causes of osteoporosis. It also
though adherence may imply that the patient is in agreement
suggests that a change in therapy is warranted.
with the treatment plan or intervention.

Changing Therapy: A Rational Approach


Persistence Table 2 shows the seven major osteoporosis therapies available in
Persistence is time from initiation of therapy to discontinuation the United States. The numbers in the boxes represent statistically
of that therapy. significant relative risk reductions in spine, nonvertebral, and hip
In general, 6 12 months of uninterrupted OT are necessary fractures based on meta-analyses or randomized clinical trials.
before concluding that an OT is not effective or that the patient The most effective agents for reducing spinal fractures are
is a nonresponder (4, 10, 11). ZA, DMAB, and teriparatide. For nonvertebral fractures, re-
sults are much more modest and not all agents are effective.
Clinical Assessment of the Patient Who Fractures on OT Finally, ALN, RIS, ZA, and DMAB are the sole agents to have
The first step is to review compliance (15). significantly reduced hip fractures in clinical trials.

The Endocrine Society. Downloaded from press.endocrine.org by [${individualUser.displayName}] on 12 January 2017. at 12:19 For personal use only. No other uses without permission. . All rights reserved.
72 ENDO 2016 MEET-THE-PROFESSOR CLINICAL CASE MANAGEMENT

TABLE 1. Secondary Causes for Osteoporosis/Fractures*


Inherited Nutritional Endocrine Medications Other
Osteogenesis imperfecta Malabsorption- Hypogonadism Glucocorticoids Multiple myeloma
Homocysteinuria Alcoholism Hyperthyroidism Anticonvulsants Rheumatoid arthritis
Marfans syndrome Calcium deficit Cushings Heparin Mastocytosis
Vitamin D deficit Anorexia Excess thyroid Immobilization
Hypercalciuria PHPT GnRH agonists
Hepatic disease HAART
Aromatase inhibitors
Proton pump inhibitors

Abbreviations: PHPT, primary hyperparathyroidism; GnRH, gonadotropin-releasing hormone; HAART, highly active anti-retroviral
therapy.
*Adapted from Lewiecki EM 2003 (15).

TABLE 2. Statistically Significant Relative Risk


Reductions in Fractures (N/S not significant) greater patient satisfaction (21) when switched to DMAB
compared with those switched to oral ibandronate. Simi-
Spine Non-vertebral Hip
Drug Fractures Fxs Fractures larly, in another study, patients switched to DMAB had
significant decreases in BTMs and significant increases in
Alendronate 0.55 0.84 0.61
BMD at both spine and hip after 12 months, compared with
Risedronate 0.63 0.80 0.74
those continuing on ALN (23).
Zoledronic Acid 0.30 0.75 0.59
Bottom line: DMAB leads to greater suppression of BTMs
Ibandronate 0.48 N/S N/S
as well as generalized improvements in BMD within 1 year of
Raloxifene 0.70 N/S N/S
switching from chronic ALN.
Denosumab 0.32 0.80 0.60
Teriparatide 0.35 0.62 N/S
Switching from ALN or RIS to TPT
After 12 months, patients previously on RIS showed signifi-
cantly greater gains in BMD at both spine and hip on TPT
Only 11 studies, mainly open label, short duration (2 y), compared with those previously on ALN (24).
and with small sample sizes, have been performed to evaluate
outcomes when transitioning from oral bisphosphonates (ALN Switching from ALN or Raloxifene (RLX) to TPT vs
or RIS) to other agents (19). None of these studies include adding TPT to oral drug
fracture outcomes. A brief synopsis of results from some of the Addition of TPT to ALN increased BMD at spine and hip
larger trials is outlined below. significantly more than stopping ALN and starting TPT by
itself. With RLX, either adding or switching to TPT led to
equivalent gains in BMD at the spine and hip (25).
Switching from ALN to another oral bisphosphonate
In general, switching poorly compliant patients from ALN to
another oral bisphosphonate, such as RIS (20) or ibandronate Other combination therapies (26)
Adding TPT to ALN or ZA increases BMD at the hip beyond
(21) does not lead to any measurable improvements.
PTH alone; at the spine, however, there is no additive effect of
combination therapy.
Switching from ALN to ZA vs continuing ALN DMAB plus TPT shows additive effects at BMD of spine
Women on long-term ALN who switched over to ZA had and hip compared with either drug alone.
significantly lower BTMs at 3 months compared with those In the opinion of one expert: for patients previously
continuing on ALN. At 1 year, however, there were no differ- treated with bisphosphonates who suffer hip fractures or who
ences in BMD or bone biopsies (22). have very low or declining BMD at the hip, strong consider-
Bottom line: Oral ALN may not maximally suppress bone ation should be given to starting TPT and continuing a potent
resorption, even after 4 years of therapy. ZA leads to greater antiresorptive therapy (possibly switching to ZA or DMAB) to
suppression of BTMs in the short term but no change in BMD. improve hip BMD and strength quickly. (27)

Switching from ALN to DMAB MAIN CONCLUSIONS


Patients with poor adherence to ALN had significant de- A significant proportion of patients on OT will continue
creases in BTMs, significant increases in BMD (21, 21) and to fracture.

The Endocrine Society. Downloaded from press.endocrine.org by [${individualUser.displayName}] on 12 January 2017. at 12:19 For personal use only. No other uses without permission. . All rights reserved.
ENDO 2016 BONE, CALCIOTROPIC HORMONES, AND VITAMIN D 73

No currently available therapy can completely C. Check additional laboratory tests to rule out secondary
eliminate the risk of fracture. causes of osteoporosis.
D. Stop ALN and switch to iv ZA.
A rational approach to patients who fracture after at least 6 12
mo of OT is outlined below:
Assess for compliance and absorption (particularly if Case 2
A healthy 70-year-old woman returns for followup of general-
patient is on an oral bisphosphonate).
ized osteoporosis. She has been on ALN, 70 mg by mouth
Assess for secondary causes of osteoporosis via history,
weekly with excellent compliance for 4 years as well as ad-
physical examination, and selected laboratory testing.
equate calcium and vitamin D. A repeat DXA shows stable
Assess BTMs (serum CTX for antiresorptive and serum
values at all sites with T-scores of 2.8 at the spine (8%
P1NP for anabolic therapy), preferably with baseline
increase since baseline), 3.0 at the FN, and 2.8 at the TH
measurements for comparison.
(4.5% increase since baseline). After sustaining a wrist fracture
Assess BMD via DXA if at least 12 mo have passed
at age 65, she has done well. Two months after seeing you, she
since the previous scan.
trips over a branch while hiking in the woods, fracturing her
If there is evidence of poor compliance, malabsorption,
left humerus. Laboratory work-up reveals normal CBC, Ca,
inadequate BTM response, or significant decline in BMD
Phos, creatinine, 25OHD, serum and urine protein electropho-
in addition to a single fragility fracture, then therapy
resis, alk phos, and urinary calcium excretion. A fasting serum
should be changed. This also applies if patients sustain
CTX comes back in the lowest 25th percentile for premeno-
two or more incident fractures on a particular therapy.
pausal women.
In general, when changing OT: What is the most appropriate management?
Replace a weaker antiresorptive agent (eg, raloxifene) A. Stop ALN and begin teriparatide.
with a more potent antiresorptive agent (eg, B. Stop ALN and begin ZA.
bisphosphonate) (4). C. Stop ALN and begin DMA.B
Substituting one oral bisphosphonate for another may not D. Continue ALN for now.
improve efficacy against subsequent fractures.
Replace an oral drug with an injected drug (eg, ZA or Case 3
DMAB) (4). A 72-year-old woman with osteoporosis comes to see you with
Replace a strong antiresorptive (eg, ZA or DMAB) with acute midback pain after lifting a bag of mulch. She had taken
an anabolic (TPT) (4). ALN 70 mg weekly for 4 years followed by DMAB 60 mg sc
In cases in which hip fracture is a particular risk, consider every 6 months for the past 18 months. On examination, her
combination therapy with ALN, ZA, or DMAB TPT midback is tender and she has lost 1 in in height. Imaging
(27). Focus on modifiable risk factors such as smoking shows an acute biconcave vertebral compression fracture at
cessation, correction of vitamin D deficiency, and T10 and mild-to-moderate anterior wedge compression frac-
reducing or stopping unnecessary medications (eg, tures at 2 other vertebrae that seem to be old. Her last DXA
proton pump inhibitors). from 6 months ago revealed T-scores of 2.5 at L1-L4, 2.8 at
the left FN, and 2.5 at the left TH. All were significantly
CASES WITH QUESTIONS improved compared with the previous scan. An extensive lab-
Case 1 oratory work up including 24 hour urine free cortisol, serum
A 66-year-old woman is diagnosed with osteoporosis on a tryptase, serum and urine protein electrophoresis, and PTH
screening DXA. Her T-scores are 3.5, 3.2, and 3.0 at comes back negative. Fasting serum CTX is suppressed.
L1L4, left femoral neck (FN), and left total hip (TH), respec- What is the most appropriate management?
tively. She is 15 years postmenopausal and on no medications. A. Stop DMAB and switch to ZA.
She has no toxic habits, exercises regularly, and has never B. Stop DMAB and switch to teriparatide.
fractured. She gets adequate calcium in her diet and takes C. Switch to ZA and add teriparatide.
vitamin D, 800 IU daily. Physical examination is negative for D. Continue DMAB and add teriparatide.
kyphosis or loss of height. Her body mass index is 21 kg/m2.
CBC, serum calcium, phosphate, creatinine, 25(OH)D, and DISCUSSION
hepatic panel are normal. You start her on ALN, 70 mg by Case 1
mouth weekly. Four months later she slips on ice, falls, and There are two teaching points from this case. The first point
breaks her left wrist. is that 6 12 months of treatment are required before one can
What is the most appropriate management? call OT a failure or deem the patient a nonresponder. Four
A. Assess her compliance with the current regimen. months is too short a time and there is no need for further
B. Check a serum or urine marker of bone resorption. laboratory testing or switching to a new therapy. Bone

The Endocrine Society. Downloaded from press.endocrine.org by [${individualUser.displayName}] on 12 January 2017. at 12:19 For personal use only. No other uses without permission. . All rights reserved.
74 ENDO 2016 MEET-THE-PROFESSOR CLINICAL CASE MANAGEMENT

turnover markers would be useful in comparison with base- acid for treatment of postmenopausal osteoporosis. N Engl J Med.
2007;356:1809-1822.
line levels but BTMs were not obtained at baseline. The 6. Black DM, Reid IR, Boonen S, et al. The effect of 3 versus 6 years of
second point is that a review of compliance with oral zoledronic acid treatment of osteoporosis: A randomized extension to
bisphosphonates is always warranted given that poor com- the HORIZON-Pivotal Fracture Trial (PFT). J Bone Miner Res.
2012;27:243-254.
pliance is so common with self-administered OT. 7. Cummings SR, San Martin J, McClung MR, et al. Denosumab for preven-
tion of fractures in postmenopausal women with osteoporosis. N Engl
Case 2 J Med. 2009;361(8):756-765.
8. Papapoulos S, Lippuner K, Roux C, et al. The effect of 8 or 5 years of
This case explores what to do when a patient on potent denosumab treatment in postmenopausal women with osteoporosis:
antiresorptive therapy fractures, despite improvements in BMD Results from the FREEDOM extension study. Osteoporos Int. 2015;26
and suppression of BTMs. According to the International Os- (12):2773-2783.
9. Cairoli E, Eller-Vainicher C, Ulivieri FM, et al. Factors associated with
teoporosis Foundation Position Paper (4), she does not warrant bisphosphonate treatment failure in postmenopausal women with primary
a change in therapy unless the wrist fracture occurred after osteoporosis. Osteoporos Int. 2014;25:1401-1410.
6 12 months on OT (which it did not). However, in the real 10. Adami S, Isaia G, Luisetto G, et al. Osteoporosis treatment and fracture
incidence: The ICARO longitudinal study. Osteoporos Int. 2008;19:
world, this would clearly be a disappointing result for both 1219-1223.
patient and physician. Teriparatide significantly reduces 11. Abrahamsen B, Rubin KH, Eiken PA, et al. Characteristics of patients
nonvertebral fractures but its effect on hip fractures is not who suffer major osteoporotic fractures despite adhering to alendronate
treatment: A national prescription registry study. Osteoporos Int.
established. Intravenous ZA will not increase BMD in patients 2013;24:321-328.
already responding to ALN. In contrast, DMAB has been 12. Dez-Perez A, Adachi JD, Adami S, et al. Risk factors for treatment failure
shown to significantly increase BMD at the spine and hip, even with antiosteoporosis medication: The global longitudinal study of osteo-
porosis in women (GLOW). J Bone Miner Res. 2014;29:260-267.
in patients coming off of long-term ALN. We have no fracture 13. Lewiecki EM, Watts NB. Assessing response to osteoporosis therapy.
outcomes to support this change but extension studies suggest Osteoporos Int. 2008;19:1363-1368.
that DMAB may be superior to bisphosphonates in the long 14. Carey JJ. What is a failure of bisphosphonate therapy for osteoporosis?
Cleve Clin J Med. 2005;72:1033-1039.
term. After 8 years of DMAB, BMD continues to increase at 15. Lewiecki EM. Nonresponders to osteoporosis therapy. J Clin Densitom.
both spine and hip while TH BMD plateaus at a lower level 2003;6:307-314.
after only 4.5 years on ZA (28). 16. Burden AM, Paterson JM, Solomon DH, et al. Bisphosphonate prescrib-
ing, persistence and cumulative exposure in Ontario, Canada. Osteoporos
Int. 2012;23:1075-1082.
Case 3 17. Sheehy O, et al. Differences in persistence among different weekly oral
bisphosphonate medications. Osteoporos Int. 2009;20(8):1369-1376.
This case illustrates the patient who clearly meets criteria for a 18. Carbonell-Abella C, Pages-Castella A, Javaid MK, et al. Early (1-year)
change in therapy. She has fractured on two potent antiresorp- discontinuation of different anti-osteoporosis medications compared: A
tive agents, ALN and DMAB. Compliance is not an issue and population-based cohort study. Calcif Tissue Int. 2015;97(6):535-541.
19. Eiken P, Vestergaard P. Treatment of osteoporosis after alendronate or
her workup for secondary causes is negative. She needs ana- risedronate [published online October 5, 2015]. Osteoporos Int. doi:
bolic therapy. However, she has osteoporosis at the hip sites 10.1007/s00198-015-3334-4.
and thus is at high risk for a hip fracture. Teriparatide has not 20. Roux C, Hofbauer LC, Ho PR, et al. Denosumab compared with
risedronate in postmenopausal women suboptimally adherent to alendro-
shown efficacy against hip fractures and can lead to declines in nate therapy: Efficacy and safety results from a randomized open-label
BMD at the femoral neck. In this particular situation, combi- study. Bone. 2014;58:48-54.
nation therapy with DMAB plus TPT should be strongly con- 21. Recknor C, Czerwinski E, Bone HG, et al. Denosumab compared with
ibandronate in postmenopausal women previously treated with bis-
sidered. In clinical trials, this combination seems to lead to the phosphonate therapy: A randomized open-label trial. Obstet Gynecol.
most robust improvements in BMD at both spine and hip 2013;121:1291-1299.
although no fracture data are available. Changing to ZA would 22. McClung M, Recker R, Miller P, et al. Intravenous zoledronic acid 5 mg in
the treatment of postmenopausal women with low bone density previously
not offer any added value in this situation. treated with alendronate. Bone. 2007;41:122-128.
23. Kendler DL, Roux C, Benhamou CL, et al. Effects of denosumab on bone
mineral density and bone turnover in postmenopausal women transitioning
REFERENCES from alendronate therapy. J Bone Miner Res. 2010;25:72-81.
1. Liberman UA, Weiss SR, Broll J, et al. Effect of oral alendronate on bone 24. Miller PD, Delmas PD, Lindsay R, et al. Early responsiveness of women
mineral density and the incidence of fractures in postmenopausal osteopo- with osteoporosis to teriparatide after therapy with alendronate or
rosis. N Engl J Med. 1995;333:1437-1443. risedronate. J Clin Endocr Metab. 2008;93:3785-3793.
2. Black DM, Cummings SR, Karpf DB, et al. Randomised trial of effect of 25. Cosman F, Wermers RA, Recknor C, et al. Effects of teriparatide in
alendronate on risk of fracture in women with existing vertebral fractures. postmenopausal women with osteoporosis on prior alendronate or
Lancet. 1996;348(9041):1535-1541. raloxifene: Differences between stopping and continuing the antiresorptive
3. Francis MD, Valent DJ. Historical perspectives on the clinical dev- agent. J Clin Endocr Metab. 2009;94:3772-3780.
elopment of bisphosphonates in the treatment of bone diseases. 26. Cosman F. Combination therapy for osteoporosis: A reappraisal. Bonekey
J Musculoskelet Neuronal Interact. 2007;7(1):2-8. Rep. 2014;3:518.
4. Diez-Perez A, Adachi JD, Agnusdei D, et al. Treatment failure in osteo- 27. Cosman F. Anabolic and antiresorptive therapy for osteoporosis: Combi-
porosis. Osteoporos Int. 2012;23:2769-2774. nation and sequential approaches. Curr Osteoporos Rep. 2014;12:385-395.
5. Black DM, Delmas PD, Eastell R, et al. Once-yearly zoledronic 28. Reid IR. Denosumab after 8 years. Osteoporos Int. 2015;26:2759-2761.

The Endocrine Society. Downloaded from press.endocrine.org by [${individualUser.displayName}] on 12 January 2017. at 12:19 For personal use only. No other uses without permission. . All rights reserved.
ENDO 2016 BONE, CALCIOTROPIC HORMONES, AND VITAMIN D 75

Osteoporosis Drug Holidays: Data and Opinions

M51 United States, four bisphosphonates are approved for the


Presented, April 1 4, 2016 prevention and/or treatment of postmenopausal osteoporo-
sis, osteoporosis in men, and osteoporosis due to long-term
glucocorticoid therapy. Alendronate and risedronate can be
Nelson B. Watts, MD. Osteoporosis and Bone Health given orally daily, weekly, or monthly; zoledronate is a
Services, Mercy Health, Cincinnati, Ohio 45242, E-mail: once-yearly iv infusion and ibandronate can be given either
nelson.watts@hotmail.com orally (monthly) or intravenously (every third month).
Bisphosphonates bind strongly to hydroxyapatite crystals
INTRODUCTION in bone (zoledronate most strongly, risedronate least
Historical Overview strongly; alendronate and ibandronate have intermediate af-
Osteoporosis has been clinically recognized for centuries. In the finity for bone). Drug that does not bind with bone is rapidly
1940s, Fuller Albright made the connection between osteoporosis excreted by the kidneys. In the process of bone resorption,
and estrogen loss with menopause, and for many years, estrogen bisphosphonates are released from the bone and enter the
was the treatment of choice. Bisphosphonates are compounds with osteoclasts, causing loss of resorptive function and acceler-
several different commercial and medical uses. First available in ating apoptosis. There is a rapid and substantial decrease in
the mid 1990s, they have been the most widely used agents for bone turnover markers with a maximum effect in 3 6
prevention and treatment of osteoporosis. months and modest increases in bone density in the first few
years of treatment (35%) that then plateau. With continued
treatment, the new steady state is maintained for 10 years (2,
SIGNIFICANCE OF THE CLINICAL PROBLEM 3) and probably longer.
Osteoporosis is a common disorder, one of several risk Bisphosphonates have proven efficacy for prevention of
factors for fracture risk. Fractures cause significant disabil- bone loss due to aging, estrogen deficiency, and glucocorti-
ity and even death. Bisphosphonates are unique in that they coid use. Three of the four (alendronate, risedronate, and
accumulate in bone so after a loading dose it might be zoledronate) have been shown in placebo-controlled trials to
possible to discontinue treatment, at least temporarily, with prevent fractures of the spine, hip, and other nonvertebral
maintenance of a therapeutic benefit. Also, there are con- sites (4 6). Because of this broad-spectrum antifracture
cerns that long-term treatment (5 y) increases the risk of efficacy, bisphosphonates have been the agents of choice for
osteonecrosis of the jaw (ONJ) and atypical femur fractures, most patients with osteoporosis.
although the risk remains small. Orally administered bisphosphonates are usually well toler-
ated but may irritate the esophagus and should not be used by
BARRIERS TO OPTIMAL PRACTICE patients who cannot remain upright, who have active upper
There is confusion and concerns among providers and pa- gastrointestinal symptoms, or have delayed esophageal empty-
tients regarding safety of bisphosphonate treatment and op- ing. Up to one third of patients receiving their first iv dose of
timal duration. zoledronate or monthly oral dose of ibandronate or risedronate
experience one or more symptoms of acute-phase reactions
(fever, muscle aches, etc.) (79), but these are usually mild,
LEARNING OBJECTIVES resolve within a few days, and rarely recur with repeated
As a result of participating in this session, learners should be
administration. Hypocalcemia may occur but is usually mild
able to:
and not clinically recognized (10). Iritis has been described
Describe the benefits and risks of bisphosphonate
with bisphosphonates (more with iv than oral) but is rare (1
treatment.
per 1000).
Describe the rationale for drug holidays from
Although there is no evidence of renal toxicity from oral
bisphosphonates.
bisphosphonates, the only route of elimination is by the
Review data regarding when a drug holiday should be
kidneys, so they should be used with caution if at all by
offered and opinion as to how long a holiday should
patients with reduced kidney function. Renal toxicity may
last.
occur with rapid iv administration of zoledronate; its use is
contraindicated for patients with creatinine clearance less
STRATEGIES FOR DIAGNOSIS, THERAPY, than 35 mL/min.
AND/OR MANAGEMENT Bisphosphonates used in the United States were approved
Bisphosphonates have been widely used for treatment of based on placebo-controlled trials of 3 4 years duration. Sev-
osteoporosis and other metabolic bone diseases (1). In the eral of these studies have been extended, with two alendronate

The Endocrine Society. Downloaded from press.endocrine.org by [${individualUser.displayName}] on 12 January 2017. at 12:19 For personal use only. No other uses without permission. . All rights reserved.
76 ENDO 2016 MEET-THE-PROFESSOR CLINICAL CASE MANAGEMENT

cohorts followed for 10 years (2, 3), risedronate cohorts fol- for osteoporosis or Pagets disease. Retrospective review of
lowed for 4 (11) and 7 years (12), and zoledronate cohorts records from the HORIZON trial with iv zoledronate for
followed for 6 years (12). No new safety concerns have been osteoporosis, two cases of ONJ were identified: one who
found in these studies. Although there have been some con- received zoledronate and one who received placebo. It is
cerns about possible oversuppression of bone turnover, iliac estimated that there have been more than 200 million pre-
crest biopsies after up to 10 years of treatment have not shown scriptions in the United States for oral bisphosphonates and
oversuppression (2). more than 6 million patients treated with iv bisphosphonates
Since their approval and widespread use, a number of for cancer worldwide. Epidemiologic data suggest an inci-
potential safety concerns have been identified but with no dence of ONJ in oral bisphosphonate users ranging from
clear cause-and-effect relationship but at least some evi- 1:10 000 to 1:250 000 (14).
dence of increased risk (albeit rare) with therapy of 5 years A causal link between bisphosphonate use and ONJ is likely
or longer. These include musculoskeletal pain, atrial fibril- but has not been conclusively established. ONJ has also been
lation, esophageal cancer, ONJ, and atypical femur frac- seen in patients receiving high-dose denosumab for treatment
tures. The latter two have been subject to close scrutiny and of advanced cancer spread to bone (15). Possible mechanisms
widespread discussion in the medical literature and in the include oversuppression of bone turnover (failure of osteoclasts
lay press. to remove diseased necrotic bone) or interference with clear-
ance of microfilms.
ONJ The American Society for Bone and Mineral Research
In 2003, a letter to the editor reported ONJ in 36 patients with Task Force performed a comprehensive review (13) and the
advanced cancer who were being treated with high doses of iv American Dental Association published guidelines in 2011
bisphosphonates (approximately 10 times higher than the doses (16). Patients receiving bisphosphonates should be informed
used to treat osteoporosis). Subsequent reports included pa- that there are risks of any treatment, including a low risk of
tients receiving lower doses of bisphosphonates for treatment ONJ with long-term therapy. Regular dental visits and main-
of osteoporosis, but, to date, well over 90% of reported cases tenance of good oral hygiene including routine dental clean-
have been in patients with cancer. This subject was extensively ing and needed restorative procedures are important. Ideally,
addressed by a Task Force of the American Society for Bone patients who need invasive dental procedures should have
and Mineral Research (14). procedures performed and healing complete before starting
This condition has received considerable public exposure, bisphosphonate therapy, if circumstances permit. Patients
which caused misconceptions among medical and dental pro- already taking a bisphosphonate may elect to take some time
fessionals as well as the public regarding the seriousness and off therapy, although there is no evidence that this will
frequency of this condition. Some patients decided to stop improve outcomes.
bisphosphonate treatment although they were at high risk of
fracture and low risk of ONJ. Atypical Femur Fractures
ONJ is a clinical diagnosis: Although bisphosphonates reduce the rates of fractures due
Exposed necrotic bone in the maxillofacial region not to osteoporosis, reports have suggested a link between
healing after 8 weeks of appropriate therapy in patients bisphosphonate use and the development of so called atypi-
with no history of craniofacial radiation (14). cal insufficiency fractures.
The bone may be yellow or white, the borders smooth or Proximal femur fractures (hip) due to osteoporosis have
ragged. no warning symptoms, are unilateral (on the side of a
ONJ often follows an invasive procedure, such as dental fall), have an acute angle, and may be comminuted.
extraction, or occurs in patients with poorly fitting Atypical fractures are often preceded by weeks of
dentures or bony exostoses. months of prodromal pain in the thigh or groin, require
There may be pain, swelling, paresthesias, drainage, soft little or no trauma.
tissue ulceration, sinus tracks, and loosening of teeth but Thirty percent of atypical fractures are bilateral.
many patients are asymptomatic.
The location is in the subtrochanteric region of the
Some lesions heal slowly or not at all, but healing has
femoral shaft.
been reported and is probably the rule rather than the
They are transverse or oblique angle with a medial spike
exception.
and little or no comminution.
ONJ has been seen in subjects not using
They begin as a localized periosteal reaction of the
bisphosphonates, but the background incidence is not
lateral cortex.
known.
Cortical bone is unusually thick (and this is not a
ONJ was not identified prospectively in any of the clinical something that bisphosphonates do) and healing is often
trials that included more than 60 000 patient-years in studies delayed.

The Endocrine Society. Downloaded from press.endocrine.org by [${individualUser.displayName}] on 12 January 2017. at 12:19 For personal use only. No other uses without permission. . All rights reserved.
ENDO 2016 BONE, CALCIOTROPIC HORMONES, AND VITAMIN D 77

Bone biopsies in such patients often show severely reduced used to decide when to end a drug holiday, but the
bone turnover, although I have seen a patient with one of risedronate study showed that fracture risk remained re-
these subtrochanteric fractures whose iliac crest biopsy was duced despite what seemed to be unfavorable changes in
completely normal. Several retrospective studies have also these parameters (11). Conversely, there is no evidence that,
suggested an association between bisphosphonate use and off treatment, fracture risk is reduced if BMD is stable or
atypical fractures (1720). The association between long- bone turnover marker is low.
term bisphosphonate use and atypical fractures does not
demonstrate causality. Additional large-scale studies are Monitoring Therapy
needed to further clarify this issue. BMD by dual-energy x-ray absorptiometry has been the
accepted technique for monitoring therapy, usually per-
Possible Side Benefits of Bisphosphonate Therapy formed every 12 years. Although treatments produce, on
Not all the news about long-term bisphosphonate use is bad. average, gains in BMD (more for some agents than for
There is evidence from controlled trials and observational others), the association between BMD gains and fracture
studies that bisphosphonate treatment is associated with a risk reductions are, for the most part, not terribly strong, and
decreased risk of breast cancer (2124), colorectal cancer not linear. I tell my patients that Im happy if BMD is not
(25), gastric cancer (26), stroke (27), and myocardial infarc- going down. One of my patients said it well: If youre not
tion (28), as well as improved survival (29). losing, youre winning.

Drug Holidays
Bisphosphonates are unique in that the drugs accumulate in MAIN CONCLUSIONS
bone and there seems to be residual benefit in terms of In my view, once we start treatment for osteoporosis, we
fracture reduction for some time after a 35-year course of must continue doing something to reduce fracture risk in-
bisphosphonate treatment (11). definitely and therapy be monitored by serial BMD testing.
The few data we have suggest that for higher-risk pa- Because bisphosphonates are avidly bound to bone, a reser-
tients, continuing treatment for 6 10 years is better than voir of drug accumulates after years of treatment that is
stopping after 35 years (3, 13, 30). Although the drug gradually released over months or years and seems to result
holiday concept has been widely accepted (31), data are not in a lingering antifracture benefit for some time after therapy
robust regarding how long to treat, how long the holiday is stopped. This makes it possible to consider drug
should be, when the holiday should be stopped, or effective- holidaystime off bisphosphonate therapy (but possibly on
ness of treatment after restarting). The good news is that a another agent, but not another potent antiresorptive drug)
break in bisphosphonate therapy may reset the clock on and then resuming therapy. Although there is no strong
ONJ and atypical femur fractures. science to guide us, we believe that some time off treat-
The American Society for Bone and Mineral Research con- ment should be offered to most patients on long-term
vened a task force on this issue (32). The algorithm they bisphosphonate therapy. The duration of treatment and the
produced is shown in Figure 1. length of the holiday should be tailored to individual patient
It has been suggested that a decrease in bone mineral circumstances, including the risk of fracture and the binding
density (BMD) or increase in bone turnover marker might be affinity of the particular bisphosphonate used.

FIGURE 1: ASBMR Task Force on Long-term Bisphosphonate Use Recommendations for Patient Management.

The Endocrine Society. Downloaded from press.endocrine.org by [${individualUser.displayName}] on 12 January 2017. at 12:19 For personal use only. No other uses without permission. . All rights reserved.
78 ENDO 2016 MEET-THE-PROFESSOR CLINICAL CASE MANAGEMENT

CASES alendronate on risk of fracture in women with existing vertebral fractures.


Lancet. 1996;348:1535-1541.
Case 1 is a 52-year-old woman diagnosed with osteopenia 4
5. Harris ST, Watts NB, Genant HK, et al. Effects of risedronate treatment on
years ago (femoral neck T-score, 2.2). She has been treated vertebral and nonvertebral fractures in women with postmenopausal osteopo-
with a weekly oral bisphosphonate since then with no signifi- rosis: A randomized controlled trial. Vertebral Efficacy With Risedronate
Therapy (VERT) Study Group. JAMA. 1999;282(14):1344-1352.
cant change in her BMD. Is this an appropriate patient for a
6. Black DM, Delmas PD, Eastell R, et al. Once-yearly zoledronic acid for
drug holiday? treatment of postmenopausal osteoporosis. N Engl J Med. 2007;356:
Case 2 is a 70-year-old woman who has been on a weekly 1809-1822.
oral bisphosphonate for 5 years. Her baseline femoral neck 7. Adami S, Bhalla AK, Dorizzi R, et al. The acute-phase response after
bisphosphonate administration. Calcif Tissue Int. 1987;41:326-331.
T-score was 2.8 and now is 1.8. Is this an appropriate 8. Gallacher SJ, Ralston SH, Patel U, Boyle IT. Side-effects of pamidronate.
patient for a drug holiday? Lancet 1989;2:42-43.
Case 3 is a 70-year-old woman who has been on 9. Zojer N, Keck AV, Pecherstorfer M. Comparative tolerability of drug thera-
pies for hypercalcaemia of malignancy. Drug Safety. 1999;21(5):389-406.
denosumab, 60 mg twice yearly for 4 years. Her baseline 10. Maalouf NM, Heller HJ, Odvina CV, Kim PJ, Sakhaee K.
femoral neck T-score was 2.8 and now is 1.8. Is this an Bisphosphonate-induced hypocalcemia: Report of 3 cases and review
appropriate patient for a drug holiday? of literature. Endocrine Practice. 2006;12:48-53.
11. Watts NB, Chines A, Olszynski WP, et al. Fracture risk remains reduced one
Case 4 is a 70-year-old woman who has been on a weekly year after discontinuation of risedronate. Osteoporos Int. 2008;19:365-372.
oral bisphosphonate for 5 years. Her baseline femoral neck 12. Mellstrom DD, Sorensen OH, Goemaere S, Roux C, Johnson TD, Chines
T-score was 2.8 and now is 2.8. Is this an appropriate AA. Seven years of treatment with risedronate in women with postmeno-
pausal osteoporosis. Calcif Tissue Int J. 2004;75(6):462-468.
patient for a drug holiday? 13. Black DM, Reid IR, Boonen S, et al. The effect of 3 versus 6 years of
Case 5 is a 75-year-old woman who was has taken weekly zoledronic acid treatment of osteoporosis: A randomized extension to
alendronate for 13 years. Her baseline femoral neck T-score the HORIZON-Pivotal Fracture Trial (PFT). J Bone Miner Res.
2012;27:243-254.
was 5.1. Her most recent femoral neck T-score was 4.8. Is
14. Khosla S, Burr D, Cauley J, et al. Bisphosphonate-associated osteone-
this an appropriate patient for a drug holiday? What other crosis of the jaw: Report of a task force of the American Society for
options should be considered? Bone and Mineral Research. J Bone Miner Res. 2007;22:1479-1491.
15. Saad F, Brown JE, Van Poznak C, et al. Incidence, risk factors, and
outcomes of osteonecrosis of the jaw: Integrated analysis from three
DISCUSSION OF CASES AND ANSWERS blinded active-controlled phase III trials in cancer patients with bone
metastases. Ann Oncol. 2012;23:1341-1347.
Case 1 met the guidelines for pharmacologic therapy prior to 16. Hellstein JW, Adler RA, Edwards B, et al. Managing the care of
2008 but does not meet the current guidelines. I would stop her patients receiving antiresorptive therapy for prevention and treatment
bisphosphonate therapy but not for a drug holiday because she of osteoporosis: Executive summary of recommendations from the
American Dental Association Council on Scientific Affairs. J Am Dent
does not need it.
Assoc. 2011;142:1243-1251.
Case 2 has had a nice increase in BMD with 5 years of 17. Goh SK, Yang KY, Koh JS, et al. Subtrochanteric insufficiency fractures
bisphosphonate treatment, is no longer at high risk of fracture, in patients on alendronate therapy: A caution. J Bone Joint Surg Br.
and is a good candidate for a drug holiday. 2007;89(3):349-353.
18. Neviaser AS, Lane JM, Lenart BA, Edobor-Osula F, Lorich DG. Low-
Case 3 has had a nice increase in BMD with 4 years of energy femoral shaft fractures associated with alendronate use. J Orthop
denosumab therapy but denosumab effects go away very Trauma. 2008;22:346-350.
quickly so she is not a candidate for a drug holiday. 19. Lenart BA, Neviaser AS, Lyman S. et al. Association of low-energy
femoral fractures with prolonged bisphosphonate use: A case control
Case 4 has avoided the loss of bone that would have been study. Osteoporos Int. 2009;20:1353-1362.
expected but is still at high risk. Studies suggest that pa- 20. Park-Wyllie LY, Mamdani MM, Juurlink DN, et al. Bisphosphonate use
tients like this do better with treatment of longer duration. and the risk of subtrochanteric or femoral shaft fractures in older women.
JAMA. 2011;305:783-789.
Case 5 remains at high risk despite 13 years of treatment 21. Vestergaard P, Fischer L, Mele M, Mosekilde L, Christiansen P. Use of
with bisphosphonate. I would give her a bisphosphonate bisphosphonates and risk of breast cancer. Calcif Tiss Int. 2011;88:255-262.
holiday but treat her with teriparatide for 2 years after 22. Dreyfuss JH. Oral bisphosphonate use associated with a decreased risk of
breast cancer. Cancer. 2010;60:343-344.
stopping her bisphosphonate and reconsider treatment op- 23. Cheblowski RT, Chen Z, Cauley JA, et al. Oral bisphosphonate use and
tions when the 2 years of teriparatide are up. breast cancer incidence in postmenopausal women. J Clin Oncol.
2010;28:3528-3590.
24. Rennert G, Pinchev M, Rennert HS. Use of bisphosphonates and risk of
REFERENCES postmenopausal breast cancer. J Clin Oncol. 2010;28:3577-3581.
1. Russell RG, Rogers MJ. Bisphosphonates: From the laboratory to the 25. Rennert G, Pinchey M, Rennert HS, Gruber SB. Use of bisphosphonates and
clinic and back again. Bone. 1999;25(1):97-106. reduced risk of colorectal cancer. J Clin Oncol. 2011;29:1146-1150.
2. Bone HG, Hosking D, Devogelaer JP, et al. Ten years experience with 26. Abrahamsen B, Pazianas M, Eiken P, Russell RG, Eastell R. Esophageal
alendronate for osteoporosis in postmenopausal women. New Engl J Med. and gastric cancer incidence and mortality in alendronate users. J Bone
2004;350:1189-1199. Miner Res. 2012;27:679-686.
3. Black DM, Schwartz AV, Ensrud KE, et al. Effects of continuing or 27. Steinbuch M, DAgostino RB, Mandel JS, et al. Assessment of mortal-
stopping alendronate after 5 years of treatment: The Fracture Interven- ity in patients enrolled in a risedronate clinical trial program: A retro-
tion Trial Long-term Extension (FLEX): A randomized trial. JAMA. spective cohort study. Regul Toxicol Pharmacol. 2002;35(3):320-326.
2006;296:2927-2938. 28. Wolfe F, Bolster MB, OConnor CM, Michaud K, Lyles KW, Colon-
4. Black DM, Cummings SR, Karpf DB, et al. Randomised trial of effect of Emeric CS. Bisphosphonate use is associated with reduced risk of

The Endocrine Society. Downloaded from press.endocrine.org by [${individualUser.displayName}] on 12 January 2017. at 12:19 For personal use only. No other uses without permission. . All rights reserved.
ENDO 2016 BONE, CALCIOTROPIC HORMONES, AND VITAMIN D 79

myocardial infarction in patients with rheumatoid arthritis. J Bone Miner 31. Whitaker M, Guo J, Kehoe T, Benson G. Bisphosphonates for
Res. 2013;28:984-991. osteoporosisWhere do we go from here? N Engl J Med. 2012;366:
29. Grey A, Bolland MJ. The effect of treatments for osteoporosis on 2048-2051.
mortality. Osteoporos Int. 2013;24:1-6. 32. Adler RA, Fuleihan GE, Bauer DC, et al. Managing osteoporosis in
30. Schwartz AV, Bauer DC, Cauley JA, et al. Efficacy of continued alendro- patients on long-term bisphosphonate treatment: Report of a Task Force
nate for fractures in women without prevalent vertebral fracture: The of the American Society for Bone and Mineral Research [published
FLEX trial (abstract). J Bone Miner Res. 2010;25(5):976-982. online September 9, 2015]. J Bone Miner Res. doi:10.1002/jbmr.2708.

The Endocrine Society. Downloaded from press.endocrine.org by [${individualUser.displayName}] on 12 January 2017. at 12:19 For personal use only. No other uses without permission. . All rights reserved.
80 ENDO 2016 MEET-THE-PROFESSOR CLINICAL CASE MANAGEMENT

Vitamin D Replacement in Patients With Malabsorption Disorders

M55 25-hydroxyvitamin D of 40 60 ng/mL and recognized up to


Presented, April 1 4, 2016 100 ng/mL is safe; toxicity is usually seen when blood levels
are greater than 150 ng/mL. Whereas the IOM recommended
that most children and all adults up to age 70 years require only
Michael F. Holick, MD, PhD. Section of Endocrinology, 600 IU of vitamin D daily the Endocrine Society recommended
Nutrition and Diabetes, Department of Medicine, Boston that children should receive 600 1000 IU daily and adults
University Medical Center, Boston, Massachusetts 02118, 1500 2000 IU daily. Obese adults required 23 times more
E-mail: mfholick@bu.edu vitamin D to satisfy their requirement.
The Endocrine Society recommends that treating vitamin D
INTRODUCTION deficiency with 50 000 IU of vitamin D2 once a week for 8
Historical Overview weeks is effective. To prevent recurrence the recommendation
Vitamin D deficiency became a major health problem for is to maintain patients on 50 000 IU of vitamin D every 2
industrialized countries beginning in the mid 1600s. By the weeks or approximately 1500-2000 IU daily.
turn of the last century upwards of 90% of children living in
northeastern United States and in the industrialized centers in
BARRIERS TO OPTIMAL PRACTICE
Europe had evidence of the bone-deforming disease, rickets.
One of the major barriers is to obtain a blood level of
The discovery that cod liver oil and sunlight were both effec-
25-hydroxyvitamin D to determine the patients vitamin D
tive in preventing this devastating disease resulted in the forti-
status. This can be caused by lack of knowledge or interest by
fication of milk with vitamin D and government agencies
the physician or a problem with the insurance company reim-
recommending that infants and children be exposed to sensible
bursing for the test. Both the IOM and Endocrine Society do
sun light. Vitamin D fortification became extremely popular in
not recommend broad screening for vitamin D status. The
the 1930s and 1940s and not only milk but also bread, custard,
assay for 25-hydroxyvitamin D should be used in circum-
shaving cream, hot dogs, and even beer were fortified with
stances for patients who were at risk for vitamin D deficiency
vitamin D. However, an outbreak of infantile hypercalcemia
or for patients who may have a metabolic abnormality making
that was associated with mental retardation, facial abnormali-
them sensitive to vitamin D therapy. Risk factors would include
ties, and heart problems in Great Britain, which was incorrectly
fat malabsorption syndromes, obesity, and medications that influ-
thought to be due to over fortification of milk with vitamin D,
ence vitamin D metabolism. Patients with granulomatous disor-
led to the banning of vitamin D fortification throughout Europe
and most of the rest of the world with the exception of the ders such as sarcoidosis must be monitored carefully to prevent
United States and Canada. Its likely that this was due to the hypercalcemia.
rare genetic disorder, Williams syndrome. Children with this Another barrier is that there are a variety of vitamin D
disorder have elfin facies, mild mental retardation, heart prob- supplements available and that the only pharmaceutical form
lems, and a hypersensitivity to vitamin D that can cause hyper- available is vitamin D2. There has been a lot of controversy and
calcemia. misconception that vitamin D2 was less effective than vitamin
D3 in maintaining blood levels of 25-hydroxyvitamin D. Sev-
eral studies, however, have demonstrated that vitamin D2 is as
SIGNIFICANCE OF THE CLINICAL PROBLEM effective in maintaining 25-hydroxyvitamin D levels in the
Concern that sun exposure increases risk for the most common
preferred range of 40-60 ng/mL as vitamin D3.
cancer (ie, skin cancer) has led to widespread use of sunscreens
and the recommendation to avoid all direct sun exposure. This has
led to a worldwide vitamin D deficiency epidemic. It is now LEARNING OBJECTIVES
estimated that 40 and 60% of children and adults throughout the As a result of participating in this session, learner should be
world are vitamin D deficient and insufficient, respectively. able to:
In 2010 The Institute of Medicine (IOM) made recommen- 1. Learn how to determine a persons vitamin D status
dations using a population model for vitamin D intake for and how to use the serum 25-hydroxyvitamin D assay
children and adults in the United States. They recommended in a clinical setting.
that to maximize bone health a 25-hydroxyvitamin D should be 2. Learn how to treat and prevent vitamin D deficiency in
greater than 20 ng/mL. The Endocrine Society recommended children and adults.
that vitamin D deficiency should be defined as a 25-hydroxyvitamin 3. Learn how to use the assay for 1,25-dihydroxyvitamin
D less than 20 ng/mL; insufficiency, 2129 ng/mL; and suffi- D when evaluating patients with calcium, phosphate,
ciency 30 ng/mL. They recommended the preferred level of and bone metabolism disorders.

The Endocrine Society. Downloaded from press.endocrine.org by [${individualUser.displayName}] on 12 January 2017. at 12:19 For personal use only. No other uses without permission. . All rights reserved.
ENDO 2016 BONE, CALCIOTROPIC HORMONES, AND VITAMIN D 81

4. Learn how to treat and prevent vitamin D deficiency with vitamin D. However, caution is required in treating vita-
the in patients with malabsorption syndromes. min D deficiency in patients with chronic granulomatous dis-
orders such as sarcoidosis and tuberculosis. They have in their
granuloma-activated macrophages that efficiently convert
STRATEGIES FOR DIAGNOSIS, THERAPY,
25-hydroxyvitamin D3 to 1,25-dihydroxyvitamin D3. This ac-
AND/OR MANAGEMENT
tive metabolite enters the circulation and can result in hypercalciuria
25-Hydroxyvitamin D is the major circulating form of vitamin D
and hypercalcemia.
that should be used for the purpose of evaluating a persons
Patients with vitamin D deficiency often have high-normal or
vitamin D status. Blood levels of 1,25-dihydroxyvitamin D are
elevated levels of PTH. Chronic vitamin D deficiency can result in
often normal or elevated in a vitamin D deficient state and thus
significant parathyroid hyperplasia, which can ultimately results in
provide no value when determining a persons vitamin D status.
autonomy causing tertiary hyperparathyroidism.
This assay, however, can be of value in acquired and inherited
Patients with chronic kidney disease, no matter what the
disorders of calcium and phosphate metabolism associated with
degree of renal failure, should be treated for vitamin D defi-
alterations in the production of 1,25-dihydroxyvitamin D.
ciency and maintain a blood level of 25-hydroxyvitamin D
Vitamin D deficiency is common in both children and
of at least 30 ng/mL. This may help reduce the development of
adults. In the United States the U.S. Centers for Disease Con-
secondary hyperparathyroidism. Patients with early stages of
trol and Prevention reported that 32% of children and adults are
Chronic Kidney Disease (CKD) that cannot adequately excrete
at risk for vitamin D deficiency (ie, a 25-hydroxyvitamin D
phosphate results in an increase in serum phosphate levels.
20 ng/mL). Based on the literature regarding blood levels of
This provides a signal to osteocytes to produce fibroblast
25-hydroxyvitamin D that is not associated with the vitamin D
growth factor 23, which travels to the kidney to increase
deficiency metabolic bone disease osteomalacia and the plateau
phosphate excretion by internalizing the sodium-phosphate
of PTH levels as it relates to blood levels of 25-hydroxyvitamin
cotransporter. It unfortunately also inhibits the renal production
D, the Endocrine Society in its guidelines recommended that of 1,25-dihydroxyvitamin D. This sets up a vicious cycle, ie, a
for maximum bone health, a 25-hydroxyvitamin D should be at decrease in 1,25-dihydroxyvitamin D results in a decrease in in-
least 30 ng/mL. The Endocrine Society also recommended that testinal calcium absorption. The resulting transient decrease in
maintenance of a 25-hydroxyvitamin D of at least 30 ng/mL the ionized calcium is recognized by the calcium sensor in the
was associated with improved muscle function, especially in parathyroid glands resulting in an increase in the production
the elderly. and secretion of PTH. The constant stimulation of the parathy-
There are a variety of strategies to treat and prevent vitamin roid glands can lead to parathyroid hyperplasia. Patients with
D deficiency in children and adults. For infants who may be end-stage renal disease are unable to produce an adequate amount
only seen once in the emergency department or by their pedia- of 1,25-dihydroxyvitamin D3 even when the blood phosphate level
trician, a single 250 000-IU dose of vitamin D orally or im has is normal. These patients require treatment with either
been effective in helping to reduce risk for rickets and is 1,25-dihydroxyvitamin D3 or one of its active analogs.
known as Stoss therapy. The Endocrine Society recommends Patients with hypoparathyroidism and pseudohypoparathyroid-
that all infants from the time they are born should receive ism have a decreased capacity to produce 1,25-dihydroxyvitamin
400-1000 IU daily. This is especially important for infants who D. These patients should be treated with vitamin D for vitamin D
were being breast fed given that human breast milk contains deficiency as well as with 1,25-dihydroxyvitamin D3 to reduce
very little vitamin D, usually not more than 25 IU/L. For risk for hypocalcemia.
children over 1 year of age 50 000 IU of vitamin D once a Patients with chronic granulomatous disorders are at risk for
week for 6 weeks or 2000 IU daily is effective. A maintenance developing hypercalciuria and hypercalcemia when blood lev-
dose of 600-1000 IU is recommended to prevent recurrence. els of 25-hydroxyvitamin D are greater than 30 ng/mL. These
For all adults 50 000 IU of vitamin D once a week for 8 weeks patients, however, should not be made vitamin D deficient and
is effective in correcting vitamin D deficiency. To prevent this can lead to the painful bone disease osteomalacia and
recurrence 50 000 IU of vitamin D every 2 weeks for up to 6 severe proximal muscle weakness. These patients are treated
years has been effective. For obese adults with a body mass with lower doses of vitamin D to maintain blood levels of
index greater than 30 kg/m2, 23 times more vitamin D is often 25-hydroxyvitamin D in the range of 20-29 ng/mL. Serum
required to treat and prevent vitamin D deficiency. Patients calcium levels should be carefully monitored to be sure that the
with fat malabsorption syndromes or who are on medications patient does not become hypercalcemic.
such as glucocorticoids, antiseizure medications, and AIDS There are several genetic disorders affecting production or
medications often required significantly more vitamin D to recognition of 1,25-dihydroxyvitamin D. Pseudovitamin D de-
treat and prevent vitamin D deficiency. Patients with primary ficiency rickets is caused by a mutation of the renal 1 alpha
hyperparathyroidism should be treated for vitamin D deficiency hydroxylase. There is a rare genetic disorder of the hepatic
given that it has been demonstrated that they will not signifi- 25-hydroxylase that can cause rickets. A 24-hydroxylase defi-
cantly increased their serum calcium when they are treated ciency has been associated with hypercalcemia during infancy.

The Endocrine Society. Downloaded from press.endocrine.org by [${individualUser.displayName}] on 12 January 2017. at 12:19 For personal use only. No other uses without permission. . All rights reserved.
82 ENDO 2016 MEET-THE-PROFESSOR CLINICAL CASE MANAGEMENT

Older children and adults who present with a history of kidney vitamin D, an alternative is to use the U.S. Food and Drug
stones, hypercalciuria, and hypercalcemia should be evaluated Administrationsanctioned Sperti lamp. This lamp emits UVB
for a 24-Hydroxylase deficiency. radiation and is effective in raising blood levels of 25-hydroxyvitamin
D and maintaining these levels in patients with that malabsorp-
tion syndromes.
CASES AND DISCUSSION
Case 1
Case 3: Mystery Case
A 35-year-old white female with long-standing history of
A 45-year-old male presents with a serum calcium of 13.5 mg/dL
Crohns disease with multiple surgeries resulting in a short
and a PTH of 30 pg/mL. An ultrasound and sestamibi scan
bowel syndrome with only 4 feet of small intestine presented
suggested a possible parathyroid adenoma and it was surgically
with excruciating unrelenting aches and pains in her bones and
removed. Intraoperative PTH level decreased from 30 to 10
muscles. A serum 25-hydroxyvitamin D was undetectable and
pg/mL. Twenty-four hours after surgery serum calcium was 11.0
her PTH level was 560 pg/mL. mg/dL with a PTH of 25 pg/mL. A repeat sestamibi scan sug-
1. What is the most likely cause for her global aches and gested another possible foci and he underwent a surgical explora-
pains? tion and a parathyroid gland was removed. Twenty-four hours
2. What strategies can be used to improve her vitamin D after surgery his PTH was undetectable and he had a calcium of
status? 10.9 mg/dL. A serum 25-hydroxyvitamin D was 57 ng/mL and
1,25-dihydroxyvitamin D was 57 pg/mL. The patient denies tak-
Case 2 ing any calcium or vitamin D supplementation.
A 55-year-old white female who underwent gastric bypass 5. What additional testing would be appropriate to
surgery presents with a serum 25-hydroxyvitamin D of 5 ng/mL and determine the cause of his persistent hypercalcemia?
PTH of 120 pg/mL. Her bone mineral density revealed T-scores,
0.5 and 3.0 in the lumbosacral spine and femoral neck. REFERENCES
3. What strategies would you use to treat her vitamin D 1. Farraye FA, Nimitphong H, Stucchi A, et al. Use of a novel vitamin D
bioavailability test demonstrates that vitamin D absorption is decreased in
deficiency?
patients with quiescent Crohns disease. Inflamm Bowel Dis. 2011;17:2116-
4. Why is there such a discrepancy in the T-scores for the 2121.
lumbosacral spine and femoral neck? 2. Koutkia P, Lu Z, Chen TC, Holick MF. Treatment of vitamin D deficiency due
to Crohns disease with tanning bed ultraviolet B radiation. Gastroenterol.
2001;121:1485-1488.
Conclusions 3. Pramyothin P, Biancuzzo RM, Lu Z, Hess DT, Apovian CM, Holick MF.
There are several strategies for treating and preventing vitamin Vitamin D in adipose tissue and serum 25-hydroxyvitamin D After Roux-en-Y
gastric bypass. Obesity. 2011;19(11):2228-2234.
D deficiency in patients with malabsorption syndromes. A
4. Pramyothin P, Holick MF. Vitamin D supplementation: Guidelines and
vitamin D absorption test has been developed which can help evidence for subclinical deficiency. Curr Opin Gastroenterol. 2012; 28(2):
the clinician in evaluating whether a patient with a malabsorp- 139-150.
tion syndrome is able to absorb any vitamin D. If so pharma- 5. Dabai NS, Pramyothin P Holick MF. The effect of ultraviolet radiation from
a novel portable fluorescent lamp on serum 25-hydroxyvitamin D3 levels in
cologic doses of vitamin D may be appropriate. However for healthy adults with Fitzpatrick skin types II and III. Photodermatol
patients who are unable to absorb any dietary or supplemental Photoimmunol Photomed. 2012;28(6):307-311.

The Endocrine Society. Downloaded from press.endocrine.org by [${individualUser.displayName}] on 12 January 2017. at 12:19 For personal use only. No other uses without permission. . All rights reserved.
DIABETES AND
GLUCOSE METABOLISM

The Endocrine Society. Downloaded from press.endocrine.org by [${individualUser.displayName}] on 12 January 2017. at 12:19 For personal use only. No other uses without permission. . All rights reserved.
84 ENDO 2016 MEET-THE-PROFESSOR CLINICAL CASE MANAGEMENT

Using CGM For Day-To-Day Insulin-Dosing Decisions

M03 days but sometimes weeks or months later would review its
Presented, April 1 4, 2016 data retrospectively, after the wear period.
The first unblended (real-time) CGM devices did not arrive
until the mid 2000s with the MiniMed Guardian RT-System
Steven Edelman, MD. Taking Control of Your and the Dexcom STS-CGM. Such real-time CGMs are now
Diabetes, University of CaliforniaSan Diego, Del Mar, available for both short-term professional trials and long-
California 92014, E-mail: sedelman@ucsd.edu term personal use (Dexcom).

INTRODUCTION LEARNING OBJECTIVES


Historical Overview
Understand the significance of CGM in the management
Diabetes management focuses on achieving target hemoglo-
of type 1 diabetes.
bin A1c (HbA1c) levels to reduce acute and chronic com-
plications. Self monitoring of blood glucose (SMBG) with Explain why the trend arrow is so important in day to
glucose meters remains the mainstay of glycemic monitor- day insulin adjustments made by patients.
ing in most people with type 1 and type 2 diabetes but has Review the pitfalls of using blinded CGM in patients
major drawbacks. By simply reporting one blood glucose with type 1 diabetes.
(BG) reading at a single point in time, SMBG fails to
provide information about direction, both from where the SIGNIFICANCE OF THE CLINICAL PROBLEM
BG is coming from and to where it is going. Even testing AND BARRIERS TO OPTIMAL PRACTICE
with SMBG four to six times each day, which is much Since its introduction, real-time CGM use has been vali-
greater than the national average, can miss persistent hyper- dated by multiple studies to improve glycemic control in
glycemia and potentially dangerous hypoglycemia.
both children and adults, while concurrently reducing the
In contrast, continuous glucose monitors (CGM) sample
incidence of hypoglycemia. As a result, real-time CGM is
interstitial glucose every 5 minutes (288 readings each day),
now endorsed by leading organizations such as the ADA,
providing extensive analytics including glucose variability,
AACE, and The Endocrine Society as a component of stan-
hourly statistics and trends, modal day views, and estimated
dard of care management for diabetes.
A1c values, all of which are extremely accurate and helpful
to the patient primarily and provider secondarily. Recent Despite the increased availability and demonstrated effec-
advances in sensor accuracy have lowered the mean absolute tiveness of real-time CGM, providers continue to use
relative difference of current generation CGMs (Dexcom blinded CGM during short-term wear periods. Unlike the
G5) to less than 10%, matching that of glucose meters. multiple studies supporting real-time CGM, the evidence
When first introduced in 1999, CGMs were exclusively regarding blinded, professional CGM use is less conclusive.
used for short-term, blinded, professional use. The For short-term professional wear periods, a common ar-
Minimed (Medtronic) CGMs was worn for 3 days, but its gument for using blinded CGM is that it better captures a
BG measurements were hidden from the patient. Then, the patients regular day, when their behavior is uninfluenced
device would be returned and a health care provider usually by knowledge of their BG data. When analyzing the data

FIGURE 1. Scenarios in which respondents continuous glucose monitoring device showed a glucose value of 110 mg/dL
and they were planning to eat 50 g of carbohydrates (two arrows up/two arrows down).

The Endocrine Society. Downloaded from press.endocrine.org by [${individualUser.displayName}] on 12 January 2017. at 12:19 For personal use only. No other uses without permission. . All rights reserved.
ENDO 2016 DIABETES AND GLUCOSE METABOLISM 85

FIGURE 2. Impact of the direction and rate of glucose change on a mealtime insulin dose at euglycemia (110 mg/dL).
Left panel (A) indicates the percentage of respondents who increased their insulin dosages 0 to 400% when 2 UP arrows
were displayed. Right panel (B) indicates the percentage of respondents who decreased their insulin dosages 0 to 100%
when 2 DOWN arrows were displayed.

retrospectively, the provider hopes to unearth hidden trends coefficient of variation from day-to-day pharmacodynamics
of glycemic variability that will inform interventions that within the same patient to be between 27 and 59% for basal
optimize diabetes management. insulins and 20 30% for short-acting insulins.
Although these intentions are noble, there is unfortunately In addition, a short 37-day trial presents an extremely
no such thing as a regular day of BG values in type 1 limited sample size for capturing a regular day. A given
diabetes. Intrapatient variability of subcutaneously injected 72-hour trial period can easily span both workweek and
insulin can vary greatly in their rate of absorption and time weekend days, active and inactive days, and days with
course of action. Glucose clamp studies have measured the different food portion sizes, compositions, and timings.

FIGURE 3. Glucose levels captured by the retrospective continuous subcutaneous glucose monitoring system (CGMS)
for the evening before and the morning of the patients death. The calibrations measured and entered by the patient are
represented by the 4 circles. The timing of the patients meals, exercise, and correction insulin boluses are represented
by the bars along the bottom of the graph. The precipitous decrease in glucose level after the correction doses can be
observed to start just after midnight, and possible counterregulatory efforts are noted once the glucose level declined to
below 30 mg/dL shortly after 2 AM.

The Endocrine Society. Downloaded from press.endocrine.org by [${individualUser.displayName}] on 12 January 2017. at 12:19 For personal use only. No other uses without permission. . All rights reserved.
86 ENDO 2016 MEET-THE-PROFESSOR CLINICAL CASE MANAGEMENT

CASE 1. CASE 2B.

Real-time Monitoring Is an Intervention 1). The respondents were asked how they would adjust their
In contrast, the full potential of CGM lies in its ability to insulin dose in each scenario.
empower and educate patients in real time, a goal that can Shown in Figure 2, more than two thirds of respondents
effectively improve long-term glucose control. would increase their dose by over 40% in response to having
A recent survey of approximately 300 successful CGM users two up arrows, and nearly 90% would decrease their dose
(mean A1c of 6.9% with minimal hypoglycemia) with type 1 and with two down arrows (15% of whom would not take any
type 2 diabetes detailed how the real-time display of glucose insulin at all). Such varied reactions to the same BG reveal
information such as BG and trend arrows provided actionable how valuable real-time CGM data can be.
insights. A majority of respondents reported initiating therapeutic In fact, the survey of successful regular CGM users re-
interventions overnight after being awoken by CGM alarms for vealed that more than 80% of respondents found that real-
both hypoglycemia and hyperglycemia. A majority of users in this time analysis of their CGM data was more useful than
survey reported more frequent insulin boluses or injections per retrospective analysis. The limitations of retrospective anal-
day after starting CGM. Respondents also reported that continu- ysis are further highlighted by the fact that 19% of those
ous glucose data led to adjustments in the timing and quantity of surveyed never or rarely downloaded their CGM data during
insulin doses before meals and at times when they were correcting clinician office visits. If the overwhelming benefit to CGM
for elevated glucose values. wearers lies in real-time analysis, why should we restrict
this feature?
Example of Strategies for Management of Insulin
Adjustments Protection Against Hypoglycemia and the Dead-in-Bed
The survey quantified these adjustments by presenting the Syndrome
above scenarios, which paired the same euglycemic BG of 110 CGM has proven to be an invaluable tool to protect against
mg/dL with a different glucose curve and trend arrows (Figure hypoglycemia, both mild and especially severe hypoglyce-

CASE 2A. CASE 3.

The Endocrine Society. Downloaded from press.endocrine.org by [${individualUser.displayName}] on 12 January 2017. at 12:19 For personal use only. No other uses without permission. . All rights reserved.
ENDO 2016 DIABETES AND GLUCOSE METABOLISM 87

they deal with their disease day in and day out. As such,
except for research studies, all usage of CGM should be
unblended.
For type 1 diabetes, unblended CGM is the standard of care
plain and simple. If circumstances require the use of short-term
CGM, then unblended CGM should be selected. Real-time
availability of hypoglycemic alarms and trend arrows add in-
formation that is exponentially more insightful than a static
glucose meter reading.
Thanks to David Ahn, MD for his help in preparing this
handout.

CASES
CASE 4. Case 1
Example of how a static BG measurement can lead to over-
and underdosing of insulin.
mia. This factor is of extreme importance in patients with
hypoglycemia unawareness. Case 2A: Jeremy
A case report documents a 23-year-old with type 1 diabe-
tes who suffered a lethal hypoglycemic event in his sleep Case 2B
while undergoing an observation period with blinded, short- The correct answer is D. A common but wrong answer is C.
term professional CGM to investigate recurrent severe hy- Because the BG value is increasing dramatically (3 mg/dL/min)
poglycemia (Figure 3; Ref. 7). On the first evening of his giving a correction based solely on the correction factor will
trial, he exercised at the gym after dinner and then went to underdose the patient. If the trend arrow had been horizontal
bed. At approximately 0900 hours the next morning, he was then 3.3 U would have been the correct answer.
discovered by his family to be dead in bed. He unfortunately
did not respond to glucagon or resuscitative efforts by para- Case 3: Robyn
medics. Robyn looks at her Dexcom (CGM) at 0300 hours and her
Postmortem analysis of his insulin pump and blinded CGM BS was now greater than 350 mg/dL and she gave herself
device revealed glucose readings showing severe hypoglyce- another Rage Bolus 5.0 U. At 0800 hours she had a
mia (50 mg/dL), highlighted by the arrows on the graph, hypoglycemic reaction and needed to ingest glucose tabs.
for at least 3 hours prior to his death. Had he been wearing Robyn did stack her insulin dose, which is not uncommon in
an unblended CGM, a hypoglycemic alarm would have CGM users as they do not see their BG values changing fast
sounded at a higher glucose threshold, likely triggering a enough after a bolus. SQ insulin just has too slow an onset and
life-saving intervention by the patient himself or a nearby hangs around for an extended period of time leading to exces-
family member. sive delayed hypoglycemia.
Although this case of dead-in-bed syndrome is admittedly Barbara is a 69-year-old female who was diagnosed with
a worst-case scenario, severe hypoglycemic or hyperglycemic type 1 diabetes 7 years ago. She is on an Omnipod with
events that occur during a blinded CGM wear period lead to insulin aspart (CHO, 12:1 and CF, 1:40). She is also on
financially and emotionally costly events that are easily pre- pramlintide premeals.
vented by unblinding the CGM. In the United States and Barbara ate a late dinner at 2000 hours.
around the world there are frequent cases of patients passing Predinner BS was 205 mg/dL.
away from severe hypoglycemia. In fact, recent studies suggest She planned on eating 70g CHO.
that 4 10% of deaths in people with type 1 diabetes are due to Premeal does of fast acting insulin: 6 2 8 U.
hypoglycemia.
Case 4: Barbara (continued)
CONCLUSION: THE VALUE OF PATIENT
EMPOWERMENT REFERENCES
I believe that many of the underlying motivations for using 1. Tamborlane WV, Beck RW, Bode BW, et al. Continuous glucose monitor-
blinded professional CGM are rooted in the paternalistic and ing and intensive treatment of type 1 diabetes. N Engl J Med.
2008;359:1464-1476.
anachronistic perspective that the practitioner always knows
2. Battelino T, Conget I, Olsen B, et al. The use and efficacy of continuous
best. I also believe that the most valuable and sustaining glucose monitoring in type 1 diabetes treated with insulin pump therapy:
interventions empower and further educate the patient as A randomised controlled trial. Diabetologia. 2012;55:3155-3162.

The Endocrine Society. Downloaded from press.endocrine.org by [${individualUser.displayName}] on 12 January 2017. at 12:19 For personal use only. No other uses without permission. . All rights reserved.
88 ENDO 2016 MEET-THE-PROFESSOR CLINICAL CASE MANAGEMENT

3. Bergenstal RM, Tamborlane WV, Ahmann A, et al. Effectiveness of sensor- Care in Diabetes2015. Diabetes Care. 2015;38(Suppl 1):S33-S40.
augmented insulin-pump therapy in type 1 diabetes. N Engl J Med. 6. Pettus J, Price DA, Edelman SV. How patients with type 1 diabetes
2010;363:311-320. translate continuous glucose monitoring data into diabetes management
4. Klonoff DC, Buckingham B, Christiansen JS, et al. Continuous glucose decisions. Endocr Pract. 2015;21:613-620.
monitoring: an Endocrine Society Clinical Practice Guideline. J Clin 7. Tanenberg RJ, Newton CA, Drake AJ. Confirmation of hypoglycemia in
Endocrinol Metab. 2011;96:2968-2979. the dead-in-bed syndrome, as captured by a retrospective continuous
5. American Diabetes Association. Glycemic targets. Sec. 6 in Standards of Medical glucose monitoring system. Endocr Pract. 2010;16:244-248.

The Endocrine Society. Downloaded from press.endocrine.org by [${individualUser.displayName}] on 12 January 2017. at 12:19 For personal use only. No other uses without permission. . All rights reserved.
ENDO 2016 DIABETES AND GLUCOSE METABOLISM 89

When and How to Use U500 (or Other Concentrated) Insulin

M10 insulin per day (4). Severe insulin resistance is somewhat


Presented, April 1 4, 2016 arbitrarily defined in the medical literature as an insulin re-
quirement of more than 200 U per day or more than 2 U per
kilogram of body weight daily (1). With increasing obesity,
Wendy Lane, MD. Mountain Diabetes and Endocrine patients with type 2 diabetes may require several hundred units
Center, Asheville, North Carolina 28803, E-mail: of insulin per day. Most patients with type 2 diabetes are not
mountaindiabetes@msn.com meeting glycemic targets, and in patients with increasing insu-
lin resistance driving the need for large doses of insulin, gly-
INTRODUCTION cemic control becomes even more suboptimal (5).
Historical Overview
U500R insulin, a product of Eli Lilly and Company, is 5-fold BARRIERS TO OPTIMAL PRACTICE
concentrated regular insulin containing 500 U of insulin per High insulin doses are clinically challenging to deliver. Insulin
mL vs the standard U100 or 100 U/mL insulin. It was first syringes only hold up to 1 mL of insulin, which contains 100 U
introduced into clinical practice in 1952 to address the high of a U100 insulin preparation. In a patient requiring more than
insulin requirements of patients with diabetes who had devel- 100 U of basal insulin daily, this would require two or more
oped severe insulin resistance caused by high titers of insulin insulin injections to deliver the basal insulin dose alone. Insulin
antibodies to nonhuman insulin preparations (1). U500R insu- pens can only deliver 80 U of basal insulin analogs glargine (in
lin was originally a beef insulin, which was replaced by pork either its U100 or U300 formulation) or detemir with a single
U500R insulin in 1980 and subsequently by U500R human injection. Only insulin degludec, recently marketed as Tresiba
insulin in 1997. U500 insulin usage declined with the develop- by Novo Nordisk, is available in a U200 formulation in a pen
ment of recombinant DNA technology and the manufacturing capable of delivering 160 U (0.8 mL) with a single injection.
of human regular insulin in 1982 as insulin antibody-mediated Large insulin depots can be poorly absorbed and painful to
insulin resistance was no longer a common problem for inject, leading to glycemic variability and suboptimal glycemic
insulin-treated patients. The increase of the obesity epidemic control both from incomplete absorption and from patient non-
over the past 25 years gave rise to a twin epidemic of insulin compliance owing to the discomfort of the large-volume injec-
resistance, reviving the need for a concentrated insulin to meet tion (1, 6). Thus, there is a need for the development of
the insulin needs of an increasingly obese diabetic population. concentrated insulin to reduce the number of insulin injections
Novo Nordisk also produced a concentrated human insulin required daily as well as the insulin volume itself in patients
formulation, Actrapid U-500, which was withdrawn from the with large insulin requirements. In 2015, the first concentrated
European market in 2008. From 2008 through 2015, U500R rapid-acting insulin analog, U200 lispro, also became available
insulin was the only commercially available concentrated insu- to address the mealtime insulin needs of many patients with
lin worldwide to address the needs of patients with high insulin average-to-high insulin requirements on basal/bolus insulin
requirements on the basis of severe insulin resistance (1). In therapy.
2015, several other concentrated insulin preparations became Continuous subcutaneous insulin infusion (CSII), or insu-
available, including U300 glargine (Toujeo, Sanofi Aventis), lin pump therapy, has been shown to improve glycemic
U200 lispro (Humalog, Eli Lilly), and most recently U200 control in patients with type 1 and type 2 diabetes (7, 8), and
degludec (Tresiba, Novo Nordisk). to reduce complications of diabetes and diabetes-related
morbidity and mortality in patients with type 1 diabetes (7,
SIGNIFICANCE OF THE CLINICAL PROBLEM 9). However, delivering large insulin doses by CSII is chal-
The global obesity epidemic has given rise to an epidemic of lenging because of the limited size of the insulin reservoir of
insulin resistance and type 2 diabetes, and increasing body most pumps (1.8 3 mL, with a capacity of 180 to 300 U of
mass index (BMI) is accompanied by a parallel increase in a U100 rapid-acting insulin analog). Thus, patients requiring
insulin doses in insulin-treated patients. In one recent study of more than 200 300 U of insulin daily would need to change
patients with type 2 diabetes, approximately 35% of patients the pump reservoir daily. Furthermore, high basal insulin
required maintenance basal insulin doses of 60 U of insulin or delivery rates by pump (3.5 4 U/h) may cause induration
more (2). In another treat-to-target basal insulin trial involving and scarring of the sc tissue, and the large-volume insulin
insulin-nave patients, 21% of subjects required more than 80 boluses for meals can also have incomplete or inconsistent
U of basal insulin by the end of the trial (3). Market research absorption, leading to glycemic instability (1). Furthermore,
for an insulin delivery system has revealed that 17% of insulin- high insulin delivery rates require frequent battery and res-
treated patients with type 2 diabetes use more than 100 U of ervoir changes of the insulin pump, driving up the cost of

The Endocrine Society. Downloaded from press.endocrine.org by [${individualUser.displayName}] on 12 January 2017. at 12:19 For personal use only. No other uses without permission. . All rights reserved.
90 ENDO 2016 MEET-THE-PROFESSOR CLINICAL CASE MANAGEMENT

this already-expensive therapy. For these reasons, there has (1), protease inhibitors (13), atypical antipsychotics
been increasing use of concentrated U500 insulin via CSII in (14)].
high-dose insulin users during the past 10 years (10). Sev- Gestational diabetes or type 2 diabetes and pregnancy.
eral small case series (11) and a small, single, uncontrolled Congenital or acquired lipodystropy with severe insulin
prospective trial (12) have shown that U500 delivered by resistance.
CSII in patients with high insulin requirements (an off-label Acquired endocrinopathy (Cushings disease,
use of U500 insulin) results in improved glycemic control acromegaly, glucagonoma, pheochromocytoma).
(11, 12), reduced glycemic variability by continuous glucose
monitoring (12), and improved patient satisfaction (12). Pharmacokinetic and Pharmacodynamic (PK/PD)
These studies have led to an ongoing large, randomized, Properties of U500 Insulin
prospective multicenter study to evaluate the safety of U500 Compared with U100 regular insulin, U500 insulin has a time-
insulin in CSII compared with multiple daily injections action profile with an extended duration or tail. Insulin clamp
(MDI) (Eli Lilly; NCT02561078; ClinicalTrials.gov). Tan- studies have shown the time to onset is within 30 minutes,
similar to U100 regular insulin, with peak insulin levels at a
dem, Inc has created an insulin pump with a 480 U insulin
mean of 3 hours, peak glucose infusion rate at 3.5 4.5 hours,
reservoir, the t:flex, to address the needs of insulin pump
and duration of action 6.510 hours after sc injection (15).
patients with higher insulin requirements. The larger reser-
Thus, U500 insulin has a similar time to onset and slightly
voir would reduce the frequency of insulin cartridge changes
slower time-to-peak than U100 regular insulin and a duration
in this population, but the problems associated with high
of action similar to NPH insulin. This gives U500 insulin both
basal insulin delivery rates and large volume boluses of the
basal and bolus insulin characteristics, making it suitable for
current rapid-acting U100 insulin analogs would still be
use as an insulin monotherapy.
problematic for the reasons stated above, even using a
larger-reservoir insulin pump.
U500 Studies
Clinical studies of U500 insulin have consisted mostly of
LEARNING OBJECTIVES retrospective case series (11) and one small uncontrolled pro-
As a result of participating in this session, learners should be spective trial (12). A recent meta-analysis and review of the
able to: U500 medical literature reviewed nine MDI studies and six
Identify appropriate candidates for U500/concentrated CSII studies of U500 insulin. Nine MDI studies involving 310
insulin therapy. patients showed an HbA1c reduction of 1.59% upon conver-
Describe the PK/PD profile of U500 insulin. sion from U100 to U500 insulin (95% confidence interval [CI],
Convert U100 insulin regimens to U500 insulin 1.26 1.92), which was associated with a mean weight gain of
regimens, including both MDI and pump therapy. 4.38 kg (95% CI, 2.35 6.41). Six CSII studies involving 55
Recognize the new concentrated insulin analogs and patients showed a 1.64% overall reduction in HbA1c (95% CI,
detail medical situations warranting their potential use. 1.14 2.14) and was associated with a 2.99 kg weight gain
Appreciate the benefits of combining insulin-sparing (95% CI, 1.837.81; not significant) (11).
therapies with insulin and detail medical situations
warranting their potential use. Dosing of U500 Insulin
U500 insulin is dosed according to total daily insulin require-
ment. If the baseline HbA1c is less than 8.0%, it is recom-
MANAGEMENT OF THE PATIENT WITH mended to reduce the total daily insulin dose by 10 20%; if the
TYPE 2 DIABETES AND HIGH INSULIN baseline HbA1c is greater than 10%, the dose should be in-
REQUIREMENTS creased 10 20%. For baseline HbA1c between 8 and 10%, the
Appropriate Candidates for U500/Concentrated Insulin baseline total daily insulin dose should be converted to U500
Patients with type 2 diabetes who require more than 2 U/kg or insulin on a unit-per-unit basis (1). It is common to see im-
more than 200 U of insulin daily on the basis of obesity proved absorption with conversion to U500 insulin, which may
without any secondary factor increasing insulin resistance are result in a lowered insulin dose after conversion.
candidates for U500 insulin. However, patients with moderate U500 insulin may be dosed using either a U100 insulin
insulin-dose requirements (100 U/d of insulin) under circum- syringe or a tuberculin syringe. There is no U500 insulin
stances of illness or stress may have worsening insulin resis- syringe; thus, unit markings on an insulin syringe must be
tance and require higher-than-usual insulin doses, which may multiplied by 5 to show the actual number of insulin units
become high enough to require the use of concentrated insulin being delivered. The prescription should be written volu-
therapy. Such conditions may include (1): metrically (in mL) as well as in unit markings on an insulin
Medications that worsen insulin resistance [high-dose syringe and stating the actual number of units to be delivered.
glucocorticoids, post-transplant antirejection medications Thus, if the insulin dose is 50 U prior to each meal, the

The Endocrine Society. Downloaded from press.endocrine.org by [${individualUser.displayName}] on 12 January 2017. at 12:19 For personal use only. No other uses without permission. . All rights reserved.
ENDO 2016 DIABETES AND GLUCOSE METABOLISM 91

prescription should be written thus: U500 insulin: Inject 0.1 ity factor should be set at 100, the usual blood glucose target is
mL or 10 U markings on an insulin syringe, 50 U, prior to 100 mg/dL, and the insulin-on-board time should be set at 6
each meal. Patients should be carefully instructed where to hours for U500 insulin.
draw up the insulin on the syringe to avoid overdosing errors. Patients should monitor blood glucose levels at least 4 times
Patients with insulin requirements between 150 and 300 U per day upon conversion to U500 insulin, and check occasional
per day may use either of the following regimens: 0200 0300 hours blood glucose (BG) readings when night
Twice-daily regimen: 60% of the total daily dose before infusion rate changes are made.
breakfast and 40% of the total daily dose before the Patients should be educated on signs and symptoms of
evening meal. hypoglycemia and instructed in the treatment of hypoglycemia.
Thrice-daily regimen: 40% of the total daily dose before Patients beginning U500 insulin should be given prescriptions
breakfast, 30 40% before the noon meal, and 20 30% for glucagon for emergency use.
before the evening meal.
A recent 24-week prospective trial comparing the efficacy and Other Currently Available Concentrated Insulin
Options
safety of twice-daily U500 to twice-daily U500 insulin showed
U300 Glargine
no difference in glycemic control between the two regimens,
U300 glargine (Toujeo, Sanofi Aventis) is identical to 100
but the thrice-daily regimen was associated with a lower inci-
glargine (Lantus) in formulation but the injection volume is
dence of hypoglycemia (16).
reduced by 2/3 (ie, 1 mL contains 300 U glargine instead of
Patients with insulin requirements between 300 and 600 U
100 U). This three-times concentrated glargine formulation has
should use the thrice-daily regimen as described above. If
a flatter pharmacokinetic profile than U100 insulin glargine
fasting blood glucose levels are above target, a fourth injection
with a longer duration of action (24 h) with a lower incidence
consisting of 10% of the total daily insulin dose may be
of nocturnal hypoglycemia than U100 glargine (17). U300
administered at bedtime (with 30% of the total daily dose given
glargine is less bioavailable than U100 glargine in glucose
before each meal).
clamp studies and in clinical trials, with 27% lower area under
Patients requiring more than 600 U of insulin per day should
the curve glucose infusion rate over 24 hours in glucose clamp
use four daily injections as described above.
studies (Toujeo PI) (18) and requires an 1117% higher dose
compared with U100 glargine in clinical trials (17). Also, it is
Dosing U500 Insulin in CSII currently only available in an injection pen, which can deliver
Patients already using U500 insulin in an MDI regimen may
a maximum of 80 U of insulin (no dose conversion necessary;
convert to insulin pump settings by using the following algo-
the pen reads the actual number of units of insulin glargine).
rithm: take 50% of the total daily insulin dose, divide by 24,
Each pen contains 1.5 mL of U300 glargine, or a total of 450
and administer as a single basal hourly infusion rate via insulin
U. The fact that U300 glargine requires an increased dose
pump with the other 50% of the total daily insulin dose distributed
compared with the U100 preparation and that patients requiring
as premeal insulin boluses. At first, it is prudent to reduce the
more than 80 U of basal insulin will still need two basal insulin
nocturnal basal rate between midnight and 0600 hours by 10 15%
injections make this option of limited use in the high-dose
to avoid nocturnal hypoglycemia, as insulin absorption may fur-
insulin-requiring patient.
ther improve when converting U500 from MDI to CSII.
For patients using U100 insulin regimens who are going to
be using U500 insulin by CSII, the total daily insulin dose must U200 Lispro
first be divided by 5, then divided again by 24 to yield the U200 lispro (U200 Humalog, Eli Lilly): This is a twice-
initial hourly U500 infusion rate. Currently, no insulin pumps concentrated, rapid-acting insulin analog for prandial use that
are able to convert pump settings for U500 insulin (which is contains 200 U of lispro insulin per mL vs 100 U. It is of equal
not U.S. Food and Drug Administrationapproved for use with bioavailability to U100 lispro (19), reduces the volume of
an insulin pump). Therefore, it must be recognized that in mealtime insulin by half, and is available only in a pen. No
patients using U500 insulin in CSII, the pump will be deliver- dose conversion from U100 lispro is necessary. The pen con-
ing five times the amount of insulin per hour shown as the tains a total of 600 U of insulin lispro in 3 mL. This may be a
hourly basal rate, and 5 times the number of units shown by the good option for patients on MDI regimens who require high
pump for meal boluses. doses of basal and bolus insulin. This insulin is not available in
Patients who use carbohydrate counting should multiply a vial, and has not been studied in CSII.
their carbohydrate factor by 5. For example, if a patient uses 1
U of insulin per 3 g of carbohydrate, this will become an U200 Degludec
insulin-to-carbohydrate ratio of 1:15 in the pump. Patients U200 degludec (Tresiba, Novo Nordisk): Degludec is an
using preset meal boluses should divide the bolus amount by 5 ultra-long-acting basal insulin analog that forms soluble
for the corresponding U500 meal boluses. The initial sensitiv- multihexamers upon injection from which insulin hexamers

The Endocrine Society. Downloaded from press.endocrine.org by [${individualUser.displayName}] on 12 January 2017. at 12:19 For personal use only. No other uses without permission. . All rights reserved.
92 ENDO 2016 MEET-THE-PROFESSOR CLINICAL CASE MANAGEMENT

are slowly and steadily released into the circulation. There is nightly glargine and lispro insulin, but she required more than
a secondary albumin-binding effect that further protracts the 100 U of basal insulin per day, was taking six insulin injections
action of the insulin, giving it a duration of action of up to per day, and still experienced fasting hyperglycemia with fast-
42 hours (20). Degludec is available in injection pens in ing BGs from 150 200 mg/dL. Therefore, her PCP switched
both U100 and U200 formulations. Degludec has a her to NPH and regular insulin with a total of 90 U of NPH and
pharmacokinetic profile that is flatter and longer in duration 70 U of regular insulin in four injections per day (50 of N with
than U100 glargine; it has not been studied in a head-to- 20 of R before breakfast, 20 of R with lunch and supper, and
head trial against U300 glargine. U200 degludec has been 40 of N with 10 of R at bedtime.). She monitors BGs reliably
shown to be equivalent in bioavailability and PK profile to four times per day and is compliant with her insulin regimen.
the U100 formulation in glucose clamp studies (21), unlike Although she has fairly good control on this regimen (HbA1c
U300 glargine. There is an approximately 10% lower basal 7.1%) she is referred for a more convenient regimen and for
insulin dose requirement for insulin degludec vs 100 insulin consideration of insulin pump therapy.
glargine in patients with type 1 diabetes (22).

Question 1
Adjunct (Insulin-Sparing) Agents in Patients With
What are the shortcomings of insulin glargine in a patient
High-Dose Insulin Requirements
Adjunct therapies to insulin can be useful in patients with high requiring insulin doses like Marys?
insulin requirements to lower the insulin dose, reduce insulin- High doses of insulin glargine can be poorly absorbed and
associated weight gain (excepting the thiazolidinediones), and require multiple injections per dose. This can occur with doses
in some cases, reduce the glycemic variability associated with greater than 60 U per dose.
insulin therapy. Oral agents that reduce weight when added to
high-dose insulin therapy include metformin and the SGLT-2 Question 2
inhibitors. Although the thiazolidinediones have an insulin- Is Mary a feasible insulin pump candidate? What barriers does
sparing effect, their use in combination with high-dose insulin she have to initiation of CSII?
therapy is associated with edema, weight gain, and heart failure Mary is a good pump candidate. She is compliant with a
(23) and therefore is not recommended. The GLP-1 receptor complex insulin regimen, monitoring BGs regularly and moti-
agonist liraglutide has been successfully combined with U500 vated to improve and maintain good glycemic control. How-
insulin and has resulted in improved glycemic control, weight ever, her large insulin requirement will make insulin pump
loss, and lower insulin doses (24) as well as reduced glycemic therapy with a U100 insulin analog impractical because she
variability by continuous glucose monitoring (25). will require a basal insulin rate of 3 4 U per hour, boluses that
may sometimes be greater than 30 U and exceed the bolus
MAIN CONCLUSIONS capability of the pump, and she will require a cartridge change
Patients with type 2 diabetes and large insulin requirements every 11.5 days using a U100 insulin analog.
present a therapeutic challenge. These patients may improve
glycemic control, weight, and glycemic variability by use of
Medications
concentrated insulin with adjunct insulin-sparing therapies. Un-
Insulin (N and R, total 170 180 U, approximately 1.8 U/kg),
til 2015, the only concentrated insulin available in the United
furosemide, benazepril, metformin, amitriptyline, atorvastatin,
States was U500 regular human insulin, and the author and
aspirin.
others have accumulated a body of clinical experience showing
best clinical practices in using U500 insulin in the insulin-
resistant patient with type 2 diabetes with large insulin require- Physical Examination
ments. Newer concentrated insulins have recently become Height, 65; weight, 221 lbs; BMI, 36.8 kg/m2; normal fundi;
commercially available, and it is now our task to discover how normal thyroid; lungs clear; cardiac examination showed regu-
to optimally use these in our patients requiring delivery of large lar rhythm without murmur; abdomen obese and nontender;
insulin doses. dusky rubor of both legs and feet without edema or lesions;
pulses in feet diminished; knee and ankle jerk reflexes are
absent over the feet and monofilament and bone vibratory
CASES
Case 1: Mary sensation are absent to the ankle.
Mary is a 59-year-old woman with a 17-year history of type 2
diabetes with microalbuminuria and advanced peripheral neu- Plan
ropathy, hypertension, dyslipidemia, and recurrent urinary tract Begin CSII with U500 insulin. Initial pump settings:
infections. She had failed oral agents 1 year prior and had been Basal rate 0.6 pump units per hour (equivalent to
begun on an intensive insulin regimen that initially involved 3.0 U/h) 24 hours.

The Endocrine Society. Downloaded from press.endocrine.org by [${individualUser.displayName}] on 12 January 2017. at 12:19 For personal use only. No other uses without permission. . All rights reserved.
ENDO 2016 DIABETES AND GLUCOSE METABOLISM 93

Meal doses: set meal doses of 5 pump units (25 tor less attractive as an adjunct therapy. She is a good candidate
actual units) U500 for breakfast and lunch, 7 pump for addition of a GLP-1 RA to U500 insulin by CSII.
units (35 actual units) for supper. Plan: begin liraglutide at 0.6 mg sq qd, increasing to 1.2 mg
Target BG, 100 mg/dL; insulin sensitivity factor, 100. sq in 1 week.
Continue metformin.

Question 6
Question 3 How should Marys U500 pump settings be reduced upon
Marys basal insulin setting by pump is equivalent to 72 U per
addition of liraglutide to her regimen?
24 hours, in contrast with the 90 U of basal insulin she took in
Mary has good glycemic control on her present U500 insulin
her MDI regimen. Why was this reduced?
doses by CSII. Liraglutide is a long-acting GLP-1 RA that
Mary has fairly good glycemic control already prior to
reduces both basal and bolus insulin requirements. Mary
initiation U500 by CSII. Thus, a 20% insulin reduction at the
should have her U500 basal and meal boluses reduced by
initiation of U500 pump therapy is a prudent initial reduction,
1525% upon initiation of liraglutide. Her present U500 set-
as she is likely to improve her insulin absorption, moving from
tings are MN-6 am: 0.6 pump units (3.0 U/h) and 6 am-MN,
glargine to U500 by CSII and require 10 20% less insulin.
0.7 pump units (3.5 U/h). She is using preset meal boluses of
This also will reduce the chance that she will experience
35 pump units (1525 U) for meals. Her basal pump settings
hypoglycemia upon conversion to U500 by CSII.
should be reduced to MN-6 am: 0.5 pump units (2.5 U/h); 6
am-MN, 0.6 pump units (3.0 U/h). Her meal boluses are re-
Question 4 duced to 2 4 pump units. Her correction dose remains the same
What other instructions should Mary receive upon initiation of (target BG, 100 mg/dL; insulin sensitivity factor, 100).
U500 via insulin pump?
Mary should be instructed in recognition and treatment of
Follow-up Visits
hypoglycemia and have glucagon prescribed. She should be
Follow-up visit 4 weeks later: HbA1c, 6.0%; weight, 223 lbs.
provided with contact information for the insulin pump nurse
Follow-up visit 4 weeks later: HbA1c, 5.4%. Weight, 221
or CDE and instructed to contact her with BG readings in
lbs. Having some mild-to-moderate nocturnal hypoglycemia
24 48 hours, as insulin requirements may decrease dramati-
(lowest BG, 52 mg/dL; several BGs in 60s). Basal rates and
cally and quickly upon conversion from U100 insulin to U500
bolus doses reduced.
insulin.
Follow-up visit 3 months later: HbA1c, 5.8%; weight,
215 lbs.
Follow-up Visits Next 5 years of followup: Mary remains on CSII with U500
Follow-up visit 3 months later: HbA1c, 6.0%. No hypoglyce- insulin by CSII with liraglutide, with stabilized basal infusion
mia (lowest BGs in 60s and occur with exercise only). Weight, rates of 0.35 0.45 pump units per hour and preset meal
226 lbs. Mary states, I love the pump! boluses on average of 3 pump units, ie, 15 U per meal. Her
Follow-up visit 6 months later: HbA1c, 5.8%. No significant HbA1cs have ranged from 5.8 6.9%. Her weight has ranged
hypoglycemia. Weight, 221 lbs. from 215220 lbs. She is pleased with the comfort and conve-
Follow-up visits over next 5 years: HbA1c, 5.0 6.7%; BGs nience of her insulin regimen.
stable and well controlled without hypoglycemia. Weight in-
creases gradually to 230 lbs. She requires a slight increase of Case 2: Rodney
basal rate (now has two basal rates of 0.6 pump units (3.0 U), Rodney is a 42-year-old man with a 15-year history of type 2
from MN-6 am and 0.7 pump units (3.5 U) per hour from 6 diabetes. He failed oral combination therapy with metformin
am-MN to maintain euglycemia. Mary is concerned about her and glyburide, so basal insulin (glargine) was added and
gradual increase in her insulin dose, weight, and HbA1c (now titrated to 100 U per day without therapeutic benefit; all his
at its highest since beginning U500 by CSII at 6.7%). BGs on oral agents plus glargine remained greater than 200
mg/dL. Six weeks ago, Rodneys PCP intensified his insulin
Question 5 regimen to a regimen of glulisine, 20 U prior to meals and
What adjunct therapies are options for Mary to limit insulin- U300 glargine, 100 U at bedtime (a total of five daily injections),
induced weight gain, lower her insulin dose, and lower her in combination with maximal metformin and glyburide. Rodney
HbA1c? monitors BGs three times per day; all readings range from
Adjunct therapies to high-dose insulin that reduce insulin 250 350 mg/dL. Rodney has never experienced hypoglycemia.
dose and weight include metformin, GLP-1 receptor agonists, Rodneys medications include insulin (1.3 U/kg): U300
and SGLT-2 inhibitors. Mary is already on her maximal- glargine, 100 U/d; glulisine, 20 U before meals three times
tolerated dose of metformin, and has a history of recurrent daily; metformin, glyburide, losartan, omeprazole, atorvastatin,
urinary tract infections, making addition of an SGLT-2 inhibi- sertraline, vitamin D, and aspirin.

The Endocrine Society. Downloaded from press.endocrine.org by [${individualUser.displayName}] on 12 January 2017. at 12:19 For personal use only. No other uses without permission. . All rights reserved.
94 ENDO 2016 MEET-THE-PROFESSOR CLINICAL CASE MANAGEMENT

Examination and supper. Instruct Rodney to consume three meals equal in


Height, 70; weight, 280 lbs; BMI, 40.6 kg/m2; blood carbohydrate content approximately 5 6 hours apart (ie, 0700,
pressure (BP), 140/100 mm Hg. Examination is 1200, 1800 h).
unremarkable except for abdominal obesity and absence
of ankle jerk reflexes. HbA1c is 8.3%. Renal function is
normal. Question 10
Which adjunct glucose-lowering therapies should Rodney be
treated with?
Question 7
Rodney should continue metformin. In addition, Rodney
Discuss Rodneys insulin regimen. Assuming he is taking his
should try a long-acting GLP-1 RA. He will begin dulaglutide,
insulin as directed and not eating between meals, what prob-
0.75 mg injected once per week in combination with his new
lems could be associated with his present insulin regimen?
U500 insulin regimen.
What, if any, adjunct therapies to insulin might be of benefit to
Rodney?
Rodney requires a high dose of basal insulin. Insulin Question 11
glargine is poorly absorbed at high doses, and U300 glargine How should Rodney be followed?
has lower bioavailability than U100 glargine, making it even Rodney will continue to monitor his BGs three or more
less efficient in delivering a high basal insulin dose. Rodney times daily and phone, fax, or email them to clinic for review
also is on relatively low mealtime insulin doses compared with and insulin adjustment weekly. He should be educated on
his basal insulin dose, and is likely to require higher mealtime signs, symptoms, and treatment of hypoglycemia, and in-
insulin doses than he is currently taking. structed to call the clinic if he experiences any hypoglycemia.
Rodney is not likely to be obtaining benefit from glyburide
while taking high-dose insulin therapy, and this medication, Followup
which also has an adverse cardiovascular safety profile, should Rodney begins his U500 insulin regimen in combination with
be discontinued. Rodney is likely to be obtaining benefit from dulaglutide and continues metformin. He faxes his BGs weekly
the insulin-sparing effects of metformin as well as nonglycemic for review. On his initial U500 insulin doses, glucose readings
anti-inflammatory benefits of this agent, and should continue improve and range from 103300 mg/dL. Rodney tolerates
metformin. dulaglutide (0.75 mg). During the next several weeks, U500
Liraglutide would have been a good adjunct therapy for
insulin doses are increased to 125 U three times daily before
Rodney in that it might have lowered his insulin dose, contrib-
meals (25 U markings on insulin syringe) and dulaglutide is
uted to glycemic control, and helped mitigate insulin-
increased to 1.5 mg per week.
associated weight gain, but he was unable to tolerate it. He
might be able to tolerate another GLP-1 RA, however, and
combining a once-weekly GLP-1 RA with an intensive insulin Discussion of Rodneys Insulin Doses
regimen and metformin would be the optimal treatment regi- Rodney is now on a total of 375 U of insulin (as U500 insulin)
men for Rodney. Rodney is also a candidate for addition of an daily. He is on three insulin injections daily, and a GLP-1 RA
SGLT-2 inhibitor, especially in light of his elevated BP. Rod- weekly. He is now absorbing his insulin doses and insulin
neys BP management should also be intensified. titration is now resulting in glycemic improvement. When he
was on glargine, despite insulin up-titration, Rodney did not
show further glycemic improvement owing to the poor absorp-
Question 8
tion of high doses of glargine.
Is Rodney a candidate for U500 insulin?
Rodney currently uses 160 U of insulin per day, but is
presently underinsulinized. He also is showing poor Three-Month Followup
absorption/action of his basal insulin. He is likely to require Rodneys BGs are now close to 100% at target (range: 94 180
much more insulin than he is presently taking, and thus is mg/dL) on 125 U of U500 insulin (25 U markings on insulin
likely to require greater than 200 U of insulin per day when syringe) before meals three times daily, metformin and
optimally insulinized. He is thus an appropriate candidate dulaglutide, 1.5 mg per week. Rodney has not had any episodes
for U500 insulin. of hypoglycemia. He is pleased with his glycemic control and
reports feeling better with more energy. His weight is up 10 lbs
to 290 lbs; HbA1c, 5.9%.
Question 9
How would we convert Rodneys insulin regimen to a U500
insulin regimen? Question 12
Begin U500 insulin at 50 U (10 U markings) on an insulin What additional treatment options (if any) are appropriate for
syringe before breakfast and 45 U (9 U markings) before lunch Rodney at this time?

The Endocrine Society. Downloaded from press.endocrine.org by [${individualUser.displayName}] on 12 January 2017. at 12:19 For personal use only. No other uses without permission. . All rights reserved.
ENDO 2016 DIABETES AND GLUCOSE METABOLISM 95

Rodney is doing well from a glycemic standpoint, although 11. Reutrakal S, Wroblewski K, Brown RL. Clinical use of U-500 regular insulin:
Review and meta-analysis. J Diab Sci Technol. 2012;6(2):412-420.
he has gained weight with improved glycemic control. Appro-
12. Lane WS, Weinrib SL, Rappaport JM, Przestrzelski T. A prospective trial
priate additional therapies to consider include addition of an of U500 insulin delivered by Omnipod in patients with type 2 diabetes
SGLT-2 inhibitor to reduce insulin dose and help with weight mellitus and severe insulin resistance. Endocr Pract. 2010;16(5):778-784.
control. If Rodney would like to use an insulin pump, he is an 13. Ovalle F. Clinical approach to the patient with diabetes mellitus and very
high insulin requirements. Diabetes Res Clin Pract. 2010;90:231-242.
appropriate candidate for either U500 insulin in an insulin 14. Teff KL, et al. Antipsychotic-induced insulin resistance and postprandial
pump (off-label use), or for U200 lispro (off-label and not hormonal dysregulation independent of weight gain or psychiatric disease.
available in vials as of the time of this writing) via insulin American Diabetes Association, Chicago, Illinois, June 2125, 2013; Ab-
stract 150-OR.
pump. Additional therapies to consider include weight loss 15. Khan M, et al. Pharmacokinetic and pharmacodynamics properties of
medications or bariatric surgery. regular U-500 insulin in healthy subjects. Diabetes. 2007;56 (suppl):1294.
16. Hood RC, Arakaki RF, Wysham C, et al. Two treatment approaches for
human regular U-500 insulin in patients with type 2 diabetes not achieving
REFERENCES adequate glycemic control on high-dose U-100 insulin therapy with or
1. Lane WS, Cochran EK, Jackson JA, et al. High-dose insulin therapy: Is it without oral agents: A randomized, titration-to-target clinical trial. Endocr
time for U-500 insulin? Endocr Pract. 2009;15(1):71-79. Pract. 2015;21(7):782-794.
2. Rodbard H, Handelsman Y, Gough S, et al. American Association of 17. Rosselli JL, Archer SN, Lindley NK, et al. J Pharm Technol. 2015;10.1177/
Clinical Endocrinologists 21st Scientific Meeting and Clinical Congress, 8755 122515584193.
Philadelphia, PA, May 23-27, 2012 (Abstract 241). 18. Toujeo package insert.
3. Gough SC, Bhargava A, Jain R, Mersebach H, Rasmussen S, Bergenstal 19. European Medicines Agency/Science Medicines Health. Committee for
RM. Low-volume insulin degludec 200 units/ml once daily improves Medicinal Products for Human Use. Assessment Report. 24 July, 2014.
glycemic control similarly to insulin glargine with a low risk of hypogly- Accessed from: http://www.ema.europa.eu/docs/en_GB/document_library/
cemia in insulin-naive patients with type 2 diabetes: A 26-week, random- EPAR_-_Assessment_Report_-_Variation/human/000088/WC500176634.pdf.
ized, controlled, multinational, treat-to-target trial: The BEGIN LOW 20. Heise T, Nosek L, Bttcher SG, Hastrup H, Haahr H. Ultra-long-acting
VOLUME trial. Diabetes Care. 2013;36:2536-2542. insulin degludec has a flat and stable glucose-lowering effect in type 2
4. Data from 2011 US Roper Patient Diabetes Market Study, provided by diabetes. Diabetes Obes Metab. 2012;14:944-950.
Gfk Custom Research LLC, with permission. 21. Korsatko S, Deller S, Koehler G, et al. A comparison of the steady-state
5. Schofield CJ, Sutherland C. Disordered insulin secretion in the development pharmacokinetic and pharmacodynamic profiles of 100 and 200 U/mL
of insulin resistance and type 2 diabetes. Diabet Med. 2012;29:972-979. formulations of ultra-long-acting insulin degludec. Clin Drug Investig.
6. Binder C, Lauritzen T, Faber O, Pramming S. Insulin pharmacokinetics. 2013;33:515-521.
Diabetes Care. 1984(7):188. 22. Heller S, Buse J, Fisher M, et al. Insulin degludec, an ultra-longacting
7. Jeitler K, Horvath K, Berghold A, et al. Continuous subcutaneous basal insulin, versus insulin glargine in basal-bolus treatment with meal-
insulin infusion versus multiple daily insulin injections in patients with time insulin aspart in type 1 diabetes (BEGIN Basal-Bolus Type 1): A
diabetes mellitus: Systematic review and meta-analysis. Diabetelogia. phase 3, randomised, open-label, treat-to-target non-inferiority trial. Lan-
2008;51(6):941-951. cet. 2012;379:1489-1497.
8. Reznik Y, Cohen O, Aronson R. Insulin pump treatment compared with 23. Singh S, Loke YK, Furberg CD. Thiazolidinediones and heart failure: A
multiple daily injections for treatment of type 2 diabetes (OpT2Mise): A teleo-analysis. Diabetes Care. 2007;30(8):2148-2153.
randomized open-label controlled trial. Lancet. 2014;37:1499-1508. 24. Lane W, Weinrib S, Rappaport J. The effect of liraglutide added to U-500
9. Gudbjornsdottir S, et al. European Association for the Study of Diabetes insulin in patients with type 2 diabetes and high insulin requirements.
Annual Meeting, September 18, 2014 (Abstract 196). Diabet Technol Ther. 2011;13(5):592-595.
10. Eby EL, Wang P, Curtis BH, et al. Cost, healthcare resource utilization, 25. Lane W, Weinrib S, Rappaport J, Hale C. The effect of addition of
and adherence of individuals with diabetes using U-500 or U-100 insulin: liraglutide to high-dose intensive insulin therapy: A randomized prospec-
A retrospective database analysis. J Med Econ. 2013;16(4):529-538. tive trial. Diab Obes Metab. 2014;16:827-832.

The Endocrine Society. Downloaded from press.endocrine.org by [${individualUser.displayName}] on 12 January 2017. at 12:19 For personal use only. No other uses without permission. . All rights reserved.
96 ENDO 2016 MEET-THE-PROFESSOR CLINICAL CASE MANAGEMENT

Exercise Resistance in Type 2 Diabetes

M14 are expected to increase, with the most recent projections by the
Presented, April 1 4, 2016 Center for Disease Control predicting that as many as 40% of
Americans will have type 2 diabetes within their lifetime (8).
Type 2 diabetes is characterized by insulin resistance and
Lauren M. Sparks, PhD, Florida Hospital/Sanford- dysregulated blood glucose levels. Impaired glycogen synthesis
Burnham Medical Research Institute, Translational and mitochondrial dysfunction, as well as ectopic lipid accu-
Research Institute for Metabolism and Diabetes, Orlando,
mulation, are also associated with the disorder (9). Although it
Florida 32804, E-mail: lauren.sparks@flhosp.org
is well established that regular exercise can prevent or delay
the onset of type 2 diabetes (4, 10), two main risk factors in the
EXERCISE RESISTANCE development of type 2 diabetes are obesity and a lack of
Historical Perspective physical activity (11, 12). Thus, increased sedentary behavior
Classic genetic studies from the 1980s by Bouchard and col-
and prevalence of obesity have popularized exercise interventions
leagues have shown that the extent of the exercise training
as both an investigative tool of the health benefits associated with
response is largely heritable (1-4); whereby individuals within
regular exercise and as a feasible lifestyle modification (13). All of
a family respond more similarly than those between families.
these factors have led to a movement centered on the exercise is
This suggests there is either a sequence variation or an
medicine philosophy. In support, randomized controlled clinical
epigenetic modification that dictates this exercise response.
In addition to this work, the turn of the century brought more trials show that exercise training improves glycemic control in
sophisticated approaches to measuring metabolic endpoints in individuals with type 2 diabetes (14).
response to exercise training interventions. It was not until
2008 that Lawrence Mandarino (28) coined the phrase exer- BARRIERS TO OPTIMAL PRACTICE
cise resistance when referring to one of his study populations Management of type 2 diabetes with exercise is paramount to
that failed to increase expression of key muscle substrate me- ending its epidemic. However, exercise benefits most but not
tabolism genes in response to an acute bout of exercise. Later all individuals with type 2 diabetes. As a result, we are facing
in 2009 and 2012, Roden and colleagues (5, 29) found that a a growing subset of the population that is essentially exercise
genetic predisposition (ie, the relatives of individuals with type
resistant. We and others have found that 1520% of indi-
2 diabetes) could actually modify the individual response of an
viduals with type 2 diabetes fail to improve their glucose
in vivo measurement of muscle metabolism to chronic exercise
homeostasis, insulin sensitivity, and muscle mitochondrial den-
training. These investigators found that a single nucleotide
sity after supervised exercise training interventions (14-17).
polymorphism in the NDUFB6 gene, which encodes for a
protein in the Complex I subunit of the mitochondrial electron Interindividual variability (of the metabolic response to exer-
transport system, reduces their exercise response of in vivo cise) in individuals with type 2 diabetes is large. Genetic
mitochondrial function (5). Also in 2012, exciting new evi- variability may be one explanation (18); however, ambient
dence suggested that DNA hypomethylation of the promoter hyperglycemia (19), pancreatic -cell function (20), and medi-
region in genes known to be involved in muscle metabolism cation use (21) may also contribute these disparities. As such,
(PGC1, PPAR/, PDK4) is an early event in contraction- prescribing exercise as a prevention and/or treatment to indi-
induced gene activation in skeletal muscle (6). In 2013, I was viduals at risk for development of or living with type 2 diabetes
awarded a grant from the American Diabetes Association to becomes complex.
investigate the underlying (epi)genetic mechanisms responsible
for the interindividual variations in exercise responses in a
LEARNING OBJECTIVES
population of men and women with type 2 diabetes. The study
As a result of participating in this session, learners should be
will be completed in June 2016.
able to:
Understand the interindividual variability in the response
SIGNIFICANCE OF THE CLINICAL PROBLEM to exercise in terms of glucose control and other
By 2050 the number of people with diagnosed diabetes in the metabolic factors among individuals with type 2
United States will reach 29 million, or roughly more than 30%
diabetes.
of the population (7). The American Diabetes Association has
reported a steady increase in the incidence of diabetes and the Begin to consider creative, efficient, and cost-effective
tremendous financial burden of this rapidly growing epidemic, ways in which you as a practitioner can capture your
which is estimated to cost $245 billion in 2012. These numbers patients adherence to a prescribed exercise regimen.

The Endocrine Society. Downloaded from press.endocrine.org by [${individualUser.displayName}] on 12 January 2017. at 12:19 For personal use only. No other uses without permission. . All rights reserved.
ENDO 2016 DIABETES AND GLUCOSE METABOLISM 97

Table 1. Clinical Characteristics of Nonresponders and 16-24 hours after the last bout of exercise, whereas persons in
Responders the other two exercise groups maintained increased insulin
Clinical P sensitivity up to 2 weeks after the last exercise bout (15). Thus,
Characteristics Responders Non-Responders Value persistence of improved insulin action may be obtained with
Age, y 57.0 3.35 55.7 2.13 NS aerobic exercise, depending on the characteristicsand not
Sex, M/F 4/5 5/4 NS necessarily the volume of the training program.
Height, cm 168.04 2.85 171.0 1.83 NS In a more recent study, similar results were observed in
% HbA1c 12.92 3.38 3.16 1.93 .0008 response to 26 weeks of either aerobic or resistance training in
% Body fat 6.76 2.3 0.17 1.27 .0258 relatives of individuals with T2D. Increased in vivo ATP syn-
BMI 3.31 1.14 0.77 0.56 .0052 thase flux was observed in 86% of individuals with the previ-
% mtDNA 50.68 12.76 23.39 10.87 .0004 ously identified responder allele NDUFB6 single nucleotide
polymorphosm (rs540467 G/G); however, only 25% of indi-
Abbreviation: F, female; M, male; NS, not significant; viduals with the nonresponder allele (rs540467 G/A) showed
BMI, body mass index; %, % change after 9 months of ex- an increase in in vivo ATP synthase flux (24). Pre-existing
ercise training. hyperglycemia has also been shown to blunt favorable HbA1c
responses to exercise in individuals with type 2 diabetes (Fig-
ure 1) (25). In addition to genetic variability and hyperglyce-
STRATEGIES FOR MANAGEMENT OF mia, medication use is another important factor when assessing
EXERCISE RESISTANCE an individuals response or resistance to exercise. Braun and
Background colleagues have shown that the biguanide metformin artificially
A recent study by our laboratory demonstrated that of the 42 increases heart rate (thereby reducing prescribed exercise
individuals with type 2 diabetes that underwent nine months of workloads), and exercise interferes with the glucose-lowering
exercise training, nine of them showed no improvements in 1) effects of metformin in individuals with type 2 diabetes (26).
hemoglobin A1c (HbA1c), 2) body mass index (BMI), 3) percent Given that biguanides have recently been identified as a new
body fat, and 4) muscle mitochondrial density (17) (Table 1). class of mitochondrial Complex I and ATP synthase inhibitors
These four factors have been previously shown to be significantly (27), it is uncertain how will this affect exercise prescription to
augmented in response to the exercise training among this entire individuals with type 2 diabetes currently taking metformin and
cohort (14, 16). These so-called Non-Responders were also distin- other antidiabetic drugs.
guished from Responders at baseline (pretraining) by the tran-
scriptional (mRNA) profiles of their muscle tissue. Genes in- Clinical Features and Diagnostic Approach
volved in substrate metabolism were among those found to be Exercise resistance is a relative term in clinical practice. In
significantly lower before training in Nonresponders. These find- terms of the clinical features of exercise resistance, it is impor-
ings support the premise of exercise resistance and also validate tant to use a predefined set of metabolic criteria by which
the use of this metabolic panel examined herein as a means of responses and lack thereof (ie, resistance) to exercise are
defining exercise resistance in individuals with type 2 diabetes. evaluated. The most clinically relevant outcomes are glucose
In 2012, Gregg et al (22) showed that an intensive lifestyle control (HbA1c), fasting blood glucose, and blood glucose
intervention program that incorporated exercise, diet manage- following a 2-hour oral glucose tolerance test (OGTT). In our
ment, and sustained frequent (twice monthly) counseling for 4 clinical and translational research settings, we use a broader
years led to nearly complete remission in 9% of persons (n spectrum of metabolic endpoints, yet all of our metabolic
2241). However, only 2% of persons in a less involved measurements relate (to some degree) with the three aforemen-
control group (3 sessions/y; n 2262) exhibited type 2 diabe- tioned clinical outcomes.
tes remission. Partial or complete remission was defined as a
return to a prediabetic or nondiabetic level of glycemia (fasting Management
plasma glucose 126 mg/dL and HbA1c 6.5%) with no Exercise is a first line of defense for practitioners when dealing
antihyperglycemic medication (22). These findings suggest that with their patients with type 2 diabetes. Unfortunately, when
the volume of exercise is the key to metabolic improvements in these patients are prescribed an exercise regimen to lower their
individuals with type 2 diabetes. However, in the US National HbA1c and show no signs of improvement at a follow-up visit,
Institutes of Health multicenter STRRIDE trial (23), investiga- the conclusion tends to be one of noncompliance. As such,
tors examined the effects of three 8-month aerobic exercise medication is usually prescribed or dosage increased at this
training interventions: 1) low volume/moderate intensity, 2) stage. However, it is possible that these patients are compliant
low volume/vigorous intensity, and 3) high volume/vigorous and doing their prescribed exercise/physical activity but are not
intensity (15). They showed that individuals in the low reaping the metabolic rewards due to some barrier (genetics,
volume/vigorous intensity group did not maintain the initial pre-existing hyperglycemia, medication use). As such, it is
exercise-mediated increase in insulin sensitivity observed imperative to find an efficient and cost-effective way for prac-

The Endocrine Society. Downloaded from press.endocrine.org by [${individualUser.displayName}] on 12 January 2017. at 12:19 For personal use only. No other uses without permission. . All rights reserved.
98 ENDO 2016 MEET-THE-PROFESSOR CLINICAL CASE MANAGEMENT

Figure 1. Pretraining and exercise-induced changes. Individuals with impaired glucose tolerance or type 2 diabetes
underwent 12-16 wk moderate-intensity exercise training, 5 d/wk, 60 min/d. Individual subject data points are plotted on
both panels; the x-axis represents the pretraining variable, and the y-axis indicates the exercise-induced change such
that the data points above the axis indicate an exercise-induced increase and vice versa. Open circles represent impaired
glucose-tolerant subjects, and open triangles represent subjects with type 2 diabetes. The solid line represents the
regression curve, and the dotted line represents the 95% confidence interval. A, There was a nonlinear quadratic
relationship between pretraining 2-h OGTT level and the training-induced change in 2-h OGTT level (y 0.06x2
1.5x 7.6 [r2 0.26; P .06] [N 105]). For every 1 mmol/L increase in pretraining 2-h glucose level above 13.1
mmol/L (the inflection point of the curve), there was a 0.2-mmol/L loss of improvement in 2-h glucose level following
exercise (to convert glucose to milligrams per deciliter, divide by 0.0555). B, There was also a nonlinear quadratic
relationship between pretraining HbA1c level and the training-induced change in HbA1c level (y 0.31x2 3.8x
11.7 [r2 0.33; P .02] [n 52]). For every1-percentage-point increase in pretraining HbA1c level above 6.2% (the
inflection point of the curve), there was a 0.2-percentage-point loss of improvement in HbA1c level following exercise.

titioners to monitor their patients compliance with their pre- tizers vs insulin)? With the rapid increase in the prevalence of
scribed exercise/physical activity. I realize that practitioners are type 2 diabetes, it is imperative that translational studies are
extremely busy and time with each patient is limited, so finding diverted toward the elucidation of the mechanisms behind
simple ways to track compliance without adding burden to the exercise resistance and that researchers work together with
practitioner is ideal. One option would be the use of free practitioners to develop novel treatments that will circumvent
physical activity smartphone apps to record activity of the the programmed failure to respond in a significant proportion
patient as proof of compliance. These apps are free and most, of patients with type 2 diabetes.
but not all, people have access to smartphones. Therefore, we
must continue to search for ways to track patients physical
activity so that practitioners can identify those patients who are CASE WITH QUESTIONS
Non-Responders/Low-Responders to try other treatment strate- A 57-year old man with a family history of type 2 diabetes,
gies such as different medications that can consistently lower HbA1c of 6.1%, and fasting blood glucose of 115 mg/dL
hyperglycemia and not interfere with the medications beneficial comes to your office for a routine checkup. The patient is not
effects. Prescribing other types of exercise such as weight training currently taking any medication but reports that he walks 5
or exercise at higher intensities for shorter duration are other times per week in the evenings with his wife and has done so
alternatives in the evolution of this newly identified population of for the past 3 months. The patient is otherwise in general good
exercise resistant individuals with type 2 diabetes. health.
What is the most appropriate management?
MAIN CONCLUSIONS A. Diagnose with type 2 diabetes and prescribe 500 mg of
A lack of positive metabolic result does not necessarily reflect metformin once a day.
lack of effort; therefore, clinical studies aimed at diagnosing B. Diagnose with type 2 diabetes, prescribe 500 mg of
exercise resistance prior to starting an exercise program are metformin three times a day, and schedule to come
critical, and better predictive models and clinical biomarkers back in 2 months.
are needed. Traditional clinical approaches also need to be C. Diagnose with type 2 diabetes, prescribe 3 days of
re-examined: Does strict glycemic control allow the muscle to walking or other physical activity for at least 30
respond favorably in exercise resistance patients? Does minutes per day using a tracking log, and schedule to
medical management need to be retargeted (eg, insulin sensi- come back in 2 months.

The Endocrine Society. Downloaded from press.endocrine.org by [${individualUser.displayName}] on 12 January 2017. at 12:19 For personal use only. No other uses without permission. . All rights reserved.
ENDO 2016 DIABETES AND GLUCOSE METABOLISM 99

D. Diagnose with type 2 diabetes, prescribe insulin, and REFERENCES


schedule to come back in 2 months. 1. Feitosa MF, Rice T, Rankinen T, et al. Evidence of QTLs on chromosomes
13q and 14q for triglycerides before and after 20 weeks of exercise training:
The HERITAGE Family Study. Atherosclerosis. 2005;182:349-360.
DISCUSSION OF CASE AND ANSWER 2. Lakka TA, Lakka HM, Rankinen T, et al. Effect of exercise training on
plasma levels of C-reactive protein in healthy adults: The HERITAGE
This case very simply deals with an issue that many practitio- Family Study. Eur Heart J. 2005;26:2018-2025.
ners face on a daily basis: a patient with family history of 3. An P, Teran-Garcia M, Rice T, et al. Genome-wide linkage scans for
diabetes and an early indication of deteriorating blood glucose prediabetes phenotypes in response to 20 weeks of endurance exercise
training in non-diabetic whites and blacks: The HERITAGE Family Study.
control who self reports habitual physical activity. I realize that
Diabetologia. 2005;48:1142-1149.
the quickest and most robust way to get their blood glucose 4. Boule NG, Weisnagel SJ, Lakka TA, et al. Effects of exercise training on
under control is to prescribe medication, usually metformin, glucose homeostasis: The HERITAGE Family Study. Diabetes Care.
and it does the job, as well as assume that the patient is not 2005;28:108-114.
5. Kacerovsky-Bielesz G, Chmelik M, Ling C, et al. Short-term exercise
being truthful about his habitual physical activity. However, training does not stimulate skeletal muscle ATP synthesis in relatives of
given what you have now learned about exercise resistance, in humans with type 2 diabetes. Diabetes. 2009;58:1333-1341.
addition to Metformins potentially negative interaction with 6. Barres R, Yan J, Egan B, et al. Acute exercise remodels promoter meth-
ylation in human skeletal muscle. Cell Metab. 2012;15:405-411.
exercise and its effect on heart rate, prescribing medication 7. Boyle JP, Thompson TJ, Gregg EW, Barker LE, Williamson DF. Projec-
may not be the best answer. Exercise is a great alternative (to tion of the year 2050 burden of diabetes in the US adult population:
medication) as a first line of defense, particularly in a dynamic modeling of incidence, mortality, and prediabetes prevalence.
Population Health Metrics. 2010;8:29.
medication-nave patient such as this one.
8. Gregg EW, Zhuo X, Cheng YJ, Albright AL, Narayan KM, Thompson TJ.
The patient should be educated about the fact that at such an Trends in lifetime risk and years of life lost due to diabetes in the USA,
early stage in the disease process, exercise alone can be used to 1985-2011: A modelling study. The lancet. Diabetes, endocrinology.
2014;2:867-874.
manage his diabetes. I would convey to him that adherence to
9. Strasser B, Pesta D. Resistance training for diabetes prevention and
physical activity at least 3 times per week for at least 30 therapy: Experimental findings and molecular mechanisms. Biomed Res
minutes each time is sufficient to control blood glucose in most Int. 2013;2013:805217.
patients with type 2 diabetes, but that sometimes people do not 10. Boule NG, Haddad E, Kenny GP, Wells GA, Sigal RJ. Effects of exercise
on glycemic control and body mass in type 2 diabetes mellitus: A meta-
respond to the first type of exercise that they try or that analysis of controlled clinical trials. JAMA. 2001;286:1218-1227.
medications can even sometimes interfere with exercise. As 11. Avery L, Flynn D, van Wersch A, Sniehotta FF, Trenell MI. Changing
mentioned previously, you should ask the patient to use a free physical activity behavior in type 2 diabetes: A systematic review and meta-
analysis of behavioral interventions. Diabetes Care. 2012;35:2681-2689.
app on his smartphone to track his physical activity and show it 12. Taylor R. Pathogenesis of type 2 diabetes: Tracing the reverse route from
to you at the next visit. cure to cause. Diabetologia. 2008;51:1781-1789.
Should this patient return in 2 months with proof (from his 13. Uusitupa M, Louheranta A, Lindstrom J, et al. The Finnish Diabetes
Prevention Study. Br J Nutr. 2000;83 Suppl 1:S137S142.
app) that he complied with the prescribed physical activity 14. Church TS, Blair SN, Cocreham S, et al. Effects of aerobic and resistance
regimen and his HbA1c is 6.1%, then consider that he may training on hemoglobin A1c levels in patients with type 2 diabetes: A
be exercise resistant and in need of an alternative type of randomized controlled trial. JAMA. 2010;304:2253-2262.
15. Bajpeyi S, Tanner CJ, Slentz CA, et al. Effect of exercise intensity and
exercise or intensity. Variations in time and intensity, as well
volume on persistence of insulin sensitivity during training cessation.
as types of physical activity, should also be recommended to J Appl Physiol (1985). 2009;106:1079-1085.
improve this adherence. The best exercise for anyone is the 16. Sparks LM, Johannsen NM, Church TS, et al. Nine months of combined
one that they will do! Another important take-home message training improves ex vivo skeletal muscle metabolism in individuals with
type 2 diabetes. J Clin Endocrinol Metab. 2013;98:1694-1702.
is that genetics do play a role in development and manage- 17. Stephens NA, Xie H, Johannsen NM, Church TS, Smith SR, Sparks LM.
ment of type 2 diabetes. Research in this area of genetic A transcriptional signature of exercise resistance in skeletal muscle of
predictors of exercise response is rapidly advancing with the individuals with type 2 diabetes mellitus. Metabolism. 2015;64:999-1004.
18. Ruchat SM, Rankinen T, Weisnagel SJ, et al. Improvements in glucose
ultimate goal of shifting the paradigm to allow interventions homeostasis in response to regular exercise are influenced by the PPARG
to be targeted to those most likely to benefit and identify Pro12Ala variant: Results from the HERITAGE Family Study. Diabetologia.
novel approaches to treat those who do not. 2010;53:679-689.
19. Malin SK, Kirwan JP. Fasting hyperglycaemia blunts the reversal of
The future of T2D research is certainly moving in the impaired glucose tolerance after exercise training in obese older adults.
direction of personalized or precision medicine. Current re- Diabetes Obes Metab. 2012;14:835-841.
search in this area continues to discover genes and signaling 20. Dela F, von Linstow ME, Mikines KJ, Galbo H. Physical training may
enhance beta-cell function in type 2 diabetes. Am J Physiol Endocrinol
pathways that differ among even the most homogenous groups
Metab. 2004;287:E1024 E1031.
of individuals. These differences ultimately lead to variations 21. Malin SK, Gerber R, Chipkin SR, Braun B. Independent and combined
in their physiological responses to medications and treatments. effects of exercise training and metformin on insulin sensitivity in indi-
It is therefore imperative that we use these findings to develop viduals with prediabetes. Diabetes Care. 2012;35:131-136.
22. Gregg EW, Chen H, Wagenknecht LE, et al. Association of an intens-
strategies to exploit these differences among individuals to ive lifestyle intervention with remission of type 2 diabetes. JAMA.
maximize each persons response to a prevention or treatment. 2012;308:2489-2496.

The Endocrine Society. Downloaded from press.endocrine.org by [${individualUser.displayName}] on 12 January 2017. at 12:19 For personal use only. No other uses without permission. . All rights reserved.
100 ENDO 2016 MEET-THE-PROFESSOR CLINICAL CASE MANAGEMENT

23. Kraus WE, Torgan CE, Duscha BD, et al. Studies of a targeted risk diabetes: examining treatment modality interactions. Diabetes Care.
reduction intervention through defined exercise (STRRIDE). Med Sci 2011;34:1469-1474.
Sports Exerc. 2001;33:1774-1784. 27. Bridges HR, Jones AJ, Pollak MN, Hirst J. Effects of metformin and other
24. Kacerovsky-Bielesz G, Kacerovsky M, Chmelik M, et al. A single nucle- biguanides on oxidative phosphorylation in mitochondria. Biochem J.
otide polymorphism associates with the response of muscle ATP synthesis 2014;462:475-487.
28. De Filippis E, Alvarez G, Berria R, et al. Insulin-resistant muscle is
to long-term exercise training in relatives of type 2 diabetic humans.
exercise resistant: evidence for reduced response of nuclear-encoded mi-
Diabetes Care. 2012;35:350-357.
tochondrial genes to exercise. Am J Physiol Endocrinol Metab.
25. Solomon TP, Malin SK, Karstoft K, Haus JM, Kirwan JP. The influ- 2008;294:E607-E614.
ence of hyperglycemia on the therapeutic effect of exercise on glyce- 29. Kacerovsky-Bielesz G, Kacerovsky M, Chmelik M, et al. A single nucle-
mic control in patients with type 2 diabetes mellitus. JAMA Intern Med. otide polymorphism associates with the response of muscle ATP synthesis
2013;173:1834-1836. to long-term exercise training in relatives of type 2 diabetic humans.
26. Boule NG, Robert C, Bell GJ, et al. Metformin and exercise in type 2 Diabetes Care. 2012;35(2):350-357.

The Endocrine Society. Downloaded from press.endocrine.org by [${individualUser.displayName}] on 12 January 2017. at 12:19 For personal use only. No other uses without permission. . All rights reserved.
ENDO 2016 DIABETES AND GLUCOSE METABOLISM 101

Diabetic Neuropathies

M15 lence rates increase with diabetes duration from 30% to up to


Presented, April 1 4, 2016 50% after 10 years of diabetes (6). In addition, recent evidence
from youth with T1D and T2D participating in the SEARCH
for Diabetes in Youth study showed that prevalence rates for
Rodica Pop-Busui, MD, PhD. Professor of Internal DSPN approached rates reported in adult populations, particu-
Medicine, Division of Metabolism, Endocrinology, and larly in youth with T2D (7). CAN prevalence is low in newly
Diabetes, Department of Internal Medicine, University of diagnosed patients with, but increases substantially with diabe-
Michigan, Ann Arbor, Michigan 48105, E-mail: tes duration (6), and prevalence rates of at least 30% were
rpbusui@umich.edu observed in the DCCT/EDIC cohort after 20 years of diabetes
duration (4). In T2D, the prevalence of CAN also increases
HISTORICAL OVERVIEW with diabetes duration and is present in at least 60% of patients
The presence of a diabetic sensorimotor polyneuropathy in with T2D after 15 years (8).
patients with diabetes has been described as early as 1885.
Between 1885 and 1890, Althaus, Leyden, and Bruns have BARRIERS TO OPTIMAL PRACTICE
described various neurological deficits complicating diabetes, A timely and appropriate diagnosis is important, because it
such as proximal diabetic, truncal, median, and ulnar neuropa- may have prognostic implications, but it is often overlooked in
thies. Leyden eventually reported that the various manifesta- practice. Clinicians are often unaware of simple, sensitive and
tions associated with diabetic polyneuropathy appear to include cost-effective algorithms to diagnose DN, which will be dis-
3 main clinical subtypes (painful, ataxic, and paralytic), which cussed. Despite the recent major advances in elucidating the
are associated with the presence of peripheral nerve degenera- pathogenesis of DN, there remains a lack of treatment options
tion on autopsies studies of these patients. Since then, it be- that effectively target the natural history of DSPN (9). Several
came apparent that diabetic neuropathy (DN) is a complex pathogenetic pharmacotherapies have been investigated, but
disorder with a wide spectrum of clinical manifestations evidence from randomized clinical trials (RCTs) for a disease
modifying therapy is disappointing (9, 10). Pain is often the
SIGNIFICANCE OF THE CLINICAL PROBLEM presenting symptom for many patients, and its treatment had
DNs are the most prevalent chronic complications of diabe- been challenging. Despite multiple classes of agents with vari-
tes, with multiple manifestations, consistent risk factors, and ous degrees of effectiveness in the treatment of neuropathic
complex pathogenetic mechanisms. DNs are a major cause pain, opioids continue to be too early prescribed, creating the
of disability, high mortality, and poor quality of life (1, 2). premise for abuse, addiction, and other psychosocial issues.
For instance, patients with distal symmetrical polyneurop- For these reasons, opioids should not be used before failure of
athy (DSPN) have a 25% cumulative risk of a lower extrem- other agents that do not have these associated concerns and
ity amputation. The 3-year survival in patients with DNs is referral to a pain clinic should be considered before opioid use.
20% less than in age- and sex-matched diabetic patients
without DN (3). LEARNING OBJECTIVES
There are multiple types of DNs, and each presents with a As a result of participating in this session, learners should be
unique array of symptoms and clinical signs. Among the vari- able to:
ous forms of DN DSPN and diabetic autonomic neuropathy Learn simple and effective diagnostic strategies for
(AN) are by far the most prevalent and most current evidence patients with DN.
refers to these forms. Understand the management strategies of patients with DN.
Epidemiologic studies have focused upon the most common,
DSPN, and the most common form of AN, cardiovascular AN
(CAN). Evidence from several large observational cohorts and STRATEGIES FOR DIAGNOSIS AND
particularly from the Diabetes Control and Complications Trial MANAGEMENT
(DCCT) and its observational follow-up Epidemiology of Dia- Distal Symmetrical Polyneuropathy
betes Interventions and Complications (EDIC) study suggests The diagnosis of DSPN is principally a clinical one and com-
that DSPN occurs in at least 20% of type 1 diabetes (T1D) prises documentation of characteristic symptoms and signs.
patients after 20 years of disease duration (4). Although the
prevalence of DSPN is highly variable in studies of type 2 Symptoms
diabetic (T2D) patients, DSPN may be present in at least Patients usually present with symptoms that include pain,
10%15% of newly diagnosed patients with T2D (5). Preva- dysesthesias (unpleasant abnormal sensations of burning and

The Endocrine Society. Downloaded from press.endocrine.org by [${individualUser.displayName}] on 12 January 2017. at 12:19 For personal use only. No other uses without permission. . All rights reserved.
102 ENDO 2016 MEET-THE-PROFESSOR CLINICAL CASE MANAGEMENT

tingling), and a stabbing, electric shock-like sensation with an mm Hg) or diastolic (10 mm Hg) blood pressure (BP) in
insensate, numb foot due to predominant large fiber loss. Pa- response to a postural change from supine to standing is usu-
tients may also frequently state their feet feel like they are ally easy to document in the office, but it is a late manifestation
wrapped in wool, or they are walking on thick socks. of CAN (8). In many (but not all) cases of CAN, there is no
compensatory increase in the heart rate, despite hypotension.
Signs Standardized cardiovascular autonomic reflex tests, which as-
Signs may be documented with simple clinical tests that in- sess the changes in the R-R interval on ECG recordings with
clude: assessment of small fiber function by evaluating pin- deep breathing, standing, and Valsalva maneuver are consid-
prick sensation, and assessment of large fiber function by ered to date the gold standard for clinical autonomic testing (8,
evaluating the vibration threshold using a 128-Hz tuning fork, 10) but reserved mainly for research. Screening for symptoms
light touch perception using a 10-g monofilament, and ankle and signs of CAN should be considered in patients with more
reflexes. Assessments should follow the typical DSPN pattern, advanced disease (10 y of duration; history of poor glucose
starting distally (the dorsal aspect of the hallux) on both sides control, presence of other chronic complications or forms of
and move proximally until a sensory threshold is identified. DN, additional risk factors). Excluding other comorbidities or
The 10-g monofilament is a useful clinical tool for detecting in drug effects/interactions that could mimic CAN should be
general more advanced neuropathy and identifying patients at performed.
increased risk of ulceration and amputation. Combining at least
2 examinations will increase the sensitivity and specificity of Diabetic Gastropathy (Gastroparesis)
detecting DSPN as demonstrated by few available clinical Gastroparesis may be clinically silent in most cases. When
instruments (1113). clinically manifest may present with symptoms and signs such
Electrophysiological testing or referral to a neurologist is as anorexia, nausea, vomiting, dyspepsia. However, these are
rarely needed for screening, except in situations where the nonspecific, resemble many other conditions, and in general do
clinical features are atypical, the diagnosis is unclear and a not correspond with severity of gastropathy. A splashing sound
different etiology is suspected (14). Atypical features, which on abdominal succussion, 1 hour after a meal, is highly sug-
warrant referral, include motor greater than sensory neuropa- gestive. Gastroparesis may be a cause of high glucose variabil-
thy, asymmetry of symptoms and signs and rapid progression. ity and/or unexplained severe hypoglycemia, with an apparent
Clinicians should screen patients after 5 years duration of lack of relationship between blood glucose excursions and the
T1D, all patients with T2D at diagnosis and thereafter annually known onset and duration of actions of the hypoglycemic
and should consider screening patients with metabolic syn- agents. Exclusion of organic causes of gastric outlet obstruc-
drome, insulin resistance, and/or impaired glucose tolerance. tion or peptic ulcer disease (with esophagogastroduodenoscopy
or a barium study of the stomach), and the concomitant use of
Autonomic Neuropathies medications with effects on GI motility (opioids, glucagon-like
AN can involve a number of different systems, including car- peptide-1 [GLP-1] receptor agonists, pramlintide) that are fre-
diovascular, gastrointestinal (GI), urogenital, and sudomotor. quently used in the management of patients with diabetes, is
needed before considering referring for more specialized test-
Cardiovascular AN ing for gastroparesis.
a) Symptoms
Most common symptoms of CAN occur upon standing and Urogenital AN
include lightheadedness, weakness, palpitations, dizziness, Erectile dysfunction, lower urinary tract symptoms, and female
faintness, and syncope. These symptoms usually occur in later sexual dysfunction may be consequences of AN (15) and are
stages of disease. Simple questions to elicit these symptoms frequent in patients with diabetes. Simple questions in clinical
may be used in the office. practice may unveil characteristic symptoms. Hypogonadism
or effects of medication should be excluded before a diagnosis
of erectile dysfunction (ED) is established.
b) Signs
Impaired heart rate variability is the earliest sign of CAN and
may be completely asymptomatic (8). In general requires lon- MANAGEMENT
ger electrocardiogram (ECG) monitoring and specialized Prevention
equipment for analysis that are not routinely available in the Intensive glucose control applied early in the course of the
office. However, simple tests such as measuring resting heart disease in patients with T1D dramatically reduces the incidence
rate or assessing heart rate change with deep breathing are done of DSPN and CAN (78% and 45% relative risk reduction,
easily in the office. In more advanced cases, patients may respectively) (4). In contrast, enhanced glucose control in pa-
present with resting tachycardia, exercise intolerance (8). Or- tients with T2D reduces the risk of developing DSPN modestly
thostatic hypotension which is defined as a fall in systolic (20 (5%9% relative risk reduction) (16). The presence of multiple

The Endocrine Society. Downloaded from press.endocrine.org by [${individualUser.displayName}] on 12 January 2017. at 12:19 For personal use only. No other uses without permission. . All rights reserved.
ENDO 2016 DIABETES AND GLUCOSE METABOLISM 103

comorbidities and risk factors including hypertension, hyper- use in the long-term treatment of gastroparesis beyond 5 days
lipidemia, and obesity among most T2D patients included in is no longer recommended and is reserved for severe cases that
these trials, the polypharmacy required to reach glucose targets, are unresponsive to other therapies (22).
and the high incidence of hypoglycemia and weight gain might
have attenuated the effects of glucose control and contributed
CASES WITH QUESTIONS
to inconsistent findings among T2D study populations (6).
Case 1
However, a multifactorial intervention targeting glucose and A 55-year-old Hispanic woman presents to the office for
many other cardiovascular disease risk factors plus lifestyle in progressive severe, shooting pain from both feet up to her
patients with T2D reduced the risk of CAN by 60% (17). ankles, worse at night. She claims her skin is on fire, and
Diabetes Prevention Program (DPP) (18) and the Impaired she cannot tolerate even the touch of clothing or bed sheet.
Glucose Tolerance Neuropathy (IGTN) Study (19) reported Reports no other known medical problems and was not
benefits of the lifestyle interventions on measures of CAN and taking any medications. On initial physical examination:
DSPN, respectively, these trials did not include subjects with patients height is 5 ft 3 in, weight is 182 lbs, waist circum-
established diabetes. ference 40 in, BP 145/90 mm Hg, and heart rate is 80
beats/min. Head/neck, lungs, cardiovascular unremarkable,
Pathogenetic Treatment extremities no edema. Initial laboratory are obtained and
Multiple pathogenetic pharmacotherapies with promising data consistent with comprehensive metabolic panel, TSH, com-
in experimental studies have been investigated in humans, so plete blood count, folate, protein electrophoresis within nor-
far the evidence from RCTs for a disease modifying therapy is mal levels, and urine negative for protein. A random glucose
disappointing (9, 10). was 138 mg/dL and hemoglobin A1c (HbA1c), 6.4%.
What is the most appropriate next diagnostic step:
Symptomatic Treatment a) Assess 8-point light touch sensation with 10-g
Treatment for Pain monofilament on plantar aspect of both feet.
At present, only 2 medications, pregabalin and duloxetine, b) Assess vibration and pin-prick sensation bilaterally
have received regulatory approval for the treatment of neuro- starting at the great toes.
pathic pain in diabetes by the United States Federal Drug c) Assess light touch sensation with 10-g monofilament on
Agency (FDA), Health Canada, and the European Medicines dorsal aspect of great toes bilaterally.
Agency. The opioid, tapentadol, has regulatory approval in the d) Refer for nerve conduction studies and neurological
United States and Canada. Several recent guidelines and sys- consultation.
tematic reviews have addressed the evidence base for the treat-
ment of neuropathic pain (20, 21) and will be reviewed.
Case 2
A 46-year-old male with 30 years of T1D, complicated by
Treatment for orthostatic hypotension nonproliferative diabetic retinopathy, nephropathy, and painful
Treatment for orthostatic hypotension is challenging and usu-
neuropathy presents with nausea, postprandial fullness, chronic
ally involves both pharmacological and nonpharmacological
constipation, and erectile dysfunction. He is on basal/bolus sc
interventions which will be revised. Midodrine, a peripheral,
insulin regimen with glargine at bedtime and insulin aspart
selective, direct 1-adrenoreceptor agonist, is an FDA-
with meals. HbA1c is 7.4%, but patient presents with frequent
approved drug for the treatment of orthostatic hypotension;
episodes of hypoglycemia, mainly 12 hours following his
however, ED titrations are often difficult and limited by side
meals followed by later hyperglycemic episodes. His medica-
effects, including supine hypertension.
tion regimen also comprises lisinopril 20 mg/d, hydro-
chlorthiazide 25 mg/d, atorvastatin 20 mg/d, amitriptyline 75
Treatment for Gastroparesis and GI Dysmotility mg/d, and oxycodone 15 mg q.6hours.
Treatment for diabetic gastroparesis is very challenging. Sev- What is the most appropriate next step in the management of
eral useful nonpharmacological clinical practices include: di- this patient:
etary changes such as eating multiple small meals, decreasing a) Perform gastric emptying studies.
dietary fat and fiber, and withdrawing drugs with effects on GI
b) Start formal carbohydrates counting teaching and adjust
motility, opioids, anticholinergics, tricyclic antidepressants,
bolus accordingly.
GLP-1RA, / DPP-4 inhibitors (22). Only metoclopramide,
c) Readjust pain medication.
a prokinetic agent, is approved by the FDA for the treatment of
d) Start metoclopramide.
gastroparesis. However, the level of evidence regarding the
benefits metoclopramide for the management of gastroparesis
is weak, and given the risk for serious adverse effects (extrapy-
REFERENCES
1. Tesfaye S, Boulton AJ, Dyck PJ, et al. Diabetic neuropathies: update on
ramidal symptoms, such as acute dystonic reactions; drug- definitions, diagnostic criteria, estimation of severity, and treatments. Dia-
induced parkinsonism; akathisia; and tardive dyskinesia), its betes Care. 2010;33:2285-2293.

The Endocrine Society. Downloaded from press.endocrine.org by [${individualUser.displayName}] on 12 January 2017. at 12:19 For personal use only. No other uses without permission. . All rights reserved.
104 ENDO 2016 MEET-THE-PROFESSOR CLINICAL CASE MANAGEMENT

2. Standards of medical care in diabetes-2015: summary of revisions. Diabe- 13. Feldman EL, Stevens MJ, Thomas PK, Brown MB, Canal N, Greene DA.
tes Care. 2015;38(suppl):S4. A practical two-step quantitative clinical and electrophysiological assess-
3. Ramsey SD, Newton K, Blough D, et al. Incidence, outcomes, and cost of ment for the diagnosis and staging of diabetic neuropathy. Diabetes Care.
foot ulcers in patients with diabetes. Diabetes Care. 1999;22:382-387. 1994;17:1281-1289.
4. Martin CL, Albers JW, Pop-Busui R. Neuropathy and related findings in 14. Callaghan BC, Kerber KA, Lisabeth LL, et al. Role of neurologists and
the diabetes control and complications trial/epidemiology of diabetes in- diagnostic tests on the management of distal symmetric polyneuropathy.
terventions and complications study. Diabetes Care. 2014;37:31-38. JAMA Neurol. 2014;71:1143-1149.
5. Young MJ, Boulton AJ, MacLeod AF, Williams DR, Sonksen PH. A 15. Pop-Busui R, Hotaling J, Braffett BH, et al. Cardiovascular autonomic
multicentre study of the prevalence of diabetic peripheral neuropathy neuropathy, erectile dysfunction and lower urinary tract symptoms in
in the United Kingdom hospital clinic population. Diabetologia. men with type 1 diabetes: findings from the DCCT/EDIC. J Urol.
1993;36:150-154. 2015;193:2045-2051.
6. Ang L, Jaiswal M, Martin C, Pop-Busui R. Glucose control and diabetic 16. Callaghan BC, Cheng HT, Stables CL, Smith AL, Feldman EL. Diabetic
neuropathy: lessons from recent large clinical trials. Curr Diab Rep. neuropathy: clinical manifestations and current treatments. Lancet Neurol.
2014;14:528. 2012;11:521-534.
7. Jaiswal M, Lauer A, Martin CL, Bell RA, et al. Peripheral neuropathy in 17. Gaede P, Vedel P, Larsen N, Jensen GV, Parving HH, Pedersen O.
adolescents and young adults with type 1 and type 2 diabetes from the Multifactorial intervention and cardiovascular disease in patients with type
SEARCH for Diabetes in Youth follow-up cohort: a pilot study. Diabetes 2 diabetes. N Engl J Med. 2003;348:383-393.
Care. 2013;36:3903-3908. 18. Carnethon MR, Prineas RJ, Temprosa M, Zhang ZM, Uwaifo G, Molitch
8. Spallone V, Ziegler D, Freeman R, et al. Cardiovascular autonomic neu- ME. The association among autonomic nervous system function, incident
ropathy in diabetes: clinical impact, assessment, diagnosis, and manage- diabetes, and intervention arm in the diabetes prevention program. Diabe-
ment. Diabetes Metab Res Rev. 2011;27:639-653. tes Care. 2006;29:914-919.
9. Boulton AJ, Kempler P, Ametov A, Ziegler D. Whither pathogenetic 19. Smith AG, Russell J, Feldman EL, et al. Lifestyle intervention for pre-
treatments for diabetic polyneuropathy? Diabetes Metab Res Rev. diabetic neuropathy. Diabetes Care. 2006;29:1294-1299.
2013;29:327-333. 20. Bril V, England JD, Franklin GM, et al. Evidence-based guideline: treat-
10. Ziegler D, Keller J, Maier C, Pannek J, German Diabetes A. Diabetic ment of painful diabetic neuropathyreport of the American Association of
neuropathy. Exp Clin Endocrinol Diabetes. 2014;122:406-415. Neuromuscular and Electrodiagnostic Medicine, the American Academy
11. Herman WH, Pop-Busui R, Braffett BH, et al. Use of the Michigan of Neurology, and the American Academy of Physical Medicine, Rehabili-
Neuropathy Screening Instrument as a measure of distal symmetrical tation. Muscle Nerve. 2011;43:910-917.
peripheral neuropathy in type 1 diabetes: results from the Diabetes Control 21. Finnerup NB, Attal N, Haroutounian S, et al. Pharmacotherapy for neuro-
and Complications Trial/Epidemiology of Diabetes Interventions and pathic pain in adults: a systematic review and meta-analysis. Lancet
Complications. Diabet Med. 2012;29:937-944. Neurol. 2015;14:162-173.
12. Zilliox LA, Ruby SK, Singh S, Zhan M, Russell JW. Clinical neuropathy 22. Pop-Busui R, Stevens M. Autonomic neuropathy in diabetes. In Umpierrez
scales in neuropathy associated with impaired glucose tolerance. J Diabe- GE, ed. Therapy for Diabetes Mellitus and Related Disorders. 6th ed.
tes Complications. 2015;29:372-377. Alexandria, VA, American Diabetes Association; 2014:834-863.

The Endocrine Society. Downloaded from press.endocrine.org by [${individualUser.displayName}] on 12 January 2017. at 12:19 For personal use only. No other uses without permission. . All rights reserved.
ENDO 2016 DIABETES AND GLUCOSE METABOLISM 105

Inpatient Management of Hyperglycemia

M21 guidelines recommending good glycemic control in noncriti-


Presented, April 1 4, 2016 cally ill surgical and medical patients as well as critically ill
patients in the intensive care setting (6, 7).
The need for good glycemic control in hospitalized patients is
Cecilia C. Low Wang, MD. Department of Medicine, now recognized in many institutions and hospital systems, with
University of Colorado School of Medicine, Aurora, more widespread use of insulin infusion protocols, insulin order
Colorado 80045, E-mail: cecilia.lowwang@ucdenver.edu sets, intensive hospital-wide education, and quality improvement
approaches to the care of patients with diabetes in the hospital
INTRODUCTION setting as well as patients with hyperglycemia without a prior
Historical Overview diagnosis of diabetes. However, a number of gaps in clinical
The Diabetes Mellitus Insulin Glucose Infusion in Acute Myo- practice and systems-based practice persist (8). For example, pa-
cardial Infarction (DIGAMI 1) study, published in 1995, dem- tients with hyperglycemia (BG 250 mg/dL) may unfortunately
onstrated a significant reduction in mortality in patients with remain hyperglycemic because of what are felt to be more acute
diabetes and suspected myocardial infarction who were ran- and pressing clinical issues, glucocorticoids are used for more
domly assigned to receive an insulin-glucose infusion in the indications than before including postoperative or chemotherapy-
hospital setting followed by 3 months of an outpatient associated nausea and vomiting, coordination of diabetes care
multidose sc insulin regimen, compared with patients receiving within and between hospital units is often far from ideal and may
conventional therapy (1). Van den Berghe and colleagues (2) lead to lapses in monitoring and therapy, standardization of bench-
later published a pivotal study in 2001 examining the use of marking and glucometrics and better outcomes studies are needed,
intensive insulin therapy in patients receiving mechanical ven- and optimal strategies for inpatient management of hyperglycemia
tilation in a surgical intensive care unit (SICU) (ICU), and under various clinical circumstances have not been well studied.
found that targeting a blood glucose (BG) range of 80 110 This session will focus on the clinical management of a few
mg/dL resulted in a marked reduction in mortality compared clinical scenarios seen in the inpatient setting.
with conventional therapy targeting a BG range of 180 200
mg/dL, in which insulin infusion was initiated if BG exceeded
215 mg/dL. BARRIERS TO OPTIMAL PRACTICE
A number of barriers exist that prevent optimal practice in the
Subsequent studies by these groups and other investigators
management of inpatient hyperglycemia:
were mixed, and results of the Normoglycemia in Intensive
Care EvaluationSurvival Using Glucose Algorithm Regulation Not prioritizing hyperglycemia management in the
(NICE-SUGAR) trial (3) led to caution regarding intensity of setting of other acute clinical issues.
glycemic control, because of increased harm from a tight glycemic Less-than-optimal coordination of clinical care within
target of 81108 mg/dL vs less than 180 mg/dL in patients and between hospital units and entities.
admitted to a mixed medical/surgical ICU. However, these two Gaps in clinical knowledge and expertise regarding
landmark studies set the stage for an explosion of interest and initiation and adjustment of basal bolus therapy.
clinical research in the inpatient management of hyperglycemia in Fear of causing hypoglycemia.
the past 2 decades. Not reviewing BG measurements on a daily basis and
making changes to insulin therapy as needed.
Persistent use of sliding-scale insulin.
SIGNIFICANCE OF THE CLINICAL PROBLEM
Of the 5.3 million hospitalizations with diabetes as any
listed diagnosis in the United States in 2010, 11.5% in- LEARNING OBJECTIVES
volved diabetes as a first-listed diagnosis (4), yet a signifi- As a result of participating in this session, learners should be
cant number of patients are admitted with hyperglycemia able to:
without a prior diagnosis of diabetes. A strong association Identify BG targets for hospitalized patients with
between hyperglycemia with or without prior diabetes and hyperglycemia.
subsequent mortality and morbidity has been demonstrated Describe initial steps in the management of hospitalized
in the inpatient setting by multiple investigators (57). Al- patients with diabetes.
though tight glycemic control in the ICU setting is not Recognize factors placing patients at high risk for
recommended (3), and it is unclear whether hyperglycemia developing hyperglycemia.
is a cause vs a marker of poor clinical outcomes, profes- Adjust insulin therapy when used to treat hyperglycemia
sional societies have published consensus statements and in hospitalized patients.

The Endocrine Society. Downloaded from press.endocrine.org by [${individualUser.displayName}] on 12 January 2017. at 12:19 For personal use only. No other uses without permission. . All rights reserved.
106 ENDO 2016 MEET-THE-PROFESSOR CLINICAL CASE MANAGEMENT

Provide recommendations for patients transitioning from Hypoglycemia order set or clear parameters for
hospital to ambulatory care. management of hypoglycemia.
Order scheduled insulin dosing with or without basal
insulin. Rapid-acting insulin orders should include a
STRATEGIES FOR DIAGNOSIS, THERAPY, scale for meals and a correction-only scale for nil per os
AND/OR MANAGEMENT (NPO) or when the patient eats only a small portion (eg,
Diagnosis 25%) of the meal.
Patients admitted to the hospital without a prior diagnosis of A hemoglobin A1c should be checked on any patient
diabetes may present with stress hyperglycemia due to the without a documented A1c within the previous 3 months
acute medical condition that led to the hospitalization, or de- to help guide both inpatient and outpatient therapy.
velop hyperglycemia because of concomitant therapies such as Education by nursing staff regarding calling for POC
glucocorticoids, enteral or parenteral nutrition, or pressors. glucose prior to meals and with symptoms of
Patients who have acute or chronic pancreatitis, trauma to the hypoglycemia, and for insulin injections.
Inpatient diabetes educator consultation if needed.
pancreas, or have undergone partial or total pancreatectomy are
at high risk for developing hyperglycemia and subsequent
diabetes. An A1c of 6.5% or greater is consistent with pre- Ongoing Management
existing undiagnosed diabetes, whereas an A1c less than 5.7% Daily evaluation of BG pattern (including POC and
is consistent with stress hyperglycemia. Patients with A1c serum values).
Daily adjustment of insulin regimen, if needed.
between 5.7 and 6.5% may have had prediabetes vs stress
Consider planned discharge regimen.
hyperglycemia (if a more chronic presentation is suspected).
The BG threshold that should prompt point-of-care (POC)
fingerstick BG monitoring and initiation of insulin therapy Planning for Discharge
is 180 mg/dL. Patients receiving glucocorticoids, enteral Consideration of factors that may affect the feasibility
nutrition/tube feeding, or parenteral nutrition should undergo and limit complexity of the outpatient regimen.
Modification of the outpatient regimen as needed.
POC glucose monitoring for 24 48 hours, or longer if gluco-
Scheduling of timely outpatient followup.
corticoid doses are increased or tube feeding/parenteral Patient education.
nutrition/enteral nutrition (TF/PN/EN) rates are increased, to
determine whether hyperglycemia develops and whether
Initiation of the Insulin Regimen
therapy is needed. Patients who have undergone pancreatec- Basal Insulin
tomy should receive POC glucose monitoring until they are For patients with pre-existing diabetes on more than 1 glucose-
able to tolerate per os intake, given that hyperglycemia may not lowering agent and/or on insulin, basal insulin should be initi-
develop until they receive a meal challenge. ated upon admission. The basal insulin dose may be deter-
mined using a weight-based method (0.1 0.2 U/kg/d) or the
Management/Therapy (9) home basal insulin dose, with some consideration of A1c (if
Patients with a prior diagnosis of diabetes should undergo known), reported hypoglycemia on the home regimen, and degree
evaluation of key elements of diabetes history including of adherence/frequency and type of missed doses. Situations that
type of diabetes, glucose-lowering therapy, level of glucose should prompt caution and empiric use of an initial basal insulin
control, review of glucose patterns, complications, and dose that is lower than the prescribed home dose include:
frequency/severity of hypoglycemia. Fairly well-controlled A1c (7.5%)
The patients inpatient glycemic goals should be deter- Basal insulin dose much greater than 50% of the total
mined: daily insulin dose (for patients on a basal-bolus insulin
For critically ill patients, the BG goal should be less regimen)
than 180 mg/dL. High total basal insulin dose (1 U/kg/d).
For noncritically ill patients, the premeal goal should be
less than 140 mg/dL, whereas the random BG should be
Bolus Insulin Using Rapid-Acting Insulin Analogs
less than 180 mg/dL. Meal Dosing. Various methods may be used to determine a
Patients receiving TF/PN are in a postprandial state and fixed dose of insulin for meals, depending on total daily dose
the goal range for BG is 140 180 mg/dL. (TDD) of insulin and/or estimated degree of insulin resistance.
Tighter BG goals may be appropriate if these can be Certain hospital units may be able to use carbohydrate counting
achieved with minimal hypoglycemia. to determine meal doses of insulin, although the available
clinical evidence has not demonstrated that this method is more
Initial Steps beneficial than fixed insulin dosing.
Discontinue all noninsulin medications. Depending on the type of diet ordered (eg, regular vs
Schedule POC glucose monitoring. carbohydrate-controlled), an estimated amount of carbohydrate

The Endocrine Society. Downloaded from press.endocrine.org by [${individualUser.displayName}] on 12 January 2017. at 12:19 For personal use only. No other uses without permission. . All rights reserved.
ENDO 2016 DIABETES AND GLUCOSE METABOLISM 107

in grams per meal can be used along with an estimated carbo- Planning for Discharge
hydrate to insulin ratio for each level of insulin sensitivity. The patients plan for discharge should be kept in mind begin-
For example, a patient with an estimated TDD of 50 U may ning early in the course of the hospitalization. If a patient is
have an estimated carbohydrate:insulin ratio of 10:1 using the believed to require intensive insulin therapy with a basal bolus
constant of 1600/TDD to calculate the correction factor (CF), regimen, important factors to consider include whether the
and CF/3 to calculate carbohydrate to insulin ratio. In other patient has the ability, motivation, self-management skills, re-
words, 1 U of insulin would be needed for every 10 g of sources, and support to be discharged on basal bolus therapy.
carbohydrates consumed. If the estimated amount of carbohy- Some patients may be discharged on their home regimen.
drate consumed per meal is 60 75 g, then a conservative However, patients with high A1c and/or high insulin require-
estimated meal dose of rapid-acting insulin would be 4 6 U. ments in the inpatient setting should have the home regimen
This information could be incorporated into 3 4 sets of reevaluated and modified.
scales ranging from insulin sensitive to extremely insulin resis- Patients who have limited financial resources should be
tant, with the option of a custom scale for clinicians to custom- placed on the planned modified insulin regimen (possibly with
ize the fixed meal dose as well as the correctional insulin neutral protamine Hagedorn (NPH) insulin and regular human
(below). insulin) at least 1 (and ideally more than 1) day prior to
discharge so that enough information can be accumulated and
doses can be adjusted for discharge. Patients who are unable or
Correctional Insulin Dosing. Correction-dose insulin can be
unwilling to take more than 23 insulin injections per day need
used at meal times and added to the meal dose of insulin, or
to be placed on types of insulin (such as premixed insulins, or
used alone for patients who are NPO, to correct hyperglyce-
70/30) or ratios of insulin at least 1 day (and ideally more than
mia. The amount of insulin used is based on an estimate of the
1 day) prior to discharge to allow for enough BG data to make
patients degree of insulin sensitivity.
dose adjustments for discharge.
For example, in the patient above with an estimated TDD of
Prior to discharge, important issues that must be addressed
50 U, the estimated CF using the equation CF 1600/TDD
include diabetes education including insulin teaching, diabetes
would be 30, ie, 1 U of rapid-acting insulin would be expected
survival skills for patients without a prior diagnosis of dia-
to decrease the patients BG by 30 mg/dL.
betes, when to monitor POC glucoses at home, what BG
The correctional insulin dosing can be incorporated into
parameters should prompt a call to a health care provider, and
tables of scales for ease of use, or calculated each time with a
scheduling of timely outpatient followup with a primary care
specific target BG in mind. Given that the target for most
provider and/or endocrinologist.
inpatients before meals is a BG of less than 140 mg/dL, one
could use a target of 140 mg/dL.
Specific Circumstances
Diabetic Ketoacidosis (10)
Making Adjustments to the Insulin Regimen For patients with uncomplicated diabetic ketoacidosis
Daily review of the patients BG values is imperative for good (DKA) without an identifiable precipitating medical
management of hyperglycemia in the hospital setting. The total cause, and as long as the serum potassium is 3.3
amount of insulin administered within the previous 24 hours mmol/L, sc insulin therapy can be instituted using rapid-
should be summed and reviewed in the context of the changing acting insulin (0.3 U/kg bolus, then 0.2 U/kg every 2 h).
clinical situation (was the patient NPO? How is the renal If an iv insulin infusion is used, a bolus is not necessary,
function? PO intake?). When a patients BGs are outside of the and the infusion can be started at 0.14 U/kg/hr.
desired range, a number of questions must be asked. If there If the serum glucose does not decrease by 10% after the
was one or more BG lower than the target range, causative or first hour, then an iv bolus of 0.14 U/kg should be
given, and therapy continued.
contributing factors may include: mistimed insulin, delayed
When BG reaches 200 mg/dL, switch to 0.1 U/kg sc
meal, missed meal, consumption of less than anticipated, over-
every 2 hours or 0.05 0.1 U/kg/h iv until DKA resolves.
estimation of carbohydrate content of the meal, stacking of Maintain glucose at 150 200 mg/dL.
correction doses, worsening renal function, etc., The TDD of
insulin should be reduced by at least 10 20%, sometimes Transitioning from iv to sc insulin
more, depending on the severity of the hypoglycemia and the Patients should be on a stable iv infusion rate of less
than 23 U/hr with BG 120 130 mg/dL.
surrounding circumstances.
Underlying clinical conditions should be
If there is one or more BG above the target range, factors to
resolving/resolved.
be considered include: missed insulin dose, mistimed insulin, As long as the patient was NPO, the infusion rate from
lack of insulin therapy for meals and/or snacks, underestima- the previous 6 8 hours can be used to extrapolate the
tion of carbohydrate intake, acute worsening of or new clinical patients estimated daily basal insulin dose.
condition, inadequately treated pain, initiation of glucocorti- The basal dose should be 50 80% of the estimated
coid therapy, etc. basal dose requirement.

The Endocrine Society. Downloaded from press.endocrine.org by [${individualUser.displayName}] on 12 January 2017. at 12:19 For personal use only. No other uses without permission. . All rights reserved.
108 ENDO 2016 MEET-THE-PROFESSOR CLINICAL CASE MANAGEMENT

To prevent rebound hyperglycemia, patients with DKA cation, keeping in mind patient-related factors and arranging
may be transitioned by starting the basal insulin dose for follow-up care are all key elements needed for successfully
with the start of the insulin infusion (11). transitioning a patient from the hospital setting to home or
Overlap basal insulin with the insulin infusion by at least outpatient care.
2 hours.
Enteral nutrition/tube feeding/parenteral nutrition (12) CASES
Patients on EN or PN should be monitored with POC Case 1
glucose testing every 4 6 hours for at least 24 48 N.K. is a 62-year-old man with type 2 diabetes who is postopera-
hours after starting therapy and with increases in
tive day 13 seconds/p 4-vessel coronary artery bypass graft. He
infusion rates or carbohydrate amounts.
developed a sternal wound infection, went into septic shock, and
Glucose monitoring can be discontinued in patients
without a prior history of diabetes if BGs are less than was intubated and transferred to the ICU. He has now been
140 mg/dL without insulin therapy for 24 48 hours successfully extubated but has had minimal PO intake, and you
after reaching goal calorie intake. are consulted because he was started on tube feeding, and has
Scheduled insulin should be started in patients with uncontrolled BGs. You evaluate him and find out that he has a
greater than 140 mg/dL with or without known diabetes, 10-year history of type 2 diabetes without diabetes complications
and who have a persistent requirement for correction except for coronary artery disease. He had been on an iv insulin
dose insulin. infusion but this was stopped 2 days ago, and he is now on
A BG target of 140 180 mg/dL should be used (13). Glargine, 25 U every morning with Humalog, 3 U every 6 hours.
Insulin orders should specify that they are intended for His BGs have ranged from 211368 mg/dL since his continuous
EN and should be held if EN is interrupted.
TF reached the goal rate of 60 cc per hour yesterday. He weighs
An iv dextrose infusion (D5 or D10) should be initiated
94 kg. He is receiving 12 g dextrose/60 cc.
at the same infusion rate as the EN/TF, and continued
1. What is the target range for his BG?
for as long as the expected duration of the longest
injected insulin dose. 2. How would you modify his insulin regimen to address
For continuous EN/TF, consider starting 70/30 insulin his uncontrolled hyperglycemia on the continuous tube
every 8 hours using a carbohydrate ratio of 1:25 in feeding?
patients without prior diabetes, or 1:15 in patients
with a pre-existing diagnosis, along with rapid-acting Case 2
insulin every 4 hours using a CF of 1:50 150 mg/dL A.R. is a 56-year-old woman who was admitted with temporal
For cyclic EN/TF, consider using 70/30 at the start of arteritis. She was started on prednisone, 60 mg PO every
the TF, and another dose halfway through. morning. The plan is to taper the prednisone slowly during the
For bolus EN/TF use rapid-acting insulin dosed with next several months depending on the resolution of her vascu-
each bolus.
litis. She has no prior diagnosis of diabetes, but her admission
For PN, consider starting with regular insulin using a
BG was 194 mg/dL, and she had a random BG of 287 mg/dL
1:151:25 carbohydrate ratio in the TPN bag.
on a mid-day blood draw after starting the prednisone. She
Steroids weighs 86 kg. Her A1c was 6.2%.
NPH insulin added to an existing insulin regimen can be 3. Does she have undiagnosed diabetes?
used for once- or twice-daily prednisone or 4. How should her steroid-induced hyperglycemia be
methylprednisolone, timed with the glucocorticoid dose.
managed?
Start with NPH 10 U with prednisone 40 mg or
equivalent in insulin-sensitive patients, or NPH 20 U in
insulin resistant patients. DISCUSSION OF CASES AND ANSWERS
Glucocorticoids dosed more than twice daily or Case 1
dexamethasone can be managed with increasing basal The goal range for a patient receiving enteral nutrition/tube
and bolus doses of insulin. feeding or parenteral nutrition is 140 180 mg/dL. He is receiv-
Taper NPH insulin with glucocorticoid taper. ing approximately 12 g of dextrose per hour or approximate
290 g per day. One possible strategy is to use basal insulin with
MAIN CONCLUSIONS correction-dose insulin. The corrections would be with rapid-
Management of inpatient hyperglycemia is an important part of acting insulin using a ratio of 1:25 or 1:50 mg/dL above 150
the care of the hospitalized patient. Many barriers exist, but a mg/dL. Another strategy would be to use NPH or 70/30 insulin
number of strategies are available for good glycemic control two to four times a day in addition to basal insulin. To use
and optimal management in this setting. Recognition and ap- 70/30 insulin or NPH, one could use a carbohydrate (CHO)
propriate management of situations that predispose to stress ratio of 1:10 1:15. For 29 g CHO per day and using 1 Unit for
hyperglycemia, expertise in management of patients with 15 g CHO 20 Units/24 hours 6 7 U of 70/30 insulin
known diabetes, and planning for discharge with needed edu- every 8 hours.

The Endocrine Society. Downloaded from press.endocrine.org by [${individualUser.displayName}] on 12 January 2017. at 12:19 For personal use only. No other uses without permission. . All rights reserved.
ENDO 2016 DIABETES AND GLUCOSE METABOLISM 109

Answer: Continue the same basal insulin. Add 70/30 insulin betes as Any-Listed Diagnosis, Adults Aged 18 Years and Older, United
1 Unit for 15 g of CHO (20 Units) in three divided doses (6 7 States, 2010. http://www.cdc.gov/diabetes/statistics/hosp/adulttable1.htm, ac-
cessed Nov 9, 2015.
U every 8 h). Add correction-dose insulin.
5. Tuna M, Manuel DG, Bennett C, et al. One- and five-year risk of death and
cardiovascular complications for hospitalized patients with hyperglycemia
Case 2 without diagnosed diabetes: An observational study. J Hosp Med.
Although her admission BG is elevated, this likely reflects 2014;9(6):365-371.
stress hyperglycemia from her acute presentation with tem- 6. Umpierrez GE, Hellman R, Korytkowski MT, et al. Management of
hyperglycemia in hospitalized patients in non-critical care setting: An
poral arteritis. Her A1c of 6.2% is consistent with this, endocrine society clinical practice guideline. J Clin Endocrinol Metab.
although she may have pre-existing prediabetes. She could 2012;97(1):16-38.
be started on NPH 20 U given at the same time as the 7. Moghissi ES, Korytkowski MT, DiNardo M, et al. American Association of
prednisone. She will require education regarding self moni- Clinical Endocrinologists and American Diabetes Association consensus state-
toring of BG, recognition/management of hypoglycemia, ment on inpatient glycemic control. Diabetes Care. 2009;32(6):1119-1131.
8. Draznin B, Gilden J, Golden SH, et al. Pathways to quality inpatient
adjustment of the NPH dose to achieve good glycemic
management of hyperglycemia and diabetes: a call to action. Diabetes
control, and outpatient followup for her stress hyperglyce- Care. 2013;36(7):1807-1814.
mia and tapering of NPH insulin during her prednisone 9. Low Wang CC, Draznin B. Insulin use in hospitalized patients with
taper. diabetes: Navigate with care. Diabetes Spectr. 201326(2):124-130.
10. Umpierrez GE, Mendez CE. Chapter 37. Therapy for diabetes mellitus and
related disorders, 6th ed. Guillermo Umpierrez, ed. Alexandria: American
REFERENCES Diabetes Association, 2014;627.
1. Malmberg K, Ryden L, Efendic S, et al. Randomized trial of insulin- 11. Hsia E, Seggelke S, Gibbs J, et al. Subcutaneous administration of glargine
glucose infusion followed by subcutaneous insulin treatment in diabetic
to diabetic patients receiving insulin infusion prevents rebound hypergly-
patients with acute myocardial infarction (DIGAMI study): effects on
cemia. J Clin Endocrinol Metab. 2012;97(9):3132-3137.
mortality at 1 year. J Am Coll Cardiol. 1995;26(1):57-65.
2. Van den Berghe G, Wouters P, Weekers F, et al. Intensive Insulin Therapy 12. Low Wang CC, Draznin B. Practical approach to management of inpatient
in Critically Ill Patients. N Engl J Med. 2001;345:1359-1367. hyperglycemia in select patient populations. Hosp Pract. 2013;41(2):45-53.
3. Finfer S, Chittock DR, Su SY, et al. Intensive versus Conventional Glu- 13. McMahon MM, Nystrom E, Braunschweig C, Miles J, Compher C.
cose Control in Critically Ill Patients. N Engl J Med. 2009;360:1283-1297. A.S.P.E.N. clinical guidelines: Nutrition support of adult patients with
4. Distribution of First-Listed Diagnoses Among Hospital Discharges with Dia- hyperglycemia. JPEN J Parenter Enteral Nutr. 2013;37:23-36.

The Endocrine Society. Downloaded from press.endocrine.org by [${individualUser.displayName}] on 12 January 2017. at 12:19 For personal use only. No other uses without permission. . All rights reserved.
110 ENDO 2016 MEET-THE-PROFESSOR CLINICAL CASE MANAGEMENT

Management of Hypoglycemia and Impaired Awareness of


Hypoglycemia in Diabetes

M22 Particular attention must be paid to recognizing and managing


Presented, April 1 4, 2016 patients with impaired awareness of hypoglycemia. These are
the knowledge gaps that will be addressed in this session.

Elizabeth R. Seaquist, MD. Division of Diabetes and


BARRIERS TO OPTIMAL PRACTICE
Endocrinology, Department of Medicine, University of
One of the major barriers to managing hypoglycemia in diabe-
Minnesota, Minneapolis, Minnesota 55455, E-mail:
tes is lack of time to adequately review the patients glucose
seaqu001@umn.edu
records and integrate them with the history of medication doses
taken, food eaten, and exercise/activity performed. A second
INTRODUCTION barrier is that not all patients will remember to report episodes
Historical Overview of hypoglycemia to their clinician, and many fail to realize that
Hypoglycemia has been noted to be a consequence of insulin they have impaired awareness of hypoglycemia. Clinicians
therapy ever since insulin was first used as a therapy in the need a systematic approach to determining whether their pa-
1920s. Optimizing glycemic control without causing hypogly- tients have impaired awareness of hypoglycemia.
cemia is a key principle in diabetes management.

LEARNING OBJECTIVES
SIGNIFICANCE OF THE CLINICAL PROBLEM As a result of participating in this session, learners should be
Hypoglycemia is the limiting factor that prevents patients with able to:
type 1 and advanced type 2 diabetes from obtaining optimal Identify factors that contribute to hypoglycemia in
glycemic control. The consequences of hypoglycemia can patients with diabetes.
range from inconvenience to confusion, seizures, and death. Up Recognize impaired awareness of hypoglycemia in
to 10% of the deaths in patients with type 1 diabetes are
patients with diabetes.
estimated to be attributable to hypoglycemia (1). More than
Develop treatment regimens that minimize the risk of
one third of patients with type 1 diabetes experience at least
hypoglycemia in patients with diabetes.
one episode of severe hypoglycemia (defined as an event of
Assist patients with diabetes and impaired awareness of
hypoglycemia that required the assistance of another) each year
hypoglycemia regain their awareness of hypoglycemia.
(2, 3). Approximately 10% of patients with type 2 diabetes
experience severe hypoglycemia each year and clinical trials
have repeatedly demonstrated that such an event predicts an STRATEGIES FOR DIAGNOSIS, THERAPY,
increased risk for mortality in subsequent months (4). AND/OR MANAGEMENT
Hypoglycemia in diabetes is the consequence of treatment Diagnosis
with insulin and/or insulin secretagogues. When prescribing The diagnosis of impaired awareness of hypoglycemia in pa-
such therapies, clinicians must ensure their patients understand tients with diabetes is made by determining whether the patient
how to anticipate, recognize, and treat hypoglycemia. Recur- has symptoms of a low blood glucose when they have a value
rent episodes of hypoglycemia can blunt the counterregulatory less than 70 mg/dL. Although healthy people can have blood
response to a subsequent episode of hypoglycemia in the next sugars lower than 70 mg/dL without symptoms, this level of
few days. As a result of this blunting, patients must reach a glucose should be viewed as an alert value in patients on
lower and lower glucose concentration before eliciting the insulin or secretagogues and additional questions should be
catecholamine release that leads to the adrenergic symptoms asked. The Hypoglycemia Workgroup report published in 2013
that trigger recognition of hypoglycemia in the patient (5). If includes a patient questionnaire than be used to determine the
the glucose level at which this response is elicited decreases hypoglycemia risk of any given patient (8).
below that which causes neuroglycopenia, the first symptom The questions listed below are particularly helpful in
the patient has of hypoglycemia may be unconsciousness. Im- identifying patients with impaired awareness of hypogly-
paired awareness of hypoglycemia may occur in up to one third cemia.
of all patients with type 1 diabetes (6, 7), which puts many 1. To what extent can you tell by your symptoms that your
patients at risk for accidents and injury. blood glucose is low?
To optimize diabetes control without hypoglycemia requires A. Never
caregivers to be aware of which patients are at risk for hypo- B. Rarely
glycemia and be knowledgeable in how to mitigate this risk. C. Sometimes

The Endocrine Society. Downloaded from press.endocrine.org by [${individualUser.displayName}] on 12 January 2017. at 12:19 For personal use only. No other uses without permission. . All rights reserved.
ENDO 2016 DIABETES AND GLUCOSE METABOLISM 111

D. Often secretagogues. Because of the dangers associated with hypo-


E. Always glycemia, care must be taken to help patients avoid low blood
2. In a typical week, how many times will your blood sugars. This is best done by providing the patient with educa-
glucose go below 70 mg/dL? tion about how to anticipate, detect, and treat hypoglycemic as
_____ per week well as insure that the drugs and treatment goals applied are
3. How many times have you had a severe hypoglycemic appropriate for the patient. Impaired awareness of hypoglyce-
episode (when you needed someones help and were mia occurs in patients with recurrent episodes of hypoglycemia
unable to treat yourself)? over a short period of time. Because such patients experience
A. Since the last visit: _____ times. loss of consciousness as their first symptom of hypoglycemia,
B. In the last year: _____ times. it is imperative that clinicians identify patients with impaired
awareness in their practice. Structured educational programs to
Therapy and Management reduce hypoglycemia may be particularly helpful in reversing
When hypoglycemia becomes a problem, the diabetes health- impaired awareness of hypoglycemia in patients with diabetes.
care provider should review the risk factors commonly associ-
ated with hypoglycemia (see Risk Factors for Hypoglycemia)
(9). Working with the patient, the provider should make adjust- CASES WITH QUESTIONS
ments in the treatment regimen to ensure that euglycemia is Case 1
achieved by a combination of insulin/insulin secretagogue A 78-year-old woman with a 30-year history of type 2 diabetes
taken, food eaten, and activity performed. A referral to a mellitus is brought in to your office by her daughter because of
comprehensive diabetes education program should also be concerns about hypoglycemia. She had stopped by to visit at
made for patients with new problems with hypoglycemia. 1400 hours the previous day and found her mother unrespon-
Health care providers should also be alert to identifying sive on the couch. The paramedics were called and blood
patients with diabetes who may have impaired awareness of glucose was found to be 32 mg/dL. Inez does not know what
hypoglycemia. The questions listed above from the Hypogly- happened but remembers sitting down to watch TV at 0930
cemia Questionnaire developed by the Work Group on Hypo- hours. Today she is fine except she is angry that her daughter
glycemia from the American Diabetes Association (ADA) and does not think she should live alone any longer. Past medical
Endocrine Society should be asked of all patients at risk for history is significant for coronary artery disease with stent place-
hypoglycemia during the clinic visit. In addition, patients with ment 3 years ago. Current meds are clopidogrel; atorvastatin;
a history of previous episodes of severe hypoglycemia, abso- lisinopril; glyburide, 5 mg twice a day; metformin, 2000 mg each
lute insulin deficiency as is seen in long duration type 1 day; and sitagliptin, 100 mg each day.
diabetes, and overly aggressive A1c targets may be at particu- What additional information do you need to assist Inez in
lar risk for the development of impaired awareness of hypogly- her diabetes management?
cemia (9). Restoring awareness of hypoglycemia in patients A. Log book of home blood glucoses.
with impairment is often difficult, but referral to a structured B. History of content and timing of meals/snacks ingested
diabetes education program should be mandatory for such in last week.
patients (10). Re-evaluation of treatment goals and drugs used C. Record of activity done by Inez in the last week.
to manage diabetes should also be done to ensure that the D. All of above.
regimen best meets the needs of the patient.
Case 2
Risk Factors for Hypoglycemia A 49-year-old man with type 1 diabetes since age 11 comes for
Doses of insulin or insulin secretagogue are excessive or assistance with diabetes management. His wife is concerned
ill timed. because he had a seizure at night last week from a low sugar.
The amounts of carbohydrates eaten are done so without He has always strived to maintain good glycolic control but his
consideration of the anticipated effect of insulin or
hemoglobin A1c values have ranged from 8.19.7% over the
insulin secretagogue taken.
last several years. He has had severe hypoglycemia in the past,
Activity is increased without a change in food eaten or
insulin/insulin secretagogue taken. but the last episode was more than a year ago. He checks his
Sensitivity to insulin has increased such as with weight sugars before meals and at bedtime and has an average of 197
loss or improved glucose control. mg/dL during the last month. He usually feels low sugars if
Alcohol ingestion (impairs hepatic glucose production). they are 50 mg/dL or less. His current medications are glargine,
Insulin clearance is reduced, such as in renal failure. 21 U in the morning; lispro, 1 U for 7 g of carb 1:50
correction with a target of 100; lisinopril; simvastatin; and
MAIN CONCLUSIONS aspirin.
Hypoglycemia is a common occurrence in the life of patients 1. Which of the following information tells you he currently
with diabetes who are treated with insulin and insulin has impaired awareness of hypoglycemia?

The Endocrine Society. Downloaded from press.endocrine.org by [${individualUser.displayName}] on 12 January 2017. at 12:19 For personal use only. No other uses without permission. . All rights reserved.
112 ENDO 2016 MEET-THE-PROFESSOR CLINICAL CASE MANAGEMENT

A. He is striving for good glycemic control. is a risk for subsequent episodes of severe hypoglycemia but
B. He had an episode of severe hypoglycemia more than does not necessarily mean he has impaired awareness of
a year ago. hypoglycemia. Patients with long-duration type 1 diabetes
C. He does not recognize low sugars until they are 50 or are at greater risk for hypoglycemia, but most do not have
less. impaired awareness of hypoglycemia.
D. He has had type 1 diabetes for more than 30 years. Answer 2: A
2. Which of the following recommendations would be most A structured diabetes education program has been demon-
likely to restore his awareness of hypoglycemia? strated to be effective in restoring awareness of hypoglycemia
A. Enrollment in a comprehensive diabetes education in some but not all patients with type 1 diabetes and impaired
program that focuses on adjusting insulin doses to
awareness of hypoglycemia (10). Changes in the mode of
meet the metabolic needs of the moment.
insulin administration or the type of insulin or the addition of a
B. Switching him from insulin injections to a pump.
bedtime snack by themselves are not likely to restore impaired
C. Asking him to eat a bedtime snack every night.
awareness of hypoglycemia in this patient.
D. Switching from U100 glargine to U300 glargine.

REFERENCES
DISCUSSION OF CASES AND ANSWERS 1. Skrivarhaug T, Bangstad HJ, Stene LC, Sandvik L, Hanssen KF, Joner G.
Case 1 Long-term mortality in a nationwide cohort of childhood-onset type 1
Answer: D diabetic patients in Norway. Diabetologia. 2006;49(2):298-305.
Glycemia at any one point of time is determined by the 2. Pedersen-Bjergaard U, Pramming S, Heller SR, et al. Severe
hypoglycaemia in 1076 adult patients with type 1 diabetes: Influence of
medication taken, food ingested, and activity performed in the
risk markers and selection. Diabetes Metab Res Rev. 2004;20(6):479-486.
preceding hours. When patients present with a history of severe 3. Donnelly LA, Morris AD, Frier BM, et al. Frequency and predictors of
hypoglycemia, it is critical to determine the factor(s) that led to hypoglycaemia in type 1 and insulin-treated type 2 diabetes: A population-
the episode so that their regimen can be altered to prevent based study. Diabet Med. 2005;22(6):749-755.
4. Heller SR, Choudhary P, Davies C, et al. Risk of hypoglycaemia in types
episodes from occurring in the future. In addition, the clinician
1 and 2 diabetes: effects of treatment modalities and their duration.
must examine how the patients glycemia has responded to Diabetologia. 2007;50(6):1140-1147.
changes in food, medication, and activity in the home setting, 5. Cryer PE. Mechanisms of hypoglycemia-associated autonomic failure in
which is best done by examining a record that reports home diabetes. N Engl J Med. 2013;369(4):362-372.
glucose values, medications taken, food ingested, and activities 6. Geddes J, Schopman JE, Zammitt NN, Frier BM. Prevalence of impaired
awareness of hypoglycaemia in adults with Type 1 diabetes. Diabet Med.
performed. 2008;25(4):501-504.
7. Jordan LV, Robertson M, Grant L, et al. The Tayside insulin management
Case 2 course: An effective education programme in type 1 diabetes. Int J Clin
Pract. 2013;67(5):462-468.
Answer 1: C
8. Seaquist ER, Anderson J, Childs B, et al. Hypoglycemia and diabetes: A
He does not have symptoms of hypoglycemia until his report of a workgroup of the American Diabetes Association and the
sugar is 50 mg/dL or less, which means he has a blunted Endocrine Society. Diabetes Care. 2013;36(5):1384-1395.
counterregulatory response to hypoglycemia. Striving for 9. Cryer PE, Axelrod L, Grossman AB, et al. Evaluation and Management of
good glycemic control has put patients in clinical trials at Adult Hypoglycemic Disorders: An Endocrine Society Clinical Practice
Guideline. J Clin Endocr Metab. 2009;94(3):709-728.
greater risk of hypoglycemia, but achieving optimal control 10. Yeoh E, Choudhary P, Nwokolo M, Ayis S, Amiel SA. Interventions that
in the absence of hypoglycemia will not impair the counter restore awareness of hypoglycemia in adults with type 1 diabetes: A system-
regulatory response. Having severe hypoglycemia in the past atic review and meta-analysis. Diabetes Care. 2015;38(8):1592-1609.

The Endocrine Society. Downloaded from press.endocrine.org by [${individualUser.displayName}] on 12 January 2017. at 12:19 For personal use only. No other uses without permission. . All rights reserved.
ENDO 2016 DIABETES AND GLUCOSE METABOLISM 113

Exercise Prescriptions for Patients With DM2

M24 diabetes are unable to offset the increased diabetes preva-


Presented, April 1 4, 2016 lence and earlier age of onset.
The benefits of exercise/physical activity have clearly been
demonstrated in diabetes. A 2001 metaanalysis of exercise
Irene Schauer, MD, PhD. University of Colorado intervention studies showed a significant 0.66% decrease in
Anschutz Medical Campus, Aurora, Colorado, 80045, and hemaglobin A1c after at least 8 weeks of intervention even in
Denver VA Medical Center, Denver, Colorado, 80220, the absence of improved body mass index (BMI) (3). Physical
E-mail: irene.schauer@ucdenver.edu activity and/or physical fitness have clearly been demonstrated
to correlate with improved mortality in populations with diabe-
HISTORICAL OVERVEW tes, as well as those without. Wei et al first demonstrated the
The prevalence of diabetes and prediabetes is increasing annu- relationship of both low fitness and inactivity to mortality in
ally. Exercise is clearly a cornerstone of treatment and preven- diabetes, and several other studies and analyses have since
tion of type 2 diabetes, yet people with type 2 diabetes and reinforced this relationship (4 6). McAuley et al (7) followed
prediabetes continue to exercise less than the average popula- 831 men with type 2 diabetes for 5 years. The subgroup of men
tion. Defining the type, duration, and intensity of exercise with peak exercise capacity of less than 5 metabolic equiva-
needed to benefit and determining how best to prescribe exer- lents (MET), corresponding to a maximum oxygen consump-
cise and achieve compliance with this prescription remain tion (VO2max) of 17.5 ml/kg/min, had a relative risk for
active areas of investigation. This session briefly reviews the all-cause mortality of 1.7 compared with those with peak ca-
current state of knowledge and discusses guidelines for and pacity more than or equal to 5 METs. They concluded that each
approaches to prescribing exercise in type 2 diabetes. additional MET of exercise capacity conferred a 10% survival
benefit (7). In fact, the term sedentary death syndrome has been
SIGNIFICANCE OF THE CLINICAL PROBLEM coined to underscore the relationship of inactivity (and result-
It is well known that diabetes, especially type 2 diabetes, ing poor fitness) to increased risk of mortality. It has been
prevalence is increasing to epidemic proportions. A 2005 proposed that we have evolved to be physically active and
report suggested that by 2025, 300 million individuals require activity to maintain a normal state of metabolic health.
would be affected worldwide (1). By 2013, The Interna- The message has been complicated slightly by the recent
tional Diabetes Federation estimated that type 2 diabetes demonstration of the existence of nonresponders to exercise.
already affected more than 380 million individuals world- Several studies have now reported adverse changes in
wide (2), and estimates of the rate of increase appear to cardiometabolic parameters with exercise interventions (Table
continue to rise annually. Accompanying this epidemic of 1) in individuals with and without diabetes (8). It is, however,
diabetes is an increase in complication rates and mortality of important to note that 1) most individuals have beneficial
diabetes, most notably cardiovascular disease incidence and changes in all the measured parameters, 2) adverse responses
mortality, as improvements in the metabolic management of (AR) are typically only in a single parameter, and 3) only a

TABLE 1. Low Prevalence of Adverse Cardiometabolic Response to Exercise


HERITAGE DREW INFLAME STRRIDE MARYLAND JYVASKYLA TOTAL
N subjects 723 326 70 303 160 105 1687
% 1 VO2max 18 9 14 12 12 13
Prevalence of adverse responders TOTAL %
1 fasting insulin 55 36 12 17 4 2 126 8.3
1 HDL 47 87 21 32 8 27 222 13.3
1 Triglycerides 56 51 9 34 11 11 172 10.3
1 SBP 44 58 11 43 10 166 12.2
Any 1 AR 145 13 32 71 54 35 468 31
Any 2 ARs 28 37 9 9 5 7 95 6
34 ARs 3 9 1 0 0 0 13 0.8

Adapted from Bouchard et al. 2012 PLOSone.

The Endocrine Society. Downloaded from press.endocrine.org by [${individualUser.displayName}] on 12 January 2017. at 12:19 For personal use only. No other uses without permission. . All rights reserved.
114 ENDO 2016 MEET-THE-PROFESSOR CLINICAL CASE MANAGEMENT

very small percentage of individuals had ARs to more than 2 als as measured by VO2 peak. This difference is independent of
parameters. In addition, other parameters where physical activ- baseline physical activity and BMI. Other differences include
ity has clear morbidity and potential mortality benefits were not 1) slower heart rate and VO2 kinetics (the rates at which heart
studied, including effects on sleep quality, cognitive function, rate and oxygen consumption increase to meet increased de-
mood, overall glycemic control, weight, pain management, mand); 2) increased lactate levels at lower rates of exertion;
increased lean body mass, and overall functional status and fall and 3) evidence of diastolic dysfunction during exercise. Per-
risk. In a recent study of resistance training in older individu- haps most important to consider when prescribing exercise is
als, the authors conclude that there are no nonresponders based that individuals with diabetes report a higher perceived rate of
on the fact that, despite wide heterogeneity in response, all exertion when exercising at a given percent of maximal exer-
individuals improved in at least 1 measure of strength or cise capacity. The cause and effect relationship between diabe-
function (9). tes and rate of perceived exertion may be debatable, but the
Overall, the benefits of exercise in type 2 diabetes are not in bottom-line is that exercise appears to be more painful in
question. The potential to improve metabolic health, morbidity, people with type 2 diabetes, an obvious barrier to exercise
and mortality through multiple beneficial effects is clear, and compliance.
guidelines for diabetes management universally include physi-
cal activity/exercise. However, individuals with diabetes are on
LEARNING OBJECTIVES
average more sedentary. Furthermore, we, as healthcare pro-
As a result of participating in this session, learners should be
viders, do not universally promote/prescribe physical activity
able to:
to the degree that we prescribe medications and promote di-
List multiple benefits of and specific barriers to exercise
etary changes. What are the barriers to physical activity that
in type 2 diabetes.
perpetuate this noncompliance with a clearly beneficial inter-
Understand the recommendations regarding amount and
vention and how can we overcome them?
type of exercise for people with type 2 diabetes.
List practical approaches to prescribing exercise in type
BARRIERS TO OPTIMAL EXERCISE 2 diabetes.
PARTICIPATION
Some of the most obvious barriers to regular physical activity
or exercise for everyone are the environmental challenges, STRATEGIES FOR INCREASING PHYSICAL
including lack of access to safe and pleasant places to exercise ACTIVITY IN TYPE 2 DIABETES
(sidewalks, parks, or other affordable facilities). In addition, Before devising a strategy, it is important to define our goal
the higher prevalence of type 2 diabetes in lower socioeco- clearly. The terms physical activity and exercise have been
nomic groups increases these and adds other environmental used together so far, but what do they really mean? Techni-
barriers, including potentially dangerous neighborhoods, in- cally, physical activity is defined as any bodily movement
ability to afford the available facilities, and higher traffic den- produced by skeletal muscle and resulting in energy expendi-
sity. In addition, societal changes have removed much of the ture above the resting metabolic rate. Exercise is often used to
need and opportunity for physical activity in daily life with refer to a subset of physical activity that is more structured,
increased reliance on motor vehicles and automation. Some planned, repetitive, and purposeful in that it has the goal of
cultures and ethnicities impose additional barriers to exercise improving some measure of physical fitness. However, many
that must be recognized when attempting to prescribe exercise recreational activities such as hiking and skiing are generally
in diabetes. For instance, in one study, 29% of nonexercising also considered exercise even though their purpose is not spe-
African American women reported their hair style as a signifi- cifically improvement of fitness. Because it is often felt that the
cant barrier to exercise. Sensitivity to such cultural issues is term exercise has negative connotations, especially in a
important in providing a specific exercise prescription. population that is inclined to be sedentary, many guidelines
Additional barriers to exercise, including clear physiological have gradually shifted to referring to physical activity rather
barriers, also exist in type 2 diabetes. The complications of than exercise. However, their recommendations technically re-
diabetes, including neuropathy, vascular disease, amputations, main exercise guidelines, because they specify a dose and
and retinopathy, can profoundly affect an individuals ability to frequency of a repetitive structured activity. Although the spe-
exercise and/or available modalities and opportunities for ex- cific guidelines for physical activity goals in type 2 diabetes
ercise. These complications can also introduce an element of should always be kept in mind, it is important to recognize that
fear of injury or further complication that is not present in the we do not really have a good answer to how much is
average population. enough? and that pushing for too much may end up getting us
In addition, specific exercise defects have been demon- nothing. Thus, the right strategy will always be patient specific
strated in diabetes (10, 11). On average, individuals with dia- and will require some time, motivational interviewing, and
betes have a lower exercise capacity than nondiabetic individu- creativity to work out.

The Endocrine Society. Downloaded from press.endocrine.org by [${individualUser.displayName}] on 12 January 2017. at 12:19 For personal use only. No other uses without permission. . All rights reserved.
ENDO 2016 DIABETES AND GLUCOSE METABOLISM 115

The Evidence exercise OR an equivalent combination of the 2)


Greater daily walking time correlated with improved survival (grade A).
(12). In approximately 3000 adults with diabetes, all-cause and Spread over at least 3 days of the week with no more
cardiovascular disease (CVD) mortality were decreased by than 2 consecutive days without exercise (grade A).
39% and 34%, respectively, in individuals who walked at least In the absence of contraindications, adults with type 2
2 h/wk, and by 54% and 53%, respectively, in those who diabetes should be encouraged to perform resistance
training at least twice per week, with each session
walked 3 4 h/wk. The analysis concluded that 1 life could be
consisting of at least 1 set each of 5 or more different
saved per year for every 61 people who could be persuaded to resistance exercises involving the large muscle groups
walk (no specific intensity) at least 2 h/wk. Another study (grade A).
followed more than 1600 people (347 with type 2 diabetes) for More than 7 h/wk of activity may be required where
10 years. All-cause mortality was not reduced in the light weight loss maintenance is a goal.
walkers (1 mile or 3000 steps per day). Moderate walking
American Association of Clinical Endocrinologists
(1 mile or 3000 steps/d), however, correlated with dramati- Patients should engage in at least 150 minutes per week
cally improved all-cause mortality (hazard ratio 0.54), specifi- of moderate-intensity exercise such as brisk walking (15-
cally in the subgroup with diabetes (13). An analysis of CVD to 20-min mile) or its equivalent (grade B).
risk by walking pace in men with type 2 diabetes from the Persons with type 2 diabetes should also incorporate
Health Professionals follow-up study found that all-cause and flexibility and strength-training exercises (grade B).
CVD mortality decreased steadily and independently with in- Patients must be evaluated initially for contraindications
creasing average walking pace and with walking time per week and/or limitations to physical activity, and then an
(14). Thus, greater intensity and greater duration both yield exercise prescription should be developed for each
greater benefit, but there is no clear evidence for a threshold patient according to both goals and activity limitations;
physical activity programs should begin slowly and build
intensity below which no benefit occurs. From these and other
up gradually (grade D).
studies, we can conclude that our immediate goal is to increase
physical activity from baseline. We should provide ultimate Other/international guidelines typically include similar recom-
guideline-driven goals, but the actual prescription for exercise mendations, although some go further and recommend 210
must be patient specific and take many factors into account. min/wk.
The above studies showed correlations between fitness or
physical activity and some measure of disease-free survi- The Prescription
val, and for these and other reasons, we believe that When we prescribe a medication, we try to prescribe what we
exercise/physical activity is good and important. But interven- think is absolutely best for our patients. How do we do that for
tion studies with hard clinical outcomes to fully establish cause exercise?
and effect are difficult, expensive, necessarily small, and rare.
Those that do exist use a variety of different interventions and Preevaluation
all are confounded by the issue of noncompliance. As a result, Symptoms of cardiac ischemia. Beyond this, screening
the best study in a metabolically impaired (prediabetic) popu- for CVD (eg, stress test) is not routinely recommended,
lation in which a large standardized intervention was applied in because the exercise progression in diabetes should be
a randomized and placebo-controlled manner with study of a slow with a goal of sustained, regular moderate intensity
hard clinical outcome remains the Diabetes Prevention Pro- exercise.
Neuropathy, including autonomic neuropathy.
gram. In this famous trial, the lifestyle intervention that in-
Foot lesions or deformities.
cluded an exercise program significantly slowed the progres-
Glycemic control: hyperglycemia before exercise and
sion to diabetes and outperformed a comparison intervention poor overall glycemic control have been shown to
with metformin. This study remains the primary support for the decrease the response to exercise in terms of both
specific details of the guidelines endorsed by the American glycemic control and exercise capacity (16); frequent
Diabetes Association and the American Heart Association for hypoglycemia increases the risk of severe exercise-
physical activity goals in type 2 diabetes. induced hypoglycemia.
Psychosocial limitations to exercise.
The Guidelines
American Diabetes Association, American Heart Patient specific: shared decision making, motivational
Association: Physical activity recommendations (15) interviewing
Children with diabetes or prediabetes should be Stage of readiness for change (precontemplation,
encouraged to engage in at least 60 minutes of physical contemplation, decision, action, maintenance).
activity each day (grade B). Patient goals (weight loss, glycemic control,
More than or equal to 150 min/wk of moderate intensity avoidance of insulin, survival, avoidance of
aerobic exercise (OR 75 min/wk of vigorous aerobic complications, mood, sleep, etc).

The Endocrine Society. Downloaded from press.endocrine.org by [${individualUser.displayName}] on 12 January 2017. at 12:19 For personal use only. No other uses without permission. . All rights reserved.
116 ENDO 2016 MEET-THE-PROFESSOR CLINICAL CASE MANAGEMENT

Limitations (pain, injury, mobility, financial, recommended duration. More is better, but anything is
environmental, etc). better than nothing. The sedentary death syndrome is a
Patient likes and dislikes. useful framework to keep in mind when prescribing
Key is to identify (or rather, help the patient identify)
exercise in diabetes in that our initial goal is likely
the intrinsic motivation.
simply to decrease inactivity and sedentary time.

Components of the prescription


Official components (FITT) CASE STUDIES WITH QUESTIONS
Case 1: A 45-year-old obese woman with type 2 diabetes for
Frequency: near daily; the bout effect is important meta-
12 years comes to you for help with her diabetes. Her primary
bolically and lasts approximately 24 36 hours.
care provider has warned her that she needs insulin if she does
Intensity: start low and work up; moderate intensity is the

recommended goal (3 6 METs, 150%160% of resting not shape up. She took this to heart and joined a gym with
heart rate or heart rate 120; a level that allows conver- the goal of losing 70 pounds in the next year and improving her
sation). glycemic control to avoid insulin. She makes herself go and
Time: again can start low and work up; goal is at least work out hard every day and is very disheartened, because she
150 minutes per week. feels terrible and her morning blood sugars have been higher
Type: aerobic (anything that gets the heart rate up; water since she started exercising.
aerobics or stationary cycling are good options for indi-
viduals with neuropathy or joint limitations) and twice Question: What Should You Do?
weekly resistance training. 1. Advise her to stop exercising as she is apparently a
Unofficial components: motivation, creativity, partners, nonresponder to exercise.
realistic expectations, measures of success 2. Review and lighten her exercise regimen.
Find the intrinsic motivation. 3. Have her check more frequent blood sugars and look
Discuss specific goals. for patterns suggestive of postexercise hypoglycemia.
Discuss specific limitations. 4. Discuss her dietary changes with exercise.
Discuss interests, exercise opportunities (dog? friends?
neighborhood? community center/church classes? danc- Answer: 2
ing? nature?). Response 2 is the best answer (17), although 3 and 4 are also
Start small and work up; encourage any increase in ac- reasonable. Her exercise regimen is too intense for her
tivity. starting out. She likely is driving her blood sugars up
Methods of monitoring and feedback can help (pedom- through stress responses to the intensive regimen (and ex-
eters, Fitbit) as can nonfood rewards. pectations). Increased food consumption is possible, but
with her weight loss goals, she is less likely to be overin-
MAIN CONCLUSIONS dulging. Lows leading to rebound highs are possible, but it
Physical activity has clear benefits in type 2 diabetes, sounds like her control was suboptimal to start with and
including metabolic, vascular, and other benefits. Greater lows are relatively unlikely. The immediate goal is to pro-
intensity and greater duration increase benefit, but any vide her with recommendations regarding intensity and fre-
increase over baseline helps. quency with a gradual progression to longer and more in-
Multiple barriers interfere with exercise compliance in tense work-outs, the occasional day off, limiting resistance
type 2 diabetes, including physiological defects that training to 23 times a week, and moderating her expecta-
cause exercise to feel harder, time constraints, tions on weight loss while encouraging her that exercise
environmental barriers, fear, and cultural and even without weight loss will reduce her changes of needing
psychosocial barriers.
insulin in the near future.
Guidelines for physical activity in type 2 diabetes
Case 2: A 60-year-old overweight male with type 2 diabetes is
recommend a total weekly dose of 150 minutes of
referred to you for exercise-induced hypoglycemia. His diabetes
moderate aerobic exercise with near-daily bouts and
is currently managed with metformin, a dipeptidyl peptidase 4
twice weekly resistance training.
Exercise prescription has to be patient specific, taking inhibitor, a thiazolidinedione, and basal insulin at 50-U daily. His
into account patient complications and glycemic control, last hemoglobin A1c was 8.0. He has mild neuropathy and no
limitations, goals and expectations, and especially other known complications. He recently retired and has started
interests and motivating factors. taking a brisk hour-long walk every morning. He feels great but
The primary goal is to get people to be more active than has started having symptomatic low blood sugars (as low as 40s)
baseline and working towards at least the guidelines- overnight.

The Endocrine Society. Downloaded from press.endocrine.org by [${individualUser.displayName}] on 12 January 2017. at 12:19 For personal use only. No other uses without permission. . All rights reserved.
ENDO 2016 DIABETES AND GLUCOSE METABOLISM 117

Question: What Advise Should You Give? pearance, mood or sleep (in this likely recently postmenopausal
1. Decrease basal insulin dose. empty-nester), avoidance of insulin or additional medications,
2. Decrease pace or duration of his walk. or avoidance of complications. Because she obviously has
3. Discontinue one of his oral medications. friends that are important to her, these may be a source of
4. Eat a snack at bedtime to avoid lows. motivation and provide partners to help increase her activity
level. If she is at all open to being more active, creative
Answer: 1 suggestions of how to do this without additional time or finan-
Exercise-induced hypoglycemia can occur during or immedi- cial requirements may get her started. Finally, many people
ately after exercise or can be delayed and occur anytime for at respond to a hard outcome like step count or some fitness ap
least 24 hours after exercise. In this case, the improved insulin or technology as a motivator. Overall, this is a patient who is
sensitivity and possibly glycogen depletion from the exercise is not going to leave your office that day ready to exercise. It will
causing him to be overinsulinized overnight. Discontinuing one likely take time and repeated conversations to get her there.
of his orals could relieve this by making him more insulin
resistant again, but this is not the best goal. In addition, eating
REFERENCES
a snack at bedtime could protect him from lows, but encourag- 1. Senemmari B. Combating the diabetes epidemic. Caring. 2005;24(6):6-12.
ing extra eating is rarely a good approach in type 2 diabetes. 2. http://www.healthline.com/health/type-2-diabetes/statistics#3.
Decreasing his basal insulin dose will reduce his overnight 3. Boule NG, Haddad E, Kenny GP, Wells GA, Sigal RJ. Effects of exercise
on glycemic control and body mass in type 2 diabetes mellitus: a meta-
insulin (even though he takes it in the morning) and potentially
analysis of controlled clinical trials. JAMA. 2001;286(10):1218-1227.
improve his chances of losing weight with his exercise (18). 4. Sluik D, Buijsse B, Muckelbauer R, et al. Physical activity and mortality in
Case 3: A 55-year-old woman with type 2 diabetes without individuals with diabetes mellitus: a prospective study and meta-analysis.
complications but with worsening glycemic control. She comes Arch Intern Med. 2012;172(17):1-11.
5. Wei M, Gibbons LW, Kampert JB, Nichaman MZ, Blair SN. Low cardio-
in for a routine visit. She works at a desk job and her children respiratory fitness and physical inactivity as predictors of mortality in men
are grown and out of the house. Her husband travels a lot for with type 2 diabetes. Ann Intern Med. 2000;132(8):605-611.
his work. Her evenings are typically spent in front of the TV or 6. Lee DC, Artero EG, Sui X, et al. Mortality trends in the general popula-
tion: the importance of cardiorespiratory fitness. J Psychopharmacol.
on the phone with friends. She flatly states that she is just not 2010;24(4 suppl):27-35.
interested in exercise. 7. McAuley PA, Myers JN, Abella JP, Tan SY, Froelicher VF. Exercise
capacity and body mass index as predictors of mortality among male
veterans with type 2 diabetes. Diabetes Care. 2007;30(6):1539-1543.
Question: How Can We Help Her Become More 8. Bouchard C, Blair SN, Church TS, et al. Adverse metabolic response to
Active? regular exercise: is it a rare or common occurrence? PLoS One.
1. Find a motivating factor for some kind of physical 2012;7(5):e37887.
activity. 9. Churchward-Venne TA, Tieland M, Verdijk LB, et al. There are no
nonresponders to resistance-type exercise training in older men and
2. Explore her disinterest in exercise and/or avoid using
women. J Am Med Dir Assoc. 2015;16:400-411.
the word exercise in any recommendations. 10. Reusch JEG, Bridenstein M, Regensteiner, JG. Type 2 diabetes mellitus
3. Ask her what her friends like to do and suggest talking and exercise impairment. Rev Endocr Metab Disord. 2013;14(1):77-86.
to them on walks or at the mall instead of on the 11. Huebschmann AG, Kohrt WM, Herlache L, et al. Type 2 diabetes exag-
phone. gerates exercise effort and impairs exercise performance in older women.
BMJ Open Diabetes Res Care. 2015;3:e000124.
4. Suggest putting some weights or a stationary bike by
12. Gregg EW, Gerzoff RB, Casperson CJ, Williamson DF, Narayan KM.
the couch and doing some light activity while watching Relationship of walking to mortality among US adults with diabetes. Arch
TV. Intern Med. 2003;163(12):1440-1447.
5. Offer her a pedometer and suggest that she try to 13. Smith TC, Wingard DL, Smith B, Kritz-Silverstein D, Barrett-Connor E.
increase her daily step count to more than 3000 steps a Walking decreases risk of cardiovascular disease mortality in older adults
with diabetes. J Clin Epidemiol. 2007;60(3):309-317.
day and work up from there.
14. Tanasescu M, Leitzmann MF, Rimm EB, Hu FB. Physical activity in
6. All of the above. relation to cardiovascular disease and total mortality among men with type
2 diabetes. Circulation. 2003;107:2435-2439.
Answer: 6 15. American Diabetes Association. Standards of medical care in diabetes
2014. Diabetes Care. 2014;37(suppl 1):S14-S80.
This is the type of exercise-resistant patient that we see so 16. Solomon TPJ, Malin SK, Karstoft K, Haus JM, Kirwin JP. The influ-
often. She is definitely in the precontemplation phase (19) and ence of hyperglycemia on the therapeutic effect of exercise on glyce-
either has no interest in or motivation to change her level of mic control in patients with type 2 diabetes mellitus. JAMA Intern Med.
2013;173(19):1834-1835.
activity or has some specific fear of exercise that leads to her
17. Waryasz GR, McDermott AY. Exercise prescription and the patient with
unqualified dismissal of exercise. Exploring her lack of interest type 2 diabetes: a clinical approach to optimizing patient outcomes. Am
could reveal an addressable fear of exercise. This alone could Acad Nurse Pract. 2010;22:217-227.
be enough to get her more active. Otherwise finding some 18. Younk LM, Mikeladze M, Tate D, Davis SN. Exercise-related hypoglyce-
mia in diabetes mellitus. Expert Rev Endocrinol Metab. 2011;6(1):93-108.
motivating force could help move her into the contemplation 19. Norcross JC, Krebs PM, Prochaska JO. Stages of change. J Clin Psychol.
and ultimately the action stage. Motivators could include ap- 2011;67:143-154.

The Endocrine Society. Downloaded from press.endocrine.org by [${individualUser.displayName}] on 12 January 2017. at 12:19 For personal use only. No other uses without permission. . All rights reserved.
118 ENDO 2016 MEET-THE-PROFESSOR CLINICAL CASE MANAGEMENT

Controversies and Consequences of Gestational Diabetes for Mother


and Child: More than Glucose

M36 Fetal Medicine; Ref. 25) were published demonstrating that


Presented, April 1 4, 2016 identification and treatment of GDM reduces excess fetal
growth and related complications (1). The Hyperglycemia and
Adverse Pregnancy Outcomes (HAPO) was ongoing during
Linda A Barbour, MD, MSPH. Departments of these interventional trials and was designed as a multicenter,
Medicine and Obstetrics and Gynecology; Divisions of multinational epidemiologic study of nearly 25 000 women to
Endocrinology and Maternal-Fetal Medicine, University of definitely determine, in a blinded fashion, at which level of
Colorado School of Medicine, Aurora, Colorado 80045, mild hyperglycemia adverse pregnancy outcomes occurred and
E-mail: lynn.barbour@ucdenver.edu was published in 2008 (19). It was the basis for the newest
diagnostic criteria proposed by the International Association of
Diabetes and Pregnancy Study Groups (IADPSG).
INTRODUCTION
Historical Overview
It was first noted in 1828 by Heinrich Gottleib Bennewitz (1) SIGNIFICANCE OF THE CLINICAL PROBLEM
that diabetes could manifest in pregnancy with subsequent GDM is a major public health concern due to rapidly increas-
resolution postpartum. He described a patient exhibiting excess ing rates and the recognized transgenerational risk of the intra-
production of sugar who delivered a baby of such robust and uterine environment of obesity and diabetes resulting in an
healthy character. . . you would have thought Hercules had increased risk of offspring obesity and diabetes (4), yet there is
begotten. It was noted a century later in the 1920s that there no consensus on how to diagnose GDM. Given that overweight
seemed to be a physiological glycosuria in pregnancy which or obesity are present in two thirds of women of childbearing
did not persist after pregnancy in some women (2) and the age, the GDM rates are escalating with 10 25% of women
significance was controversial. Fasting blood glucoses and glu- being diagnosed with GDM for those obstetric practices adopt-
cose tolerance tests were recommended to further investigate ing the IADPSG criteria, which include the Endocrine Society
the glycosuria in the 1930s to diagnose impaired carbohydrate (5), the American Diabetes Association (6), and the WHO (7).
intolerance of pregnancy, which could be associated with However, the American College of Physicians (ACOG) argues
macrosomia, toxemia, and an increased risk of perinatal mor- that treating up to one third of pregnant women for GDM will
tality. In 1964, OSullivan and Mahan published a study using outstrip existing resources. ACOG asserts that there are no
whole blood glucose and what has been considered to be the adequately powered randomized controlled trials (RCTs) using
gold standard for the diagnosis of gestational diabetes mellitus these criteria which demonstrate improved pregnancy out-
(GDM) in the United States. It was based on a 100-g oral comes compared with the currently used ACOG criteria, (8)
glucose tolerance test (OGTT) in 1013 pregnant women and which results in a prevalence of 510% of pregnant women,
validated on the maternal risk of developing type 2 diabetes in which was supported by the National Institutes of Health (NIH)
the next 8 years, not on adverse pregnancy outcomes. The Consensus Conference (9). The paradigm shift in the in the
authors also screened a group of women without known risk diagnosis of GDM and the unresolved controversies on the
factors by determining a 1-hour blood glucose value after the importance of glucose, lipids, and obesity in the contribution of
ingestion of a glucose load of 50 g. Carpenter and Coustan excess fetal fat accretion has also resulted in a complete lack of
converted OSullivans whole blood glucose which used the consensus on the most appropriate diet and medical strategies
Somogyi-Nelson method to plasma measured enzymatically (1). to treat GDM. This session will review why there remains no
Sacks subsequently confirmed the conversion of OSullivans val- diagnostic standard, no agreement on whether excess dietary
ues to those in venous plasma by glucose oxidase (1). However, carbohydrate or fat should be reduced, and whether oral hypo-
the World Health Organization (WHO) used 75-g 2-hour glycemic therapy is safe given the unknown offspring effects
OGTT and in 1996 Sacks et al (3) published an open observa- on hepatic, pancreatic, or fat development.
tional study, Toward Universal Criteria for Gestational Diabe-
tes, using a 75-g OGTT in 3505 women and demonstrated a BARRIERS TO OPTIMAL PRACTICE
continuous relationship between fasting, 1-hour, and 2-hour New diagnostic criteria proposed by the IADPSG will triple the
OGTT values and birth-weight percentile and macrosomia. In prevalence of GDM to 18% in a normal-risk population and
2005 and 2009, two randomized controlled trials using the 75-g as high as 30% in a higher-risk population (10, 11). Whether
2 hour OGTT WHO criteria (Australian Carbohydrate Intoler- the treatment of women with these milder degrees of hypergly-
ance Study in Pregnant Women; ACHOIS; Ref. 24) and the cemia will improve pregnancy outcomes and is cost effective is
100 g 3-hour OGTT Carpenter and Coustan criteria (Maternal- suggested by a trial in Madrid Spain (11) but unproven in the

The Endocrine Society. Downloaded from press.endocrine.org by [${individualUser.displayName}] on 12 January 2017. at 12:19 For personal use only. No other uses without permission. . All rights reserved.
ENDO 2016 DIABETES AND GLUCOSE METABOLISM 119

United States due to a lack of RCTs and is questioned given the risk by 1.75-fold for large-for-gestational-age (LGA) infants
powerful effect of obesity alone on excess fetal growth. There and an elevated cord-blood C-peptide, consistent with fetal
are also data that question the prescription of the conventional hyperinsulinemia. Furthermore, the FBG was more strongly
carbohydrate-restricted diet supported by ACOG (8) and the predictive of these outcomes than the 1-hour or 2-hour values.
Endocrine Society (5) and that substituting fat for carbohydrate The IADPSG and later the American Diabetes Association
to blunt postprandial glucoses may be counterproductive and adopted the new diagnostic criteria using a 2-hour 75-g OGTT
worsen maternal insulin resistance (IR), resulting in a flux of based on a 1.75-fold increased risk of LGA in hopes that, for
all excess nutrients to the fetus. Furthermore, there are data the first time, all international groups could agree on using the
supporting that free fatty acids (FFAs) and triglycerides (TGs) same glucose load and the same diagnostic criteria (1, 20). The
may be an even more powerful substrate for excess fetal fat IADPSG also sought to modify the previous definition of GDM
accretion and a higher complex-carbohydrate/low-fat diet may as a glucose-intolerant state with onset or first recognition
be more advantageous in blunting worsening maternal IR and during pregnancy given that many women with undiagnosed
excess fetal growth (12-15). Although it is attractive to use oral pre-existing (overt) diabetes were being referred to as GDM.
hypoglycemic agents for this growing number of GDM women Given that undiagnosed women with DM have a much higher
for whom diet therapy fails, metformin has a high failure rate, risk of maternal and fetal complications, including major mal-
it freely crosses the placenta, and there are limited data that it formations if their glycated hemoglobin (HgbA1c) is at least
might increase offspring sc fat (12, 16). There are also concerns 6.5%, the IADPSG and ADA recommended that overt (pre-
about the efficacy of glyburide, which crosses the placenta more existing) diabetes (not GDM) be diagnosed if any of the fol-
than previously appreciated, and that its pharmacokinetics may lowing criteria are fulfilled during pregnancy: HgbA1c of at
result in significant maternal hypoglycemia (17, 18). least 6.5; FBG at least 126 mg/dL, or random glucose at least
200 mg/dL, the same criteria for DM outside of pregnancy (1,
6, 20). Furthermore, although they recommended screening all
LEARNING OBJECTIVES
pregnant women with one of these tests early in pregnancy for
As a result of participating in this session, learners should be
pre-existing diabetes, they did not mandate that a 75-g OGTT
able to:
be done on all women at high risk for GDM (obesity, polycys-
Understand the basis for the different diagnostic criteria
tic ovary syndrome, previous macrosomic infant, first-degree
for GDM between organizations, why it is hotly
relative with diabetes), unlike the ACOG criteria, which
contested, and how to diagnosis pre-existing diabetes
screens all women at risk for GDM early with a 50-g glucose
mellitus (DM) for the first time in pregnancy.
challenge as part of two-step testing approach (8).
Appreciate why there is no consensus on the optimal
Adopting the IADPSG criteria resulted in a tripling of the
diet for GDM women and the potential for a lower-
prevalence of GDM (1, 8, 10, 11, 20, 21). As a result of the
carbohydrate/higher-fat diet to worsen maternal IR,
lack of any RCTs demonstrating that adopting the new criteria
increase fetal fat accretion, and promote the fetal
and treating milder GDM will improve pregnancy outcomes,
programming of obesity or nonalcoholic fatty liver
the Carpenter and Coustan diagnostic criteria continue to be
disease.
used by most U.S. obstetricians using two abnormal values on
Recognize the risks/limitations of using metformin or
a 3-hour 100-g OGTT (FBG 95 mg/dL; 1 h 180; 2 h
glyburide in GDM pregnancies as a result of their
155; 3 h 140) if a 50-g glucose screen is abnormal
transplacental crossing, effectiveness, pharmacokinetics,
( 130-140 mg/dL) (8). Critics of the IADPSG criteria includ-
and potential long-term consequences for the mother and
ing Canada (22) emphasized that a 2.0-fold increase in LGA
offspring.
risk instead of 1.75-fold could have been chosen, which has
been adopted as the preferred diagnostic criteria according to
STRATEGIES FOR DIAGNOSIS, THERAPY, the Canadian guidelines, although the alternative diagnostic
AND/OR MANAGEMENT choice is according to the IADPSG. However, it is estimated
Diagnosis of GDM that U.S. women have a prediabetes rate of approximately 26%
There is compelling evidence based on the landmark HAPO and not very different than the predicted GDM rate using the
(Hyperglycemia and Adverse Pregnancy Outcomes) trial in IADPSG criteria at 1.75 increased risk (1). Although the
23 316 women (1, 19, 20) which used a 2-hour 75-g OGTT and IADPSG initially suggested that a FBG of 92 mg/dL may be
demonstrated that the level of maternal hyperglycemia at which used to diagnose GDM early in pregnancy, this recommenda-
adverse pregnancy outcomes increase are lower than the diag- tion has been rescinded due to recent data from China demon-
nostic criteria typically used for GDM in the United States strating a clear decrease in FBG in early pregnancy and show-
(Carpenter and Coustan criteria for the 3-hour 100-g OGTT). ing that this level is poorly predictive of GDM at 24-28 weeks
The HAPO trial (1, 19, 20) demonstrated that a fasting blood gestation (21). Data from New Zealand demonstrated that an
glucose (FBG) of at least 92 mg/dL, a 1-hour value at least 180 glycated hemoglobin of at least 5.9% identified all cases of
mg/dL, or a 2-hour value of at least 153 mg/dL increased the WHO-diagnosed GDM in pregnancy and was associated with a

The Endocrine Society. Downloaded from press.endocrine.org by [${individualUser.displayName}] on 12 January 2017. at 12:19 For personal use only. No other uses without permission. . All rights reserved.
120 ENDO 2016 MEET-THE-PROFESSOR CLINICAL CASE MANAGEMENT

2-fold risk of congenital anomalies, preeclampsia, shoulder mended by IADPSG would significantly benefit the much
dystocia, and a 3-fold risk of perinatal deaths (21). However, larger population making criteria for GDM in the United
the use of the A1C to diagnose GDM early has not currently States, potentially outstripping the resources to treat it. They
been officially endorsed by major national bodies. Further- also argued that 78% of LGA infants occurred in mainly obese
more, the HgbA1c is the least-sensitive test to diagnosis either women who did not make criteria for GDM in the HAPO trial.
prediabetes or diabetes (6), especially in anemia or high red A recent retrospective review of nearly 10 000 women who
blood cell turnover states such as pregnancy. Although the were diagnosed with GDM using the IADPSG criteria showed
FBG is most often abnormal in Latina populations who have an overall GDM prevalence of 24% but the vast majority of
hepatic IR, it is less sensitive than an OGTT, especially for LGA was attributable to being overweight and obese (10). The
Asian women who have been shown to typically have normal NIH further provided data from a pooled meta-analysis of five
FBGs and elevated 1-2-hour values (23). This raises the ques- RCTs showing treatment of GDM resulted in an absolute
tion as to whether early diagnostic testing recommended by the difference in birth weight of less than 150 g and only a 6%
IADPSG to screen for pre-existing diabetes will miss many absolute risk reduction of LGA. The NIH recommended that
women with pre-existing glucose intolerance or impaired fast- further RCTs be performed to pit the diagnostic criteria against
ing glucose. According to IADPSG, diagnosing and treating
each other to see whether adverse outcomes are improved in a
early GDM is yet to be supported due to the absence of any
cost effective manner (Table 1).
RCTs but high-risk populations for early GDM are screened,
diagnosed, and treated per ACOG.
Optimal Diet Therapy for GDM
Currently there is no consensus about the adoption of the
Determining the optimal macronutrient composition for dietary
IADPSG criteria over the ACOG criteria with the Endocrine
management of GDM has tremendous potential to improve
Society (ES) adopting IADPSG, the WHO adopting IADPSG
(based on weak evidence), and the ADA most recently chang- perinatal outcomes and deter the need for expensive medical
ing their endorsement to either criteria (6). The NIH held a management, especially given a GDM prevalence of 10 30%
Consensus Conference in March of 2013 (8) concluding that if IADPSG is used. Furthermore, fetal metabolic programming
the 75-g OGTT would be beneficial to standardize diagnostic from excess maternal nutrient availability including glucose,
criteria internationally and it is only the test in which glucose TGs, and FFAs has been strongly implicated in human epide-
thresholds for adverse fetal outcomes have been directly vali- miologic studies and animal models as an independent predic-
dated. However, the two RCTs showing benefit for the identi- tor for the development of obesity, metabolic syndrome, and
fication and treatment of GDM women did not use the diabetes in the offspring (4, 12, 26). A minimum of 175 g of
IADPSG criteria (24, 25).The NIH also voiced concerns that carbohydrate is recommended for fetal-placental energy needs
using a single value to diagnose GDM could result in more but there remains no consensus on the optimal diet for women
false positives given that approximately 25% of women have with GDM to minimize both the need for medication and
different results if performed at different times (9). The NIH prevent excess fetal overgrowth. Surprisingly, there are less
concluded that there were insufficient data from RCTs demon- than a dozen RCT diet trials in GDM women (13) and no large
strating that adopting the lower glucose thresholds recom- trials which directly provide the maternal diet to ensure com-

TABLE 1. Diagnostic Criteria for Gestational Diabetes


ACOG IADPSG ADA ES WHO Canada
2 step: A1C 6.5% A1C 6.5 Same as IADPSG Same as IADPSG Preferred:
50 g screen 130-140 mg/dl FBG 126 mg/dL FBG 126 but data weak 2 step
Random 200 Random 200 50 g 140
is Overt DM is Overt DM 75 OGTT
1 step Either 1 step or 2 step 95
Early Diagnosis: 191
A1C 5.9?
162
(2 Risk)
100 g OGTT (2) 75 mg OGTT (1) Either ACOG or IADPSG 75 mg OGTT 75 mg OGTT
(initially favored IADPSG)
95 92 (1) (1)
180 180 92 92
155 153 180 180
145 153 153

The Endocrine Society. Downloaded from press.endocrine.org by [${individualUser.displayName}] on 12 January 2017. at 12:19 For personal use only. No other uses without permission. . All rights reserved.
ENDO 2016 DIABETES AND GLUCOSE METABOLISM 121

pliance given the limited accuracy of dietary questionnaires largest experience with metformin has been in GDM women
and food records from adherence difficulties and recall bias. later in pregnancy from the Metformin in Gestation (MiG) trial
Although historically, diet therapy has focused on lowering (31) in which 751 women with GDM were randomly assigned
simple carbohydrates to blunt the postprandial glucose excur- to metformin vs insulin after exclusion of hypertension and
sion, women often substitute fat for carbohydrates given that fetal growth restriction. Metformin did not seem to increase
protein content is held remarkably constant at 1520% of total any adverse outcomes although it was associated with a slight
calories. In addition to maternal glucose, studies support that increase in preterm birth. Importantly, 46% of the women in
maternal TG and FFAs can be used by the placenta and may be the metformin group required supplemental insulin. In another
a stronger predictor of excess fetal fat accretion than maternal smaller RCT, metformin had a 32% failure rate and these
glucose, raising the question as to whether glycemia should be women were more obese, had higher FBG levels, and exhibited
the sole criteria for diet recommendations (13). Furthermore, an earlier need for pharmacologic treatment (17). The offspring
the substitution of carbohydrate with fat, especially saturated in the MiG trial were followed at 2 years and this study
fat, may worsen maternal IR (27), resulting in a flux of all demonstrated that the children exposed to metformin had larger
nutrients (glucose, lipids, amino acids) across the placenta and measures of sc fat (32). A greater increase in TG was seen in
drive fetal overgrowth. Small RCTs in which a high complex- the metformin group, and maternal TG, C-peptide, and mater-
carbohydrate/lower-fat (60%/25%) diet was compared with the nal body mass index were correlated with LGA and anthropo-
conventional low-carbohydrate /higher-fat (40%/45%) diet in metric measures of infant adiposity (33). Another RCT in
which all meals were provided to the subjects suggest that women with polycystic ovary syndrome comparing metformin
higher complex carbohydrates are well tolerated, control post- to placebo showed that although women randomized to receive
prandial glycemia by continuous glucose monitoring (CGM), metformin gained less weight during pregnancy, at 1 year
and result in lower postprandial FFAs, resulting in an attenua- postpartum the women who used metformin in pregnancy lost
tion of IR and a trend toward decreasing infant adiposity (14, less weight and their infants were heavier (34).
15). A high-fat diet has also been shown in nonhuman primates Metformin is concentrated in the fetal compartment with
to result in lipid deposition in the fetal liver in addition to umbilical artery and vein levels being at least equal to the
causing changes in the offspring metabolome, appetite regula- maternal serum (17). Hypothetically, if metformin increases
tion, and may affect serotonergic systems resulting in anxiety insulin sensitivity in the fetus, it might be possible for excess
(4, 12, 13, 27). The effect of a high-fat diet on the microbiome nutrient flux across the placenta to result in increased fetal
may also be associated with endotoxin release, resulting in adipogenesis. Its potential long-term effect on gluconeogenic
worsening maternal inflammation and an obesogenic enzymes in the fetal liver is unknown. Although the ADA
microbiome (28). Overall, the available evidence would sup- recommends that continuing metformin beyond the first trimes-
port that a diet higher in complex carbohydrate and fiber, low ter should be studied in the context of a trial, ACOG recently
in simple sugar (low glycemic index), and lower in saturated stated that insulin and oral medications are equivalent in
fat may be effective in blunting maternal postprandial hyper- efficacy, yet they acknowledge that 20 45% of women fail
glycemia and the worsening of maternal IR and preventing metformin alone, necessitating that insulin be added (8). They
excess fetal growth. However, studies on optimal diet therapy also suggest a role for counseling women that although current
in this rapidly growing population is an area in grave need for data do not demonstrate any adverse short-term outcomes,
high-quality RCTs (29, 30) as evident in Table 2. long-term outcomes have yet to be studied.
Glibenclamide (glyburide) is the only sulfonylurea that has
Controversies in the Use of Metformin or Glyburide in been studied in large RCTs in women with GDM (12, 17, 18),
Pregnancy crosses the placenta less well than metformin, and was ap-
Metformin is an attractive option to treat the growing number proved by the ADA and ACOG as a possible alternative to
of women with GDM given that it does not cause maternal insulin in GDM women (8). In some trials, maternal glycemic
hypoglycemia and is fairly inexpensive and easy to dose. The control, macrosomia (body weight 4000 g), neonatal hypo-

TABLE 2. Recommendations for Macronutrient Diet Composition in Gestational Diabetes


Endocrine AHA/ACC Lifestyle to
ADA 5th International GDM 2007 ADA MNT ACOG #137 Society 2013 Reduce CVD Risk 2013*
Diabetes 2013* 2013
Insufficient Evidence Inconclusive Carbs: 33-40% Carbs: 35-45% Carbs: 55-59%
recommendation withdrawn
*Individualization Fat: 40% Fat: 26-27%
needed
Pro: 15-18%

The Endocrine Society. Downloaded from press.endocrine.org by [${individualUser.displayName}] on 12 January 2017. at 12:19 For personal use only. No other uses without permission. . All rights reserved.
122 ENDO 2016 MEET-THE-PROFESSOR CLINICAL CASE MANAGEMENT

glycemia, and neonatal outcomes were not different between implications is thwarted by a lack of international consensus on
groups although in others, there was a significantly greater rate its diagnosis and clearly there is a critical need for carefully
of macrosomic infants in the glyburide group (17, 18). Al- executed RCTs to also address the optimal diet and treatment
though it was initially thought not to appreciably cross the for GDM and obese women.
placenta or significantly affect fetal insulin levels, a recent
publication using HPLC mass spectrometry suggested a modest
CASES
amount does cross (17). An RCT compared the efficacy of Case 1
metformin with glyburide for glycemic control in GDM (17) A 31-year-old Gravida 1 Para 0 at 28 weeks gestation comes
showed a failure rate of 35% in the metformin group and to you for a second opinion because she has been told conflict-
16% in the glyburide group, resulting in a need for insulin. ing opinions about the results of her glucose tolerance tests.
Kahn (35) and our group (12) demonstrated that it has a high She had an initial 50-g glucola of 200 mg/dL at 24 weeks. This
failure rate in women diagnosed with GDM at less than 24 was followed by an A1c of 6.6%. At 26 weeks she received a
weeks gestation and in women with fasting hyperglycemia. 75-g OGTT with a FBG of 90 mg/dL, a 1-hour of 178 mg/dL,
Although it does not appear to be teratogenic, it, like and a 2-hour of 153 mg/dL.
metformin, is very likely to fail as monotherapy in women with Which of the following statements are supported by the
type 2 diabetes mellitus. Due to its peak at 2-4 hours, many data?
women have inadequate control of their 1- or 2-hour postpran- A. The 50-g OGTT of 200 mg/dL already gave her a
dial glucoses and then become hypoglycemic 3-4 hours later. diagnosis of GDM and the 75-g OGTT was unnecessary.
Data also suggest that serum concentrations with usual doses B. Her A1c of 6.6% at 26 weeks already gave her the
are lower in pregnant women (17, 18). If used, it should be diagnosis of overt diabetes according to IADPSG
given 30 minutes to 1 hour before breakfast and dinner and criteria and the 75-g OGTT was unnecessary.
should not be given before bedtime due to the risk of early C. Given that the A1c of 6.6% was performed after 24
morning hypoglycemia in light of its 2-4-hour peak (similar to weeks, she does not meet criteria for overt diabetes by
Regular insulin). A retrospective cohort study using insurance IADPSG.
claims database in 9173 women treated with glyburide or D. The 75-g OGTT does not make criteria for GDM by
insulin demonstrated that newborns treated with glyburide IADPSG but given the 50-g glucose of 200 mg/dL, a
were more likely to experience adverse outcomes than those 100-g OGTT should be obtained to confirm the GDM
treated with insulin (18). A recent meta-analysis suggested that diagnosis.
glyburide was inferior to insulin and treatment failures were
higher with metformin alone than glyburide alone (17).
Case 2
A patient of yours with GDM is getting contradictory recom-
MAIN CONCLUSIONS mendations of which diet to adhere to in order to avoid medical
There is unabated controversy about the ideal diagnostic ap- therapy. She was recommended a low-carbohydrate diet
proach to GDM and the optimal treatment with regard to both (35%) and was told that if she instead substituted eggs and
diet and medical therapy. Yet the stakes could not be higher sausage for cereal she would have lower postprandial glucose
given the increasing prevalence of GDM and the compelling excursions and a lower risk of needing medical therapy. How-
recent data that the intrauterine environment characterized by ever, she was warned by another practitioner that eating too
obesity, diabetes, and nutrient excess may have long-term met- much fat could worsen her insulin resistance.
abolic programming effects on the offspring (4, 12, 26). Future Despite the paucity of rigorous RCTs, which is best sup-
studies must address whether adopting the IADPSG guidelines ported by the data?
in the United States, which triples the prevalence of GDM, will A. Carbohydrate intake in women with GDM is optimal at
improve pregnancy outcomes and be cost effective or whether 125 g due to the clear utility in blunting postprandial
maternal obesity is the major driver of adverse pregnancy glucose excursions resulting in improved pregnancy
outcomes in women with GDM. Currently, a diet restricted in outcomes.
simple carbohydrates (lower glycemic index), saturated fats, B. The placenta has more of an ability to transport
and trans fats, which liberalizes vegetables, fiber, and promotes maternal glucose for fetal fat accretion than TG or
the use of monounsaturated fatty acids and Omega-3 fatty acids FFAs; thus, dietary fat can be liberalized but
is best supported. Although insulin is likely to be the most carbohydrates should be restricted to 125 g.
effective in treating women who fail diet given that it also C. Both carbohydrates and fat should be restricted and
suppresses FFAs, the use of metformin or glyburide is superior ideally, a diet with at least 35% protein has been
to not treating hyperglycemia for women who are highly reti- shown to enhance lean mass development in the baby
cent to use insulin or in whom cost is prohibitive (16). Advanc- and minimize the risk for macrosomia.
ing our understanding of GDM, and its major public health D. None of the above.

The Endocrine Society. Downloaded from press.endocrine.org by [${individualUser.displayName}] on 12 January 2017. at 12:19 For personal use only. No other uses without permission. . All rights reserved.
ENDO 2016 DIABETES AND GLUCOSE METABOLISM 123

Case 3 Case 3
A patient referred to you has failed diet and was given the Answer: A
option by her OB/GYN of starting metformin, glyburide, or Although the combination of longer-acting insulins such as
insulin. She has fasting hyperglycemia in addition to some mild Levemir with rapid-acting Aspart or Lispro have an advantage
postprandial hyperglycemia and was told she could try in achieving better glycemic control without hypoglycemia,
glyburide before bedtime or metformin instead of insulin. She especially in women with type 1 DM, NPH and Regular have
was also given an option of being started on twice a day effectively been used in GDM women and are likely to be
NPH/Regular with breakfast and dinner. more efficacious than either glyburide or metformin alone.
Which are true statements about glyburide, metformin, and Metformin crosses the placenta more readily than glyburide,
insulin? has a higher failure rate, and neither is superior to insulin.
A. Levemir and rapid-acting insulins (Aspart, Lispro) have
not been shown to be more effective in improving
REFERENCES
pregnancy outcomes compared with NPH and Regular 1. Mcintyre HD, Colagiuri S, Roglic G, Hod M. Diagnosis of GDM: A
insulin in women with GDM. suggested consensus. Best Pract Res Clin Obstet Gynaecol. 2015;29(2):
B. Metformin is superior to insulin given that it does not 194-205.
2. Mestman JH. Historical notes on diabetes and pregnancy. Endocrinologist.
cause maternal hypoglycemia and has been shown to
2002;12:224-242.
improve pregnancy outcomes as a result of increasing 3. Sacks DA, Greenspoon JS, Abu-Fadil S, Henry HM, Wolde-Tsadik G,
maternal insulin sensitivity. Yao JF. Toward universal criteria for gestational diabetes: the 75-gram glu-
C. Glyburide crosses the placenta more freely than cose tolerance test in pregnancy. Am J Obstet Gynecol. 1995;172(2 Pt
1):607-614.
metformin and should be avoided except in cases of
4. Barbour LA. Changing perspectives in pre-existing diabetes and obesity in
fasting hyperglycemia in which the patient refuses pregnancy: Maternal and infant short- and long-term outcomes.. Curr Opin
insulin. Endocrinol Diabetes Obes. 2014;21(4):257-263.
D. Glyburide failures in pregnancy are more common than 5. Blumer I, Hadar E, Hadden DR, et al. Diabetes and pregnancy: An
Endocrine Society clinical practice guideline. J Clin Endocrinol Metab.
metformin failures, and if oral hypoglycemic therapy is 2013;98:4227-4249.
preferred by the patient, metformin is more likely to be 6. American Diabetes Association. Standards of medical care in diabe-
effective. tes2014. Diabetes Care. 2014;37 Suppl 1:S14 S80.
7. World Health Organization. Diagnostic Criteria and Classification of
Hyperglycaemia First Detected in Pregnancy. Geneva, World Health Org,
DISCUSSION OF CASES AND ANSWERS 2013 (WHO/NMH/MND/13.2).
Case 1 8. American College of Obstetricians and Gynecologists. ACOG Practice
Bulletin: Gestational Diabetes Mellitus Vol. 137. Washington, DC, Ameri-
Answer: B can College of Obstetricians and Gynecologists, 2013, p. 1-11.
Per IADPSG criteria, an A1c of at least 6.5% makes criteria 9. National Institutes of Health. National Institutes of Health Consensus
for overt or pre-existing DM at any time during pregnancy. Development Conference: Diagnosing Gestational Diabetes Mellitus,
A 50-g OGTT of at least 200 alone does not make a diagnosis 2013. Accessed from: http://prevention.nih.gov/cdp/conferences/2013/gdm/
resources.
of GDM (although some practices may treat this as equivalent 10. Black MH, Sacks DA, Xiang A, et al. The relative contribution of
criteria) and a single 2-hour value of 153 mg/dL does meet prepregnancy overweight and obesity, gestational weight gain, and
IADPSG criteria for GDM. IADPSG-defined gestational diabetes mellitus to fetal overgrowth. Diabe-
tes Care. 2013;36:56-62.
11. Duran A, Saenz S, Torrejon MJ, et al. Introduction of IADPSG criteria for
Case 2 the screening and diagnosis of gestational diabetes mellitus results in
Answer: D improved pregnancy outcomes at a lower cost in a large cohort of pregnant
It is recommended that a minimum of 175 g of carbohydrate women: The St. Carlos Gestational Diabetes study. Diabetes Care.
2014;37:2442-2450.
be consumed in pregnancy given that most of the energy 12. Barbour LA. Unresolved controversies in gestational diabetes: Implica-
demands of the placenta and fetus require glucose due to an tions on maternal and infant health. Curr Opin Endocrinol Diabetes Obes.
inability to oxidize fat. Furthermore, maternal TGs and FFAs 2014;21(4):264-270.
can be used by the placenta for fetal fat accretion due to 13. Hernandez, TL, Anderson, MA, Chartier-Logan C, Friedman JE, Barbour
LA. Strategies in the nutritional management of gestational diabetes. Clin
placental lipoprotein lipase and fatty acid transport proteins. A Obstet Gynecol. 2013;56(4):803-815.
diet high in saturated fat may worsen maternal IR and complex 14. Hernandez TL, Van Pelt RE, Anderson MA, et al. Higher complex carbo-
carbohydrates with low glycemic indices are well tolerated in hydrate diet in gestational diabetes achieves glucose targets and lowers
postprandial lipids: A randomized crossover study. Diabetes Care.
pregnancy. Although there is no consensus, the data support
2014;37:1254-1262.
minimizing simple sugars, carbohydrates with a high glycemic 15. Hernandez TL, Van Pelt RE, Anderson Barbour LA, et al. Women with
index, saturated fats, and trans fats, and promote the intake of gestational diabetes randomized to a higher complex carbohydrate/low fat
lean protein, complex carbohydrates with lower glycemic indi- diet manifest lower adipose tissue insulin resistance, inflammation, glu-
cose, and free fatty acids. Diabetes Care. 2015. [Epub ahead of print]
ces, and healthier fats, which include monounsaturated fatty
16. Damm P, Mathiesen ER. Therapy for gestational diabetes mellitusTime
acid and polyunsaturated fatty acid, especially of the omega-3 for a change? Nat Rev Endocrinol. 2015;11(6):327-328.
type. 17. Balsells M, Garcia-Patterson A, Sola I, et al. Glibenclamide, metformin,

The Endocrine Society. Downloaded from press.endocrine.org by [${individualUser.displayName}] on 12 January 2017. at 12:19 For personal use only. No other uses without permission. . All rights reserved.
124 ENDO 2016 MEET-THE-PROFESSOR CLINICAL CASE MANAGEMENT

and insulin for the treatment of gestational diabetes: A systematic review tional diabetes mellitus: mild neonatal effects, a long-term threat to global
and meta-analysis. BMJ. 2015;350. health. J Pediatr. 2014;164(30):445-450.
18. Camelo Castillo CW, Boggess K, Sturmer T, et al. Association of Adverse 27. Deer J, Koska J, Ozias M, Reaven P. Dietary models of insulin resistance.
pregnancy outcomes with glyburide vs insulin in women with gestational Metabolism. 2015;64(2):163-171.
diabetes. JAMA Pediatr. 2015;169(5):452-458. 28. Soderborg, TK, Borengasser SJ, Barbour LA, Friedman JE. Microbial
19. HAPO Study Cooperative Research Group. Hyperglycemia and adverse transmission from mothers with obesity and diabetes: An innovative op-
pregnancy outcomes. N Engl J Med. 2008;358:1991-2002. portunity to interrupt a vicious cycle. Diabetologia. 2015, in press.
20. Metzger BE, Gabbe SG, Persson B, et al. International association of 29. Viana LV, Gross JL, Azevedo MJ. Dietary intervention in patients with
diabetes and pregnancy study groups recommendations on the diagnosis gestational diabetes mellitus: A systematic review and meta-analysis of
and classification of hyperglycemia in pregnancy. Diabetes Care. 2010;33: randomized clinical trials on maternal and newborn outcomes. Diabetes
676-682. Care. 2014;37:3345-3355.
21. McIntyre HD, Sacks DA, Barbour LA, et al. Issues with the diagnosis and 30. Evert AB, Boucher JL, Cypress M, et al. Nutrition therapy recommenda-
classification of hyperglycemia in early pregnancy. Diabetes Care. 2015 tions for the management of adults with diabetes. Diabetes Care. 2013;36:
[Epub ahead of print]. 3821-3842.
22. Thompson D, Berger H, Feig D, et al. Diabetes in Pregnancy. Can J 31. Rowan JA, Hague WM, Gao W, et al. Metformin versus insulin for the
Diabetes. 2013;37 Suppl 1:S168 S83. treatment of gestational diabetes. N Engl J Med. 2008;358:2003-2015.
23. Sacks DA, Hadden DR, Maresh M, et al. HAPO Study Cooperative 32. Rowan JA, Rush EC, Obolonkin V, Battin M, Wouldes T, Hague WM.
Research Group. Frequency of gestational diabetes mellitus at collaborat- Metformin in gestational diabetes: the offspring follow-up (MiG TOFU):
ing centers based on IADPSG consensus panel-recommended criteria: the Body composition at 2 years of age. Diabetes Care. 2013;36(7):1941-
Hyperglycemia and Adverse Pregnancy Outcome (HAPO) Study. Diab 1946.
Care. 2012;35:526-528. 33. Barrett HL, Dekker NH, Jones L. Determinants of maternal triglycerides in
24. Crowther CA, Hiller JE, Moss JR, et al. Effect of treatment of gestational women with gestational diabetes in the Metformin in Gestational Diabetes
diabetes mellitus on pregnancy outcomes. N Engl J Med. 2005;352:2477- (MIG) study. Diabetes Care. 2013;36:529-536.
2486. 34. Carlsen SM, Martinussen MP, Vanky E. Metformins effect on first-year
25. Landon MB, Spong CY, Thom E, et al. A multicenter, randomized trial of weight gain: A follow-up study. Pediatrics. 2012;130:e1222 e1226.
treatment for mild gestational diabetes. N Engl J Med. 2009;361:1339- 35. Kahn B, Davies J, Lynch A, Reynolds R, Barbour L. Predictors of
1348. glyburide failure in the treatment of gestational diabetes. Obstet Gynecol.
26. Mitanchez D, Burguet A, Simeoni U. Infants born to mothers with gesta- 2006;107:1306-1309.

The Endocrine Society. Downloaded from press.endocrine.org by [${individualUser.displayName}] on 12 January 2017. at 12:19 For personal use only. No other uses without permission. . All rights reserved.
ENDO 2016 DIABETES AND GLUCOSE METABOLISM 125

Management of DM1 in Athletes

M37 motes. Many of the strategies are similar to what Dr Lawrence


Presented, April 1 4, 2016 had learned on himself when he was rowing or playing tennis (7).
Take less insulin before and after aerobic exercise and take a
snack if glucose levels still decrease to hypoglycemic levels.
Michael C. Riddell, PhD. School of Kinesiology and Work on the prevention of postexercise, late-onset, nocturnal
Health Science, Muscle Health Research Centre, York hypoglycemia, as was first described to be a major clinical problem
University, Toronto M4G 2X1, Canada, E-mail: mriddell@ by MacDonald (12), remains an active area of research. Strategies to
yorku.ca manage competitive stress, intense exercise and exercise-associated
hyperglycemia are now starting to emerge (13).
INTRODUCTION
Historical Overview SIGNIFICANCE OF THE CLINICAL PROBLEM
Regular exercise has been recommended for hygienic and pre-
Regular exercise, although beneficial for a variety of health and
ventative reasons since before the times of ancient Greece (1).
fitness reasons, causes major disturbances in glucose homeo-
The Shushruta Samhita, an ancient Indian Sanskrit text (600
stasis in patients with type 1 diabetes. Normally, in nondiabet-
BC), noted that regular exercise could cause reductions in the
ics, insulin levels decrease and glucose counterregulatory hor-
sweetness of urine of people who had the affliction (2). In
mones increase on the onset of moderate-intensity aerobic
Greece, Celsus (25 BC to 50 AD) prescribed exercise for
exercise. A decrease in the insulin-to-glucagon ratio in the
persons with diabetes to improve their wellbeing (3). In the
portal circulation facilitates an increase in hepatic glucose
1800s, the French pharmacist A. Bouchardat (4) demonstrated
production that closely matches the increase in glucose dis-
that exercise improved the tolerance of carbohydrates in diabe-
posal into contracting muscle. The precise coordination of
tes, while in the late 1800s and early 1900s, Allen (5) and
Joslin (6) were the two main advocates of exercise for people hormones and glucose flux in type 1 diabetes is lost and
with diabetes. In Total Regulation in the Treatment of Diabetes patients typically have a decrease in glucose during aerobic
(1919), Allen and colleagues (5) describe several cases studies activities. The increase in insulin sensitivity in recovery, along
of the benefits of exercise of an aerobic nature on the blood with diminished glycogen stores, sets the stage for increased
glucose levels and glycosuria of different classes of diabetic late-onset (often nocturnal) hypoglycemic risk. In contrast,
patients. They had subjects climb upwards of 150 flights of large increases in catecholamines associated with brief intense
stairs each day and were able to demonstrate normal glucose exercise can exaggerate glucose production by the liver that
tolerance in a majority of their diabetic patients. Just after the exceeds glucose disposal and hyperglycemia can ensue in dia-
discovery of insulin, a physician named R.D. Lawrence (7), betes unless a physiological increase in circulating insulin
who happened to have juvenile diabetes (now called type 1 occurs. Understanding the complex coordination of hormones
diabetes), demonstrated that his own blood glucose levels de- and substrates to various forms of exercise are critical to
creased much more rapidly, often resulting in hypoglycemia, improve physiological insulin replacement and minimize gly-
when his insulin injection was followed by vigorous exercise. cemic disturbances in athletes with type 1 diabetes.
At approximately the same time, Hetzel (8) proposed that
severe persons with diabetes were best not to perform strenu- BARRIERS TO OPTIMAL PRACTICE
ous exercise (of an intensity that could only last 5 min), Barriers to optimal management of athletes with type 1 diabetes is
given that it promoted a further elevation in glucose levels. the myriad of factors that can affect glucose control during exer-
These early studies provided clues that short-term intense cise (competition stress, timing of exercise, insulin levels, exercise
exercise and prolonged moderate exercise cause different gly-
type and duration, fitness level, food intake, etc.) and the inferior
cemic responses in diabetes. The importance of glucose as a
way in which insulin is delivered. A fear of hypoglycemia in
fuel for exercise, and the links between hypoglycemia and
athletes with type 1 diabetes can often promote excessive strate-
fatigue, were also demonstrated in the 1920s and 1930s (9, 10).
gies such as over consuming carbohydrates and the withholding of
In the late 1970s, the first official exercise and diabetes confer-
insulin altogether. Postexercise late-onset hypoglycemia is a major
ence was held in California with much of the work presented
concern for athletes as is postexercise hyperglycemia after the end
on the mechanisms of normal metabolism during exercise and
of exhaustive exercise.
the disruptions caused by insulin-dependent and -independent
forms of the disease (11).
Today, thousands of athletes with type 1 diabetes compete at LEARNING OBJECTIVES
all levels of competition with clinical strategies in place to help As a result of participating in this session, learners should be
offset the rapid changes in glucose turnover that exercise pro- able to:

The Endocrine Society. Downloaded from press.endocrine.org by [${individualUser.displayName}] on 12 January 2017. at 12:19 For personal use only. No other uses without permission. . All rights reserved.
126 ENDO 2016 MEET-THE-PROFESSOR CLINICAL CASE MANAGEMENT

List the factors that determine the patterns in blood glucose TABLE 1. Guidelines for the Reduction of Pre-Meal Dose
responses to exercise in athletes with type 1 diabetes. of Rapid-Acting Insulin (e.g. Lispro, Aspart or Glulisine)
Apply nutritional and insulin adjustment strategies that for Patients With Type 1 Diabetes Mellitus in
will allow for better glucose management during and Anticipation of Post-Meal Aerobic Exercise
after intense exercise. Percent Reduction in Pre-Meal Bolus
Aerobic
Be facile with the benefits and limitations of continuous Exercise Insulin Dose
sc insulin infusion and continuous glucose monitoring Intensity 30 min of exercise 60 min of exercise
(CGM) for improved patient control during and after Low 25 50
intense exercise. Moderate 50 75
High 75

MANAGEMENT STRATEGIES
Regular physical activity should be encouraged, as it is critical
for health in diabetes. Health care providers should realize, aerobic activities, likely because of increases in catecholamine
however, that increasing regular physical activity does not levels (23). In these situations, less need exists for insulin dose
typically lower A1c levels in adults with type 1 (14). Different reductions to maintain euglycemia. Resistance-type exercise
types of exercise produce different disturbances in glucose (weight lifting, wrestling, etc.) (24) and sprint-based activities
homeostasis. Prolonged aerobic exercise typically promotes a (25) tend to increase glucose levels if done for a limited
reduction in glycemia, whereas brief intense activities may pro- duration (seconds to minutes). Postexercise late-onset hypogly-
cemia risk may be associated with these types of activities too
mote an increase (15). The types of exercise that athletes with
(26), perhaps because muscle glycogen stores must be replen-
diabetes perform vary greatly, from high-intensity anaerobic ac-
ished. In situations of exercise-associated increases in glucose
tivities (eg, combat sports, powerlifting, golf, sprints, diving) to
concentration, a conservative insulin bolus can be adminis-
slower, continuous, low-intensity aerobic movements (eg, long-
tered, as long as the administration is not close to bedtime for
distance running and cycling). Each type of activity has its own
fear of increased nocturnal hypoglycemia risk. A 50% bolus
tendency for change in glucose concentration. Moreover, the
insulin correction bolus seems to be a reasonable correction
blood glucose response to a given exercise activity is variable
dose in these situations (14).
from patient to patient and from day to day within a patient for a
To date, no guidelines exist for the optimal blood glucose
number of other reasons (insulin concentrations during the activ-
concentration for performance and competition. Theoretically,
ity, ambient temperature, patient training status, previous episodes
glucose levels that are too high might help reduce hypoglyce-
of hypoglycemia, previous bouts of exercise, etc.). Understanding
mia risk during exercise but may set the stage for earlier
what type of exercise is being performed on a regular basis is a
glycogen depletion and increased risk for dehydration (15).
critical first step in helping the patient.
The target range for glucose concentrations for prolonged
Blood glucose responses to a given exercise task is primarily aerobic activities (running, long-distance swimming, and indi-
affected by the timing of exercise relative to the last insulin vidual and team sports), based on a general consensus (27), is
bolus administration and the time of day in which the exercise between 90 and 250 mg/dL (5.0 13.9mM), depending on the
is performed (16-18). Understanding whether exercise is being timing of exercise relative to the last meal and insulin dose.
performed during times of peak insulin action is a second Anything above or below this wide range should be considered
critical step in helping the patient on appropriate management detrimental from an exercise performance and safety perspec-
strategies. In general, the more exercise that athletes with tive. In situations of pre-exercise hyperglycemia (blood glucose
diabetes perform, the more the hypoglycemia risk (19). concentration 250 mg/dL [14mM]), aerobic exercise can be
If aerobic exercise is being performed within 3 hours after a initiated as long as blood ketone levels are not elevated. A
meal with insulin administration, then hypoglycemic risk can conservative insulin correction may be considered to help
be mitigated by reductions in the mealtime insulin dose by lower glycemia toward a more suitable range and to help limit
2575%, depending on the intensity and duration of the activity excessive ketone production during the exercise. Most athletes
(see Table 1) (20). If prolonged aerobic exercise is being with type 1 diabetes demonstrate considerable decreases in
performed before meals or in a fasted state, then basal insulin blood glucose concentration during exercise even if starting
dose reductions are typically required. For patients on multiple- glucose concentrations are between 14 and 22mM even without
daily-insulin therapy, long-acting insulin dose can be reduced bolus insulin correction for the pre-exercise hyperglycemia (28,
by 20% on the evening before exercise and on the evening after 29). Only if insulin has been withheld for prolonged periods
exercise (21). If patients are on pump therapy, then temporary (eg, skipped insulin administration, pump basal insulin inter-
basal rate reductions can be performed during the exercise ruption due to infusion set malfunction, intentionally prolonged
period itself (17) and in the postexercise overnight period (22). pump disconnection) will glucose levels likely increase during
Intermittent higher-intensity-type exercise seems to have purely aerobic exercise performed in the fasting/postmeal absorptive
slightly less risk of hypoglycemia compared with continuous state (30).

The Endocrine Society. Downloaded from press.endocrine.org by [${individualUser.displayName}] on 12 January 2017. at 12:19 For personal use only. No other uses without permission. . All rights reserved.
ENDO 2016 DIABETES AND GLUCOSE METABOLISM 127

In general, before prolonged aerobic exercise, appropriate still cannot control for hypoglycemia. Her midafternoon exercise
meal planning includes adequate sources of both complex car- routine consists of 60 minutes of stationary cycling followed
bohydrate and protein up to 3 hours before the activity. Where by 20 minutes of elliptical work at a moderate-to-high inten-
exercise occurs less than 3 hours after a meal, the kinetics of sity, three to four times per week.
rapid-acting insulin analogs (aspart, lispro, glulisine) are such 1. Which of the following best explains her hypoglycemia?
that a dose reduction should be considered to help minimize A. Abnormal gastrointestinal motility
active insulin during the activity. Some research has shown B. Abnormal insulin absorption
pre-exercise meal protein (an average of 30 g 2 h prior to C. Incorrect insulin apsart-to-CHO ratio
aerobic exercise in adolescents with type 1 diabetes) decreases D. Incorrect basal insulin (glargine) dosing
hypoglycemia risk during aerobic exercise, albeit not as effec- E. In appropriate exercise regimen
tively as consuming an immediate pre-exercise carbohydrate 2. Based on your assessment of her hypoglycemia, which of
snack (31). Where no adjustment to premeal insulin delivery the following is the best next step in helping prevent
has occurred, up to 1 g of carbohydrate (CHO) per kg of body these episodes?
weight per hour of activity, dispersed throughout the exercise A. CHO intake of 0.51.0 g/min exercise prior to exer-
period, is considered optimal for performance and glucose cise session
stability (29, 32). Where pre-exercise insulin has been reduced, B. CHO intake of 1530 g after exercise session
then 0.3 0.5 g/kg per hour will likely be sufficient (33). Com- C. Reduce by 50% aspart dose for meal prior to each
petitions lasting less than 1 hour can be handled with just small exercise session
carbohydrate supplements prior to their start (1530 g) (27). D. Reduce by 25% the glargine dose on the night prior
Insulin-pump therapy is associated with lower rates of severe to each exercise session
hypoglycemia (34). Continuous glucose monitoring (CGM) can as- E. Delay aspart dose for meal prior to each exercise
sist young athletes with monitoring of glucose levels while session by 2 hours
detecting time-based changes and trends in glucose levels (35).
During exercise, stress hormones increase and influence glu-
Case 2: Post-Exercise Hyperglycemia and Nocturnal
cose metabolism, and changes are often rapid and unpredict- Hypoglycemia
able. By continuously monitoring glucose levels, these devices A 17-year-old boy (body mass index, 24 kg/m2) who has had
show promise for improved glucose control during exercise type 1 diabetes for seven years has been experiencing hyper-
(36). Automatic suspension of insulin delivery in insulin glycemia after competing in hockey games and then later that
pumps, facilitated by CGM, pumps seem to offer some preven- night he often experiences a significant hypoglycemic episode.
tion of postexercise nocturnal hypoglycemia, although thresh- He is currently managed with an insulin pump with lispro
old settings may need to be adjusted if a recent bout of insulin. He is more concerned with the overnight (0200 0300
hypoglycemia has occurred (37). Improved glucose data in real h) lows and has tried adding a large bedtime snack when he has
time with a predictive element may help inform appropriate had a game.
measures to avoid immediate postactivity hypoglycemia as 3. Which of the following is the most likely mechanism to
well as delayed hypoglycemia. explain this patients hyperglycemia after the hockey
game?
MAIN CONCLUSIONS A. Increase catecholamine tone
Glucose control during exercise remains a challenge for ath- B. Poor insulin absorption during exercise
letes during exercise. Although prolonged aerobic exercise C. Inadequate basal rate of insulin infusion
necessitates either reduced insulin levels or increase carbohy- D. Incorrect type of insulin in the pump
drate intake, intense exercise requires cautious insulin admin- 4. Which of the following therapeutic interventions would
istration in early recovery to help combat hyperglycemia. be most likely to prevent the hypoglycemic episodes?
A. Further increase in bedtime CHO intake
CASES WITH QUESTIONS B. Change to insulin aspart
Case 1: Aerobic Exercise and Hypoglycemia C. Temporary reduction in basal rate from bedtime to
A 26-year-old athletic woman (weight, 55 kg; runner) who has 0300 h following competition
had type 1 diabetes for 12 years expresses concern to her health D. Place pump on suspend for 2 hours after competition
care team about repeated episodes of hypoglycemia during her
aerobic workout (cycling and training on an elliptical ma- REFERENCES
chine). She is using a multiple-daily-injection insulin regimen, 1. Berryman JW, Park RJ. Sport and Exercise Science: Essays in the History
of Sports Medicine. Chicago: University of Illnois Press, 1992.
taking insulin glargine at bedtime and insulin aspart at meal-
2. Tipton CM. Susruta of India, an unrecognized contributor to the history of
times. She takes her aspart with every meal and glargine each exercise physiology. J Appl Physiol. 2008;104(6):1553-1556.
night. She begins exercising 4 hours after her mealtime injection and 3. Schadewaldt H. Diabetes: Its medical and cultural history. In: The History

The Endocrine Society. Downloaded from press.endocrine.org by [${individualUser.displayName}] on 12 January 2017. at 12:19 For personal use only. No other uses without permission. . All rights reserved.
128 ENDO 2016 MEET-THE-PROFESSOR CLINICAL CASE MANAGEMENT

of Diabetes Mellitus. von Engelhardt D, ed. Berlin Heidelberg: Springer- Preventing post-exercise nocturnal hypoglycemia in children with type 1
Verlag, 1989. diabetes. J Pediatr. 2010;157(5):784-8.e1.
4. Bouchardat M. Training or forced exercise in the treatment of diabetes. 23. Guelfi KJ, Ratnam N, Smythe GA, Jones TW, Fournier PA. Effect of
Am J Med Sci. 1867;254-255. intermittent high-intensity compared with continuous moderate exercise on
5. Allen FM, Stillman E, Fitz R. Total dietary regulation in the treatment of glucose production and utilization in individuals with type 1 diabetes.
diabetes. New York: Rockefeller Institute for Medical Research, 1919. Am J Physiol Endocrinol Metab. 2007;292(3):E865-E870.
6. Joslin EP. The Prevention of Diabetes Mellitus. JAMA. 1922;76(2):79-84. 24. Turner D, Gray BJ, Luzio S, et al. Similar magnitude of post-exercise
7. Lawrence RD. The effect of exercise on insulin action in diabetes. Br Med hyperglycemia despite manipulating resistance exercise intensity in type 1
J. 1926;1:648-650. diabetes individuals [published online April 28, 2015]. Scand J Med Sci
8. Hetzel KS. Muscular exercise in diabetes mellitus. Br J Med. 1925;1:102- Sports. doi:10.1111/sms.12472.
106. 25. Fahey AJ, Paramalingam N, Davey RJ, Davis EA, Jones TW, Fournier PA.
9. Levine S, Gordon B, Derick C. Some changes in the chemical constituents The effect of a short sprint on postexercise whole-body glucose production
of the blood following a marathon race. JAMA. 1924;82:1778-1779. and utilization rates in individuals with type 1 diabetes mellitus. J Clin
10. Christensen EH, Hansen O. Arbeitsfahigkeit und Ernahrung. Skand Arch Endocrinol Metab. 2012;97(11):4193-4200.
Physiol. 1939;81:160-171. 26. Yardley JE, Kenny GP, Perkins BA, et al. Resistance versus aerobic
11. Vranic M, Horvath S, Wahren J. Proceedings of a conference on diabetes exercise: Acute effects on glycemia in type 1 diabetes. Diabetes Care.
and exercise. Sponsored by the Kroc Foundation, Santa Ynez Valley, CA. 2013;36:537-542.
Diabetes. 1979;28(Suppl. 1):1-113. 27. Robertson K, Riddell MC, Guinhouya BC, Adolfsson P, Hanas R. ISPAD
12. MacDonald MJ. Postexercise late-onset hypoglycemia in insulin- Clinical Practice Consensus Guidelines 2014. Exercise in children and
dependent diabetic patients. Diabetes Care. 1987;10(5):584-588. adolescents with diabetes. Pediatr Diabetes. 2014;15 Suppl 20:203-223.
28. Tansey MJ, et al. The effects of aerobic exercise on glucose and counter-
13. Turner D, Luzio S, Gray BJ, et al. Algorithm that delivers an individual-
regulatory hormone concentrations in children with type 1 diabetes. Dia-
ized rapid-acting insulin dose after morning resistance exercise counters
betes Care. 2006;29:20-25.
post-exercise hyperglycaemia in people with Type 1 diabetes [published
29. Riddell MC, Bar-Or O, Ayub BV, Calvert RE, Heigenhauser GJ. Glucose
online July 29, 2015]. Diabet Med. doi:10.1111/dme.12870.
ingestion matched with total carbohydrate utilization attenuates hypogly-
14. Kennedy A, Nirantharakumar K, Chimen M, et al. Does exercise improve
cemia during exercise in adolescents with IDDM. Int J Sport Nutr. 1999;
glycaemic control in type 1 diabetes? A systematic review and meta-
9:24-34.
analysis. PLoS One. 2013;8(3):e58861.
30. Berger M, Berchtold P, Cuppers HJ, et al. Metabolic and hormonal effects
15. Galassetti P, Riddell MC. Exercise and type 1 diabetes (T1DM). Compr
of muscular exercise in juvenile type diabetics. Diabetologia. 1977;13(4):
Physiol. 2013;3(3):1309-1336.
355-365.
16. Franc S, Daoudi A, Pochat A, et al. Insulin-based strategies to prevent
31. Dube MC, Lavoie C, Galibois I, Weisnagel SJ. Nutritional strategies to
hypoglycaemia during and after exercise in adult patients with type 1 prevent hypoglycemia at exercise in diabetic adolescents. Med Sci Sports
diabetes on pump therapy: the DIABRASPORT randomized study. Diabe- Exerc. 2012;44:1427-1432.
tes Obes Metab. 2015;17(12):1150-1157. 32. Adolfsson P, Mattsson S, Jendle J. Evaluation of glucose control when a
17. Dube MC, Weisnagel SJ, Prudhomme D, Lavoie C. Is early and late new strategy of increased carbohydrate supply is implemented during
post-meal exercise so different in type 1 diabetic lispro users? Diabetes prolonged physical exercise in type 1 diabetes. Eur J Appl Physiol. 2015;
Res Clin Pract. 2006;72(2):128-134. 115(12):2599-2607.
18. Gomez AM, Gomez C, Aschner P, et al. Effects of performing morning 33. Murillo S, Brugnara L, Del Campo E, Yague I, Duenas B, Novials A.
versus afternoon exercise on glycemic control and hypoglycemia fre- Carbohydrate management in athletes with type 1 diabetes in a 10 km run
quency in type 1 diabetes patients on sensor-augmented insulin pump competition. Int J Sports Med. 2015;36(10):853-857.
therapy. J Diabetes Sci Technol. 2015;9(3):619-624. 34. Misso ML, Egberts KJ, Page M, OConnor, D, Shaw J. Continuous
19. Devadoss M, Kennedy L, Herbold N. Endurance athletes and type 1 subcutaneous insulin infusion (CSII) versus multiple insulin injections for
diabetes. Diabetes Educ. 2011;37(2):193-207. type 1 diabetes mellitus. Cochrane Database Syst Rev. 2010(1):CD005103.
20. Rabasa-Lhoret R, Bourque J, Ducros F, Chiasson JL. Guidelines for 35. Adolfsson P, Nilsson S, Lindblad B. Continuous glucose monitoring sys-
premeal insulin dose reduction for postprandial exercise of different inten- tem during physical exercise in adolescents with type 1 diabetes. Acta
sities and durations in type 1 diabetic subjects treated intensively with a Paediatr Oslo Nor. 2011;100(12):1603-1609.
basal-bolus insulin regimen (ultralente-lispro). Diabetes Care. 2001;24(4): 36. Riddell MC, Milliken J. Preventing exercise-induced hypoglycemia in type
625-630. 1 diabetes using real-time continuous glucose monitoring and a new
21. Campbell MD, Walker M, Bracken RM, et al. Insulin therapy and dietary carbohydrate intake algorithm: An observational field study. Diabetes
adjustments to normalize glycemia and prevent nocturnal hypoglycemia Technol Ther. 2011;13:819-825.
after evening exercise in type 1 diabetes: a randomized controlled trial. 37. Garg SK, Brazg RL, Bailey TS, et al. Hypoglycemia begets hypoglycemia:
BMJ Open Diabetes Res Care. 2015;3(1):e00008512. the order effect in the ASPIRE in-clinic study. Diabetes Technol Ther.
22. Taplin CE, Cobry E, Messer L, McFann K, Chase HP, Fiallo-Scharer R. 2014;16:125-130.

The Endocrine Society. Downloaded from press.endocrine.org by [${individualUser.displayName}] on 12 January 2017. at 12:19 For personal use only. No other uses without permission. . All rights reserved.
ENDO 2016 DIABETES AND GLUCOSE METABOLISM 129

Individualizing Management with Insulin Pumps

M54 hard to keep up with these practice tools and their


Presented, April 1 4, 2016 appropriate usage.

BARRIERS TO OPTIMAL PRACTICE


Anthony L. McCall, MD, PhD. Department of
Practicing and training clinicians often have not had
Medicine/Endocrinology and Metabolism and Center for
formal instruction nor experience regarding insulin
Diabetes Technology, University of Virginia Health
pumps and new tools, such as CGM and online data
System, Charlottesville, VA 22908, E-mail: alm3j@
analysis of monitoring.
virginia.edu
Practice patterns and pump and CGM use are changing
rapidly.
INTRODUCTION Practical aspects of pump use and CGM use are not
Brief Historical Overview of Insulin Pump Therapy clear from the current medical literature.
Early work in 1963 by Arnold Kadish (1) resulted in prototypes
of an iv insulin infusion for both glucagon and insulin. The size
of the device was that of an overstuffed large backpack. The LEARNING OBJECTIVES
concept emerging from this work was to establish a more As a result of participating in this session, learners should be
physiological delivery of insulin. This early prototype was able to:
inadequate for home clinical use. In the late 1970s a wearable Select appropriate candidates for an insulin pump or
device was developed. Two aspects of diabetes care made the CGM and delineate preparation for their use of insulin
pump practical. One was the ability to self-monitor finger stick pumps and CGM
blood glucose, albeit accuracy for reading colors on test strips Identify basic elements of pump management and
was modest at best. Another advance indirectly important to assessment of basal rates, meal bolus types, and
pumps was technology development for Hemoglobin A1c test- correction dosing safely
ing: this gave a way to quantify the potential long-term benefit Appreciate the advanced features of insulin pumps
of monitoring and use of pumps. including use of temporary basal rates and alternatives to
Concepts gradually came into practice with insulin pumps usual bolus
for variable basal rates that could be individually adjusted to Develop an approach for troubleshooting basal bolus
account for temporary basal rate needs such as for exercise and imbalance, pattern therapy, CGMs
the dawn phenomenon (2). In addition, use of correction dosing
to help catch up when temporary hyperglycemia occurred STRATEGIES FOR DIAGNOSIS, THERAPY,
now was possible. Gradually with clinical studies, mathemati- AND/OR MANAGEMENT
cal algorithms and increasing sophistication of equipment, we Why Choose an Insulin Pump
are now at the stage where insulin pumps have been connected Pros
to accurate continuous glucose monitoring, and can function in
early prototypes of an artificial or bionic endocrine pancreas. Flexible and adjustable basal rates help deal with daily
Research on artificial pancreas is on the verge of widespread rhythms
More reliable absorption
clinical applicability and offers relief for patients of some of
Less variability
the self-care burden of type 1 diabetes management and per-
Temp basal rates allowing one to deal better with sick
haps also for those with type 2 diabetes.
days and exercise
Precision in dosing
SIGNIFICANCE OF THE CLINICAL PROBLEM Variable bolus formats and bolus calculators to aid meal
Type 1 diabetes affects more than a million people in the
control
United States and continues to increase in prevalence.
Cons
As insulin deficiency advances, insulin pumps have
increasingly become a preferred therapy to achieve mimicry Body image issues
of physiological insulin delivery, potentially reducing It is just a machine and machines do fail
hypoglycemia and achieving better glucose control. Not for everyone
Insulin pumps keep evolving with new types and Requires knowledge and skill for effective and safe use
features including tools such as Continuous Glucose Involves dexterity, technical skills, and good hearing
Monitoring (CGM) that are increasingly integrated. It is It is more, not less work for patients

The Endocrine Society. Downloaded from press.endocrine.org by [${individualUser.displayName}] on 12 January 2017. at 12:19 For personal use only. No other uses without permission. . All rights reserved.
130 ENDO 2016 MEET-THE-PROFESSOR CLINICAL CASE MANAGEMENT

Figure 1. From Case 1 see below.

Who is a Pump Candidate? (see references 5,6,7) CASES NOTE MAINTENANCE OF


A pump candidate is someone who is: CERTIFICATION (MOC) WITH SEVERAL CASES
Realistic, mature, and motivated Case 1: A Patient With Poor Control and Multiple
Committed and capable of learning carbohydrate Complications
counting and its nuances This case illustrates recognition of basal bolus imbalance and
Willing to monitor blood glucoses frequently and keep discusses how to rectify this with gradual adjustments to basal
accurate records and meal insulin.
Dexterous with adequate visual acuity A 55-year-old man with type 1 diabetes mellitus (DM) and
Financially committed long history of poor glycemic control, known retinopathy,
Commited to working with and communicating with neuropathy, gastroparesis. The patient is fearful of bolus
Health Care Team insulin and is constantly grazing, especially in the evening. His
Limited by Multiple daily injections (eg, very active, A1c is 9.7%, he has been on insulin pump therapy for 3 years,
very insulin sensitive) and he has just started working with a DM educator to achieve
better control.
Preparing for Insulin Pump Use Due to his poor glycemic control and reported eating patterns,
Assess the patients understanding of advanced you suspect that there is a mismatch between his basal insulin,
carbohydrate counting and its nuances bolus insulin, and carbohydrate intake.
Establish the patients carbohydrate ratio/insulin 1. Which of the following best describes the usual distribution
sensitivity and glucose target of basal and bolus insulin in most adult patients on insulin
Have patient demonstrate knowledge of carbohydrate- pumps? See next page for best answer (MOC 1)
counting skills A. 6580% basal, 2035% bolus
Have patient send in blood glucose records or share B. 5065% basal, 3550% bolus
electronic information with health care providers to have C. 3550% basal, 5065% bolus
adequate information to assess knowledge base D. 2035% basal, 6580% bolus

The Endocrine Society. Downloaded from press.endocrine.org by [${individualUser.displayName}] on 12 January 2017. at 12:19 For personal use only. No other uses without permission. . All rights reserved.
ENDO 2016 DIABETES AND GLUCOSE METABOLISM 131

Figure 2. From Case 2 see below.

Question 1 MOC best answer C basal and meal insulin (4). He still struggled with monitoring
With too high a basal insulin level a patient may risk frequent enough to estimate meal dosing needs and remained inconsistent
hypoglycemia despite inadequate control. Practically speaking, it in use of extended boluses for his gastroparesis.
is difficult to rapidly change the balance of basal and meal insulin
quickly. To improve this it is suggested to make a small reduction Case 2: Worsening DM control-See CareLink
(eg, 0.1 U/h) for most or all of the day and try to use this to information above
provide more insulin for meal control and repeat this gradually A 22-year-old woman has had type 1 diabetes since the age of
over weeks to months until the appropriate balance is restored 3 years without complication. She has been using a Medtronic
[Refer to book by Kaufman et al (5)]. pump for several years. During the last 3 years, her A1c has
By gradually decreasing the high basal insulin (12 U at a time) increased from 7.2to 9.8% despite checking her blood sugars
and adding back meal insulin, this man acheived better balance of 4 times per day.

The Endocrine Society. Downloaded from press.endocrine.org by [${individualUser.displayName}] on 12 January 2017. at 12:19 For personal use only. No other uses without permission. . All rights reserved.
132 ENDO 2016 MEET-THE-PROFESSOR CLINICAL CASE MANAGEMENT

Figure 3. Highlighted areas of concern based on Case 2 CareLink information.

2. How will you figure out what the focus of your advice MOC question 2 on case 2
should be based on? A. Up to 3 times daily
3. What do you notice? B. 3-5 times daily
Figure 3 interpretation C. 5-8 times daily
A. Basal overdose D. At least 9 times daily
B. Boluses Blood glucose tests See MOC question 2 answer below Fig. 4.
C. Infrequent monitoring (25 glucose tests daily) Fig. 5 next page shows her pattern of basal rates.
D. Infrequent fills (not changing every 3 d) 5. What do you think it means when this patient
E. Not very consistent carbohydrates increases the basal rates at 0300, 0900, and 1800
To help this patient, it is clear that the wide range of glucose hours?
values on any given day reflect her infrequent home glucose
monitoring. You should consider the possibility and inquire whether
4. Which of the following would be your recommendation these are meal times.
to her for the minimal optimum frequency for home 6. Would 0300 hours usually be appropriate for a dawn
blood glucose tests? phenomenon?

The Endocrine Society. Downloaded from press.endocrine.org by [${individualUser.displayName}] on 12 January 2017. at 12:19 For personal use only. No other uses without permission. . All rights reserved.
ENDO 2016 DIABETES AND GLUCOSE METABOLISM 133

Figure 4. High testing frequency benefits pump users Figure 6. Basal rate testing possibilities.
[Davidson et al. (5)].

Figure 6.
MOC question 2 best answer is C
9. What if she does not do basal rate testing?
7. This patient states that she cannot raise her meal bolus
insulin or she becomes low within 2-3 hours. Which of Consider timing of the 0300 hours basal insulin increase:
the following would be the most likely explanation? is it dawn phenomenon or meal-related adjustments?
A. Excess reliance on basal insulin Ask about meal times
B. Excess insulin-to-carbohydrate bolus dosing Ask about meal composition
C. Incorrect carbohydrate counting for meals Reduce her basal rates overall and/or increase her meal
D. Abnormal gastric emptying causing insulin/carbohydrate insulin
mismatch
10. What is her bedtime-to-morning BG reduction?
8. What should you do next?
(BEMR)watch this.
Basal rate testing (diagrammed above) is appropriate. 11. What did you observe in CareLink?
Right dose 30 mg/dL, drift up means too little basal,
drift down means too much basal Case 3: How can this patient improve glycemic control
Preferably test when the patient is in: safely? See figure 7 on next page below
Moderate control, no exercise, no food or bolus for A 25-year-old woman has had type 1 diabetes for 10 years
several hours, and easy with continuous glucose without complication. Her A1c is 8.7%.
monitoring (CGM) (6). 12. Describe what the CareLink report suggests. Fig 7.
No stress, fasting till the end, 6-8 hours maximum
13. One aspect of the use of insulin pumps that is often
advantageous is the adaptability to changes in lifestyle
from day to day. For this patient, which of the
following changes would be the best first step in
allowing her to manage the wide excursions in
carbohydrate intake?
MOC question 3
A. Increase insulin sensitivity factor
B. Increase insulin-to-carbohydrate ratio
C. Decrease in basal insulin during daytime
D. Match home glucose monitoring to meals
Observations from CareLink: The basal is overdosed. The
number of BG tests is not consistent enough and does not
match the number of boluses given (then what is the basis
for the bolus?). Carbohydrates are not consistent. Possible
Figure 5. Basal insulin pattern Case 2. stacking.

The Endocrine Society. Downloaded from press.endocrine.org by [${individualUser.displayName}] on 12 January 2017. at 12:19 For personal use only. No other uses without permission. . All rights reserved.
134 ENDO 2016 MEET-THE-PROFESSOR CLINICAL CASE MANAGEMENT

Figure 7. CareLink information Case 3.

Best answer MOC question 3 is D. following would be the appropriate next step in her
management?
Case 4: Cannot Get the Sugars to Less Than 300 mg/dL MOC question 4
A 31-year-old woman with type 1 DM since the age of
A. Check urine ketones 4
13 years with no known complications calls on a holiday
B. Check urine pregnancy test
weekend stating that she received a steroid injection into
C. Add scheduled insulin bolus every 4 hours on top
her hand yesterday and cannot get her sugar to come to less
of Insulin sensitivity factor and insulin/carb ratio
than 300 mg/dL and she is noting urinary frequency and
dosing
some thirst. She gave herself several correction doses with
D. Initiate temporary increase in basal rate of 25% and
little improvement in glycemic control and is asking for
increase glucose tests to every 2-3 hours 4
advice.
E. Encourage fluid hydration without change of insu-
14. Her basal rates were recently raised from 0.7 to 0.8
lin dosing
U/h through the 24-hour day due to A1c of 10%.
16. What do you think is going on here?
She checks her sugar level 4-6 times daily. What do
you suggest?
Case 5
15. You review her total daily dose, basal insulin settings, This is an example from John Walsh of a 71-year-old man
insulin sensitivity factor, and insulin-carbohydrate ratios. with a high total daily dose of insulin who takes post-meal
She seems facile with the insulin pump. Which of the insulin and overcorrects. He is dangerously overtreated but

The Endocrine Society. Downloaded from press.endocrine.org by [${individualUser.displayName}] on 12 January 2017. at 12:19 For personal use only. No other uses without permission. . All rights reserved.
ENDO 2016 DIABETES AND GLUCOSE METABOLISM 135

Figure 8

is happy to have an A1c of less than 6.5%. He probably has with. If A1c is 10%, no lows very difficult to estimate
impaired hypoglycemia awareness. Insulin must be reduced accurately
and timing is adjusted to be more appropriate. Some apps are pretty good but most have not had
Best answer to MOC question 4 is D although A is also independent and thorough vetting: they may not take
correct. into account the mise en scene
Patient wishing tight control may correct to as low as
Issues With Correction Dosing 80-100: this is too aggressive and leads to overcorrection
Accurate estimation of ISF 1800 rule commonly
Use of other rules of thumb and correction dose Correction at bedtime is dangerous and yet can
estimation apps usually be done sparingly for marked hyperglycemia
Checking on target of correction: often too aggressive with an assumption that ISF is doubled or close to
and leads to overcorrection that and correction target is modest such as 150 mg/dL
Correction at bedtime (half or none) after hypoglycemia Pump patients do not always use Insulin on Board
rebound estimation or bolus calculators that track IOB and
Correction at meals vs after meals thus may wind up stacking doses for post prandial
Using CGM trends to adjust correction dosing highs

Issues With Correction Dosing continued Another way to deal with persistent hyperglycemia is to try
Use of a rule such as 1800 (1700) rule is dependent to use a temporary basal rate initially with a moderate
upon what kind of overall control the patient has to start increase

The Endocrine Society. Downloaded from press.endocrine.org by [${individualUser.displayName}] on 12 January 2017. at 12:19 For personal use only. No other uses without permission. . All rights reserved.
136 ENDO 2016 MEET-THE-PROFESSOR CLINICAL CASE MANAGEMENT

Figure 9. Modal day Continuous Glucose Monitoringnote middle of the night highs and very large drop by morning in
blood sugars.

Typically, most patients would start at 20 30% increase 20. What should you do? See references 4-7 for
(ie, 120 130% increase in temp basal for the period of being background
awake and check BG or use CGM to guide when to cancel
Consider use of Dual Wave Bolus for high-fat meals
the temp basal
Regular Bolus
With a rapid decrease of greater than 1 mg/dL/min consider
Used for low-fat, moderate-carbohydrate meals, some
10% reduction; with more than 2 mg/dL/min decrease probably
moderate snacks
cancel temp if getting near goal or decrease 20%
Dual Wave (Combined) Bolus
If initial temp basal not enough go up to 140%
Use mostly for large meals with high-fat, high-protein
In addition to temp basal with steroid injection (usually
last approximately 2.5 d with variable effect), may also need pizza, richer meals, eating out etc.
Start with 50/50 with 1-2-hour extension
substantial increase in insulin-to-carbohydrate ratio (lower
Proceed to extend based on glycemic escape (late BG
number that is) for a day or two.
increase)
Square Wave (Rectangle extended) Bolus
Case 6
A 39-year-old man has a 15-year history of type 1 diabetes Traditionally used for anticipated grazing
complicated by background diabetic retinopathy, HTN, and Sometimes used for gastroparesis or drugs causing de-
hyperlipidemia. His A1c is 8.5% and he is frustrated by his layed appearance of meal-related hyperglycemia
resistant fasting hyperglycemia. He eats out a lot and often Figure 10.
eats late. At bedtime (a couple of hours after eating), his Potential pearls for insulin pumps and CGM
BGs are pretty well controlled. Careful selection, training and follow up coaching for all
17. What do you recommend? Always ask about use of temp basal and extended
18. What are the causes of fasting hyperglycemia? (4) boluses
Somogyi effect (rare) Inquire about hypo frequency, timing, treatment, and
Dawn phenomenon (moderately common) symptoms
Extended postdinner/snack hyperglycemia (very common) Know common eating patterns of your patient
19. How do you determine which is the cause? In order Inquire about physical activity, types, duration, intensity,
below: timing
A. Hypoglycemia in the middle of the night Confirm their pump settings: basal, bolus settings (max)
B. Normal glycemia in the middle of the night Ask them I/C, ISF review (correction targets)
C. Hyperglycemia in the middle of the night Document TDD and basal/bolus balance every time

The Endocrine Society. Downloaded from press.endocrine.org by [${individualUser.displayName}] on 12 January 2017. at 12:19 For personal use only. No other uses without permission. . All rights reserved.
ENDO 2016 DIABETES AND GLUCOSE METABOLISM 137

Figure 10. Insulin needs and high fat meals.

Ask about calibration, CGM time used per day, week, 3. Wolpert HA, et al. Dietary fat acutely increases glucose concentrations and
etc. insulin requirements in patients with type 1 diabetes: Implications for
carbohydrate-based bolus dose calculation and intensive diabetes manage-
Document actual site changes and amount of monitoring. ment. Diabetes Care. 2013;36(4):810-816.
Beware the multiple-meter patient 4. McCall A. Insulin Therapy and Hypoglycemia. Endo Clinics NA, 2012.
Emphasize trends and their rapidity, not absolute 5. Kaufman FR, Westfall E. Insulin pumps and continuous glucose monitoring: A
numbers users guide to effective diabetes management. Alexandria, VA: ADA, 2012.
6. Scheiner G. Practical CGM: A guide to improving outcomes through con-
Ask about confirmation of lows and highs
tinuous glucose monitoring. Alexandria, VA: ADA, 2015.
7. Walsh J, Roberts R. Pumping insulin: Everything you need for success on
REFERENCES an insulin pump a new chapter on CGMs. San Diego: Torrey Pines Press,
1. Kadish AH. Automation control of blood sugar. A servomechanism for 2015.
glucose monitoring and control. Trans Am Soc Artif Int Org. 1963;19:363- 8. Davidson PC, et al. A cause-and-effect-based mathematical curvilinear
367. model that predicts the effects of self-monitoring of blood glucose fre-
2. Pickup JC. Banting Memorial Lecture 2014* Technology and diabetes care: quency on hemoglobin A1c and is suitable for statistical correlations.
Appropriate and personalized. Diabet Med. 2015;32(1):3-13. J Diabetes Sci Technol. 2007;1(6):850-856.

The Endocrine Society. Downloaded from press.endocrine.org by [${individualUser.displayName}] on 12 January 2017. at 12:19 For personal use only. No other uses without permission. . All rights reserved.
138 ENDO 2016 MEET-THE-PROFESSOR CLINICAL CASE MANAGEMENT

Diabetes in the Older Patient

M56 and ageism among staff. Behavioral change can be challenging


Presented, April 1 4, 2016 given that there can be cognitive inflexibility, poor self moti-
vation, lack of family support, financial challenges, and diffi-
culty setting priorities.
Graham T. McMahon, MD, MMSc. Accreditation
Council for Continuing Medical Education, Chicago,
BARRIERS TO OPTIMAL PRACTICE
Illinois 60654, E-mail: gmcmahon@accme.org
Insufficient time to provide comprehensive diabetes care
for these complex patients.
INTRODUCTION Inadequate team resources to be able to deliver ongoing
Historical Overview collaborative team care.
In 1900 the average survival for a patient diagnosed with
diabetes at age 30 years was approximately 4 years, and at age
LEARNING OBJECTIVES
50 years was approximately 8 years. Care of the older person
Recognize the opportunities to improve QOL for the
with diabetes had little relevance given that so few survived
older patient with diabetes.
very long. With the advent of therapies from the early 1900s
Appropriately select treatment targets based on the
including lower-carbohydrate diets, insulin, and a range of oral
presence of comorbidities.
medications, survival and quality of life (QOL) have improved
Choose medications appropriately for the older patient
dramatically, a trajectory that needs to continue.
with diabetes.
Anticipate the typical complications associated with
SIGNIFICANCE OF THE CLINICAL PROBLEM diabetes that impair QOL in the older patient.
The prevalence of diabetes increases with each decade. More
than 25% of the population of the United States greater than Case 1: Maria: Part 1
age 65 years has diabetes, and an even higher proportion of Maria, 76 years old, has been experiencing urinary inconti-
nursing home residents have diabetes. An additional one third nence and showing signs of confusion. She has not had fre-
of patients with diabetes are undiagnosed (1, 2). The increased quent urination or excessive thirst, but seems slightly dehy-
prevalence of diabetes in older people is attributable to several drated. Maria has no family history of diabetes and believes
factors: insulin resistance increases with age related to adipos- its a waste of time to screen her.
ity, sarcopenia, and physical activity; and insulin secretory 1. Is there any benefit in diagnosing diabetes in the older
function seems to decline with age. patient at all?
Diabetes can meaningfully affect the functional status and life
expectancy of older patients, and increase the risk for institution- Diabetes Prevention and Screening in the Older Patient
alization. Polypharmacy, falls, chronic pain, incontinence, cogni- Screening for diabetes is appropriate when there is a likelihood
tive impairment, and depression are all more common among of benefit of an intervention if diabetes is diagnosed. Screening
older patients with diabetes. Older patients have the highest rates is likely to be helpful in a reasonably healthy 70-year-old but
of lower-extremity amputation, myocardial infarction, visual im- less- or unhelpful in a 90-year-old with advanced comorbidi-
pairment, and end-stage kidney disease of any age group (3, 4). ties. In patients who are found to have features of prediabetes,
Although recently declining, deaths from hyperglycemic crises reversible causes [such as use of antipsychotic medicines for
remain significantly greater in older adults compared with younger example (5)] should be sought.
patients, and patients older than 75 years of age have double the In a patient such as Maria, screening her for diabetes can
rate of emergency department visits for hypoglycemia than the help identify a contributor to her incontinence and help pre-
general population with diabetes. vention the decline in cognitive function that can accompany
However, appropriate and effective management can mitigate untreated chronic hyperglycemia, while also helping to prevent
some of these risks. The patients age affects the social, medical, a hyperglycemic emergency.
and behavioral approaches to the patient. Caring for diabetes in Followup of the Diabetes Prevention Program cohort for 10
older patients necessitates consideration of their functional and years after randomization showed that lifestyle interventions
care status and comorbidities when setting treatment priorities, were especially effective in older participants (49% risk reduc-
and should prioritize the principal of doing no harm. tion in those age 60 y at randomization vs 34% for the total
Diabetes care in the older patient creates a particular burden cohort) and additional benefits of the lifestyle intervention that
on the care team given that there can be communication diffi- might affect older adults, such as reduction in urinary inconti-
culties, inadequate services for the complexity of the patient, nence, improvement in several QOL domains, and improve-

The Endocrine Society. Downloaded from press.endocrine.org by [${individualUser.displayName}] on 12 January 2017. at 12:19 For personal use only. No other uses without permission. . All rights reserved.
ENDO 2016 DIABETES AND GLUCOSE METABOLISM 139

ments in several cardiovascular risk factors. Thus, when inter- more than 70 000 patients with type 2 diabetes age greater than
vention is needed, lifestyle interventions are recommended 60 years demonstrated a U-shaped relationship between A1c
given that they seem to be especially useful in older patients. and mortality with lowest mortality at an A1c of approximately
Older patients do not tend to respond well to metformin. 7.5% (13).
In the absence of directive clinical trial data for this popu-
Maria: Part 2 lation, and given the potential harm associated with treatment
Maria is diagnosed with type 2 diabetes, and she and her that targets an A1c level below 6.5%, the general A1c target
family are agreeable to consider treatments that will enhance recommended by the American Geriatric Society (3) among
her QOL. others is 7.5% when it can be safely achieved in patients with
2. Which therapeutic targets would you prioritize for good functional status and few comorbidities. An A1c target of
Maria? between 7.5 and 8% is appropriate for many older patients, and
3. How would your approach to her care differ given her a target above 8% is appropriate for older adults with multiple
comorbidities? comorbidities, poor health, and limited life expectancy (1, 2).

Diabetes Care for the Older Patient Routine Screening of Older Patients for Diabetes
Complications
Choosing a Glycemic Target
Screening recommendations for older patients with diabetes
Older patients represent a wide variety of patient types, ranging
largely match those for all patients with diabetes (3). Depending
in life expectancy, functional status, comorbidities, and their
on the individualized glycemic goals, measuring A1c every 6
care environment. This heterogeneity must be factored into
months is considered adequate for most, with a minimum recom-
decision making when setting glycemic and other therapeutic
mended frequency of once yearly. Self monitoring of glucose
goals. Factors such as vulnerability to hypoglycemia, ability to
levels should be at the discretion of the physician to meet particu-
self manage, cognitive status, comorbidities, and life expec-
lar patient needs, and that can support disease awareness, inform
tancy must all be considered (1, 2).
insulin dosing decisions, and avoid hypoglycemia.
A tighter glycemic target may be appropriate for older pa-
Kidney function is best monitored and followed using the
tients who have tolerated therapies without hypoglycemia and estimated GFR given that creatinine can be variable in this
for whom protection from microvascular disease is a priority. population. As with other patients, microalbuminuria and foot
Higher glycemic targets are appropriate for those with a high screening should be completed at least annually. Eye screening
cardiovascular risk, a history of hypoglycemia, a longer dura- is recommended every 2 years except in those with A1c above
tion of diabetes and multiple comorbidities. 8%, type 1 diabetes mellitus, or uncontrolled hypertension.
Although older patients may tolerate hyperglycemia to a
greater extent than younger patients owing to their lower glo- Interventions for Diabetes Care
merular filtration rate (GFR), hyperglycemia leading to symp- Diabetes Education
toms or risk of acute hyperglycemic complications should be Diabetes education may need to engage a carer or care team.
avoided in all patients. The education itself may need to be especially sensitive to
Hypoglycemia is often poorly tolerated by older patients and the cognitive capacity of the learner and cognitive load during the
can lead to traumatic falls and other consequences that can be activity. Some strategies (Table 1) that can improve effectiveness
especially dangerous for isolated patients and those who live of the education include limiting information, using short sen-
alone. Hypoglycemia may precipitate acute cardiovascular events tences, using frequent repetition, combining verbal and written
and lead to hospitalization, which can increase the risk of
delirium.
Prespecified subgroup analyses of the the Action to Control TABLE 1. Supplemental Resources for Older Patients
Cardiovascular Risk in Diabetes (ACCORD) Trial suggested with Diabetes
that hypoglycemia and other adverse effects of treatment were The following products may be helpful for older patients who
more common in older participants in the ACCORD trial, but have dexterity, visual or hearing problems:
the disproportionate cardiovascular mortality risk in the inten- Large print, audio and visual educational materials;
sive glycemic control group was in participants younger than Blood glucose meter with a backlight or audio;
age 65 years as opposed to older participants (10). There were Blood glucose meter with a drum of preloaded test strips,
no detectable differences in the primary outcome data in the eliminating the need to insert a test strip each time;
ADVANCE trial by age group (11). In the Veterans Affairs Lancing devices with a drum containing multiple lancets;
Diabetes Trial, those with diabetes duration less than 15 years Insulin or GLP1 pens that combine the medication and a
had a mortality benefit in the intensive arm, whereas those with syringe in a single device; and
duration of 20 years or more had higher mortality in the Syringe magnifier, a clear device that slips over or clips to a
syringe to magnify its markings.
intensive arm (12). Finally, a retrospective cohort study of

The Endocrine Society. Downloaded from press.endocrine.org by [${individualUser.displayName}] on 12 January 2017. at 12:19 For personal use only. No other uses without permission. . All rights reserved.
140 ENDO 2016 MEET-THE-PROFESSOR CLINICAL CASE MANAGEMENT

instructions, providing plenty of relevant examples, and soliciting clinical trial data in patients over the age of 75 years: a statin
feedback and understanding. study in older adults (participants age 70 82 y) found a 15%
reduction in coronary artery disease events with pravastatin (7).
Lifestyle Modification
Changing nutrition for older patients can be challenging due to Aspirin
the presence of established food preferences, cooking capabil- The benefit of aspirin is greatest among those patients with the
ity, financial constraints, concerns about fluid intake, reduced highest cardiovascular risks. However, the risks of bleeding
activity, and changes in dentition and swallowing. tend to also increase with comorbidities. Aspirin not recom-
mended for primary prevention in older persons. Aspirin at 75
Choice of Antidiabetic Drug mg per day is appropriate for patients with known cardiovas-
Metformin remains first-line therapy for older patients with type 2
cular disease who are not taking other anticoagulant therapy.
diabetes unless there is evidence of renal impairment or other
When aspirin is used in older persons, use of a proton-pump
contraindications; its dose should be increased gradually over the
inhibitor should be considered.
initial weeks to minimize gastrointestinal intolerance. A sulfonyl-
urea with a low risk of hypoglycemia such as glipizide or a
Dipeptidyl peptidase-4 (DPP4) inhibitor can be used if metformin Diabetes Comorbidities in the Older Patient
is not tolerated, is contraindicated, or is inadequate (2). The rapid- Depression
onset insulin secretagogues (the glinides) have been demonstrated Depression is more common among patients with diabetes
to be safe and effective for glucose management in older patients, and depression screening is important. Screen using the geriat-
and associated with less hypoglycemia but similar effects on A1c ric depression scale or patient health questionnaire (PHQ9).
as sulfonylureas. Sodium/glucose cotransporter 2 (SGLT2) inhibi- Screen at diagnosis, or with any cognitive decline or change in
tors seem to be safe and effective, and are associated with clinical status. Pharmacologic and psychological care of older
improvements in mortality and cardiovascular events in patients with depression improves QOL and functional status
older patients with a history of cardiovascular disease (6). and reduces depressive symptoms. Collaborative care is key.
GLP1 agonists are suitable for older patients if weight loss
would not be problematic. When insulin is indicated, a
Impaired Cognition
long-acting basal insulin combined with ongoing metformin
Alzheimer-type dementia and multi-infarct dementia are both
therapy is appropriate. If prandial insulin is needed it can be
twice as common in patients with diabetes as those without.
added first to the largest meal of the day, with addition
Hyperglycemia is associated with cognitive decline (8). Cogni-
before other meals if needed. Use of a prefilled insulin pen
can reduce dosing errors and improve compliance. tive decline seems to be exacerbated by and increase the risk of
Choosing agents with a low risk for hypoglycemia is espe- hypoglycemia. Thus, it can be useful to assess cognition at
cially important in patients who are frail or who have dementia. diagnosis of diabetes in the older patient, and again with any
Patients with dementia may be unable to recognize or respond change in functional or clinical status using a tool such as the
to hypoglycemia whereas frail patients may have diminished Montreal Cognitive Assessment.
glycogen reserve and be susceptible to developing severe hy-
poglycemia quickly. Falls
Falls are increased in the older patient and related to the
Hypertension Management presence of peripheral neuropathy, Charcot foot, muscle weak-
There is consistent evidence that lowering blood pressure (BP) ness, orthostatic hypotension, decreased vision, nocturia and
from very high levels (eg, systolic BP, 170 mm Hg) to moder-
incontinence, and polypharmacy. It is important to clarify the
ate targets (eg, systolic BP, 150 mm Hg) reduces cardiovascu-
causes of any falls. A basic falls evaluation includes an assess-
lar risk in older adults with diabetes. Target BP is less than
ment of injuries and evaluation for reversible causes including
140/90 mm Hg, but no lower than 120 mm Hg systolic. Several
medications and environmental factors. The risk factors for
antihypertensive groups including ACE inhibitors and diuretics
have demonstrated efficacy in older patients. Measure K and falls include balance disorders, functional impairment (includ-
kidney function within 2 weeks of initiating angiotensin ing muscular weakness), visual deficits, cognitive impairment,
converting-enzyme inhibitor (ACEI) or angiotensin receptor and certain medicines (including anticonvulsants, antipsychot-
blocker (ARB), with dose change, and at least yearly. ics, benzodiazepines, hypnotics, tricyclic antidepressants, and
selective serotonin uptake inhibitors). Falls can be prevented
Lipid Management with medication management, exercise, assessment in the liv-
Statin use is appropriate in older patients with diabetes, par- ing environment, orthostatic BP measurements, vision assess-
ticularly those who have had diabetes for more than 10 years, ment, gait and balance evaluation, followed by appropriate
but treatment to target levels is not necessary. There is limited preventive advice (9).

The Endocrine Society. Downloaded from press.endocrine.org by [${individualUser.displayName}] on 12 January 2017. at 12:19 For personal use only. No other uses without permission. . All rights reserved.
ENDO 2016 DIABETES AND GLUCOSE METABOLISM 141

Polypharmacy standard approach to glycemic control can be modified to


Patients or their carers should maintain an updated list of minimize the risk of hypoglycemia.
medications and share it with the care provider team. Medica- Maria requires nuanced care to ensure that treatment choices,
tion reconciliation at discharge is key. Interactions are espe- targets, medications, and lifestyle interventions are individualized
cially common in patients with diabetes given that the rate of and take into account the variety of comorbidities that coexist with
polypharmacy is so high among these patients. It can be useful diabetes in her case. Patients like Maria with cognitive decline
to review the American Geriatric Society (AGS) Beers criteria should be managed collaboratively, with particular attention to
to recognize inappropriate medications for older patients. Com- minimizing the risk of hypoglycemia and falls.
bination pills (such as a combination between metformin and With these approaches, the wellbeing, longevity, and QOL
glipizide) can improve compliance and minimize pill burden. of these older patients with diabetes can be optimized.

Urinary Incontinence REFERENCES


Urinary incontinence is often underreported and has a high 1. Sinclair A, Morley JE, Rodriguez-Manas L, et al. Diabetes mellitus in
older people: Position statement on behalf of the International Association
prevalence in older patients with diabetes. It can lead to falls, of Gerontology and Geriatrics (IAGG), the European Diabetes Working
fractures, depression, and social isolation. Its presence should Party for Older People (EDWPOP), and the International Task Force of
Experts in Diabetes. J Am Med Dir Assoc. 2012;13:497-502.
be sought with annual screening.
2. Sinclair A, Dunning T, Colagiuri S. International Diabetes Federation;
Managing older people with type 2 diabetes. Global Guideline. 2013.
Persistent Pain Accessed from: https://www.idf.org/sites/default/files/IDF-Guideline-for-
older-people-T2D.pdf.
Neuropathic pain can affect as many as 50% of older patients 3. Kirkman MS, Briscoe VJ, Clark N, et al. Diabetes in older adults. Diabetes
with diabetes and substantially affect QOL and function. Pain Care. 2012;35(12):2650-2664.
treatment with pharmacologic and nonpharmacologic ap- 4. Huang ES, Laiteerapong N, Liu JY, et al. Rates of complications and
mortality in older patients with diabetes mellitus: The Diabetes and Aging
proaches should be individualized. Study. JAMA Intern Med. 2014;174(2):251-258.
5. Lipscombe LL, Levesque L, Gruneir A, et al. Antipsychotic drugs and
hyperglycemia in older patients with diabetes. Arch Intern Med.
Osteoporosis 2009;169(14):1282-1289.
Osteoporosis can increase the risk of fall-related fractures. 6. Zinman B, Wanner C, Lachin JM, et al. Empagliflozin, cardiovasc-
Diabetes seems to increase the risk for osteoporosis. Screening ular outcomes, and mortality in type 2 diabetes. N Engl J Med.
2015;373(22):2117-2128.
and treatment is appropriate. 7. Shepherd J, Blauw GJ, Murphy MB, et al. Pravastatin in elderly individu-
als at risk of vascular disease (PROSPER): A randomised controlled trial.
Vision and Hearing Problems Lancet. 2002;360:1623-1630.
8. Whitmer RA, Karter AJ, Yaffe K, Quesenberry CP, Selby, JV. Hypogly-
Nearly one in five older adults with diabetes report visual cemic episodes and risk of dementia in older patients with type 2 diabetes
impairment, and hearing impairment is twice as common in mellitus. JAMA. 2009;301(15):1565-1572.
patients with diabetes; both problems can be associated with 9. Chau D, Edelman SV. Clinical management of diabetes in the elderly. Clin
Diabetes. 2001;19(4):172-175.
vascular disease and neuropathy. 10. Gerstein HC, Miller ME, Byington RP, et al. Action to control cardiovas-
cular risk in diabetes study group: effects of intensive glucose lowering in
type 2 diabetes. N Engl J Med. 2008;358:2545-2559.
MAIN CONCLUSIONS 11. Patel A, MacMahon S, Chalmers J, et al. Intensive blood glucose control
Managing diabetes becomes more challenging as patients grow and vascular outcomes in patients with type 2 diabetes. N Engl J Med.
2008;358:2560-2572.
older. Complicating comorbidities are more common, and the 12. Duckworth W, Abraira C, Moritz T, et al. Glucose control and vascular
patients do not tolerate adverse effects or polypharmacy well. complications in veterans with type 2 diabetes. N Engl J Med.
Behavioral modification can be especially difficult in older 2009;360:129-139.
13. Huang E, Liu JY, Moffet HH, John PM, Karter AJ. Glycemic control,
patients whose activity and dietary repertoire may be very complications, and death in older diabetic patients: The Diabetes and
limited. The treatment target must be individualized, and the Aging Study. Diabetes Care. 2011;34: 1329-1336.

The Endocrine Society. Downloaded from press.endocrine.org by [${individualUser.displayName}] on 12 January 2017. at 12:19 For personal use only. No other uses without permission. . All rights reserved.
The Endocrine Society. Downloaded from press.endocrine.org by [${individualUser.displayName}] on 12 January 2017. at 12:19 For personal use only. No other uses without permission. . All rights reserved.
GENERAL
ENDOCRINOLOGY

The Endocrine Society. Downloaded from press.endocrine.org by [${individualUser.displayName}] on 12 January 2017. at 12:19 For personal use only. No other uses without permission. . All rights reserved.
144 ENDO 2016 MEET-THE-PROFESSOR CLINICAL CASE MANAGEMENT

Endocrine Tumor Genetics: Challenging Issues

CMF1 dent on an understanding of normal organ function and organ


Presented, April 1 4, 2016 pathology, but also on the hereditary aspects of diseases, which
were only widely recognized in the 20th century. With the
advances of molecular genetics, the definition of new syn-
Tobias Else, MD; Marta Korbonits, MD, PhD. Department dromes is an ongoing active field of research as exemplified by
of Internal Medicine, Division of Metabolism, the fairly recent description of new syndromes, such as familial
Endocrinology & Diabetes University of Michigan Health isolated pituitary adenomas (FIPA) or DICER1 syndrome.
System, Ann Arbor, Michigan 48109, E-mail: telse@umich.edu;
and Department of Endocrinology, Barts and The London
School of Medicine, London EC1A 6BQ, United Kingdom, SIGNIFICANCE OF THE CLINICAL PROBLEM
E-mail: m.korbonits@qmul.ac.uk All hereditary endocrine tumor syndromes are rare syndromes.
However, in general it is estimated that 10% of all cancers arise
in patients with a predisposition syndrome due to a single gene
INTRODUCTION
alteration. Therefore, although hereditary disorders are rare
Historical Overview
Endocrine tumor syndromes are fairly recent discoveries. Most when considering each single syndrome, a hereditary predispo-
of them were only described over the course of the 20th sition of any kind is present in a significant minority of pa-
century. However, the history of their core components dates tients. The diagnosis of a hereditary syndrome has great effect
back significantly longer with descriptions of paragangliomas on the patients health, diagnostic and therapeutic approach,
dating back to the 17th century, and pheochromocytomas de- and is often beneficial to the patients family, preventing mor-
scribed as a unique tumor entity in the late 19th century (1). In bidity and mortality by regular surveillance of patients carrying
the early 20th century, the first surgeries for pheochromocy- a pathogenic mutation.
toma and paraganglioma were conducted, initially bearing a
significant amount of morbidity and mortality given that alpha
BARRIERS TO OPTIMAL PRACTICE
blockade was only widely available in the 1960s to 1970s. The
The main barriers to care for patients with hereditary endocrine
first description of a pheochromocytoma-associated syndrome
tumor syndromes lie in the fact that this specialized care often
was in 1882 by von Recklinghausen, who gave the first de-
requires a detailed understanding and availability of several
scription of neurofibomatosis type 1 (1). Most interestingly,
one of his first patients in retrospect likely also suffered from a medical subspecialties such as clinical genetics, genetic coun-
pheochromocytoma with symptoms of headaches and profuse seling, oncology, surgery, and epidemiology in addition to
sweating (1). The association of pheochromocytoma and von endocrinology. The general practitioner or endocrinologist is
Hippel-Lindau disease (VHL) was first recognized in the often not familiar with the interpretation of genetic test results,
1920s, multiple endocrine neoplasia (MEN) type 2 (MEN2) and even patients with clear index presentations may be dis-
was initially described in the 1960s, and only recently the missed given that knowledge about rare syndromes is often
hereditary paraganglioma syndrome, caused by SDHx muta- lacking. Furthermore, there are a limited number of guidelines
tions, was finally entirely recognized as its own syndrome in providing physicians with support in decision making for diag-
the early 2000s. nosis, therapy, and surveillance of patients with hereditary
Another example of a long history of endocrine genetics is endocrine syndromes. Very few dedicated centers with spe-
the illustration of a familial pituitary adenoma family suffering cialized endocrine tumor genetics clinics have a sufficient
from gigantism seen in the National Portrait Gallery in London volume of patients to gather experience in care for these rare
from the 18th century. The underlying mutation was identified syndromes.
in DNA from the skeleton of one of the family members
suffering from gigantism, which is exhibited in the Hunterian
Museum of the Royal College of Surgeons in London, and that LEARNING OBJECTIVES
exact mutation is present in 18 currently living families with As a result of participating in this session, learners should be
the same disease and the exact same mutation (2). able to:
The field of endocrine tumor syndromes is fast moving, with Identify patients who might benefit from genetic
regular discovery of new associations, such as renal cell cancer evaluation.
and pituitary adenomas with SDHx mutations. The history for Understand the process of genetic testing for the proband
most other hereditary endocrine tumor syndromes follows and family.
along the lines described for paragangliomas or pituitary tu- Understand the basic approach to interpretation of
mors. The definition of these syndromes was not only depen- genetic test results.

The Endocrine Society. Downloaded from press.endocrine.org by [${individualUser.displayName}] on 12 January 2017. at 12:19 For personal use only. No other uses without permission. . All rights reserved.
ENDO 2016 GENERAL ENDOCRINOLOGY 145

Understand the underlying principles of surveillance for TABLE 1. Familial Cancer Syndromes With Endocrine
carriers of gene mutations predisposing to endocrine Manifestations and Guidelines/Publications With Expert
tumor development. Opinion
Disease Guideline/Expert Opinion
STRATEGIES FOR DIAGNOSIS, THERAPY, General Gene reviews (http://www.ncbi.nlm.
AND/OR MANAGEMENT nih.gov/books/NBK1116/)
Approach to the Patient with Endocrine Tumors Concise Handbook of Familial Cancer
Focusing on Case Detection for Hereditary Syndromes Susceptibility Syndromes (5)
Evaluation of a patient with an endocrine tumor for a heredi- NCCN guidelines
tary predisposition should include a detailed personal history, (http://www.nccn.org/professionals/
physician_gls/f_guidelines.asp)
family history, physical examination, and review of systems,
MEN1 Brandi et al (6), Thakker et al (7)
and often requires review of laboratory parameters and surgical
MEN2 Brandi et al (6), Wells et al (8)
specimens. All information is integrated to make a working
Von Hippel-Lindau VHL Alliance and handbook (http://
diagnosis of a potential hereditary syndrome and to decide on
disease www.vhl.org/)
the recommendation of genetic testing for the patient or other
Neurofibromatosis type 1 Ferner et al (9), Radtke et al (10)
family members.
Hereditary Paraganglioma Benn et al (11), Fishbein et al (12),
In Endocrine Tumor Genetics the focus is not only on Syndrome Lenders et al (13)
malignant tumors, but often also on benign lesions and hor- Cowdens Disease (PTEN See NCCN, Bubein et al (14), Ngeow
mone excess syndromes. In general, hereditary syndromes with Hamarto-matous Tumor and Eng (15), Nieuwenhuis et al (16)
involvement of endocrine organs can be subdivided into the Syndrome, PHTS)
classical endocrine tumor syndromes (eg, MEN type 1 [MEN1] DICER1 syndrome See GeneReviews
and MEN2), syndromes with major endocrine tumor manifes- Lynch Syndrome American Gastroenterological
tations plus involvement of other organs (eg, VHL), and syn- Association (17), Hegde et al (18),
dromes with primarily nonendocrine manifestations, which Rubenstein et al (19), Stoffel et al
(20), Syngal et al (21)
may present with major endocrine tumors (eg, Cowdens dis-
Carney Complex Correa et al (22)
ease [PTEN-hamartomatous tumor syndrome; PHTS], Li-
Familial Adenomatous Vasen et al (23)
Fraumeni syndrome, or Lynch syndrome) (Table 1, Figure 1). Polyposis
This underscores the importance of a thorough general evalua- CDC73-related disorder See GeneReviews
tion of the patient not only limited to endocrine or oncological (HPT and Jaw Tumor
manifestations. This evaluation truly should be multidisci- syndrome)
plinary, including at least endocrinologists, physicians fa- Beckwith-Wiedemann See GeneReviews
miliar with hereditary diseases, genetic counselors, oncolo- syndrome
gists, pathologists, and potentially other subdisciplines. McCune-Albright See GeneReviews
Several lines of evidence can be used to identify index syndrome
patients with hereditary syndromes. A thorough personal his- Li-Fraumeni syndrome See NCCN (24)
tory can identify other diseases and syndromic manifestations FIPA Korbonits et al (25)
suggestive of a genetic predisposition. For example, thyroid
Abbreviation: HPT, hyperparathyroidism.
cancer in a young female patient with a history of breast cancer
and macrocephaly (95th percentile) raises the suspicion for
PHTS. Physical examination can identify other syndrome
manifestations, such as neurofibromas in a patient with pheo-
chromocytoma suggesting neurofibromatosis type 1, or col- recommend genetic testing. A child with an adrenocortical
lagenomas and angiofibromas in a young patient with primary cancer should always receive the recommendation for germline
hyperparathyroidism (pHPT) suggesting MEN1. A key compo- TP53 testing regardless of family history. Up to 25% of pa-
nent in the evaluation of a patient is the family history. A tients with TP53 mutations harbor de novo mutations and will
four-generation pedigree should be collected, ideally by a dedi- not have any family history, and young patients with medullary
cated professional health care provider (eg, genetic counselor). thyroid cancer will have a very high percentage of de novo
The family history should be reviewed for the presence of other RET mutation and MEN2. The family history can also be
tumors of the syndrome-associated spectrum, mode of potential negative because of adoption, imprinting, nonpaternity, or a
inheritance, ages of onset, and ideally histological diagnosis of low penetrance of the phenotype. Penetrance of genetic dis-
tumors. As an example, the co-occurrence of neuroendocrine eases can vary significantly and manifestation of different ele-
tumors, parathyroid tumors, and pituitary tumors would raise ments of a syndrome is also variable. For example, whereas
the suspicion for MEN1. It is important to note that a positive MEN1-associated pHPT is present in greater than 95% of
family history is not a necessary prerequisite for the decision to carriers by 50 years of age, other manifestations such as pitu-

The Endocrine Society. Downloaded from press.endocrine.org by [${individualUser.displayName}] on 12 January 2017. at 12:19 For personal use only. No other uses without permission. . All rights reserved.
146 ENDO 2016 MEET-THE-PROFESSOR CLINICAL CASE MANAGEMENT

Tumor syndromes with endocrine manifestaon

Mosaic
genec Germline
mutaon abnormality

McCune- Mulple MEN2a Neurof Familial non-


Albright PGL, SSoma, Endocrine Carney SDHx DICER Cowden Li- LLynch
syndr. Polycythemia MEN2b VHL ibroma HLRCC medullary PBMAH
Neoplasia complex syndr. syndr. syndr. F
Fraumeni ssyndr.
1&4 FMTC tosis thyr. cancer

PRKAR1A ARMCS
MLH1,2,6 PTEN
GNAS HIF2A MEN1 PRKACA SDHA,B,C, MEN1
RET 1 VHL
DICER1 NF1 PTEN TP53
3 PMS2 FH APC
PDE11 D,AF2 FH
CDKN1B,1A,2B,2C EPCAM CHEK2
2q16 PDE11A
HAPB2
MC2R
DICER1
PRKACA

Figure 1. Tumor syndromes with endocrine manifestations.

itary tumors occur in only a fraction of all MEN1 patients and Considerations Regarding Genetic Testing
indeed may be totally absent, and pancreatic neuroendocrine Based on the clinical evaluation and integration of above-
tumors manifest clinically in only 60 70% of cases. In au- mentioned information a working diagnosis is formed. Occa-
topsy, however, pancreatic tumors are present in almost all sionally the diagnosis can be made on clinical grounds alone. A
MEN1 patients. In hereditary paraganglioma syndrome caused patient with two of the three main manifestations of MEN1 will
by an SDHx mutation, several proven or obligate carrier family receive the clinical diagnosis of MEN1. Genetic confirmation
members might be unaffected. This is particularly true for gene should always be considered as it enables testing of at-risk
mutations that follow a nonclassical autosomal-dominant inher- family members. In general the threshold for genetic testing
itance, such as SDHD mutations, which are only pathogenic lies at least in the range of 510% expected positive results, but
when inherited from the father due to imprinting of the SDHD is of course influenced by the clinical effect a diagnosis has on
locus. The ages of onset in a family are also important as, for the future health of a patient and family. Genetic testing should
example, in general a thyroid cancer or parathyroid adenoma only be recommended if there is evidence or logical support
that it will reduce morbidity or mortality for the patient or other
above age 60 years is less likely to be caused by genetic
family members. Often the benefit of testing will be rather in
predisposition. Taken together, the family history often pro-
cascade family screening of hitherto-unaffected individuals
vides significant clues in patient evaluation but can, in some
rather than in the patient itself. However, there are certainly
instances for various reasons, also be entirely negative. A
also treatment decisions that are influenced by genetic testing
thorough family history might also identify an individual
(eg, a patient with completely resected adrenal cancer, and a
within the family that might be a better candidate to recom-
TP53 mutation should be advised against adjuvant radiation
mend testing than the patient who is present in clinic.
because of the risk of secondary malignancies).
Laboratory evaluation and review of pathological specimens
It is important to underscore that patients with a clinical
can also provide important information. For example, parathy- diagnosis of a hereditary syndrome should still be treated in
roid cancer and 4-gland hyperplasia are more likely associated accordance with the working diagnosis regardless of testing
with CDC73-related disorder or MEN1, respectively, than the results. Although genetic testing is very good for some diseases
usual single adenoma. Follicular thyroid cancer is the classical (eg, nearly 100% of patients with von Hippel-Lindau disease will
manifestation of PHTS, although papillary variants can be have an identifiable VHL mutation after genetic testing) it is less
observed. In a patient with pheochromocytoma, a personal or often confirmatory for others (eg, only 75% of patients with
family history of a papillary type 2 renal cell cancer is sugges- Carney complex will have a mutation in PRKAR1A). Another
tive of hereditary leiomyomatosis and renal cell cancer rather reason for negative genetic testing is the existence of phenocopy,
than any of the other syndromes predisposing to pheochromocy- in which an individual in a family with known hereditary disease
toma or paraganglioma. Finally, the localization and hormonal may have a sporadic occurrence of the disease or tumor type. For
phenotype can influence the genetic differential diagnosis. example, even mutation negative members of a family with
MEN1-associated neuroendocrine tumors are almost invariably MEN1, who therefore do not carry the diagnosis of MEN1, still
localized in foregut derivatives and VHL-associated pancreatic might develop primary hyperthyroidism by chance, just to the
neuroendocrine tumors (pNETs) are exclusively nonfunctional. same degree as the rest of the population.

The Endocrine Society. Downloaded from press.endocrine.org by [${individualUser.displayName}] on 12 January 2017. at 12:19 For personal use only. No other uses without permission. . All rights reserved.
ENDO 2016 GENERAL ENDOCRINOLOGY 147

Genetic testing should be streamlined in accordance with the disease causing, can leave physician and patient in an unclear
current knowledge of syndromes and the patients clinical state, often causing anxiety and overtreatment.
findings and should be conducted in a targeted fashion includ- The most important step in genetic testing is the interpreta-
ing all genes that would be considered for the patients indi- tion of test results. According to the American College of
vidual clinical findings. We recommend that genetic testing Medical Genetics and Genomics (ACMG) guidelines, several
should only be conducted after careful counseling by a genetic lines of evidence can be used to classify a mutation in five
counselor and patient written informed consent, which indeed categories: benign, likely benign, uncertain significance, likely
in some countries is required to conduct any genetic testing, pathogenic or pathogenic (Table 2) (3). The laboratory and
particularly when evaluating unaffected family members for interpreting geneticist and clinical pathologist will follow this
gene carrier status to identify their risk. classification and provide the physician with a categorization.
Genetic testing includes a variety of methods, such as meta- However, the laboratory will often lack clinical information on
phase analysis to investigate for chromosomal aberrations as in patient phenotype and family history. Therefore, they will
Klinefelter or Turner syndrome. Chromosomal microarrays can mainly take into account molecular characteristics, published
detect medium-sized to large deletions and duplications. For literature reports, population databases, and disease-specific
single gene analysis the main methods are next-generation databases. Often the first step is to compare it to mutations,
sequencing (NGS), traditional Sanger sequencing of exons and which are known to cause a certain disease and if the change is
already classified, the laboratory will follow with its interpre-
exon-intron boundaries, and deletion/duplication analysis (usu-
tation. Some molecular changes, such as splice-site mutations,
ally performed by multiplex ligation-dependent probe amplifi-
larger deletions, or nonsense mutations in a disease-causing
cation [MLPA]).
gene will also more likely be classified as pathogenic. In
Sanger sequencing and NGS are nowadays almost equiva-
addition, in vitro analysis, if available, such as disrupted pro-
lent in their reliability to detect single-base-pair mutations.
However, they often fail to identify medium-to-large-sized
deletions or duplications. Deletion/duplication analysis by TABLE 2. ACMG Categories and Criteria for Variant
MLPA was only introduced into regular clinical care in the Calling (Overview)
2000s. Therefore, it is very important to review older genetic Category
test results when patients present to clinic. For example, muta- Benignnot disease causing
tions in the gene CDC73, coding for parafibromin, in hyper- Likely benign
parathyroidism jaw tumor syndrome, are often caused by larger VUScannot be classified as disease causing or benign
deletions and therefore patients evaluated by Sanger sequenc- Likely pathogenic
ing will not show any genetic changes. In these cases perform- Pathogenicdisease causing
ing dedicated deletion/duplication analysis can often complete Evidence Used for Categorization
genetic testing and confirm the diagnosis due to partial gene Type of mutation
deletions. The advantage of NGS panels over Sanger sequenc- Nonsense, frameshift, canonical 1 or 2 splice sites,
ing is that it allows for the analysis of multiple genes rather initiation codon, single or multi-exon deletion
than single genes. This decreases time to diagnosis and is often Prediction algorithms
cost effective because it does not require sequential gene test- eg, Polyphen, SIFT
ing from highest suspicion to lowest suspicion. However, most Disease-specific databases
available gene panels offered by academic institutions or com- Known disease causing mutations, hotspot (oncogenic)
mutations (e.g. RET)
mercial providers are fixed with regard to the genes they
Population databases
contain, leaving little space for flexibility. Although it is advan-
Absent in individuals that do not have the phenotype (eg,
tageous to cover, for example, all SDHx genes for a patient 1000 genomes, ExAC)
with a head and neck paraganglioma, panels would not need to Tracking with phenotype within pedigree
include genes that are not applicable (eg, RET for MEN2). Segregation of the variant with the phenotype (more
Panels also sometimes include genes that never require a diag- weight the larger the pedigree and the more distant the
nosis by genetic testing such as for neurofibromatosis type 1, relation). Especially useful when there is no segregation
which remains an entirely clinical diagnosis as the penetrance as then disease causing status is less likely although
phenocopies can interfere
of the cutaneous phenotype, and Lisch nodules is almost 100%.
In vitro data
However, this also underscores the importance of conducting a
eg, in vitro loss of protein function, altered splicing or
full physical examination and review of systems. Although it expression
seems preferable to get more analysis for the same cost, genes
that are tested without a clear indication increase the chance of Abbreviations: ACMG, American College of Medical Ge-
results that are difficult to interpret. Variants of uncertain netics and Genomics; VUS, variant of uncertain significance.
significance (VUSs), which cannot be classified as benign or

The Endocrine Society. Downloaded from press.endocrine.org by [${individualUser.displayName}] on 12 January 2017. at 12:19 For personal use only. No other uses without permission. . All rights reserved.
148 ENDO 2016 MEET-THE-PROFESSOR CLINICAL CASE MANAGEMENT

tein function or loss of heterozygosity in available tumor tissue, for retinal angiomas in VHL starts very early in life. The
can classify a mutation as more likely pathogenic. However, procedure is low risk and can save a patients eyesight. How-
although loss of heterozygosity is the rule for several heredi- ever, screening for pheochromocytoma and neuroendocrine tu-
tary syndromes caused by classical tumor suppressor genes, it mors in VHL does not start until preteen/teenage years, when
is, for example, the exception for other syndromes, such as incidence increases and patients can cooperate for imaging
PTEN mutations in PHTS. In case a mutation in a gene known procedures without general anesthesia. The same is true for
to be associated with a hereditary syndrome has never been hereditary paraganglioma syndrome, when it is probably advis-
described before, it will be reviewed in larger population data- able to start with blood pressure surveillance in early child-
bases. When the allele frequency of a genetic change clearly is hood, with addition of whole-body magnetic resonance imag-
greater than the disease in the population (for most of the ing (MRI) when general anesthesia is not needed.
hereditary conditions 1%) the change will be classified as The principles of surveillance for patients with hereditary
benign. Family history should be taken into account, and if a diseases are well comparable to established population screen-
mutation tracks with a rare clinical phenotype (segregates), it ing. The usual recommendation for colon cancer screening in
may be more likely regarded as pathogenic. A specific chal- the general population in the United States is to start screening
lenge is a rare mutation in a gene that is classified as a VUS in colonoscopy at age 50 years, which is a compromise of missing
a patient with highly suggestive features or even a clinical some cases of sporadic colon cancer, but also keeping the
diagnosis. Clinical judgment is needed in this situation to either procedure-associated morbidity and costs at a necessary mini-
regard the variant as a true VUS or to embark on treatment, mum on the population basis. For a rare disease, such as
surveillance, and family testing as if the mutation was truly MEN1, this means that one can start early with screening for
disease causing. In general, clinical evidence should be very insulinoma and pituitary tumor, which can be performed by
strong to consider action, meaning the patient should have a physical examination, review of systems, and measurement of
clinical diagnosis of the syndrome in question or a very strong insulin and prolactin levels. These tests are not too difficult to
family history (Table 2). organize and do not require general anesthesia. Screening for
other MEN1-associated pNETs should start later once children
Surveillance for Mutation-Positive Individuals can cooperate as necessary for imaging procedures. Of course,
Once a patient or a family member is found to carry a patho- there might be descriptions of MEN1-associated nonfunctional
genic variant, decisions must be made regarding the right pNET as early as 5-10 years, but all screening is a tradeoff to
clinical care to prevent associated morbidity and mortality. prevent costs and screening-associated risks. For some syn-
Although this is very individually distinct for the different dromes, tumor risk can be age dependent and screening can be
diseases, there are major principles that must be considered by implemented at the time of highest tumor risk and then stopped
the treating physician. Surveillance should always be an indi- later. For example, some manifestations of DICER1 syndrome
vidualized process, integrating patient wishes and beliefs, sug- almost exclusively occur in the first 4 years of life, such as
gestions by accepted guidelines, and expert opinions, as well as pleuropulmonary blastoma or pituitary blastoma. In Beckwith-
morbidity and mortality associated with the conditions to Wiedemann syndrome there is significant cancer risk early in
screen for and any potential risks associated with the surveil- life that does not seem to significantly increase in adulthood.
lance itself. Although some of the syndromes have well- Surveillance frequency and intervention depend on the
established guidelines (Table 1), these do not exist for all. speed at which tumors occur and grow. Pheochromocytoma
General points of contention and uncertainty include the extent, and paraganglioma associated with VHL or MEN2 are slow
frequency, and age of onset of screening. The range is based on growing and surveillance is not needed more than annually.
a general view to aim to diagnose all diseases in patients with Given that MEN2-associated pheochromocytomas almost in-
a hereditary syndrome vs a more reasoned approach consider- variably proceed through a stage of hyperplasia and always
ing cost effectiveness and rationale regarding risks associated produce metanephrine, simple laboratory surveillance is suffi-
with screening. cient. MEN1-associated pNETs only develop a potential to
In general, screening that takes place more often than once a metastasize once they grow larger than 1-2 cm and only then
year is usually a significant strain on the patient. Patient edu- should surgery be a consideration. Treatment with pancreatic
cation and counseling about the possibility of syndrome-related surgery is a high-risk procedure with high morbidity and even
findings as well as potential incidental findings (which are mortality. In addition, therapy should always be weighed
often more common on imaging procedures than targeted find- against preservation of organ function. Postpancreatectomy
ings) is important to gain acceptance and understanding by the (for neuroendocrine tumors) diabetes mellitus and post adrena-
patient/family. lectomy (for pheochromocytomas) adrenal insufficiency are
Another common question is the age of onset of screening, states that significantly affect quality of life and increase mor-
which in the past has often aimed to diagnose all associations bidity and mortality.
as early as possible. However, screening should be guided by Whenever guidelines are present, we recommend screening
the age-adjusted incidence of findings. For example, screening in accordance with the guidelines. If no guidelines are in place,

The Endocrine Society. Downloaded from press.endocrine.org by [${individualUser.displayName}] on 12 January 2017. at 12:19 For personal use only. No other uses without permission. . All rights reserved.
ENDO 2016 GENERAL ENDOCRINOLOGY 149

then best expert opinion and clinical judgment should be ap- Discussion
plied. And in any case, surveillance should be a team approach, This is a male patient with early onset pHPT due to
including a multidisciplinary team of physicians and other hyperplasia/multiple adenomas, the kidney stones suggesting
healthcare providers such as genetic counselors. First and fore- the onset before age 35. In general everybody with pHPT prior
most it should consider the patient as the integral part of the to age 35 (or even 45) years should be considered for genetic
team. Counseling and consenting are an important part of this evaluation. In addition, he has ongoing hyperparathyroidism
process. And finally, the decision for a screening plan is always from parathyroid tissue in the forearm. These facts are sugges-
individualized to a certain patient, condition, and available tive of a hereditary predisposition even in the absence of a
screening procedures. positive family history (Answer C is incorrect). At minimum
the patient should undergo genetic testing for MEN1 and
MAIN CONCLUSIONS CDC73 (Answer B is correct). Testing beyond MEN1 and
Making the diagnosis of a genetic endocrine tumor syndrome is CDC73 is not necessary as a first round of genetic testing
an integrative process, involving clinical evaluation and careful and simple NGS analysis would also miss deletions and dupli-
consideration of the right genetic test. Genetic test results often cations (Answer D is incorrect). In addition, he should be
need detailed interpretation, integrating clinical findings to con- worked up for familial hypocalciuric hypercalcemia although
struct a working diagnosis. Surveillance for gene carriers is an this diagnosis is less likely in the presence of kidney stones and
individualized process, taking into account patient preferences, the degree of hypercalcemia. He did not have a low calcium/
published evidence, physician experience, and availability of creatinine clearance. Genetic testing revealed an intronic muta-
diagnostic tools. tion in CDC73 that was classified as a VUS. However, review
of the mutation shows that it potentially activates a cryptic
splice site resulting in an early stop codon. Therefore, taken
DISCUSSION OF CASES AND ANSWERS
together with the very rare phenotype we decided to treat this
Case 1
as a pathogenic mutation (further in vitro analysis showed loss
A 52-year-old male patient with pHPT presents for evaluation
of the wild type allele in all parathyroid tissue). Identifying this
of possible genetic predisposition for pHPT. Multiple episodes
mutation allows for family diagnosis and screening and will
of kidney stones. At age 35 diagnosis of pHPT, calcium (Ca),
12.6 mg/dL; PTH, 242pg/mL; followed by surgery R lower and likely steer the therapy of this patient in a more aggressive
upper parathyroid gland (parathyroid adenoma). Postsurgical fashion (Answer A is incorrect). The patient is diagnosed with
labs: Ca, 9.5 mg/dL. Eight years later recurrent hypercalcemia a working diagnosis of CDC73-related disorder, hyperparathy-
(12.4 mg/dL), surgery L lower and upper parathyroid gland roidism, and jaw tumor syndrome. Screening did not show any
(parathyroid adenoma) and autotransplant L forearm; Ca, 9.7 jaw tumors. Other manifestations of this disorder are not well
mg/dL. After 3 years again recurrent hypercalcemia, surgery described. There is evidence for an increased risk of uterine
revision forearm, hyperplastic parathyroid tissue. Currently on- fibroids, kidney cysts, and possible kidney cancer. Although
going hypercalcemia, kidney stones, negative sestamibi scan. expert opinion recommends a baseline ultrasound of the kid-
neys, which was negative in this patient, there is certainly not a
high risk for kidney cancers (Answer E is incorrect).
Family History
Four-generation pedigree obtained. Mother status post hyster-
Case 2
ectomy, likely for fibroids, no other family members affected.
A 19-year-old male with para-adrenal paraganglioma and bone
metastasis, status post right adrenalectomy and nephrectomy.
Question 1 Presurgical elevation of normetanephrine levels and very mild
Which of the following statements are true? elevation of metanephrine level.
A. This patient does not need to undergo genetic testing
because he already had all parathyroid glands removed.
B. The patient should be recommended genetic testing for Family History
MEN1, possibly CDC73 by sequencing and deletion/ No paragangliomas or pheochromocytomas in four generation
duplication analysis. pedigree, maternal aunt with kidney cancer.
C. The patient does not need testing in the absence of any
family history. Question 2
D. It is likely that the patient has something hereditary Which of the following statements are correct?
and a broad panel or whole exome sequencing is A. The patient should only have targeted sequencing of
indicated. the SDHB gene because he has a malignant
E. In case the patient tests positive, he will have a high normetanephrine-producing tumor and SDHB is likely
risk for renal cell cancer. to harbor a mutation.

The Endocrine Society. Downloaded from press.endocrine.org by [${individualUser.displayName}] on 12 January 2017. at 12:19 For personal use only. No other uses without permission. . All rights reserved.
150 ENDO 2016 MEET-THE-PROFESSOR CLINICAL CASE MANAGEMENT

B. This patient should undergo NGS and nent jaw; he is 186 cm, which is 12 cm taller than his
deletion/duplication panel for at least SDHB, SDHD, midparental height. His IGF-I level is 2.5 times higher than
VHL, TMEM127, and MAX. upper limit of age-adjusted normal range.
C. The patients mother does not need to be tested
because she is already 55 years old and unlikely to
Family History
develop a tumor and screening will likely not be cost
A male paternal first cousin suffered a pituitary apoplexy at the
effective.
age of 13 years, he is 210 cm tall. A 35-year-old female
D. If the patient tests positive for an SDHB mutation he
paternal first cousin had recent-onset amenorrhea and infertility
needs yearly endoscopies and pituitary MRIs for GIST
and microprolactinoma was identified.
and pituitary adenoma screening.
E. The patient does not need testing because it does not
influence further therapy. Question 3
The most likely genetic diagnosis is:
A. MEN1
Discussion
B. Carney complex
This is a young patient with an unusual tumor and he should
C. FIPA-AIP
definitely undergo genetic testing. Taken the normetanephrine
production and the malignant phenotype, a mutation in SDHB D. X-linked acrogigantism
is the most likely outcome. The patient was tested by an NGS E. SDHx-related paranganglioma syndrome
panel (SDHB, SDHC, SDHD, SDHA, SDHAF2, VHL, TMEM127,
RET, NF1, and MAX) and deletion/duplication analysis by Discussion
MLPA (Answer B is correct). He was found to have a deletion The presentation with two relatives with early-onset gigantism/
of exon 1 of SDHB, which was only detected in MLPA, acromegaly due to a GH-secreting tumor is very suggestive of
underscoring the importance for deletion analysis (Answer a hereditary syndrome and well in accordance with the diagno-
A is incorrect). Full evaluation by a chromosomal microar- sis of FIPA (Answer C is correct). MEN1 and Carney complex
ray showed restriction of the deletion to only exon 1 of can present with acromegaly, but most likely one would expect
SDHB. As a next step, first-degree relatives should be tested other disease manifestations or findings on physical examina-
for the deletion, including his parents and siblings (Answer tion and family history with syndromic aspects (Answers A
C is incorrect). Of note, he has a younger brother with a and B are incorrect). Both are more likely to present later in
history of a neuroblastoma (described with deletions in life than childhood. X-LAG presents before the age of 5 years
SDHB) as well as a paraganglioma diagnosed shortly after in all the published cases and most often results from a de novo
genetic diagnosis of hereditary paraganglioma syndrome and mutation, whereas in familial cases follows an X-linked inher-
an aunt with kidney cancer, and both tested positive for the itance; therefore, other affected family members on the pater-
deletion. His mother was started on surveillance with plasma nal side of a male patient are unlikely (Answer D is incorrect).
metanephrines every year and MRI whole body every 2 Carriers of SDHx mutations have a slightly increased risk of
years, which is the protocol most centers currently recom- pituitary tumor development; however, these cases usually
mend in the absence of guidelines. She was not found to
present at a later age (mean age, 50 y) and the majority are
have any tumor, which is in accordance with the incomplete
prolactinomas (60%) and are associated with personal or fam-
penetrance of tumors with SDHB mutations resulting in the
ily history of paragangliomas or pheochromocytomas (Answer
fact that not everybody with a mutation develops a tumor.
E is incorrect).
However, she will be advised to undergo regular screening
to evaluate the onset of any tumors in the future (Answer D
is incorrect). It is true that the knowledge of an SDHB Question 4
mutation does not immediately influence therapy. However, What is the next step?
there is evidence that patients with SDHB mutations respond A. Testing in the male and female first cousin is not
better to I131MIBG therapy and possibly certain cytotoxic necessary because they do have a genetic disease of
chemotherapies and the possibility of family testing likely FIPA, because their cousin carries an AIP mutation.
will prevent morbidity in the family (Answer E is incorrect). B. Following testing of the proband, the other two
The patient is currently without therapy other than bisphosphonates affected subjects should be tested followed by cascade
with stable disease for 3 years. testing of their-first degree relatives.
C. No further genetic testing is needed.
Case 3 D. AIP mutations are usually due to a postzygotic
A 20-year-old male presents with increasing headaches, poor mutations causing mosaicism and therefore are not
sleep, and excessive sweating. He has a long face with promi- inherited.

The Endocrine Society. Downloaded from press.endocrine.org by [${individualUser.displayName}] on 12 January 2017. at 12:19 For personal use only. No other uses without permission. . All rights reserved.
ENDO 2016 GENERAL ENDOCRINOLOGY 151

Discussion TABLE 3. Glossary of Terms


The next step in evaluating this family is 2-fold and should Term Definition
definitely be recommended (Answer D is incorrect). One
Phenocopy Manifestation of a syndrome associated
should offer genetic testing in first-degree relatives and then phenotype in a mutation-negative
recommend genetic testing for all individuals in the family that person in a family with proven
are first-degree relatives of a mutation carrier (cascade testing) hereditary syndrome (sporadic disease
(Answer B is correct). It is also feasible to test affected family mimicking syndrome manifestation)
members. Interestingly, in this case the paternal female cousin Obligate carrier Untested individual in direct line with
two gene carriers who can be assumed
did not carry the AIP mutation (her phenotype was not typical
to be positive for the mutation without
of AIP mutationpositive cases anyway) and therefore does not testing
carry the diagnosis of FIPA (4). This finding is referred to as Penetrance Proportion of patients with a disease-
phenocopy (in this case due to a sporadic microprolactinoma) causing mutation who develop a
(Answer A is incorrect). There are diseases that are invariably specific phenotype
due to mosaicism (eg, McCune-Albright syndrome) as a full Deletion/duplication Deletion or duplication of stretches of
deletion is not compatible with early embryonic development DNA
and life. However, mosaicism or somatic mutation has never Point mutation Single base pair change in DNA
been described for AIP mutationpositive FIPA cases (Answer Insertion Insertion of one to several base pairs
into the DNA
D is incorrect). Although there are no formal guidelines yet for
De novo mutation Proven mutation in an affected patient
surveillance of AIP-mutation carriers, best clinical judgment with both parents being negative for
and discussion with the patient is warranted. At our institution that mutation. Mutation arose in this
we perform clinical screening of mutation carriers with base- patient
line pituitary hormones, oral glucose tolerance test for GH, and Gene imprinting Parental pattern of methylation
a baseline MRI. If these are negative, we follow them with affecting gene expression (either
yearly clinical examination and basal pituitary blood test until maternal or paternal allele silent)
Parent of origin Disease-causing mutation has to be
the age of 30 years. If at that point all investigations are
effect inherited from either mother or father
negative, the chance of developing disease later is low. in order to be pathogenic (eg, SDHD
mutations only cause disease when
Case 4 paternally inherited
A 15-year-old boy is referred to genetic clinic due to recent Phenotype Disease expression of a certain genetic
operation of a large multinodular goiter. He has no other health change (genotype)
concerns and had normal development. Cascade testing Follow-up family testing, ideally
testing all first-degree family
members of gene mutation carriers
Family History Mosaicism Variegated genotype in the same
His brother, mother, maternal grandmother all had a thyroid organism. Usually due to a
operation at the age of 14-15 years. postzygotic mutation that will cause
all daughter cells of that particular
initial cell in which the mutation
Question 5 arose to carry the mutation, while the
Which of the following answers are correct? rest of the organism does not carry
A. The patients head size needs to be measured this mutation.
B. Multinodular goiter (is a common disease and no Next-generation A method to identify the bp sequence
further testing is needed in this case) sequencing of the DNA using high-throughput
(NGS) method, usually involving small
C. Multiple thyroid operation in family should lead to full
fragments of DNA sequenced at 30-
testing for all of the following genes: RET, PTEN, 100-1000 times, ie, depth
APC, CHEK2, HAPB2, DICER1
D. The patient will likely be at risk for pleuropulmonary
blastoma and pituitary blastoma Without any other manifestation DICER1 syndrome is the most
E. If the patient had a sister she might be at risk for likely diagnosis. With regard to the presentation with goiter,
Sertoli-Leydig Cell Tumors one could also entertain several other syndromes affecting
thyroid hormone synthesis (eg, Pendred syndrome). However,
Discussion most of these would have some other manifestations (eg, hear-
Multinodular goiter is a common disease, but the early presen- ing loss) and mostly follow an autosomal-recessive pattern,
tation and the number of affected individuals in the family which is unlikely in this case of three generations affected with
suggest a hereditary predisposition (Answer B is incorrect). goiter. In case the patients had other manifestations of

The Endocrine Society. Downloaded from press.endocrine.org by [${individualUser.displayName}] on 12 January 2017. at 12:19 For personal use only. No other uses without permission. . All rights reserved.
152 ENDO 2016 MEET-THE-PROFESSOR CLINICAL CASE MANAGEMENT

Cowdens disease, (eg, macrocephaly or macular speckling), 7. Thakker RV, Newey PJ, Walls GV, et al. Clinical practice guidelines for
multiple endocrine neoplasia type 1 (MEN1). J Clin Endocrinol Metab.
testing for a PTEN mutation should be prioritized (Answer A is
2012;97(9):2990-3011.
correct). Although one could entertain a panel of genes, RET 8. Wells SA Jr, Asa SL, Dralle H, et al. Revised American Thyroid Associa-
and APC do predispose to medullary and papillary thyroid tion guidelines for the management of medullary thyroid carcinoma.
cancer, respectively, but usually do not cause multinodular Thyroid. 2015;25(6):567-610.
9. Ferner RE, Huson SM, Thomas N, et al. Guidelines for the diagnosis and
goiter. CHEK2 increases the risk for several cancers, but it is management of individuals with neurofibromatosis 1. J Med Genet. 2007;
still not entirely clear whether mutation carriers truly need to 44(2):81-88.
undergo enhanced screening for cancers other than breast can- 10. Radtke HB, Sebold CD, Allison C, Haidle JL, Schneider G. Neurofibro-
matosis type 1 in genetic counseling practice: Recommendations of the
cer. A HAPB2 mutation has recently been shown in one family National Society of Genetic Counselors. J Genet Couns. 2007;16(4):
with papillary thyroid cancer, but not goiter. It is in general 387-407.
debatable whether very recently described genes should be 11. Benn DE, Gimenez-Roqueplo AP, Reilly JR, et al. Clinical presentation
and penetrance of pheochromocytoma/paraganglioma syndromes. J Clin
included in diagnostic genetic testing or whether more-detailed Endocrinol Metab. 2006;91(3):827-836.
knowledge needs to be ascertained regarding increased risk 12. Fishbein L, Nathanson KL. Pheochromocytoma and paraganglioma: Un-
increase and ability to screen for the disease in gene carriers. derstanding the complexities of the genetic background. Cancer Genet.
2012;205(1-2):1-11.
Sequencing of DICER1 revealed a pathogenic mutation 13. Lenders JW, Duh QY, Eisenhofer G, et al. Pheochromocytoma and para-
and cascade testing should be initiated. Because DICER1 ganglioma: An endocrine society clinical practice guideline. J Clin
syndromeassociated manifestations follow an age-dependent Endocrinol Metab. 2014;99(6):1915-1942.
14. Bubien V, Bonnet F, Brouste V, et al. High cumulative risks of cancer in
risk all described family members do not need any screening at
patients with PTEN hamartoma tumour syndrome. J Med Genet. 2013;
this point. Most manifestations, such as pituitary blastoma (0-2 50(4):255-263.
y) and pleuropulmonary blastoma (0-4 y) occur in early child- 15. Ngeow J, Eng C. PTEN hamartoma tumor syndrome: Clinical risk assess-
hood (Answer D is incorrect). Sertoli-Leydig cell tumors arise ment and management protocol. Methods. 2015;77-78:11-19.
16. Nieuwenhuis MH, Kets CM, Murphy-Ryan M, et al. Cancer risk and
in woman during adolescence or early adulthood (Answer E is genotype-phenotype correlations in PTEN hamartoma tumor syndrome.
correct). In case the patient had a sister in this age, one should Fam Cancer. 2014;13(1):57-63.
suggest screening by laboratory workup and ultrasound. An- 17. American Gastroenterological Association. Lynch syndrome: AGA Patient
Guideline Summary. Gastroenterology. 2015;149(3):814-815.
other very important clinical tool is patient education and 18. Hegde M, Ferber M2, Mao R, et al. ACMG technical standards and
simple discussion about the symptoms of associated diseases. guidelines for genetic testing for inherited colorectal cancer (Lynch syn-
In this case mentioning of menstrual irregularities, hirsutism, drome, familial adenomatous polyposis, and MYH-associated polyposis).
Genet Med. 2014;16(1):101-116.
and virilization would empower the patient and their family to 19. Rubenstein JH, Enns R, Heidelbaugh J, Barkun A. American
actively be involved in screening. Gastroenterological Association Institute Guideline on the Diagnosis and
Management of Lynch Syndrome. Gastroenterology. 2015;149(3):777-
782; quiz e16 e17.
REFERENCES 20. Stoffel EM, Mangu PB1, Gruber SB, et al. Hereditary colorectal cancer
1. Else T. 15 Years of paraganglioma: Pheochromocytoma, paraganglioma syndromes: American Society of Clinical Oncology Clinical Practice
and genetic syndromes: A historical perspective. Endocr Relat Cancer. Guideline endorsement of the familial risk-colorectal cancer: European
2015;22(4):T147T159. Society for Medical Oncology Clinical Practice Guidelines. J Clin Oncol.
2. Chahal HS, Stals K, Unterlander M, et al. AIP mutation in pituitary 2015;33(2):209-217.
adenomas in the 18th century and today. N Engl J Med. 2011;364(1): 21. Syngal S, Brand RE, Church JM, Giardiello FM, Hampel HL, Burt RW.
43-50. ACG clinical guideline: Genetic testing and management of hereditary
3. Richards S, Aziz N, Bale S, et al. Standards and guidelines for the gastrointestinal cancer syndromes. Am J Gastroenterol. 2015;110(2):223-
interpretation of sequence variants: A joint consensus recommendation of 262; quiz 263.
the American College of Medical Genetics and Genomics and the Asso- 22. Correa R, Salpea P, Stratakis CA. Carney complex: An update. Eur J
ciation for Molecular Pathology. Genet Med. 2015;17(5):405-424. Endocrinol. 2015;173(4):M85M97.
4. Williams F, Hunter S, Bradley L, et al. Clinical experience in the screening 23. Vasen HF, Moslein G, Alonso A, et al. Guidelines for the clinical man-
and management of a large kindred with familial isolated pituitary ad- agement of familial adenomatous polyposis (FAP). Gut. 2008;57(5):
enoma due to an aryl hydrocarbon receptor interacting protein (AIP) 704-713.
mutation. J Clin Endocrinol Metab. 2014;99(4):1122-1131. 24. Villani A, Tabori U, Schiffman J, et al. Biochemical and imaging surveil-
5. Lindor NM, et al. Concise handbook of familial cancer susceptibility lance in germline TP53 mutation carriers with Li-Fraumeni syndrome: A
syndromes-second edition. J Natl Cancer Inst Monogr. 2008;(38):1-93. prospective observational study. Lancet Oncol. 2011;12(6):559-567.
6. Brandi ML, Gagel RF, Angeli A, et al. Guidelines for diagnosis and 25. Korbonits M, Storr H, Kumar AV. Familial pituitary adenomasWho
therapy of MEN type 1 and type 2. J Clin Endocrinol Metab. 2001;86(12): should be tested for AIP mutations? Clin Endocrinol (Oxf). 2012;77(3):
5658-5671. 351-356.

The Endocrine Society. Downloaded from press.endocrine.org by [${individualUser.displayName}] on 12 January 2017. at 12:19 For personal use only. No other uses without permission. . All rights reserved.
ENDO 2016 GENERAL ENDOCRINOLOGY 153

Endocrine Consequences of Opiate Therapy

M04 BARRIERS TO OPTIMAL PRACTICE


Presented, April 1 4, 2016 There is limited clinical awareness of the neuroendocrine effect
of opiates required for education of patients initiating therapy,
which is often for the long term. The neuroendocrine effects
Ken Ho, MD, FRACP. Princess Alexandra Hospital and are rarely included in the medical curriculum and do not hold
The University of Queensland, Brisbane 4102, Australia, the same prominence that the behavioral and addictive aspects
E-mail: ken.ho@health.qld.gov.au do in public health and undergraduate educational programs.
Practitioners who initiate opioid therapy are not endocrinolo-
INTRODUCTION gists but those in other disciplines such as anesthesiologists,
Historical Overview clinical pharmacologists, and pain practitioners. Opioid-induced
Opioids have been used for millennia. Homer conveys its endocrinopathies are difficult to diagnose because the symptoms
effects in The Odyssey as a drug that had the power of robbing are nonspecific, often overlapping with those for which opioids
grief and anger of their sting and banishing all painful memo- were initially prescribed. The clinical significance of some of the
ries. Morphine was first isolated from opium in 1805 by a hormonal perturbations is not well understood and management
German pharmacist, Wilhelm Serturner. In 1874, Alder regimens are based on limited evidence.
Wright, an English pharmacist, produced heroin by boiling
morphine and acetic acid. In 1898, the German pharmaceutical LEARNING OBJECTIVES
giant, Bayer, marketed heroin commercially. It became the best After participating in this session, learners will be able to:
selling brand of all time (http://www.opiates.net). Understand the effects of opioids on neuroendocrine
The social consequences of opioids arising from endocrine function.
dysfunction were described by Charles Bruce in 1839, that Recognize the high prevalence of opioid-induced
opium has kept, and does now keep down the population: the hypogonadism in both men and women.
women have fewer children than those of other countries the Detail the mechanism(s) of suppression for opioids on
feeble opium-smokers of Assam are more effeminate than the pituitary adrenal axis.
women (1). The neuroendocrine inhibitory effects of opioids Develop a strategy to manage opioid-induced
on gonadal function were first reported over 30 years and on endocrinopathies.
adrenal function a few years later (2).
STRATEGIES FOR DIAGNOSIS AND
SIGNIFICANCE OF THE CLINICAL PROBLEM MANAGEMENT
Opioids are among the most widely abused substances globally Background
(3). Liberalization of laws on the prescription of opiates for Opioids act on the central nervous system through three types of
treatment of chronic pain has markedly escalated opiate use in receptors: , , and . The major endogenous ligands for these
the community. There has been a 4-fold increase in sales of receptors are endorphins, enkephalins, and dynorphins and their
receptor selectivities are shown in Table 1. Control of LH
opiates in the United States from 1999 to 2010 during which
more than 238 million prescriptions were written (4). In 2008,
approximately 14.8 million opioid prescriptions were dis-
pensed in England. The scale of opiate use is increasing glob- TABLE 1. Endogenous and Exogenous Opioids and
ally and public health programs minimizing their use and harm Receptor Selectivity. Modified from (4)
are health priorities in many countries. Receptor Type
The opioid system exerts diverse central and peripheral Opiods
effects on the endocrine system (5). Opiates also affect the Endogenous
respiratory, cardiovascular, and immune system. Among the Endorphins
wide-ranging effects are suppression of the reproductive and Enkephalins 0
adrenal axes (2). Opiates reduce gonadotropin secretion caus- Dynorphins 0
ing hypogonadism in men and menstrual dysfunction in Exogenous
Morphine 0
women. Opioid suppression of the pituitary adrenal axis can
Heroin 0
cause profound hypocortisolemia (6). Opioids also stimulate
Methadone 0
the GH system and prolactin secretion to a lesser extent (2, 5).

The Endocrine Society. Downloaded from press.endocrine.org by [${individualUser.displayName}] on 12 January 2017. at 12:19 For personal use only. No other uses without permission. . All rights reserved.
154 ENDO 2016 MEET-THE-PROFESSOR CLINICAL CASE MANAGEMENT

secretion is mediated mainly by receptors and of ACTH secre- acutely (7) but suppression after chronic use (7). The clinical
tion by and receptors (5, 7). significance of these perturbations is uncertain.

Gonadal System Diagnosis


The term opiate-induced androgen deficiency (OPIAD) was intro- Opioid endocrinopathy occurs with iv, transdermal, oral, or
duced to refer to hypogonadism in men using opiates (2, 8). intrathecal routes of administration (2, 7). The diagnosis is
Exogenous opiates inhibit GnRH secretion as well as T synthesis. dependent on awareness of the neuroendocrine effects or of
Intravenous administration of heroin reduces LH and T concentra- their use, which may be covert. The symptoms are nonspecific
tions within 4 6 hours. A similar rapid effect on gonadotropin and frequently are a mix of psychological and physical ail-
secretion also occurs in women (9). Several cross-sectional studies ments affecting mood, sexual function, and general wellbeing.
have documented high prevalence of hypogonadism among opi- The symptoms are difficult to distinguish from the psycho-
oid users (10). The degree of T suppression is dose related and logical consequences of chronic pain and diseases for which
reversible with opiate antagonists. A meta-analysis has reported a opiates were prescribed or from personality disorders, which
mean reduction in T of 165 ng/dL (5.7 nmol/L) among male lead to illicit use. The diagnosis is dependent on:
opioid users (10). It is unclear whether the hypogonadal potential Excluding other causes of impaired pituitary function
differs between different opiates. Recognizing the pattern of pituitary system dysfunction
Similar impairment of the pituitary gland occurs in women that typify opioid use
but is less-well studied (1). Up to 50% report some disturbance Excluding medications that suppress pituitary function
of menstrual function and up to 60% a reduction in libido (11). and its component systems for example, concurrent use
A cross-sectional study of women with chronic pain observed of synthetic glucocorticoids, androgens or estrogens,
that over 50% of women managed on opioids had amenorrhea, which are usually not detected in conventional
a prevalence more than 2-fold higher than a group managed immunoassays.
with nonopiate-based analgesics (11). In the same study, the
Measurement of baseline pituitary function is helpful. The
mean levels of LH and estradiol were much lower in the opiate
expected finds are shown in Table 2. Dynamic tests of the
users. Some studies have reported an increased prevalence of
pituitary-adrenal axis such as CRH or ACTH stimulation tests
osteoporosis in opioid users (12).
may be helpful in confirming defective responsiveness. The
value of GH stimulation test is not established.
Adrenal System
There is a strong body of evidence that opioids suppress the
hypothalamic-pituitary-adrenal system, although this is poorly rec- MANAGEMENT
ognized (6). Opioids acutely inhibit CRF release, ACTH secre- General
tion, and the cortisol response to ACTH stimulation (13). Serum Management is a challenging problem that requires a multidis-
cortisol decreases within hours of opioid administration (14), an ciplinary team. The general principles are as follows.
effect reversed by naloxone (13). Dehydroepiandrosterone A nonopiate mode of pain management should be pursued.
(DHEAS) levels are lower in opioid users in both men and women Wherever possible, patients should be placed in a methadone
(6). program, which offers a chance for dosage control, reduction,
It is not known whether reduced cortisol concentrations and withdrawal. It is not yet established whether different
represent an adaptive response or a pathological state of gluco- opioids differ in potency of endocrine suppression relative to
corticoid deficiency. There are reports of the occurrence of their analgesic effect. Buprenorphine was reported to induce a
hypotension and cardiovascular collapse, which have been in- lesser degree of gonadotropin suppression and lower incidence
terpreted as adrenal crisis because of attendant low cortisol
levels (7). It is not established that the hypocortisolemia is TABLE 2. Baseline Pituitary Function Tests and Findings
pathogenic because it has not been possible to dissect the in Opioid Induced Endocrinopathy
contributions to hypotension from the respiratory and cardio-
Hormone Finding
vascular effects of opioids (15). Morphine may cause cardio-
LH Low
genic shock (16). Some case reports have described profound
Testosterone Low
hypocortisolism without hypotension and spontaneous recov-
Estradiol Low or normal
ery with no adverse clinical consequence on opioid withdrawal
ACTH low
(17, 18). The suppressive effects of opioids on adrenal function
Cortisol Low
are well established but the clinical significance is unclear.
DHEAS Low
Prolactin Normal or mild elevation
Other Pituitary Effects
TSH No change
Opioids increase prolactin secretion (6, 7). Opioid effects on
IGF-I Normal or low
the GH axis seem to be time dependent, inducing stimulation

The Endocrine Society. Downloaded from press.endocrine.org by [${individualUser.displayName}] on 12 January 2017. at 12:19 For personal use only. No other uses without permission. . All rights reserved.
ENDO 2016 GENERAL ENDOCRINOLOGY 155

of sexual dysfunction than methadone. However, this finding Table 3.


requires confirmation in larger trials. There is a lack of evi- Hormones Results Normal
dence that opiate swapping helps, although, this regimen is Cortisol nmol/liter 210 200600
sometimes recommended (2). LH U/liter 0.6 28
General principles of managing hypogonadism should apply FSH U/liter 2.0 26
in opioid users. An open-labeled study has reported that T Testosterone nmol/liter 2.4 820
replacement improved sexual function in man with opioid- Prolactin mU/liter 480 500
induced hypogonadism (19). Osteoporosis is more prevalent in
this population (12, 20) such that sex steroid supplementation
should be considered for those with hypogonadism. Discussion
This case shows typical suppression of the pituitary gonadal
Hypoadrenalism axis by opiates and the reversibility after drug withdrawal.
Given that the significance of hypercortisolaemia is unknown,
the question as to whether glucocorticoid supplementation is to Case 2
be initiated depends on the clinical context. In an emergency, A 28-year-old woman with manic-depressive illness and mul-
the general cardiopulmonary principles of resuscitation includ- tiple hospital admissions for mood management presents with a
ing glucocorticoid therapy with a priori knowledge of cortisol 3-year history of chronic pain from a traumatic skiing accident.
status. Where hypocortisolism is detected incidentally, the car- She was managed initially with morphine, which led to nar-
diovascular status should be evaluated and the patient observed cotic abuse before being placed in a methadone program.
and monitored. A decision to initiate glucocorticoid supple- Suffering an exacerbation of depression, she was admitted to a
mentation or sick day management should be guided by clini- private clinic. Her medications included the oral contraceptive
cal judgment. pill, aprozolam (benzodiadapine), aripripazole (antipsychotic),
paroxitiene (SSRI), and methadone, 120 mg a day. Blood tests
were undertaken to exclude an endocrine cause for depression.
CONCLUSION
The results are shown in Table 4.
Opioid use affects hypothalamic-pituitary function, especially
She had a flat affect, slow speech manifesting general disin-
the gonadal and adrenal axes, which are frequently suppressed.
terest. She walked without assistance and cardiovascular as-
Physicians should evaluate symptoms and evidence of hypogo-
sessment revealed a regular pulse rate of 72 bpm and a blood
nadism, offering sex hormone supplementation as clinically pressure of 110/80 mm Hg without a significant change from a
appropriate. The prevalence of hypocortisolism is high but sitting to standing position. She reported no dizziness in getting
clinical significance not yet known. The need for glucocorti- out of bed. On account of the low cortisol, which was recon-
coid treatment is guided by clinical context and judgment. firmed, she was placed on hydrocortisone at a dose of 20 mg
Avoiding or minimizing the use of opioids remains a corner- per day (Table 2).
stone in managing opioid-induced endocrinopathy. Her mood improved gradually over the ensuing 3 months
and she was discharged. She did not keep her appointment with
CASES AND DISCUSSION the endocrine clinic. She made good progress on the metha-
Case 1 done program, coming off opiates 6 months later. She was then
A 49-year-old man sustained a midthoracic vertebral fracture evaluated at the endocrine clinic after a number of repeat
and a fracture of the right neck of femur following a traumatic appointments. She divulged that she had stopped hydrocorti-
accident while bicycling. In the emergency department, he was sone supplementation at the time of discharge. She had not lost
given morphine for pain relief before proceeding to hip sur- weight and denied lassitude or dizzy spells. A blood sample
gery, which was uneventful. Postoperatively, his convalescence taken in the late morning returned a normal cortisol level of
was slow because of pain and he was discharged on oxycontin 380 nmol/L.
at a dose of 20 mg daily. He was referred to the endocrinol-
ogy department to exclude osteoporosis and risk factors for
Table 4.
fractures. The results of endocrine evaluations are shown in
Hormones Results Normal
Table 3.
The low T concentration was noted by an experienced en- TSH mU/liter 0.6 0.42.0
fT4 pmol/liter 12 1020
docrinologist and recommended an evaluation after complete
Prolactin mU/liter 1050 500
convalescence when the patient was expected to be off analge-
Estradiol pmol/liter 50 100500
sics. At that assessment 4 months later, the patients T concen-
Cortisol nmol/liter 40 200600
tration had increased to 13.1 nmol/L with correspondence LH
24 h UFC nM 30 250
and FSH levels of 3.2 and 2.6 U/L, respectively.

The Endocrine Society. Downloaded from press.endocrine.org by [${individualUser.displayName}] on 12 January 2017. at 12:19 For personal use only. No other uses without permission. . All rights reserved.
156 ENDO 2016 MEET-THE-PROFESSOR CLINICAL CASE MANAGEMENT

Discussion 8. Daniell HW. Hypogonadism in men consuming sustained-action oral


opioids. J Pain. 2002;3:377-384.
This case highlights the not uncommon complex psychosocial
9. Hemmings R, Fox G, Tolis G. Effect of morphine on the hypothalamic-
and drug history of patients taking multiple medications that pituitary axis in postmenopausal women. Fertil Steril. 1982;37:389-391.
can affect hypothalamic pituitary function. In this instance, 10. Bawor M, Bami H, Dennis BB, et al. Testosterone suppression in opioid
users: A systematic review and meta-analysis. Drug Alcohol Depend.
aripripazole, an antipsychotic may have contributed to prolac- 2015;149:1-9.
tin elevation and methadone to low cortisol status. The signifi- 11. Daniell HW. Opioid endocrinopathy in women consuming prescribed
cance of hypocortisolemia was uncertain and questionable be- sustained-action opioids for control of nonmalignant pain. J Pain.
2008;9:28-36.
cause of normal cardiovascular and blood pressure status at the 12. Pedrazzoni M, Vescovi PP, Maninetti L, et al. Effects of chronic heroin
time. Nevertheless, glucocorticoid supplementation was pre- abuse on bone and mineral metabolism. Acta Endocrinol (Copenh).
1993;129:42-45.
scribed, an action unlikely to cause harm. Recovery of adrenal
13. Grossman A, Gaillard RC, McCartney P, Rees LH, Besser GM. Opiate
function occurred after withdrawal from methadone. modulation of the pituitary-adrenal axis: Effects of stress and circadian
rhythm. Clin Endocrinol (Oxf). 1982;17:279-286.
14. Facchinetti F, Comitini G, Petraglia F, Volpe A, Genazzani AR. Re-
REFERENCES duced estriol and dehydroepiandrosterone sulphate plasma levels in
1. Reddy RG, Aung T, Karavitaki N, Wass JA. Opioid induced hypogonad- methadone-addicted pregnant women. Eur J Obstet Gynecol Reprod
ism. BMJ. 2010;341:c4462. Biol. 1986;23:67-73.
2. Katz N, Mazer NA. The impact of opioids on the endocrine system. Clin J 15. Zelis R, Mansour EJ, Capone RJ, Mason DT. The cardiovascular effects of
Pain. 2009;25:170-175. morphine. The peripheral capacitance and resistance vessels in human
3. United Nations Office on Drugs and Crime. 2008. Opium / heroin market. subjects. J Clin Invest. 1974;54:1247-1258.
Accessed from: http://www.unodc.org/documents/wdr/WDR_2008/WDR 16. Feeney C, Ani C, Sharma N, Frohlich T. Morphine-induced cardiogenic
2008_Opium_heroin_market.pdf. shock. DICP. 2011;45:e30.
4. Manchikanti L, Helm S 2nd, Fellows B, et al. Opioid epidemic in the 17. Mussig K, Knaus-Dittmann D, Schmidt H, Morike K, Haring HU. Second-
ary adrenal failure and secondary amenorrhoea following hydromorphone
United States. Pain Physician. 2012;15:ES9-ES38.
treatment. Clin Endocrinol (Oxf). 2007;66:604-605.
5. Vuong C, Van Uum SH, ODell LE, Lutfy K, Friedman TC. The effects of
18. Policola C, Stokes V, Karavitaki N, Grossman. A Adrenal insufficiency in
opioids and opioid analogs on animal and human endocrine systems.
acute oral opiate therapy. Endocrinol Diabetes Metab Case Rep.
Endocr Rev. 2010;31:98-132. 2014;2014:130071.
6. Rhodin A, Stridsberg M, Gordh T. Opioid endocrinopathy: A clinical 19. Daniell HW, Lentz R, Mazer NA. Open-label pilot study of testosterone
problem in patients with chronic pain and long-term oral opioid treatment. patch therapy in men with opioid-induced androgen deficiency. J Pain.
Clin J Pain. 2010;26:374-380. 2006;7:200-210.
7. Abs R, Verhelst J, Maeyaert J, et al. Endocrine consequences of 20. Dursteler-MacFarland KM, Kowalewski R, Bloch N, Wiesbeck GA,
long-term intrathecal administration of opioids. J Clin Endocrinol Kraenzlin ME, Stohler R. Patients on injectable diacetylmorphine mainte-
Metab. 2000;85:2215-2222. nance have low bone mass. Drug Alcohol Rev. 2011;30:577-582.

The Endocrine Society. Downloaded from press.endocrine.org by [${individualUser.displayName}] on 12 January 2017. at 12:19 For personal use only. No other uses without permission. . All rights reserved.
ENDO 2016 GENERAL ENDOCRINOLOGY 157

Neurohumoral Syndromes

M16 SIGNIFICANCE OF THE CLINICAL PROBLEM


Presented, April 1 4, 2016 Neuroendocrine tumors (NETs) are a heterogeneous group of
tumors that arise from the diffuse endocrine system. They
derive from the embryological endocrine system predominantly
Aaron Vinik, MD, PhD, FCP, MACP, FACE. Department in the gut in the gastric mucosa, the small and large intestine,
of Internal Medicine, Eastern Virginia Medical School, and the rectum but are also to be found in the pancreas, lungs,
Norfolk, Virginia 23510, E-mail: vinikai@evms.edu and ovaries. NETs are tumors that. are slow growing and
capable of storing and secreting different peptides and
INTRODUCTION neuroamines (1). Some of these substances cause specific clini-
Historical Overview cal syndromes, others do not (2).
The origin of endocrinology is in the gastrointestinal (GI) tract. Some of these substances cause specific clinical syndromes
It began with Bayliss and Starling (29), who instilled acid in whereas others do not. For convenience they are separated into
the duodenum of dogs and showed the stimulation of the functional in which the consequence is a clinical syndrome
denervated pancreas secreted a bicarbonate-rich fluid. They derived from the hormone or amine being produced, or non-
reasoned that this must be a hormone, which means to excite, functional in which case the syndrome derives from the tumor
thus gave birth to hormonology or endocrinology. Shown be- bulk and the effect of metastases usually to liver, lymph nodes,
low is a graphic developed by Tom ODorisio (personal com- and to bone. Although considered rare, the annual incidence of
munication) (Figure 1), which shows the multiple Nobel Prize NETs has increased to 40-50 cases per million due to the
events that followed the initial discovery. Most notable of these availability of improved techniques for tumor detection. A
was the discovery of Insulin assay by Yalow and Berson (30), review of the SEER database showed a 5-fold increase in the
which was the first peptide therapy to be followed by soma- incidence of NETs from 1.09/100 000 in 1973 to 5.25/100 000
tostatin as the second peptide introduced into therapy. Its im- in 2004. In the United States, the prevalence is estimated to be
portance in the localization of GI endocrine tumors, 103 312 cases, which is twice the prevalence of gastric and
(Octreoscan) and subsequent use of its radiolabeled analog for pancreatic cancers combined (3).
ablation of tumors is illustrated. This evolution identified the Similar estimates have been reported from England and
secretion of peptides and amines into the lumen of the GI Sweden. These tumors occur at all ages with the highest inci-
tract referred to as lumenology and subsequently the la-
dence in fifth decade onward, except for appendiceal carcinoid,
beling of somatostatin with gallium a much more potent agent
which occurs at around 40 years of age and the genetic syn-
both for diagnostics (Gallium DOTATOC and DOTATATE
drome such as von Hipell-Lindau, neurofibromatosis, tube-
and peptide-receptor radiotherapy).
rosclerosis, and multiple endocrine neoplasia (MEN) -1 and
MEN-2 have their onset many years earlier. Life expectancy is
determined by the current grading system of tumors based
upon the Ki67 index of cell proliferation and the mitotic index.

BARRIERS TO OPTIMAL PRACTICE


There are impediments to the diagnosis of NETs. They are not
first in the differential because they comprise less than 2% of
the GI malignancies. Symptoms are often nonspecific and the
manifestations mimic a variety of disorders. A delay in diag-
nosis can also happen when the biopsy material is not exam-
ined for the secretory peptides. Tumors may then be labeled
erroneously as adenocarcinoma, affecting the management and
underestimating prospects for survival (4). There is typically a
delay of many years before the right diagnosis is made, by
which time metastases have occurred and survival has directly
been affected, as shown in Figure 2. Learning to recognize the
Figure 1. The evolution of major discoveries along the symptoms is very important for early diagnosis. Clinically
path of development of neurohumoral syndromes and suspicious symptoms necessitate biochemical testing. Clini-
their management. cally suspicious symptoms necessitate biochemical testing.

The Endocrine Society. Downloaded from press.endocrine.org by [${individualUser.displayName}] on 12 January 2017. at 12:19 For personal use only. No other uses without permission. . All rights reserved.
158 ENDO 2016 MEET-THE-PROFESSOR CLINICAL CASE MANAGEMENT

Diarrhea insulin, pancreatic polypeptide (PP), etc., and followed over


time. Potential diagnostic markers include chromogranin A,
(CgA) B (CgB), and C (CgC), 5 hydroxyindole acetic acid
Secretory persists Non-secretory
(5-HIAA), pancreastatin, and pancreatic polypeptide.
with fasng Improves with fasng
Endocrine Gastroenterological
Carcinoid Heart Disease
Carcinoid Heart Disease is characterized by fibrous endocardial
Figure 2. Differences in secretory and non-secretory thickening that mainly involves the right side of the heart. This
diarrhea. fibrous tissue characteristically devoid of elastic fibers is
known as carcinoid plaque. It causes retraction and fixation of
the tricuspid and pulmonary valves, which leads to valvular
LEARNING OBJECTIVES regurgitation, but pulmonary and tricuspid stenosis may also
As a result of participating in this session learners should be occur (9). The cause is unclear but direct actions of serotonin
able to: and bradykinin have been implicated in animal studies (10).
Recognize the clinical syndromes and different types of The clinical presentation is that of right-sided heart failure with
NETs responsible for the constellation of these features fatigue, dyspnea, ascites, edema, and cardiac cachexia. Left
Provide aids to differentiate NETs from masquerading heart disease is uncommon.
diseases
Understand the choices of management and the use of an Bronchoconstriction
algorithm for clinical, biochemical, and radiological Bronchoconstriction is clinically apparent as wheezing. The
diagnosis differential diagnosis includes asthma and chronic obstruc-
Understand and make informed choices based upon a tive pulmonary disease. The bronchospasm is usually caused
decision tree for management of NETs. by substance P, histamine, or serotonin (4).

STRATEGIES FOR DIAGNOSIS, THERAPY, Blood and Urine Biomarkers Potentially Useful for
AND/OR MANAGEMENT Diagnosis and Followup
The most important strategy is an alertness to the possibility of Several circulating tumor markers have been evaluated for the
a NET. The most common variety is the carcinoid tumor. It is diagnosis and followup of NETs; however, a tissue confirma-
the result of hypersecretion of vasoactive amines (eg, sero- tion is needed to make the diagnosis. Measurement of specific
tonin, histamine, tachykinins, and prostaglandins). It is com- hormones may be helpful and used in conjunction with imag-
mon with small-intestine NETs but also occurs with bronchial, ing to follow clinical status and treatment response. There is
ovarian, and other foregut carcinoids (5). Because the liver can controversy on the need for biomarkers and the frequency with
inactivate these substances, the carcinoid syndrome typically which they should be sampled in following progress and or
presents after hepatic metastasis have occurred. But this is not response to intervention. In some instances the relationship
essential in foregut NETs. between the clinical syndrome and the hormone implicated is
The clinical manifestations are: flushing (which occurs in clear, in which case the specific hormone causing the clinical
84% of patients), Diarrhea (70%), and heart disease (37%), but syndrome should be measured and followed over time (eg,
symptoms could also be widespread to include bronchospasm gastrin in gastrinoma syndrome). Other markers may also be
(17%) and myopathy (7%) (6, 7). Other recently recognized secreted by less-well-differentiated tumors and nonfunctioning
associated symptoms include abnormal increase in skin pig- ones (2). The key is to identify few biomarkers in a particular
mentation, which is a pellagra-like eruption (5%), arthropathy, patient and follow them over time in conjunction with symp-
paraneoplastic neuropathy, and edema (8). Mesenteric fibrosis toms and measurements of tumor bulk.
is associated with midgut carcinoids even in the absence of a Potential diagnostic markers include CgA, CgB, CgC,
visible mass and could compress the vessels, which leads to 5-HIAA, pancreastatin, and pancreatic polypeptide.
bowel ischemia and malabsorption.
The specific etiologic substances of each of the manifestations Chromogranin A
are not known. Serotonin, prostaglandin, 5 hydroxytryptophan CgA is a most important marker. It is a 49-KDa acidic poly-
(5-HTP), Substance P, kallikrein, histamine, dopamine, and peptide present in the secretory granules of all neuroendocrine
neuropeptide K are thought to be involved. Pancreatic polypep- cells. Its sensitivity varies between 53 and 68% and the speci-
tide and motilin levels are often elevated. ficity between 84 and 98% (1115). A recent meta-analysis of
Several circulating tumor markers have been evaluated for 13 studies has shown a high sensitivity of 73% and specificity
the diagnosis and follow up of NETs; however, a tissue confir- of 95% for the diagnosis of NETs (16). CgA level should be
mation is needed to make the diagnosis. The specific hormone measured fasting and exercise should be avoided before the
causing the clinical syndrome should be measured eg, gastrin, testing because both eating and exercise lead to increased

The Endocrine Society. Downloaded from press.endocrine.org by [${individualUser.displayName}] on 12 January 2017. at 12:19 For personal use only. No other uses without permission. . All rights reserved.
ENDO 2016 GENERAL ENDOCRINOLOGY 159

levels (15). Somatostatin analogs affect the CgA level so the Table 4 and lists tests suggested to help with the differential
serial measurements should be performed at the same interval and Table 5 suggests the various biomarkers.
from the injections.
There are caveats to the use of CgA as a universal tumor Features Associated With Different Flushing
marker for NETs. First, the level of CgA correlates with tumor Syndromes
volume (17); hence, small tumors may be associated with a When the flushing is dry it is due to a carcinoid tumor until
normal level. Second, false-positive measurements are reported proven otherwise.
in common conditions including decreased renal function, liver The flush in foregut tumors tends to be of protracted dura-
or heart failure, chronic gastritis, inflammatory bowel disease, tion, is often a purplish or violaceous hue, and frequently
hyperthyroidism, proton pump inhibitor (PPI) use, and even results in telangiectasia and hypertrophy of the skin of the face
benign essential hypertension and exercise-induced physical and upper neck. The face may assume a leonine characteris-
stress (18, 19). Also, elevations of CgA are reported in malig- tic resembling that seen in leprosy or acromegaly. The flush in
nant nonneuroendocrine tumors such as breast cancer and midgut tumors is of a faint pink to red color and involves the
hepatocellular carcinoma (15). These problems are not seen face and upper trunk down to the nipple line. It is initially
with CgB or pancreastatin (18). provoked by alcohol and tyramine-containing foods such as
blue cheese, chocolate, red wine, and red sausage. With time it
Markers Useful in Followup becomes spontaneous. It usually lasts for few minutes and
Pancreastatin may help monitor response to surgery and occurs many times a day. It generally does not lead to perma-
predict tumor growth. nent discoloration of the skin.
Neurokinin A is a possible prognosticating marker when
followed during treatment. Diarrhea
Neuron-specific enolase has a very low false-negative It is secretory in nature as are all endocrine diarrheas. As opposed
rate, which makes it a reasonable marker for follow up. to osmotic diarrhea, it generates a large amount of stool with no
MicroRNA profiling has entered the arena and when it osmotic gap and the key is that it persists with fasting. It occurs
becomes clearer what is being measured and what this in other syndromes such as watery diarrhea hypokalemia, hyp-
reflects may remain a prophecy yet to be fulfilled.
ochlorhydria, acidosis WDHHA syndrome (Verner-Morrison
NETs can also be nonfunctional and present with signs and syndrome/VIPoma), Zollinger-Ellison syndrome (Gastrinoma),
symptoms due to the mechanical complications (pain, obstruc- calcitonin-secreting tumors (medullary carcinoma of the thyroid or
tion, bleeding), but those silent tumors can at any point in time C cell hyperplasia), PPoma, and Substance Psecreting tumors.
start producing hormones and become syndromic (6). The In the gastrinoma syndrome the diarrhea is associated with
substance secreted by one tumor may change with time and steatorrhea and it improves with administration of a PPI or
yield an entirely different clinical syndrome. Indeed, metasta- histamine 2 blockers. The acidity in the duodenum and small
ses are known to each secrete different hormones than the intestine inactivates lipase, amylase, and trypsin, damages the
parent tumor. NETs can also secrete other substances not mucosa of the small intestine, and precipitates the succus
related to their original cell properties such as cytokines, auto- entericus, thereby causing malabsorption and steatorrhea.
antibodies, etc., which results in paraneoplastic syndromes In Verner-Morrison syndrome the diarrhea is associated
(20). See Table 1 for the frequent manifestations as well as the with hypercalcemia. VIP stimulates GI secretions and in-
seldom-discussed and unusual syndromes. The clinical presen- creases the rate of fluid delivery from the proximal to the
tations, the syndromes that are produced, the tumor type the distal small bowel so that it exceeds its absorptive capacity.
sites where the tumors are to be found, and the hormones, The diarrhea is watery and there is great loss of bicarbonate
amines and peptides synthesized and released into the periph- and potassium.
ery and that are responsible for many of the clinical features C-cell hyperplasia syndrome is a more recently described
are given in Table 1 below. In general NETs are named cause of secretory diarrhea and flushing. Total thyroidectomy
according to the hormone they produce (eg, gastrinoma if is the treatment of choice (Figure 2).
gastrin secreting, VIPoma if vasoactive intestinal peptide [VIP]
secreting). We do suggest an approach to diagnosing a NET Character of Secretory and Osmotic Diarrhea
based on the clinical presentation and the biochemical markers, Secretory
as summarized in Table 1.
Large-volume stools
Persists during fasting
Flushing
2 [Na K1 stool osmolality
Although a cardinal manifestation of carcinoid syndrome, it
Osmotic
occurs in other conditions such as menopause, panic attacks,
medullary thyroid cancer, autonomic neuropathy, mastocytosis, Small volume 1 L/d
and simultaneous ingestion of chlorpropamide, and alcohol. Disappears with fasting

The Endocrine Society. Downloaded from press.endocrine.org by [${individualUser.displayName}] on 12 January 2017. at 12:19 For personal use only. No other uses without permission. . All rights reserved.
160 ENDO 2016 MEET-THE-PROFESSOR CLINICAL CASE MANAGEMENT

Table 1. Clinical Presentation of NETs, Syndrome, Tumor Type, Site and Hormone Produced
Clinical Presentation Syndrome Tumor Type Site Hormone
Flushing Carcinoid Carcinoid Mid/foregut adrenal Serotonin, GCRP, calcitonin
medulla gastric
Medullary carcinoma C cell tumor Thyroid C cells Metanephrine and
of thyroid normetanephrine
Pheochromocytoma Tumor of chromaffin Adrenal and sympathetic
cells nervous system
Diarrhea abdominal pain Carcinoid, WDHHA, Carcinoid, VIPoma, As above, pancreas, mast As above, VIP, gastrin, PP,
and dyspepsia ZE, PP, MCT gastrinoma, cells, thyroid calcitonin
PPoma, medullary
carcinoma thyroid
mastocytoma
Diarrhea/steatorrhea Somatostatin Somatostatinoma, Pancreas Somatostatin
neurofibromatosis
Bleeding GI tract Duodenum
Wheezing Carcinoid Carcinoid Gut/pancreas/lung SP, CGRP, serotonin
Ulcer/dyspepsia Zollinger Ellison, Gastrinoma Pancreas/duodenum Gastrin
Hypoglycemia Whipples triad Insulinoma, sarcoma, Pancreas, retroperitoneal Insulin, IGF-I, IGF-II
hepatoma liver
Dermatitis Sweet syndrome Glucagonoma Pancreas Glucagon
Pellagra Carcinoid Midgut Serotonin
Dementia Sweet syndrome Glucagonoma Pancreas Glucagon
Diabetes Glucagonoma Glucagonoma Pancreas Glucagon
Somatostatin Somatostatinoma Pancreas Somatostatin
DVT, steatorrhea, Somatostatin Somatostatinoma Pancreas Somatostatin
cholelithiasis
Neurofibromatosis Duodenum
Silent, liver METS Silent PPoma Pancreas PP
Fever With weight loss and Any Any Cytokines (IL-6, NF-B, TNF-)
cachexia
Bone metastasis Pain/fracture/spinal Any Any Bone Alk phos N-telopeptide
compression
Paraneoplastic Peripheral neuropathy, Any Any Antibodies to calcium channels,
syndromes myopathy, acetylocholine receptors, C-
mystenia, CIDP, ANCA, P-ANCA, Hu
Lambert Eaton,
cerebellar ataxia

Abbreviations: ANCA, anti-neutrophil cytoplasmic antibodies; CGRP, calcitonin gene-related peptide; CIDP, chronic inflammatory
demyelinating polyneuropathy; DVT, deep vein thrombosis; MCT, medullary carcinoma of thyroid; WDHHA, watery diarrhea hypo-
chlorhydria hypokalemic acidosis; ZE, Zollinger-Ellison.
Data are from Vinik A et al (1).

2 [Na K1 stool osmolality ie, osmotic gap, Surreptitious laxative abuse


search for idiogenic osmoles Idiopathic
Causes of Secretory Diarrhea
Imaging of NETs
Watery diarrhea, hypokalemia, hyperchlorhydria, acido- The goal of imaging is to help make the diagnosis, determine
sis syndrome the tumor burden, and assess the potential for surgical resec-
Zollinger-Ellison syndrome tion, as well as establish the prognosis and determine the
Carcinoid potential for nonconventional therapies, especially in inoper-
Medullary carcinoma of thyroid able disease. Modalities include standard cross-sectional tech-
Secreting villous adenoma of rectum nique and nuclear functional imaging.

The Endocrine Society. Downloaded from press.endocrine.org by [${individualUser.displayName}] on 12 January 2017. at 12:19 For personal use only. No other uses without permission. . All rights reserved.
ENDO 2016 GENERAL ENDOCRINOLOGY 161

Table 2. Tests to Identify Causes of Flushing Although small familial clusters of midgut carcinoids have
Flushing Syndrome Associated Features been described, they are not associated with known genetic
Carcinoid Diarrhea, wheezing cancer syndromes. Among sporadic midgut carcinoids, several
Medullary carcinoma Mass in neck, family history studies using comparative genomic hybridization or microsatel-
thyroid lite markers have shown frequent allelic deletion of chromo-
Pheochromocytoma Paroxysmal hypertension, pallor some 18 (23, 24). On an epigenetic level, midgut NETs have
tachycardia been found to have global hypomethylation (25).
Diabetes Autonomic neuropathy, chlorpropramide Genetic analysis should be performed in suspected cases of
Menopause Cessation of menses MEN1, von Hippel-Landau, neurofibromatosis-1, and tuberous
Panic syndrome Phobias and anxiety sclerosis. Genetic counseling should be sought prior to testing
Mastocytosis Dyspepsia, peptic ulcer, dermatographia in all patients. Germline DNA testing is recommended in the
Polycythemia, renal Plethora presence of a positive family history of MEN1, if there are
cell carcinoma suspicious clinical findings, or if multiple tumors or precursor
Food Alcohol, MSG, nitrites, cheese, Tyramione
lesions are present. Somatic (tumor) DNA testing is not recom-
containing foods, red wine, dark
chocolate, hot dogs, dried fruit mended (26 28).
Drugs Niacin, phosphodiesterase inhibitors
Idiopathic Destitute MAIN CONCLUSIONS
NETs are slow-growing neoplasms capable of storing and
Abbreviation: MSG, monosodium gluconate. secreting different peptides and neuroamines. Some of these
Data are from Vinik A et al (1).
substances cause specific symptom complexes whereas others
are silent. They usually have episodic expression and the diag-
Table 3. Clinical Conditions an Biomarkers of Flushing nosis is often made at a late stage. Although considered rare,
the incidence of NETs is increasing. For these reasons a high
Clinical Condition Biomarkers
index of suspicion is needed. NETs on the increase require a
Carcinoid Plasma 5-HIAA, (22) 5-HTP, SP,
CGRP, CgA, pancreastatin,
high level of suspicion. We need to monitor the tumor burden,
neurokinin A Pro BNP the clinical response, and the appropriate biomarkers. There are
Medullary carcinoma Calcitonin, Calcium infusion RET biomarkers for specific NETS (eg, 5HIAA for carcinoid and
of the thyroid proto-oncogene hormones such as insulin, gastrin etc.) for secretory tumors.
Pheochromocytoma/ Plasma fractionated Markers for tumor behavior are Ki 67 sand mitotic indices of the
paraganglioma metanephrines and tumor histology and circulating levels of pancreastatin, neurokinin
catecholamines,
A, CgA, and neuron-specific enolase. Surgical removal of the
methoxytyramine, SHDBC
primary tumor is the first choice and one needs to be aggressive
Autonomic neuropathy Heart rate variability, 2-h post
prandial glucose and use tumor-debulking procedures of both the primary tumor
Menopause FSH and the metastases judiciously. Octreotide/lanreotide controls
Epilepsy Electroencephalogram symptoms and may cause biochemical and tumor burden im-
Panic Pentagastrin/ACTH provement. Tumor growth can be arrested. One must treat to
Mastocytosis Plasma histamine, urine tryptase target plasma octreotide/lanreotide plasma levels around 10 000
Hypomastia, Mitral Cardiac echo pg/mL, which is half saturation of the somatostatin 2 and 5
valve prolapse receptors present on NETs. Management of liver metastases
may require ancillary measures such as bland embolization,
Abbreviations: BNP, brain natriuretic peptide; CGRP, Strontium/Yttrium labeled resin microspheres therapy, radio-
calcitonin gene-related peptide; SHDBC, succinate dehy- frequency ablation, and chemoembolization. Therapy for
drogenase D, B & C.
somatostatin-positive tumors can use somatostatin peptide recep-
Data are from Vinik A et al (1).
tor radiotherapy now becoming available in the United States.
New therapies with agents acting on the RTK/PI3-K/AKT/mTOR
The sensitivity and specificity for some imaging modalities pathway and new somatostatin analogs are in the pipeline.
are shown in the Table 4. For a detailed discussion, see Vinik In this article we review the different clinical syndromes and
and Chaya (21). the pathophysiology of each tumor as well as the new and
Sensitivity, Specificity, Positive and Negative Predictive emerging biochemical markers and imaging techniques that
Values for Radiologic Diagnosis of NETS (6) should be used to facilitate an early diagnosis, followup, and
Gene Studies as Biomarkers of NETs prognosis.
The genetics of neuroendocrine tumorigenesis have yet to be As Moertel once said 3 decades ago, this is an Odyssey in
elucidated. the land of slow growing tumors. (31) We present here the

The Endocrine Society. Downloaded from press.endocrine.org by [${individualUser.displayName}] on 12 January 2017. at 12:19 For personal use only. No other uses without permission. . All rights reserved.
162 ENDO 2016 MEET-THE-PROFESSOR CLINICAL CASE MANAGEMENT

Table 4. Tests for Localization of NETs and Metastases


Test Sensitivity Specificity PPV NPP
CT, % 83 76
MRI, % 93 88
Ultrasound, % 5085 76-97
Octreoscan, % 5278 93 98 47
PET/CT 68Ga-DOTATOC, % 97 92
PET/CT 68Ga-DOTANOC, % 78 93
PET/CT 18F-FDG-PET, % (NETs 92
with proliferation index 15%)

Abbreviations: 68Ga-DOTATOC, gallium tetraazacyclododecane-tetraacetic acid; MRI, magnetic resonance imaging; NPP, negative
predictive valve; PET/CT, positron emission tomography/computerized tomography; PPV, positive predictive valve.

Table 5. Frequency of Major Clinical Manifestation and Cytokine Product


Major Clinical Manifestations Percent Tumor Production of Cytokines
Symptom/Sign TNF , IL-6, NF-B
Flushing 84 Fever, fatigue, weight loss, cachexia
Diarrhea 79
Heart disease 37
Bronchospasm 17
Seldom-discussed features Tumor stimulation of antibody formation
(Ca2 channels(PQ), achreceptors, CANCA, PANCA, Hu
Diabetes, metabolic syndrome, NASH 37
Hypertension 50 Neurological syndromes
Neuromyopathy 7 Somatic and autonomic neuropathy
Pigmentation arthropathy 5 Cerebellar ataxia
Hyper/hypoglycemia, NIHHPS 1 Eaton Lambert syndrome
Ulcer disease, skin rashes 1 Myaesthenia
Psychological disturbances 1 CIDP

Abbreviations: CANCA, cytoplasmic anti-neutrophil cytoplasmic antibopdies; CIDP, chronic inflammatory demyelenating polyneu-
ropathy; NASH, nonalcoholic steatohepatitis; NIHHPS, non insulinoma hyperinsulinemic, hypoglycemia syndrome; PANCA, peri-
nuclear anti-neutrophil cytoplasmic antibodies.

evolution of this odyssey and the rapid rate of progress that has ing pulse , 73 bpm. His physical examination was normal
been made in earlier and better identification with an increasing except for an ejection click at the left parasternal border with a
awareness and better tools for detection. soft systolic crescendo, decrescendo murmur at the left para-
sternal border. He has no myopathy, neuropathy, wheezing, or
CASES prolonged Forced Expiratory Volume clinically, and no proxi-
Case 1 mal myopathy. He had an ileocolectomy for a 1.5-cm tumor in
Mr. J.N. is a 51-year-old male who is being seen today for the appendix that was positive for synaptophysin and CgA. The
followup of his carcinoid syndrome. He presented in 2007 with mitotic index as 2% and the Ki67 index was 1%. He had
episodes of chest pain, dyspnea, feeling faint, and flushing, features of metabolic syndrome, mitral valve prolapse, degen-
which was dry. erative disc disease, dyslipidemia, and a history of night
The episodes occurred frequently, over the face and of the sweats.
rest of the body, lasted a few minutes and then disappeared His main complaints today are: 1) No new issues; 2) Maybe
without the development of telangiectasia. His 5HIAA was 50 a few less night sweats; 3) He cut back on the atenolol:
pg/mL. weight, 200 lbs; Waist, 39 in; hip, 40 in; waist/hip stopping over the past month (tapered off over several weeks).
ratio, 0.98; supine blood pressure, 122/78 mm Hg; supine 4) He denies shortness of breath, palpitations, or other issues.
pulse, 58 bpm; sitting blood pressure, 136/90 mm Hg; sitting 5) His chest x-ray was normal and his purified protein deriva-
pulse, 60 bpm, standing blood pressure, 128/84 mm Hg; stand- tive was negative. He did not do any new labs. 6) His lower

The Endocrine Society. Downloaded from press.endocrine.org by [${individualUser.displayName}] on 12 January 2017. at 12:19 For personal use only. No other uses without permission. . All rights reserved.
ENDO 2016 GENERAL ENDOCRINOLOGY 163

back is doing fine. He exercises several times a week: treadmill (0.0 58.8) (was 70 on 1/3/2012 and 76.5 on 12/12/2011);
and abdominal and running. Because of the family history of calcitonin 2.0 pg/mL; gastrin, 14; pancreastatin, 77 (was 93
colon cancer and he has had the gene analysis. on 1/3/2012). On 1/9/2012, urine 24-Hour 5-HIAA, 4.6.
He has had a return of the flushing postoperatively but now On 8/26/13 he presented with prolongation of FEV-1 and we
it is wet, lasts a few minutes, but his face is red and remains so ordered pulmonary function tests. It was done: and was told,
after the flush. His carcinoid score is 2/13 with the only good for his age.
positive feature being the wet flushing. He has not developed 2014: He has had some dyspepsia and reflux, has become
shortness of breath or pedal edema and there is no liver en- tired and lethargic, his skin is dry, hair is falling out, and his
largement, ascites, or pedal edema. The laboratory results are voice is croaky. All NET makers normal; B12, 1951; VIP, 61.9
normal CMP and Erythrocyte Sedimentation Rate. Calcitonin (0 58.8); histamine, 0.46 (1.0); gastrin 10 (0 20); 5HIAA
2 (0 8.4 pg/mL); gastrin, 18 (0 115 pg/mL); VIP 16.8 10, T4, F, 1.4; T3, 80; TSH, 7.0; electrocardiogram, low
(0 58.8 pg/mL); pancreatic glucagon, 80 (40 130 pg/mL); voltage; and his resting heart rate is 54.
pancreastatin, 37 (10 135 pg/mL); neurokinin A, 20 (40
pg/mL); CgA, 2 (0 6.0 pg/mL); Substance P, 106 (40 270 Case 3
pg/mL). His plasma 5HIAA, 10 (0 22 ng/mL); FSH, 3.0 A.J. is a 70-year-old female who presented in March 2008
mIU/ml (1.6 8); LH, 1.5 mIU/mL (1.7 8.6); erythrocyte sedi- with 4-5 months of increasing spells of palpitations, head-
mentation rate, 2 mm/h; total testosterone, 438 (398 1197 aches, flushing, tremors, weakness with paroxysmal epi-
ng/dL); free testosterone, 7.88 pg/mL (46 224); %free testos- sodes of hypertension (240/150 mm Hg), hypertension for
terone, 1.80; sex hormone binding globulin, 48.9 (19.376.4 many years.
nmoles/L). She was treated with Coreg CR, 80 mg/d; Benicar, 40 mg/d;
Tekturna, 300 mg/d; Clonidine, 0.1 mg four times daily;
Case 2 Verelan, PM 200 mg/night; Lorazepam, 1 mg every 4 hours for
Mr J.W. was first seen here 11 years ago when he was 67 years anxiety; Zocor, 20 mg/d; Plavix, 75 mg/d; Lasix, 20 mg/d as
of age. He complained of dyspepsia and reflux, was tired and needed; potassium, 20 mEq with Lasix; Fosamax, 70 mg/wk;
fatigued, and found to have a macrocytic anemia hemoglobin, Carafate, 1 g four times daily; albuterol nebulizer; Allegra, 10
8.0 g% and a routine colonoscopy and upper endoscopy was mg/d as needed.
performed and he was found to have a 1-cm carcinoid tumor in Past medical/surgical histories: Vitamin D deficiency, Iron
the stomach with a mitotic index of 2% and a Ki 67 index of deficiency since Billroth procedure (42 y earlier) for a bleeding
2.0%. He had no diarrhea, wheezing, or flushing. on exami- ulcer, hypertension, carotid artery stenosis, Asthma, gastro-
nation BP, 157/86 mm Hg; pulse, 54 bpm; weight, 154 lb; and esphogeal reflux disease, osteoarthritis, total abdominal hyster-
he had no myopathy but he had some loss of vibration detec- ectomy (1993) secondary to fibroid tumors, bladder lift (1987),
tion threshold in the feet, loss of ankle jerks, and the laser sinus operations (1994 and 1996).
Doppler blood flow indicated increase in blood flow in the feet Examination: BP, 178/79 mm Hg; Pulse, 78 bpm; Coughing
as is seen in Charcot neuroarthropathy. spells with wheezing; Regular rate and rhythm with a systolic
Biochemistries: Serotonin, 1119 ng/mL; CgA, 69 nmol/L; murmur and left carotid bruit.
CA, 19 9, 2.5 U/mL; carcinoembryonic antigen, 0.6 mcg/L; Biochemistries: plasma norepinephrine, 1072 pg/mL (0-399
histamine, 0.27 pg/mL; gastrin, 1551 pg/mL; calcitonin, 7 pg/mL); epinephrine, 92 pg/mL; dopamine, 33 pg/mL; total
pg/mL; somatostatin, 128 pg/mL; 5HIAA urine, 375 mg/24 h; catecholamine, 1197 pg/mL (0-642 pg/mL); plasma, metanephrines,
insulin, 3.4 micro Iu/mL; prolactin, 6.5 pg/mL (14 64); PTH, 59 pg/mL; normetanephrines, 125 pg/mL (18-111 pg/mL);
39.2 pg/mL (14 64); Ca, 9.9 mg/dL; cortisol, 26.8 mg/dL; CgA, 10 nmol/L (0-5 nmol/L); urine 24-hour 5-HIAA, 10.5
gastric parietal antibodies, 38.9 (0-20); ACTH 13 pg/mL. mg/d (0-8 mg/d).
We offered him surgical therapy but he was reticent until 1 year Discontinue clonidine; start dibenzyline (Phenoxy-
later and he underwent an antrectomy, which was extended to benzamine), titrate slowly up from 10 mg once daily; Demser
66% gastrectomy when more carcinoid nodules were found in the (metyrosine), 250 mg as a backup; sandostatin LAR, 30 mg
stomach and a Billroth 11 was completed successfully. once monthly; left carotid artery stenting procedure performed
Three years later he was feeling well without complaints in May 2008.
except for mild dyspepsia and his primary care physician gave Family history: negative for carcinoid or neuroendocrine
him Nexium, which he said helped. The biochemical workup tumors; positive for cancer (mother, cervical; brother, prostate).
revealed: CMP, normal except for fasting glucose, which was June 2008: exploratory laparotomy; lysis of adhesions; ex-
113; blood urea nitrogen, 23 mg/dL; CO2, 31; globulin, 1.5 tensive small-bowel resection including mesenteric mass, and
g/dL; A/G ratio, 3.0. Substance P, 243 pg/mL; CgA, 8 nmol/L anastomosis more than 20 tumors (size, 0.3-2.0 cm); several
(0 5) (was 4 on 12/12/2011); glucagon, 75 pg/mL; histamine, had invaded the serosa with positive satellite lesions in the
0.33 p/mL; neurokinin A, 5.0 pg/mL; pancreatic polypeptide, mesentery; perineural and lymphovascular invasion with 5/6
186.5 pg/mL; serotonin, 104 ng/mL; VIP, 81.4 pg/mL lymph nodes positive for metastasis.

The Endocrine Society. Downloaded from press.endocrine.org by [${individualUser.displayName}] on 12 January 2017. at 12:19 For personal use only. No other uses without permission. . All rights reserved.
164 ENDO 2016 MEET-THE-PROFESSOR CLINICAL CASE MANAGEMENT

August/September 2008 Followup: Still flushing (like eggs Norvasc, 5 mg/d; chlorthalidone, 25 mg/d; and Sunitinib, 50
frying on my forehead and cheek), which signifies hyperten- mg/d, 2 weeks on, 2 weeks off.
sive attack; Zero energy. August 2008: BP, 135/69 mm Hg; December: No flushing, two syncopal episodes; BP,
pulse, 78 bpm. September 2008: BP, 165/78 mm Hg; pulse, 80 120/60-150/100 mm Hg; reduced all medications, became hy-
bpm; still getting hypertensive episodes (240-250/120-130 mm pothyroid and need T4 replacement. Plasma catecholamines:
Hg) and going to the emergency department every few weeks. norepinephrine, 3757 pg/mL (0-400 pg/mL); Epinephrine, 104
Blood pressure medications: Dibenzyline, 240 mg four times pg/mL (0-100 pg/mL); dopamine, 167 pg/mL (0-143 pg/mL);
daily; Demser, 250 mg thrice daily; Coreg, 80 mg/d; Benicar, total catecholamines, 4028 pg/mL (0-643 pg/mL); plasma
40 mg/d; Tekturna, 300 mg/d; Verelan, 200 mg/d. metanephrines, 90 ug/mL (0-62 ug/mL); plasma normetane-
Biochemistries: plasma norepinephrine, 1003 pg/mL; epineph- phrines, 535 pg/mL (0-145 pg/mL); fasting glucose, 214 mg/dL
rine, 49 pg/mL; dopamine, 16 pg/mL; total catecholamines, 1068 (65-99 mg/dL). CT chest, no metastases; discontinued
pg/mL; plasma metanephrines, 87 pg/mL (12-60 pg/mL); sandostatin and started Somatulin, 120 mg twice monthly.
normetanephrines, 192 pg/mL (18-111 pg/mL). I-(123) meta-
July 2010: BP, 130-140/80-90 mm Hg with rare readings at
iodobenzylguanidine scan.
180/100 mm Hg.
August 2008: No definitive evidence for of metaio-
Biochemistries: CgA, 6 nmol/L; pancreastatin, 63 pg/mL;
dobenzylguanidine-avid tumor or significant interval change.
plasma metanephrines, 45 pg/mL; plasma normetanephrines,
OctreoScan, September 2008: Decreased but residual
112 pg/mL. Magnetic resonance image of abdomen/pelvis with
somatostatin-positive disease in the midline of the abdomen
and without contrast (June 2010), no evidence of tumor recur-
and pelvis. No new area of disease identified.
rence or metastatic disease.
Abdominal exploration, November 2008 w/gamma probe: re-
moval of mesenteric lymph nodes, 4/10 positive for carcinoid.
Pathology: Synaptophysin/chromogranin positive; S-100, Case 4
negative except for rare entrapped dendritic cells and Ms. L.W. is a 58 year-old female whose carcinoid was diag-
adipocytes; Ki-67 1%. Fewer spells next few months but BP nosed in 3/19/2010 (Figure 3). Her symptoms of edema, short-
unchanged and fractionated metanephrines slight improvement. ness of breath, and fatigue with flushing occurred several times
February 2009: still having hypertensive episodes (200s/100s a day as did diarrhea, which occurred when she was not eating.
mm Hg) although no flushing to alert to episode, just feel funny; She was found to have bilobar hepatic metastases and peripor-
BP, 169/78 mm Hg; Pulse, 58 bpm. 5HIAA normal. Octreoscan- tal and periaortic lymph node involvement. The serotonin was
negative MIBG diffuse uptake in the lungs. Computed tomogra- 800 and the CgA, 333. She initially received Octreotide, 90
phy (CT) innumerable pulmonary nodules bilateral. mg Q3/52; and on 5/12/2010 underwent an ileocolectomy. The
September 2009: Octrescan negative and MIBG scan nega- cardiac ejection fraction was 55%. Between 2010 and 2013 she
tive. Still hypertensive 162/66 mm Hg; pulse, 61 bmp. Added underwent SIRS, embolization of liver metastases, or debulk-

Figure 3. Clinical Course of Patient LW with Metastatic Carcinoid.

The Endocrine Society. Downloaded from press.endocrine.org by [${individualUser.displayName}] on 12 January 2017. at 12:19 For personal use only. No other uses without permission. . All rights reserved.
ENDO 2016 GENERAL ENDOCRINOLOGY 165

ing with little effect on her symptoms of biochemistries. She to gastric parietal cells, which fail to secrete acid and the
had developed ascites, massive liver enlargement, and periph- loss of suppression of gastrin unbridles its secretion.
eral edema with markedly elevated pulsatile jugular venous Gastrin is trophic to enterochromaffin like cells cells and
pressure. The ejection fraction was 15% and she was shown to increase growth and proliferation leading to a gastric
have mitral, tricuspid, and pulmonary valve incompetence with carcinoid. Although this can be controlled with
fibrosis. She underwent mitral, tricuspid, and pulmonary valve somatostatin this is very expensive, requires shots for life,
replacement and the cardiac status stabilized and she is pit of and there are failures and a small but significant rate of
failure. However, her biochemistries were grossly abnormal, malignancies. The hypergastrinemia is not due to the PPI,
she had profuse diarrhea, lost a great deal of weight, and was which for the most part does not increase of gastrin to
admitted for failure to thrive. The diarrhea was only controlled 400 pg/mL, and there is no need to do a secretin test.
with LAR, 90 mg Q3 weekly and somatostatin analog, 500 ug The presence of parietal antibodies combined with a
neutral gastric pH would have been enough. Antrectomy is
(morning) and 1000 ug (evening). She underwent a CT chest,
entirely appropriate. Type 2 occurs with MEN and there
abdomen, and pelvis. This showed no evidence of bone metas-
were no other hormone abnormalities. Type 3 is sporadic
tases. Intra-abdominal carcinomatosis no change. Iliac lymph
and has the highest rates of malignancy and should require
nodes increasing in size. No additional changes. A trial of
total gastrectomy.
Affinitor caused a pulmonary infection with nasal bleeding and
thrombocytopenia. Xeloda was not tolerated and she received
Case 3
palliative 5FU and leukovorin. Her EF returned to 55% and she
Question: What suggested to you that this patient had a para-
left hospital having gained 20 lb and was in great spirits. Her
ganglioma combined with carcinoid?
serotonin is 1500 ng/dL and the pancreastatin 20 000 pg/mL
A. Hypertensive crises
and the neurokinin A is 80 pg/mL. We reviewed her at tumor
board and are considering further SIRS therapy. Alternatively, B. Elevation of both 5HIAA and fractionated
we will do radioembolization. metanephrines
C. Immunohistochemistry for synaptophysin, CgA in the
CASE DISCUSSION AND ANSWERS mesenteric nodes
Case 1
Is the cause of the red face due to: D. Responsiveness to Sunitinib
A. Alcohol consumption E. The need for more than three dugs to control BP
B. Hyperthyroidism including Demser
Answer: None of the above. The secretion of
C. Diabetes facialis diabeticorum
norepinephrine and not epinephrine and the increased
D. Hindgut carcinoid dopamine excluded a pheochromocytoma and suggested
E. Midgut carcinoid paraganglioma. Hypertensive crises occur in both
pheochromocytoma and paraganglioma and the high levels
F. Mitral valve prolapse of norepinephrine could derive from both
Answer: F, mitral valve prolapse. He does not drink pheochromocytoma and paraganglioma and the
alcohol, he is clinically euthyroid and not diabetic.
fractionated metanephrines would not have made the
Hindgut NETs to not case flushing and do not secrete
diagnosis. Immunohistochemistry for CgA and
serotonin or other vasodilators and midgut carcinoid flush
synaptophysin simply recognizes an endocrine tumor. The
is ephemeral lasting only minutes and do not leave a
hypertension in pheochromocytoma and paraganglioma is
permanent discoloration. The relief of symptoms with
such to require large doses of antihypertensive agents. The
atenolol is typical of mitral valve prolapse.
response to Sunitinib of paragangliomas has now been
established since this index case was reported.
Case 2
1. Is this a type 1 carcinoid?
Case 4
2. Is this a type 2 carcinoid? Question: What are the findings that alert you to the possibility
3. Is this a type 3 carcinoid? of carcinoid heart disease?
4. Is the hypergastrinemia due to the PPI? A. The symptoms of diarrhea and flushing

5. Was it correct to do an antrectomy? B. The cardiac EF 50%


Answer: This is a type 1 carcinoid of the stomach. There C. The elevated serotonin to 800 pg/mL
are three types of gastric carcinoid. Type 1 occurs with
atrophic gastritis, the most common cause being antibodies D. Pedal edema

The Endocrine Society. Downloaded from press.endocrine.org by [${individualUser.displayName}] on 12 January 2017. at 12:19 For personal use only. No other uses without permission. . All rights reserved.
166 ENDO 2016 MEET-THE-PROFESSOR CLINICAL CASE MANAGEMENT

E. Shortness of breath, elevated JVP, and a mitral valve for diagnosis and follow-up of endocrine tumours. Clin Endocrinol (Oxf).
2004;60:644-652.
murmur of incompetence
14. Bajetta E, Ferrari L, Martinetti A, et al. Chromogranin A, neuron specific
Answer: D, pedal edema suggesting possible right heart enolase, carcinoembryonic antigen, and hydroxyindole acetic acid evalua-
failure. The symptoms of diarrhea and flushing occur in tion in patients with neuroendocrine tumors. Cancer. 1999;86:858-865.
carcinoid syndrome. Cardiac EF is near normal. Carcinoid 15. Lawrence B, Gustafsson BI, Kidd M, Pavel M, Svejda B, Modlin IM. The
clinical relevance of chromogranin A as a biomarker for gastroen-
heart disease is almost always associated with serotonin teropancreatic neuroendocrine tumors. Endocrinol Metab Clin North Am.
values 1000 pg/mL. Although it occurs, left ventricular 2011;40:111-134, viii.
dysfunction is rare and not the rule in carcinoid diseases 16. Yang X, Yang Y, Li Z, et al. Diagnostic value of circulating chromogranin
a for neuroendocrine tumors: A systematic review and meta-analysis. PLos
that shortness of breath with a mitral valve murmur is One. 2015;10:e0124884.
unexpected. The case illustrates how well we can do in the 17. Kolby L, Bernhardt P, Sward C, et al. Chromogranin A as a determinant of
face of widely metastatic tumor, objectively addressing the midgut carcinoid tumour volume. Regul Pept. 2004;120:269-273.
18. Stridsberg M, Eriksson B, Fellstrom B, Kristiansson G, Tiensuu Janson E.
cardiac component and reducing tumor bulk to enhance Measurements of chromogranin B can serve as a complement to chromo-
responsiveness to standard therapy. granin A. Regul Pept. 2007;139:80-83.
19. Takiyyuddin MA, Cervenka JH, Hsiao RJ, Barbosa JA, Parmer RJ,
OConnor DT. Chromogranin A. Storage and release in hypertension.
REFERENCES Hypertension. 1990;15:237-246.
1. Massironi S, Sciola V, Peracchi M, Ciafardini C, Spampatti MP, Conte D. 20. Li SC, Khan M, Caplin M, Meyer T, Oberg K, Giandomenico V. Soma-
Neuroendocrine tumors of the gastro-entero-pancreatic system. World J tostatin analogs treated small intestinal neuroendocrine tumor patients
Gastroenterol. 2008;14:5377-5384. circulating microRNAs. PLos One. 2015;10:e0125553.
2. Eriksson B, Oberg K, Stridsberg M. Tumor markers in neuroendocrine 21. Vinik AI, Chaya C. Clinical presentation and diagnosis of neuroendocrine
tumors. Digestion. 2000;62 Suppl 1:33-38. tumors. Hematol Oncol Clin North. 2016;30(1):21-48.
3. Yao JC, Hassan M, Phan A, et al. One hundred years after carcinoid: 22. Tellez MR, Mamikunian G, ODorisio TM, Vinik AI, Woltering EA. A
Epidemiology of and prognostic factors for neuroendocrine tumors in single fasting plasma 5-HIAA value correlates with 24-hour urinary
35,825 cases in the United States. J Clin Oncol. 2008;26:3063-3072. 5-HIAA values and other biomarkers in midgut neuroendocrine tumors
4. Vinik A, ODorisio T, Woltering E, Go VL. Neuroendocrine tumors: A (NETs). Pancreas. 2013;42:405-410.
comprehensive guide to diagnosis and management. 1st ed. Inglewood, 23. Wang GG, Yao JC, Worah S, et al. Comparison of genetic alterations in
CA: Inter-Science Institute, 2006. neuroendocrine tumors: Frequent loss of chromosome 18 in ileal carcinoid
5. Davis Z, Moertel CG, McIlrath DC. The malignant carcinoid syndrome. tumors. Mod Pathol. 2005;18:1079-1087.
Surg Gynecol Obstet. 1973;137:637-644. 24. Kytola S, Hoog A, Nord B, et al. Comparative genomic hybridization
6. Vinik AI, Woltering EA, Warner RR, et al. NANETS consensus guidelines identifies loss of 18q22-qter as an early and specific event in tumorigenesis
for the diagnosis of neuroendocrine tumor. Pancreas. 2010;39:713-734. of midgut carcinoids. Am J Pathol. 2001;158:1803-1808.
7. Creutzfeldt W, Stockmann F. Carcinoids and carcinoid syndrome. Am J 25. Choi IS, Estecio MR, Nagano Y, et al. Hypomethylation of LINE-1 and
Med. 1987;82:4-16. Alu in well-differentiated neuroendocrine tumors (pancreatic endocrine
8. Vinik AI, Gonzales MR. New and emerging syndromes due to neuroen- tumors and carcinoid tumors). Mod Pathol. 2007;20:802-810.
docrine tumors. Endocrinol Metab Clin North Am. 2011;40:19-63, vii. 26. Brandi ML, Gagel RF, Angeli A, et al. Guidelines for diagnosis and
9. Tormey WP, FitzGerald RJ. The clinical and laboratory correlates of an therapy of MEN type 1 and type 2. J Clin Endocrinol Metab.
increased urinary 5-hydroxyindoleacetic acid. Postgrad Med J. 1995;71: 2001;86:5658-5671.
542-545. 27. Thakker RV. Multiple endocrine neoplasia type 1 (MEN1). Best Pract Res
10. Fox DJ, Khattar RS. Carcinoid heart disease: Presentation, diagnosis, and Clin Endocrinol Metab. 2010;24:355-370.
management. Heart. 2004;90:1224-1228. 28. Toumpanakis CG, Caplin ME. Molecular genetics of gastroenteropancreatic neu-
11. Goebel SU, Serrano J, Yu F, Gibril F, Venzon DJ, Jensen RT. Prospective roendocrine tumors. Am J Gastroenterol. 2008;103:729-732.
study of the value of serum chromogranin A or serum gastrin levels in the 29. Bayliss WM, Starling EH. Preliminary communication on the causation of
assessment of the presence, extent, or growth of gastrinomas. Cancer. the so-called peripheral reflex secretion of the pancreas. Lancet.
1999;85:1470-1483. 1902;1:813-814.
12. Bernini GP, Moretti A, Ferdeghini M. A new human chromogranin A 30. Yalow RS, Berson SA. Immunoassay of endogenous plasma insulin in
immunoradiometric assay for the diagnosis of neuroendocrine tumours. man. J Clin Invest. 1960;39:1157-1175.
Br J Cancer. 2001;84:636-642. 31. Moertel CG. Karnofsky memorial lecture: an odyssey in the land of small
13. Nehar D, Lombard-Bohas C, Olivieri S, et al. Interest of Chromogranin A tumors. J Clin Oncol. 1987;5:1502-1522.

The Endocrine Society. Downloaded from press.endocrine.org by [${individualUser.displayName}] on 12 January 2017. at 12:19 For personal use only. No other uses without permission. . All rights reserved.
ENDO 2016 GENERAL ENDOCRINOLOGY 167

Hypoglycemic Disorders

M31 Operator-dependence/lack of reproducibility of some


Presented, April 1 4, 2016 localization studies
Poor characterization of histologic changes in the endocrine
pancreas associated with postprandial hypoglycemia (3-6)
Adrian Vella, MD. Division of Endocrinology, Diabetes,
Metabolism and Nutrition, Mayo Clinic, Rochester, Knowledge Gaps
Minnesota 55905, E-mail: vella.adrian@mayo.edu Inability to predict insulinoma behavior based on
imaging and histologic characteristics
INTRODUCTION The utility of an abnormal Selective Arterial Calcium
Historical Overview Stimulation Test (SACST) in the absence of documented
Hypoglycemia was recognized as a biochemical anomaly in hypoglycemia
humans in the early part of the twentieth century. However, it Pathogenesis/pathophysiology of postprandial
was not until the introduction of insulin therapy for the treat- hypoglycemia after Roux-en-Y gastric bypass (7)
ment of diabetes in 1922 that hypoglycemia gained importance
in the medical literature. The first clinical descriptions of hy-
poglycemia appeared in 1922 concurrently with the description LEARNING OBJECTIVES
of the use of insulin for the treatment of diabetes. In 1936, the As a result of participating in this session, participants will be
pancreatic surgeon, Allen O. Whipple, was the first to describe able to:
the difference between hypoglycemia, which occurred during Understand the principles behind the diagnostic criteria
fasting and that which occurred only in response to the inges- for hypoglycemia (8).
tion of carbohydrates (11). Whipples research showed that Develop a differential diagnosis for a patient with a
only patients with symptoms associated with fasting hypogly- hypoglycemic disorder (9).
cemia benefited from pancreatectomy. These patients were Recognize the roles and limitations of imaging studies in
often found to have an islet cell tumor. In an article titled, The the evaluation of patients with hypoglycemia (10).
Surgical Therapy of Hyperinsulinism (1), Whipple proposed
that no pancreatic surgery to look for an insulinoma be per-
STRATEGIES FOR DIAGNOSIS, THERAPY,
formed unless certain criteria were met. This diagnostic test,
AND/OR MANAGEMENT
now known as Whipples triad, is an essential tool in the
Hypoglycemic disorders have been typically classified by the
diagnosis of hypoglycemia. The use and significance of Whip-
timing of hypoglycemia (fasting vs postprandial). However, it
ples triad have evolved over the last century as the understand-
is important to understand that such classification provides no
ing of the forms of hypoglycemia has increased and diagnostic
information about the underlying disorder. Moreover, patients
tests and imaging procedures have improved.
can be uncertain as to the actual timing of their symptoms.
Finally, fasting and postprandial symptoms can coexist in the
SIGNIFICANCE OF THE CLINICAL PROBLEM
same patient. As such we prefer a classification based on the
Although true hypoglycemic disorders are generally a very small
part of endocrine practice, patients with symptoms suggestive of context in which hypoglycemia occurs (12). The differential
hypoglycemia are very common. As such it is important to de- diagnosis of hypoglycemia in a sick, critically ill patient is very
velop a rational approach to patients with these symptoms to avoid different from that in an otherwise-healthy patient. Hence, a
unnecessary investigation, surgery, or medical treatment with their history with particular attention paid to the medications that the
associated costs and comorbidities. Although the investigation and patient has been exposed to is a key part of the evaluation
management of patients with fasting hypoglycemia seems to be especially in the hospitalized patient (9).
well understood, there are significant gaps in our ability to suc- The diagnosis of hypoglycemia depends on fulfilling Whip-
cessfully diagnose and treat patients with postprandial symptoms, ples triad. This dictates that it is necessary to document low
most especially after Roux-en-Y gastric bypass. This presentation blood glucose at the time of symptoms compatible with
is intended to discuss our current practice and management of neuroglycopenia. These symptoms should be ameliorated by
patients with hypoglycemic symptoms. correction of the hypoglycemia. Only after these criteria are
fulfilled should one embark on testing to determine the mecha-
BARRIERS TO OPTIMAL PRACTICE nism by which hypoglycemia occurs (2). In such circumstances
Challenges one may utilize a spontaneous episode of hypoglycemia (pro-
Lack of standardization among immunoassays (2) vided the context is known) or else undertake provocative tests
Limited availability of sulfonylurea screens, which such as a supervised fast or a mixed-meal challenge to replicate
reliably detect fourth-generation sulfonylureas the conditions that provoke hypoglycemia. Such tests are neces-

The Endocrine Society. Downloaded from press.endocrine.org by [${individualUser.displayName}] on 12 January 2017. at 12:19 For personal use only. No other uses without permission. . All rights reserved.
168 ENDO 2016 MEET-THE-PROFESSOR CLINICAL CASE MANAGEMENT

sary to differentiate between insulin-mediated and noninsulin- the former tests typically encompass transabdominal ultrasound
mediated hypoglycemia. Assuming that hyperinsulinemia is the and triple-phase computed tomography, whereas the latter en-
underlying cause of hypoglycemia, it is also important to ex- compass endoscopic ultrasound and the SACST. The combina-
clude factitious causes of hypoglycemia by screening for sul- tion of noninvasive tests can localize an insulinoma 70 80%
fonylurea abuse using a sulfonylurea screen that can detect of the time, whereas invasive testing is necessary for the
compounds such as glimepiride and repaglinide. remainder (10).

Supervised Fast SACST


The supervised fast is usually undertaken for 72 hours and The SACST has been (probably mistakenly) characterized as
should be performed in a unit with experience in its conduct. In both a diagnostic and localization test (3, 17). It is indicated in
a group of 170 insulinoma patients, 93% had a positive fast by patients with hyperinsulinemic hypoglycemia in the absence of
48 hours of fasting and 99% by 72 hours (13). Increasingly, a lesion or multiple lesions on imaging (where the source of
this is being conducted initially as an outpatient given the high insulin production is uncertain) and surgery is contemplated.
frequency of hypoglycemia in the first 24 hours (10) and the This technique is highly dependent on operator experience and
utility of insulin surrogates in determining whether a fast ability to interpret the angiogram that accompanies the func-
should proceed (14, 15). Other criteria for ending the super- tional testing. The procedure includes puncture of the femoral
vised fast include documentation of Whipples triad or a glu- vein with catheterization of the right hepatic vein via the
cose reading less than 55 mg/dL with prior documentation of inferior vena cava. Calcium gluconate is selectively injected
Whipples triad even in the absence of neuroglycopenia. into the gastroduodenal artery, superior mesenteric artery, and
Ending the fast may be a difficult, especially in the setting of splenic artery. An increase in hepatic vein insulin of at least
nonspecific symptoms with a glucose concentration near the 2-3-fold at 20, 40, and 60 seconds after injection of calcium
hypoglycemic threshold. Confounding this difficulty is the fact will localize the excess insulin secretion to head of the pan-
that low serum glucose may be physiological, and glucose creas (gastroduodenal artery), the uncinate (superior mesenteric
levels in the 40-50 mg/dL range are not uncommonly seen, artery), and the body or tail (splenic artery) if typical anatomy
particularly in young lean women. Simple bedside tests of is present.
neurocognitive function should be performed at scheduled in- More recently we have characterized the effect of aberrant
tervals and at the time of symptoms to establish whether arterial anatomy on the interpretation of SACST (18) for
neuroglycopenia is occurring. proven insulinoma, biochemical results were positive in more
than one arterial distribution in approximately 25% of patients.
Mixed-Meal Study The sensitivity for insulinoma localization was 54.8% using
Standards have not been established for the mixed-meal study. only the diagnostic arteriography information, increasing to
The study is performed in patients with symptoms of postpran- 73.8% when biochemical data was included, and increasing to
dial hypoglycemia. The test is performed over 5 hours after the 92.8% when arteriographic and biochemical data was com-
patient ingests a meal that typically produces symptoms (although bined with anatomic and perfusion data. The biochemical data
there is one important caveat). A positive result includes symp- have some utility in differentiating insulinoma from other
toms suggestive of neuroglycopenia in the setting of a glucose less causes of hypoglycemia (19) in which both maximum and
than or equal to 50 55 mg/dL. It is important to note that a relative-fold increase in hepatic venous insulin concentration
positive test does not provide a diagnosis, but rather confirmation over baseline following calcium injection into the dominant
of Whipples triad in the postprandial state. The criteria for artery were significantly higher in insulinomas compared with
hyperinsulinemic hypoglycemia should not be used (or used with other causes of hypoglycemia.
extreme caution) in the postprandial setting (2).
CASES
Insulin Antibodies Case 1: Establish the Presence of a Hypoglycemic
Insulin antibodies should be measured in all cases of Disorder
hyperinsulinemic hypoglycemia. Antibody-mediated hypogly- A 38-year-old woman is referred for evaluation of noninsulin-
cemia is a rare disorder more commonly seen in Asians of mediated hypoglycemia. The patient was investigated for over-
Korean or Japanese descent (16). Patients often have a history whelming fatigue associated with tachycardia, sweating, and
of other autoimmune disorders. This disorder arises from the tremor that occurred throughout the day. These episodes had no
unregulated release of insulin bound to the antibodies indepen- clear association with food intake but seemed to be precipitated
dent of the prevailing serum glucose. Symptoms of hypoglyce- by exertion. Because of this she was admitted for a 72-hour
mia may range from mild to severe, and can occur in both the supervised fast. This proceeded uneventfully until the 60th
fasting and postprandial state. hour when at 0100 hours a reflectance meter glucose obtained
In hyperinsulinemic hypoglycemia, localization tests could by finger stick was low at 45 mg/dL. The patient was subse-
be divided into noninvasive and invasive tests. In my practice quently awoken from sleep and asked about accompanying

The Endocrine Society. Downloaded from press.endocrine.org by [${individualUser.displayName}] on 12 January 2017. at 12:19 For personal use only. No other uses without permission. . All rights reserved.
ENDO 2016 GENERAL ENDOCRINOLOGY 169

symptoms. She reported the presence of her usual symptoms. mg/dL increase in glucose in response to glucagon at the end of
End of fast procedures were commenced. Lab values revealed fast. Hemolysis might explain low insulin concentrations but
glucose of 51 mg/dL with undetectable insulin and C-Peptide. would not explain the absence of other characteristics compatible
-Hydroxybutyrate was 7.4 mmol/L ( 0.4 mmol/L) and the with an insulin-mediated process.
response to glucagon was flat. The diagnosis of a hypoglycemic disorder rests on the dem-
onstration of the Whipple triad (ie, symptoms that are compat-
Questions ible with neuroglycopenia occur at the time of a low venous
1. Which of the following is the most likely diagnosis? glucose and are corrected by correction of the hypoglycemia).
A. Insulinoma These criteria are not fulfilled in this instance because the fast
B. Surreptitious insulin administration was stopped not by the development of neuroglycopenia but by
C. Noninsulin mediated hypoglycemia a glucose value below 55 mg/dL. The patients symptoms
D. Hypoglycemia caused by an insulinlike factor developed when she was awoken and at that time were typical
E. Cannot determine from the available history of the symptoms she experienced frequently, but not typical for
2. Has Whipples triad been established? neuroglycopenia in which fatigue is not the main symptom.
A. Yes For the reasons expounded in the answers above I do not
B. No think that further management aimed at treating a hypoglyce-
3. Which of the following would be the best approach to mic disorder is indicated.
management for this patients symptom complex?
A. Frequent meals Case 2: Same but Different
B. Diazoxide A 28-year-old female is referred for evaluation, 6 weeks post-
C. No further management at this time partum. She had an elective cesarean section at 38 weeks and
D. Dietary modification this was uneventful, leading to delivery of a healthy 7 lbs 7 oz
E. Surgical consultation baby. The postpartum course was uneventful with establish-
ment of breast feeding, etc., until the third day after delivery,
when the patient was found unresponsive in her room. Venous
Answers
glucose was low at 36 mg/dL but she responded to treatment
The laboratory data are compatible with normal prolonged
with iv dextrose. A hypoglycemic agent screen obtained sub-
fasting in which insulin secretion is suppressed and ketones
sequently was negative. The patient feels well at the present
serve as the primary substrate for metabolism. -cell function
time. She has not experienced a recurrence of her symptoms
is suppressed when glucose values decrease below the 65-60
although she is eating frequent small meals. Examination is
mg/dL range. Typically, it is best to interpret detectable con-
remarkable for a body mass index of 19 kg/m2 and a blood
centrations of -cell polypeptides when glucose is below the
pressure of 110/65 mmHg. There is no orthostatism. After
55-60 mg/dL range and it is important to bear in mind that
some of these polypeptides have prolonged half-lives in the discussion with the patient, an outpatient fast was initiated, and
circulation (proinsulin, C-peptide). The data are interpretable then continued as an inpatient. This was stopped after 32 hours
given the glucose value at the end of the fast. The data would of fasting when the patient developed symptoms of hypoglyce-
be compatible with noninsulin mediated hypoglycemia if the mia. At the time glucose was 39 mg/dL with undetectable
patient had clear evidence of neuroglycopenia. In such circum- insulin and C-peptide. Administration of glucagon did not raise
stances, waking up a patient who was sleeping makes this un- glucose by greater than 25 mg/dL.
likely. It is important to document mental status continuously
during the fast using a simple method adapted to the patients Questions
baseline (eg, subtraction of serial 7s if the patient is able to do 1. Which of the following is the most likely diagnosis?
this without error at baseline). In this circumstance the patient was A. Insulinoma
prompted to report symptoms that were not really compatible with B. Surreptitious insulin administration
neuroglycopenia. To be compatible with the presence of an C. Abuse of oral hypoglycemic agents
insulin-like factor causing hypoglycemia, one would expect D. Hypopituitarism
-hydroxybutyrate to be suppressed and that there would be E. Cannot determine
enough hepatic glycogen reserves which, when mobilized in re- 2. Has Whipples triad been established?
sponse to glucagon, generate a 25 mg/dL increase in glucose A. Yes
concentrations. This would strongly suggest the presence of a B. No
circulating insulin-like hormone such as IGF-II, which is often a 3. Which of the following would be the best approach to
mediator of tumor-associated hypoglycemia. However, these cri- management for this patients symptom complex?
teria are not fulfilled. Insulin-mediated hypoglycemia would also A. Pancreatic exploration
be characterized by suppressed -hydroxybutyrate and a 25 B. Steroid replacement

The Endocrine Society. Downloaded from press.endocrine.org by [${individualUser.displayName}] on 12 January 2017. at 12:19 For personal use only. No other uses without permission. . All rights reserved.
170 ENDO 2016 MEET-THE-PROFESSOR CLINICAL CASE MANAGEMENT

Answers 3. Which of the following would be the best approach to


In this patient, the -cell polypeptides at the time of hypogly- management for this patients symptom complex?
cemia are appropriately suppressed; therefore, this process is A. Enucleation of the pleural tumor
not insulin mediated. The hypoglycemia is also not compatible B. Medical management of hypoglycemia
with the presence of an insulin-like factor given that there was C. Other therapeutic modalities
no response to glucagon at the end of the fast. Noninsulin-
mediated hypoglycemia is uncommon, but can occur in the
Answers
setting of severe malnutrition such as anorexia nervosa or The criteria necessary to fulfill Whipples triad are fulfilled,
primary or secondary adrenal insufficiency. The fact that the and therefore the diagnostic process moves toward elucidating
patients BMI is 19 kg/m2 and she was able to become preg- the mechanism of hypoglycemia. Measurement of -cell poly-
nant and establish breastfeeding makes a diagnosis of anorexia peptides in this instance clearly demonstrates that they are
nervosa unlikely. The patient had an undetectable ACTH level appropriately suppressed and this process is therefore not insu-
and a cortisol concentration of 2 g/dL (0800 h), compatible lin mediated. The data are compatible with the presence of an
with secondary adrenal insufficiency. There was a blunted insulin-like factor causing hypoglycemia. Despite suppressed
response to 250 mcg of exogenous ACTH (cosyntropin). A insulin concentrations at the time of hypoglycemia, there were
urine screen for synthetic steroids was negative. enough hepatic glycogen reserves that, when mobilized in
In this case, Whipples triad is fulfilled given the change in response to glucagon, generated a greater than 25 mg/dL in-
cognition accompanying low blood glucose (with reversal crease in the glucose concentration. This strongly suggests the
when hypoglycemia is corrected). Subsequently a 72-hour fast presence of a circulating insulin-like hormone such as IGF-II,
that was stopped prematurely documented the presence of which is often a mediator of tumor-associated hypoglycemia.
neuroglycopenia at the time of a glucose of 39 mg/dL. Whipples Triad has been clearly established during the outpa-
Pituitary magnetic resonance imaging suggested autoim- tient fast. At the time of initial presentation, the fingerstick glucose
mune hypophysitis. Corticosteroid replacement has ensured precludes strong conclusions regarding fulfilment of the Triad.
that the patient continues to be symptom free. However, subsequent formal evaluation clearly demonstrates hy-
poglycemia with accompanying neuroglycopenia.
Case 3: Insulin Surrogates Are Important Thoracic fibromas often present in the manner described
A 52-year-old man is undergoing an outpatient evaluation for above and may be amenable to resection. However, anatomy
dyspnea and accompanying chest pain in a radicular distribu- often precludes complete resection and a combination of thera-
tion. He is noted to be confused and dysarthric on awakening peutic interventions is necessary. This includes embolization of
one morning and is taken to the emergency department where a the tumor, frequent timed caloric intake, and other therapies
fingerstick glucose concentration is 45 mg/dL. He receives intended to decrease secretion of insulin-like factors by the
treatment with iv dextrose, which leads to prompt resolution of tumor.
his symptoms and he is admitted for evaluation. This episode
prompts a request to you as an endocrinologist to evaluate the REFERENCES
patient. Findings on physical examination are unremarkable. A 1. Whipple AO. The surgical therapy of hyperinsulinism. J Internat Chir.
chest x-ray performed as part of his initial evaluation reveals a 1938;3:237-276.
2. Cryer PE, Axelrod L, Grossman AB, Heller SR, Montori VM, Seaquist
dense, pleural-based mass approximately 5 cm in diameter. ER, Service FJ. Evaluation and management of adult hypoglycemic disor-
After discussion with the patient, an outpatient fast is initiated. ders: An Endocrine Society Clinical Practice Guideline. J Clin Endocrinol
This is stopped after 4 hours when the patient develops symp- Metab. 2009;94:709-728.
3. Service GJ, Thompson GB, Service FJ, Andrews JC, Collazo-Clavell ML,
toms of hypoglycemia. At the time glucose was 44 mg/dL with Lloyd RV. Hyperinsulinemic hypoglycemia with nesidioblastosis after
undetectable insulin and C-peptide. Administration of glucagon gastric-bypass surgery. N Engl J Med. 2005;353:249-254.
raised glucose by greater than 25 mg/dL. 4. Meier JJ, Butler AE, Galasso R, Butler PC. Hyperinsulinemic hypoglyce-
mia after gastric bypass surgery is not accompanied by islet hyperplasia or
increased beta-cell turnover. Diabetes Care. 2006;29:1554-1559.
Questions 5. Patti ME, McMahon G, Mun EC, et al. Severe hypoglycaemia post-gastric
bypass requiring partial pancreatectomy: Evidence for inappropriate insu-
1. Which of the following is the most likely diagnosis? lin secretion and pancreatic islet hyperplasia. Diabetologia. 2005;48:2236-
A. Insulinoma 2240.
B. Surreptitious insulin administration 6. Rumilla KM, Erickson LA, Service FJ, et al. Hyperinsulinemic hypogly-
cemia with nesidioblastosis: Histologic features and growth factor expres-
C. Tumor-mediated hypoglycemia
sion. Mod Pathol. 2009;22:239-245.
D. Hypopituitarism 7. Vella A, Service FJ. Incretin hypersecretion in post-gastric bypass
E. Antibody-mediated hypoglycemia hypoglycemiaPrimary problem or red herring? J Clin Endocrinol
2. Has Whipples triad been established? Metab. 2007;92:4563-4565.
8. Cryer PE, White NH, Santiago JV. The relevance of glucose counter-
A. Yes regulatory systems to patients with insulin-dependent diabetes mellitus.
B. No Endocr Rev. 1986;7:131-139.

The Endocrine Society. Downloaded from press.endocrine.org by [${individualUser.displayName}] on 12 January 2017. at 12:19 For personal use only. No other uses without permission. . All rights reserved.
ENDO 2016 GENERAL ENDOCRINOLOGY 171

9. Service FJ. Hypoglycemic disorders. N Engl J Med. 1995;332:1144-1152. 16. Basu A, Service FJ, Yu L, Heser D, Ferries LM, Eisenbarth G. Insulin
10. Placzkowski KA, Vella A, Thompson GB, et al. Secular trends in the autoimmunity and hypoglycemia in seven white patients. Endocr Pract.
presentation and management of functioning insulinoma at the Mayo 2005;11:97-103.
Clinic, 1987-2007. J Clin Endocrinol Metab. 2009;94:1069-1073. 17. Doppman JL, Miller DL, Chang R, Shawker TH, Gorden P, Norton JA.
11. Marks V, Rose FC. Hypoglycaemia, Second Edition. Oxford, UK: Insulinomas: Localization with selective intraarterial injection of calcium.
Blackwell Scientific Publications; 1981. Radiology. 1991;178:237-241.
12. Service FJ. Diagnostic approach to adults with hypoglycemic disorders.
18. Thompson SM, Vella A, Service FJ, Grant CS, Thompson GB, Andrews
Endocrinol Metab Clin North Am. 1999;28:519-532, vi
JC. Impact of variant pancreatic arterial anatomy and overlap in regional
13. Service FJ, Dale AJ, Elveback LR, Jiang NS. Insulinoma: Clinical and
perfusion on the interpretation of selective arterial calcium stimulation
diagnostic features of 60 consecutive cases. Mayo Clin Proc. 1976;51:417-
429. with hepatic venous sampling for preoperative localization of occult
14. Service FJ, OBrien PC. Increasing serum betahydroxybutyrate concentra- insulinoma. Surgery. 2015;158:162-172.
tions during the 72-hour fast: Evidence against hyperinsulinemic hypogly- 19. Thompson SM, Vella A, Thompson GB, Rumilla KM, Service FJ, Grant
cemia. J Clin Endocrinol Metab. 2005;90:4555-4558. CS, Andrews JC. Selective arterial calcium stimulation with hepatic ve-
15. OBrien T, OBrien PC, Service FJ. Insulin surrogates in insulinoma. nous sampling differentiates insulinoma from nesidioblastosis. J Clin
J Clin Endocrinol Metab. 1993;77:448-451. Endocrinol Metab. 2015;100:4189-4197.

The Endocrine Society. Downloaded from press.endocrine.org by [${individualUser.displayName}] on 12 January 2017. at 12:19 For personal use only. No other uses without permission. . All rights reserved.
172 ENDO 2016 MEET-THE-PROFESSOR CLINICAL CASE MANAGEMENT

Whats New in the Management of Multiple Endocrine


Neoplasia Type 2?

M32 SIGNIFICANCE OF THE CLINICAL PROBLEM


Presented, April 1 4, 2016 Substantial advances in our understanding and management of
MEN2, the association of MTC, hyperparathyroidism and uni-
lateral or bilateral PHEOs, have reduced morbidity and death
Robert F. Gagel, MD. Professor of Medicine, MD. for most patients with this genetic syndrome. Early thyroidec-
Anderson Cancer Center, Houston, Texas 77030, E-mail: tomy and central node dissection for MTC and laparoscopic or
rgagel@mdanderson.org cortical sparing adrenalectomy for treatment of PHEO have
reduced mortality to very low levels. However, delay in appro-
priate treatment can lead to metastasis of MTC and death or
HISTORICAL OVERVIEW
morbidity from PHEO.
Medullary thyroid carcinoma (MTC) was first identified as a
separate entity in 1959 (1), and during the same year, Copp
described the hypocalcemic peptide, calcitonin (2), later shown to BARRIERS TO OPTIMAL PRACTICE
be from the thyroid gland by Hirsch (3) and Foster (4). In 1961, a The greatest threat to surgical cure of hereditary MTC is an
medical resident, John Sipple, attended the autopsy of a patient he incomplete or delayed thyroidectomy. The thyroid C cells are
had seen in consultation for hypertension and cerebral hemor- distributed throughout the thyroid gland with the greatest con-
rhage. He later stated I was amazed when I saw large, bilateral centration occurring at the junction of the upper one-third and
pheochromocytomas and a 2 cm pale tan mass in each lobe of the lower two-thirds of each lobe along a hypothetical cephalad-
thyroid gland and nodular enlargement of the only parathyroid caudal central axis (12). Mutant RET is expressed in each C
gland we could find (5). He could find no reported association cell and an incomplete thyroidectomy may result in residual
between pheochromocytomas (PHEOs) and thyroid and parathy- cells that may transform later in life. Similarly, delay of thy-
roid neoplasms. So he dug deeper in the library stacks and was roidectomy may result in the development of nodal metastasis.
able to find 5 other cases of carcinoma of the thyroid in patients Total thyroidectomy in a young child is a challenging proce-
with pheochromocytoma (5). Although he and his colleagues dure and should be performed by an individual knowledgeable
incorrectly identified the type of thyroid cancer, he is generally about this procedure and this disease. Performance of cortical-
credited with piecing together the several components of this sparing adrenalectomy is a technically challenging procedure
syndrome (6). From that point forward the field moved very when performed laparoscopically and should also be performed
quickly. In 1966, Williams hypothesized that neuroendocrine cells by a surgeon experienced in the technique (22).
in the thyroid were the transformed cell type in MTC and pro-
duced calcitonin (7). In 1968, Sipple syndrome was renamed LEARNING OBJECTIVES
multiple endocrine neoplasia type 2 (MEN2) to distinguish it from As a result of participating in this session, learners will be
MEN1 (8). The identification of a cohesive family in the greater able to:
Boston area in 1968 led Melvin, Miller, and Tashjian to use a 1. Understand how mutations of the RET proto-oncogene
newly developed calcitonin RIA to diagnose hereditary MTC in cause transformation of the C cell and development of
the context of MEN2A before any clinical evidence of the disease PHEOs.
(9, 10). This led to prospective screening for hereditary MTC by 2. Properly apply genetic testing in MEN2 kindreds.
measurement of serum calcitonin after provocative calcium or 3. Recognize the importance of timely and appropriate
pentagastrin tests, leading to the recognition that hyperplasia of C surgical treatment of MTC and PHEO.
cells preceded the development of MTC (11, 12) and that early 4. Explain the declining importance of
thyroidectomy and central lymph node dissection could cure these hyperparathyroidism in prospectively screened families.
children and young adults (13, 14). During this period, it was also 5. Demonstrate how the identification of RET mutations
recognized that hyperplasia of adrenal medullary cells preceded has led to targeted therapy with tyrosine kinase
the development of PHEOs (15, 16). Large and well-defined inhibitors (TKIs) for treatment of metastatic MTC.
kindreds identified by prospective screening were used to geneti-
cally map the causative disease gene to chromosome 10q (17, 18), STRATEGIES FOR DIAGNOSIS, THERAPY, OR
and in 1993, mutations of the RET (REarranged during Transfec- MANAGEMENT
tion) proto-oncogene were identified (19, 20). Genetic testing was MEN2 is an autosomal dominant genetic syndrome character-
rapidly adopted as the basis for thyroidectomy in gene carriers and ized by the presence of MTC, parathyroid neoplasia and hyper-
is the current standard for diagnosis (21). parathyroidism, and unilateral or bilateral PHEO. All 3 of these

The Endocrine Society. Downloaded from press.endocrine.org by [${individualUser.displayName}] on 12 January 2017. at 12:19 For personal use only. No other uses without permission. . All rights reserved.
ENDO 2016 GENERAL ENDOCRINOLOGY 173

manifestations are caused by one of several germline mutations tial to transform and metastasize at a later date. These
of the RET proto-oncogene. There are 2 broad categories of considerations must be balanced again the very real con-
MEN2, MEN2A and MEN2B. MEN2A is characterized by the cerns of hypothyroidism, hypoparathyroidism, and recurrent
presence of MTC (100%), hyperparathyroidism (10%), and laryngeal nerve damage in a young child. For this reason, it
unilateral or bilateral PHEOs (50%). MEN2B is characterized is important that the surgeon be experienced with thyroidec-
by early onset MTC (100%), PHEOs (50%), and mucosal tomy in the context of MEN2.
neuromas in the lips, eyelids, oral cavity, and throughout the A question that is raised repetitively is how long can one
gastrointestinal tract and Marfanoid-like skeletal features. Al- wait to perform a thyroidectomy in a child with a moderate risk
though both syndromes are rare, MEN2A is far more common mutation (27). In truth, there are insufficient data to make firm
than MEN2B. The MTC associated with MEN2B is more recommendations regarding this question; expert opinion sug-
aggressive than that associated with MEN2A. gests that delaying thyroidectomy (in a patient with a moderate
risk mutation) to the age of 10 years may be a reasonable
How Does Activated RET Cause MTC? course of action.
Activating mutations of RET cause autophosphorylation of the After thyroidectomy, monitoring with annual serum basal
receptor and phosphorylation of downstream signaling mol- calcitonin and thyroid hormone measurements and a high-
ecules causing activation of MAPK and c-Jun N-terminal resolution ultrasound of the head and neck (every 35 y) is
kinases pathways (23, 24). One of the hallmarks of hereditary appropriate.
MTC is the accumulation of C cells that occurs before the
development of MTC. Recent studies have provided potential Management of Hyperparathyroidism
insight into a mechanism for accumulation of C cells (25). Evaluation for hyperparathyroidism by measurement of serum
After its activation (through ligand interaction or mutation), calcium every other year is appropriate. Long-term studies
RET is translocated to the nucleus, where it interacts with and have demonstrated that hyperparathyroidism in patients with
phosphorylates activating transcription factor 4 (ATF4), a MEN2A is uncommon (certainly less than the 10%15% re-
cAMP response element-binding protein transcription factor. ported before the beginning of prospective screening in the
ATF4 is an important activator of apoptosis through its effect 1970s). It has been speculated that the loss of parathyroid
to enhance expression of NOXA (phorbol-12-myristate-13-ac- tissue may occur with a prophylactic total thyroidectomy, ex-
etate-induced protein 1) and PUMA (the p53 upregulated plaining the current low incidence of hyperparathyroidism.
modulator of apoptosis), activators of apoptosis. Down-
regulation of ATF4 causes reduced cell death. In addition, RET Management of PHEO
translocation to the nucleus raises the very real possibility that The clinical features of MEN2-associated PHEO differ from
it interacts with other factors that may be important in the that found in sporadic or other entities such as von Hippel
transformation process. Lindau (VHL) syndrome or hereditary paraganglioma. Patients
with MEN2A are more likely to present with palpitations,
Management of MTC intermittent tachycardia, attacks of nervousness and intermit-
Prospective screening of MEN2A families by calcitonin mea- tent headaches, reflecting the greater production of epinephrine
surements in the 1970s1980s led to the recognition that C cell by these tumors (13, 28). Although death from cardiac arrest
abnormalities could be identified before metastasis and treated caused by PHEO occurred with some frequency in the early
in childhood or teenage years by total thyroidectomy. days of the recognition of this syndrome (Sipples original case
Intermediate-term follow up studies indicate that prophylactic likely died from cerebral hemorrhage caused by PHEO), death
thyroidectomy resulted in surgical cure in 80%90% of cases from PHEO is currently uncommon.
operated at a mean age of 15 years (26). After the identification Prospective screening for PHEO can be performed by an-
of RET proto-oncogene mutations in 1993, a consensus was nual measurement of plasma fractionated metanephrines (the
quickly reached that children with the most common RET most sensitive) or plasma catecholamines, beginning after the
codon 634 mutation should have a total thyroidectomy per- age of 5 years (28). The peak age for development of PHEO is
formed by the age of 6 years. This age was chosen because of during the second and third decade, but occasionally may occur
a single child with this mutation who presented with metastatic in children under the age of 10 years. Parents should be
MTC at age 6 years. Experience with aggressiveness of MTC attentive for tachycardia or other intermittent adrenergic symp-
in kindreds with mutations of other codons has permitted a toms in their children. Computerized tomography provides the
stratification of mutations from highest risk to medium risk and best quality images and should be performed only in children
recommendations for age of thyroidectomy (27). or young adults with adrenergic symptoms or abnormal test
It is important that a total thyroidectomy and central results to reduce radiation exposure. Radionuclide scanning
lymph node dissection be performed. Because the germline with radiolabeled octreotide or met-iodo benzyl guanidine is
RET mutation is expressed in all C cells, any residual C occasionally helpful to improve sensitivity, but is unneeded in
cells not removed by the surgical procedure have the poten- most cases.

The Endocrine Society. Downloaded from press.endocrine.org by [${individualUser.displayName}] on 12 January 2017. at 12:19 For personal use only. No other uses without permission. . All rights reserved.
174 ENDO 2016 MEET-THE-PROFESSOR CLINICAL CASE MANAGEMENT

The treatment for PHEOs in the context of MEN2 has lial growth factor (VEGF) receptor (VEGFR) and epidermal
evolved from laparoscopic adrenalectomy to retroperitoneal growth factor (EGFR) active ATP analog reduced tumor size
laparoscopic adrenalectomy (29). The advantages of a retro- by 30% or greater in 45% of patients in a phase III trial and
peritoneal approach are substantial, it is not necessary to enter prolonged progression-free survival by 11 months (31).
the peritoneal cavity, making it possible to discharge a patient Cabozantinib, a TKI that targets RET, VEGFR, EGFR, and the
from the hospital 24 hours following a successful procedure. proto-oncogene MET (c-Met protein) prolonged progression-
A second trend that has emerged over the past 2 decades is the free survival in a phase III trial from 4 to 11.2 months with an
goal of preserving adrenal cortical function. Several deaths from
overall response rate of 28% for active drug vs 0% for placebo
acute adrenal insufficiency in adrenalectomized patients have been
(32). Vanetanib was approved for treatment of progressive
reported and has led to a resurgence of interest in cortical sparing
metastatic MTC in 2011 and cabozantinib in 2012. Both are
adrenalectomy, a procedure in which it is possible to bisect the
now used regularly in patients with metastatic MTC and have
adrenal gland, preserving the blood supply, and excise a small
intramedullary PHEO (30). This procedure is less feasible in altered outcomes in this disease, although the duration of effect
patients with larger pheochromoctyomas. It is more challenging, is unclear at this time. Other agents have activity in MTC but
but technically possible, to perform a cortical sparing adrenalec- have not been approved for this use by either the American or
tomy using a laparscopic approach; there is limited experience European regulatory agencies (Table 1).
with cortical sparing adrenalectomy using the posterior retroperi- These agents have significant toxicity and are best adminis-
toneal laparoscopic approach. The advantages to the patient are tered by oncologists with experience treating this disease or
significant, no need to take corticosteroid and mineralocorticoid endocrinologists who have acquired skill in the management of
replacement. The procedure is successful in maintaining adequate toxicity associated with the use of these compounds. The im-
adrenal function in approximately 80% 85% of cases when per- portant role of endocrinologists in the development of these
formed by an experienced surgical team. Inevitably adrenal med- agents has led to the development of oncologic endocrinology
ullary tissue remains after successful cortical sparing adrenalec- as subspecialty of endocrinology.
tomy and the recurrence rate for PHEO in the operated gland is
10%15%.
MAIN CONCLUSIONS
1. The management of MEN2 has evolved steadily over
Management of Metastatic MTC
Ten years ago there were no effective therapies for treatment of the past 30 years; in 2016, death from either MTC or
metastatic MTC. Response rates for cytotoxic chemotherapy PHEO is uncommon in kindreds where prospective
ranged from 10% to 25% with few durable responses. The screening, either by calcitonin or genetic testing has
development of tyrosine kinase inhibitors (TKIs) that target been applied.
several tyrosine kinase receptors including RET has provided a 2. Successful management of MTC and PHEOs requires
more effective therapy. Vandetanib, a RET, vascular endothe- skilled surgeons with unique technical expertise. In no

TABLE 1. Tyrosine Kinase Inhibitors Studied in Medullary Thyroid Carcinoma


Receptor Tyrosine Kinase Activity Efficacy in MTC
Compound IC50 (nM) Partial Response Rate
VEGFR2 RET Other
Vandetanib 40 100 EGFR 500 Phase III
47%
Cabozantinib 0.035 4.5 C-MET 1.8 Phase III
28%
Axitinib 0.25 PDGFR 1.7 Phase II
18%
Lenvatinib 4 35 FGFR 46 Phase II
36%
Sorafenib 90 49 PDGFR 58 Phase II
25%, 6%
Sunitinib 9 41 Phase II
50%
Pazopanib 30 2800 PDGFR 74 Phase II
14%

The Endocrine Society. Downloaded from press.endocrine.org by [${individualUser.displayName}] on 12 January 2017. at 12:19 For personal use only. No other uses without permission. . All rights reserved.
ENDO 2016 GENERAL ENDOCRINOLOGY 175

other disease is the dictum get it right the first time (normal 2 pg/mL in a thyroidectomized patient), which
more important. stimulates to a value of 97 pg/mL after a short calcium
3. The discovery of RET mutations has led to the infusion. Ultrasound examination of the neck shows no
development of tyrosine kinase inibitors, now approved abnormal lymph nodes.
for treatment of MTC, that have changed the 1. Does this patient have metastatic MTC?
management of metastatic MTC caused by hereditary or 2. Should the patient have microsurgical bilateral neck
sporadic disease. lymph node dissection?
3. Should this patient have external beam radiotherapy to
Case 1 the neck and upper mediastinum?
A 35-year-old male presents with new onset tremulousness, 4. Should RET proto-oncogene testing be performed?
palpitations, and attacks of nervousness over the past 6 8
months. On your initial evaluation you note a 1- to 1.5-cm Case Discussion
left-sided thyroid nodule with no adenopathy. Plasma It is likely this patient has metastatic MTC. A normal postop-
metanephrines and catecholamines are elevated 3 times above erative serum calcitonin value in a thyroidectomized male,
the upper range of normal. A computerized tomography scan using current 2-site assay technology is less than 2 pg/mL, and
shows a left adrenal mass. A serum calcium is normal. The the rise in serum calcitonin following calcium stimulation also
patient describes a deceased uncle had a tumor of the adrenal supports this conclusion. During the period from 1990 to 2005,
gland removed almost 40 years ago. there was enthusiasm for performing detailed bilateral lymph
1. Which of the following diagnostic studies should be per- node dissection in such patients, a procedure that required a 4-
formed in this patient before treatment of the PHEO? to 6-hour operative procedure. Several series demonstrated that
a. An ultrasound of the thyroid such treatment lower the basal serum calcitonin to undetectable
b. A fine needle aspiration (FNA) of the thyroid nodule levels in 5%10% of cases; however, the incidence of hypo-
c. A serum calcitonin parathyroidism and XII nerve damage after such procedures
d. A RET proto-oncogene analysis was significant and physicians who treat this disorder have
e. A FNA with molecular analysis of the aspirate become less enthused about extensive neck dissection without
2. You elect to biopsy the thyroid nodule and FNA reveals a evidence of lymph node disease by ultrasound or other imaging
benign colloid cyst. Which of the following genetic techniques. External beam radiotherapy in a patient with no
causes of PHEO is most likely? disease identified by imaging would not be appropriate and is
a. MEN1 generally considered only in patients with extensive lymph
b. MEN2 node metastasis or those with soft tissue extension of disease.
c. VHL syndrome Testing for germline mutations of the RET proto-oncogene
d. Hereditary paraganglioma should be performed in all patients with apparent sporadic
e. Neurofibromatosis MTC. A total of 5%7% will have a germline mutation of the
RET proto-oncogene identified that may impact treatment of
Case Discussion children or other relatives.
This patient has a likely PHEO. The PHEO should be ad-
dressed first and appropriate treatment would include - and Case 3
possibly -adrenergic antagonists followed by a laparoscopic A 54-year-old male presents with a 20 year history of meta-
adrenalectomy. The finding of a colloid nodule makes MEN2 static MTC. On his annual CT scan of the neck, he is noted to
and MTC unlikely in a 35-year-old male, but if either the have a new left paratracheal focus of metastatic MTC that
ultrasound or the serum calcitonin were abnormal, it would be appears to invade into the wall of the trachea. Surgical consul-
prudent to perform a RET proto-oncogene mutation analysis tation indicates that a complete resection would require total
because of the vague and unconfirmed history of an adrenal laryngectomy. The patient is hesitant and eventually declines
tumor in an uncle. If the RET analysis shows evidence of surgical intervention. Should TKI therapy be considered?
mutation known to cause MEN2, a total thyroidectomy with or The case with images will be discussed at the Meet the
without neck dissection should be performed. If negative other Professor session.
causes of hereditary PHEO (VHL, hereditary paraganglioma,
VHL, or neurofibromatosis) should be considered.
REFERENCES
1. Hazard JB, Hawk WA, Crile G Jr. Medullary (solid) carcinoma of the thyroid:
Case 2 a clinicopathologic entity. J Clin Endocrinol Metab. 1959;19:152-161.
A 45-year-old male presents with apparent sporadic MTC. 2. Copp DH, Davidson AGF, Cheney BA. Evidence for a new parathyroid
hormone which lowers blood calcium. Proc Canad Fed Biol Soc.
Total thyroidectomy is performed at another hospital and
1961;4:17.
MTC was identified in 2 of 6 lymph nodes sampled. Post- 3. Hirsch PF, Voelkel EF, Munson PL. Thyrocalcitonin: hypocalcemic,
operatively, the patient has a serum calcitonin of 8 pg/mL hypophosphatemic principle of the thyroid gland. Science. 1964;146:412-414.

The Endocrine Society. Downloaded from press.endocrine.org by [${individualUser.displayName}] on 12 January 2017. at 12:19 For personal use only. No other uses without permission. . All rights reserved.
176 ENDO 2016 MEET-THE-PROFESSOR CLINICAL CASE MANAGEMENT

4. Foster GV, Baghdiantz A, Kumar MA, Slack E, Soliman HA, Macintyre I. are associated with MEN 2A and FMTC. Hum Mol Genet.
Thyroid origin of calcitonin. Nature (London). 1964;202:1303-1305. 1993;2(7):851-856.
5. Sipple JH. Multiple endocrine neoplasia type 2 syndromes: historical 20. Mulligan LM, Kwok JB, Healey CS, et al. Germ-line mutations of the
perspectives. Henry Ford Hosp Med J. 1984;32(4):219-221. RET proto-oncogene in multiple endocrine neoplasia type 2A. Nature.
6. Sipple JH. The association of pheochromocytoma with carcinoma of the 1993;363(6428):458-460.
thyroid gland. Am J Med. 1961;31:163-166. 21. Brandi ML, Gagel RF, Angeli A, et al. Guidelines for diagnosis and
7. Williams ED. Histogenesis of medullary carcinoma of the thyroid. J Clin therapy of MEN type 1 and type 2. J Clin Endocrinol Metab.
Pathol. 1966;19:114-118. 2001;86(12):5658-5671.
8. Steiner AL, Goodman AD, Powers SR. Study of a kindred with pheochro- 22. Castinetti F, Qi XP, Walz MK, et al. Outcomes of adrenal-sparing surgery
or total adrenalectomy in phaeochromocytoma associated with multiple
mocytoma, medullary carcinoma, hyperparathyroidism and Cushings dis-
endocrine neoplasia type 2: an international retrospective population-based
ease: multiple endocrine neoplasia, type 2. Medicine. 1968;47:371-409.
study. Lancet Oncol. 2014;15(6):648-655.
9. Melvin KEW, Miller HH, Tashjian AH Jr. Early diagnosis of medullary
23. Salvatore D, Barone MV, Salvatore G, et al. Tyrosines 1015 and 1062 are
carcinoma of the thyroid gland by means of calcitonin assay. N Engl in vivo autophosphorylation sites in ret and ret-derived oncoproteins.
J Med. 1971;285:1115-1120. J Clin Endocrinol Metab. 2000;85(10):3898-3907.
10. Melvin KEW, Tashjian AH Jr, Miller HH. Studies in familial (medullary) 24. Santoro M, Carlomagno F, Romano A, et al. Activation of RET as a
thyroid carcinoma. Recent Prog Horm Res. 1972;28:399-470. dominant transforming gene by germline mutations of MEN 2A and MEN
11. Wolfe HJ, Melvin KE, Cervi-Skinner SJ, et al. C-cell hyperplasia preced- 2B. Science. 1995;267:381-383.
ing medullary thyroid carcinoma. N Engl J Med. 1973;289:437-441. 25. Bagheri-Yarmand R, Sinha KM, Gururaj AE, et al. A novel dual kinase
12. Wolfe HJ, Voelkel EF, Tashjian AH Jr. Distribution of calcitonin- function of the RET proto-oncogene negatively regulates activating transcrip-
containing cells in the normal adult human thyroid gland: a correlation tion factor 4-mediated apoptosis. J Biol Chem. 2015;290(18):11749-11761.
of morphology with peptide content. J Clin Endocrinol Metab. 26. Skinner MA, Moley JA, Dilley WG, Owzar K, Debenedetti MK, Wells SA
1974;38:688-694. Jr. Prophylactic thyroidectomy in multiple endocrine neoplasia type 2A.
13. Gagel RF, Tashjian AH Jr, Cummings T, et al. The clinical outcome of N Engl J Med. 2005;353(11):1105-1113.
prospective screening for multiple endocrine neoplasia type 2a: an 18-year 27. Wells SA Jr, Asa SL, Dralle H, et al. Revised American thyroid associa-
experience. N Engl J Med. 1988;318(8):478-484. tion guidelines for the management of medullary thyroid carcinoma. Thy-
14. Graze K, Spiler IJ, Tashjian AH Jr, et al. Natural history of familial roid. 2015;25(6):567-610.
medullary thyroid carcinoma: effect of a program for early diagnosis. 28. Eisenhofer G, Lenders JW, Linehan WM, Walther MM, Goldstein DS,
N Engl J Med. 1978;299(18):980-985. Keiser HR. Plasma normetanephrine and metanephrine for detecting pheo-
15. Carney JA, Sizemore GW, Tyce GM. Bilateral adrenal medullary hyper- chromocytoma in von Hippel-Lindau disease and multiple endocrine neo-
plasia type 2. N Engl J Med. 1999;340(24):1872-1879.
plasia in multiple endocrine neoplasia, type 2: the precursor of bilateral
29. Walz MK, Peitgen K, Hoermann R, Giebler RM, Mann K, Eigler FW.
pheochromocytoma. Mayo Clin Proc. 1975;50(1):3-10.
Posterior retroperitoneoscopy as a new minimally invasive approach for
16. DeLellis RA, Wolfe HJ, Gagel RF, et al. Adrenal medullary hyperplasia. A
adrenalectomy: results of 30 adrenalectomies in 27 patients. World J Surg.
morphometric analysis in patients with familial medullary thyroid carci- 1996;20(7):769-774.
noma. Am J Pathol. 1976;83(1):177-196. 30. Castinetti F, Taieb D, Henry JF, et al. Management of endocrine disease:
17. Mathew CG, Chin KS, Easton DF, et al. A linked genetic marker for outcome of adrenal sparing surgery in heritable pheochromocytoma. Eur J
multiple endocrine neoplasia type 2A on chromosome 10. Nature. Endocrinol. 2016;174(1):R9-R18.
1987;328(6130):527-528. 31. Wells SA Jr, Gosnell JE, Gagel RF, et al. Vandetanib for the treatment of
18. Simpson NE, Kidd KK, Goodfellow PJ, et al. Assignment of multiple patients with locally advanced or metastatic hereditary medullary thyroid
endocrine neoplasia type 2A to chromosome 10 by linkage. Nature. cancer. J Clin Oncol. 2010;28(5):767-772.
1987;328(6130):528-530. 32. Elisei R, Schlumberger MJ, Muller SP, et al. Cabozantinib in progressive
19. Donis-Keller H, Dou S, Chi D, et al. Mutations in the RET proto-oncogene medullary thyroid cancer. J Clin Oncol. 2013;31(29):3639-3646.

The Endocrine Society. Downloaded from press.endocrine.org by [${individualUser.displayName}] on 12 January 2017. at 12:19 For personal use only. No other uses without permission. . All rights reserved.
NEUROENDOCRINOLOGY
AND PITUITARY

The Endocrine Society. Downloaded from press.endocrine.org by [${individualUser.displayName}] on 12 January 2017. at 12:19 For personal use only. No other uses without permission. . All rights reserved.
178 ENDO 2016 MEET-THE-PROFESSOR CLINICAL CASE MANAGEMENT

Acromegaly: Navigating the Difficult Cases

CMF6 the mechanisms and underpinning the more common presenta-


Presented, April 1 4, 2016 tion of acromegaly; older patients with a milder phenotype,
slower-growing tumors that are treatment sensitive.

Katja Kiseljak-Vassiliades, DO; Peter Trainer, MD,


SIGNIFICANCE OF THE CLINICAL PROBLEM
FRCP. University of Colorado Aurora CO, 80045, USA,
Acromegaly, a disorder of abnormal skeletal and visceral tissue
E-mail: katja.kiseljak-vassiliades@ucdenver.edu; and the
growth, in the vast majority of cases, is caused by a GH pituitary
Christie NHS Foundation Trust, Manchester M20 4BX,
adenoma with an incidence of five per million (5, 6). Approxi-
United Kingdom, E-mail: peter.trainer@manchester.ac.uk
mately 65% of the GH tumors present as macroadenomas (tu-
mors 1 cm) that often lead to local compression of adjacent
INTRODUCTION sellar structures, which can lead to visual field loss and pitu-
Historical Overview itary hormone deficiencies (7). Excess GH causes the clinical
The history of gigantism is wrapped up in myths and legends presentation of acromegaly including coarsening of facial fea-
dating back to Goliath, but it was not until 1886 that Pierre tures, frontal bossing, jaw prognathism, overbite; enlargement
Marie (Paris neurologist, 18531940) first used the term acro- of nose and ears; and skeletal and soft tissues overgrowth
megaly (large extremities). He did, however, not appreciate characterized by increase in shoe, ring size, and skin thicken-
the significance of the pituitary in the etiology and nor did the ing. Because the process is of gradual onset, the diagnosis is
John Hunter (Scottish anatomist, 17271793) who obtained the often delayed by an average of 10 years (6). Persistently el-
skeleton of the Irish Giant, Charles Byrne (17611783). It evated serum GH levels due to GH tumors result in a 10-year
was Harvey Cushing who, in 1909, opened the skull of the reduction in life expectancy due to cardiovascular (biventricular
Irish Giant to demonstrate the enlarged pituitary fossa (1). hyperthrophy, arrhythmias, diastolic dysfunction leading to heart
Sadly, the landmark event in 1916, the year that the Endo- failure), vascular (HTN), respiratory (sleep apnea), metabolic
crine Society was created, was the death of Baptiste Hugo (230 (obesity, lipid abnormalities including increased triglycerides,
cm, 1876 1916), the older of the Hugo brothers (les Geants Lpa and ApoB), and endocrine complications (diabetes melli-
des Alpes), his younger brother (Antoine Hugo, 225 cm, tus and hypogonadism) (6, 8, 9).
18871914) having died two years earlier. The Hugo brothers
are not the only case of familial acromegaly, with other famous
examples including the Knipe twins who were Irish contempo- BARRIER TO OPTIMAL PRACTICE
raries of Charles Byrne. Hilary Mantel wrote a brilliant, fic- Early diagnosis is clearly desirable, but as alluded to above, 10
tionalized novel (The Giant, OBrien) of the circus freak show years routinely elapses between the onset of symptoms and the
life of 18th century giants, that has in many ways changed diagnosis being made and there is a dearth of evidence that
little. A unifying theme among the giants of legend was that progress is being made in speeding up the time to diagnosis.
they died young, which we now know is a consequence of Biochemical confirmation of the diagnosis is rarely a problem;
aggressive pituitary tumors and the metabolic effect of GH the challenge is increasing awareness of acromegaly among the
hypersecretion and hypopituitarism. many specialties that patients may present to.
Ninety years after Baptistes death, Vierimaa and colleagues Given that uncontrolled acromegaly is associated with in-
(2) identified mutations in the AIP gene as the likely explana- creased cardiovascular, respiratory, and metabolic morbidity,
tion of familial gigantism, such as the Hugo brothers. DNA and decreased life span, the treatment goal is biochemical
extracted from one of Charles Byrnes teeth confirmed that he normalization of GH levels and IGF-I adjusted for age and sex.
is a member of an Irish kindred with a R304X mutation in the However, the reference range of normal GH level was estab-
AIP gene that subsequently has been found in more than 70 lished using retrospectively collected data generated with less-
descendants with acromegaly and gigantism (3). sensitive assays that are in routine use nowadays. In addition,
Additional insight into the molecular mechanisms causing there is a wide variety of platforms used for IGF-I assay. There
gigantism has come from the recent report of, so-called, X- are variations in the molecular heterogeneity of the analytes
linked acrogigantism caused by an Xq26.3 genomic duplica- preparations, antibody epitope specificity, as well as technique
tion, probably of GPR101, with disease onset being in early differences to reduce interference from other binding proteins.
childhood (4). These lead to poor comparability of assay results between
There remains much to be learned, and in particular, while various laboratories, which further affects guidance for diagno-
AIP and X-linked acrogigantism may be important in the sis and treatment.
pathogenesis of aggressive tumors presenting in childhood and Recent consensus on the medical treatment of acromegaly
early adult life, there remains the challenge of understanding recommends surgical resection as first-line therapy for GH

The Endocrine Society. Downloaded from press.endocrine.org by [${individualUser.displayName}] on 12 January 2017. at 12:19 For personal use only. No other uses without permission. . All rights reserved.
ENDO 2016 NEUROENDOCRINOLOGY AND PITUITARY 179

tumors (10, 11), but choice of surgeon is critical given that hallmark characteristics of acromegaly include acral and facial
remission rates are higher and complication rates lower with changes (98%), diaphoresis (64%), headaches (55%), carpal
experienced surgeons with high throughput. Even the most tunnel syndrome (40%), sexual dysfunction, hypertension (28%),
expensive surgeon is cost effective compared with the expense sleep apnea, skin tags, and arthritis (19). Given that acromegaly is
associated with long-term medical therapy in the patient in a gradually progressive disease, the diagnosis is often delayed by
whom the chance of cure has been missed. 10 years or more, leading to presence of HTN, sleep apnea, and
All the same, given that GH tumors are often large and diabetes mellitus complications frequently present at the time of
invasive, less than 50% of patients achieve long-term bio- diagnosis. The treatment of the disease is therefore focused on the
chemical control postoperatively with surgery alone (7). In biochemical control and the reversal of comorbidities.
patients with persistent elevation of IGF-I and GH following
the surgical resection, therapy with somatostatin analogs (SSA) Diagnostic Approach
is indicated as first-line medical treatment (10-14). In the recent Significant diagnostic advancement has been made in the field
systemic review of the response rate of SSA treatment in of pituitary GH tumors in the last two decades including:
acromegaly (9 studies, N 354) the reported efficacy rates of Development of assays with increased sensitivity for GH
SSA is 31%, much lower than historically reported response and IGF-I (20).
rates (15). Dopamine agonists have a limited role in patients Widespread use of diagnostic magnetic resonance
with mildly elevated GH levels and IGF-I less than 2.5 ULN imaging (MRI) images for pituitary visualization (21).
(upper limit of normal) and likelihood of biochemical disease Novel software to assess the physical changes associated
remission is low (6, 10, 11). In addition, recent experience with with disease, although this is likely too early for
pegvisomant, GH receptor antagonist, suggests IGF-I level widespread use (22-24).
normalization in approximately 70% of patients, at least some Questionaires to assess the quality of life mostly
of which may have been attributed to inadequate dosing of the developed for disease monitoring (25).
medication (16). In patients with uncontrolled disease follow-
ing surgical and medical therapy, radiotherapy is required, but It is clinical presentation that often leads a provider to suspect
often involves a long latency period to achieve remission (50% acromegaly and obtaining screening laboratories to include GH
at 10 y) and carries a risk for cerebrovascular death (14, 17). and IGF-I levels.
In summary, up to 40% of patients with acromegaly patients
have uncontrolled, progressive disease despite surgery and GH Assay
medical therapy (18), and better clinical and biochemical mark- It is important to understand normal GH production and
ers are needed for prognostic purposes as well as to guide physiology to be able to interpret GH levels on a patient
novel therapies in the era of precision medicine. with potential GH over-rsecretion. In a normal pituitary
gland GH is pulsatile and there might be up to 5-6 physi-
LEARNING OBJECTIVES ologic pulses of GH present in a normal individual whose
As a result of participating in this session, learners should be values might overlap with the values seen in acromegalic
able to: patient. The optimal way to assess GH secretion would be
Identify patients with acromegaly whose presentation to obtain daily production by 24-hour sampling, which
and characteristics might point to a hereditary would not be practical for the patient or physician. In addi-
component of the disease. tion, although the development of new IRMA (Im-
Understand how to manage these patients as well as munoradiometric assay) and ILMA (mmunoluminometric
provide genetics counseling to the family. assay) assays have improved the state of the field there is
Understand the present GH and IGF-I assay variability, still significant variability between different labs attributed
and how to approach patient management in the setting to use of different antibodies and their abilities to recognize
of assay differences. different GH isoforms, GH-binding protein issues, separa-
Understand how to use histologic biomarkers to help tion techniques, and calibration issues.
with prognosis as well as guide medical treatment in a GH suppression testing with oral glucose load has been used
patient. as confirmatory for the diagnosis of acromegaly (26) with a
Understand rare but significant addictive behavior nadir GH of more than 0.4 ug/L supportive of diagnosis in
component related to treatment with dopamine agonists. conjunction of clinical symptoms and elevated IGF-I. GH sup-
pression testing is of limited value in high catabolic states such
STRATEGIES FOR MANAGEMENT OF as stress, liver and renal impairment, obesity, pregnancy, dia-
ACROMEGALY betes mellitus, and adolescents and patients on estrogen re-
Clinical Features placement (27). In patients on medical therapy for acromegaly
Chronic GH oversecretion leads to abnormal growth of organs, GH suppression testing is not useful for followup due to incon-
skeletal and connective as well as soft tissue swelling. The sistent results (10).

The Endocrine Society. Downloaded from press.endocrine.org by [${individualUser.displayName}] on 12 January 2017. at 12:19 For personal use only. No other uses without permission. . All rights reserved.
180 ENDO 2016 MEET-THE-PROFESSOR CLINICAL CASE MANAGEMENT

IGF-I Assay Control the symptoms of disease and reduce/reverse the


Compared with GH, IGF-I has a long half-life (18-20 h) and is associated comorbidities.
stable throughout the day, so it can be used as a screening tool Biochemical normalization of GH and IGF-I levels
as well as the marker of disease activity (26). Although there adjusted for age and sex.
are significant advantages of using IGF-I compared with GH Partial or complete removal the pituitary tumor to
levels, a clinician needs to be aware that multiple physiologic alleviate compression on the adjacent structures while
factors can affect IGF-I levels (27): restoring visual fields and normal pituitary function.
False-negative IGF-I: exogenous estrogen, malnutrition,
liver and renal failure. Surgery
False-positive IGF-I: pregnancy and adolescence. Transspheinoidal approach surgical resection is the first-line
In addition, problems with assay can arise from interference treatment for GH tumors (10, 11). The success of surgery is
from binding protein and the tendency for IGF-I levels to correlated with the access to an experienced neurosurgeon and
plateau at GH mean level greater than 20 ug/L (28). presence of multidisciplinary team. In the hands of a skilled
neurosurgeon the surgical cure is achieved in approximately
70% of patients (100% for microadenomas, and 61% for
Imaging
Once the biochemical diagnosis of acromegaly has been made, macroadenomas) (44). It has also been shown that regardless of
contrast-enhanced MRI of the sella turica is indicated to visualize tumor size DG adenomas respond to surgery better than SG
the pituitary tumor and determine the extent of the sellar structural tumors (68 vs 14%) (39).
involvement. The GH tumor presents as macroadenoma in ap-
proximately 65% of the cases (7). The tumor is usually hypointense Medical Therapy
on T1-weighed MRI images. Computed tomography may be Somatostatin analogs are first-line medical therapy in patients
used if MRI images are not available or contraindicated. who do not achieve remission with surgical resection (10, 11).
Patients with DG tumors and those with high SSTR2 expres-
sion on immunohistochemistry are more likely to respond to
Pathology
Using electron microscopy (EM) imaging GH tumors were treatment with SSA levels (39, 42, 43). Patients with SG
initially subdivided into two subtypes with distinct ultrastruc- tumors are less likely to respond to SSA (39). Dopamine
ture features; densely (DG) and sparsely (SG) granulated ad- agonists can be considered in patients with mild elevation of
enomas (29, 30). DG tumors show well-developed organelles IGF-I levels, and therefore have limited use in acromegaly.
and abundant secretory granules on EM. SG tumors, In con- High-dose dopamine agonists for treatment of Parkinsons dis-
trast, contain sparse neurosecretory granules. With the ad- ease have been associated with valvular dysfunction, which has
vancement of pathological techniques, immunohistochemistry not been seen with doses used for acromegaly (45). Dopamine
for cytokeratin (CAM5.2) has been widely used as a surrogate agonists have, however, been associated with addictive behav-
for EM (31). Although the cytoplasmic keratin filaments pres- ior (see Case 4).
ent as fibrous bodies in SG adenomas, in DG tumors cytoker- GH receptor antagonist pegvisomant is used in patients who
atin staining has diffuse perinuclear pattern (32). The prognos- do not respond (defined as normalization of GH and IGF-I
tic behavior of GH secreting adenomas based on tumor subtype level adjusted for age and sex) to somatostatin analogs (11).
has been suggested (33-35). SG GH tumors often are more The effectiveness of pegvisomant normalizing IGF-I levels has
common in the young (50 y), with larger tumors, and lower been well established (46).
GH/IGF-I levels than DG GH tumors (33, 36-39). SG tumors
are thought to be less responsive to SSAs than DG GH tumors, Radiotherapy
but the mechanisms underlying the differences in tumor behav- Radiation therapy is generally considered as third-line treat-
ior has not been elucidated (34, 39-41). ment in patients who do not have tumor growth control or
Immunohistochemistry for somatostatin receptors (SSTR15) normalization with surgery and medical therapy. Radiation
has also been evaluated in several studies, as main targets of therapy might be useful in patients receiving GH receptor
first-line medical therapy with SSAs (somatostatin analogues). antagonist who show evidence of tumor expansion (47).
Most studies agree that increased SSTR2 expression on IHC
correlates with positive response to treatment with SSA in MAIN CONCLUSIONS
vivo, although the resistance to SSA cannot be always ex- Acromegaly is a slow, progressive, and under-recognized dis-
plained by SSTR2 expression levels (42, 43). ease associated with comorbidities, and if uncontrolled, with
decreased life-span. Although significant advancement in diag-
Treatment nostic techniques have been made in last two decades, a clini-
The goal of treatment for acromegaly is focused on several cian needs to be aware and familiar with current method-
aspects: specific assays, hormone standards, and assays sensitivity and

The Endocrine Society. Downloaded from press.endocrine.org by [${individualUser.displayName}] on 12 January 2017. at 12:19 For personal use only. No other uses without permission. . All rights reserved.
ENDO 2016 NEUROENDOCRINOLOGY AND PITUITARY 181

specificity. Pituitary tumor resection by an experienced skilled 181 ng/mL (45-173) and prolactin level is 9 ng/mL (3-20).
surgeon provides the only chance for cure, and referral to a What is the most appropriate management?
center of excellence should be highly considered. Patients with A. Obtaining oral glucose GH suppression test
persistent disease require treatment with medical therapy. Tu- B. Adding cabergoline to the regiment
mor size and histological markers might be useful prognostica- C. Repeating IGF-I with a different assay after
tors that might help direct treatment modalities. investigating assay methodology
D. Adding pegvisomant to the regiment
CASES WITH QUESTIONS
Case 1a Case 3
In 2003, the patient, age 20 years, presented with a 1 year of The patient is a 43-year old man who was initially complaining
headache. His tall stature was noted and a head scan identified of fatigue and on workup was found to have low T. His
a pituitary macroadenoma and biochemistry confirmed acro- hypogonadism was evaluated further and deemed to be a cen-
megaly. He underwent transsphenoidal surgery (TSS) on two tral process given that LH and FSH were low. Additional labs
occasions 10 months apart but had residual tumor (1.3 0.8 revealed IGF-I, 984 ng/mL (109-284); prolactin 5, ng/mL; and
cm) abutting his left cavernous sinus, and there was biochemi- GH, 8.0 ng/mL (0-2.9). The patient also underwent oral glu-
cal evidence of persisting disease activity and hypopituitarism. cose loading, which was abnormal and consistent with acro-
He was commenced on SSA therapy with minimal benefit (IGF-I, megaly. The patient underwent MRI imaging, and although it
was initially read as normal, on further review there was
857 ng/mL; RR (reference range), 116-368). The addition of
increased amount of tissue anteriorly in the gland and on the
pegvisomant, ultimately 20 mg per day, resulted in normalization
left side of the sella. The patient underwent TSS and pathology
of IGF-I and for the last 11 years tumor volume has remained
showed GH adenoma demonstrating numerous fibrous bodies
stable.
on CAM5.2 staining consistent with SG tumor. Three months
following the surgery patients IGF-I level was 275 ng/mL
Case 1b
(121-237) and GH 1.29 ng/mL (0.05-3.0). IGF-I level was
In 1983, a patient, age 18 years and great than 213 cm tall,
repeated 1 month later and was 325 ng/mL (125-333), and GH,
acromegaly was diagnosed and TSS undertaken and followed
1.45 ng/mL (0.01-3.0). At 1-year postop, IGF-I, 275 ng/mL
by three-field, multifractional pituitary radiotherapy. Thirty
(121-237) and in conjunction with abnormal GH suppression
years later the patient is GH deficient and has cardiomyopathy
testing patient was started on lanreotide, 90 mg monthly and
and atrial fibrillation.
then up titrated to 120 mg monthly. His IGF-I level on this
What additional testing should be performed in these patients?
dose is 276 ng/mL (121-237) and GH, 1.78 (0.05-3.0).
A. AIP mutation testing
B. PIT1 mutation testing
What is the most appropriate management?
C. MEN1 testing
A. Obtaining MRI imaging
D. X-linked acrogigantism
B. Oral load glucose suppression testing
E. G protein alpha subunit mutation testing C. Adding cabergoline to the regiment
D. Switching to pegvisomant
Case 2
The patient is a 59-year-old woman with a history of thyroid Case 4
cancer, s/p thyroidectomy, and central neck dissection 2 years In 2006, the patient, age 27, presented to a dentist with facial
ago, who noted swelling of her hands, difficulty with closing pain and acromegaly was suspected and confirmed by nonsup-
her fists, as well as paresthesias in her hands. Her laboratory pression of GH during an oral glucose tolerance test, an el-
work evaluation revealed IGF-I, 679 ng/mL (45-173), and evated IGF-I, and pituitary macroadenoma. Prolactin was el-
subsequent oral glucose load revealed a baseline GH of 5.52 evated at 1484 mU/L (RR 400). The patient was commenced
ng/mL (0.01-3.661), a 1-hour level of 4.77 ng/mL, and a on bromocriptine preoperatively and 1 year later underwent
2-hour level of 7.02 ng/mL. An MRI of the pituitary was TSS (2007) and due to persisting disease activity transcranial
obtained on and revealed a 0.6 0.9 0.7 cm right pituitary surgery (2008) followed by postoperative three-field, multifractional
mass, and another smaller hypoenhancing structure within the radiotherapy (2008). Ongoing bromocriptine therapy was supple-
left pituitary measuring approximately 0.6 0.5 0.56 mm mented with octreotide LAR without disease control being
that might be contiguous with the larger right-sided lesion. The achieved.
patient has undergone TSS and pathology was consistent with In 2012 he was jailed for defrauding family members of
mixed prolactin-GH adenoma consistent with prolactin DG GH approximately $200 000 to pay off gambling debts.
adenoma. At 3-month followup the patients IGF-I was 443 On direct questioning he gave a history of hypersexuality.
ng/mL (53-287) and she was started on lanreotide 120 mg SQ On release from prison, his IGF-I was 678 ng/mL (RR,
once monthly. Three months later the patients IGF-I level is 78-232), bromocriptine was discontinued and pegvisomant was

The Endocrine Society. Downloaded from press.endocrine.org by [${individualUser.displayName}] on 12 January 2017. at 12:19 For personal use only. No other uses without permission. . All rights reserved.
182 ENDO 2016 MEET-THE-PROFESSOR CLINICAL CASE MANAGEMENT

commenced and in February 2015 he underwent had Gamma Oral glucose load GH testing in patients on medical therapy for
Knife therapy. His most recent IGF-I is 298 ng/mL. acromegaly is not useful due to inconsistent results (10).
Which dopamine agonists are associated with addictive
behavior? Case 3
A. Bromocriptine This case involves a patient with aggressive GH adenoma,
B. Quinigolide which on immunohistochemistry has been shown to have nu-
C. Cabergoline merous fibrous bodies consistent with SG tumor. Although the
D. Pergolide patient had a 6-month postoperative period of apparent remis-
E. All of the above sion after surgery his IGF-I increased again and proved to be
unresponsive to somatostatin analogs as the recent literature
DISCUSSION OF CASES AN ANSWERS would suggest (34, 39, 42, 43). At this point the patient was
Case 1 switched to pegvisomant, and clinically he had a very nice
Studies by Marta Korbonits (m.korbonits@qmul.ac.uk) demon- response to pegvisomant with normalization of IGF-I. Obtain-
strated the same mutation in the AIP gene in both patients. ing MRI on the patient would likely be less useful, and recent
Their presentations are typical of patients with AIP mutations: guidelines recommend MRI imaging at 3-month postop and
they both presented as young adults with gigantism and pitu- serial MRIs in patient receiving pegvisomant (11). Adding
itary macroadenomas that could not be fully resected at sur- dopamine agonist to patients regiment is unlikely to normalize
gery. The first patient was treated with somatostatin analogs IGF-I levels. Recent guidelines suggest that either SSA or
but with minimal benefit (the second patient was diagnosed pegvisomant can be used as initial adjuvant medical therapy,
before somatostatin analogs were available). The latter pa- and with recent literature suggesting that SG tumors usually do
tients daughter is an asymptomatic carrier. not respond to SSA one might consider pegvisomant as first-
They are both members of nine Caucasian origin, not- line therapy in a patient with GH tumor showing aggressive
knowingly related c.805_825dup-positive pedigrees (four characteristics on imaging and immunohistochemistry.
FIPA, five sporadic; five from UK, three from US, 1 from
France) that include 16 affected (nine gigantism/four Case 4
acromegaly/two nonfunctioning pituitary adenoma patients Numerous small series have reported dopamine agonistinduced
with macroadenomas and one prospectively diagnosed addictive behavior in patients, most commonly in patients with
microadenomaacromegaly) and nine unaffected carriers. All Parkinsons disease but also in those with acromegaly (48-50).
pedigrees shared a 2.79 Mbp haploblock around AIP, indicat- Pathological gambling and hypersexuality may be class effects of
ing the existence of a common ancestor. The c.805_825dup dopamine agonists. Compulsive eating, compulsive shopping, and
allele associates a severe clinical phenotype, with high fre- excessive alcohol consumption have also been reported. Patients
quency of gigantism, especially in males. All known commencing dopamine agonists should be warned of the risk of
c.805_825dup-positive pedigrees share a common ancestor, addictive behavior.
the English founder, who lived approximately 1175 years
ago (assuming 25 y per generation). An estimated 96 carriers REFERENCES
may exist at present, with 25 identified so far (Salvatori et al, 1. de Herder WW. Acromegaly and gigantism in the medical literature. Case
article submitted). descriptions in the era before and the early years after the initial publica-
tion of Pierre Marie (1886). Pituitary. 2009;12(3):236-244.
2. Vierimaa O, Georgitsi M, Lehtonen R, et al. Pituitary adenoma predisposi-
Case 2 tion caused by germline mutations in the AIP gene. Science. 2006;312(5777):
This case involves a patient with GH tumor who still has 1228-1230.
3. Chahal HS, Stals K, Unterlander M, et al. AIP mutation in pituitary
persistent disease after surgical resection. The patient is now on adenomas in the 18th century and today. N Engl J Med. 2011;364(1):
maximum dose of SSA with IGF-I level just barely above 43-50.
upper limit of normal range. This IGF-I test was performed 4. Trivellin G, Daly AF, Faucz FR, et al. Gigantism and acromegaly due to
Xq26 microduplications and GPR101 mutation. N Engl J Med.
through a commercial laboratory using blocking RIA (Radio-
2014;371(25):2363-2374.
immunoassay) (RAI) after acid:alcohol extraction; with signifi- 5. Asa SL, Ezzat S. The pathogenesis of pituitary tumors. Annu Rev Pathol.
cant proportion of IGF-I values recently resulting higher than 2009;4:97-126.
expected. The testing of IGF-I level was repeated at least twice 6. Melmed S. Acromegaly pathogenesis and treatment. J Clin Invest.
2009;119(11):3189-3202.
subsequently using a different commercial laboratory using 7. Shimon I, Cohen ZR, Ram Z, Hadani M. Transsphenoidal surgery for
quantitative chemiluminescent immunoassay with all the val- acromegaly: Endocrinological follow-up of 98 patients. Neurosurgery.
ues in this patient falling within normal range. The patient has 2001;48(6):1239-1243; discussion 1244-1245.
8. Coculescu M, Niculescu D, Lichiardopol R, Purice M. Insulin resistance
been therefore continued on lanreotide and is clinically doing
and insulin secretion in non-diabetic acromegalic patients. Exp Clin
well. Adding second-line therapy in this patient, such as Endocrinol Diabetes. 2007;115(5):308-316.
cabergoline or pegvisomant, would therefore not be necessary. 9. Boero L, Manavela M, Gomez Rosso L, et al. Alterations in biomarkers of

The Endocrine Society. Downloaded from press.endocrine.org by [${individualUser.displayName}] on 12 January 2017. at 12:19 For personal use only. No other uses without permission. . All rights reserved.
ENDO 2016 NEUROENDOCRINOLOGY AND PITUITARY 183

cardiovascular disease (CVD) in active acromegaly. Clin Endocrinol 31. Yamada S, Aiba T, Sano T, et al. Growth hormone-producing pituitary
(Oxf). 2009;70(1):88-95. adenomas: Correlations between clinical characteristics and morphology.
10. Giustina A, Chanson P, Kleinberg D, et al. Expert consensus document: A Neurosurgery. 1993;33(1):20-27.
consensus on the medical treatment of acromegaly. Nat Rev Endocrinol. 32. Asa SL. Tumors of the pitutary gland. Washington, DC: Armed Forces
2014;10(4):243-248. Institute of Pathology, 1998.
11. Katznelson L, Laws ER Jr, et al. Acromegaly: An endocrine society clinical 33. Besser GM, Burman P, Daly AF. Predictors and rates of treatment-
practice guideline. J Clin Endocrinol Metab. 2014;99(11):3933-3951. resistant tumor growth in acromegaly. Eur J Endocrinol. 2005;153(2):
12. Stevenaert A, Beckers A. Presurgical Octreotide: Treatment in acro- 187-193.
megaly. Metabolism. 1996;45(8 Suppl 1):72-74. 34. Bhayana S, Booth GL, Asa SL, Kovacs K, Ezzat S. The implication of
13. Clemmons DR, Chihara K, Freda PU, et al. Optimizing control of acro- somatotroph adenoma phenotype to somatostatin analog responsiveness in
megaly: Integrating a growth hormone receptor antagonist into the treat- acromegaly. J Clin Endocrinol Metab. 2005;90(11):6290-6295.
ment algorithm. J Clin Endocrinol Metab. 2003;88(10):4759-4767. 35. Ezzat S, Kontogeorgos G, Redelmeier DA, Horvath E, Harris AG, Kovacs
14. Carmichael JD, Bonert VS, Mirocha JM, Melmed S. The utility of oral K. In vivo responsiveness of morphological variants of growth hormone-
glucose tolerance testing for diagnosis and assessment of treatment out- producing pituitary adenomas to octreotide. Eur J Endocrinol. 1995;133(6):
comes in 166 patients with acromegaly. J Clin Endocrinol Metab. 686-690.
2009;94(2):523-527. 36. Obari A, Sano T, Ohyama K, et al. Clinicopathological features of growth
15. Colao A, Auriemma RS, Lombardi G, Pivonello R. Resistance to soma- hormone-producing pituitary adenomas: Difference among various types
tostatin analogs in acromegaly. Endocr Rev. 2011;32(2):247-271. defined by cytokeratin distribution pattern including a transitional form.
16. Trainer PJ, Ezzat S, DSouza GA, Layton G, Strasburger CJ. A random- Endocr Pathol. 2008;19(2):82-91.
ized, controlled, multicentre trial comparing pegvisomant alone with com- 37. Bando H, Sano T, Ohshima T, et al. Differences in pathological findings
bination therapy of pegvisomant and long-acting octreotide in patients and growth hormone responses in patients with growth hormone-
with acromegaly. Clin Endocrinol (Oxf). 2009;71(4):549-557. producing pituitary adenoma. Endocrinol Jpn. 1992;39(4):355-363.
17. Sherlock M, Reulen RC, Alonso AA, et al. ACTH deficiency, higher doses 38. Bakhtiar Y, Hirano H, Arita K, et al. Relationship between cytokeratin
of hydrocortisone replacement, and radiotherapy are independent predic- staining patterns and clinico-pathological features in somatotropinomae.
tors of mortality in patients with acromegaly. J Clin Endocrinol Metab. Eur J Endocrinol. 2010;163(4):531-539.
2009;94(11):4216-4223. 39. Kiseljak-Vassiliades K, Carlson NE, Borges MT, et al. Growth hormone
18. Freda PU. Somatostatin analogs in acromegaly. J Clin Endocrinol Metab. tumor histological subtypes predict response to surgical and medical
2002;87(7):3013-3018. therapy. Endocrine. 2015;49(1):231-241.
19. Molitch ME. Clinical manifestations of acromegaly. Endocrinol Metab 40. Brzana J, Yedinak CG, Gultekin SH, Delashaw JB, Fleseriu M. Growth
Clin North Am. 1992;21(3):597-614. hormone granulation pattern and somatostatin receptor subtype 2A corre-
20. Clemmons DR. Consensus statement on the standardization and evaluation late with postoperative somatostatin receptor ligand response in acro-
of growth hormone and insulin-like growth factor assays. Clin Chem. megaly: A large single center experience. Pituitary. 2013;16(4):490-498.
2011;57(4):555-559. 41. Fougner SL, Casar-Borota O, Heck A, Berg JP, Bollerslev J. Adenoma
21. Freda PU, Beckers AM, Katznelson L, et al. Pituitary incidentaloma: An granulation pattern correlates with clinical variables and effect of soma-
endocrine society clinical practice guideline. J Clin Endocrinol Metab. tostatin analogue treatment in a large series of patients with acromegaly.
2011;96(4):894-904. Clin Endocrinol (Oxf). 2012;76(1):96-102.
22. Schneider HJ, Kosilek RP, Gunther M, et al. A novel approach to the 42. Taboada GF, Luque RM, Neto LV, Machado Ede O, Sbaffi BC,
detection of acromegaly: Accuracy of diagnosis by automatic face classi- Domingues RC, et al. Quantitative analysis of somatostatin receptor sub-
fication. J Clin Endocrinol Metab. 2011;96(7):2074-2080. types (1-5) gene expression levels in somatotropinomas and correlation to
23. Miller RE, Learned-Miller EG, Trainer P, Paisley A, Blanz V. Early in vivo hormonal and tumor volume responses to treatment with octreotide
diagnosis of acromegaly: Computers vs clinicians. Clin Endocrinol (Oxf). LAR. Eur J Endocrinol. 2008;158(3):295-303.
2011;75(2):226-231. 43. Casarini AP, Jallad RS, Pinto EM, et al. Acromegaly: Correlation between
24. Giustina A, Bevan JS, Bronstein MD, et al. SAGIT: Clinician-reported expression of somatostatin receptor subtypes and response to octreotide-lar
outcome instrument for managing acromegaly in clinical practice- treatment. Pituitary. 2009;12(4):297-303.
development and results from a pilot study. Pituitary. 2015 [Epub ahead of 44. Jane JA Jr, Starke RM, Elzoghby MA, et al. Endoscopic transsphenoidal
print]. surgery for acromegaly: Remission using modern criteria, complications, and
25. Webb SM, Prieto L, Badia X, Albareda M, Catala M, Gaztambide S, et al. predictors of outcome. J Clin Endocrinol Metab. 2011;96(9):2732-2740.
Acromegaly Quality of Life Questionnaire (ACROQOL) a new health- 45. Maione L, Garcia C, Bouchachi A, et al. No evidence of a detrimental
related quality of life questionnaire for patients with acromegaly: Develop- effect of cabergoline therapy on cardiac valves in patients with acro-
ment and psychometric properties. Clin Endocrinol (Oxf). 2002;57(2): megaly. J Clin Endocrinol Metab. 2012;97(9):E1714-E1719.
251-258. 46. van der Lely AJ, Biller BM, Brue T, et al. Long-term safety of pegvisomant in
26. Giustina A, Barkan A, Casanueva FF, et al. Criteria for cure of acromegaly: A patients with acromegaly: Comprehensive review of 1288 subjects in
consensus statement. J Clin Endocrinol Metab. 2000;85(2):526-529. ACROSTUDY. J Clin Endocrinol Metab. 2012;97(5):1589-1597.
27. Cordero RA, Barkan AL. Current diagnosis of acromegaly. Rev Endocr 47. Melmed S, Colao A, Barkan A, et al. Guidelines for acromegaly manage-
Metab Disord. 2008;9(1):13-19. ment: An update. J Clin Endocrinol Metab. 2009;94(5):1509-1517.
28. Barkan AL, Beitins IZ, Kelch RP. Plasma insulin-like growth factor- 48. Klos KJ, Bower JH, Josephs KA, Matsumoto JY, Ahlskog JE. Pathological
I/somatomedin-C in acromegaly: Correlation with the degree of growth hypersexuality predominantly linked to adjuvant dopamine agonist therapy
hormone hypersecretion. J Clin Endocrinol Metab. 1988;67(1):69-73. in Parkinsons disease and multiple system atrophy. Parkinsonism Relat
29. Kovacs K, Horvath E, Corenblum B, Sirek AM, Penz G, Ezrin C. Pituitary Disord. 2005;11(6):381-386.
chromophobe adenomas consisting of prolactin cells: A histologic, im- 49. McKeon A, Josephs KA, Klos KJ, et al. Unusual compulsive behaviors
munocytological and electron microscopic study. Virchows Arch A Pathol primarily related to dopamine agonist therapy in Parkinsons disease and
Anat Histol. 1975;366(2):113-123. multiple system atrophy. Parkinsonism Relat Disord. 2007;13(8):516-519.
30. Horvath E, Kovacs K. Ultrastructural classification of pituitary adenomas. 50. MHRA Public Assessment Report Dopamine Agonists. Pathological gam-
Can J Neurol Sci. 1976;3(1):9-21. bling and increased libido. 2007.

The Endocrine Society. Downloaded from press.endocrine.org by [${individualUser.displayName}] on 12 January 2017. at 12:19 For personal use only. No other uses without permission. . All rights reserved.
184 ENDO 2016 MEET-THE-PROFESSOR CLINICAL CASE MANAGEMENT

Molecular Imaging and Radionuclide Therapy of Functional


Neuroendocrine Tumors (NETS)

CMF8 tumor heterogeneity in metastatic disease limits the accu-


Presented, April 1 4, 2016 racy of a single random biopsy to prognosticate and direct
systemic therapy. Molecular imaging and conventional
CT/MRI is necessary for confident localization to guide
David Pattison, MBBS, MPH, FRACP, FAANMS. surgical excision, accurate staging to prevent morbidity as-
Peter MacCallum Cancer Centre, East Melbourne, sociated with futile surgery in unresectable disease, and in
Australia. E-mail: david.pattison@petermac.org and vivo disease characterization to guide the choice of systemic
treatment (including radionuclide therapy) and assess its
Thomas ODorisio, MD. University of Iowa, Iowa City, response.
IA 52242 E-mail: thomas-odorisio@uiowa.edu

BARRIERS TO OPTIMAL PRACTICE


HISTORICAL OVERVIEW The rarity and protean symptomatology of NETs is a signifi-
Neuroendocrine tumors (NETs) were first described by Sieg- cant barrier to diagnosis in primary care, where a lack of
fried Oberndorfer in 1907, when he described tumors of the awareness of the condition may delay diagnosis such that 50%
small intestine as karzinoide (carinoid or carcinoma like). Sub- of patients present with metastatic disease. Subsequent man-
sequent work by Pearse in the 1960s to define NETs as amine agement is best provided in large specialist cancer centers due
precursor uptake decarboxylase (APUD) tumors and the acci- to the need for multidisciplinary care, and lack of access for
dental discovery of somatostatin as a peptide inhibiting the geographic or demographic reasons is a significant barrier to
release of GH in rats in 1973 underpin the targets commonly optimal practice. The rarity and heterogeneity of tumor biology
used for molecular imaging and radionuclide therapy today.
has also limited the capacity to conduct large clinical trials in
The radiolabeled somatostatin analog 111In-DTPA-octreotide
this field. Furthermore, different regulatory barriers to the use
was developed by Krennings group at Erasmus Medical Cen-
of emerging technologies has limited the access to new mo-
tre in the Netherlands for imaging NETs in 1989 (1) and
lecular imaging techniques and radionuclide therapies in some
peptide receptor radionuclide therapy (PRRT) in the early
regions of the world.
1990s. Subsequent innovation has included the use of higher
resolution PET/CT imaging using 68Ga-based somatostatin
analogs and therapeutic application of -emitting 177Lu- and LEARNING OBJECTIVES
90
Y-analogs for PRRT. As a result of participating in this session, learners should be
able to:
1. Understand the principles behind molecular imaging
SIGNIFICANCE OF THE CLINICAL PROBLEM
of NETs using 68Ga-DOTATATE and 18F-fluorode-
This session will focus upon NETs arising from the diges-
oxyglycose (FDG) PET/CT, including 1) indications
tive tract, gastroenteropancreatic (GEP) NETs, and chromaf-
for these scans and 2) interpretation of results to
fin cells, including phaeochromocytoma and paraganglioma,
guide personalized therapy.
whereas other primary sites, including the lung, thyroid
2. Understand the principles of treating NETs with PRRT,
(medullary thyroid carcinoma), and pituitary, will not be
including patient selection, contraindications, potential
discussed further. The 5-fold increase in incidence of NETs
complications, and expected clinical outcomes.
over the last 3 decades (5.25/100 000 persons/y) likely rep-
3. Recognize the important role of the endocrinologist
resents an increase in diagnosis by access to more sensitive
within the NET multidisciplinary team.
blood tests, improved imaging techniques and more wide-
spread use of endoscopy and endoscopic ultrasound. NETs
often cause significant impairment of quality of life (QOL) STRATEGIES FOR DIAGNOSIS, THERAPY,
due to secretion of various hormonal syndromes, which AND/OR MANAGEMENT
reflect the function of the (neuro)endocrine cell of origin, in Diagnosis
addition to direct oncologic effects. NETs exist on a spec- The diagnosis of NETs is based upon clinical symptomatology,
trum from well to poorly differentiated phenotypes, classi- biochemical evidence of disease (including hormonal secre-
fied according to the European Neuroendocrine Tumor So- tion), and histopathologic confirmation of malignancy. The
ciety and World Health Organization grading system into 3 carcinoid syndrome is classically characterized by cutaneous
categories based on expression of the Ki-67 antigen with flushing, secretory diarrhea, and, in longstanding cases, cardiac
cut-offs at 2% and 20% (2). However, the presence of valvular lesions caused by secretion of serotonin (quantified by

The Endocrine Society. Downloaded from press.endocrine.org by [${individualUser.displayName}] on 12 January 2017. at 12:19 For personal use only. No other uses without permission. . All rights reserved.
ENDO 2016 NEUROENDOCRINOLOGY AND PITUITARY 185

24-h urinary collection of its metabolite 5HIAA) and other precursors to produce hormones stored in secretory granules
vasoactive peptides into the systemic circulation from meta- within tumor cells) and the expression of peptide hormone
static small bowel carcinoids. Pancreatic NETs may be non- receptors (including somatostatin and glucagon-like peptide
functional, or characterized by hormonal syndromes such as [GLP]-1 receptors), in addition to nonspecific glycolytic me-
insulinoma, gastrinoma, VIPoma, glucagonoma, ectopic ACTH tabolism via 18F-FDG (see Figure 1). Molecular imaging roles
secretion, or somatostatinoma. Hindgut tumors are typically include localization, staging, restaging, and theranostics. A
nonfunctional. Phaeochromocytoma and sympathetic paragan- theranostic system integrates a diagnostic imaging test, which
gliomas (PPGLs) are classically associated with episodic can be performed before and after a therapy (using the same
symptoms, including headache, palpitations, sweating, and hy- compound) to determine suitability for treatment and monitor
pertension, and should be initially evaluated with measure- its effects.
ment of plasma-free metanephrines or urinary fractionated
metanephrines. Over one third of PPGLs are associated with Somatostatin Receptor (SSTR) Imaging
a germline mutation, which are classified in 2 categories: Radiolabeling of octreotide with the -emitter 111In has been
cluster 1 demonstrates increased expression of genes asso- largely superseded by the positron emitter 68Ga labeled to
ciated with angiogenesis and hypoxia (pseudohypoxic clus- octreotate, which provide the advantages of the greater spatial
resolution of PET technology, lower radiation dose, greater
ter) comprising succinate dehydrogenase (SDH) complex
patient convenience, and overall lower cost. Three compounds
units, assembly factor 2 and von-Hippel-Lindau. Cluster 2
are now in use, including [68Ga-DOTA0-Ty3]octreotate
includes rearranged during transfection, neurofibromatosis
(DOTA-TATE), [68Ga-DOTA0-Ty3]octreotide (DOTA-TOC),
1, myc-associated factor X, and transmembrane protein 127
and [68Ga-DOTA0-NaI3]octreotide (DOTA-NOC). These have
susceptibility genes involved in kinase signaling pathways.
varying SSTR subtype specificity with highest affinity
for SSTR2, SSTR5, and SSTR3/5, respectively. 68Ga-
MOLECULAR IMAGING MODALITIES DOTATATE is the preferred tracer at Peter MacCallum Cancer
NET cells may be specifically targeted by the APUD mecha- Centre given that SSTR2 is the predominant subtype
nism (the ability to concentrate and take up amino acids and overexpressed in more than 90% of GEP NETs (3). The USA

FIGURE 1. Schematic representation of mechanisms for molecular imaging and radionuclide therapy.

The Endocrine Society. Downloaded from press.endocrine.org by [${individualUser.displayName}] on 12 January 2017. at 12:19 For personal use only. No other uses without permission. . All rights reserved.
186 ENDO 2016 MEET-THE-PROFESSOR CLINICAL CASE MANAGEMENT

68
TABLE 1. Suggested Clinical Indications for Ga-DOTATATE/DOTATOC PET/CT
Staging Prior to resection of apparently localized disease
Localization Primary site in patients with biochemical suspicion of NET
Unknown primary with metastatic NET
Theranostic SSTR density & distribution to guide suitability for SSA therapy or PRRT
Restaging Therapeutic response assessment
Suspected disease recurrence post-surgery (e.g. rising tumor markers)

use both the DOTATATE and DOTATOC carriers for the most amenable biopsy, it is increasingly recognized this
68Ga-PET scan. cannot be considered representative given tumor heteroge-
Numerous studies have demonstrated superior accuracy of neity with different imaging and histopathological pheno-
68
Ga-DOTATATE/DOTATOC compared with either conven- types possible at different sites (9). Confirmatory biopsy of
tional radiologic imaging or 111In-octreotide scintigraphy for an unusually FDG avid site may significantly alter the tumor
evaluation of NET, with a pooled sensitivity of 93% and grade and thus treatment approach. FDG PET/CT is also
specificity of 96% in a recent large meta-analysis (4). More useful for response assessment to therapy and exclusion of
importantly, 68Ga-DOTATATE demonstrates high manage- discordant FDG avid disease while assessing suitability for
ment impact with intermodality change in 47% of patients PRRT. The Peter MacCallum Cancer Centre reserves use of
(such as directing curative surgery from localization of primary FDG PET/CT for patients meeting the criteria listed in Table
site or directing patients with unresectable metastatic disease to 2. Similarly, the University of Iowa NET center routinely
systemic therapy) and intramodality change in an additional offers 18F-FDG PET/CT for all grade 2 or 3 tumors with
10% (5). A summary of suggested clinical indications for negative SST2A receptor staining of the primary or meta-
SSTR PET/CT is provided in Table 1, with similar criteria used static tumor.
at the University of Iowa NET center in the IMPACT trial.
18
F-Fluorodopa (FDOPA)
18 18
F-Fluorodeoxyglycose F-FDOPA is a radiolabeled analog for L-DOPA, a dopamine
18
F-FDG is a glucose analog taken up by cancer cells precursor, which enters cells via large amino acid transporter 2
via facilitated transport by glucose transporters (especially before undergoing decarboxylation and storage in secretory
GLUT-1) and after phosphorylation by glucose-6-phosphatase vesicles. Its role is not yet clearly defined, although it is a
(also known as hexokinase) remains trapped in the cell. sensitive investigation for staging of medullary thyroid carci-
18
F-FDG uptake is considered representative of the tumors noma and appears useful for imaging PPGL and localization of
glycolytic metabolism and is higher in poorly differentiated focal congenital hyperinsulinism. However, it is not routinely
cells dependent on inefficient aerobic glycolysis (the Warburg used at our institution due to lack of a theranostic target for
effect) to fuel their metabolic needs. The pseudohypoxic clus- radionuclide therapy and time-consuming synthesis process.
ter of PPGL (especially SDHx disease) are an important excep-
tion, which may exhibit very intense FDG uptake despite an 123
I/124I Metaiodobenzylguanidine
entirely benign phenotype (6) via constitutive activation of MIBG is a noradrenaline analog which enters cells via the
hypoxia-inducible factors with resultant inhibition of oxidative norepinephrine transporter and is stored in neurosecretory
phosphorylation and induction of similar glycolytic pathways. granules via vesicular monoamine transporter (1 or 2). Al-
Historically, the utility of FDG PET/CT was underesti- though MIBG is traditionally radiolabeled with 123I for planar
mated as a diagnostic test for NETs given their often low scintigraphy or SPECT/CT imaging of PPGL, 124I PET/CT
intrinsic metabolic activity and lack of tumor specificity, provides better spatial resolution and precise quantitative do-
deemed only of use if other molecular imaging agents, eg, simetry for planning radionuclide therapy with its theranostic
SSTR or metaiodobenzylguanidine (MIBG), were negative partner 131I-MIBG. Although often used for localization of
(7). However, 18F-FDG plays a powerful role in disease
characterization as treatment selection is increasingly
guided by the tumor grade (defined by Ki-67 index) and the TABLE 2. Suggested Indications for FDG PET/CT in
distribution of disease (eg, resectable vs PRRT vs liver NETs
directed vs systemic). Numerous studies demonstrate that Patients with Ki,67 index greater than or equal to 5%
FDG PET/CT represents the single most powerful prognos- Patients with clinical or imaging findings of progressive disease
tic tool in patients with metastatic NET, exceeding tradi- within a 6-month period
tional markers such as Ki-67, chromogranin A (CgA), and Patients with sites of disease identified on CT that do not have
uptake on SSTR PET/CT and are of concern as sites of poorly
anatomic stage (8). Although tumor grade is often based
differentiated disease
upon a single random histopathologic assessment of a site

The Endocrine Society. Downloaded from press.endocrine.org by [${individualUser.displayName}] on 12 January 2017. at 12:19 For personal use only. No other uses without permission. . All rights reserved.
ENDO 2016 NEUROENDOCRINOLOGY AND PITUITARY 187

pheochromocytoma/paraganglioma, it has relatively low sensi- control arm. Other series have described favorable results treat-
tivity for detection of distant metastatic disease, which limits ing patients with unresectable metastatic PPGL and bronchial
its utility for radionuclide therapy. In our experience, the com- carcinoid tumors.
parative intensity and distribution of uptake favors treatment Functional NETs require effective control of the associated
with 177Lu-DOTATATE in metastatic disease, which fortu- hormonal syndrome and symptoms (eg, malignant insulinoma)
itously also does not require hospitalization for radiation safety in addition to the underlying oncologic disease and often pres-
precautions. 123I-MIBG uptake is also present in up to 52% of ent a management challenge to endocrinologists. It is critical to
other (non-PPGL) NETs (11). clarify treatment goals in the management of unresectable
functional NETs which are often characterized by severe hor-
GLP-1 Receptor Imaging monal syndromes impacting on QOL but relatively indolent
The GLP-1 analog exendin-4 can be radiolabeled with either (non-FDG avid) tumor behavior refractory to oncologic thera-
111
In (SPECT/CT) or 68Ga (PET/CT) for localization of pies. In such cases, symptomatic treatments of carcinoid syn-
insulinoma. A recent prospective multicenter study evaluating drome (including somatostatin analog therapy, antidiarrheal
111
In-exendin-4 SPECT/CT demonstrated a sensitivity of 95% agents, and investigational drugs such as the peripheral trypto-
in 24 patients, performing substantially better than combined phan hydroxylase inhibitor telotristat etiprate) are initially pref-
CT and MRI (45%). An early small series of patients with erable to often poorly tolerated oncologic therapy. In addition
68
Ga-exendin-4 PET/CT appears superior to SPECT/CT as to delivering effective oncologic control, PRRT is also a highly
expected. GLP-1 receptor imaging is not yet developed in the effective treatment for refractory hormonal syndromes includ-
United States. ing carcinoid syndrome (14), and functional pancreatic NETs
including metastatic insulinoma (15) with minimal toxicity.
TREATMENT
There is an increasing variety of treatments for NETs, includ- Radionuclide Therapy
177
ing PRRT, pharmacological, hormonal, and liver-directed Lu- and 90Y-DOTATATE are the 2 most commonly used
therapies, and surgical approaches. Pharmacologic options in- radionuclides for SSTR-targeted PRRT. After binding to the
clude the mTOR inhibitor everolimus, the tyrosine kinase in- SSTR2 receptor, their complex is internalized and emission of
hibitor sunitinib, and a variety of combination chemotherapy high-energy -particles damages DNA in adjacent cells. The
regimens, including capecitabine/temozolomide (CAPTEM). longer path length of 90Y (5 mm vs 1 mm for 177Lu) provides
Our practice has favored treatment with PRRT for suitable greater dosimetry in large lesions more than 4 cm, whereas
177
patients (see Table 3) given the markedly greater progression- Lu is preferentially used for lesions below this threshold.
free survival (PFS) in large prospective series (40 48 mo) (12, Radionuclide therapy is typically administered in 4 cycles
13) compared with other available therapies (eg, 11 mo for approximately 8 weeks apart and is coadministered with a
everolimus and sunitinib) and high symptomatic and imaging renoprotective amino acid infusion to reduce the renal radiation
response rates in patients previously uncontrolled by conven- dose. Radiosensitizing chemotherapy may also be used.
tional therapies. The recently presented interim results of Patients are selected for treatment with PRRT according to
NETTER-1 study, a phase III randomized controlled trial com- the suggested criteria in Table 3. In particular, treatment is
paring octreotide LAR 30 mg 177Lu-DOTATATE PRRT vs based upon demonstration of tracer uptake greater than liver
octreotide LAR 60 mg in patients with progressive disease, has uptake at all sites of disease that are not below the spatial
not yet reached a median PFS after more than 30-month fol- resolution of the imaging technique used and therefore subject
low-up in the PRRT arm compared with only 8.4 months in the to partial volume effects. Confirmed progressive disease within

TABLE 3. PRRT treatment Criteria and Contraindications at Peter MacCallum Cancer Centre
Criteria for treatment with PRRT at Peter MacCallum Cancer Centre:
- Unresectable locally advanced or metastatic neuroendocrine tumor (NET); and
- Somatostatin receptor (SSTR) scan uptake liver (i.e. Krenning score 3 or 4); and
- No evidence of macroscopic SSTR ve/FDG ve disease; and
- Either:
o uncontrolled symptoms due to hormone secretion/tumour burden; or
o radiologic, scintigraphic or biochemical evidence of progression in prior 12 months; or
o G2 or G3 disease with significant tumor burden or impaired performance status
Contraindications:
- Hypoalbuminaemia (25g/liter), GFR 30 ml/min, platelet count 50 000 or pancytopaenia
- ECOG Performance score 4 or expected survival 3 months.

The Endocrine Society. Downloaded from press.endocrine.org by [${individualUser.displayName}] on 12 January 2017. at 12:19 For personal use only. No other uses without permission. . All rights reserved.
188 ENDO 2016 MEET-THE-PROFESSOR CLINICAL CASE MANAGEMENT

12 months or refractory hormonal symptoms despite maximal icterus and peripheral edema. Initial biochemical tests included
medical therapy is a requirement for treatment at our institu- hypokalaemia (K 2.1 mmol/L, N 3.55.0) and cholestatic
tion. Comparison with FDG PET/CT is performed for specific jaundice (ALP 407 U/L, N 40 100; GGT 1854 U/L, N 732;
indications (see Table 2) to exclude spatially discordant dis- Bili 35 mol/L; N 35). CT abdomen and pelvis identified a
ease, which would require a different treatment modality. 5-cm pancreatic mass, regional nodal disease in porta hepatis,
PRRT with 177Lu-DOTATATE is comparatively well tol- and extensive bilobar hepatic metastases. Biopsy of liver lesion
erated with rare complications in large prospective series of confirmed a diagnosis of metastatic pancreatic NET (ENETs
patients treated with long-term follow-up. Abdominal dis- G2, Ki-67 10%). Additional biochemistry demonstrated
comfort or vomiting within 24 hours occurs in approxi- 24-hour urinary-free cortisol: 49 489 nmol/24 hours (N
mately 10% of patients and subacute hematological toxicity 350); ACTH, 57 pmol/L (N 20); gastrin, more than 447
(WHO grade 3 or 4) is seen at 4 8 weeks after 3.6% of pmol/L (N 55); and CgA, 630 g/L (N 17). Ga68 DOTA-
administrations (12). Serious hepatic toxicity may rarely octreotate PET/CT demonstrated intense tracer uptake (Kren-
occur in the context of diffuse hepatic disease (4% of pa- ning score 4; ie, uptake spleen) in all sites of disease with
tients), and our institution has a threshold of albumin more concordant moderate FDG uptake on FDG PET/CT.
than 25 g/L to ensure adequate hepatic reserve. Prolonged
follow-up of 802 patients treated with either 90Y than 177Lu- Question: What Is the Most Appropriate Management
DOTATATE showed hematologic and nephrotoxicity is After Immediate Correction of Hypokalaemia?
more common with 90Y and myelodysplastic syndrome is a A. Sunitinib steroidogenesis inhibitor
rare (2.3%) but significant delayed complication (16). The B. Everolimus steroidogenesis inhibitor
risk of transient worsening of hormonal symptoms after C. PRRT steroidogenesis inhibitor
treatment of functional NETs with PRRT is well recognized. D. Debulking surgery
In a series of 479 patients treated with PRRT, 1% required E. Bilateral adenalectomy
prolonged hospitalization for severe symptoms (17).
Pretherapy stabilization of hormonal excretion is recom- Case 2
mended, somatostatin analog therapy should be reinstituted A 66-year-old man presented to our NET clinic to see our
soon after treatment and close observation (including hospi- endocrine surgeon, James R. Howe, and myself. He relates a
talization) is required for those at high risk. Particularly care 1-year history of night sweats, and a 6-month history of loose
is required in cases of VIPoma, insulinoma, and phae- stools (3 4/d, described as greasy and foul-smelling) and
ochromocytoma, in which life-threatening hormonal release persistent facial flushing and reddening on the palms of his
syndromes can occur. These patients should ideally be ad- hands. Before visit, as part of a chronic obstructive pulmonary
mitted to hospital, at least for the first cycle of treatment. disease trial, in which he was participating, a CT of chest noted
liver nodules. Core biopsy revealed metastatic low-grade neu-
roendocrine carcinoma (WHO, grade 2, Ki-67, 5%). He had
MAIN CONCLUSIONS not been on any somatostatin analogs before his visit. Exami-
NETs are a heterogeneous group of tumors that may impair
nation revealed an up-beat well-nourished man with persistent
QOL by secretion of hormonal syndromes in addition to direct
crimson facial appearance and deeply reddened palms. Electro-
oncologic effects. Molecular imaging is necessary for confident
lytes were normal as were creatinine and BUN. Hemogram was
localization to guide surgical excision, accurate staging to pre-
completely normal. Except for slightly elevated total bilirubin
vent morbidity associated with futile surgery in unresectable
(1.4, NL 1.2, possibly core biopsy related), liver function
disease, and in vivo disease characterization to guide the choice
studies were normal. Initial biomarkers returned: serotonin,
of systemic treatment (including radionuclide therapy) and as-
960 (N 200); CgA, 168 (N 95); and pancreastatin, 899
sess its response. Surgery is the preferred treatment for resect-
(N 135). Initial MR was negative for primary tumor of the
able disease and clarification of treatment goals is important in
abdomen, and initial octreoscan was read as negative for pri-
patients with metastatic functional NETs to minimize potential
mary small intestinal tumor; however, the Ga68-DOTATOC
toxicity. PRRT is a safe and effective treatment for patients
PET scan localized a single primary small bowel lesion.
with GEP-NETs, providing both symptomatic relief of hor-
Octreoscan addendum read suggests primary small bowel tu-
monal syndromes and recent phase III trial evidence of pro-
mor, but first report said primary was probably lung.
longed oncologic disease control.

Question: After Initiation of Somatostatin Analogs,


CASES WITH QUESTIONS What Is the Most Appropriate Therapeutic Option?
Case 1 A. Locate the primary tumor and exploratory surgery if
A 61-year-old woman presented with subacute painless jaun- possible
dice, weight loss, and peripheral edema in May 2012. Exami- B. Hepatic arterial embolization (HAE)
nation revealed a slender tanned Caucasian woman with scleral C. Antiangiogenic therapy (EG everolimus)

The Endocrine Society. Downloaded from press.endocrine.org by [${individualUser.displayName}] on 12 January 2017. at 12:19 For personal use only. No other uses without permission. . All rights reserved.
ENDO 2016 NEUROENDOCRINOLOGY AND PITUITARY 189

D. PPRNT with either LU177-DOTATATE or NET with this rare combination of gastrin and ectopic ACTH
Y90-DOTATOC secretion (18).
E. Adjust somatostatin analogues and follow with serial She received a total of 5 cycles of PRRT with 177Lu-DOTA-
CT/MR and biomarkers. octreotate with an ongoing complete metabolic response (FDG)
and near complete resolution of disease on 68Ga-DOTA-
Case 3 octreotate PET/CT and normalization of biochemistry (24-h
A 46-year-old man who presented with a painful mandibular UFC 90, ACTH 4.6) now approximately 3 years after com-
metastasis and biopsy was consistent with NET. Staging FDG mencement of therapy.
PET/CT demonstrated very intense widespread activation of
brown adipose tissue and very intense FDG uptake (SUVmax Case 2
19) in the mandible metastasis and a 8-cm para-aortic mass. This case points up almost classic presentation of metastatic
The patient had a fasting BSL of 8.1 mmol/L at time of PET midgut carcinoid due to the pathohumoral action of serotonin
scan and a history of hypertension. Given the highly unusual excess when sustained; namely facial flushing, nocturnal per-
brown fat distribution and exceptionally intense FDG avidity of spiration, and hypermotile-fat malabsorptive loose stools. The
the disease, the reporting nuclear medicine physician suggested other 2 forms of diarrhea, which can be associated with high
this may represent a metastatic SDHB-associated paragan- sustained blood levels of serotonin, are secretory diarrhea and
glioma from an organ of Zuckerkandl primary. Plasma diarrhea from postobstructive bowel. Regarding the therapeutic
normetadrenaline was subsequently tested at 18000 pmol/L options available, numerous retrospective studies suggest that
(N 900), and immunohistochemical stains confirmed this removal of the primary NET even in the face of metastasis
diagnosis. favorably improves the time to progression and overall sur-
vival. This should be considered the first best option of therapy
if possible, particularly if the patient is symptomatic from the
Question: What Is the Best Staging Investigation in
primary site. Continued use and appropriate adjustment of the
This Case?
A. Repeat 18F-FDG PET/CT with propranolol to suppress somatostatin analogs with 4- to 6-month CT/MR scans and
brown fat uptake regular biomarker monitoring (especially if initial biomarkers
B. 123I-MIBG SPECT/CT are elevated), helps to guide the most optimal therapies and
C. CT and MRI may help to change therapies that include HAE or even bridg-
D. 68Ga-DOTA-octreotate PET/CT ing therapy with antiangiogenic drugs such as everolimus or
E. 18F-FDOPA PET/CT chemotherapy drugs such as CAPTEM, both of which are out
of indication for midgut carcinoids.
DISCUSSION OF CASES AND ANSWERS
Case 1 Case 3
This patient presents with a functional metastatic pancreatic This uncommon presentation highlights several important as-
pects of FDG PET/CT imaging in pheochromocytoma and
NET secreting ectopic ACTH and gastrin, with secondary hy-
paraganglioma (PPGL). The extensive brown fat activation in a
peraldosteronism due to mineralocorticoid effect of high circu-
perinephric distribution despite high ambient temperature was
lating cortisol levels. Clearly control of both the hormonal
an important diagnostic clue. Although brown fat activation is
syndrome and underlying oncologic disease is required in this
well known to be stimulated by cold exposure and inversely
case. Options for hormonal control include steroidogenesis
related to body mass index, knowledge of its association with
inhibitors, glucocorticoid receptor antagonists, or bilateral ad-
PPGL is not widespread despite being first reported in 1957
renalectomy. In this case, metyrapone was commenced due to
(19). The usual administration of -blockers to suppress BAT
efficacy, rapid onset of action, side effect profile, and clinician uptake for repeat FDG imaging is not recommended, because
preference. High-dose proton pump inhibitor therapy was com- it may precipitate hypertensive crisis due to unopposed
menced for hypergastrinaemia. Traditionally bilateral adrena- -adrenergic activation. The second important clue to the diag-
lectomy was often performed due to difficulty achieving du- nosis was very intense FDG uptake (SUVmax 19) associated
rable control of the underlying disease. Noncurative surgery is with the tumor, which is indicative of a pseudohypoxic met-
not recommended given the unresectable disease and effective abolic signature, which can be observed with imaging of
systemic therapies. Sunitinib and everolimus both have evi- cluster 1 PPGL. Incidental fasting hyperglycaemia and history
dence for a modest increase in PFS in metastatic pancreatic of hypertension were additional clues to the diagnosis.
NETs (11 mo vs 5 mo on placebo). However, PRRT is An important recent study by Janssen et al (20) prospec-
preferred in this case given the intensity/distribution of SSTR tively evaluated these imaging modalities in 17 patients with a
expression and the substantially longer PFS in large prospec- total of 289 lesions. 68Ga-DOTATATE PET/CT demonstrated
tive series (40 58 mo). There are also case reports of particu- lesion based detection rate of 98.6%, substantially better than
larly durable treatment response in previous cases of pancreatic any of the other comparators (CT and MRI, 18F-FDG, 123I-

The Endocrine Society. Downloaded from press.endocrine.org by [${individualUser.displayName}] on 12 January 2017. at 12:19 For personal use only. No other uses without permission. . All rights reserved.
190 ENDO 2016 MEET-THE-PROFESSOR CLINICAL CASE MANAGEMENT

MIBG, and 18F-FDOPA). In this case, it identified an addi- tomography in targeted radionuclide therapy for endocrine malignancies.
PET Clin. 2015;10:461-476.
tional 2 sites of bony disease and critically it demonstrated
11. Binderup T, Knigge U, Loft A, et al. Functional imaging of neuroendo-
suitability for treatment with PRRT, which was administered crine tumors: a head-to-head comparison of somatostatin receptor scintig-
with excellent long-term clinical, scintigraphic, and biochemi- raphy, 123I-MIBG scintigraphy, and 18F-FDG PET. J Nucl Med.
cal response. 2010;51:704-712.
12. Kwekkeboom DJ, de Herder WW, Kam BL, et al. Treatment with the
radiolabeled somatostatin analog [ 177Lu-DOTA 0,Tyr 3]octreotate: tox-
REFERENCES icity, efficacy, and survival. J Clin Oncol. 2008;26:2124-2130.
1. Lamberts SW, Bakker WH, Reubi JC, Krenning EP. Somatostatin-receptor 13. Kashyap R, Hofman MS, Michael M, et al. Favourable outcomes of
imaging in the localization of endocrine tumors. N Engl J Med. (177)Lu-octreotate peptide receptor chemoradionuclide therapy in patients
1990;323(18):1246-1249. with FDG-avid neuroendocrine tumours. Eur J Nucl Med Mol Imaging.
2. Rindi G, Capella C, Solcia E. Introduction to a revised clinicopathological 2015;42(2):176-185.
classification of neuroendocrine tumors of the gastroenteropancreatic tract. 14. Khan S, Krenning EP, van Essen M, Kam BL, Teunissen JJ, Kwekkeboom
Q J Nucl Med. 2000;44(1):13-21. DJ. Quality of life in 265 patients with gastroenteropancreatic or bronchial
3. Reubi JC, Schar JC, Waser B, Wenger S, Heppeler A, Schmitt JS. Affinity neuroendocrine tumors treated with [177Lu-DOTA0,Tyr3]octreotate.
profiles for human somatostatin receptor subtypes SST1-SST5 of soma- J Nucl Med. 2011;52:1361-1368.
tostatin radiotracers selected for scintigraphic and radiotherapeutic use. 15. van Schaik E, van Vliet, Feelders RA, et al. Improved control of severe
Eur J Nucl Med. 2000;27(3):273-282. hypoglycemia in patients with malignant insulinomas by peptide receptor
4. Geijer H, Breimer LH. Somatostatin receptor PET/CT in neuroendocrine radionuclide therapy. J Clin Endocrinol Metab. 2011;96(11):3381-3389.
tumours: update on systematic review and meta-analysis. Eur J Nucl Med 16. Bodei L, Kidd M, Paganelli G, et al. Long-term tolerability of PRRT in
Mol Imaging. 2013;40(11):1770-1780. 807 patients with neuroendocrine tumours: the value and limitations of
5. Hofman MS, Kong G, Neels OC, Eu P, Hong E, Hicks RJ. High manage- clinical factors. Eur J Nucl Med Mol Imaging. 2015;42(1):5-19.
ment impact of Ga-68 DOTATATE (GaTate) PET/CT for imaging neu- 17. de Keizer B, van Aken MO, Feelders RA, et al. Hormonal crises
roendocrine and other somatostatin expressing tumours. J Med Imaging following receptor radionuclide therapy with the radiolabeled soma-
Radiat Oncol. 2012;56:40-47. tostatin analogue [177Lu-DOTA0,Tyr3]octreotate. Eur J Nucl Med Mol
6. Taieb D, Sebag F, Barlier A, et al. 18F-FDG avidity of phae- Imaging. 2008;35:749-755.
ochromocytomas and paragangliomas: a new molecular imaging signa- 18. Davi MV, Bodei L, Ferdeghini M, et al. Multidisciplinary approach in-
ture? J Nucl Med. 2009;50:711-717. cluding receptor radionuclide therapy with 90Y-DOTATOC ([90Y-
7. Adams S, Baum R, Rink T, et al. Limited value of fluorine-18 DOTA0, Tyr3]-octreotide) and 177Lu-DOTATATE ([177Lu-DOTA0,
fluorodeoxyglucose positron emission tomography for the imaging of Tyr3]-octreotate) in ectopic Cushing syndrome from a metastatic
neuroendocrine tumours. Eur J Nucl Med. 1998;25:79-83. gastrinoma: a promising proposal. Endocr Pract. 2008;14(2):213-218.
8. Binderup T, Knigge U, Loft A, et al. 18F-fluorodeoxyglucose positron 19. Melicow MM. Hibernating fat and pheochromocytoma. AMA Arch Pathol.
tomography predicts survival of patients with neuroendocrine tumours. 1957;63:367-372.
Clin Cancer Res. 2010;16:978-985. 20. Janssen I, Blanchet EM, Adams K, et al. Superiority of [68Ga]-
9. Hofman MS, Hicks RJ. Changing paradigms with molecular imaging of DOTATATE PET/CT to other functional imaging modalities in the local-
neuroendocrine tumours. Discov Med. 2012;14(74):71-81. ization of SDHB-associated metastatic pheochromocytoma and paragan-
10. Pattison DA, Hofman MS. Role of fluorodeoxyglucose PET/computed glioma. Clin Cancer Res. 2015;21(17):3888-3895.

The Endocrine Society. Downloaded from press.endocrine.org by [${individualUser.displayName}] on 12 January 2017. at 12:19 For personal use only. No other uses without permission. . All rights reserved.
ENDO 2016 NEUROENDOCRINOLOGY AND PITUITARY 191

ACTH-Dependent Cushings Syndrome: Challenging Cases

CMF11 LEARNING OBJECTIVES


Presented, April 1 4, 2016 As a result of participating in this session, learners should be
able to:
Understand the diagnostic tests, and potential pitfalls, for
Lynnette K. Nieman, MD; Janice M. Kerr, MD. Program establishing Cushings disease (CD).
in Reproductive and Adult Endocrinology, Eunice Individualize treatment options for patients with CS.
Kennedy Shriver National Institute of Child Health and Understand the radiographic approach to localizing
Human Development, National Institutes of Health, ectopic ACTH tumors.
Bethesda, Maryland 20892, E-mail: niemanl@mail.nih.
gov; Department of Endocrinology, Metabolism and STRATEGIES FOR DIAGNOSIS AND
Diabetes, University of Colorado Anschutz Medical MANAGEMENT
Campus, Aurora, Colorado 80045, E-mail: Janice.kerr@ The Endocrine Society guidelines recommend that the first step
ucdenver.edu in the diagnosis of CS is to exclude exogenous glucocorticoid
use (10). Having done that, endogenous hypercortisolism is
evaluated by measurement of a 24-hour urine cortisol, late night
INTRODUCTION serum or salivary cortisol and/or cortisol after dexamethasone, 1
Historical Overview mg the evening before. Once endogenous hypercortisolism is
In a 1910 monograph, Harvey Cushing presented the case of confirmed, the initial step in the differential diagnosis is mea-
Minnie G, who presented with syndrome of painful obesity, surement of plasma ACTH with a high-performance assay
hypertrichosis, and amenorrhea, which he characterized as a (11). Normal or high values suggest an ACTH-dependent
pluriglandular syndrome related to a pituitary abnormality (1). cause, either pituitary adenoma (so-called CD) or ectopic
In 1932, he published an additional 11 cases, most collected ACTH secretion from another tumor. Low or undetectable
from the literature, and ascribed them to a basophilic tumor of ACTH levels indicate an ACTH-independent cause, and imag-
the pituitary gland (2). Around that time, Walters et al (3) ing of the adrenal glands is indicated to identify the site(s) of
reported on the suprarenal cortical syndrome in 10 cases, abnormal tissue. Further testing to differentiate the causes of
some of whom had adrenal adenoma or carcinoma; others had ACTH-dependent CS should be performed during consistent
suspected pituitary abnormalities. The differential diagnosis of hypercortisolism to ensure that the normal pituitary corticotropes are
the syndrome was enlarged by the description of ectopic fully suppressed, as IPSS and other tests do not distinguish
ACTH secretion in 1962 (4), and the later identification of between normal individuals and those with CD (12). An ACTH
other adrenal causes (eg, primary pigmented nodular gradient between a peripheral vein and the petrosal sinus,
during IPSS, is characteristic of CD, if values of more than 2
adrenocorticol disease, primary macronodular adrenal hyper-
before CRH administration, or more than 3 after CRH are
plasia) (5, 6). The development of RIA for ACTH in the 1960s
achieved. IPSS has high diagnostic accuracy (95%) (9).
allowed for differential diagnosis (7), as did development of
However, lack of suppression of normal corticotropes or ve-
the CRH stimulation test (8) and inferior petrosal sinus sam-
nous abnormalities may lead to falsely positive and negative
pling (IPSS) in the 1980s (9). results, respectively (13, 14). Measurement of prolactin to
normalize the petrosal sinus ACTH values may help identify
SIGNIFICANCE OF THE CLINICAL PROBLEM inadequate cannulation (15). A response to high-dose (8 mg)
Cushings syndrome (CS), although rare, is potentially lethal if dexamethasone (DST) or CRH may also help to identify CD,
not treated. Deaths are due to venous thromboembolism (myo- albeit with less diagnostic accuracy than IPSS. Magnetic
cardial infarction [MI], stroke, pulmonary embolism) and in- resonance imaging (MRI) of the pituitary is generally per-
fections. Comorbidities reduce quality of life. Because treat- formed before IPSS, given that a large mass (6 mm) is
ment improves these conditions, it is important to efficiently virtually diagnostic of CD and obviates the need for invasive
sampling (16).
make the diagnosis and normalize signs and symptoms.
Management of CS comorbidities should be initiated while
the diagnostic workup is ongoing; the goal is normalization
BARRIERS TO OPTIMAL PRACTICE (17). According to Endocrine Society guidelines on the treat-
Gaps in knowledge related to interpretation of diagnostic ment of CS, resection of the tumor(s)/hyperplastic tissue ac-
tests for CS and individualization of management cording to the specific cause is optimal management. Anatomi-
prevent optimal practice. cal and/or functional imaging with MRI, computed tomography

The Endocrine Society. Downloaded from press.endocrine.org by [${individualUser.displayName}] on 12 January 2017. at 12:19 For personal use only. No other uses without permission. . All rights reserved.
192 ENDO 2016 MEET-THE-PROFESSOR CLINICAL CASE MANAGEMENT

(CT), MIBG or somatostatin analog nuclear medicine studies, TABLE 2.


are often needed to identify an ectopic source of ACTH (18). Basal Cortisol
Given that these tumors are usually located in the chest, initial IPSS ACTH, pg/mL Level, mcg/dL
imaging of the thorax has the best diagnostic yield. When Time (min before/after CRH) 0 3 5
tumor resection is not possible, is not successful, or there is a R petrosal 11 14 12 8
recurrence, second-line therapy should be individualized. Re- L petrosal 12 12 18 7
currence in CD is more likely with cavernous sinus invasion, Peripheral 8 9 11 9
large tumors, and a short period of dependence on glucocorti-
coid replacement (19). In patients with recurrent or persistent
CD, and who are not candidates for repeat transsphenoidal C. ACTH assay variability or sample degradation.
resection, the choices for second-line treatment include: medi- D. An ectopic CRH-producing tumor.
cal therapy (steroidogenesis inhibitors (20), glucocorticoid an- Correct answer: D.
tagonist, corticotrope-directed treatment with cabergoline or The indeterminate plasma ACTH levels has created a diagnos-
pasireotide), radiation therapy/radiosurgery, and bilateral adre- tic quandary, between ACTH-dependent and ACTH-independent
nalectomy. hypercortisolism; a dilemma not seen with ectopic CRH-
producing tumors.
MAIN CONCLUSIONS 2. You next recommend all of the following now except:
The choice of tests for the diagnostic evaluation of CS A. Review the IPSS venogram (if available), or check
should be individualized to the patient. petrosal prolactin levels.
Treatment options for recurrent/persistent CD include B. Re-evaluate for persistent hypercortisolism with re-
surgery, radiation and medical therapies, and require an peat screening tests.
individualized approach. C. Review medications/consider synthetic glucocorti-
coid screen.
D. Repeat the IPSS with a different ACTH assay.
CASES AND DISCUSSION Correct answer: D.
Case 1
Although current ACTH assays have limited sensitivity at the
A contrast-enhanced, 3T pituitary MRI showed no abnormali-
lower diagnostic range, repeating IPSS with a different ACTH
ties. IPSS 1 month later revealed (Table 2):
assay is unlikely to be helpful.
A 56-year-old Caucasian female reported a 15-pound cen-
The patients venogram was normal. She continued to deny
tral weight gain, fatigue, and subjective muscle weakness
exogenous steroid exposure. Additional testing revealed mildly
for the past 6 months. Her medical history was significant
elevated urinary free cortisol levels (UFC, 75 84 mcg/d); mid-
for scleroderma, hypothyroidism, hypertension, and type II
night salivary cortisol levels, 1.8 2.1-fold elevated; and low
diabetes mellitus (for 7 y). She denied any recent steroid
ACTH levels (1520 pg/mL by ARUP/Immunolite 2000 assay).
exposure. Physical examination revealed an overweight
3. You consider all of the following except:
woman (body mass index, 27 kg/m2) with minimal clinical
A. An adrenal CT scan.
features of CS. She had mild moon facies and central obe-
B. A dehydroepiandrosterone sulfate (DHEA-S) level or
sity, but otherwise no facial plethora, atrophic skin
CRH stimulation test.
change/ecchymoses, violaceous striae, or muscle weakness.
C. Exploratory transsphenoidal surgery (TSS).
The initial endocrine labs showed (Table 1).
D. A synthetic glucocorticoid screening test.
1. The history, laboratory results, and IPSS results are most
Correct answer: C.
consistent with all of the following except:
TSS is not recommended in the absence of a central ACTH
A. Cyclic CS or intermittent exogenous steroid expo-
gradient.
sure.
Review of a recent abdominal CT scan, for a diverticulitis
B. Failed petrosal sinus cannulation.
flare, showed no evidence of an adrenal adenoma. A DHEA-S
level was normal (21). Consideration was given to a synthetic
glucocorticoid test, although the patient finally acknowledged
TABLE 1.
to using hydrocortisone-containing hemorrhoid creams inter-
Lab 4/11 4/12 5/22 5/23 mittently (and copiously). Subsequent evaluation off the oint-
24 h UFC (nml 45 mcg/d) 331 56 ment for 1 month showed normalized UFC tests for the past 6
A.m. cortisol (nml 522 mcg/dl) 29 months.
Cortisol after 1 mg DST 12.8
(nml 1.8 mcg/dl)
Case 2
ACTH (0800 h) (nml 658 pg/mL) 35 14
A 57-year-old Hispanic female presented with a 1-year history
Salivary cortisol (nml 80 ng/dl) 110 157
of marked central weight gain (40 lbs) and newly-diagnosed

The Endocrine Society. Downloaded from press.endocrine.org by [${individualUser.displayName}] on 12 January 2017. at 12:19 For personal use only. No other uses without permission. . All rights reserved.
ENDO 2016 NEUROENDOCRINOLOGY AND PITUITARY 193

impaired glucose tolerance, hypertension, and dyslipidemia. At a 1-year followup, the patient noted mild weight regain,
She had facial plethora/moon facies, increased supraclavicular worsening hypertension, and insomnia, and was concerned
fat pads, central obesity (body mass index, 39 kg/m2), and about recurrent Cushings disease. A pituitary MRI showed a
atrophic lower-extremity muscles. She had no ecchymoses or minimally larger, left-sided, cavernous sinus abnormality.
violaceous abdominal striae. Initial endocrine labs showed UFCs were normal (38 and 47 mcg/d). The morning serum
(Table 3). cortisol levels were 10 12 mcg/dL, and ACTH levels were
4. You next recommend: 36 45 pg/mL.
A. An adrenal-directed therapy for symptomatic hyper- 6. You next recommend:
cortisolism and likely ectopic ACTH tumor (based on a A. Continue periodic monitoring with 24-hour urinary
failed 8-mg dexamethasone suppression test). free cortisol tests (every 36 mo).
B. Pituitary MRI. B. A 1-mg dexamethasone suppression test.
C. CT scan-neck/chest/abd. C. Midnight salivary cortisol tests.
D. IPSS. D. Empiric ketoconazole.
Correct answer: B. Correct answer: C.
The 8-mg DST has suboptimal diagnostic test characteristics An elevated midnight salivary, or serum cortisol values, is
compared with IPSS. For example, failed cortisol suppression the earliest abnormality with recurrent CD.
with 8 mg DST is common with ACTH-secreting macro- The patients midnight salivary cortisol tests returned
adenomas. The next recommended step, in this case, is a pituitary 22.5-fold elevated.
MRI. 7. Consideration should now be given to all of the following
A pituitary MRI showed a 2.0 1.1 0.9-cm mass abut- except:
ting the optic chiasm, with left lateral extension. The patient A. Tumor-directed therapy with cabergoline or pasireotide.
subsequently underwent a successful TSS. The pathology dem- B. Radiation/stereotactic radiosurgery.
onstrated an ACTH/Crooke cell adenoma, and the 72-hour C. Treatment with steroidogenesis inhibitors or a gluco-
postoperative cortisol level was 1.8 mcg/dL. Prednisone-5 mg a corticoid antagonist.
day was initiated. A 3-month postoperative MRI showed a D. Repeat transsphenoidal resection.
subtle left-sided cavernous sinus hypodensity. Clinically, she Correct answer: D.
did well with significant improvement in hypertension and Repeat TSS is unlikely to be curative with cavernous sinus
impaired glucose tolerence and weight loss of approximately disease, and therefore not recommended. Conversely, radiation
15 lbs. She was tapered off glucocorticoids after approximately therapy, or several medical therapies, could be considered for
5 months. patients with recurrent or persistent CS.
5. The patient is at risk for persistent/recurrent CD based on
all of the following factors, except: Case 3
A. The tumor size and possible cavernous sinus A 55-year-old African-American female presented with poorly
involvement. controlled diabetes mellitus (A1c, 9.6%), HTN, and hypokalemia.
B. The postoperative cortisol level. She had moon facies, marked alopecia, and proximal muscle
C. The duration of secondary adrenal insufficiency. weakness, but no violaceous striae or atrophic skin changes. Labs
D. The ACTH tumor histological subtype. indicated ACTH-dependent CS: UFC, 3737 mcg/d; ACTH,
Correct answer: B. 226 pg/mL. Based on the marked hypercortisolism, her endocri-
A postoperative cortisol level of less than 2 mcg/dL gener- nologist initiated a workup for an ectopic ACTH source, but the
ally portends CD cure, although this has been studied mostly in neck/chest/abd/pelvic CT, chest MRI, Octreotide scan, and FDG-
microadenomas. Conversely, recurrence rates are higher in CD PET/CT scans did not identify a tumor. A pituitary MRI was also
patients with macroadenomas (including the aggressive Crooke unremarkable, and an IPSS showed no ACTH gradient or step-up
cell adenoma variant), and with duration of secondary adrenal with CRH.
insufficiency less than 1 year. 8. For the most rapid control of hypercortisolism, you rec-
ommend consideration of the following medical therapies
except:
TABLE 3.
A. Metyrapone
Lab Tests 12/1 12/2 12/3 12/4 B. Ketoconazole
24 h UFC (mcg/d, nml 45) 304 356 C. Mifepristone
A.M. Cortisol (nml 522 mcg/dl) 22 D. Etomidate
Cortisol after 1 mg DST 18 Correct answer: C.
(nml 1.8 mcg/dl)
In cases of fulminant, life-threatening CS, rapid cortisol
8 mg DST 23
normalization through medication therapies, ACTH tumor re-
ACTH (0800 h) (nml 658 pg/mL) 187
section, or bilateral adrenalectomy is recommended. Among

The Endocrine Society. Downloaded from press.endocrine.org by [${individualUser.displayName}] on 12 January 2017. at 12:19 For personal use only. No other uses without permission. . All rights reserved.
194 ENDO 2016 MEET-THE-PROFESSOR CLINICAL CASE MANAGEMENT

the medical treatments listed, mifepristone has a relatively slow protein kinase A and adrenocortical tumors. Front Cell Dev Biol. 2015;
onset of action, so is not recommended as initial therapy. 3:26.
7. Berson SA, Yalow RS. Radioimmunoassay of ACTH in plasma. J Clin
The patient was started on ketoconazole, but her liver func- Invest. 1968;47:2725-2751.
tion tests significantly increased on maximum-dose ketoco- 8. Nieman LK, Chrousos GP, Oldfield EH, et al. The ovine corticotropin-
nazole, with alanine aminotransferase (ALT), 497 U/L (normal, releasing hormone stimulation test and the dexamethasone suppression test
752 U/L); and aspartate aminotransferase (AST), 305 U/L in the differential diagnosis of Cushings syndrome. Ann Intern Med.
1986;105:862-867.
(normal, 1239 U/L). The patients ketoconazole was discon-
9. Oldfield EH, Doppman JL, Nieman LK, et al. Petrosal sinus sampling with
tinued and she was switched to metyrapone, with improvement, and without corticotropin-releasing hormone for the differential diagnosis
but not resolution, of the hypercortisolism. She was then re- of Cushings syndrome. N Engl J Med. 1991;325:897-905.
ferred for further imaging studuies. 10. Nieman LK, Biller BM, Findling JW, et al. The diagnosis of Cushings
9. Limited studies suggest the following may be the most syndrome: An Endocrine Society Clinical Practice Guideline. J Clin
Endocrinol Metab. 2008;93:1526-1540.
sensitive imaging test for an occult, ectopic ACTH tumor 11. Pecori Giraldi F, Saccani A, Cavagnini F. Assessment of ACTH assay
after a nonlocalizing CT scan: variability: A multicenter study. Eur J Endo. 2011;164:505-512.
A. MRI of neck/chest/abdomen. 12. Yanovski JA, Cutler GB Jr, Doppman JL, et al. The limited ability of
B. High-dose Octreotide scan. inferior petrosal sinus sampling with corticotropin-releasing hormone to
distinguish Cushings disease from pseudo-Cushing states or normal
C. 68Gallium-SSTR-PET/CT scan.
physiology. J Clin Endocrinol Metab. 1993;77:503-509.
D. FDG-PET/CT scan. 13. Swearingen B, Katznelson L, Miller K, et al. Diagnostic errors after
Correct answer: C. inferior petrosal sinus sampling. J Clin Endocrinol Metab. 2004;89:3752-
Although there are very limited studies comparing the vari- 3763.
ous anatomic or functional imaging modalities, 68Gallium- 14. Doppman JL, Chang R, Oldfield EH, Chrousos G, Stratakis CA, Nieman
LK. The hypoplastic inferior petrosal sinus: A potential source of false-
DOTOTAC PET/CT scan may hold promise as a sensitive negative results in petrosal sampling for Cushings disease. J Clin
method for locating occult, ectopic ACTH tumors. Endocrinol Metab. 1999;84:533-540.
15. Sharma ST, Nieman LK. Is prolactin measurement of value during inferior
Research created by U.S. government employees and pre- petrosal sinus sampling in patients with adrenocorticotropic hormone-
pared as part of their official duty is not subject to copyright. dependent Cushings Syndrome? J Endocrinol Invest. 2013;36:1112-1116.
Therefore, this work remains in the Public Domain. 16. Woo YS, Isidori AM, Wat WZ, et al. Clinical and biochemical character-
istics of adrenocorticotropin-secreting macroadenomas. J Clin Endocrinol
Metab. 2005;90:4963-4969.
REFERENCES 17. Nieman LK, Biller BM, Findling JW, et al. Treatment of Cushings
1. Cushing H. The pituitary body and its disorders: Clinical states produced syndrome: An Endocrine Society clinical practice guideline. J Clin
by disorders of the hypophysis cerebri. Philadelphia: JB Lippincott, 1912 Endocrinol Metab. 2015;100:2807-2831.
(monograph). 18. Isidori AM, Sbardella E, Zatelli MC, et al. Conventional and nuclear
2. Cushing H. The basophil adenomas of the pituitary body and their clinical
medicine imaging in ectopic Cushings syndrome: A systematic review.
manifestations (pituitary basophilism). Bull Johns Hopkins Hosp.
J Clin Endocrinol Metab. 2015;100:3231-3244.
1932;50:137-195.
19. Bochicchio D, Losa M, Buchfelder M. Factors influencing the immediate
3. Walters W, Wilder RM, Kepler EJ. The suprarenal cortical syndrome with
presentation of ten cases. Ann Surg. 1934;100:670-688. and late outcome of Cushings disease treated by transsphenoidal surgery:
4. Meador CK, Liddle GW, Island DP, et al. Cause of Cushings syndrome in A retrospective study by the European Cushings Disease Survey Group.
patients with tumors arising from nonendocrine tissue. J Clin Endocrinol J Clin Endocrinol Metab. 1995;80:3114-3120.
Metab. 1962;22:693-703. 20. Castinetti F, et al. Ketoconazole in Cushings disease: Is it worth a try?
5. Fragoso MC, Alencar GA, Lerario AM, et al. Genetics of primary J Clin Endocrinol Metab. 2014;99:1623-1630.
macronodular adrenal hyperplasia. J Endocrinol. 2015;224:R31R43. 21. Yamaji T, et al. Serum dehydroepiandrosterone sulfate in Cushings syn-
6. Berthon AS, Szarek E, Stratakis CA. PRKACA: The catalytic subunit of drome. J Clin Endocrinol Metab. 1984;59:1164-1167.

The Endocrine Society. Downloaded from press.endocrine.org by [${individualUser.displayName}] on 12 January 2017. at 12:19 For personal use only. No other uses without permission. . All rights reserved.
ENDO 2016 NEUROENDOCRINOLOGY AND PITUITARY 195

Diabetes Insipidus: Principles of Diagnosis and Treatment

M06 BARRIERS TO OPTIMAL PRACTICE


Presented, April 1 4, 2016 The major barrier to optimal practice in disorders of water
metabolism is a general unfamiliarity with these disorders by
virtue of their relative rarity. Therefore, reviewing basic con-
Joseph G. Verbalis, MD. Professor of Medicine and cepts of diagnosis and treatment of DI can markedly enhance
Chief, Division of Endocrinology and Metabolism, an endocrinologists ability to manage these patients when they
Georgetown University, Washington, DC 20007, E-mail: present. A second barrier is the difficulty with performing
verbalis@georgetown.edu water deprivation tests and the lack of uniform criteria for both
executing and interpreting the results of these tests, which will
HISTORICAL OVERVIEW be discussed in detail during this presentation
Diabetes insipidus (DI) is one of the oldest recognized hor-
mone deficiency disorders. Just as diabetes mellitus, DI results LEARNING OBJECTIVES
in voluminous urine output but without the high glucose con- As a result of participating in this session, learners should be
tent of diabetes mellitus. The English physician Sir Thomas able to:
Willis reported the sweet taste of urine in diabetes mellitus as a 1. Make a diagnosis of DI; know when and how to
result of glucosuria in the 1600s, and later physicians realized perform both overnight and formal water deprivations
the absence of the sweetness in the urine of patients with DI, tests; appropriately interpret the results of water
hence the term insipidus. DI is also one of the earliest deprivation tests; understand the usefulness and the
hormone deficiency disorders that was successfully treated limitations of assessing the posterior pituitary bright
with hormone replacement therapy. Simultaneous reports by spot on magnetic resonance imaging (MRI).
German (von den Velden) and Italian (Farini) investigators in 2. Prescribe desmopressin for treatment of DI; understand
1913 described amelioration of the polyuria of DI by treatment how dosing differs for parenteral, intranasal, and oral
with posterior pituitary extracts. Once the active hormone of desmopressin use; appreciate the potential
the posterior pituitary was discovered to be vasopressin, complications of desmopressin treatment and how to
therapy subsequently evolved using synthetic preparations of avoid them.
arginine vasopressin (AVP) and lysine vasopressin. However, a 3. Differentiate among the patterns of postoperative and
seminal advancement in the field of peptide hormone therapy posttraumatic DI; understand the pathophysiology and
occurred in 1978 when the Czech chemist Zaoral described the time course of the triphasic response to pituitary stalk
synthesis of the AVP V2-receptor-specific agonist desmopres- section, and how isolated hyponatremia after traumatic
sin, the first peptide synthesized with structural modifications brain injury (TBI) is a variant of the triphasic response.
to enhance both the specificity and the duration of hormone 4. Appreciate the differences between DI and
action at desired receptors. Since then, DI has remained one of osmoreceptor dysfunction; know the clinical situations
the endocrine diseases with the most targeted and effective where osmoreceptor dysfunction should be considered
therapies. as a possibility; know the similarities and differences
between treating osmoreceptor dysfunction and DI.
SIGNIFICANCE OF THE CLINICAL PROBLEM
DI is not a common disorder (0.005% 0.01% or 1520 000 STRATEGIES FOR DIAGNOSIS, THERAPY,
cases worldwide, as compared with diabetes mellitus, 1.6% AND MANAGEMENT
ages 20 39 to 19.3% ages 75 or 1520 000 000 cases Hyperosmolality
worldwide). However, because of its relative rarity, even Hyperosmolality indicates a deficiency of water relative to
good endocrinologists are sometimes unfamiliar with the solute in the extracellular fluid (ECF). Because water moves
criteria necessary for the diagnosis of DI and the appropri- freely between the intracellular fluid (ICF) and ECF, this also
ate use of antidiuretic agents for the therapy of DI, often indicates a deficiency of total body water relative to total body
resulting in ineffective and sometimes dangerous treatment solute. Although hypernatremia can be caused by an excess of
of this disorder. The frequent occurrence of DI and associ- body sodium, the vast majority of cases are due to losses of
ated disorders of water metabolism after neurosurgery for body water in excess of body solutes, caused by either insuffi-
sellar and suprasellar lesions makes this a particularly chal- cient water intake or excessive water excretion. Consequently,
lenging area that endocrinologists must be familiar with in most of the disorders causing hyperosmolality are those asso-
order to appropriately manage these complicated postopera- ciated with inadequate water intake and/or deficient pituitary
tive patients. AVP secretion. Although hyperosmolality from inadequate wa-

The Endocrine Society. Downloaded from press.endocrine.org by [${individualUser.displayName}] on 12 January 2017. at 12:19 For personal use only. No other uses without permission. . All rights reserved.
196 ENDO 2016 MEET-THE-PROFESSOR CLINICAL CASE MANAGEMENT

ter intake is seen frequently in clinical practice, this is usually Osmoreceptor Dysfunction
not due to an underlying defect in thirst but rather results from The primary osmoreceptors that control AVP secretion and
a generalized incapacity to obtain and/or ingest fluids, often thirst are located in the anterior hypothalamus, and lesions of
stemming from a depressed sensorium. this region in animals cause hyperosmolality through a combi-
nation of impaired thirst and osmotically stimulated AVP se-
Etiologies and Diagnosis cretion (6). Initial reports in humans described this syndrome
Evaluation of the patients ECF volume status is important as a as essential hypernatremia, and subsequent studies used the
guide to fluid replacement therapy but is not as useful for term adipsic hypernatremia in recognition of the profound
differential diagnosis, because most hyperosmolar patients will thirst deficits found in most of the patients. All of these syn-
manifest some degree of hypovolemia. Rather, assessment of dromes can be grouped together as disorders of osmoreceptor
urinary concentrating ability provides the most useful data with function (7). Most of the cases reported to date have repre-
regard to the type of disorder present. Using this approach, sented various degrees of osmoreceptor destruction associated
disorders of hyperosmolality can be categorized as those in with different brain lesions (8). In contrast to lesions causing
which renal water conservation mechanisms are intact but are central DI, these lesions usually occur more rostrally in the
unable to compensate for inadequately replaced losses of hy- hypothalamus. For all cases of osmoreceptor dysfunction, it is
important to remember that afferent pathways from the brain-
potonic fluids from other sources, or those in which renal
stem to the hypothalamus generally remain intact; therefore,
concentrating defects are a contributing factor to the deficiency
these patients will usually have normal AVP and renal concen-
of body water (1).
trating responses to baroreceptor-mediated stimuli such as hy-
povolemia and hypotension.
Diabetes Insipidus
DI can result from either inadequate AVP secretion (central or
Differential Diagnosis
neurogenic DI) or inadequate renal response to AVP (nephro-
Distinguishing between central and NDI in a patient who is
genic DI [NDI]). Central DI is caused by a variety of acquired
already hyperosmolar entails simply evaluating the response to
or congenital anatomic lesions that disrupt the neurohypophy-
a trial of AVP or desmopressin. Administration of AVP (5 U
sis, including pituitary surgery, tumors, trauma, hemorrhage,
sc) or, preferably, the selective AVP V2R agonist desmopres-
thrombosis, infarction, or granulomatous disease (2). Severe
sin (2 g sc or iv), should cause a significant increase in urine
NDI is most commonly congenital due to defects in the gene
osmolality within 12 hours after injection in patients with
for the AVP V2 receptor (X-linked recessive pattern of inher-
central DI, indicating insufficient endogenous AVP secretion.
itance) or in the gene for the aquaporin-2 water channel (auto-
An absent or suboptimal response suggests renal resistance to
somal recessive pattern of inheritance) (3), but relief of chronic
AVP effects and, therefore, NDI. Although conceptually
urinary obstruction or therapy with drugs such as lithium can simple, interpretational difficulties often arise because the wa-
cause an acquired form sufficient to warrant treatment. Ac- ter diuresis produced by AVP deficiency causes a down-
quired NDI can result from hypokalemia or hypercalcemia, but regulation of AQP2 synthesis along with a wash-out of the
the mild concentrating defect generally does not by itself cause renal medullary concentrating gradient, such that increases
hypertonicity and responds to correction of the underlying in urine osmolality in response to administered AVP or
disorder (4). Regardless of the etiology of the DI, the end result desmopressin are not as great as would be expected (see Inter-
is a water diuresis due to an inability to concentrate urine pretation of urine concentration after AVP/desmopressin,
appropriately. Table 1).
Because patients with DI do not have impaired urine sodium Because patients with DI generally have an intact thirst mecha-
conservation, the ECF volume is generally not markedly de- nism, such patients often do not present with hyperosmolality but
creased, and regulatory mechanisms for maintenance of os- rather have normal plasma osmolality and serum sodium levels
motic homeostasis are primarily activated: stimulation of thirst with polyuria and polydipsia (2). In these cases, a fluid deprivation
and pituitary AVP secretion (to whatever degree the neurohy- test should be performed in order to raise the serum osmolality
pophysis is still able to secrete AVP). In cases where AVP and confirm the diagnosis of DI (see Table 1 for the procedure and
secretion is totally absent (complete DI), patients are dependent interpretation of a fluid deprivation test).
entirely on water intake for maintenance of water balance. When a diagnosis of central DI is made, MRI of the hypo-
However, in cases where some residual capacity to secrete thalamus and neurohypophyseal tract is mandatory to rule out a
AVP remains (partial DI), plasma osmolality can eventually neoplasm or granulomatous disease as an etiology. In individu-
reach levels that allow moderate degrees of urinary concentra- als with a normal posterior pituitary, the presence of pituitary
tion. Although untreated DI can lead to both hyperosmolality bright spot (ie, a hyperintense signal in the absence of contrast
and volume depletion, until the water losses become severe, administration, representing the AVP-containing neurosecre-
volume depletion is minimized by osmotic shifts of water from tory granules) is usually seen on T1-weighted noncontrast
the ICF into the more osmotically concentrated ECF (5). sagittal images. Conversely, in patients with central DI, the

The Endocrine Society. Downloaded from press.endocrine.org by [${individualUser.displayName}] on 12 January 2017. at 12:19 For personal use only. No other uses without permission. . All rights reserved.
ENDO 2016 NEUROENDOCRINOLOGY AND PITUITARY 197

TABLE 1. Procedure and Interpretation of the Fluid Deprivation Test for the Diagnosis of Diabetes Insipidus
Procedure
1. Initiation of the deprivation period depends on the severity of the DI; in routine cases, the patient should not ingest any food or fluids
after dinner, whereas in cases with more severe polyuria and polydipsia, this may be too long a period without fluids and the water
deprivation should be begun early on the morning (e.g. 6 am) of the test.
2. Obtain plasma and urine osmolality, serum electrolytes and a plasma AVP level at the start of the test.
3. Measure urine volume and osmolality hourly or with each voided urine.
4. Stop the test when body weight decreases by 3%, the patient develops orthostatic blood pressure changes, the urine osmolality
reaches a plateau (i.e. 10% change over two or three consecutive measurements), or the serum Na 145 mmol/liter.
5. Obtain plasma and urine osmolality, serum electrolytes, and a plasma AVP level at the end of the test, when the plasma osmolality is
elevated, preferably 300 mOsm/kg H2O.
6. If the serum Na 146 mmol/liter or the plasma osmolality 300 mOsm/kg H2O when the test is stopped, then consider a short
infusion of hypertonic saline (3% NaCl at a rate of 0.1 ml/kg/min for 12 hr) to reach these endpoints.
7. If hypertonic saline infusion is not required to achieve hyperosmolality, administer AVP (5 U) or desmopressin (1 g) SC and
continue following urine osmolality and volume for an additional 2 hr.
Interpretation
1. An unequivocal urine concentration after AVP/desmopressin (50% increase) indicates central DI and an unequivocal absence of
urine concentration (10%) strongly suggests nephrogenic DI (NDI) or primary polydipsia (PP).
2. Differentiating between NDI and PP, as well as for cases in which the increase in urine osmolality after AVP/desmopressin
administration is more equivocal (e.g. 10%50%), is best done using the relation between plasma AVP levels and plasma osmolality
obtained at the end of the dehydration period and/or hypertonic saline infusion and the relation between plasma AVP levels and urine
osmolality under basal conditions

bright spot is usually absent. However, this test is not defini- ered in the previous section discussing pathophysiology of
tive, because the pituitary bright spot decreases with age and specific types of DI. Interestingly, patients with DI typically
with disorders that cause dehydration, and it can be present in describe a craving for cold water, which appears to quench
up to 5% of patients with DI due to the presence of pituitary their thirst better. Patients with central DI also typically de-
oxytocin, which also leads to a hyperintense signal on scribe a precipitous onset of their polyuria and polydipsia,
T1-weighted imaging (5). which simply reflects the fact that urinary concentration can be
Evaluation of anterior pituitary function should be per- maintained fairly well until the number of AVP-producing
formed in all patients with central DI, especially if glucocorti- neurons in the hypothalamus decreases to 10%15% of normal,
coid administration and/or replacement unmasks underlying after which plasma AVP levels decrease to the range where
DI. Adrenal insufficiency can cause hypersecretion of AVP, urine output increases dramatically (11).
which may be due in part to reductions in systemic blood However, patients with DI, and particularly those with os-
pressure (BP) and cardiac output caused by cortisol deficiency, moreceptor dysfunction syndromes, can also present with vary-
thereby stimulating pituitary AVP release. Cortisol deficiency ing degrees of hyperosmolality and dehydration depending on
is also known to cause increased AVP release from the median their overall hydration status. It is therefore important to be
eminence into the pituitary portal circulation in an attempt to aware of the clinical manifestations of hyperosmolality as well.
increase ACTH secretion via effects at pituitary vasopressin These can be divided into the signs and symptoms produced by
V1b receptors on corticotrophs, which may also contribute to dehydration, which are largely cardiovascular, and those
increased systemic AVP levels (9). caused by the hyperosmolality itself, which are predominantly
The recent development of a commercial assay for the C- neurological and reflect brain dehydration as a result of os-
terminal glycoprotein of the AVP prohormone, copeptin, offers motic water shifts out of the central nervous system (CNS) (1).
the possibility of a more stable and easier to measure marker of Cardiovascular manifestations of hypertonic dehydration in-
AVP secretion in response to induced dehydration and hyper- clude hypotension, azotemia, acute tubular necrosis secondary
tonicity (10). However, the clinical use of copeptin levels as a to renal hypoperfusion or rhabdomyolysis, and shock. Neuro-
surrogate marker of AVP secretion during fluid deprivation logical manifestations range from nonspecific symptoms such
tests will require standardization of normal and abnormal re- as irritability and cognitive dysfunction to more severe mani-
sponses relative to plasma AVP levels. festations of hypertonic encephalopathy such as disorientation,
decreased level of consciousness, obtundation, chorea, sei-
Clinical Manifestations zures, coma, focal neurological deficits, subarachnoid hemor-
The characteristic clinical symptoms of DI are the polyuria and rhage, and cerebral infarction. The severity of symptoms can
polydipsia that result from the underlying impairment of uri- be roughly correlated with the degree of hyperosmolality, but
nary concentrating mechanisms, which have already been cov- individual variability is marked and for any single patient the

The Endocrine Society. Downloaded from press.endocrine.org by [${individualUser.displayName}] on 12 January 2017. at 12:19 For personal use only. No other uses without permission. . All rights reserved.
198 ENDO 2016 MEET-THE-PROFESSOR CLINICAL CASE MANAGEMENT

level of serum [Na] at which symptoms will appear cannot be avoid producing cerebral edema during treatment, particu-
accurately predicted. larly in pediatric patients (15).
Similar to hypoosmolar syndromes, the length of time over It should be remembered that formulae which estimate body
which hyperosmolality develops can markedly affect the water deficits do not take ongoing water losses into account.
clinical symptomatology. Rapid development of severe Consequently, frequent serum electrolyte determinations must
hyperosmolality is frequently associated with marked neuro- be made, and the administration rate of oral water or iv 5%
logical symptoms, whereas gradual development over several dextrose in water should be adjusted accordingly. For example,
days or weeks generally causes milder symptoms. In this case, the estimated water deficit of a 70-kg patient whose serum
the brain counteracts osmotic shrinkage by increasing intracel- [Na] is 160 mmol/L would be 5.25 L. In such an individual,
lular content of solutes. These include electrolytes such as administration of water at a rate more than 200 mL/h would be
potassium and a variety of organic osmolytes, which previ- required simply to correct the established deficit over 24 hours,
ously had been called idiogenic osmoles; for the most part but additional fluid would be needed to keep up with any
these are the same organic osmolytes that are lost from the ongoing losses in a patient with DI until a response to treat-
brain during adaptation to hypoosmolality (12). The net effect ment has occurred.
of this process is to protect the brain against excessive shrink- A variety of antidiuretic agents have been used to treat
age during sustained hyperosmolality. However, once the brain central DI, but desmopressin is the treatment of choice for this
has adapted by increasing its solute content, rapid correction of disorder. Desmopresin was synthesized as a selective antago-
the hyperosmolality can produce brain edema, because it takes nist of AVP V2 receptor, and it is particularly useful therapeu-
a finite time (24 48 h in animal studies) to dissipate the tically, because it has a much longer half-life than AVP and is
accumulated solutes, and until this process has been completed, devoid of the pressor activity of AVP at vascular AVP V1a
the brain will accumulate excess water as plasma osmolality is receptors (16). Desmopressin is generally administered
normalized (13). This effect is most often seen in dehydrated intranasally (10 20 g every 8 24 h) but can be given paren-
pediatric patients who can develop seizures with rapid rehydra- terally in acute situations (12 g iv, im, or sc). For both the
tion, but it has been described only rarely in adults. intranasal and parenteral preparations, increasing the adminis-
tered dose generally has the effect of prolonging the duration of
Therapy antidiuresis rather than increasing its magnitude; consequently,
The general goals of treatment of all hyperosmolar disorders altering the dose can be useful to reduce the required frequency
are 1) correction of preexisting water deficits, and 2) reduction of administration. Synthetic AVP (Pitressin) can also be used
in ongoing excessive urinary water losses. The specific therapy to treat central DI, but its use is limited by a much shorter
required varies with the clinical situation. Awake ambulatory half-life necessitating more frequent dosing or a continuous
patients with DI and normal thirst have little body water deficit infusion, and the production of pressor effects due to vasocon-
but benefit from relief of the polyuria and polydipsia that striction.
disrupt normal activities. In contrast, comatose patients with or NDI is more difficult to treat because the kidney is resistant
without DI are unable to drink in response to thirst, and in to all AVP-type agents. Limited responses can sometimes be
these patients, progressive hypertonicity may be life threaten- achieved using thiazide diuretics (any drug of the thiazide class
ing. The established water deficit may be estimated using the may be used with equal potential for benefit). Thiazides cause
following formula (14): natriuresis by blocking sodium absorption in the cortical dilut-
ing site; when combined with dietary sodium restriction a
Water deficit 0.6 premorbid weight modest hypovolemia results, which stimulates isotonic proxi-
mal tubular solute reabsorption and diminishes solute delivery
1 140/serum Na mmol/L to the distal parts of the nephron. Together, these effects
diminish free water clearance independently of actions of AVP,
This formula is dependent on several assumptions (total thereby decreasing the polyuria of patients with NDI (17).
body water is 60% of body weight, no body solute is lost Monitoring for hypokalemia is necessary and K supplemen-
as hypertonicity developed, and the premorbid [Na] is 140 tation is occasionally required. Care must be exercised when
mmol/L) but nonetheless provides a valid estimate of the treating patients taking lithium with diuretics, because the in-
approximate total body water deficit. To reduce the risk of duced contraction of plasma volume may increase lithium
CNS injury from protracted exposure to severe hypertonic- concentrations by increasing proximal tubular absorption and
ity, the serum osmolality should be lowered to the range of worsen potential toxic effects of the therapy (18). Because
330 mOsm/kg H2O within the first 24 hours of therapy. As prostaglandins increase renal medullary blood flow and dimin-
noted previously, because the organic osmolytes accumu- ish medullary solute concentration, effects that modestly de-
lated in the brain during chronic hyperosmolality cannot be crease the interstitial gradient for water reabsorption, drugs that
immediately dissipated, further correction to a normal osmo- block renal prostaglandin synthesis (eg, nonsteroidal antiin-
lality should be spread over the subsequent 13 days to flammatory agents) can increase non-AVP-mediated water re-

The Endocrine Society. Downloaded from press.endocrine.org by [${individualUser.displayName}] on 12 January 2017. at 12:19 For personal use only. No other uses without permission. . All rights reserved.
ENDO 2016 NEUROENDOCRINOLOGY AND PITUITARY 199

absorption and impair urinary dilution, thereby reducing free hypothalamus; unlike DI, which produces polyuria and
water clearance and urine output. Although these agents are polydipsia, osmoreceptor dysfunction causes a selective
somewhat effective in central DI, their main usefulness is as loss of AVP secretion in response to osmotic but not
adjunctive therapy in NDI, in which more direct antidiuretic baroreceptor stimuli, and adipsia because of absent
therapies are limited. stimulation of thirst centers as a result of hyperosmolality;
osmoreceptor dysfunction should be considered with
lesions involving the anterior wall of the third ventricle
MAIN CONCLUSIONS
1. The diagnosis of DI is based upon suboptimal urine (craniopharyngiomas, anterior communicating artery
concentration in the presence of hyperosmolality and aneurysms, granulomatous diseases such as sarcoidosis,
the absence of a solute diuresis; these criteria are rarely etc); osmoreceptor dysfunction should be treated with
met on initial presentation, because polydipsia is desmopressin in combination with a prescription for fluid
sufficient to maintain normal water homeostasis, so intake based on the patients urinary plus insensible fluid
overnight or formal fluid deprivations testing is losses.
frequently required to confirm a diagnosis; in formal
fluid deprivation testing the indirect (desmopressin CASES
administration) and direct (AVP measurement) Case 1
approaches should be combined to maximize diagnostic A 24-year-old male was referred for increasing polyuria and
accuracy; absence of a posterior pituitary bright spot on polydipsia over the last 2 years. He reports urinating every 2
MRI is supportive evidence for central DI but by itself hours during the day with nocturia 23 times nightly. He drinks
cannot be relied upon diagnostically. 6 7 L/d and craves cold water. Baseline labs showed serum
2. Desmopressin is the preferred treatment of DI; effective [Na] 143 mEq/L and urine osmolality (Uosm) 73
intranasal doses (10 20 g) are 10 parenteral doses mOsm/kg H2O. A 24-hour urine showed total volume 7.0 L
(12 g), and effective oral doses (100 200 g) are (55 mL/kgh). After overnight fluid deprivation, [Na] 141
10 intranasal doses; oral doses should not be mEq/L and Uosm 83 mOsm/kg H2O. He was admitted for a
administered with food to decrease proteolytic formal fluid-deprivation test (see Table 2).
destruction of desmopressin; the major potential
complication of desmopressin treatment is Case 2
hyponatremia, which can be prevented by avoiding A 28-year-old male previously in normal health was found
excessive fluid ingestion/administration in patients unconscious at the roadside after a motorcycle accident. He
taking desmopressin. was not wearing a helmet. On arrival at the Emergency Room,
3. Postoperative and posttraumatic DI can take the form of a computerized axial tomography scan showed a basilar skull
several different patterns; the triphasic response to fracture. In the first 24 hours, his urine output was 11.0 L with
pituitary stalk section or damage consists of DI, a urine osmolality of 102 mOsm/kg H2O. Despite more than 7
followed by syndrome of inappropriate antidiuresis L of iv fluids, his serum [Na] increased to 160 mmol/L. After
(SIADH) as a result of unregulated AVP release from 5 days of treatment with vasopressin, his DI resolved, but his
degeneration of a denervated posterior pituitary, and serum [Na] then decreased to 128 mmol/L with a urine
then a return of DI; isolated hyponatremia after TBI is osmolality of 955 mOsm/kg H2O despite cessation of vasopres-
a variant of the triphasic response, in which the sin therapy. The patient had a grand mal seizure requiring
pituitary stalk damage is not sufficient to cause DI but therapy with 3% NaCl. A schematic diagram of the patients
does result in SIADH as a result of unregulated AVP hospital course is shown in Figure 1.
release from degeneration of a partially denervated
posterior pituitary. Case 3
4. Osmoreceptor dysfunction is a variant of DI caused by A 49-year-old male underwent a transcranial resection of a
damage to osmoreceptive cells in the anterior 3.5 4.2-cm craniopharyngioma. Postoperatively he devel-

TABLE 2. Fluid Deprivation Test for the Diagnosis of Diabetes Insipidus


Time Serum Na Uosm Posm Urine Volume
0 h 143 77 301 1125
9 h 147 128 625
DDAVP 1 g
11 h 210 230
12 h 511 95

The Endocrine Society. Downloaded from press.endocrine.org by [${individualUser.displayName}] on 12 January 2017. at 12:19 For personal use only. No other uses without permission. . All rights reserved.
200 ENDO 2016 MEET-THE-PROFESSOR CLINICAL CASE MANAGEMENT

FIGURE 1. Schematic diagram of the patient described in Case 2. I/O input/output; pAVP plasma AVP level;
p[Na] plasma sodium concentration.

oped hypotonic polyuria and hyperosmolality that was treated sellar region. The triphasic pattern is relatively uncommon,
with desmopressin 1 mcg sc q12h. Despite the desmopressin, occurring in 1.1% (first 2 phases only) and 3.4% (all 3 phases)
he remained hyperosmolar with a serum [Na] 158 mEq/L. of patients undergoing transphenoidal surgery. The first phase
The desmopressin was increased to a fully antidiuretic dose (2 of DI typically lasts 57 days and then transitions into a second
mcg sc q8h) with Uosm more than 800 mOsm/kg H2O but antidiuretic phase of the SIADH, as in this case. This second
without change in clinical status. His 24-hour urine volume phase is due to the uncontrolled release of AVP from degener-
decreased to 1.4 L/d, but fluid intake was only 1.2 L/d, and the ating posterior pituitary tissue. In this phase, the urine becomes
patient denied being thirsty. concentrated and urine output markedly decreases. Continued
administration of excess water during this period can quickly
CASE DISCUSSIONS lead to hyponatremia and hypoosmolality, as occurred in this
Case 1 is a classic presentation of spontaneous DI (ie, patient on posttrauma day 6. The duration of the second phase
nontraumatic or postsurgical). Although the presenting labora- is variable and can last from 2 to 14 days. This case manifested
tory results are consistent with central DI, they are not diag- a relatively short duration of antidiuresis lasting 4 5 days.
nostic. The overnight fluid deprivation test also failed to con- Some patients with more limited damage to the neurohypophy-
firm a diagnosis of DI, because the serum [Na] stayed in the sis manifest the phenomenon of an isolated second phase,
normal range, thus requiring formal fluid deprivation testing. which has been reported to produce hyponatremia in up to
After 9 hours, the patient met criteria for DI with hypotonic 8%21% of patients after pituitary surgery. After AVP stores
polyuria despite hyperosmolality. The response to desmopres- are depleted from the degenerating posterior pituitary tissue,
sin (50% increase in Uosm after 12 h; indirect test) then the third phase of chronic DI often, but not always, ensues. In
confirmed a diagnosis of central DI. Measurement of plasma this phase, there are insufficient remaining AVP neurons ca-
AVP level before desmopressin administration (0.5 pg/mL; pable of synthesizing additional AVP, thereby resulting in
direct test) further verified the diagnosis of central DI. Once permanent DI. Previous studies showed that the major determi-
confirmed, the patient underwent testing for the etiology of nant of permanent DI following pituitary stalk sectioning is the
central DI, manly consisting of a brain MRI to rule out mass level of the lesion: the closer the lesion to the magnocellular
or inflammatory lesions. A final diagnosis of lymphocytic cell bodies in the hypothalamus, the more likely that the hypo-
infundibuloneurohypohysitis was made and the patient was thalamic cell bodies will degenerate, resulting in permanent DI.
begun on desmopressin therapy, 10 mcg intranasally twice Case 3 is represents an example of osmoreceptor dysfunc-
daily with a good response. The DI has persisted for more than tion. The primary osmoreceptors that control AVP secretion
5 years. and thirst are located in the anterior hypothalamus and lesions
Case 2 is a classic presentation of a triphasic response to of this region in animals cause hyperosmolality through a
pituitary stalk damage following head trauma or surgery in the combination of impaired thirst and osmotically stimulated AVP

The Endocrine Society. Downloaded from press.endocrine.org by [${individualUser.displayName}] on 12 January 2017. at 12:19 For personal use only. No other uses without permission. . All rights reserved.
ENDO 2016 NEUROENDOCRINOLOGY AND PITUITARY 201

secretion. Initial reports in humans described this syndrome as 2. Robertson GL. Diabetes insipidus. Endocrinol Metab Clin North Am.
1995;24(3):549-572.
essential hypernatremia, and subsequent studies used the term
3. Fujiwara TM, Bichet DG. Molecular biology of hereditary diabetes insipi-
adipsic hypernatremia in recognition of the profound thirst dus. J Am Soc Nephrol. 2005;16(10):2836-2846.
deficits found in most of the patients. All of these syndromes 4. Khanna A. Acquired nephrogenic diabetes insipidus. Semin Nephrol.
can be grouped together as disorders of osmoreceptor function. 2006;26(3):244-248.
5. Robinson AG, Verbalis JG. Posterior pituitary. In: Melmed S, Polonsky
Most of the cases reported to date have represented various
KS, Larsen PR, Kronenberg HM, eds. Williams Textbook of Endocrinol-
degrees of osmoreceptor destruction associated with different ogy. Philadelphia, PA: W.B. Saunders; 2011:291-323.
brain lesions. In contrast to lesions causing central DI, these 6. Johnson AK, Buggy J. Periventricular preoptic-hypothalamus is vital for
lesions usually occur more rostrally in the hypothalamus. For thirst and normal water economy. Am J Physiol. 1978;234(3):R122-R129.
7. Baylis PH, Thompson CJ. Osmoregulation of vasopressin secretion and
all cases of osmoreceptor dysfunction it is important to remem-
thirst in health and disease. Clin Endocrinol (Oxf). 1988;29(5):549-576.
ber that afferent pathways from the brainstem to the hypothala- 8. Baylis PH, Thompson CJ. Diabetes insipidus and hyperosmolar syndromes.
mus generally remain intact; therefore, these patients will usu- In: Becker KL, ed. Principles and Practice of Endocrinology and Metabolism.
ally have normal AVP and renal concentrating responses to Philadelphia,, PA: Lippincott Williams, Wilkins; 2001:285-293.
9. Oelkers W. Hyponatremia and inappropriate secretion of vasopressin (an-
baroreceptor-mediated stimuli such as hypovolemia and hypo-
tidiuretic hormone) in patients with hypopituitarism. N Eng J Med.
tension. Although such patients need to be treated with 1989;321:492-496.
desmopressin for their DI, this isnt sufficient to maintain a 10. Morgenthaler NG, Struck J, Jochberger S, Dunser MW. Copeptin: clinical
normal plasma osmolality in the absence of thirst. Conse- use of a new biomarker. Trends Endocrinol Metab. 2008;19(2):43-49.
11. Heinbecker P, White HL. Hypothalamico-hypophyseal system and its
quently, their therapy must include a prescribed amount of
relation to water balance in the dog. Am J Physiol. 1941;133:582-593.
daily fluid intake in addition. Typically, fluid intake should be 12. Gullans SR, Verbalis JG. Control of brain volume during hyperosmolar
500-1000 mL above urine output to account for insensible and hypoosmolar conditions. Annu Rev Med. 1993;44:289-301.
losses. However, in practice the amount needed must be deter- 13. Verbalis JG. Brain volume regulation in response to changes in osmolality.
Neuroscience. 2010;168(4):862-870.
mined empirically by trial and error while checking the serum
14. Robinson AG, Verbalis JG. Diabetes insipidus. Curr Ther Endocrinol
[Na] frequently, generally at least weekly. For some patients, Metab. 1997;6:1-7.
a home sodium analyzer (eg, iStat) significantly aids chronic 15. Adrogue HJ, Madias NE. Hypernatremia. N Engl J Med. 2000;342
management. (20):1493-1499.
16. Robinson AG. DDAVP in the treatment of central diabetes insipidus.
N Engl J Med. 1976;294(10):507-511.
REFERENCES 17. Sands JM, Bichet DG. Nephrogenic diabetes insipidus. Ann Intern Med.
1. Verbalis JG. Disorders of water balance. In: Taal MW, Chertow GM, 2006;144(3):186-194.
Marsden PA, Skorecki K, Yu ASL, Brenner BM, eds. Brenner, Rectors 18. Grunfeld JP, Rossier BC. Lithium nephrotoxicity revisited. Nat Rev
The Kidney. Philadelphia, PA: Elsevier; 2012:540-594. Nephrol. 2009;5(5):270-276.

The Endocrine Society. Downloaded from press.endocrine.org by [${individualUser.displayName}] on 12 January 2017. at 12:19 For personal use only. No other uses without permission. . All rights reserved.
202 ENDO 2016 MEET-THE-PROFESSOR CLINICAL CASE MANAGEMENT

Non-Functioning Pituitary Adenomas and Incidentalomas

M26 tumors have an important burden on the health care system.


Presented, April 1 4, 2016 Therefore, optimal resource distribution for both clinical care
and research activities aiming to improve the outcome for these
patients is needed (4, 6, 7).
Jens Bollerslev, MD, DMSc; Kristin Astrid Berland
ystese, MD. Section for Specialized Endocrinology,
LEARNING OBJECTIVES
Rikshospitalet, Oslo University Hospital, 0424 Oslo,
As a result of participating in this session, learners should be
Norway, E-mail: jens.bollerslev@medisin.uio.no able to:
Diagnose patients with NFPA and pituitary
INTRODUCTION incidentalomas.
Historical Overview Handle the patients, knowing indications for treatment
Pituitary adenomas (PAs) are common, comprising approxi- (surgery/radiotherapy), and schedule a follow-up regime.
mately one third of all intracranial tumors (1). With the
increasing access to cerebral imaging, pituitary tumors are
STRATEGIES FOR DIAGNOSIS, THERAPY,
increasingly identified by chance, when an investigation is
AND/OR MANAGEMENT
performed for another reason than a suspicion of a pituitary
Presentation
diseasethe definition of a pituitary incidentaloma. Although usually benign, NFPAs present therapeutic chal-
lenges because of their location close to the optic chiasm and
Epidemiology optic nerves, and the proximity to the healthy pituitary tissue.
Already in the 1930s there was a genuine interest in pituitary NFPAs become clinically manifest late in the development, at
anatomy and function, and also in pituitary tumors without the time when they cause mass effect on surrounding tissue
known clinical manifestations (2). Several early studies from (visual impairment, headache) or compress pituitary hormonal
this period reported unsuspected pituitary masses in approxi- secretion. This is contrary to hormone-secreting PAs normally
mately 10% of routine autopsies with a diameter of 125 mm detected at an earlier stage because of characteristic signs and
[reviewed in the thesis by RT Costello (2)]. In his own series of symptoms (8).
1000 autopsies, Costello found a prevalence of more than 22% The PA are in most cases sporadic; however, up to 5% are
(2). Using modern imaging techniques, (ie, computed tomog- familial adenomas, either as part of the multiple endocrine
raphy [CT] or magnetic resonance imaging [MRI] scans) al- neoplasia or familial isolated PAs, often caused by mutations in
most the same figures for pituitary incidentalomas seem to the AIP gene (9). Also, an increasing number of incidentalomas
come out, the vast majority being tumors less than 10 mm are found on MRI or computed tomography (CT) performed
(microadenomas), reviewed by Freda et al (3). for other reasons, raising questions of how it should be
The exact prevalence of PA is difficult to estimate because handled (10).
of the variation in clinical presentation and the study methods The study and treatment of NFPA is challenging due to the
or the registers used. A recent nationwide observational study location at the base of the skull and because the complex
in Iceland spanning almost six decades demonstrated an overall functions of the pituitary gland, but also due to the restricted
prevalence of nonfunctioning pituitary adenomas (NFPAs) for tissue availability and the lack of animal models adequately
2012 of 43 per 100 000 inhabitants, NFPA being the next mimicking human pituitary tumorigenesis.
most-prevalent-diagnosed pituitary tumor. For the entire pe-
riod, NFPA were the most prevalent, comprising 43% of all Workup and Imaging
diagnosed adenomas. The standard incidence rate for PA in- A pituitary tumor, either being diagnosed due to mass effect or
creased during the observed period, estimated to be 5.8 per incidentally should be systematically investigated for hormonal
100 000 by the end of the observation period (4). The incidence overproduction and potential hormonal deficiencies secondary
rate and distribution seems to be comparable with recent esti- to the tumor mass effect (3, 11). If the tumor is close to the
mates from the Swedish Pituitary Register Study, demonstrat- optic chiasm or abuts the optic nerves an ophtalmological
ing an overall incidence rate of PA of 3.9 per 100 000 per year investigation is indicated. Potential hormonal deficiencies of
(5). Also here, NFPAs were the most common (54%), followed importance (cortocitroph and thyreotroph) should be substi-
by the hormone-secreting adenomas (ie, prolactinomas [32%], tuted before any further treatment (3, 11).
GH producing [9%], ACTH producing [4%], and TSH produc- MRI imaging is essential for the preoperative assessment of
ing [0.7%]). Thus, it seems that the prevalence of NFPA is patients, but also in characterizing the growth of tumors during
4-fold higher than previously reported, suggesting that pituitary followup. T1-weighted MRI technique is the standard modality

The Endocrine Society. Downloaded from press.endocrine.org by [${individualUser.displayName}] on 12 January 2017. at 12:19 For personal use only. No other uses without permission. . All rights reserved.
ENDO 2016 NEUROENDOCRINOLOGY AND PITUITARY 203

FIGURE 1. T1-weighted magnetic resonance scan in the coronal and sagittal plane of a 63-year-old man with a
pituitary macroadenoma.

of pituitary imaging (Figure 1). Coronal and sagittal planes gery, the patient should be investigated for new pituitary defi-
before and after iv contrast are used. T2-weighted images have ciencies by dynamic testing and adequately substituted (11).
increasingly been used in recent years and gives further quali-
tative information about tumor tissue (12), especially for Growth and Regrowth
The progression rate of NFPA is difficult to estimate. Some
hormone-producing adenomas. The qualitative importance of
patients with NFPA experience a slow progression rate neces-
T2 for NFPA has so far not been elaborated.
sitating a long observation time, whereas other NFPAs act
more aggressively and invade the neighboring structures,
CASE STUDY thereby requiring rapid neurosurgical intervention to prevent
A man born in 1946 was referred to our clinic and seen for the long-term impairment on visual field or pituitary deficiency (8).
first time in July 2009. He was diagnosed with arterial hyper- So far, little is known about tumor growth and behavior, before
tension some years earlier, well treated. Due to hearing prob- and after surgery (14, 15). Knowledge of adenoma growth
lems, he was referred to a CT scan of the petrosal bones, kinetics (ie, linear, logistic, or exponential growth) collaborated
revealing a pituitary mass and no other abnormalities. An MRI with data on tumor biology would substantially enhance our
was performed with a pituitary protocol (Figure 1). A typical clinical understanding and help to adjust and individualize
pituitary macroadenoma was disclosed. At the consultation, he MRI-scanning protocols and follow-up investigations (16). So
seemed unremarkable with no signs of hormonal over- or far, the only published systematic study investigating specific
underproduction, confirmed biochemically without dynamic growth patterns of NFPA, albeit with a limited number of
testing. He described no symptoms related to the pituitary patients (n 15), concluded that the linear growth model is not
mass. His case was discussed at the following transdisciplinary suitable and that an exponential growth model or a logistic
pituitary meeting. The consensus was to simply follow the case model (Figure 2) is more accurate to describe the growth
clinically and by means of repeated magnetic resonance scans, kinetics of NFPA (16). Moreover, follow-up studies have
the first time after 6 months (spring 2010). shown that macroadenomas (tumors 1 cm) tend to progress
more frequently than microadenomas (1719).
Tumor volume doubling time (TVDT; Figure 2) has been
Management and Treatment
investigated both preoperatively and postoperatively for
Compared with other PAs, and to add to the complexity of the
NFPAs, although data are limited to a few studies and poten-
issue, there are no approved medical therapies for NFPA. If
tially hampered by selection bias, being based on relatively few
indicated, the first-line treatment of NFPA is transsphenoidal
patients and a retrospective design (1, 15).
surgery (11). Unfortunately, successful removal of the entire
tumor is possible in only up to a third of the patients, and most Different Growth Patterns
of the patients will therefore be in need of long-term clinical, Of growing tumors, most will initially grow exponentially,
biochemical, and radiological followup (13). Following sur- later achieving a logistic growth pattern. When a TVDT has

The Endocrine Society. Downloaded from press.endocrine.org by [${individualUser.displayName}] on 12 January 2017. at 12:19 For personal use only. No other uses without permission. . All rights reserved.
204 ENDO 2016 MEET-THE-PROFESSOR CLINICAL CASE MANAGEMENT

FIGURE 2. Different growth patterns of PAs. Based on and adapted from Hoenegger (16).

been estimated, a prognosis for maximal further growth can be years, giving a TVDT of 14.5 years, if the tumor growth
established. There is no indication in the literature that continued to be exponential.
NFPAs will show accelerated growth, (ie, a growth pattern
with decreasing TVDT) (16). Growth and Regrowth
Most NFPA will grow exponentially in the beginning fol-
lowed by a plateau phase, most probably due to limited
Followup
blood supply (16). The growth rate is very individual; in
The patient was followed at regular intervals, with new MRI scans
general NFPAs have a lower TVDT in younger than in older
performed in May 2010, followed by scans in the spring of 2011,
patients (16, 20).
2012 and 2013. At all appointments, the patient seemed
unremarkabe, with no specific symptoms related to the NFPA. All MAIN CONCLUSIONS
scans were described as unchanged compared with the previous NFPAs are common and constitute a major part of PAs
investigation. At the consultation in the spring of 2013 it was and incidentalomas. If indicated, first-line therapy is
decided that we should wait 18 months until the next appointment transsphenoidal surgery. Patients should be evaluated for
with a new MRI. In October 2014, the patient was in good hormonal deficiencies before and after surgery, and
condition and he did not receive any medication, except for the substituted if indicated.
antihypertensive treatment. The hormonal workup was unremark- The growth pattern of the primary tumor or the remnant
able. The MRI scan was again described without significant tumor after surgery can give valuable information as a
changes compared with the investigation in 2013. We decided to basis for an individualized follow-up protocol.
go over the scans for the whole period (Figure 3). It is of importance always to study the whole series of
Assuming that the tumor had a form of an ellipsoid, the scans for growth/regrowthnot only the two last
increase in volume was calculated to be 28.5% over the 5 images.

The Endocrine Society. Downloaded from press.endocrine.org by [${individualUser.displayName}] on 12 January 2017. at 12:19 For personal use only. No other uses without permission. . All rights reserved.
ENDO 2016 NEUROENDOCRINOLOGY AND PITUITARY 205

FIGURE 3. Coronal, T1-weighted MRI scans of the case from 2009 (left) and 2014 (right). A significant increase in
tumor volume is indicated.

REFERENCES 11. Chanson P, Raverot G, Castinetti F, Cortet-Rudelli C, Galland F, Salenave


1. Aflorei ED, Korbonits M. Epidemiology and etiopathogenesis of pituitary S. Management of clinically non-functioning pituitary adenoma. Ann
adenomas. J Neurooncol. 2014;117(3):379-394. Endocrinol (Paris). 2015;76(3):239-247.
2. Costello RT. Subclinical adenoma of the pituitary gland. Am J Pathol. 12. Patronas NJ, Liu CY. State of art imaging of the pituitary tumors.
1936;12(2):205-216.1. J Neurooncol. 2014;117(3):395-405.
3. Freda PU, Beckers AM, Katznelson L, et al. Pituitary incidentaloma: An 13. Cortet-Rudelli C, Bonneville JF, Borson-Chazot F, et al. Post-surgical
Endocrine Society clinical practice guideline. J Clin Endocrinol Metab. management of non-functioning pituitary adenoma. Ann Endocrinol
2011;96(4):894-904. (Paris). 2015;76(3):228-238.
4. Agustsson TT, Baldvinsdottir T, Jonasson JG, et al. The epidemiology of 14. Chen Y, Wang CD, Su ZP, et al. Natural history of postoperative
pituitary adenomas in Iceland, 1955-2012: A nationwide population-based nonfunctioning pituitary adenomas: A systematic review and meta-
study. Eur J Endocrinol. 2015;173(5):655-664. analysis. Neuroendocrinology. 2012;96(4):333-342.
5. Tjornstrand A, Gunnarsson K, Evert M, et al. The incidence rate of 15. Monsalves E, Larjani S, Loyola Godoy B, et al. Growth patterns of
pituitary adenomas in western Sweden for the period 2001-2011. Eur J pituitary adenomas and histopathological correlates. J Clin Endocrinol
Endocrinol. 2014;171(4):519-526. Metab. 2014;99(4):1330-1338.
6. Fernandez A, Karavitaki N, Wass JA. Prevalence of pituitary adenomas: A 16. Honegger J, Zimmermann S, Psaras T, et al. Growth modelling of non-
community-based, cross-sectional study in Banbury (Oxfordshire, UK). functioning pituitary adenomas in patients referred for surgery. Eur J
Clin Endocrinol (Oxf). 2010;72(3):377-382. Endocrinol. 2008;158(3):287-294.
7. Daly AF, Rixhon M, Adam C, Dempegioti A, Tichomirowa MA, Beckers 17. Molitch ME. Management of incidentally found nonfunctional pituitary
A. High prevalence of pituitary adenomas: A cross-sectional study in the tumors. Neurosurg Clin N Am. 2012;23(4):543-553.
province of Liege, Belgium. J Clin Endocrinol Metab. 2006;91(12):4769- 18. Scangas GA, Laws ER Jr. Pituitary incidentalomas. Pituitary. 2014;17(5):
4775. 486-491.
8. Cooper O, Melmed S. Subclinical hyperfunctioning pituitary adenomas: 19. Lenders N, Ikeuchi S, Russell AW, Ho KK, Prins JB, Inder WJ. Longitu-
The silent tumors. Best Pract Res Clin Endocrinol Metab. 2012;26(4):447- dinal evaluation of the natural history of conservatively managed non-
460. functioning pituitary adenomas [published online August 20, 2015]. Clin
9. Vierimaa O, Georgitsi M, Lehtonen R, et al. Pituitary adenoma predispo- Endocrinol (Oxf). doi:10.1111/cen.12879.
sition caused by germline mutations in the AIP gene. Science (New York, 20. Tanaka Y, Hongo K, Tada T, Sakai K, Kakizawa Y, Kobayashi S. Growth
NY). 2006;312(5777):1228-1230. pattern and rate in residual nonfunctioning pituitary adenomas: Correla-
10. Esteves C, Neves C, Augusto L, et al. Pituitary incidentalomas: Analysis tions among tumor volume doubling time, patient age, and MIB-1 index.
of a neuroradiological cohort. Pituitary. 2015;18(6):777-781. J Neurosurg. 2003;98(2):359-365.

The Endocrine Society. Downloaded from press.endocrine.org by [${individualUser.displayName}] on 12 January 2017. at 12:19 For personal use only. No other uses without permission. . All rights reserved.
206 ENDO 2016 MEET-THE-PROFESSOR CLINICAL CASE MANAGEMENT

Growth Hormone DeficiencyControversies in the Clinical


Management in Adults

M44 industry as the best surrogates for efficacy and safety of GH


Presented, April 1 4, 2016 therapy of adult GHD.
Also less-well recognized is that the occurrence of newer
possible indications for testing and treatment of GHD very
Ulla Feldt-Rasmussen, MD, DMSc. Department of likely has a lower a priori likelihood of the disease than the
Medical Endocrinology and Metabolism, Rigshospitalet, severely hypopituitary patients initially investigated, so we are
Copenhagen University Hospital, Copenhagen DK-2100, dealing with other patient populations where neither testing nor
Denmark, E-mail: ufeldt@rh.dk treatment efficacy have been investigated and where current
guidelines do not apply, even if they indicate to do so (35).
INTRODUCTION
Historical Overview BARRIERS TO OPTIMAL PRACTICE
The first use of the term midget as description of a propor- Most recommendations and guidelines on management of
tionate dwarf was in 1816, but it was not until 1912,that GHD in adults have a low level of evidence based on retro-
Harvey Cushing in The Pituitary Gland proposed the exis-
spective studies and expert opinions. Moreover, the definition
tence of a hormone of growth promoting skeletal growth. GH
of GHD is heterogeneous in the literature.
or somatropin was first extracted from cadaveric pituitaries in
Management of patients with GHD should first of all con-
the late 1950s, and soon after, Maurice Raben (1) described
sider who should be tested for GHD; next which stimulation
other more metabolic actions of this hormone in humans. The
test to be used; the proper diagnostic cutoff concentration of
purified hormone was then used for the treatment of short
GH; and finally the relevance, efficacy, and safety of GH
stature in hypopituitary children, although in 1962 Raben al-
replacement therapy.
ready had described general health improvement after injection
Many diagnostic tests have been developed for GHD, most
of GH in a hypopituitary adult. Further developments of
of them for patients with established hypothalamo-pituitary
knowledge of GH effects in adults was brought forward by
disease with a high a priori test outcome for deficiency. The
development in 1963 of a RIA for measuring GH in serum, as
same tests are now also used for diagnosing GHD in a number
well as production of recombinant human (rh) GH in 1981.
of other potential patient populations, raising high risk of
These two discoveries made studies of GH concentrations in
misuse and wasting of resources. Further, the technical perfor-
adults as well as effects on the human body of GH deficiency
(GHD) possible, and studies on effects from replacement with mance of hormone assays is highly variable among different
rhGH could begin to take place. laboratories.
Appropriate management of GHD is multidisciplinary,
needing a high level of integration and coordination among
SIGNIFICANCE OF THE CLINICAL PROBLEM different specialists.
Adult GHD is thus a well-recognized clinical entity. It causes
abnormalities in substrate metabolism, body composition, and
physical and psychosocial function. Since the mid 1980s an LEARNING OBJECTIVES
improvement has been recognized with GH replacement, and As a result of participating in this session, learners should be
this has gradually been incorporated in clinical routine based aware that:
on the few short-term initial randomized clinical trials, which Stringency in choice of test for GHD is pertinent to
led to international guidelines in 1997 (2). correct diagnosis.
Less-well recognized is the fact that these early studies were Each center that diagnoses GHD should have its own
based on selected patient groups with very severe hypopituitar- cutoffs for the methods they use.
ism and therefore high a priori likelihood of severe GHD, few New indications for the assessment of GHD and other
study participants, short-term treatment, and supraphysiologi- hypopituitarism need special attention in order not to
cal GH doses that were calculated based on the experience overdiagnose and overtreat while awaiting more
from childhood GHD. Despite knowledge on the very high appropriate studies and guidelines.
influence of age on GH/IGF-I secretion the initially chosen GH replacement should be monitored regularly to assess
adult doses in the studies were inadvertently chosen too high. both efficacy and safety.
Most of the current recommendations and guidelines are thus GH replacement of patients with GHD should generally
based on later retrospective single center experience or data be considered safe and with a positive effect on the
from large surveillance databases run by the pharmaceutical adverse features of GHD.

The Endocrine Society. Downloaded from press.endocrine.org by [${individualUser.displayName}] on 12 January 2017. at 12:19 For personal use only. No other uses without permission. . All rights reserved.
ENDO 2016 NEUROENDOCRINOLOGY AND PITUITARY 207

STRATEGIES FOR DIAGNOSIS, THERAPY, The insulin tolerance test is thus probably the most reliable test
AND/OR MANAGEMENT although intra-individual variabilities do occur, also speaking in
GH therapy is now considered a very safe therapy according to favor of double or confirmatory testing (7), not only of GHD but
a very recent position paper (6), which should however be also of the other pituitary functions (7, 9). Further, long-term
closely monitored. Treatment itself follows very simple guide- follow up may be needed, even in patients with apparently iso-
lines, starting with low dose and increasing doses according to lated (GHD) hormone deficiency (10).
IGF-I measurements at short intervals, taking into account age,
sex (mainly estrogen status), and possible adverse effects.
MAIN CONCLUSIONS
The main controversy, although not very well recognized by
True GHD is an important clinical entity that should be treated
most clinicians, lies in the diagnosis. There are several autho-
and managed properly. In contrast, it is important to avoid false
rized guidelines in the literature and on top of numerous indi-
diagnosis, which might lead to unnecessary life-long therapy
vidual reviews. So, what is the general clinician going to rely
with GH replacement.
on? Table 1 shows some important differences in the test
The diagnosis of GHD is rather simple in patients with a
eligibility and testing recommendation in three different guide-
typical structural pathology in the hypothalamo-pituitary re-
lines, where one is an update of the original from 1998. The
gion, especially in cases and multiple pituitary hormone defi-
main purpose of that concensus was to avoid overdiagnosing
ciencies and low IGF-I concentrations in which the likelihood
adult GHD, why stringent criteria of number of tests and low
of GHD exceeds 97% and a stimulation test is not necessary.
cutoff level as the gold standard was chosen. This stringency
In all other cases it is my opinion that two tests should be
was compromised in the update based on several studies indi-
performed to avoid the risk of overtreatment on a wrong basis
cating that rightly not all cases (eg, 3 multiple pituitary
(Figure 1). The knowledge of ones own laboratory performance
hormone deficiencies; 3) needed several tests. Unfortunately,
as well as own reference population data with BMI cutoffs for
other etiologies such as traumatic brain injury (TBI) and
control persons is crucial in interpretation of results. The same
subarachnoidal hemorrhage were included as new eligible di-
holds true for the testing of the other hormone axes, some of
agnoses, without scrutinizing the facts from several studies
which have similar challenges in diagnosing correctly in
indicating that isolated GHD in these cases may have been
hypothalamo-pituitary disease states (9).
strongly overemphasized based on only single testing for GHD
Treatment itself is simply based on IGF-I titration, and both
(5, 7). The Endocrine Society update is more to the point if
safety and efficacy should be monitored.
recommendations on two tests in isolated GHD should be
standard. New indications for GHD testing should not be accepted
All tests have limitations. The gold standard was initially the without prior stringent testing by several tests, and preferably
insulin-tolerance test with a cutoff of 3.0 g/L (depending on GH in different laboratories, given that the classical GHD
method in ones own laboratory), but in the Endocrine Society phoenotype such as obesity, fatigue, and poor quality of life
guidelines and a recent review (8), the GHRH-arginine test was most often has causes other than GHD.
described with similar sensitivity and accuracy. The main limita-
tion of the GHRH-arginine test is its high dependence on body CASES
mass index (BMI), requiring BMI-dependent cutoffs (on top of Case 1
the GH method ones). Furthermore, the test is not reliable in A 26-year-old female, unemployed, and unmarried. Since age
patients in early phases of hypothalamic injury such as irradiation. 20 years progressive weight increase and now weighs 102 kg,

TABLE 1. Criteria for GHD Testing in Various Guidelines


GRS 1997 (2) GRS 2007 (3) ENDO 2011 (4)
Eligible for Testing Hypothalamic/pituitary pathology Hypothalamic/pituitary pathology Hypothalamic/pituitary pathology
For adult GHD Verified CO-GHD Verified CO-GHD Verified CO-GHD
Brain irradiation Brain irradiation Brain irradiation
Traumatic brain injury and SAH Traumatic brain injury and SAH
Recommended One stim test in MPHD No stim tests in MPHD low IGF-I No stim tests in MPHD low IGF-I
Testing for GHD Two dynamic tests in all others One stim test in other situations One stim test in most other situations
Except 2 tests recommended in isolated
GHD

GRS, Growth Hormone Research Society; ENDO, Endocrine Society; GHD, growth hormone deficiency; CO, childhood onset; SAH,
subarachnoidal haemorrhage; stim, dynamic test; MPHD, multiple pituitary hormone deficiency.

The Endocrine Society. Downloaded from press.endocrine.org by [${individualUser.displayName}] on 12 January 2017. at 12:19 For personal use only. No other uses without permission. . All rights reserved.
208 ENDO 2016 MEET-THE-PROFESSOR CLINICAL CASE MANAGEMENT

BMI, 36 kg/m2. For several years she has had loss of alertness,
aching legs, no headache, irregular periods. She had a moder-
ately severe head trauma from a fall from a bicycle at 18 years
of age. Her general physician referred her for an endocrine
evaluation, where the following was found: Normal plasma
levels of estrogen, FSH, and LH, and regular periods; serum
TSH, 1.8 mU/L, serum free T4, 19.4 pmol/L; peak serum
cortisol 30 minutes after Synacthen, 650 mmol/L; no symp-
toms of diabetes insipidus; serum IGF-I, 198 mg/L (age-related
reference range, 123 463 mg/L); GHRH-arginine test showed
peak GH, 4.9 g/L (Normal cutoff response 14.9 g/L); She
was given the diagnosis GHD, while the other pituitary hor-
mone axes were normal.
Should she receive GH replacement therapy?
A. No, she has GHD but does not qualify for therapy
because she is obese.
B. Yes, she qualifies for GHD with one deficient
stimulation test and IGF-I below 0 SDS.
C. No, because she is obese and has a different cutoff for
GH by the GHRH-arginine test (4.0 g/L), and
therefore does not have GHD.

Case 2
A 27-year-old girl with type 1 diabetes since age 5 years had
been diagnosed with pituitary tumor at age 12 years due to
reduced growth velocity. There were no signs of hormone
overproduction, and the diagnosis was thus nonfunctioning
pituitary adenoma. She had multiple pituitary hormone insuffi-
FIGURE 1. The prevalence of insufficient test responses ciency and started substitution with T4, hydrocortisone, and
in the total cohort (A) and in the subgroup undergoing GH after proper stimulation tests. She went into normal pu-
dual testing (B). A, Prevalence of insufficient test berty and had regular periods thereafter. She had regular mag-
responses to either ITT or Pyridostigmin-GHRH/ netic resonance imaging (MRI) scans to check for tumor
GHRH-arg (ie, combined tests) in the total cohort of growth. At age 27 years cranial MRI showed growth of the
TBI patients (black columns) and healthy controls pituitary tumor, but without suprasellar growth or affection of
(white columns), respectively, as defined by either local the vision. She had paused GH replacement several times
or guideline-derived cutoffs. Whiskers indicate the 95% previously (upon retesting for adult indication and while laser
confidence intervals. GHD was more frequently treating proliferative retinopathy), with occurrence of severe
diagnosed in TBI patients tested by a combined test as GHD symptoms each time.
compared with ITT, and even more so if guideline What is the most appropriate management?
cutoff values were applied instead of local cutoffs. The A. Continue GH, explain that GH therapy is not
results from healthy controls illustrate the high false- responsible for the growth, and continue surveillance of
positive rate resulting from application of guideline- the tumor.
derived cutoffs, which was significantly above the B. Stop GH immediately and continue surveillance of the
generally accepted 2.5% for the combined tests (P tumor.
.02). *, P < .005 compared with patients. B, Prevalence C. Stop GH, explain that GH therapy is responsible for
of insufficient test responses in the subgroups of the growth, and send the patient to neurosurgery.
patients (black columns; n 169) and controls (white D. Continue GH, explain that GH therapy is not
columns; n 117) undergoing dual testing, as defined responsible for the growth, and send the patient to
by either local cutoff values or guideline-derived cutoff neurosurgery.
values. Confirmed insufficiency was defined as a
concordant positive result to both the ITT and a
combined test. Whiskers indicate the 95% confidence DISCUSSION OF CASES AND ANSWERS
intervals. Reproduced with permission (7). Case 1
The case deals with one of the important diagnostic pitfalls in
testing for GHD. Obesity has a pronounced effect on the GH

The Endocrine Society. Downloaded from press.endocrine.org by [${individualUser.displayName}] on 12 January 2017. at 12:19 For personal use only. No other uses without permission. . All rights reserved.
ENDO 2016 NEUROENDOCRINOLOGY AND PITUITARY 209

response by GHRH-arginine, requiring BMI-related cutoffs 2. Consensus guidelines for the diagnosis and treatment of adults with
growth hormone deficiency: summary statement of the Growth Hor-
(g/L), generally given in the literature as normal weight,14.9
mone Research Society Workshop on Adult Growth Hormone Defi-
g/L; overweight, 8.6 g/L; obese, 3.0 g/L. However, these ciency. J Clin Endocrinol Metab. 1998;83:379-381.
cutoffs are very dependent on the method for measuring GH, 3. Ho KK. Consensus guidelines for the diagnosis and treatment of adults
so ideally all specialized centers managing patients with pitu- with GH deficiency II: A statement of the GH Research Society in
association with the European Society for Pediatric Endocrinology,
itary diseases should have their own method- and laboratory-
Lawson Wilkins Society, European Society of Endocrinology, Japan En-
derived reference ranges and cutoffs between normal and dis- docrine Society, and Endocrine Society of Australia. Eur J Endocrinol.
eased persons. Had she been biochemically GHD by that test, 2007;157:695-700.
with normal other axes, and no known structural hypothalamo- 4. Molitch ME, Clemmons DR, Malozowski S, Merriam GR, Vance ML.
Evaluation and treatment of adult growth hormone deficiency: An endo-
pituitary disease, I would have completed the diagnostic strat-
crine society clinical practice guideline. J Clin Endocrinol Metab.
egy by another stimulation test (eg, insulin tolerance test; ITT) 2011;96:1587-1609.
less influenced by BMI (but not completely diagnostically safe 5. Klose M, Feldt-Rasmussen U. Hypopituitarism in traumatic brain injuryA
either). critical note. J Clin Med. 2015;4:1480-1497.
6. Allen DB, Backeljauw P, Bidlingmaier M, et al. GH safety workshop
position paper: A critical appraisal of recombinant human GH therapy in
Case 2
children and adults. Eur J Endocrinol. 2015;174(2):P1-P9.
The case deals with the issue of regrowth of pituitary tumors 7. Klose M, Stochholm K, Janukonyte J, et al. Prevalence of posttraumatic
during GH-replacement therapy. No hard evidence exists since growth hormone deficiency is highly dependent on the diagnostic set-up:
there have been no long-term randomized clinical trials com- Results from the Danish National Study on posttraumatic hypopituitarism.
J Clin Endocrinol Metab. 2014;99:101-110.
paring tumor growth and GH or placebo. The tumors are
8. Andersen M. The robustness of diagnostic tests for GH deficiency in
generally very slowly growing and after the approval of GH for adults. Growth Horm IGF Res. 2015;25:108-114.
replacement in adult GHD, it has not been and will not be 9. Klose M, Marina D, Hartoft-Nielsen ML, et al. Central hypothyroidism
possible to perform such trials. and its replacement have a significant influence on cardiovascular risk
factors in adult hypopituitary patients. J Clin Endocrinol Metab.
2013;98:3802-3810.
REFERENCES 10. Klose M, Jonsson B, Abs R, et al. From isolated GH deficiency to multiple
1. Raben MS. Growth hormone. 2. Clinical use of human growth hormone. pituitary hormone deficiency: An evolving continuum - a KIMS analysis.
N Engl J Med. 1962;266:82-86. Eur J Endocrinol. 2009;161 Suppl 1:S75-S83.

The Endocrine Society. Downloaded from press.endocrine.org by [${individualUser.displayName}] on 12 January 2017. at 12:19 For personal use only. No other uses without permission. . All rights reserved.
210 ENDO 2016 MEET-THE-PROFESSOR CLINICAL CASE MANAGEMENT

Traumatic Brain Injury and Fatigue

M48 thy (CTE) may be the end stage of this chronic disease, but
Presented, April 1 4, 2016 more studies are needed.

BARRIERS TO OPTIMAL PRACTICE


Albert Chamberlain, MD; Randall J. Urban, MD; There remains a stigma associated with GH administration,
Melinda Sheffield-Moore, PhD. Department of Internal much like that of T only a decade ago, the concern being that
Medicine, Endocrinology and Metabolism, University of GH therapy may be unnecessary or that any dosage in border-
Texas Medical Branch, Galveston, Texas 77555, E-mail: line cases will lead to supraphysiologic GH levels. This con-
melmoore@utmb.edu, rurban@utmb.edu cern may be largely attributable to the lack of existing evidence
as to the benefit of GH supplementation in patients other than
INTRODUCTION the most severely GHD (glucagon-stimulation test [GST] 3
Historical Overview g/L). This current conservative approach to GH replacement
The first case of anterior pituitary dysfunction caused by head has resulted in high costs for GH and subsequent resistance to
trauma was reported in 1918, but by 1961 only 15 cases had insurance reimbursement for therapy.
been documented (1, 2). Even a 2000 review of published cases The most looming barrier, however, remains the lack of
by Benvenga et al (3) only found 314 cases of hypopituitarism studies that show the benefits of GH replacement in patients
following traumatic brain injury (TBI), over 80 years later. with TBI. These studies are difficult because of the time in-
Investigations into the prevalence of GH deficiency (GHD) volved to show GH treatment effects in patients and the lack of
following TBI also began in 2000 (3-5). Since then numerous a true understanding of the mechanism causing the symptoms
associated with GHD. We are currently investigating the ben-
publications have reviewed the incidence of hypopituitarism
efits of GH administration in GH-deficient patients after mild
after both moderate-to-severe and mild TBI. The most recent
TBI through functional magnetic resonance imaging and brain
review showed an approximately 30% incidence of hypopitu-
blood-flow studies.
itarism after TBI (6). The most frequent pituitary abnormality
is GHD, occurring approximately 20% of the time. Although
there is much media attention today about TBI, there is little LEARNING OBJECTIVES
discussion of its effects on the pituitary. We hope as we better As a result of participating in this session, learners should be
understand the mechanisms of pituitary dysfunction after TBI, able to:
and show how treatment can help the lives of patients, there Recognize that pituitary dysfunction in post-TBI patients
will be an increased awareness. is a chronic disease with chronic effects that may
manifest subtly over a long period of time.
Recognize that the spectrum of symptoms that often
SIGNIFICANCE OF THE CLINICAL PROBLEM accompany TBI are part of a broader disease than purely
Most of our experience with GHD occurs in the setting of the pituitary dysfunction.
occurrence of a pituitary tumor that results in damage to the
somatotroph cells either by expansion in the sella, radiation to
STRATEGIES FOR DIAGNOSIS AND
the tumor, or surgical removal. Therefore, our view of GHD is
MANAGEMENT OF GHD AFTER TBI
generally based on an event in time with either loss of function
Background
or retention of function.
In moderate-to-severe TBI, the accompanying endocrine defi-
However, we propose a different paradigm regarding pitu- ciencies (and necessary replacement strategies) that may de-
itary dysfunction after TBI. We propose that in a segment of velop in the acute phase of the injury (721 d) is well docu-
the population, TBI will induce a state of chronic brain inflam- mented (710). By comparison, little has been done to address
mation that results in a chronic disease state. One of the the chronic (1 y) endocrine developments after TBI. Notably,
manifestations of this chronic disease is GHD. The GHD oc- somatotrophs and gonadotrophs have been shown to be the
curs across a spectrum from very deficient to normal. Associ- most vulnerable anterior pituitary hormones after the acute
ated with the GHD are a well-defined group of symptoms that phase of TBI. It has been suggested that this is attributable to
are improved with GH replacement. As this chronic disease is their anatomical location and corresponding arterial supply (3,
better defined, we anticipate additional symptoms to be identi- 11, 12).
fied. As we better understand the mechanisms of this disease The primary traumatic lesion and mechanical compression
state in the brain we hope to better understand how to treat the caused by the event may be followed by a secondary hypoxic
disease. We can speculate that chronic traumatic encephalopa- and/or hypotensive insult. Brain swelling and increased cranial

The Endocrine Society. Downloaded from press.endocrine.org by [${individualUser.displayName}] on 12 January 2017. at 12:19 For personal use only. No other uses without permission. . All rights reserved.
ENDO 2016 NEUROENDOCRINOLOGY AND PITUITARY 211

pressure are thought to be the cause of this secondary insult, main diagnostic tool to identify GHD of any severity in pa-
particularly in cases without skull fracture (3, 13, 14). The tients with TBI.
definitive description of this pathophysiology remains unclear Diagnosis can be made using the peak GH observed in GST
and we believe that a chronic inflammatory state develops at with well-researched cutoffs including severe GHD (GH 3
some point during the described events. g/L) and moderate GHI (GH 3 8 g/L). Currently, our rec-
A subset of patients with TBI will develop pituitary dys- ommendation is to treat patients with a history of TBI whose
function, although which hormones will be affected and the maximum GH is less than 10 g/L on glucagon stimulation if
timing after the injury follow no currently discernible pattern. they present with the symptoms described above.
The most common pituitary dysfunction is seen in GH with
approximately 1525% of TBI patients having a degree of GH
Management
abnormality at some point after injury (35, 1517). GHD in Our recommendation is that once the diagnosis of GHD or GHI
TBI manifests with two distinct symptoms. The first is fatigue.
has been established, GH should be replaced in stepwise fash-
This fatigue is profound, many times causing the patient to
ion with 2-month intervals beginning at 0.2 mg sc once daily.
reduce or quit their current job. The second is loss of cognition
At the 2-month mark, serum IGF-I levels should be drawn, and
of three specific functions: executive function, short-term
if IGF-I is less than 400 g/L, then the once-daily dosage
memory retention, and processing speed index (4, 1719).
should be increased to 0.4 mg. This increase will be followed
Our previous work, which included both patients with GH
by another 2-month interval at the end of which a second
insufficiency (GHI) and GHD has shown that 1 year of GH
serum IGF-I level should be obtained. A dosage increase to 0.6
replacement therapy leads to improvements in cognitive func-
tion including increased processing speed, cognitive flexibility, mg sc once daily will begin at month 5 if serum IGF-I levels
and verbal learning. Improvements in upper extremity motor continue to remain less than 400 g/L. In some rare instances,
speed were also significant in those subjects receiving GH it may be necessary in reproductive-age females to increase
therapy (17). this dose further to 0.8 mg after a third 2-month-long period.
This study illustrates that post-TBI patients with symptoms At any point in this therapy regimen, if IGF-I is more than
of fatigue and cognitive dysfunction will benefit from GH 400 g/L, then the daily dosage should be stepped down using
replacement. this same interval pattern to the next highest daily dose that
does not elevate serum IGF-I greater than 400 g/L.
Diagnostic Approach Potential adverse effects of GH replacement therapy include
Patients with a history of moderate-to-severe TBI are much peripheral or generalized edema, mono- or polyarticular
more likely to have a documented medical history of the TBI arthralgias, carpal tunnel syndrome, and glucose intolerance,
compared with those patients having suffered a mild TBI. although the latter is extremely rare in clinical practice because
Patients with mild TBI can be more difficult to treat because of the low doses of GH used in these patients.
they may not be able to identify or elucidate the event or time
range when injury occurred. Regardless of the severity, there is
MAIN CONCLUSIONS
no established timeline for the development of post-TBI signs
In conclusion, we put forth the notion that TBI is a chronic
and symptoms. Patients may live normal lives for years before
disease. We believe that the chronic nature of TBI may be
noticing a change in their daily demeanor or activity level. This
derived from a persistent inflammatory condition that arises
makes identification of post-TBI patients challenging given
over time, making TBI a progressive disease. In addition, if
that many complaints, including memory impairment, fatigue,
pituitary dysfunction occurs in response to a TBI, we purport
decreased physical activity, weight gain, anxiety, concentration
that this circumstance can induce a multitude of other symp-
impairment, depression, and deteriorating social skills have a
toms (fatigue, foggy brain, cognitive dysfunction in areas of
vast array of possible psychological and endocrine origins (20).
Therefore, it is up to the physician to garner an ever-present executive function, memory loss, etc.) that could lead the
suspicion and derive clues from the history including any past patient or the treating physician to question whether further
vehicular accidents, risk-taking behavior, and participation in testing is warranted but these patients are rarely followed or
contact sports, to identify patients suffering the effects of a treated. We challenge the mainstream approach of treat and
previous TBI. The patients symptoms should be the driving forget in the postconcussive period whereas the progressive
factor for pituitary function testing once their history has been nature of the post-TBI syndrome demands long-term oversight
established. as the evolution of this chronic condition is ever changing.
We have shown that using 175 g/L as a threshold for Although research in this area is scant, our data and that of
serum IGF-I can be helpful in choosing whom to evaluate others strongly suggests that patients with TBI who develop
further with a GST (21). However, IGF-I should not be used in GHI as a chronic consequence of head trauma can benefit from
place of GST to diagnose GHD or GHI. We use the GST as the GH replacement therapy much like severe GHD patients.

The Endocrine Society. Downloaded from press.endocrine.org by [${individualUser.displayName}] on 12 January 2017. at 12:19 For personal use only. No other uses without permission. . All rights reserved.
212 ENDO 2016 MEET-THE-PROFESSOR CLINICAL CASE MANAGEMENT

CASES WITH QUESTIONS C. YES: Measurement of his IGF-I as a screen for the
Case 1 status of his GH axis is warranted. If found to be
You see a 45-year-old male in clinic with a history of moderate GHD, then begin GH replacement.
TBI from a 4-wheeler accident. He is complaining of severe D. NO: Giving additional thyroid replacement in hopes his
fatigue. His cortisol, prolactin, and thyroid function tests are fatigue will improve will only increase the risk of
normal but his T is low. You initiate T replacement and see hyperthyroidism and not improve the fatigue.
him back in clinic after 3 months. His T level is now between E. NO: Although referring him to a psychologist might
midrange and the upper limit of normal. The patient reports his help with the depression from his reduced abilities, it
fatigue is slightly better but it is still profound and he cannot will not reverse the fatigue and cognitive problems.
continue to work.
What would the next best step be? Case 2
A. Increase his T replacement to help further his profound What would the next best step be?
fatigue. A. NO: Although referring her to a neurologist for
B. Recommend he return to his primary care physician for assessment is likely warranted given her history of
additional evaluation of his fatigue because you have head injury, given that she has not lost consciousness
in any of her prior head blows, assessment of her
done all you can from an endocrine perspective.
pituitary is recommended as a first course of screening.
C. Measure an IGF-I as a screen for the status of his GH
B. NO: In this setting she most likely would have
axis.
secondary hypothyroidism and is in need of a more
D. Start thyroid replacement in hopes his fatigue will
complete assessment of anterior pituitary function.
improve.
C. NO: Although it is possible that her nutrition is poor as
E. Refer him to psychologist to better understand why he
a college student, she is active and outside on a regular
is not coping with life. basis playing soccer and is physically fit. Thus, low
vitamin D is unlikely to be causing her foggy brain.
Case 2 D. YES: Foggy brain and other cognitive impairments
You see a 20-year-old female in clinic who plays soccer at the can result from pituitary hormone deficiency.
local college. She has been playing soccer since junior high Assessment of her pituitary function is warranted.
school. She reports multiple knocks to her head from either E. NO: Although referral to the campus psychologist may
headers or head-to-head collisions with other players but she help her to understand her problems, the foggy brain
never lost consciousness. Approximately 6 months ago she was and inability to function in class is a real problem
rear ended at a stop light by a student who was intoxicated. She likely induced from pituitary hormone deficiency.
did not go to the emergency room or lose consciousness. She
comes today concerned that she is now struggling in school, REFERENCES
making passing grades but not her usual As and Bs. She feels 1. Altman R, Pruzanski. Post-traumatic hypopituitarism. Anterior pituitary
insufficiency following skull fracture. Ann Intern Med. 1961;55:149-154.
she is in a fog and wonders if it is her hormones.
2. Cyran E. Hypophysenschadigung durch schadelbasisfraktur (in German).
What would the next best step be? Dtsch Med Wschr. 1918;44:1261.
A. Refer her to a neurologist. 3. Benvenga S, Campenn A, Ruggeri RM, Trimarchi F. Clinical review 113:
Hypopituitarism secondary to head trauma. J Clin Endocrinol Metab.
B. Measure her TSH looking for primary hypothyroidism.
2000;85(4):1353-1361.
C. Measure her vitamin D level given that her nutrition is 4. Kelly DF, Gonzalo IT, Cohan P, Berman N, Swerdloff R, Wang C.
probably poor. Hypopituitarism following traumatic brain injury and aneurysmal subarach-
noid hemorrhage: A preliminary report. J Neurosurg. 2000;93(5):743-752.
D. Assess her pituitary function.
5. Lieberman SA, Oberoi AL, Gilkison CR, Masel BE, Urban RJ. Prevalence
E. Refer her to the campus psychologist to better cope of neuroendocrine dysfunction in patients recovering from traumatic brain
with the pressures of academics. injury. J Clin Endocrinol Metab. 2001;86(6):2752-2756.
6. Lauzier F, Turgeon AF, Boutin A, et al. Clinical outcomes, predictors, and
prevalence of anterior pituitary disorders following traumatic brain injury:
DISCUSSION A systematic review. Crit Care Med. 2014;42(3):712-721.
7. Barton RN, Stoner HB, Watson SM. Relationships among plasma cortisol,
Case 1 adrenocorticotrophin, and severity of injury in recently injured patients.
What would the next best step be? J Trauma. 1987;27(4):384-392.
A. NO: Increasing his T further is not likely to further 8. Cernak I, Savic VJ, Lazarov A, Joksimovic M, Markovic S. Neuroendo-
crine responses following graded traumatic brain injury in male adults.
reduce his fatigue, especially if his serum T is within Brain Inj. 1999;13(12):1005-1015.
normal range from the initial dosing. 9. Pentelenyi T, Kammerer L. Alterations of the serum cortisol and blood
B. NO: It is unlikely that his primary care physician will glucose in brain-injured patients. Injury. 1984;15(6):397-402.
10. Estes SM, Urban RJ. Hormonal replacement in patients with brain injury-
feel comfortable providing medical management of induced hypopituitarism: Who, when and how to treat? Pituitary.
what we believe is a chronic endocrine disorder. 2005;8(3-4):267-270.

The Endocrine Society. Downloaded from press.endocrine.org by [${individualUser.displayName}] on 12 January 2017. at 12:19 For personal use only. No other uses without permission. . All rights reserved.
ENDO 2016 NEUROENDOCRINOLOGY AND PITUITARY 213

11. Burman P, Broman JE, Hetta J, et al. Quality of life in adults with growth 17. High WM Jr, Briones-Galang M, Clark JA, et al. Effect of growth hor-
hormone (GH) deficiency: Response to treatment with recombinant human mone replacement therapy on cognition after traumatic brain injury.
GH in a placebo-controlled 21-month trial. J Clin Endocrinol Metab. J Neurotrauma. 2010;27(9):1565-1575.
1995;80(12):3585-3590. 18. Mossberg KA, Masel BE, Gilkison CR, Urban RJ. Aerobic capacity and
12. Deijen JB, van der Veen EA. The influence of growth hormone (GH) growth hormone deficiency after traumatic brain injury. J Clin Endocrinol
deficiency and GH replacement on quality of life in GH-deficient patients. Metab. 2008;93(7):2581-2587.
J Endocrinol Invest. 1999;22(5 Suppl):127-136. 19. Kelly DF, McArthur DL, Levin H, et al. Neurobehavioral and quality of
13. Mitchell A, Steffenson N, Davenport K. Hypopituitarism due to traumatic
life changes associated with growth hormone insufficiency after compli-
brain injury: A case study. Crit Care Nurse. 1997;17:34-37, 40-42, 46-51;
cated mild, moderate, or severe traumatic brain injury. J Neurotrauma.
quiz 53-54.
14. Klein MJ. Post head injury endocrine complications. eMEd J. 2002. Ac- 2006;23(6):928-942.
cessed from: http://emedicine.medscape.com/article/326123-overview. 20. Urban RJ. Hypopituitarism after acute brain injury. Growth Horm IGF
15. Agha A, Phillips J, Thompson CJ. Hypopituitarism following traumatic Res. 2006;16 Suppl A:S25-S29.
brain injury (TBI). Br J Neurosurg. 2007;21(2):210-216. 21. Zgaljardic DJ, Guttikonda S, Grady JJ, et al. Serum IGF-1 concentrations
16. Bondanelli M, De Marinis L, Ambrosio MR, et al. Occurrence of in a sample of patients with traumatic brain injury as a diagnostic marker
pituitary dysfunction following traumatic brain injury. J Neurotrauma. of growth hormone secretory response to glucagon stimulation testing.
2004;21(6):685-696. Clin Endocrinol (Oxf). 2011;74(3):365-369.

The Endocrine Society. Downloaded from press.endocrine.org by [${individualUser.displayName}] on 12 January 2017. at 12:19 For personal use only. No other uses without permission. . All rights reserved.
214 ENDO 2016 MEET-THE-PROFESSOR CLINICAL CASE MANAGEMENT

Prolactinomas

M57 BARRIERS TO OPTIMAL PRACTICE


Presented, April 1 4, 2016 The need to evaluate for cardiac valve abnormalities in
patients taking greater than standard doses of
cabergoline.
Mark E. Molitch, MD. Northwestern University The need for careful followup of women with
Feinberg School of Medicine, Chicago, Illinois, E-mail: macroadenomas who wish to become pregnant.
molitch@northwestern.edu The need for magnetic resonance image (MRI) scanning
in most patients.
INTRODUCTION
Historical Overview LEARNING OBJECTIVES
In the late 1920s and early 1930s, a number of groups found As a result of participating in this session, learners should be able to:
that pituitary extracts could induce milk secretion. Riddle and Know which patients require echocardiographic
coworkers found that this substance, which they named prolac- monitoring when greater than standard doses of
tin (PRL), could be differentiated from the known growth- and cabergoline need to be used.
gonad-stimulating substances by showing that it stimulated Understand the relative risks of bromocriptine vs
milk production by guinea pig mammary glands and a milk- cabergoline in the woman with a prolactinoma who
like substance from the crop sacs of pigeons and doves, which wishes to become pregnant.
then served as bioassays (1). Over the ensuing years, PRL was Understand the relationship between tumor size and PRL
characterized and sequenced and specific assays were devel- levels.
oped. In 1970, Frantz and Kleinberg developed a sensitive in
vitro bioassay and, for the first time, were able to demonstrate STRATEGIES FOR DIAGNOSIS, THERAPY,
measurable PRL levels in women with puerperal and nonpuerperal AND/OR MANAGEMENT
galactorrhea (2). Shortly thereafter, bromocriptine was found to Once drug use, renal insufficiency, hypothyroidism, and preg-
inhibit PRL secretion and reverse these clinical symptoms. In the nancy have been excluded, patients with hyperprolactinemia
1990s cabergoline was found to be even more effective than must have pituitary imaging with MRI to characterize the
bromocriptine and the principal of primary medical manage- possible presence of a prolactinoma or other structural pathol-
ment of patients with prolactinomas rather than surgery became ogy (4-6). For patients with symptomatic prolactinomas, treat-
firmly established. ment is indicated to normalize hyperprolactinemia and de-
crease tumor size. Dopamine agonists have been the mainstay
SIGNIFICANCE OF THE CLINICAL PROBLEM of treatment. Greater than 95% of prolactinomas are microadenomas
Prolactinomas are the most frequent subtype of hormone- (diameter, 10 mm) (2-4). Studies examining the natural his-
secreting pituitary adenomas, with an estimated prevalence tory of untreated microprolactinomas have found that only
ranging from 100-775 per million (3). Although dopamine 5-7% progress to macroadenomas and the rest remain small
agonist therapy has become the standard treatment for patients (6). Thus, patients with microprolactinomas do not require
with prolactinomas, a number of special clinical situations have treatment to prevent tumor growth. In addition, patients who
arisen in which such treatment is not straightforward. One of are asymptomatic, eugonadal, and do not desire fertility usually
these is the patient who is resistant to standard doses of dopa- need no treatment and surgery is never indicated in such
mine agonists. A number of management alternatives can be patients (2-4). Hypogonadal women with microadenomas can
considered, including raising drug doses, using alternative be treated with either dopamine agonists or estrogens (if
medications, or surgery. Each of these alternatives has its fertility is not wanted) to alleviate symptoms, restore go-
benefits and risks and management strategies must be individu- nadal function, and prevent premature osteoporosis. Patients
alized. A second situation is the woman being treated with a with macroadenomas may require treatment to alleviate local
dopamine agonist who wishes to become pregnant. Issues here mass effects in addition to treating hyperprolactinemia and to
include the safety of the dopamine agonist for the developing correct hormonal deficits (4-6).
fetus, the risk of pregnancy-induced prolactinoma enlargement, In head-to-head randomized, prospective comparison studies
and the safety of postpartum nursing. A third issue is the (7-8), retrospective analyses (9) and general clinical experi-
differentiation of the patient with a nonfunctioning adenoma ence, cabergoline has been shown to be substantially more
and disinhibition hyperprolactinemia from a patient with a effective than bromocriptine in normalizing PRL levels and
prolactinoma. This differentiation is very important, given that reducing tumor size, with fewer adverse effects (6). Thus,
the modes of therapy are quite different. normalization of PRL and at least a 25% reduction in tumor

The Endocrine Society. Downloaded from press.endocrine.org by [${individualUser.displayName}] on 12 January 2017. at 12:19 For personal use only. No other uses without permission. . All rights reserved.
ENDO 2016 NEUROENDOCRINOLOGY AND PITUITARY 215

size can be expected in approximately two thirds of patients Discussion


with bromocriptine and more than 80% with cabergoline (4-6). Answer D: When the dose of cabergoline exceeds 2 mg/wk.
Surgical success rates are highly dependent upon the experi- When faced with a patient with dopamine agonist resistance,
ence and skill of the neurosurgeon as well as the size and inva- there are several options available. The first option is to switch
siveness of the tumor (6, 10) Experience from 53 surgical series to a dopamine agonist that has a greater efficacy rate. Studies
shows that initial normalization of PRL can be achieved in have shown that approximately 80% of patients resistant to
65-85% of patients with microadenomas with transsphenoidal bromocriptine respond to cabergoline (6). If the patient is
surgery (6). For macroadenomas this number is much lower, at already on cabergoline, it is unlikely for them to respond better
30-40% (6). Rates of recurrence after transsphenoidal surgery to bromocriptine, although that may be worth a try (16). The
increase as length of followup increases and are approximately second option is to gradually increase the dose in a stepwise
20% for both micro- and macroadenomas (6). Therefore, using a fashion, as long as there is a stepwise reduction in PRL levels.
normal PRL level as the criterion for long-term surgical cure, the Doses as high as 3-5 mg daily have been shown to be well
cure rate for microadenomas is approximately 62% and for tolerated in patients being treated for Parkinsons disease.
macroadenomas, 16% (6). Because of its greater efficacy and However, cardiac valvular disease has been found when using
safety compared with transsphenoidal surgery, dopamine such high doses in such patients (13). In patients with
agonist therapy is generally regarded as the initial treatment prolactinomas, valvular disease is not seen when conventional
of choice for almost all patients with prolactinomas. doses (2 mg/wk) are used (14, 15). But the dosing point for
Most patients with prolactinomas will respond to conven- cabergoline for which the risk for valvular lesions begins re-
tional doses of cabergoline (2 mg/wk). However, 15-20% mains unknown. Therefore, when the dose of cabergoline ex-
may require higher doses to achieve control (6, 9, 11, 12), and ceeds 2 mg per week, an echocardiogram should be performed
this is the favored approach as long as there are no adverse and then repeated yearly (Answer D). Given that it is not
effects. A 3-6 fold increased risk of cardiac valvular abnormali- known which patients will be resistant when starting cabergoline,
ties has been found in patients with Parkinsons disease who it is not cost effective to perform an echocardiogram in all patients
receive high doses of cabergoline (3-5 mg/d) (13). Fortunately, upon initiation of cabergoline (Answer B). And these valvular
multiple studies now show that when the dose is maintained lesions do not occur with bromocriptine (Answers A, C, and E).
less than 2 mg per week there is no such risk in prolactinoma
patients (14, 15). However, because the threshold dose for Case 2
causing valvular risk is unknown, echocardiograms should be A Fertile Woman Taking Cabergoline
carried out in the subset of patients exceeding a weekly dose of This 29-year-old woman with amenorrhea and galactorrhea is
2 mg. In patients who do not respond to an increase in taking cabergoline 1 mg twice weekly for control of her
cabergoline dose or who develop valvular lesions, surgery hyperprolactinemia and a 1.1 cm prolactinoma. Her PRL level has
would be an appropriate choice of treatment. normalized, her tumor has reduced in size by approximately 50%,
her menses have regularized, and her galactorrhea has resolved.
She has decided now that she wants to get pregnant.
CASES
Which of the following should be the next step in
Case 1
The Patient Resistant to Dopamine Agonists management?
This 18-year-old woman initially presented with headaches and A. Switch cabergoline to bromocriptine and continue
pubertal delay and was found to have a PRL of 3200 ng/mL (138 throughout pregnancy.
nmol/L) and a 3-cm sellar mass extending into both cavernous B. Switch cabergoline to bromocriptine and stop when a
sinuses and wrapping around both carotid arteries. She was ini- pregnancy test is positive.
tially begun on bromocriptine and the dose was gradually in- C. Continue cabergoline throughout the pregnancy.
creased to 10 mg per day as her PRL level decreased in a stepwise D. Continue cabergoline and stop when a pregnancy test
fashion to approximately 450 ng/mL. She was nauseated with this is positive.
high dose and would periodically stop the medication, restarting E. Stop the cabergoline treatment now in anticipation of
again at the same dose. She was then switched to cabergoline, the pregnancy.
dose gradually being increased to 1 mg per day as the PRL levels
decreased to 60 ng/mL and she had substantial tumor shrinkage. Discussion
In this patient, when should an echocardiogram have been Answer D: Continue cabergoline but stop when a pregnancy
performed? test is positive.
A. Upon initiation of bromocriptine Neither cabergoline nor bromocriptine should be continued
B. Upon initiation of cabergoline once the pregnancy is diagnosed (Answers A and C) so as to
C. When the dose of bromocriptine exceeded 7.5 mg/d limit the exposure of the developing fetus to either drug (17).
D. When the dose of cabergoline exceeded 2 mg/wk The dopamine agonist should be stopped once pregnancy is
E. At both C and D diagnosed (Answer B or D). The chance of clinically signifi-

The Endocrine Society. Downloaded from press.endocrine.org by [${individualUser.displayName}] on 12 January 2017. at 12:19 For personal use only. No other uses without permission. . All rights reserved.
216 ENDO 2016 MEET-THE-PROFESSOR CLINICAL CASE MANAGEMENT

cant tumor growth related to the pregnancy is approximately correct answer, not stopping treatment (Answer A). Because
24% and this is not high enough to warrant continuing drug. his cabergoline dosage has been low, an echocardiogram (An-
An exception might be made in a patient with a giant swer C) is not indicated in this patient. The PRL level has
prolactinoma (4 cm) in whom the risk of rapid tumor en- normalized with cabergoline at the current dosage, so there is
largement outweighs the risks of drug continuation. This is not no reason to switch to either bromocriptine (Answer E). Preg-
the case in this patient with a 1.1-cm tumor. When either of nancy could result in a prolactinoma enlargement and head-
these drugs is stopped within the first few weeks of pregnancy, aches, but it is very unlikely that this is a prolactinoma for the
the frequency of major malformations is not greater than would reasons discussed above and it is very rare for a nonfunctioning
be expected in the general population. Given that the risks are adenoma to enlarge during pregnancy (Answer D).
not increased for either drug, there is no justification for
switching from cabergoline to bromocriptine, which is gener- REFERENCES
ally less effective and may result in her losing the ability to 1. Riddle O, Bates WR, Dykshorn WS. The preparation, identification and
assay of prolactina hormone of the anterior pituitary. Am J Physiol.
ovulate. If the cabergoline were to be stopped now, her PRL
1933;105:191-216.
would likely increase and she would become anovulatory again 2. Frantz AG, Kleinberg DL. Prolactin: Evidence that it is separate from growth
and be unable to become pregnant (Answer E). hormone in human blood. Science. 1970 Nov 13;170(3959):745-747.
3. Daly AF, Tichomirowa MA, Beckers A. The epidemiology and genetics of
pituitary adenomas. Best Pract Res Clin Endocrinol Metab. 2009;23:
Case 3 543-554.
Distinguishing Between the Hyperprolactinemia From a 4. Casanueva FF, Molitch ME, Schlechte JA, et al. Guidelines of the Pituitary
Prolactinoma From Disinhibition Due to a Clinically Society for the diagnosis and management of prolactinomas. Clin
Endocrinol. 2006;65:265-273.
Nonfunctioning Adenoma 5. Melmed S, Casanueva FF, Hoffman AR, et al. Diagnosis and treatment of
A 27-year-old woman presented with decreased libido, amenor- hyperprolactinemia: An Endocrine Society Clinical Practice Guideline.
rhea, and galactorrhea and is found to have a PRL level of 58 J Clin Endocrinol Metab. 2011;96:273-288.
6. Gillam MP, Molitch ME, Lombardi G, Colao A. Advances in the treat-
ng/mL (2.5 nmol/L) and a 13-mm pituitary adenoma on MRI. Her
ment of prolactinomas. Endocrine Revs. 2006;27:485-534.
PRL level, libido, and menses normalize on cabergoline, 0.5 mg 7. Webster J, Piscitelli G, Polli A, Ferrari CI, Ismail I, Scanlon MF. A
twice weekly, and she has remained on this treatment for 2 years. comparison of cabergoline and bromocriptine in the treatment of
She has recently developed headaches over the past few months. hyperprolactinemic amenorrhea. Cabergoline Comparative Study Group.
N Engl J Med. 1994;331:904-909.
Her last menstrual period was 3 weeks ago. Her physical exami- 8. Pascal-Vigneron V, Weryha G, Bosc M, Leclere J. [Hyperprolactinemic
nation is normal and there is no expressible galactorrhea. amenorrhea: treatment with cabergoline versus bromocriptine. Results of a
Which of the following would be the best next step in national multicenter randomized double-blind study]. [Article in French]
Presse Med. 1995;24:753-757.
management step? 9. Di Sarno A, Landi ML, Cappabianca P, et al. Resistance to cabergoline as
A. Stop the cabergoline compared with bromocriptine in hyperprolactinemia: prevalence, clinical
B. Perform a pituitary-directed MRI definition, and therapeutic strategy. J Clin Endocrinol Metab. 2001;86:
5256-5261.
C. Perform an echocardiogram 10. Swearingen B. Update on pituitary surgery. J Clin Endocrinol Metab.
D. Perform a pregnancy test 2012;97:1073-1081.
E. Switch the cabergoline to bromocriptine 11. Ono M, Miki N, Kawamata T, et al. Prospective study of high-dose
cabergoline treatment of prolactinomas in 150 patients. J Clin Endocrinol
Metab. 2008;93:4721-4727.
Discussion 12. Delgrange E, Daems T, Verhelst J, Abs R, Maiter D. Characterization of
resistance to the prolactin-lowering effects of cabergoline in macropro-
Answer B: Perform a pituitary-directed MRI. lactinomas: A study in 122 patients. Eur J Endocrinol. 2009;160:
Generally, PRL levels parallel the size of the tumor. Of 747-752.
concern in this patient is the discrepancy between her PRL 13. Simonis G, Fuhrmann JT, Strasser RH. Meta-analysis of heart valve
abnormalities in Parkinsons disease patients treated with dopamine
level of only 58 ng/mL (2.5 nmol/L) and the size of the agonists. Mov Disord. 2007;22:1936-1942.
adenoma: 13 mm. This discrepancy could be due to inefficient 14. Valassi E, Klibanski A, Biller BM. Clinical Review: Potential cardiac
production of PRL by a prolactinoma, but it is more likely due valve effects of dopamine agonists in hyperprolactinemia. J Clin
Endocrinol Metab. 2010;95:1025-1033.
to stalk dysfunction caused by a nonfunctioning adenoma or
15. Molitch ME. Management of medically refractory prolactinoma. J Neurooncol.
some other mass lesion such as a meningioma or a Rathke cleft 2014;117:421-428.
cyst (4, 15, 18). Given this scenario, a dopamine agonist could 16. Iyer P, Molitch ME. Positive prolactin response to bromocriptine in two
patients with cabergoline resistant prolactinomas. Endocrine Pract. 2011;16:
well normalize PRL levels and correct libido and anovulation,
1-11.
but have no effect on the growth of a mass lesion that is not a 17. Molitch ME. Endocrinology in pregnancy: management of the pregnant
prolactinoma. Thus, in a patient with this type of discrepancy patient with a prolactinoma. Eur J Endocrinol. 2015;172:R205R213.
in PRL level and adenoma size, it is prudent to monitor tumor 18. Karavitaki N, Thanabalasingham G, Shore HC, et al. Do the limits of
serum prolactin in disconnection hyperprolactinaemia need re-definition?
size by MRI, as well as PRL levels, during treatment. There- A study of 226 patients with histologically verified non-functioning pitu-
fore, performing a pituitary-directed MRI (Answer B) is the itary macroadenoma. Clin Endocrinol (Oxf). 2006;65:524-529.

The Endocrine Society. Downloaded from press.endocrine.org by [${individualUser.displayName}] on 12 January 2017. at 12:19 For personal use only. No other uses without permission. . All rights reserved.
OBESITY AND LIPIDS

The Endocrine Society. Downloaded from press.endocrine.org by [${individualUser.displayName}] on 12 January 2017. at 12:19 For personal use only. No other uses without permission. . All rights reserved.
218 ENDO 2016 MEET-THE-PROFESSOR CLINICAL CASE MANAGEMENT

Bariatric Surgery Nutritional Management

M05 related comorbidity and thus would qualify medically for


Presented, April 1 4, 2016 bariatric surgery, and all of those with a BMI greater than 40
kg/m2 would qualify medically for surgery, there is a very
large at-risk population. In reality, due to patient preferences
William Troy Donahoo, MD. Division of Endocrinology, and insurance factors as well as surgical risk, a much smaller
Metabolism and Diabetes, Department of Endocrinology, number of people undergo bariatric surgery: data from the
Kaiser Permanente Colorado, University of American Society of Metabolic and Bariatric Surgery found
ColoradoDenver, Lafayette, Colorado 80026, E-mail: that there were approximately 179 000 bariatric surgeries in the
william.t.donahoo@kp.org United States in 2013 with 34% being RYGB, 14% GB, 42%
VSG, and 1% BPD-DS. There are guidelines available that
summarize the nutritional management of the bariatric surgery
HISTORICAL PERSPECTIVE
patient (5, 6); however, a recent study from an insurance
Historical Overview
The jejunoileal bypass was the first truly bariatric surgery, origi- database of more than 21 000 patients undergoing weight loss
nally performed in the 1950s. Due to the extensive malabsorption, surgery (with 84% having a RYGB procedure) showed that
this procedure was fraught with nutritional deficiencies including recommended presurgery laboratory testing for vitamin defi-
ciencies was less than 25% and by 12-month followup only
protein-calorie malnutrition and severe deficiencies in vitamins A
12% had been screened for vitamin D deficiency, 60% had
and D. To lessen these significant issues with jejunoileal bypass,
been screened for vitamin B12 deficiency, 47% had been
other malabsorptive surgeries including biliopancreatic diversion
screened for folate deficiency, and 49% for iron deficiency as
and duodenal switch (BPD-DS) have been developed. BPD-DS is
recommended (7). Thus, the overall goal of this presentation is
thought to provide superior weight loss but still has significant risk
to close the gap between recommendations and practice and to
of malnutrition due to malabsorption of fat-soluble vitamins and
help prepare the practitioner for the management of this rapidly
iron, and these procedures also can put the patient at increased risk
growing population of patients at risk for nutritional deficien-
of protein malnutrition. Since the 1960s, the gastric bypass has
cies, including deficiencies that were rarely seen in the past.
evolved to its present state with the incorporation of a Roux limb
into the Roux-en-Y gastric bypass (RYGB). Due to the minimal
malabsorptive component (exclusion of most of the stomach, the BARRIERS TO OPTIMAL PRACTICE
duodenum, and 70 100 cm of jejunum), severe nutritional defi- Patient adherence to supplement recommendations.
ciencies including protein malnutrition are much less common Time to adequately manage the patient.
although they can still occur. More recently, purely restrictive Cost of weight-loss surgery and relative lack of coverage.
procedures including gastric banding (GB) and gastroplasty Cost of laboratory monitoring.
(specifically, vertical sleeve gastrectomy [VSG]) have come
into favor, in part due to the minimal risk of malnutrition due LEARNING OBJECTIVES
to malabsorption. After participating in this session, learners should be able to:
Also worth noting in this perspective is what was perhaps Recognize the signs and symptoms of nutritional
the earliest surgical intervention for a metabolic condition: deficiencies after bariatric surgery.
ileal bypass was first described in the early 1960s as a treat- Recognize emergent complications following bariatric surgery.
ment for hypercholesterolemia (2). Interestingly, along with Determine the appropriate vitamins and nutritional
this earliest description, the risk of vitamin B12 deficiency is supplements needed for optimal health following
noted. However, no nutritional deficiencies were reported with bariatric surgery.
long-term followup of the Program on the Surgical Control of
the Hyperlipidemia (POSCH) trial, despite the outcomes in-
STRATEGIES FOR DIAGNOSIS, THERAPY,
cluding significant improvements in overall morbidity and
AND/OR MANAGEMENT
mortality from cardiovascular disease (3).
Recommended preoperative screening for nutrition disorders
includes screening for iron, vitamin B12 (with screening for
SIGNIFICANCE OF THE CLINICAL PROBLEM red blood cell folate, homocysteine, and methylmalonic acid
According to recent National Health and Nutrition Examina- (MMA), optional), folic acid, and 25-OH vitamin D (5).
tion Surveys (NHANES) data, more than 8% of the adult U.S. Screening for vitamins A and E are considered optional. Be-
population has a body mass index (BMI) of 35 40 kg/m2 and cause liver function tests are also measured in screening, an
more than 6% have a BMI greater than 40 kg/m2 (4). Given albumin level is obtained. If a more extensive malabsorptive
that most the 8% with a BMI of 35 40 kg/m2 have a weight- procedure such as a BPD-DS is planned, then consideration for

The Endocrine Society. Downloaded from press.endocrine.org by [${individualUser.displayName}] on 12 January 2017. at 12:19 For personal use only. No other uses without permission. . All rights reserved.
ENDO 2016 OBESITY AND LIPIDS 219

FIGURE 1. Gastrointestinal Location of Nutrient Absorption.

more extensive baseline testing including the fat-soluble vita- Mineral Deficiencies
mins should be considered. Current evidence is insufficient for Mineral deficiencies that can be seen following bariatric surgery
performing preoperative screening with dual energy x-ray include iron deficiency, zinc deficiency, copper deficiency, and
absorptiometry and guidelines from other societies such as the selenium deficiency. Due to both decreases in gastric acid secre-
National Osteoporosis Foundation should be followed. tion because iron is primarily absorbed in the duodenum and
It is easiest to understand the nutrition deficiencies due to proximal jejunum, iron deficiency is not uncommon, especially in
malabsorption following bariatric surgery by reviewing where menstruating women. Iron deficiency occurs in stages from iron
the nutrients are absorbed and the effect of the various types of deficiency without anemia to iron deficiency with mild anemia to
weight-loss surgery. Figure 1 summarizes this concept. iron deficiency with severe anemia. Symptoms include fatigue and
Recommendations for postoperative nutrition management exercise intolerance as well as pica, weakness, headache, and
can be divided into recommendations for macronutrients (no- irritability. Symptoms can occur with low ferritin even without
tably protein) and vitamins, micronutrients, and minerals with
anemia. Initial treatment is with oral iron (along with vitamin C,
the latter categories stratified by surgery type. For macronutrients and
which forms a chelate with ferric iron at acid pH that remains
fluids, a staged meal progression is recommended with three
soluble at alkaline pH further down the gastrointestinal tract);
small meals a day. Goal protein intake should be at least 60 g
however, severe constipation is often a limiting adverse effect of
a day divided between the meals. To optimize protein intake,
oral iron. In these cases parenteral iron is indicated.
protein-rich foods should be the first eaten, and drinking other
fluids should be avoided for at least 30 minutes. Some evidence
suggests that a higher protein intake of 1.5 g/kg ideal body Copper
weight or even up to 2.1 g/kg ideal body weight might be Copper is absorbed in the stomach and proximal duodenum and thus
might be better (Tables 1 and 2). deficiencies can be seen with both RYGB and BPD-DS. Given that

The Endocrine Society. Downloaded from press.endocrine.org by [${individualUser.displayName}] on 12 January 2017. at 12:19 For personal use only. No other uses without permission. . All rights reserved.
220 ENDO 2016 MEET-THE-PROFESSOR CLINICAL CASE MANAGEMENT

TABLE 1. Screening for Postoperative Nutritional cause and effect are difficult to separate. In addition, zinc can
Deficiencies interfere with copper absorption, so copper deficiency should
GB/VSG RYGB BPD-DS be considered in patients taking higher doses of zinc without
DXA (at 2 y) separate copper intake. Treatment with 100 mg by mouth daily
24-h U Ca (6 mo then is usually sufficient to treat zinc deficiency.
annually)
Vitamin B12 (annually) Selenium
Folic acid Selenium is absorbed in the duodenum and proximal jejunum.
Iron studies including CBC Deficiency in selenium can cause skeletal muscle dysfunction
25-OH vitamin D and even cardiomyopathy as well as possibly mood disorders.
iPTH There is a narrow therapeutic window for selenium, so replace-
Vitamin A (initially then ?
ment at Dietary Reference Intakes (DRI) is recommended.
every 6-12 mo)
Copper, zinc, selenium
(with findings) Vitamin A
Thiamine (with findings) In addition to vitamin D insufficiency/deficiency, vitamin A
deficiency must also be considered following bariatric surgery.
Abbreviations: DXA, dual x-ray absorptiometry; iPTH, intact Vitamin A is absorbed in the jejunum and ileum, and thus,
PTH. patients following RYGB and BPD-DS are at risk of vitamin A
Table adapted from Mechanick et al (5).
deficiency. Deficiency is manifest by night blindness, xeropthalmia,
keratomalacia, Bilots spots, and hyperkeratosis. Treatment is
with high-dose vitamin A (eg, 10 000 IU/d, which is 200%
copper is needed for both white blood cell and red blood cell RDI [toxicity can occur at five times the RDI]).
formation, deficiencies can result in a microcytic anemia and
neutropenia. There is also a copper-deficiency myleoneuropathy
Vitamin B1
that can result in ataxia. Given that copper deficiency results in a
Vitamin B1 (thiamine) deficiency is uncommon but can present
microcytic anemia, it can often be confused with iron deficiency;
dramatically and can be devastating if not treated rapidly and
however, the symptoms associated with copper deficiency can
appropriately. Vitamin B1 is absorbed via a carrier transport
worsen with iron treatment. Copper deficiency is treated with 2.0
mechanism in the duodenum and proximal jejunum. Symptoms
mg of elemental copper per day, and the hematological abnormali-
of vitamin B1 deficiency can be seen as early as 3 weeks after
ties usually resolve within 2 months although the neurological
surgery if limited oral intake and nausea are present and have
abnormalities can persist.
even been reported with nonmalabsorptive procedures includ-
ing VSG. Nausea is a common symptom with thiamine defi-
Zinc ciency. Thiamine deficiency can present as Beriberi with
Zinc is absorbed in the duodenum and jejunum, so patients congestive heart failure (ie, wet), aphonia, peripheral neuropa-
with BPD-DS are at most risk for deficiency although deficien- thy, Wernicke encephalopathy (nystagmus, opthalmoplegia, and
cies have been reported in patients following RYGB. There is ataxia), confusion, and coma. Treatment for patients with symp-
an association between zinc deficiency and diarrhea, although toms includes iv thiamine, 100 mg followed by daily parenteral
thiamine of 100 mg until oral intake is adequate (5 d) when
TABLE 2. Recommended Nutritional Supplements by-mouth intake of 50 100 mg should be instituted.
Supplement GB/VSG RYGB BPD-DS
Multivitamins and minerals, No. 1 2 2 Protein
pills/d Finally, protein deficiency has been described, especially fol-
Calcium citrate, 12001500 mg/da lowing BPD-DS but also after RYGB with other complica-
Vitamin D, 3000 IU titrate to tions. Protein malnutrition should be considered with increased
30 ng/dL nutrition risk, (eg, a Nutrition Risk Screening Score [NRS] of
Vitamin B12 titrate to maintain 3) (5). The NRS 2002 takes into account nutrition status
normal levels (orally if
(weight change, BMI, intake during the last week) as well as
absorbed, otherwise parenteral)
Iron, 4560 mg/d via multivitamin ?
severity of disease and age (8). As always, parenteral nutrition
should be the first line of therapy; however, if a patient is not
a
Includes diet, specific amount not definitive. able to meet their nutrition needs using the gastrointestinal tract
Table adapted from Mechanick et al (5). for 57 days with a noncritical illness or 37 days with a
critical illness, then parenteral nutrition should be considered.

The Endocrine Society. Downloaded from press.endocrine.org by [${individualUser.displayName}] on 12 January 2017. at 12:19 For personal use only. No other uses without permission. . All rights reserved.
ENDO 2016 OBESITY AND LIPIDS 221

MAIN CONCLUSIONS Which of the following is the most likely evaluation to


A growing population of obese patients are undergoing identify the cause of this patients symptoms?
bariatric surgery including procedures with malabsorption such A. Calcium and PTH levels.
as the RYGB and BPD-DS. Knowing signs and symptoms of B. Complete blood count, ferritin, iron and total iron
associated nutritional deficiencies and appropriate screening binding capacity levels.
and management is essential to prevent serious sequelae. C. Zinc and copper level.
D. CBC, B12, and MMA levels.
E. Glucose and insulin levels.
CASES WITH QUESTIONS
Case 1
Ms. L is a 45-year-old female with a presurgical BMI of 45 DISCUSSION OF CASES AND ANSWERS
kg/m2 and presurgical comorbidities including polycystic ovar- Case 1
ian syndrome, sleep apnea (using CPAP), hypertension (con- Learning Objective: Recognize the symptoms and understand the
trolled on ACE-I and diuretic), and prediabetes. She underwent treatment of thiamine deficiency following bariatric surgery.
a RYGB surgery without complications and was discharged the This patient has classic signs and symptoms of thiamine
day after surgery. She missed her 1-month follow-up appoint- deficiency following bariatric surgery including nausea, confu-
ment but called in to say she was happy with her weight loss sion, nystagmus, and ataxia. C is the correct answer: treatment
but thought she had the stomach flu and did not want to travel is with iv thiamine followed by parenteral then oral thiamine.
to the clinic. You are called from the emergency department 4 Her symptoms are not consistent with hypoglycemia following
weeks later (2 mo after surgery) and notified that Ms. L has bariatric surgery, and glucose is contraindicated to avoid pre-
been brought in by her family with nausea, confusion, nystag- cipitating (worsening) Wernicke encephalopathy.
mus, ataxia, and double vision. She has lost 35 lbs (BMI 39
kg/m2), and has continued to be nauseous. Case 2
In addition to establishing iv access, which of the following Learning Objective: Understand the etiologies of anemia fol-
would be the best first step to advise the emergency physician lowing bariatric surgery including the risk of copper deficiency
to initiate? mistaken for iron deficiency.
A. Administer 12 Amp of D50 iv bolus. This patient likely underwent a BPD-DS surgery and has cop-
B. Administer 1 Amp of calcium gluconate iv bolus. per deficiency and perhaps iron deficiency as well so E is the
C. Start 100 mg of thiamine by iv infusion. correct answer. Copper deficiency is a microcytic anemia as is
D. Administer 1000 mcg cyanocobalamine im injection. iron deficiency, and with the pica it is likely she also had iron
E. Administer hydrocortisone, 100 mg iv bolus. deficiency that was treated with the additional oral iron; however,
the addition of zinc interfered with copper uptake, thus resulting in
a persistent anemia that was worsened by the additional iron.
Case 2
Ms G. is a 27-year-old female, new to your practice and who
has only been in the United States for 3 months, who presents REFERENCES
1. American Society for Metabolic and Bariatric Surgery (ASMBS). Story of Obesity
for monitoring and management 2 years after weight-loss sur-
Surgery. Accessed from: https://asmbs.org/resources/story-of-obesity-surgery.
gery in Mexico. She is not sure which surgery was performed 2. Buchwald H, Varco RL. Ileal bypass in lowering high cholesterol levels.
but states it was the best. She says she weighed 100 kg prior Surg Forum. 1964;15:289-291.
3. Buchwald H, Varco RL, Boen JR, et al. Effective lipid modification by
to the surgery (BMI 46 kg/m2 based on current height) and
partial ileal bypass reduced long-term coronary heart disease mortality and
had diabetes that was controlled with metformin. She also morbidity: Five-year posttrial follow-up report from the POSCH. Program
states she had not had menstrual cycles for approximately 5 on the surgical control of the hyperlipidemias. Arch Intern Med.
years prior to her surgery but they have started again during the 1998;158(11):1253-1261.
4. Ogden CL, Carroll MD, Kit BK, Flegal KM. Prevalence of childhood and
last year and have been heavy. Her current weight is 50 kg adult obesity in the United States, 2011-2012. JAMA. 2014;311(8):806-814.
(BMI 23 kg/m2) and her other vital signs are stable except 5. Mechanick JI, Youdim A, Jones DB, et al. Clinical practice guidelines for
for the suggestion of orthostatic hypotension. She is happy with the perioperative nutritional, metabolic, and nonsurgical support of the bariatric
surgery patient2013 update: Cosponsored by American Association of Clini-
her weight loss but complains of a lot of fatigue. She states she cal Endocrinologists, the Obesity Society, and American Society for Metabolic
was told in Mexico she should take more iron because her & Bariatric Surgery. Surg Obes Relat Dis. 2013;9:159 191.
blood count was low and so she has been trying to eat more red 6. Heber D, Greenway FL, Kaplan LM, et al. Endocrine and Nutritional manage-
ment of the post-bariatric surgery patient: An Endocrine Society Clinical Prac-
meat and take an iron pill once or twice a day as well as taking
tice Guideline. J Clin Endocrinol Metab. 2010;95(11):4823-4843.
extra zinc to prevent colds, but it seems her symptoms have not 7. Gudzune KA, Huizinga MM, Chang HY, Asamoah V, Gadgil M, Clark JM.
improved and have perhaps worsened. On examination you Screening and diagnosis of micronutrient deficiencies before and after
note pale sclera and several broken teeth. The patient states bariatric surgery. Obes Surg. 2013;23(10):1581-1589.
8. Kondrup J, Rasmussen HH, Hamberg O, Stanga Z. Nutritional risk screen-
these dental changes are new since her surgery, probably be- ing (NRS 2002): A new method based on an analysis of controlled clinical
cause she has been craving ice. trials. Clin Nutr. 2003;22:321336.

The Endocrine Society. Downloaded from press.endocrine.org by [${individualUser.displayName}] on 12 January 2017. at 12:19 For personal use only. No other uses without permission. . All rights reserved.
222 ENDO 2016 MEET-THE-PROFESSOR CLINICAL CASE MANAGEMENT

Diet and Exercise Recommendations in Metabolic Disease:


Implementing Effective Behavior Change

M17 real-world settings (22). Subsequent guidelines that specifi-


Presented, April 1 4, 2016 cally addressed behavioral methodology for metabolic
disease management include the American College of
Cardiology/American Heart Association/The Obesity Society
Marie E. McDonnell, MD. Department of Medicine and Task Force on Practice Guidelines (2014) (23). In summary,
Endocrinology, Brigham and Womens Hospital, Boston, during the past 15 years, expert opinion and guidelines based
Massachusetts 02115, E-mail: mmcdonnell@bwh.harvard. on studies on select patient populations have become highly
edu accessible for clinicians, students, and patients.
However, the implementation of published strategies to con-
trol lifestyle-related disease has been challenging. Many
INTRODUCTION
Historical Overview studies are investigating whether modern advances in technol-
Modern behavioral and lifestyle interventions for metabolic ogy and genetics can provide direction. Although it remains
diseases were largely untested until the 20th century. By the unknown how to implement effective strategies on regional,
first few years of the 21st century, we had solid evidence that national, and international scales to change this trajectory, what
behavioral change including increased exercise and attention to seems likely is that multiple approaches will be necessary.
diet could lead to weight loss and prevent diabetes.
Prior to this, data to support the clinical benefits of SIGNIFICANCE OF THE CLINICAL PROBLEM
exercise were limited. It was not until 1998 that the first Obesity is predicted to increase 42% by 2030 in the United
clinical guideline for treating overweight and obesity in States, and the increasing prevalence of excess adiposity and
adults was completed by the National Institutes of Health sedentary behavior across the world underlies this projection
(NIH) and the National Heart Lung and Blood Institute (1). The effect of these conditions is broad and reverberates
(NHLBI) (21). The charge to that 24-member panel was to throughout the life span of individuals and their communities.
not only cull the scientific literature to derive evidence Associated conditions (eg, diabetes, sleep apnea, arthritis, fatty
based recommendations but to also develop guidelines that liver, and overall disability), lead to reduced quality of life,
would be appropriate for the practicing physician and other productivity, and longevity. Interruption of this cycle of disease
health care providers dealing with overweight patients. requires both individual and population-level behavior change,
Some accomplishments of the NIH-NHLBI guidelines in- perhaps made most evident by the fact that gestational diabetes
cluded an agreed to definition for obesity using the body in a woman increases her offsprings risk of subsequently
mass index (BMI), a treatment algorithm that could direct developing metabolic disease as an adult.
those in primary care as to when the various types of Are we prepared for the challenge? Despite the availability of
treatment should be considered, and some practical weight guidelines and statistics, physicians in training frequently fail to
loss goals and strategies were developed. Following this first recognize obesity, are unfamiliar with treatment options, and
trailblazing guideline, others followed with the central goal spend relatively little clinical time treating obesity (2, 3). Physi-
to establish obesity as a chronic metabolic disorder that cians often do not counsel patients about their weight (4). When
exists to a large degree on a continuum that includes dyslip- counseling does occur, most physicians do not discuss specific
idemia, prediabetes, and type 2 diabetes. Between 1997 and behavior change recommendations such as diet and exercise. Lack
2002, three major studies (the Da Qing study, the Finnish of training and competency in obesity management are among the
Diabetes prevention study, and the Diabetes Prevention Pro- reasons that physicians do not adequately counsel patients.
gram) were published showing for the first time that diabetes
was preventable through behavioral strategies. Just a year BARRIERS TO OPTIMAL PRACTICE
later in 2003, researchers from the United States Centers for Evidence-Based and Accessible Interventions
Disease Control and Prevention (CDC) published and One clear barrier is the lack of availability of evidence-based
widely publicized the sobering statistic that one of three and accessible interventions that can actually lead to behavior
children born in the United States after 2000 will develop change in an individual and a population. It takes years, often
diabetes by age 50 years (1). The CDC then took steps to decades, for national or universal change when it comes to
develop a nationally supported diabetes prevention program clinical standards, due in part to the time required to generate
with procedures largely based on the Diabetes Prevention solid outcome data. Many countries are working to solve this
Program (DPP) trial, a clinical trial that tested the effects of problem by offering DPP-like programs on a national scale and
a lifestyle intervention for people at high risk for diabetes in incentivizing health insurers to offer them at discounts or

The Endocrine Society. Downloaded from press.endocrine.org by [${individualUser.displayName}] on 12 January 2017. at 12:19 For personal use only. No other uses without permission. . All rights reserved.
ENDO 2016 OBESITY AND LIPIDS 223

without cost. In the United States, for example, there is the List important unknowns in the field.
national diabetes prevention program through the CDC. Pro-
grams must meet the criteria described in the Standard Operat-
STRATEGIES FOR DIAGNOSIS, THERAPY,
ing Procedure Manual, available online: http://www.cdc.gov/
AND/OR MANAGEMENT
diabetes/prevention/pdf/dprp-standards.pdf. There is now a large body of evidence that supports the clinical
goal of 35% weight loss to achieve significant clinical out-
Over-Reliance on the Availability of Specific comes (712). This modest degree of weight loss can lead to
Evidence-Based Strategies the following clinical outcomes:
In contrast, over-reliance on the availability of specific Prevention of type 2 diabetes mellitus
evidence-based strategies can also lead to clinical inertia. In Increased life expectancy.
deciding on approaches to implement in clinical care, we can Improved glycemic control.
risk focusing on one based on few successful positive studies. Lower blood pressure.
If the approach is inaccessible due to cost, the result may be to Improved serum lipid profile.
keep the status quo. Published interventions intended to pre- Decreased cancer risk? (likely, but not yet well
vent diabetes have incorporated various levels and combina- established).
tions of lifestyle/health coaching, in-person interaction, and Improved lower back pain, gastroesophageal reflux,
technology-enhanced tools. These various models differ in lower extremity arthralgias, and sleep apnea reduce need
reach, cost, and effectiveness. Interestingly, a meta-analysis of for medications for obesity-related conditions.
community DPP-type programs showed what others have also
found, which is that consistent attendance was more critical The DPP model is the most evidence-based at this time (13).
than other important programmatic details. With every addi- In the DPP, participants received instruction in diet, exercise,
tional lifestyle session attended, weight loss increased by and behavior modification. Case managers met with each par-
0.26%. They concluded that costs associated with diabetes ticipant for at least 16 sessions in the first 24 weeks and then at
prevention can be lowered without sacrificing effectiveness, least every other month individually or in groups. Participants
using nonmedical personnel, and motivating higher attendance were asked to lower fat to less than 25% of caloric intake. If
at program sessions (5). reducing fat did not result in weight loss, a calorie goal was
added. Participants received culturally sensitive instruction in
diet, exercise, self monitoring, goal setting, and problem solv-
Cost
ing. In the DPP over an average of 3.2 years of followup, there
Cost is a major barrier, which includes both out-of-pocket costs
was a 16% mean reduction in diabetes risk per kilogram weight
to patients, missed time from work, and time required by
loss. The Look AHEAD employed a DPP model for the inter-
clinicians to counsel patients. Clinical trials are in process to
vention compared with diabetes support education in individu-
test the value of Internet-based interventions and specific meth-
als with type 2 diabetes who had increased cardiovascular risk
odology (eg, multiple behavior targets vs single ones). Thus
(14). In the Look AHEAD roughly one third of the intensive
far, reports are mixed on specific elements that lead to success,
lifestyle intervention subjects lost 10% or more in the first year,
but it seems that approaches ought to be tailored to the indi-
one third lost 5% up to 10%, and one third lost less than 5%. At
vidual patient (6). It seems that in comparison with in-person
year 4, 70% of those who did very well in year 1 maintained
interventions, Internet-based approaches paradoxically reach
more than 5% loss, whereas only 22% of those who did poorly
fewer enrolled subjects over time due to dropout, yet the
in year 1 had achieved more than 5% loss at year 4. Further
potential population effect is much greater given the expansive
analysis confirmed that early success with behavior modifica-
reach of the Internet and lower costs. Time is money, it is
tion predicted long-term success.
said, and the lack of time and expertise in the clinical encounter
Practical guidelines for prescribing impactful behavioral
can prevent efficient and effective advice to patients.
change for overweight and obese individuals have been put
forth by some guidelines, for example, those from the 2013
LEARNING OBJECTIVES American College of Cardiology (ACC)/American Heart Asso-
As a result of participating in this session, learners should be ciation (AHA)/The Obesity Society (TOS) guideline (Table 1).
able to: If diet change is demonstrated to be more difficult in an
Know the benefit of exercise and dietary behavior individual, regular physical activity has a benefit independent
change in diabetes prevention. of diet. The 2010 position statement on exercise and type 2
Prescribe an evidence-based exercise and nutrition diabetes from the American College of Sports Medicine and
program to a patient with prediabetes and diabetes who the American Diabetes Association provides a useful summary
is overweight or obese. of the evidence. Prospective cohort and cross-sectional obser-
Describe unique issues that are amenable to evidence- vational studies that assessed exercise with questionnaires
based advice, namely, sleep hygiene. showed that higher physical activity levels are associated with

The Endocrine Society. Downloaded from press.endocrine.org by [${individualUser.displayName}] on 12 January 2017. at 12:19 For personal use only. No other uses without permission. . All rights reserved.
224 ENDO 2016 MEET-THE-PROFESSOR CLINICAL CASE MANAGEMENT

TABLE 1. Practice Advice difficult to predict, but no doubt that with our learning that
Use BMI to identify risk; advise patients of their risk genetic studies support the hypothesis that some benefit
Use waist circumference to identify risk; advise patients of their more than others from the metabolic effects of exercise, the
risk AHA, ACC, TOS, and appropriate partners must address the
Small sustained weight loss of 35% body weight reduces risk of questions that remain about how to translate behavioral
diabetes and improves cardiovascular risk factors. Greater change to clinical practice. For example, primary providers
benefits seen with 510% loss
should be provided with success rates of available obesity
Prescribe set number of calories per day
treatment programs. Reimbursement for successful treat-
There is no ideal diet, and adherence to a reduced calorie diet is
the most important goal ment programs, primary care physicians, and specialists has
Advise a program with in-person participation of 6 mo improved recently but require consistent support.
Electronic and commercial programs can aid in weight loss Finally, there is a growing body of evidence to suggest that
Regular contact is best for long-term maintenance not all individuals respond as robustly to particular behavioral
For weight maintenance after loss: Engage in high levels of interventions, specifically exercise, in terms of metabolic re-
physical activity (i.e., 200300 min/wk); Monitor body weight sponses in muscle. This may translate to some individuals
regularly; Consume a reduced-calorie diet preference to exercise vs to not, although this remains unclear
(17). Although it is unknown which genotypes lend themselves
Adapted from the 2013 ACC/AHA/TOS Guideline (16).
better to the exercise effect, results from recent collaborations
(eg, MAGIC), have shown that comprehensive lifestyle change
can delay the progression of diabetes regardless of genetic
underpinnings (18).
reduced risk for type 2 diabetes, regardless of method of
activity assessment, ranges of activity categories, and statistical
methods (16). Physical activity improves glucose uptake into Other Behavioral Factors
active muscles, which is the major beneficial effect in those Sleep
with diabetes. At rest, people with classical type 2 diabetes A growing body of evidence supports a functional correlation
have an impaired insulin-stimulated glucose uptake, but mus- between sleep restriction (shorter duration and/or poor quality)
cular contraction facilitates glucose transport by a separate and metabolic disorders, particularly obesity and insulin resis-
mechanism that is not impaired by insulin resistance. A com- tance. Epidemiological studies have shown increased risk of
bination of aerobic exercise and resistance training may be weight gain and insulin resistance in shift workers. Studies in
more effective in improving glucose control then either alone. humans have shown that sleep deprivation in a laboratory
However, more studies are needed to understand why this setting causes metabolic deficits characteristic of a prediabetic
might be. For optimal results in type 2 diabetes and for a state within only 10 days of exposure. In real-world studies
consistent effect, exercise events should not be separated by rotating shift-work duration positively correlates with a trend
more than 2 days, given that it has been shown that physical of increase in BMI and development of type 2 diabetes. Inter-
activity can improve insulin sensitivity for a maximum of 72 estingly, short-term sleep deprivation can temporarily increase
hours. For those individuals with neuropathy who do not have energy expenditure, but over time individuals may increase
an acute foot ulceration, moderate intensity weight bearing calorie intake and in the end be prone to weight gain and
exercise is recommended. Moderate walking does not increase obesity (19).
risk of foot ulcers or reulceration as long as there is proper
attention to foot care including daily self inspection and proper
footware.
MAIN CONCLUSIONS
Prescriptions for exercise and diet to target 35% weight
reduction can yield substantial improvement in metabolic
Practice Advice
parameters linked to morbidity and mortality.
New approaches and tools have emerged. Chronic disease
management platforms have emerged from several indus- Best evidence for successful strategies is for in-person,
tries, encompassing a spectrum that includes not only well- comprehensive programs that include group activity and
established weight loss programs, but also pharmaceutical have a duration of at least 6 months.
companies, nutrition supplement companies, and device Other behavioral factors may be critical to address in the
companies. Navigating the benefits of these online programs individual, specifically sleep. Altered circadian rhythm
on behavior modification is challenging, at best. Well- over time leads to increased metabolic disease risk.
designed studies in diabetes have failed to show a unique Two major unknowns will likely lead to changes in the
benefit of remote behavioral modification. More recently, field of behavior change in medicine, namely, whether
Health Coaching has emerged as a promising method, but as the genotype may allow for greater or lesser benefit from
with other methods, cost must be addressed. The future is specific behavior change, and the feasibility and success

The Endocrine Society. Downloaded from press.endocrine.org by [${individualUser.displayName}] on 12 January 2017. at 12:19 For personal use only. No other uses without permission. . All rights reserved.
ENDO 2016 OBESITY AND LIPIDS 225

of remote, Internet-based health behavior modification C. You should consider a high-protein, low-carbohydrate
tools. diet as it has been proven to be superior to other types
of diets in the long run.
D. You could just lift weights until your schedule settles
CASES WITH QUESTIONS
down. Resistance training three times a week is known
Case 1
to prevent diabetes.
You have recommended that your new patient with type 2
diabetes and mild peripheral neuropathy begin a scheduled
Case 3
exercise program that includes 2 and 12 hours per week of
A provider in your group announces that they are seeking
aerobic exercise. She asks you about the most bang for her
institutional support for developing and implementing an
buck when it comes to the type of exercise, stating that there
evidence-based behavioral change intervention for patients
are no athletes in my family, and my sisters and I always
who are overweight or obese with prediabetes.
thought that people who report a runners high were from You advise the following method has been shown to be most
another planet. effective:
Which of the following advice statements is most supported A. Mobile phone based behavior advice and tracking.
by evidence? B. Community health coach.
A. Individuals with diabetes do not get as much out of C. Virtual reality Televisits.
exercise because they have trouble moving sugar D. A comprehensive weight loss program that includes
(glucose) into their muscle cells. So it is probably individual or group in-person sessions that lasts for 6
better just to keep relatively active in your life without months and continues to follow engaged individuals
a major focus on exercise. over time for maintenance.
B. A combination of aerobic exercise and resistance
training may be more effective in improving glucose Case 4
control then either alone. It is not clear which you Your patient asks, My husband lost 20 pounds on the Paleo
should emphasize more, but mixing it up may help you Diet. Will this work for me?
stick to it, and will probably be most beneficial. Best evidence suggests you give the following response.
C. I recommend that you stick to the Nautilus. Individuals A. Yes, you should try it for at least 2 weeks.
with peripheral neuropathy should not do weight- B. Yes, it could work for you as long as you follow it.
bearing exercise as it could increase their risk of foot Please share some details and tell me your strategy.
ulceration. C. No. You must count calories.
D. It is a known fact that the runners high is genetic. D. Can we discuss this at our next visit?
You probably do not have the gene.
DISCUSSION OF CASES AND ANSWERS
Case 2
A 48-year-old nurse with hypertension and prediabetes (HA1c Case 1
Answer: B. This individual has mild neuropathy and early type
last 2 years, 5.8 and 6.2) and a BMI of 30 kg/m2 asks you
2 diabetes. Although it is possible she has a genetic tendency
about his next move for preventing diabetes. He has been part
toward a different physiologic effect of exercise, this evidence
of a commercial exercise and lifestyle program at the YMCA
is limited at this time. What is known, however, is that exercise
for the last 3 months and has lost approximately 5% of his
is very likely to improve her overall glycemic profile and
body weight. He reports stress around moving jobs. You dis-
nonglycemic disease outcomes (eg, blood pressure), and can
cover that he plans to take overnight shifts for the higher
improve glucose disposal substantially beyond what is attain-
hourly salary and he does not know whether he will be able to able for the patient with type 2 diabetes at rest. Although the
get to the gym. reasons are not yet clear, it does seem that aerobic exercise and
Which of the following advice statements below are based resistance training have synergistic effects in the patient with
on available evidence? diabetes, so a combination of these is ideal to recommend for
A. Night-shift work over time will likely increase your her workout (17).
risk of developing overt diabetes. I would strongly
advise you to avoid this if you can. Perhaps limit to no Case 2
more than one shift per month. Answer: A. The only factual statement is answer A. Altered
B. Lets consider pharmacologic treatment of the circadian rhythm causes insulin resistance in animals, particu-
prediabetes, as it will offer the same protection against larly if repeated and continued over time. Answer B is unsup-
progression of the diabetes as your work in the gym. ported by the DPP, which showed a higher reduction in risk of

The Endocrine Society. Downloaded from press.endocrine.org by [${individualUser.displayName}] on 12 January 2017. at 12:19 For personal use only. No other uses without permission. . All rights reserved.
226 ENDO 2016 MEET-THE-PROFESSOR CLINICAL CASE MANAGEMENT

progression to diabetes in the group who received the intensive style interventions in real-world settings that were modeled on the Diabe-
tes Prevention Program? Health Aff (Millwood). 2012;31(1):67-75.
lifestyle intervention, compared with the metformin group.
6. Hamman RF, Wing RR, Edelstein SL, et al. Effect of weight loss with
Resistance training has not been shown to be effective as a lone lifestyle intervention on risk of diabetes. Diabetes Care. 2006;29(9):2102-2107.
strategy for diabetes prevention, so answer C is incorrect. 7. Lean ME, Powrie JK, Anderson AS, Garthwaite PH. Obesity, weight loss
There are no superior diets, as suggested by answer C (see and prognosis in type 2 diabetes. Diabet Med. 1990;7:228-233.
8. Wing RR, Koeske R, Epstein LH, Nowalk MP, Gooding W, Becker D.
Case 4). Long-term effects of modest weight loss in type II diabetic patients. Arch
Intern Med. 1987;147:1749-1753.
9. Schotte DE, Stunkard AJ. The effects of weight reduction on blood
Case 3 pressure in 301 obese patients. Arch Intern Med. 1990;150:1701-1704.
Answer: D. Outcome data from both the DPP and Look 10. Dattilo AM, Kris-Etherton PM. Effects of weight reduction on blood lipids
AHEAD have shown that modest weight loss is beneficial in and lipoproteins: a meta-analysis. Am J Clin Nutr. 1992;56:320-328.
reducing development of diabetes among high-risk individuals, 11. Bianchini F, Kaaks R, Vainio H. Weight control and physical activity in
cancer prevention. Obesity Reviews 2002;3:5-8.
and that modest weight loss in patients with diabetes reduces 12. Wadden TA, Neiberg RH, Wing RR, et al. Four-year weight losses in the
cardiovascular risk factors and improved glycemic control. Look AHEAD study: factors associated with long-term success. Obesity
Presumably, however this is achieved, the results will be simi- (Silver Spring). 2011;19(10):1987-1998.
13. Knowler WC, Barrett-Connor E, Fowler SE, et al. Reduction in the
lar. At the moment, however, the evidence is strongest for an incidence of type 2 diabetes with lifestyle intervention or metformin.
in-person, group-based, comprehensive behavior change inter- N Engl J Med. 2002;346:393-403.
vention that continues for 6 months and preferably longer. 14. Wing RR, Bolin P, Brancati F, et al. Cardiovascular effects of intensive
lifestyle intervention in type 2 diabetes. N Engl J Med. 2013;369:145-154.
Overall, community health coaching in primary care seems to 15. Jensen MD, Ryan DH, Apovian CM, et al. Cardiology/American Heart
lead to significant benefits for patients from lower- Association Task Force on Practice Guidelines; Obesity Society. 2013
socioeconomic status communities with poorly controlled type AHA/ACC/TOS guideline for the management of overweight and obesity
in adults: a report of the American College of Cardiology/American
2 diabetes, such as to lower HA1c, but this method has not
HeartAssociation Task Force on Practice Guidelines and The Obesity
been shown effective for the overweight or obese patient with Society. J Am Coll Cardiol. 2014;1;63(25 Pt B):2985-3023.
type 2 diabetes. 16. Sheri R. Colberg, Ronald J. Sigal, et al. Exercise and type 2 diabetes: The
American College of Sports Medicine and the American Diabetes Asso-
ciation: joint position statement. Diabetes Care. 2010;33(12):e147-e167.
Case 4 17. Sparks LM, Johannsen NM, Church TS, et al. Nine months of combined
Answer: B. There seems to be no ideal diet for weight loss and training improves ex vivo skeletal muscle metabolism in individuals with
no superiority for any of the myriad diets reviewed. Choice of type 2 diabetes. J Clin Endocrinol Metab. 2013;98(4):1694-702.
18. Hivert MF, Christophi CA, Franks PW, et al. Lifestyle and metformin
diet composition should be made considering patients prefer- ameliorate insulin sensitivity independently of the genetic burden of estab-
ences and health status. In the POUNDS Lost study (20), there lished insulin resistance variants in Diabetes Prevention Program partici-
was no change in amount of weight loss whether on high vs pants. Diabetes. 2015;db150950. [Epub ahead of print].
19. Nohara K, Yoo SH, Chen ZJ. Manipulating the circadian and sleep cycles
low carbohydrate, fat, or protein. This was true at 6, 12, 18, to protect against metabolic disease. Front Endocrinol (Lausanne).
and, shown here, 24 months. 2015;6:3523.
20. Sacks FM, Bray GA, Carey VJ, et al. Comparison of weight-loss diets with
different compositions of fat, protein, and carbohydrates. N Engl J Med.
REFERENCES 2009;360(9):859-873.
1. Narayan KM, Boyle JP, Thompson TJ, Sorensen SW, Williamson DF. 21. National Institutes of Health. Clinical guidelines on the identification,
Lifetime risk for diabetes mellitus in the United States. JAMA. evaluation, and treatment of overweight and obesity in adultsthe evi-
2003;290(14):1884-1890. dence report [erratum in Obes Res. 1998;6:464]. Obes Res. 1998;6 Suppl
2. Ruser CB, Sanders L, Brescia GR, et al. Identification and management of 2:51S209S.
overweight and obesity by internal medicine residents. J Gen Intern Med. 22. The Diabetes Prevention Program Research Group. Reduction in the inci-
2005;20:1139-1141. dence of type 2 diabetes with lifestyle intervention or metformin. N Engl
3. Block JP, DeSalvo KB, Fisher WP. Are physicians equipped to address the J Med. 2002;346:393-403.
obesity epidemic? Knowledge and attitudes of internal medicine residents. 23. American College of Cardiology/American Heart Association Task Force
Prev Med. 2003;36:669-675. on Practice Guidelines; Obesity Society. 2013 AHA/ACC/TOS guideline
4. Jay M, Schlair S, Caldwell R, Kalet A, Sherman S, Gillespie C. From the for the management of overweight and obesity in adults: a report of the
patients perspective: The impact of training on resident physicians obe- American College of Cardiology/American Heart Association Task Force
sity counseling. J Gen Intern Med. 2010;25(5):415-422. on Practice Guidelines and The Obesity Society. J Am Coll Cardiol.
5. Ali MK, Echouffo-Tcheugui J, Williamson DF. How effective were life- 2014;63:2985-3023.

The Endocrine Society. Downloaded from press.endocrine.org by [${individualUser.displayName}] on 12 January 2017. at 12:19 For personal use only. No other uses without permission. . All rights reserved.
ENDO 2016 OBESITY AND LIPIDS 227

Post-Bariatric Surgery Hypoglycemia

M18 easier to define if the pathogenesis was understood. However,


Presented, April 1 4, 2016 pathogenesis has not been established. In the original report,
Service and coinvestigators suggested that patients with this
disorder developed hyperinsulinemia as a result of islet cell
John P. Bantle, MD. Division of Endocrinology and hyperplasia, perhaps due to an increase in glucagon like
Diabetes, Department of Medicine, University of peptide-one (GLP-1) secretion (2). Postprandial serum insulin
Minnesota, Minneapolis, Minnesota 55455, E-mail: and GLP-1 responses are known to increase after gastric by-
bantl001@umn.edu pass surgery (11-13) and GLP-1 has been shown to increase
-cell mass in rodent models (14, 15). Several investigators in
INTRODUCTION addition to Service have suggested that -cell mass is increased
Historical Overview after gastric bypass (2-4). However, this has not been a univer-
Postbariatric surgery hypoglycemia is not a new phenomenon and sal finding (8). Moreover, Meier and colleagues (16) reexam-
was mentioned in surgical reports more than 30 years ago (1). ined the pancreatic tissues of the patients in Services original
However, a more-detailed description of its potential significance report and found no evidence of increased -cell mass. They
occurred only 11 years ago. In 2005, Service and colleagues (2) suggested that postprandial hypoglycemia after gastric bypass
first described postprandial hypoglycemia causing neruoglyco- was due to the combination of gastric dumping and increased
penia after Roux-en-Y gastric bypass surgery. Several of their insulin secretion. Consistent with this, McLaughlin (17) and
patients experienced confusion and loss of consciousness during colleagues described a unique patient with this disorder. The
episodes. Subsequently, additional patients with this disorder have patient had a gastrostomy tube inserted into her remnant stom-
been described in small series and case reports (3-9).
ach. When she was given a standardized liquid test meal orally,
she developed postprandial hyperglycemia and hyperinsulin-
SIGNIFICANCE OF THE CLINICAL PROBLEM emia followed by hypoglycemia. However, when the standard-
The prevalence of postgastric bypass hyperinsulinemic hypo- ized test meal was given through the gastrostomy tube into her
glycemia is unknown but is at least 0.36% of those patients remnant stomach, no postprandial hyperinsulinemia or hypo-
having gastric bypass surgery (7). This is almost certainly an glycemia developed.
underestimate as it is likely that all patients with the syndrome Thus, it is not clear whether postgastric bypass hypoglyce-
were not identified. The syndrome seems to be specific for mia results from:
gastric bypass and does not occur after purely restrictive pro- 1. An increase in beta cell mass postgastric bypass,
cedures such as adjustable gastric banding (10). Clearly, the perhaps induced by increased GLP-1 or other beta cell
hypoglycemia produces major morbidity and potential mortal-
trophic peptides, and resultant postprandial
ity for those who have it.
hyperinsulinemia,
2. Rapid digestion and absorption of ingested nutrients
BARRIERS TO OPTIMAL PRACTICE coupled with failure to adaptively decrease insulin
The disorder is almost certainly underdiagnosed. secretion after gastric bypass-induced weight loss, or
Diagnostic criteria are not well defined. 3. Something else.
This is a difficult patient population to work with.
Treatment options are limited and often not effective.
Diagnostic criteria have not been clearly established
but we propose the following (6):
LEARNING OBJECTIVES Postprandial hypoglycemia with neuroglycopenia
At the end of the session, participants will be able to: developing 1 year after gastric bypass
Define methods for the diagnosis of postgastric bypass Spontaneous correction of hypoglycemia
hypoglycemia. Normal fasting glucose and serum insulin
Develop a rational approach to treatment. Hyperinsulinemia at the time of hypoglycemia or, after a
Know of research pathways to better treatment that are mixed meal, plasma glucose 50 mg/dL, and serum
being explored. insulin 50 uU/L.
The mixed meal we use consists of 8 oz orange juice, 6 oz
STRATEGIES FOR DIAGNOSIS, THERAPY, fat-free yogurt, one slice of bread with one teaspoonful of
AND/OR MANAGEMENT margarine, and two teaspoonsful of jam. Plasma glucose and
Both diagnostic methods and management strategies for serum insulin are sampled before and 15, 30, 45, 60, 90, 120,
postgastric bypass hyperinsulinemic hypoglycemia would be 180, and 240 minutes after the meal.

The Endocrine Society. Downloaded from press.endocrine.org by [${individualUser.displayName}] on 12 January 2017. at 12:19 For personal use only. No other uses without permission. . All rights reserved.
228 ENDO 2016 MEET-THE-PROFESSOR CLINICAL CASE MANAGEMENT

A variety of treatments for postgastric bypass hyperinsulinemic treatments were compared in random order using a crossover
hypoglycemia have been attempted. Service and colleagues (2) design. The treatments were a high-carbohydrate test meal
employed an arterial calcium stimulation test to localize exces- (control condition), a high-carbohydrate test meal after pre-
sive pancreatic insulin secretion and subsequent subtotal pan- treatment with rapid-acting lispro insulin (insulin condition),
creatic resection with improvement in symptoms in some pa- and a high-fructose test meal that was low in glucose and
tients. However, total pancreatectomy has been required to glucose-containing carbohydrates but with total carbohydrate
eliminate hypoglycemia in other patients (3) resulting in iatro- and caloric content similar to the control meal (fructose condi-
genic, insulin deficient diabetes mellitus. Gastric restriction by tion).
laparoscopic placement of a silastic ring around the gastric The hypotheses to be tested were:
pouch has been reported to be an effective treatment (8). 1. Pretreatment with lispro insulin would prevent, or at
Noninvasive treatments include low-carbohydrate diets (5, 6), least reduce, the occurrence of hypoglycemia, and
coingestion of the medication acarbose with meals containing 2. Substitution of fructose for glucose in the test meal
carbohydrate to retard carbohydrate digestion (18, 19), treat- would prevent, or at least reduce, the occurrence of
ment, in a single patient, with the somatostatin analog hypoglycemia.
pasireotide (20), and continuous infusion of the glucagon-like
peptide 1 receptor antagonist exendin 9-39 (21). The primary Study participants were 10 patients with postgastric
approach to treatment should be a low-carbohydrate diet. Hy- hyperinsulinemic hypoglycemia. For the control condition, sub-
poglycemia does not occur without the antecedent ingestion of jects received a high-carbohydrate test meal composed of or-
carbohydrate. In occasional patients, reversal of gastric bypass ange juice (8 oz), fat-free yogurt (6 oz), one piece of toasted
or pancreatectomy may need to be considered. The other ap- wheat bread, margarine (1 teapoonful) and jam (2 teaspoons-
proaches have not been consistently effective or are not gener- ful). Ten minutes before the meal, subjects received an sc
ally available. injection of saline. For the insulin condition, subjects received
Patients with postgastric bypass hyperinsulinemic hypogly- a test meal of the same composition and, 10 minutes before the
cemia demonstrated a greater-than-normal increase in plasma meal, sc injection of one unit of lispro insulin for every 15 kg
glucose after eating carbohydrate-containing meals (6, 7, 9). of body weight. For the fructose condition, subjects received a
This led to a robust insulin response, hyperinsulinemia, and test meal with similar amounts of carbohydrate, protein and fat
subsequent hypoglycemia even though insulin levels declined as in the control meal but with fructose as the principal carbo-
rapidly as plasma glucose decreased. Thus, it is plausible that hydrate. Plasma glucose and serum insulin were sampled 15
treatment that reduces the postprandial increase in plasma glu- minutes before and 15, 30, 45, 60, 75, 90, 105, 120, 135, 150,
cose after a carbohydrate-containing meal would also reduce 180, 210, and 240 minutes after all test meals. The primary
the insulin response and, thereby, the risk of hypoglycemia. study endpoint was occurrence or not of plasma glucose less
With this in mind, we studied two interventions intended to than 60 mg/dL during the 4 hours after the test meal. The
prevent postprandial hyperglycemia and subsequent hypogly- results are summarized in Table 1.
cemia (22). The first was a rapid-acting insulin analog given There were no differences among the three treatments in
before a high-carbohydrate meal. The second was replacement fasting values. After the high-carbohydrate control meal with
of glucose with fructose in a high-carbohydrate meal. Three premeal saline injection, eight of 10 subjects demonstrated a

TABLE 1: Fasting and Postprandial Test Values (Mean SD or n)


High Carb Meal With High Carb Meal With High Carb
Pre-Meal Saline Pre-Meal Insulin Fructose Meal
Injection (Control) Injection (Insulin) (Fructose)
Fasting plasma glucose, mg/dL 76 6 77 5 81 8
Fasting serum insulin, mU/liter 43 52 43
Peak postprandial plasma glucose, mg/dL 173 47 182 40 117 20
Peak postprandial serum insulin, mU/liter 134 55 148 68 45 31
Postprandial plasma glucose 60 mg/dL, n 8 (80%) 10 (100%) 2 (20%)*
Postprandial glucose nadir, mg/dL 44 15 34 10* 67 10
Time to glucose nadir, min 132 35 125 27 126 28

* P 0.05 when compared to the control meal.



P 0.001 when compared to the control meal.

The Endocrine Society. Downloaded from press.endocrine.org by [${individualUser.displayName}] on 12 January 2017. at 12:19 For personal use only. No other uses without permission. . All rights reserved.
ENDO 2016 OBESITY AND LIPIDS 229

plasma glucose nadir less than 60 mg/dL. Five subjects dem- A third episode occurred while on a hunting trip in Montana.
onstrated a nadir less than 40 mg/dL. The lowest plasma Again, he felt lightheaded and collapsed. He was taken to
glucose observed was 22 mg/dL. Despite the low plasma glu- hospital where he was evaluated and released. The most recent
cose values, testing was not stopped as no subject developed episode occurred after he had eaten banana bread, half a ham
evidence of neuroglycopenia during testing. The high- sandwich, and a snack cake at approximately 0900 hours.
carbohydrate meal with premeal insulin injection did not pre- Thirty minutes later, he collapsed at work and was taken to
vent hypoglycemia. All 10 subjects demonstrated a plasma hospital. No cause for his collapse was found. He had started
glucose nadir less than 60 mg/dL and the mean plasma glucose blood glucose monitoring and had numerous fasting blood
nadir was 34 mg/dL. After the fructose meal, only two subjects glucose readings of approximately 100 mg/dL. After seeing his
demonstrated a plasma glucose nadir less than 60 mg/dL and bariatric surgeon, he was referred to the Endocrinology Clinic
the mean plasma glucose nadir was 67 mg/dL. We concluded at the University of Minnesota.
that pretreatment with modest doses of rapid-acting insulin did
not prevent postprandial hyperglycemia, hyperinsulinemia, or
Clinic Visit
subsequent hypoglycemia; meals with fructose as the principal
Medications:
carbohydrate did not produce significant hypoglycemia; and
Lisinopril, 20 mg daily
people with this disorder have an extraordinary tolerance for
Vitamin B12, 500 mcg daily
hypoglycemia.
Multivitamin daily

MAIN CONCLUSIONS Examination:


Postgastric bypass hypoglycemia is probably more Blood pressure, 141/96 mm Hg
common than is generally appreciated. Pulse, 93 beats per minute
The best diagnostic method is a mixed-meal test with Height, 72 in
measurement of postprandial plasma glucose and serum Weight, 101 kg
insulin levels. BMI, 30.2 kg/m2
A low-carbohydrate diet, fructose-sweetened foods, and No significant abnormality detected
pancreatectomy are the only proven treatments. Glucose Laboratory values
tablets (1 or 2) preemptively 60-90 minutes after a meal Creatinine, 1.0 mg/dL
or at first symptoms of hypoglycemia seems to help. Fasting glucose, 93 mg/dL
Fasting insulin, 5 uU/L
CASES WITH QUESTIONS Alanine aminotransferase, 31 u/L
Case 1: David Albumin, 4.0 g/dL
David was a 50-year-old man seen for episodes of loss of Cortisol (0800 h), 8.4 mcg/dL
consciousness. Four years earlier, he had undergone Roux-
What further diagnostic evaluation would you recommend?
en-Y gastric bypass for obesity. Preoperative weight, 140.5 kg
and body mass index (BMI), 41.2 kg/m2. Subsequently, he lost A. Repeat determinations of fasting plasma glucose and
46 kg but eventually regained 6 kg. His first episode of loss of serum insulin
consciousness occurred approximately 2 years prior to his visit B. Plasma glucose and serum insulin obtained at the time
with me. One morning after breakfast, while antique shopping of symptoms
with his wife, he felt weak and lightheaded. He sat down in the C. Mixed-meal test with plasma glucose and serum insulin
store, temporarily felt better, but again felt weak when he stood determinations
up. He then collapsed and broke a glass case in the store. He D. ACTH stimulation test
was taken by ambulance to a hospital but regained conscious- E. Some other test
ness en route. No physical or laboratory abnormality was found
in the emergency department. Plasma glucose was normal. He Case 2: Karen
was released without treatment. Karen was a 49-year-old woman seen for episodes of loss of
A second episode occurred while at work for the Minnesota consciousness. Five years earlier, she had undergone Roux-
Department of Transportation. He had eaten a turkey sandwich en-Y gastric bypass for obesity. Subsequently, she lost 41 kg.
and a snack cake for lunch. After lunch, he felt lightheaded and Nine months prior to her visit, she had an episode of loss of
collapsed into hot asphalt. He was pulled from the asphalt by consciousness with seizure. Subsequently, she had episodes of
coworkers and taken to hospital by ambulance. He had burns fatigue, confusion, and feeling awful, which occurred 1-2
on his hands and was hospitalized for several days. An exten- hours after eating. Symptoms abated spontaneously. Blood
sive cardiac and neurologic evaluation was unrevealing. glucose values were 30-40 mg/dL during episodes.

The Endocrine Society. Downloaded from press.endocrine.org by [${individualUser.displayName}] on 12 January 2017. at 12:19 For personal use only. No other uses without permission. . All rights reserved.
230 ENDO 2016 MEET-THE-PROFESSOR CLINICAL CASE MANAGEMENT

Clinic Visit Case 2: Karen


Medications: The first treatment to try would be a low-carbohydrate diet.
Calcium with vitamin D, 600/200 mg twice daily Glucose tablets (1 or 2) taken preemptively 60-90 minutes after
Vitamin B12 1000 mcg by monthly injection a carbohydrate-containing meal or with the onset of symptoms
Multivitamin daily of hypoglycemia might be helpful. Foods containing fructose
Pantoprazole, 40 mg daily as the main carbohydrate should be well tolerated. As a last
Gabapentin, 600 mg daily at bedtime resort, reversal of gastric bypass or pancreatectomy might be
Duloxetine, 30 mg twice daily considered.
Examination:
Blood Pressure, 121/79 mm Hg REFERENCES
Pulse, 79 beats per minute 1. Halverson JD, Kramer J, Cave A, Permutt A, Santiago J. Altered
glucose tolerance, insulin response, and insulin sensitivity after mas-
Height, 62 in sive weight reduction subsequent to gastric bypass. Surgery. 1982;
Weight, 64.5 kg 92:235-240.
BMI, 26.2 kg/m2 2. Service GJ, Thompson GB, Service FJ, Andrews JC, Collazo-Clavell ML,
Lloyd RV. Hyperinsulinemic hypoglycemia with nesidioblastosis after
Decreased vibratory sense both great toes.
gastric-bypass surgery. N Engl J Med. 2005;353:249-254.
Laboratory values: 3. Patti ME, McMahon G, Mun EC, Bitton A, Holst JJ, Goldsmith J, et al.
Severe hypoglycaemia post-gastric bypass requiring partial pancreatec-
Hemoglobin, 10.8 g/dL tomy: Evidence for inappropriate insulin secretion and pancreatic islet
Creatinine, 1.0 mg/dL hyperplasia. Diabetologia. 2005;48:2236-2240.
Random glucose, 71 mg/dL 4. Clancy TE, Moore FD Jr, Zinner MJ. Post-gastric bypass hyperinsulinism
Calcium, 9.4 mg/dL with nesidioblastosis: Subtotal or total pancreatectomy may be needed to
prevent recurrent hypoglycemia. J Gastrointest Surg. 2006;10:1116-1119.
PTH, 113 pg/mL 5. Alvarez GC, Faria EN, Beck M, Girardon DT, Machado AC. Laparoscopic
25-OH D, 36 g/L. spleen-preserving distal pancreatectomy as treatment for nesidioblastosis
after gastric bypass surgery. Obes Surg. 2007;17:550-552.
6. Bantle JP, Ikramuddin S, Kellogg TA, Buchwald H. Hyperinsulinemic
hypoglycemia developing late after gastric bypass. Obes Surg. 2007;17:
TABLE 2: Mixed Meal Test Result for Case 2 592-594.
Time (min) Plasma Glucose Serum Insulin 7. Kellogg TA, Bantle JP, Leslie DB, et al. Postgastric bypass
(mg/dl) (mU/liter) hyperinsulinemic hypoglycemia syndrome: Characterization and response
to a modified diet. Surg Obes Relat Dis. 2008;4:492-499.
0 74 3 8. ZGraggen K, Guweidhi A, Steffen R, et al. Severe recurrent hypoglyce-
15 141 89 mia after gastric bypass surgery. Obes Surg. 2008;18:981-988.
30 190 183 9. Salehi M, Gastaldelli A, DAlessio DA. Altered islet function and insulin
clearance cause hyperinsulinemia in gastric bypass patients with symptoms
45 159 155
of postprandial hypoglycemia. J Clin Endocrinol Metab. 2014;99:2008-
60 89 55 2017.
90 39 18 10. Marsk R, Jonas E, Rasmussen F, Naslund E. Nationwide cohort study of
120 31 10 post-gastric bypass hypoglycaemia including 5,040 patients undergoing
surgery for obesity in 1986-2006 in Sweden. Diabetologia. 2010;53:2307-
180 73 3
2311.
240 76 3 11. Goldfine AB, Mun EC, Devine E, et al. Patients with neuroglycopenia
after gastric bypass surgery have exaggerated incretin and insulin
secretory responses to a mixed meal. J Clin Endocrinol Metab. 2007;
92:4678-4685.
What treatment would you recommend? 12. Rabiee A, Magruder JT, Salas-Carrillo R, et al. Hyperinsulinemic hypo-
A. Low-carbohydrate diet glycemia after Roux-en-Y gastric bypass: Unraveling the role of gut
B. Glucose tablets (1 or 2) at first symptoms of hormonal and pancreatic endocrine dysfunction. J Surg Res. 2011;167:199-
205.
hypoglycemia
13. Cui Y, Elahi D, Andersen DK. Advances in the etiology and manage-
C. Acarbose with meals containing carbohydrate ment of hyperinsulinemic hypoglycemia after Roux-en-Y gastric by-
D. Reversal of gastric bypass pass. J Gastrointest Surg. 2011;15:1879-1888.
E. Total pancreatectomy 14. Drucker DJ. Glucagon-like peptides: Regulators of cell proliferation, dif-
ferentiation, and apoptosis. Mol Endocrinol. 2003;17:161-171.
15. Hadjiyanni I, Baggio LL, Poussier P, Drucker DJ. Exendin-4 modulates
diabetes onset in nonobese diabetic mice. Endocrinology. 2008;149:1338-
DISCUSSION OF CASES AND ANSWERS
1349.
Case 1: David 16. Meier JJ, Butler AE, Galasso R, Butler PC. Hyperinsulinemic hypoglyce-
Plasma glucose and serum insulin values obtained at the time of mia after gastric bypass surgery is not accompanied by islet hyperplasia or
symptoms would allow one to establish the diagnosis but would increased beta-cell turnover. Diabetes Care. 2006;29:1554-1559.
17. McLaughlin T, Peck M, Holst J, Deacon C. Reversible hyperinsulinemic
be difficult to obtain. Thus, the best diagnostic test would be a
hypoglycemia after gastric bypass: A consequence of altered nutrient
mixed-meal test looking for a plasma glucose nadir less than 50 delivery. J Clin Endocrinol Metab. 2010;95:1851-1855.
mg/dL and peak a serum insulin greater than 50 uU/L. 18. Moreira RO, Moreira RB, Machado NA, Goncalves TB, Coutinho WF.

The Endocrine Society. Downloaded from press.endocrine.org by [${individualUser.displayName}] on 12 January 2017. at 12:19 For personal use only. No other uses without permission. . All rights reserved.
ENDO 2016 OBESITY AND LIPIDS 231

Post-prandial hypoglycemia after bariatric surgery: Pharmacological octreotide on insulin, glucagon, and GLP-1. Surg Obes Relat Dis. 2014;
treatment with verapamil and acarbose. Obes Surg. 2008;18:1618-1621. 10:e31 e33.
19. Valderas JP, Ahuad J, Rubio L, Escalona M, Pollak F, Maiz A. 21. Salehi M, Gastaldelli A, DAlessio DA. Blockade of glucagon-like peptide
Acarbose improves hypoglycaemia following gastric bypass surgery 1 receptor corrects postprandial hypoglycemia after gastric bypass. Gas-
without increasing glucagon-like peptide 1 levels. Obes Surg. troenterology. 2014;146:669-680.
2012;22:582-586. 22. Bantle AE, Wang Q, Bantle JP. Post-gastric bypass hyperinsulinemic
20. de Heide LJM, Laskewitz AJ, Apers JA. Treatment of severe postRYGB hypoglycemia: Fructose is a carbohydrate which can be safely consumed.
hyperinsulinemic hypoglycemia with pasireotide: A comparison with J Clin Endocrinol Metab. 2015;100:3097-3102.

The Endocrine Society. Downloaded from press.endocrine.org by [${individualUser.displayName}] on 12 January 2017. at 12:19 For personal use only. No other uses without permission. . All rights reserved.
232 ENDO 2016 MEET-THE-PROFESSOR CLINICAL CASE MANAGEMENT

Hypertriglyceridemia and Low HDL

M38 mia and low HDL-cholesterol as a major clinical problem. The


Presented, April 1 4, 2016 need to adequately perform a proper differential diagnosis, to
establish an adequate treatment plan and provide adequate
follow-up is crucial in order to prevent recurrence of acute
Maria F. Lopes-Virella, MD, PhD. Professor, cardiovascular disease (CVD) events. The fact that several
Department of Medicine, Division of Diabetes, primary and secondary lipoprotein disorders are associated
Endocrinology and Medical Genetics, Medical University with hypertriglyceridemia and low HDL-cholesterol and that
of South Carolina and Staff Physician, Ralph H. Johnson the diagnosis is not always clear and the treatment plan differ
VA Medical Center, Charleston, South Carolina 29401, with the type of lipid disorder is another significant clinical
E-mail: virellam@musc.edu problem. Finally, the fact that the usual measurement of lipo-
protein levels using cholesterol content as the main measure-
ment can provide inadequate assessment of CHD risk in this
HISTORICAL PERSPECTIVE
type of lipoprotein disorders and may preclude appropriate
The study of modern lipid chemistry began in the 17th and
treatment of the patient to prevent acute CVD events is another
18th centuries with early observations by Robert Boyle, Poul-
problem that needs to be raised in this type of lipid disorders.
letier de la Salle, Antoine Francois de Fourcroy, and others.
However, it was not until the 20th century that dyslipidemia
was accepted as a significant factor in atherosclerosis and BARRIERS TO OPTIMAL MANAGEMENT
coronary heart disease (CHD). Beginning with Anitschkows One of main barriers to optimal management is the inability to
1913 classic work on the cholesterol-fed rabbit model, the 20th adequately assess patient risk using the conventional method-
century brought many advances in the understanding of lipo- ology. Another barrier is the inability to adequately assess the
protein structure and function, and explored relationships be- efficacy of the treatment plan by conventional methodology
tween lipoproteins and disease states. These discoveries led to and therefore significantly reduce patient risk for acute CVD
the first large, randomized, double-blind primary intervention events. Proper methodology to assess risk and efficacy of
trial, the Coronary Primary Prevention Trial (1984), that treatment is available but not always requested in patients who
showed decreasing blood cholesterol (using cholestyramine) would benefit from it and the interpretation of the results is
significantly reduced CHD events. Based on the gathered evi- frequently inadequate.
dence, decreasing blood cholesterol levels for those at risk
became an official national public health goal. LEARNING OBJECTIVES
For the participants, the main learning objectives of this session
SIGNIFICANCE OF THE CLINICAL PROBLEM will be:
Hypertriglyceridemia, both associated with primary or second- 1. Understand the pathophysiology of the most common
ary lipoprotein disorders, is very prevalent in the United States lipoprotein disorders associated with
population. Hypertriglyceridemia is a component of several hypertriglyceridemia and low HDL-cholesterol.
lipoprotein disorders either in the endogenous or exogenous 2. Learn how to evaluate patients with both primary and
lipoprotein pathways. The most common lipoprotein disorder secondary dyslipidemias and assess CVD risk.
associated with hypertriglyceridemia is combined hyperlipid- 3. Learn how to develop a treatment strategy that includes
emia and that lipoprotein disorder is associated with most acute both lifestyle management as well as drug therapy in
cardiovascular events. Dysbetalipoproteinemia is also a lipo- the lipoprotein disorders discussed.
protein disorder in the endogenous lipoprotein pathway associ-
ated with hypertriglyceridemia and more prevalent nowadays STRATEGIES FOR MANAGEMENT OF
due to the marked increase in the incidence of type 2 diabetes. HYPERTRIGLYCERIDEMIA AND LOW
Finally, hypertriglyceridemia associated with disorders in the HDL-CHOLESTEROL
exogenous lipoprotein pathway are also important and although Background
of lower prevalence their incidence is sufficiently high to raise Lipoprotein metabolism has 2 main components. One is con-
problems mainly because their management is usually com- trolled by apolipoprotein (Apo)B and its main function is to
plex. Usually hypertriglyceridemia is associated with low lev- deliver cholesterol to cells and tissues. Cholesterol is an essen-
els of high density lipoprotein (HDL)-cholesterol in all types of tial component of cell membranes and a precursor for the
hypertriglyceridemia. Combined hyperlipidemia is present in synthesis of hormones and bile acids. The other pathway,
most patients with cardiovascular disease or in patients with mediated by HDL, has as main function to remove excess
postacute cardiovascular events, thus placing hypertriglyceride- cholesterol from cells and tissues (reverse cholesterol trans-

The Endocrine Society. Downloaded from press.endocrine.org by [${individualUser.displayName}] on 12 January 2017. at 12:19 For personal use only. No other uses without permission. . All rights reserved.
ENDO 2016 OBESITY AND LIPIDS 233

FIGURE 1. Endogenous lipoprotein metabolism pathway.

port), and it has been identified as playing a main protective and low density lipoprotein (LDL) can interact with LDL
role in oxidative and inflammatory processes. receptors (LDL-Rs). VLDL remnants/IDL will also interact
The ApoB-controlled component is comprised of 2 path- with another receptor called the LDL-related receptor. ApoE
ways, an endogenous pathway (Figure 1) and an exogenous is recognized by both receptors. ApoB is the main ligand for
pathway (Figure 2). The endogenous pathway starts with the the LDL-R although ApoE has a higher affinity than ApoB
synthesis of very low density lipoprotein (VLDL) in the to the LDL-R. Chylomicrons formed by the intestinal cell
liver, whereas the exogenous pathway starts with the forma- are processed similarly to VLDL and the hydrolysis of
tion of chylomicrons by the intestinal cell. Once both par- triglycerides (the main component of chylomicrons) leads to
ticles enter the circulation the triglycerides are hydrolyzed. the formation of chylomicron remnants. The uptake of rem-
The hydrolysis of triglycerides leads to a decrease in nants by the liver is mainly through LDL and LDL-related
triglycerides in the core of VLDL, the particle becomes receptor. Chylomicrons transport dietary triglycerides to the
smaller in size and it is called intermediate density lipopro- peripheral tissues and cholesterol to the liver and therefore
tein (IDL) or VLDL remnant. The amount of cholesterol and chylomicron remnants are the major transport source of
triglycerides in this type of particle is equivalent. After- exogenous cholesterol. Triglycerides are in contrast hydro-
wards further hydrolysis of triglycerides leads remnants/IDL lyzed in peripheral tissues and converted into free fatty acids

FIGURE 2. Exogenous lipoprotein metabolism pathway.

The Endocrine Society. Downloaded from press.endocrine.org by [${individualUser.displayName}] on 12 January 2017. at 12:19 For personal use only. No other uses without permission. . All rights reserved.
234 ENDO 2016 MEET-THE-PROFESSOR CLINICAL CASE MANAGEMENT

(FFAs) and glycerol. FFAs are one of the main sources of enzymes that regulate lipoprotein metabolism such as
energy, and if the amount of FFA released is excessive, ApoCII, ApoCIII, and cholesterol ester transfer protein
triglycerides are again reformed and deposited in the adi- (CETP), and an acquired form which is extremely common
pose tissue as energy storage. in patients with metabolic syndrome, diabetes, hypertension,
Lipoprotein disorders can be can be primary or secondary. central obesity, and renal failure.
Secondary dyslipidemias need to be carefully considered and Familial dysbetalipoproteinemia, also called remnant hyper-
excluded, before a diagnosis of primary dyslipidemia is estab- lipidemia, broad -disease or remnant removal disease, is
lished. In some cases, like hypothyroidism-induced hyperlipid- caused by a point mutation in ApoE, leading to the formation
emia without correcting the primary condition, lipid-lowering of ApoE2 with decreased binding affinity to LDL-R and rem-
agents, diet, exercise, will not be effective. Primary hypothyroid- nant receptors and therefore to a deficient removal of VLDL
ism is easily confirmed by increased levels of TSH. Chronic renal and chylomicron remnants by liver cells. This disorder is con-
failure and uncontrolled diabetes are also common causes of sidered primary when increased production of VLDL is also
secondary hyperlipidemia. Finally, a variety of pharmacological present and secondary when the increased VLDL production is
agents, antiretrovirals, antidepressants, antipsychotics, etc, may secondary to other diseases like hypothyroidism, diabetes, met-
also lead to secondary hyperlipidemia. abolic disorders, or renal disease.
A lipoprotein profile is essential for the initial evaluation of
a patient with suspected hyperlipidemia. When cholesterol and Physiopathology
triglycerides are both elevated, both familial combined hyper- The basic mechanism of FCHL is an increased synthesis of
lipidemia (FCHL) and dysbetalipoproteinemis are the likely ApoB100 in the liver, causing increase production of
diagnosis. If the levels of cholesterol and triglycerides are very triglyceride-rich VLDL. In addition, these patients show de-
similar, the diagnosis of dysbetalipoproteinemia needs to be layed clearance of triglycerides after a fat load, so they may
considered. We will discuss under hypertriglyceridemia and have increased levels of remnants and increased circulating
low HDL-cholesterol these 2 types of lipid disorders. Endoge- FFA after a fat-rich mean, leading to insulin resistance. Genes
nous hypertriglyceridemia, because it has a low prevalence, influencing the FCHL phenotype have not yet been fully char-
will not be discussed. acterized.
Diseases associated with acquired combined hyperlipidemia
Combined Hyperlipidemia and Dysbetalipoproteinemia include diabetes, obesity, and metabolic syndrome as well as
Combined hyperlipidemia is the most common lipid disor- renal diseases.
der, diagnosed in about 80% of patients with dyslipidemia. The increased synthesis and impaired catabolism of
There are 2 types of combined hyperlipidemia, the primary triglyceride-rich VLDL leads to increased levels of trigly-
FCHL, likely caused by polymorphisms in molecules and cerides. Because the VLDL is triglyceride enriched, it will ex-

FIGURE 3. Comparison of cholesterol and apoB content in LDL from patients with familial hypercholesterolemia and
familial combined hyperlipidemia.

The Endocrine Society. Downloaded from press.endocrine.org by [${individualUser.displayName}] on 12 January 2017. at 12:19 For personal use only. No other uses without permission. . All rights reserved.
ENDO 2016 OBESITY AND LIPIDS 235

change triglycerides by cholesterol with both LDL and HDL with cholesterol than ApoB100 (the normal ratio is 1.25:1). In com-
the help of CETP, the key enzyme in this reaction (Figure 3). As bined hyperlipidemia, the cholesterol to ApoB100 ratio is less
a consequence of the transfer, HDL and LDL loose cholesterol than 1.25. In addition, these patients have increased levels of
esters and become richer in triglycerides. The triglycerides will be plasma ApoE and ApoC3 (reflecting the increased concentra-
hydrolyzed and the particles become smaller, small, dense LDL, tion of VLDL remnants) and low HDL-C. Special profiles (eg,
and small HDL (Figure 4). The circulating concentrations of the Berkeley lipoprotein electrophoresis profile separate HDL
HDL-cholesterol and LDL cholesterol are low. That leads to poor and LDL in subfractions, including those that correspond to
estimation of the number of LDL particles in patients with com- small-size and intermediate-size LDL) and lipoprotein sizing
bined hyperlipidemia, because clinically the measurement of by nuclear magnetic resonance performed by the Liposcience
LDL-cholesterol is considered equivalent to the measurement of Laboratories allows the precise determination of LDL particle
LDL. In fact, the number of LDL particles is usually increased number and density, including intermediate density LDL and
although the LDL-cholesterol (C) is normal or sometimes even small dense LDL. These tests are recommended for the proper
low, because the cholesterol ester content in LDL is reduced. The evaluation of CVD risk in patients with suspected combined
CVD risk of these patients cannot be properly estimated by the hyperlipidemia. In patients with confirmed combined hyperlip-
measurement of serum LDL-cholesterol, but rather by the concen- idemia, the same tests should be used, as needed, for follow-up
tration of ApoB or measurement of LDL particle number. purposes, to determine whether the therapy target (ideally LDL
Dysbetalipoproteinemia can result from ApoE deficiency, particle number) is reached and maintained.
but more often is associated with the presence of ApoE2, an In dysbetalipoproteinemia, a lipid profile typically will show
ApoE isoform with lower affinity for remnant receptors. low HDL-C and very close concentrations for cholesterol and
ApoE isoforms vary by 1- to 2-amino acid substitutions in triglycerides with slightly higher triglycerides, because rem-
the binding site (arginine vs cysteine substitutions). If a nants carry very similar concentrations of cholesterol and
patient with the E2:E2 profile has excessive production of triglycerides. When plasma ultracentrifugation is available, the
VLDL, dysbetaliproteinemia will occur. diagnosis of dysbetalipoproteinemia can be made if the choles-
terol to triglyceride ratio of VLDL is greater than 0.3. Identi-
Diagnosis fication of ApoE2:E2 in plasma or ApoE deficiency are defini-
The lipid phenotype is variable, within a family with history of tive tests for the diagnosis of dysbetalipoproteinemia.
FCHL, some have mainly high cholesterol, some high
triglycerides, and even in the same patient, the levels vary at Clinical Features
different times, depending on the diet and weight loss or gain. The main clinical features of combined hyperlipidemia are
For the reasons mentioned above, a diagnosis of combined premature CHD, transient cerebral ischemia, stroke, and
hyperlipidemia can only be established by a quantitative assay intermittent claudication. Note that xanthomas are not seen
of ApoB100, and calculation of the cholesterol to ApoB ratio. in these patients. Joint involvement is also not seen in these
Normally, LDL (the main cholesterol carrier) contains more patients.

FIGURE 4. Formation of cholesterol-depleted LDL and HDL in hypertriglyceridemia.

The Endocrine Society. Downloaded from press.endocrine.org by [${individualUser.displayName}] on 12 January 2017. at 12:19 For personal use only. No other uses without permission. . All rights reserved.
236 ENDO 2016 MEET-THE-PROFESSOR CLINICAL CASE MANAGEMENT

Many patients with dysbetalipoproteinemia have insulin re- plasma triglycerides. If the triglyceride levels are below
sistance and fasting hyperglycemia, elevated uric acid, hepato- 300-mg/dL, statins are likely to be the first drug of choice.
splenomegaly, and xanthomas. Premature CHD and peripheral Nicotinic acid and fibrates are most effective drugs for
vascular disease are common in this disease and their presen- reducing plasma/serum triglycerides and therefore are the most
tation is usually early in life and CHD may precede or follow commonly used, usually in combination with statin therapy
peripheral vascular disease (PVD) but both occur within a short when the triglyceride levels are above 300 mg/dL. Two fibric
interval. The xanthomas most frequently observed are flat (pla- acid derivatives are used clinically, gemfibrozil and fenofibrate.
nar xanthomas) and tend to be localized to the palms of hands Both increase the synthesis of ApoA-I and ApoA-II and of the
and feet soles. Tuberous xanthomas are also common. ATP-binding cassette transporter ABCA1 and lead to increased
HDL levels, thus facilitating reverse cholesterol transport.
Management Fibrates also have an inhibitory effect over VLDL synthesis by
A common factor in these 2 types of lipid disorders is in- inhibiting ApoCIII synthesis and increasing the conversion of
creased plasma triglycerides and low HDL. When triglycerides FFA into acetyl-coenzyme A in the liver. Fibrates also up-
are above 400 mg/dL, direct measurement of LDL or tests regulate LPL synthesis, which helps reduce VLDL by increas-
providing the total number of LDL particles, which are consid- ing its conversion into LDL. Thus, fibrates may cause a mild
erably more useful and reliable, need to be requested. Non- increase in LDL and LDL-cholesterol, but usually, this just
HDL-cholesterol is also useful because it provides a combined reflects the change in LDL composition, which becomes richer
assessment of cholesterol and triglycerides. Lifestyle modifica- in cholesterol as a consequence of the reduction of triglyceride
tions and pharmacotherapy are usually needed to treat both synthesis, without affecting the LDL particle number. An im-
lipid disorders. portant difference between gemfibrozil and fenofibrate is that
Patients with combined hyperlipidemia and dysbetali- gemfibrozil causes an increase in the levels of statins in pa-
poproteinemia should consume a diet low in saturated fat tients receiving both drugs thus potentiating adverse side ef-
(7%10%), with no more than 200 300 mg/d of cholesterol fects, Thus, fenofibrate is preferred for patients in which a
intake per day and moderate amount of carbohydrates with combination of fibric acid derivatives and statins is prescribed.
marked restriction of simple/refined carbohydrates and sugars Fibrates by themselves are usually well tolerated.
(Figure 5). The goal is to decrease VLDL production and avoid Niacin decreases triglyceride synthesis by inhibiting VLDL
down-regulation of LDL-R expression. The patient should lose synthesis as a consequence of blocking a hormone-sensitive
weight and be as close to possible to their ideal body weight. lipase in the liver, essential for FFA mobilization, and inhibit-
Exercise is also very important in these patients and daily exercise ing diacylglycerol acyltransferase. Niacin has no effect on
of 30 45 minutes should be implemented. In general, 5 hours of LPL, so the reduction of VLDL is paralleled by a reduction in
exercise weekly, including strengthening exercises twice/week, LDL, with increased proportions of large LDL particles be-
should be recommended. cause of the decreased availability of triglycerides. HDL, on
Statins and nicotinic acid are commonly used in the treatment the other hand, increases because niacin blocks its catabolism.
of combined hyperlipidemia as well as dysbetalipoproteinemia Parallel to the decrease in VLDL synthesis, LDL will also
and statins are frequently used in the treatment of these 2 disorders decrease, whereas HDL levels increase because niacin reduces
in combination with nicotinic acid or fibrates. Bile acid the degradation of ApoA-I. The most frequent side effect of
sequestrants are not indicated to treat either of these disorders niacin is flushing. Taking one regular aspirin daily can prevent
because they may lead to overproduction of VLDL, and increase it. Other side effects include increased uric acid levels (a
special problem in patients with gout), hepatotoxicity, usually
seen in patients taking high doses of niacin (liver enzymes
should be monitored), and hyperglycemia (monitor closely
blood glucose levels in diabetic patients taking niacin).
Recently several new classes of drugs were introduced that
may be extremely useful to treat these 2 types of lipid disor-
ders, but unfortunately they have side effects and patient com-
pliance with the therapy is not easy. These new drugs were not
approved for the treatment of these disorders but for the treat-
ment of familial homozygous hypercholesterolemia such as
microsomal triglyceride transfer protein inhibitors (lomitapide)
and ApoB antisense oligonucleotide (mipomersen). Lomitapide
is associated with accelerated VLDL degradation and
mipomersen with decreased VLDL formation. The third class
of drugs, the proprotein convertase subtilisin/kexin type 9 in-
FIGURE 5. Diet therapy strategies in hypertriglyceridemia. hibitors, of which 2 compounds (evolocumab and alirocumab)

The Endocrine Society. Downloaded from press.endocrine.org by [${individualUser.displayName}] on 12 January 2017. at 12:19 For personal use only. No other uses without permission. . All rights reserved.
ENDO 2016 OBESITY AND LIPIDS 237

have been also approved by the Food and Drug Administration, orange hue to palmar creases. Labs: cholesterol (CHOL) 455,
are mainly associated with increased removal of LDL and triglycerides (TG) 525, HDL 35, LDL 54, VLDL 365.
remnants and its main mechanism of action is to prevent or Based on your assessment of this patient, which of the
reduce LDL-receptor degradation. All of the new compounds following complications is he most likely to develop?
will have a place in the treatment of combined hyperlipidemia A. Stroke
and dysbetalipoproteinemia. B. Hepatic cirrhosis
C. Cholecystitis
MAIN CONCLUSIONS D. Pancreatitis
The most important caveat in these 2 types of lipid disorders is E. PVD
to adequately diagnose, follow, and assess treatment to prevent
or reduce acute CVD events. Conventional lipid profile may be Case 3
rather misleading and preclude aggressive therapy. On the A 48-year-old female evaluated for possible dyslipidemia 3
other hand, overly aggressive therapy may be unnecessary and months ago. She was healthy and her only medication was an
decrease the degree of patient compliance. Proper characteriza- angiotensin converting enzyme inhibitor for hypertension. BMI
tion of the type of disorder being treated is crucial to allow 28.1, waist circumference 40, BP 140/85. Family history re-
implementing changes in lifestyle or therapy that minimize vealed a brother with CAD at 44 years of age and a father with
patent risk for CVD. CAD at 54 years of age and type 2 diabetes mellitus. Her diet
contained excessive number of calories and frequent fast-food
type of meals, and she had a rather sedentary lifestyle. No
CASES WITH QUESTIONS
xanthomas were observed at physical exam. Lipid panel was
Case 1
not available. You advised her to start lifestyle changes.
A 58-year-old man with coronary artery disease (CAD), dys-
On her second visit, 3 months after, she had lost 5 pounds of
lipidemia, type 2 diabetes mellitus, and hypertension. He has
weight after following an American Diabetes Association-like
history of elevated LDL-C and triglycerides (see Table 1). He
diet. She is still not exercising regularly. Requested lipid panel
was prescribed Simvastatin after having cardiac stent place-
shows: total cholesterol (TC) 257, LDL 96 (direct LDL mea-
ment, but the drug was discontinued 3 months before the visit
surement), HDL 36, and TG 496. TSH 2, creatinine 0.9, fasting
due to abnormalities in his liver function tests. Patient watches
glucose 118, hemoglobin A1C (HbA1c) 5.9%.
diet, quit smoking, drinks socially. Other meds include
In addition to intensifying lifestyle modifications including
metformin, lisinopril, hydrochlorothiazide, and aspirin. No his-
tory of hepatitis and hepatitis serology is negative. Ultrasound an exercise program, which medication(s) would be the most
shows liver parenchyma without dilatation of hepatic ducts and adequate first step?
no masses. No jaundice, hepatomegaly, or edema. Body mass A. Nicotinic acid
index (BMI) 33. B. Fenofibrate
Whats the next best step? C. Statin and fenofibrate
A. Restart therapy with a different statin D. Statin and metformin
B. Start fenofibrate E. Statin and cholestyramine
C. Prescribe niacin
D. Prescribe lomitapide DISCUSSION OF CASES AND ANSWERS
E. Prescribe evolocumab or alirocumab Case 1
This case deals with the diagnosis and dietary and drug man-
Case 2 agement of combined hyperlipidemia, possible side effects of
A 56-year-old male with elevated cholesterol and triglycerides. statins, and how to address them and modify therapy, accord-
Light smoker who quit smoking at age 45. Social ethanol ingly. Lowering LDL is essential in the treatment of this
intake. No family history of CAD. Myocardial Infarction at 49 patient and assessing the effect of dietary modifications is also
years, coronary artery bypass graft at 54 years of age. Placed crucial and seems to have played a role in his lipid levels. I
on lipid-lowering agents in the past but never for more than 18 would request LDL particle number in this patient to ad-
months. BMI 29. Blood pressure (BP) 126/81. Corneal arcus equately assess CVD risk factors and perform proper fol-
present, erythematous coalescing lesions on both elbows and low-up. Most these patients are undertreated and LDL-C is far

TABLE 1.
Lab Work TC TGL LDL HDL ALT AST ALP HbA1c
4 mo ago 161 232 74 41 126 91 84 7.5%
2 wks ago 253 216 166 44 77 59 88 6.9

The Endocrine Society. Downloaded from press.endocrine.org by [${individualUser.displayName}] on 12 January 2017. at 12:19 For personal use only. No other uses without permission. . All rights reserved.
238 ENDO 2016 MEET-THE-PROFESSOR CLINICAL CASE MANAGEMENT

from ideal in these patients to assess adequacy of treatment and Her LDL cholesterol levels could not be calculated due to the
CVD risk prevention. marked increase of her triglycerides, and therefore, it repre-
sents a direct measurement of LDL. The value of using a direct
Case 2 measurement of LDL-C vs LDL particle number will be dis-
This case deals with the diagnosis and clinical features of cussed. Considering the strong family history of CVD and the
dysbetalipoproteinemia and the increased risk to develop family history of diabetes, I would strongly recommend a
PVD not just CHD in this type of lipid disorder. The patient discussion on how aggressive the therapy needs to be to avoid
has a typical presentation and xanthomas compatible with both the development of diabetes and the occurrence of CVD
events. I would also emphasize the need to establish a regular
dysbetalipoproteinemia. The presence of a corneal arcus is
exercise program and make serious lifestyle modifications to
interesting and raises questions concerning possible genetic
avoid the use of multiple medications.
defects involved. I would also discuss adequate dietary and
drug therapy as well as special testing required to confirm
the diagnosis of dysbetalipoproteinemia.
REFERENCES
1. Lopes-Virella MF, Colwell JA. Pharmacological treatment of lipid disor-
ders in diabetes mellitus. Diabetes Metab Rev. 1987;3:691-722.
Case 3 2. Lopes-Virella MF, Jenkins C. Nutrition in diabetes mellitus. In: Berdanier
C, Dwyer J, Feldman E, eds. Handbook of Nutrition and Food. Boca
This case deals with an overweight female with hyperlipidemia Raton, FL: CRC Press LLC; 2008:785-813.
and metabolic characteristics associated with metabolic syn- 3. American Diabetes Association: Clinical Practice Recommendations. Dia-
drome. Her abdominal waist circumference is indicative of betes Care. 2014;37(suppl 1):1-155.
4. Jacobson TA, Ito MK, Maki KC, et al. National Lipid Association recom-
abdominal obesity, her BP is not ideal, and her weight loss was mendations for patient-centered management of dyslipidemia: part 1- full
not sufficient to significantly change her triglyceride levels. report. J Clin Lipidol. 2015;9:129-169.

The Endocrine Society. Downloaded from press.endocrine.org by [${individualUser.displayName}] on 12 January 2017. at 12:19 For personal use only. No other uses without permission. . All rights reserved.
ENDO 2016 OBESITY AND LIPIDS 239

Obesity Medications

M39 schools and inadequate training in residency programs on the


Presented, April 1 4, 2016 management of patients with obesity.
Other cited barriers to optimal practice include lack of reim-
bursement, limited time during office visits, competing de-
Katherine H. Saunders, MD; Alpana P. Shukla, MD, mands, low confidence in the ability to treat and change patient
MRCP; Louis J. Aronne, MD, DABOM, FTOS. Department behaviors, limited resources, and the perception that patients
of Medicine, Division of Endocrinology, Diabetes and are not motivated (6).
Metabolism, Weill Cornell Medical College, New York
City, New York 10065, E-mail: ljaronne@med.cornell.edu
LEARNING OBJECTIVES
As a result of participating in this session, learners should be
INTRODUCTION able to:
Historical Overview
Understand the importance of a multidisciplinary
More than one third of U.S. adults (34.9% or 78.6 million) are
approach to obesity including diet, exercise, behavioral
now obese, defined as body mass index (BMI) at least 30
modification, and pharmacotherapy.
kg/m2 (1). Obesity-related comorbidities include many of the
leading causes of preventable death such as heart disease, Understand mechanisms of action, dosing/administration,
stroke, type 2 diabetes, and certain types of cancer. Modest adverse effects, drug interactions, and contraindications
weight loss of 510% body weight can improve cardiovascular for the six most widely prescribed FDA-approved
disease risk factors (2). antiobesity medications.
The field of obesity medicine is growing quickly: more and
more physicians are taking the American Board of Obesity STRATEGIES FOR MANAGEMENT OF
Medicine (ABOM) examination each year, new fellowship OBESITY
programs are being developed, and four antiobesity medica- According to the 2013 American Heart Association/American
tions have been approved by the U.S. Food and Drug Admin- College of Cardiology/Obesity Society Guideline for the Manage-
istration (FDA) since 2012. ment of Overweight and Obesity in Adults, pharmacotherapy for
the treatment of obesity can be considered if a patient has:
SIGNIFICANCE OF THE CLINICAL PROBLEM A BMI 30 kg/m2
Successful treatment of obesity requires a multidisciplinary A BMI 27 kg/m2 with weight-related comorbidities
approach including diet, exercise, and behavioral modification. such as hypertension, type 2 diabetes, dyslipidemia, and
Even with significant lifestyle changes, weight loss is a chal- obstructive sleep apnea (7).
lenge for many patients as reduced calorie consumption and
increased energy expenditure are counteracted by adaptive In 2015, the Endocrine Society published clinical practice
physiologic responses (3). Reduction in body mass causes an guidelines on pharmacological management of obesity (8).
increase in appetite and a decrease in energy expenditure, These evidence-based guidelines include recommendations for
which is out of proportion to the weight loss. These changes individualized weight management and ongoing evaluation of
are associated with alterations in a range of hormones (4). medication efficacy.
In the past, medications for obesity were used as short-term Primary care physicians and endocrinologists can use these
treatment. However, newer agents have been approved for guidelines to provide appropriate care to patients with obesity.
long-term therapy given that obesity is now considered to be a Medications approved for weight management should be
chronic disease (5). Most of the antiobesity medications affect viewed as useful additions to diet and exercise for patients who
appetite mechanisms, signaling through serotonergic, dopami- have been unsuccessful with lifestyle changes alone.
nergic, or noradrenergic pathways. They primarily target the Phentermine; orlistat; phentermine/topiramate ER; lorcaserin;
arcuate nucleus of the hypothalamus to stimulate anorexigenic naltrexone SR/bupropion SR; and liraglutide, 3.0 mg are the
pro-opiomelanocortin (POMC) neurons, which promote six most widely prescribed obesity medications approved by
satiety. the FDA (9). Table 1 provides an overview of the medications.

BARRIERS TO OPTIMAL PRACTICE Phentermine


Until a few years ago, there were limited guidelines for the Phentermine was approved by the FDA in 1959 and has been
management of obesity and few FDA-approved pharmacologic the most commonly prescribed antiobesity medication in the
options. In addition, there is insufficient teaching in medical United States (9). It was approved only for short-term use (3 mo)

The Endocrine Society. Downloaded from press.endocrine.org by [${individualUser.displayName}] on 12 January 2017. at 12:19 For personal use only. No other uses without permission. . All rights reserved.
240 ENDO 2016 MEET-THE-PROFESSOR CLINICAL CASE MANAGEMENT

Table 1. Overview of FDA-Approved Antiobesity Medications


Phentermine/Topirimate Naltrexone SR/Bupropion Liraglutide 3.0 mg
Phentermine Orlistat (Xenical) ER (Qsymia) Lorcaserin (Belviq) SR (Contrave) (Saxenda)

Mechanism Adrenergic agonist Lipase inhibitor Adrenergic 5-HT2C receptor agonist Opioid receptor antagonist/ GLP-1 analog
agonist/neurostabilizer Dopamine and NE
reuptake inhibitor/
Estimated % weight loss 5.1% at 28 weeks (11) 3.1% at 1 yr (12) 6.6% at 1 yr (13) 3.6% at 1 yr (14) 4.8% at 56 weeks (15) 5.4% at 56 weeks (16)
(medication compared 15 mg daily 120 mg TID 7.5/46 mg daily 10 mg BID 16/180 mg BID 3 mg daily
to placebo, ITT data)
Dosage/administration 15 mg or 37.5 mg daily 120 mg TID with 3.75/23 mg daily with 10 mg BID 8/90 mg daily (in the 0.6 mg daily with gradual
(can also use 1/4 or meals gradual dose escalation morning) with dose dose escalation (1.2 mg
1/2 pill) (7.5/46 mg daily then escalation (8/90 mg BID daily then 1.8 mg daily
11.25/69 mg daily then then 16/180 mg in the then 2.4 mg daily then
15/92 mg daily) morning, 8/90 mg in the 3.0 mg daily)
evening then 16/180 mg
BID)
Available formulations Capsule, tablet, powder Capsule Capsule Tablet Tablet Prefilled pen for SC
injection
Approved for long-term No Yes Yes Yes Yes Yes
use
Schedule IV controlled Yes No Yes Yes No No
substance
Side effects Dizziness, dry mouth, Oily spotting, flatus Paresthesia, dizziness, Headache, dizziness, fatigue, Nausea, constipation, Nausea, hypoglycemia,
difficulty sleeping, with discharge, dysguesia, insomnia, nausea, dry mouth, headache, vomiting, diarrhea, constipation,
irritability, nausea/ fecal urgency, constipation, dry constipation, dizziness, insomnia, dry vomiting, headache,
vomiting, diarrhea, fatty/oily stool, mouth hypoglycemia, back pain, mouth, diarrhea dyspepsia, fatigue,
constipation oily evacuation, cough, fatigue dizziness, abdominal
increased pain, increased lipase
defecation, fecal
incontinence
Contraindications Pregnancy, nursing, Pregnancy, chronic Pregnancy, glaucoma, Pregnancy; caution with Pregnancy, uncontrolled Pregnancy, personal or
CVD, during or malabsorption hyperthyroidism, valvular heart disease and HTN, history of seizures family history of
within 14 days of syndrome, during or within 14 other serotinergic drugs or at risk of seizure, medullary thyroid
MAOIs, other cholestasis; days of MAOIs, other (coadministration may bulimia or anorexia, use carcinoma or multiple
sympathomimetic should not be sympathomimetic lead to serotonin of opioid agonists or endocrine neoplasia
amines, taken with amines syndrome or neroleptic partial agonists, during syndrome type 2
hyperthyroidism, cyclosporine, malignant syndrome) or within 14 days of
glaucoma, agitated levothyroxine, MAOIs
states, history of drug warfarin or
abuse, concomitant antiepileptic
alcohol use drugs

Abbreviations: 5-HT2C, serotonin; NE, norepinephrine; GLP-1, glucagon-like peptide-1; ITT, intention-to-treat; BID, twice daily;
TID, three times daily; SC, subcutaneous; CVD, cardiovascular disease; HTN, hypertension.
Adapted from Saunders KH, Igel LI, Kumar RB, Shukla AP, Aronne LJ. Pharmacotherapy for Obesity. Endocrinol Metab Clin North
Am. In press.; Apovian CM, Aronne L, Powell AG. Clinical Management of Obesity. 186-192. Professional Communications, Inc., 2015.

as there are no long-term safety trials of phentermine avoided to prevent insomnia. There is no advantage of continuous
monotherapy; however, many practitioners prescribe phentermine compared with intermittent phentermine treatment (18).
for longer durations as off-label therapy for continued weight
management. Orlistat (Xenical)
Phentermine is an adrenergic agonist that promotes weight Orlistat was FDA approved in 1999 for chronic weight man-
loss by activation of the sympathetic nervous system. Norepi- agement and remained the only FDA-approved weight loss
nephrine release causes increased resting energy expenditure medication for chronic use until 2012 (9). It is also available as
and appetite suppression (17). In the 1990s, it was prescribed in an over-the-counter medication called Alli at half the prescrip-
combination with the serotonin-releasing medication, fenfluramine; tion dose. Orlistat reduces fat absorption from the gastrointes-
however, fenfluramine was withdrawn in 1997 because of car- tinal (GI) tract by inhibiting pancreatic and gastric lipases. At
diac valvulopathy. the recommended dose, orlistat blocks absorption of approxi-
The recommended dosage of phentermine is 1537.5 mg daily, mately 30% of ingested fat (19).
but dosage should be individualized to achieve adequate response Although the GI adverse effects associated with steatorrhea
with the lowest effective dose. For some patients, a quarter tablet can limit patient tolerability and long-term use, orlistat can be
(9.375 mg) or a half tablet (18.75 mg) may be adequate. A split an attractive medication for patients with constipation. Alterna-
dose of a half tablet two times daily is also an option, but tively, the addition of a psyllium fiber supplement can reduce
administration of the second dose in the late evening should be adverse effects.

The Endocrine Society. Downloaded from press.endocrine.org by [${individualUser.displayName}] on 12 January 2017. at 12:19 For personal use only. No other uses without permission. . All rights reserved.
ENDO 2016 OBESITY AND LIPIDS 241

The recommended dosage of orlistat is one 120-mg capsule receptors, which are associated with hallucinations and cardiac
(Xenical) or one 60-mg capsule (Alli) three times a day with valve insufficiency, respectively (21).
each main meal containing fat. Patients should be advised to The recommended dose of lorcaserin is 10 mg twice daily
follow a nutritionally balanced, reduced-calorie diet with ap- with or without food. The medication should be discontinued if
proximately 30% of calories from fat. In addition, the daily at least 5% weight loss is not achieved after 12 weeks.
intake of fat, carbohydrate, and protein should be distributed Lorcaserin is listed in Schedule IV of the Controlled Sub-
over three meals. Given that orlistat decreases the absorption of stances Act.
fat-soluble vitamins (A, D, E, and K), patients should take a
multivitamin (separately from the medication) to ensure ad-
Bupropion SR/Naltrexone SR (Contrave)
equate nutrition.
Bupropion SR/naltrexone SR was approved by the FDA in
2014 (9). Bupropion is a dopamine/norepinephrine reuptake
Phentermine/topiramate ER (Qsymia)
inhibitor which was approved on its own for depression in the
The fixed-dose combination of phentermine and topiramate ER
1980s and smoking cessation in 1997. Naltrexone is an opioid
was approved by the FDA in 2012 as the first combination
receptor agonist approved for opiate dependency in 1984 and
medication for chronic weight management (9). Given that
obesity is a complex disorder that involves multiple signaling alcohol addiction in 1994. Naltrexone antagonizes an inhibitory
pathways, targeting different sites simultaneously can have an feedback loop that limits bupropions anorectic properties so
additive effect on weight loss. In addition, the smaller doses of the two medications have a synergistic effect (22). The combi-
each medication reduce the adverse-effect profile. nation works synergistically to activate POMC neurons in the
Topiramate, which was approved for epilepsy in 1996 and arcuate nucleus. This causes release of -MSH (a potent ano-
migraine prophylaxis in 2004, has been found to decrease rectic neuropeptide), which projects to other hypothalamic ar-
caloric intake. The mechanism responsible for weight loss is eas involved in feeding and body weight control.
uncertain, but thought to be mediated through its modulation of Naltrexone SR/bupropion SR tablets contain 8 mg naltrexone and
-aminobutyric acid (GABA) receptors, inhibition of carbonic 90 mg bupropion. Initial prescription should be for one tablet
anhydrase, and antagonism of glutamate to reduce food intake (6). daily with instructions to increase by one tablet a week to a
Phentermine/topiramate ER is available in four doses, which maximum dose of two tablets in the morning and two tablets in
should be taken once daily in the morning. Evening adminis- the evening (32/360 mg). The medication should be discontin-
tration should be avoided to prevent insomnia associated with ued if a patient has achieved no greater than 5% weight loss at
phentermine. 12 weeks.
Gradual dose escalation, which helps minimize risks and
adverse events, should be done as follows:
Liraglutide (Saxenda)
3.75/23 mg daily for 14 days, then 7.5/46 mg daily
Liraglutide is a glucagon-like peptide-1 (GLP-1) analog with
At 12 weeks: option to increase to 11.25/69 mg daily,
97% homology to human GLP-1, a gut-derived incretin hor-
then 15/96 mg daily
mone (9). Native GLP-1 has a half-life of only 1 to 2 minutes;
The medication should be discontinued or the dose should be however, liraglutide has a half-life of approximately 13 hours.
escalated if 3% weight loss is not achieved after 12 weeks at Liraglutide was approved in 2010 for the treatment of type 2
7.5/46 mg daily. In addition, if 5% weight loss is not achieved diabetes under the brand name Victoza at doses up to 1.8 mg
after 12 weeks on the maximum dose (15/92 mg), the medication daily. It improves hemoglobin A1c, blood pressure (BP), and
should be discontinued. Gradual discontinuation is recommended
lipids. Given that many patients on the liraglutide lost weight
to prevent seizure from rapid withdrawal of topiramate.
in a dose-dependent manner, it seemed to be a desirable treat-
All formulations of phentermine/topiramate ER are listed
ment option for obesity (23). In 2014, the FDA approved
in Schedule IV of the Controlled Substances Act because
liraglutide as Saxenda at 3.0 mg for chronic weight manage-
phentermine is a Schedule IV drug. The medication is preg-
ment in patients with obesity. Weight loss is mediated by
nancy category X and the FDA requires a Risk Evaluation and
Mitigation Strategy (20). reduced appetite and energy intake rather than increased energy
expenditure (24).
Liraglutide is administered as an sc injection once daily into
Lorcaserin (Belviq)
Lorcaserin was also approved by the FDA in 2012 for chronic the abdomen, thigh, or upper arm irrespective of meals. It is
weight management (9). It is a serotonin receptor agonist initiated at 0.6 mg daily for 1 week with instructions to in-
thought to reduce food intake and increase satiety by selec- crease by 0.6 mg weekly until 3.0 mg is reached. Slow dose
tively activating receptors on anorexigenic POMC neurons in titration is effective in managing GI adverse effects. The medi-
the hypothalamus. At the recommended dose, lorcaserin selec- cation should be discontinued if a patient has achieved less
tively binds to 5-HT2C receptors instead of 5-HT2A and 5-HT2B than or equal to 4% weight loss at 16 weeks.

The Endocrine Society. Downloaded from press.endocrine.org by [${individualUser.displayName}] on 12 January 2017. at 12:19 For personal use only. No other uses without permission. . All rights reserved.
242 ENDO 2016 MEET-THE-PROFESSOR CLINICAL CASE MANAGEMENT

MAIN CONCLUSIONS A. Phentermine


Successful pharmacotherapy for obesity depends on tailoring B. Phentermine/topiramate ER
treatment to a patients behaviors and comorbidities. When C. Liraglutide
choosing a medication, it is essential to consider medication D. Bupropion SR/naltrexone SR
interactions and risk of potential adverse effects. The chal-
lenges of weight maintenance justify a long-term approach Case 3
with chronic treatment and followup to prevent relapses. A 50-year-old man with obesity (511; 241 lbs; BMI, 33.7
Given that multiple agents are sometimes required for clini- kg/m2) s/p laparoscopic sleeve gastrectomy 2 years ago, HL,
cally significant weight loss, the future of obesity medicine will obstructive sleep apnea on CPAP, gastroesophageal reflux dis-
likely involve combinations of agents in addition to behavioral
ease, and depression presents for weight regain. His medica-
counseling. In a recent pilot study, the addition of phentermine
tions are lansoprazole, 30 mg daily; Ca, 600 mg daily; vitamin
to lorcaserin resulted in twice as much weight loss compared
D3, 2000 IU daily; and a multivitamin. His highest weight was
with lorcaserin alone (25). Other combined treatments are also
301 lbs and he lost 80 lbs in the year following his procedure
expected.
(280 3 202 lbs), but he became ravenously hungry at this low
weight and has regained 39 lbs over the past year. He con-
CASES WITH QUESTIONS stantly thinks about food and cannot control his impulses to
Case 1 buy or order in large quantities of unhealthy food. He is so
A 64-year-old woman with obesity (52; 189 lbs; BMI, 34.6
preoccupied by thoughts of food that he has difficulty concen-
kg/m2), stage II breast cancer status post (s/p) mastectomy and
trating at work.
chemotherapy (no evidence of disease), type 2 diabetes melli-
Which medication would be an appropriate next step for this
tus (HA1c, 6.1% on metformin), congestive heart failure, hy-
patient?
pertension, and hyperlipidemia presents for weight gain. Her
A. Orlistat
medications are metformin, 1000 mg twice daily; venlafaxine, 75
mg XR daily; lisinopril, 5 mg daily; carvedilol, 12.5 mg twice B. Phentermine
daily; simvastatin, 10 mg daily; aspirin, 81 mg daily; and C. Lorcaserin
exemestane, 25 mg daily. Her highest weight was 210 lbs a few D. Bupropion SR/naltrexone SR
years ago. She lost 40 lbs by reducing carbohydrate intake, but
she did not stick to the diet and regained 19 lbs. She describes DISCUSSION OF CASES AND ANSWERS
herself as a carbaholic and she binges occasionally when she Case 1
is stressed. She is most hungry in the afternoon. She walks one Answer: C, liraglutide. This is a case of a patient with well-
mile per day for exercise. controlled diabetes who describes difficulty following a diet,
Which medication would be an appropriate next step for this cravings, and binging. I would start by reviewing a low-
patient? carbohydrate diet and encourage adding weight resistance in
A. Phentermine/topiramate ER addition to her walking. Liraglutide would be a good medica-
B. Lorcaserin tion given her diabetes. It will also help her feel less hungry.
C. Liraglutide
Phentermine/topiramate ER is not a great option because of her
D. Orlistat
cardiac history. Lorcaserin could be helpful, but I would be
cautious given that she is on an serotonin-norepinephrine
Case 2 reuptake inhibitor (risk of serotonin syndrome). Finally, orlistat
A 27-year-old woman with obesity (55.5; 203 lbs; BMI, 33.3
would not be my first choice because of the adverse-effect
kg/m2), prediabetes (HA1c, 5.7%), polycystic ovary syndrome,
profile.
HL, migraines, and depression presents for weight manage-
ment. Her medications are metformin, 1000 mg twice daily;
bupropion XL, 450 mg daily; Yaz; a multivitamin; and Case 2
Excedrin PRN. She suffers from migraines twice a month. She Answer: B, phentermine/topiramate ER. This is a case of a
was 220 lbs when she graduated from college. She then lost 30 patient who continues to gain weight despite an excellent diet
lbs with diet and exercise, but plateaued at 190 lbs. The past and exercise regimen. Phentermine is a good choice given that
year has been very stressful and she gained 13 lbs despite no she is young and has no cardiac disease; however, phentermine/
change in diet or exercise. She follows a strict low-carbohydrate, topiramate ER would be a better option because she also has
portion-controlled diet and exercises (cardio and weight resis- migraines. Liraglutide would not be my first choice because it
tance) daily. is an injectable. She could be switched from bupropion to
Which medication would be an appropriate next step for this bupropion SR/naltrexone SR, but she should not take both
patient? medications together.

The Endocrine Society. Downloaded from press.endocrine.org by [${individualUser.displayName}] on 12 January 2017. at 12:19 For personal use only. No other uses without permission. . All rights reserved.
ENDO 2016 OBESITY AND LIPIDS 243

Case 3 extended-release in obese adults. Obesity (Silver Spring). 2013;21(11):2163-


Answer: D, bupropion SR/naltrexone SR. This is a case of a 2171.
12. Yanovski SZ, Yanovski JA. Long-term drug treatment for obesity: A
patient who has intrusive food thoughts and tremendous hunger systematic and clinical review. JAMA. 2014;311(1):74-86.
after losing a significant amount of weight. This happens be- 13. Gadde KM, Allison DB, Ryan DH, et al. Effects of low-dose, controlled-
release, phentermine plus topiramate combination on weight and associ-
cause of changes in hormonal pathways following weight loss.
ated comorbidities in overweight and obese adults (CONQUER): A
In addition to diet and exercise, bupropion SR/naltrexone SR randomised, placebo-controlled, phase 3 trial. Lancet. 2011;377(9774):
would be a good choice because of the addictive nature of his 1341-1352.
14. Smith SR, Weissman NJ, Anderson CM, et al. Multicenter, placebo-
presentation. The medication can help patients think differently
controlled trial of lorcaserin for weight management. Multicenter, placebo-
about food. Again, orlistat would not be my first choice be- controlled trial of lorcaserin for weight management. N Engl J Med.
cause of the adverse-effect profile. Phentermine and lorcaserin 2010;363(3):245-256.
15. Greenway FL, Fujioka K, Plodkowski RA, et al. Effect of naltrexone plus
are also options, but bupropion SR/naltrexone SR would likely bupropion on weight loss in overweight and obese adults (COR-I): A
address his behavior more effectively. multicentre, randomised, double-blind, placebo-controlled, phase 3 trial.
Lancet. 2010;376(9741):595-605.
16. Pi-Sunyer X, Astrup A, Fujioka K, et al. A Randomized, controlled trial of
REFERENCES 3.0 mg of liraglutide in weight management. N Engl J Med. 2015;373(1):
1. Ogden CL, Carroll MD, Kit BK, Flegal KM. Prevalence of childhood and 11-22.
adult obesity in the United States, 2011-2012. JAMA. 2014;311(8):806- 17. Rothman RB, Baumann MH, Dersch CM, et al. Amphetamine-type central
814. nervous system stimulants release norepinephrine more potently than they
2. Wing RR, Lang W, Wadden TA, et al. Benefits of modest weight loss in release dopamine and serotonin. Synapse. 2001;39(1):32-41.
improving cardiovascular risk factors in overweight and obese individuals 18. Munro JF, MacCuish AC, Wilson EM, Duncan LJ. Comparison of con-
with type 2 diabetes. Diabetes Care. 2011;34(7):1481-1486. tinuous and intermittent anorectic therapy in obesity. Br Med J. 1968;
3. Rosenbaum M, Hirsch J, Gallagher DA, Leibel RL. Long-term persistence 1(5588):352-354.
of adaptive thermogenesis in subjects who have maintained a reduced 19. Zhi J, Melia AT, Guerciolini R, et al. Retrospective population-based
body weight. Am J Clin Nutr. 2008;88(4):906-912. analysis of the dose-response (fecal fat excretion) relationship of orlistat in
4. Sumithran P, Prendergast LA, Delbridge E, et al. Long-term persistence of normal and obese volunteers. Clin Pharmacol Ther. 1994;56(1):82-85.
hormonal adaptations to weight loss. N Engl J Med. 2011;365(17):1597- 20. Qsymia risk evaluation and mitigation strategy (REMS). VIVUS, Inc.
1604. Available from: http://www.qsymiarems.com. Accessed October 30, 2015.
5. American Medical Association. Policy H-440.842. Recognition of obesity 21. Connolly HM, Crary JL, McGoon MD, et al. Valvular heart disease
as a disease. 2013. associated with fenfluramine-phentermine. N Engl J Med. 1997;337(9):581-
588.
6. Kushner, RF. Weight loss strategies for treatment of obesity. Prog
22. Greenway FL, Whitehouse MJ, Guttadauria M, et al. Rational design of a
Cardiovasc Dis. 2014;56(4):465-472.
combination medication for the treatment of obesity. Obesity (Silver
7. Jensen MD, Ryan DH, Apovian CM, et al. 2013 AHA/ACC/TOS guide-
Spring). 2009;17(1):30-39.
line for the management of overweight and obesity in adults: A report of
23. Nauck M, Frid A, Hermansen K, et al. Efficacy and safety comparison of
the American College of Cardiology/American Heart Association Task
liraglutide, glimepiride, and placebo, all in combination with metformin, in
Force on Practice Guidelines and The Obesity Society. J Am Coll Cardiol. type 2 diabetes: The LEAD (liraglutide effect and action in diabetes)-2
2014;63(25 Pt B):2985-3023. study. Diabetes Care. 2009;32(1):84-90.
8. Apovian CM, Aronne LJ, Bessesen DH, et al. Pharmacological manage- 24. van Can J, Sloth B, Jensen CB, Flint A, Blaak EE, Saris WH. Effects of
ment of obesity: An endocrine Society clinical practice guideline. J Clin the once-daily GLP-1 analog liraglutide on gastric emptying, glycemic
Endocrinol Metab. 2015;100(2):342-362. parameters, appetite and energy metabolism in obese, non-diabetic adults.
9. Saunders KH, Igel LI, Kumar RB, Shukla AP, Aronne LJ. Pharmaco- Int J Obes (Lond). 2014;38(6):784-793.
therapy for obesity. Endocrinol Metab Clin North Am. In press. 25. Smith SR, Garvey WT, Greenway F, et al. Combination weight manage-
10. Apovian CM, Aronne L, Powell AG. Clinical management of obesity. ment (WM) pharmacotherapy with lorcaserin (LOR) and immediate re-
West Islip: Professional Communications, Inc; 2015, 186-192. lease (IR) phentermine (phen). Obesity Week 2014. American Society for
11. Aronne LJ, Wadden TA, Peterson C, Winslow D, Odeh S, Gadde KM. Metabolic and Bariatric Surgery and the Obesity Society Joint Annual
Evaluation of phentermine and topiramate versus phentermine/topiramate Meeting. November 2-7, 2014; Boston, MA. Abstract 2053P.

The Endocrine Society. Downloaded from press.endocrine.org by [${individualUser.displayName}] on 12 January 2017. at 12:19 For personal use only. No other uses without permission. . All rights reserved.
244 ENDO 2016 MEET-THE-PROFESSOR CLINICAL CASE MANAGEMENT

Management of Severe Hypercholesterolemia

M52 gous and has extreme levels of cholesterol with LDL levels
Presented, April 1 4, 2016 often reaching the greater than 600 mg/dL (15.54 mmol/L)
range. These individuals, if untreated, develop significant ten-
dinous xanthomas and are at risk for ASCVD and death at a
Marc-Andre Cornier, MD. Division of Endocrinology, very young age (before age 20 y). More than 1600 mutations of
Metabolism and Diabetes, University of Colorado the LDL-receptor have been documented. The prevalence of
School of Medicine, Aurora, Colorado 80045, E-mail: heterozygous FH is as high as one in 300 500 persons depend-
marc.cornier@ucdenver.edu ing on the population, rendering FH as one of the most com-
mon serious genetic disorders known. Untreated patients with
INTRODUCTION heterozygous FH typically have LDL levels in the 200 300
Historical Overview mg/dL (5.18 7.77 mmol/L) range and can also develop xan-
It was not until the turn of the twentieth century that choles- thomas. They are at increased risk for premature ASCVD
terol was identified as a key component of atherosclerosis and events before the fourth and fifth decades.
that it was found that feeding cholesterol to rodents could PCSK9 is an important regulator of LDL receptor degrada-
produce atherosclerosis. It was not until the 1930s though, that tion because binding of PCSK9 to the LDL receptor results in
the genetic link between high cholesterol and myocardial in- degradation of the receptor, thereby preventing recycling of the
farctions (MIs) was identified and described as familial hyper- receptor. Although loss-of-function mutations of PCSK9 are
cholesterolemia (FH). This disorder was later known to be associated with increased LDL receptor function and have led
consistent with heterozygous FH and was associated with sig- to new treatment options for lowering LDL, gain-of-function
nificantly elevated cholesterol levels and increased risk for mutations of PCSK9 may also lead to severe hypercholesterol-
premature coronary heart disease. In the 1950s the biosynthetic emia clinically similar to FH. In this condition, PCSK9 reduces
pathways of cholesterol was further understood (1) and low- hepatic LDL receptor activity where over 70% of LDL clear-
density lipoprotein (LDL) was identified and high levels found ance occurs.
to be associated with higher risk for coronary heart disease (2). Severe hypercholesterolemia can also occur in the setting
It was not until the seminal work of Goldstein and Brown (3) in of mixed hyperlipidemias and is also associated with prema-
the 1970s that the LDL receptor was identified and that a defect ture ASCVD. familial combined hyperlipidemia is the most
in the LDL-receptor gene was discovered to be the primary common genetic disorder of mixed hyperlipidemia, occur-
defect responsible for FH. We now know that there are many ring in up to one in 250 persons. Although a specific gene
different LDL receptor gene mutations, and depending on the mutation has not been identified, these individuals have
site of the mutation, patients with homozygous FH are unable increased production of apo B containing lipoproteins. Fa-
to produce any functional receptors or produce low levels of milial dysbetalipoproteinemia most often occurs due to a
normally active LDL receptors. We also know that patients genetic variation in apo E, the E2 isoform. This is associated
with heterozygous FH have approximately half the normal with reduced clearance of remnant particles and is associ-
amount of normally functional LDL receptors. ated with planar and tuberous xanthomas and premature
ASCVD if untreated. The metabolic dyslipidemia is the
most common cause of mixed hyperlipidemia associated
SIGNIFICANCE OF THE CLINICAL PROBLEM with other features of the metabolic syndrome but is gener-
Atherosclerotic cardiovascular disease (ASCVD) remains as a ally associated with more modest elevations in cholesterol.
major cause of death in the United States and the world. Choles-
terol is at the core of the development of atherosclerosis and is one
of the major modifiable risk factors. Individuals with genetically BARRIERS TO OPTIMAL PRACTICE
caused severe hypercholesterolemia are at greatly increased risk Effectiveness of available treatment options.
for premature ASCVD, and current guidelines recommend that Adverse effects of currently available treatment
any individual with an LDL greater than 190 mg/dL (4.92 options.
mmol/L) should be treated with high-intensity statin therapy (4). Cost of therapies.
Severe hypercholesterolemia can occur in isolation as seen in
the different forms of FH, or can be associated with other lipid LEARNING OBJECTIVES
abnormalities such as concomitantly with elevated triglycerides As a result of participating in this session, learners should be
and/or low high-density lipoprotein cholesterol (HDL). able to:
Isolated severe hypercholesterolemia is usually consistent Recognize and initiate an appropriate evaluation of the
with FH. Approximately one in one million persons is homozy- hypercholesterolemic patient.

The Endocrine Society. Downloaded from press.endocrine.org by [${individualUser.displayName}] on 12 January 2017. at 12:19 For personal use only. No other uses without permission. . All rights reserved.
ENDO 2016 OBESITY AND LIPIDS 245

Provide individualized goals for treatment of 4. Laboratory analysis


hypercholesterolemia. a. LDL 330 mg/dL (8.5 mmol/L) (8 points).
Understand the different medical treatment options for b. LDL, 250329 mg/dL (6.58.4 mmol/L) (5 points).
cholesterol lowering. c. LDL, 190249 mg/dL (5.06.4 mmol/L) (3 points).
Be aware of other treatment options such as LDL d. LDL, 155189 mg/dL (4.04.9 mmol/L) (1 points).
apheresis and recognize when to refer patients to a lipid- e. HDL and triglycerides are normal.
disorders specialist. f. DNA-analysis: functional mutation LDL receptor
gene present (8 points). Somon Broome Register Di-
agnostic Criteria for Heterozygous FH.
STRATEGIES FOR DIAGNOSIS, THERAPY,
Definite FH is defined as:
AND/OR MANAGEMENT
Diagnosis 1. Total cholesterol 260 mg/dL (6.7 mmol/L) or LDL
As previously discussed, homozygous FH patients typically 4.0 mmol/L (155 mg/dL) in a child 16 y or total
have LDL levels greater than 600 mg/dL (15.54 mmol/L) cholesterol 290 mg/dL (7.5 mmol/L) or LDL 190
and family history of both parents with evidence for mg/dL (4.9 mmol/L) in an adult, as well as tendon
heterozygous FH. These patients must be identified at a xanthomas in patient or in first- or second-degree
young age and treated aggressively. Genetic testing can be relative.
performed but not all of the more than 1600 mutations are 2. Otherwise, DNA-based evidence of an LDLR mutation
identified in clinically available genetic testing. As such, or familial defective Apo lipoprotein B 100.
genetic testing, if negative, does not exclude the diagnosis Possible FH is defined as:
and is not necessary if the clinical scenario is consistent 1. Total cholesterol 260 mg/dL (6.7 mmol/L) or LDL
with the diagnosis. Heterozygous-FH patients should be 155 mg/dL (4.0 mmol/L) in a child 16 y or total
suspected in the setting of LDL greater than 160 mg/dL cholesterol 290 mg/dL (7.5 mmol/L) or LDL 190
(4.14 mmol/L) in children or LDL greater than 190 mg/dL mg/dL (4.9 mmol/L) in an adult.
(4.92 mmol/L) in adults. Most adults will have LDL levels 2. And at least one of the following:
greater than 220 mg/dL (5.70 mmol/L). FH should also be a. Family history of MI at less than 50 years in second-
suspected in the setting of the following physical examina- degree relative or less than 60 years in first-degree
tion findings: 1) TENDON xanthomas at any age (most relative.
commonly in the Achilles tendon and finger extensor ten- b. Family history of raised cholesterols, 290 mg/dL (7.5
dons), 2) corneal arcus in patient younger than 45 years of mmol/L) in adult first- or second-degree relative or
age, and 3) xanthelasmas in patients younger than 20 25 260 mg/dL (6.7 mmol/L) in child or sibling 16 y.
years of age. It is also important to also consider secondary causes of hyper-
Two different criteria have been established to help make cholesterolemia in these patients, including: hypothyroidism,
the clinical diagnosis of FH. nephrotic syndrome, cholestatic liver disease, medications, and
diet.
World Health Organization Diagnostic Criteria
Diagnosis of heterozygous FH is certain when more than 8 Goals of Treatment
points, probable when 6 8 points, possible when 35 points: The primary goal of treatment is to reduce the risk of ASCVD-
1. Family history related events. Current joint guidelines published by the
a. First-degree relative with known premature (men American College of Cardiology and the American Heart As-
55 y, women 60 y) cardiovascular disease (1 sociation in 2013 have established that all individuals with an
point). LDL of greater than 190 mg/dL (4.92 mmol/L) should be
b. First-degree relative with known LDL-cholesterol treated with high-intensity statin (atorvastatin 40/80 mg or
above the 95th percentile for age and sex, and/or with rosuvastatin 20/40 mg) (4). Specific LDL goals are less clearly
tendon xanthomata and/or arcus cornealis, or children supported by the literature. Guidelines have suggested that
less than 18 years with LDL above the 95th percen- intensification should be considered if LDL remains greater
tile for age and sex (2 points). than 150 160 mg/dL (3.88 4.14 mmol/L). Other ASCVD
2. Clinical history risk factors should also be aggressively managed, including
i. Patient has premature (men 55 y, women 60 y) hypertension, smoking, and unhealthy lifestyle.
coronary artery disease (2 points) or other cardiovas- In patients with mixed hyperlipidemia, the primary goal is to
cular disease (1 point). treat the hypercholesterolemia to reduce ASCVD risk as discussed
3. Physical examination above. Triglycerides should be targeted in those with severe el-
a. Tendon xanthomata (6 points). evations, greater than 400 500 mg/dL (10.36 12.95 mmol/L)
b. Arcus cornealis below the age of 45 years (4 points). with a primary goal of reducing the risk of pancreatitis. Treating

The Endocrine Society. Downloaded from press.endocrine.org by [${individualUser.displayName}] on 12 January 2017. at 12:19 For personal use only. No other uses without permission. . All rights reserved.
246 ENDO 2016 MEET-THE-PROFESSOR CLINICAL CASE MANAGEMENT

moderate hypertriglyceridemia can be considered as a secondary apheresis requires a well-coordinated team of experienced per-
goal although direct support for this is somewhat lacking. sonnel (11).

Treatment Options Nonlipid Treatments


Medical Therapy Lifestyle modification including smoking cessation increase
Medical therapy with high-intensity statins is the standard of physical activity and reduced saturated fat intake should be
care and cornerstone of treatment with a goal of reducing LDL recommended to all patients with hypercholesterolemia. Sec-
by 50% or more (4). Homozygous patients, however, clearly ondary causes of hyperlipidemia if present should also be
need more LDL lowering than can be achieved by statin treated. Hypertension and diabetes should also be aggressively
therapy alone. Multiple drugs are usually necessary and often treated. Consideration for low-dose aspirin therapy should also
not sufficient. Other medical treatment options to consider be made.
include ezetimibe, bile acid sequestrants, niacin, lomitapide,
mipomersen, and now the PCSK9 evolocumab.
MAIN CONCLUSIONS
Statins are the gold standard therapy due to the number of
Severe hypercholesterolemia is usually associated with a ge-
clinical trials that have shown benefit in those with and without
netic cause such as FH and is associated with premature
FH. Recent evidence suggests that combination therapy with
ASCVD. Aggressive, early treatment is warranted and neces-
ezetimibe may confer further risk reduction (5). There is less
sary to prevent ASCVD. Combination medical therapy is usu-
evidence in support of other treatment options such as combi-
ally necessary to adequately reduce the total cholesterol bur-
nation therapy with bile acid sequestrants. More recently there
den. The new PCSK9 inhibitors may offer a new mechanism to
have been two negative trials of the addition of niacin to statin
substantially lower LDL in these patients otherwise difficult to
therapy (6, 7).
treat. Finally, in some patients LDL apheresis is necessary and
Two PCSK9 inhibitors, aloricumab and evolocumab, have
should be considered in the appropriate patient.
now been approved in the United States for the treatment of
heterozygous FH (8, 9) and should be considered as an impor-
tant treatment option for those patients who do not respond CASES
adequately to statins ezetimibe. These agents lower LDL Case 1
50 60% beyond maximally tolerated statin therapy. A 45-year-old woman is referred to you for evaluation and
Evolocumab has also been approved for the treatment of ho- management of hyperlipidemia. She was otherwise healthy
mozygous FH with LDL lowering of 20 30% beyond maxi- until she suffered a transient ischemic attack and an MI with
mally tolerated statin therapy (10). stent placement 2 years ago. She was started on atorvastatin, 80
Lomitapide and mipomersen are also approved for the treat- mg after her MI for high cholesterol but developed signifi-
ment of homozygous FH. Lomitapide is a microsomal triglyc- cant muscle aches on this treatment. Her cardiologist then tried
eride transfer protein inhibitor taken orally in a dose of 5 60 her on rosuvastatin, 20 mg daily but again she developed
mg daily. Lomitapide has been shown to reduce LDL by up to muscle aches and pains. She was then tried on pravastatin, 20
40% in those who can tolerate it. Fat in the diet must be mg, which she has been able to tolerate. She has not had any
restricted to prevent diarrhea. Mipomersen inhibits the transla- further ASCVD events to this point. She is currently also
tion of apo B-100 mRNA, preventing VLDL and LDL produc- taking clopidogrel, carvedilol, lisinopril, and sertraline. Her
tion and reduces LDL by an additional 20% above other treat- other medical problems include hypertension and depression.
ments. It is dosed sc every once weekly and can cause acute Her family history is significant for her father having a MI in
flu-like symptoms. Both of these treatments can be associated his 40s and a brother undergoing coronary artery bypass graft-
with reversible hepatotoxicity. ing (CABG) in his early 50s. She does not smoke, tries to eat a
low-saturated-fat diet, and walks 2 miles every day.
Apheresis Her physical examination is unremarkable. Her fasting lab-
LDL apheresis, the direct removal of cholesterol, has been oratory tests reveal cholesterol, 247 mg/dL (6.40 mmol/L);
shown to prolong survival, especially in children with homozy- triglycerides, 100 mg/dL (2.59 mmol/L); LDL, 175 mg/dL
gous FH. Apheresis should be considered in high-risk patients (4.53 mmol/L); HDL, 52 mg/dL (1.35 mmol/L); and a normal
such as those with known ASCVD who have been refractory to complete metabolic panel.
medical therapy and/or those intolerant to medical therapy.
LDL apheresis only temporarily removes LDL particles and Question 1
thus must be repeated on a regular basis, generally on an every What should this patients LDL goal be?
12-week basis. Some patients are able to extend the frequency A. LDL 150 mg/dL
of apheresis with newer medical therapies. Because of the B. LDL 100 mg/dL
frequent need for vascular access, many if not most patients C. LDL reduction of 50%
require fistulas. Successful treatment of patients with this LDL D. Lowest LDL level she can tolerate

The Endocrine Society. Downloaded from press.endocrine.org by [${individualUser.displayName}] on 12 January 2017. at 12:19 For personal use only. No other uses without permission. . All rights reserved.
ENDO 2016 OBESITY AND LIPIDS 247

Answer C. Based on current guidelines the primary goal would trointestinal adverse effects with colesevelam, niacin led to
be to treat her with high-intensity statin therapy with a goal of intolerable flushing, and ezetimibe caused muscle aches as
reducing her LDL by 50% or more. Unfortunately, she was not well. He finally started LDL apheresis 4 years ago, which has
able to tolerate high-intensity statin therapy. Nevertheless, helped reduce the rate of new ASCVD events. He has had more
more aggressive LDL goals aiming for levels at least less than trouble with angina, however, since moving to higher altitude.
150 160 mg/dL is a reasonable secondary goal.
Question 1
Question 2 What should this patients LDL goal be?
Which of the following would be the best next step in her A. LDL less than 150 mg/dL
management? B. LDL less than 100 mg/dL
A. Try a different more efficacious statin such as C. LDL reduction of greater than 50%
simvastatin or pitavastatin D. Lowest LDL level he can tolerate
B. Retry atorvastatin or rosuvastatin at a lower dose
Answer D. Although based on current guidelines the primary
C. Add ezetimibe
goal would be to treat him with high-intensity statin therapy
D. Add a PCSK9 inhibitor
with a goal of reducing his LDL by 50% or more, he has had
E. Start LDL apheresis
persistent ASCVD. As such his LDL should be treated as
Answer C. Although all of these options are reasonable options aggressively as he can tolerate.
to consider and try, it would be most reasonable to try combi-
nation therapy. Maximizing statin therapy should first be tried.
Question 3
This could be done by pushing the dose of the current treat- Which of the following would be the best next step in his
ment, pravastatin, try a different higher-potency statin such management?
as simvastatin or pitavastatin, or retrying atorvastatin or A. Add a PCSK9 inhibitor
rosuvastatin at lower doses. It would also be reasonable to try B. Add lomitapide
combination therapy with an agent such as ezetimibe or a bile C. Add mipomersen
acid sequestrant in addition to the highest-tolerable statin. The D. No changes in management, simply continue LDL
PCSK9 inhibitors are also now approved for the treatment of apheresis
heterozygous FH in patients not controlled on maximally tol-
erated statin therapy. These agents are the most likely to Answer: All of these options are reasonable options to consider
achieve good control of severe hypercholesterolemia. Most and try. At this time, LDL apheresis should be continued given
insurance plans will require failure of combination therapies. that he has had persistent disease and apheresis has helped
Finally, aggressive medical therapy may also help prevent the reduce event rates. It would also be reasonable, though, to add
need for LDL apheresis. Apheresis should also be considered medical therapy to see whether the frequency of apheresis
as a good option if her LDL cholesterol cannot be controlled could be reduced. All three of these medical treatment options
with aggressive medical therapy and/or she continues to have are approved for homozygous-FH patients. Although all of
recurrent ASCVD events. Before the approval of the PCSK9 these agents are associated with high cost, they are much less
inhibitors, apheresis was more commonly used as a good way expensive than frequent apheresis. The PCSK9 inhibitor,
to control the overall cholesterol burden to which these patients evolocumab, is approved for the treatment of homozygous FH
are exposed. with LDL lowering of 20 30% and is a reasonable best option
as the cost and adverse effect profile are advantageous com-
Case 2 pared with the other options. Mipomersen is also associated
A 56-year-old man with a complex medical history recently with a similar LDL lowering. Lomitapide has been shown to
moved to the area and seeks your help. He was diagnosed with lower LDL by up to 40% at higher doses but can be more
homozygous FH and significant coronary artery disease and difficult to tolerate. Combinations of these newer agents have
has been treated with LDL apheresis. He was first diagnosed not been studied.
15 years ago although knew his cholesterol was very high since
the age of 30 years. He has had multiple stents and MIs and is REFERENCES
s/p coronary artery bypass grafting (CABG) before apheresis 1. Bloch, K. The biological synthesis of cholesterol. Science.
1965;150:19-28.
was started. He has had LDL cholesterol levels in the 500 2. Gofman JW, Glazier F, Tamplin A, Strisower B, De Lalla O. Lipoproteins,
mg/dL range and has xanthomas of the knuckles as well as the coronary heart disease, and atherosclerosis. Physiol Rev. 1954;34:589-607.
Achilles tendons. He has a very strong family history of heart 3. Goldstein JL, Brown MS. Familial hypercholesterolemia: Identification of
a defect in the regulation of 3-hydroxy-3-methylglutaryl coenzyme A
disease in both parents as well as cholesterol levels in family
reductase activity associated with overproduction of cholesterol. Proc Natl
members of greater than 300 mg/dL. He has been intolerant to Acad Sci U S A. 1973;70:2804-2808.
statins with significant myopathy. He has had significant gas- 4. Stone NJ, Robinson JG, Lichtenstein AH, et al. 2013 ACC/AHA

The Endocrine Society. Downloaded from press.endocrine.org by [${individualUser.displayName}] on 12 January 2017. at 12:19 For personal use only. No other uses without permission. . All rights reserved.
248 ENDO 2016 MEET-THE-PROFESSOR CLINICAL CASE MANAGEMENT

guideline on the treatment of blood cholesterol to reduce atheroscle- 8. Raal FJ, Stein EA, Dufour R, et al. PCSK9 inhibition with evolocumab (AMG
rotic cardiovascular risk in adults: A report of the American College of 145) in heterozygous familial hypercholesterolaemia (RUTHERFORD-2): A
Cardiology/American Heart Association Task Force on Practice Guide- randomised, double-blind, placebo-controlled trial. Lancet. 2015;385:331-340.
lines. Circulation. 2014;129:S1S45. 9. Kastelein JJP, Ginsberg HN, Langslet G, et al. ODYSSEY FH I and FH II:
5. Cannon CP, Blazing MA, Giugliano RP, et al. Ezetimibe added to statin 78 week results with alirocumab treatment in 735 patients with heterozy-
gous familial hypercholesterolaemia. Eur Heart J. 2015;36:2996-3003.
therapy after acute coronary syndromes. N Engl J Med. 2015;372:2387-2397.
10. Raal FJ, Honarpour N, Blom DJ, et al. Inhibition of PCSK9 with
6. The HPS2-THRIVE Collaborative Group. Effects of extended-release nia-
evolocumab in homozygous familial hypercholesterolaemia (TESLA
cin with laropiprant in high-risk patients. N Engl J Med. 2014; Part B): A randomised, double-blind, placebo-controlled trial. Lancet.
371:203-212. 2015;385:341-350.
7. The AIM-HIGH Investigators. Niacin in patients with low HDL choles- 11. Heigl F, Hettich R, Eder B, Arendt R. Lipoprotein apheresis standard for
terol levels receiving intensive statin therapy. N Engl J Med. 2011; apheresis competence centersan updated synthesis and amendment to
365:2255-2267. pre-existing standards. Atheroscler Suppl. 2013;14:57-65.

The Endocrine Society. Downloaded from press.endocrine.org by [${individualUser.displayName}] on 12 January 2017. at 12:19 For personal use only. No other uses without permission. . All rights reserved.
PEDIATRIC
ENDOCRINOLOGY

The Endocrine Society. Downloaded from press.endocrine.org by [${individualUser.displayName}] on 12 January 2017. at 12:19 For personal use only. No other uses without permission. . All rights reserved.
250 ENDO 2016 MEET-THE-PROFESSOR CLINICAL CASE MANAGEMENT

Thyroid Nodules and Cancer in Children

M09 (PTC) also frequently present as cervical adenopathy with or


Presented, April 1 4, 2016 without a palpable thyroid lesion, as a diffusely infiltrating
lesion (diffuse-sclerosing variant of PTC), or as an incidental
finding after imaging or surgery for an unrelated condition.
Gary L. Francis, MD, PhD. Childrens Hospital of Occasionally, the diagnosis is made only after the discovery of
Richmond at the Virginia Commonwealth University, distant metastases.
Richmond, Virginia 23298, E-mail: glfrancis@vcu.edu Previous recommendations for therapy included total thy-
roidectomy, lymph node dissection, and radioactive iodine
(RAI) ablation for essentially all children with DTC, but these
INTRODUCTION
Historical Overview recommendations have been revised with the American Thy-
Early reports of thyroid cancer in children opine that this is roid Association Management Guidelines for Children with
probably a very rare disease. In 1920, Basil Hughes, DSO, Thyroid Nodules and Cancer published in 2015 (4). The ratio-
MA, MB, Assistant Honorary Surgeon, Childrens Hospital of nale for these new recommendations is based on several find-
Bradford published a case, Papilliferous Carcinoma of the ings about PTC in children.
Thyroid Gland in a 13-year-old girl (1). In 1934, Schreiner and In the past, most children and adolescents with thyroid
Murphy (2) used a combination of surgery plus external beam cancer went to surgery without any preoperative staging. It was
radiation to treat differentiated thyroid cancer (DTC) but de- felt that lymphatic spread and distant metastases would be
scribed the outcome as a fatal disease with few exceptions. identified by RAI whole-body scan postoperatively and treated
They did postulate that early recognition and surgical removal with therapeutic doses of RAI. This committed all children to
might offer hope for cure. Radioactive iodine (RAI) was first RAI therapy. However, we now know that children and ado-
used for treatment of DTC in 1946, following which, patients lescents have low risk for mortality from thyroid cancer despite
presentation with regional lymph node involvement and even
typically received surgery and RAI therapy (3). Tools for early
distant metastases. Treatment with total thyroidectomy plus
recognition: puncture of thyroid (fine-needle aspiration; FNA)
RAI was highly effective (98% 40-y survival) but resulted in
in 1952 and thyroid ultrasound in the 1967 led to earlier
short-term adverse effects and probably excess mortality from
diagnosis but treatment for all children with DTC still included
second malignancies associated with radiation (external beam
surgery and RAI until very recently. Based on the historical
or RAI). Studies by Rubino et al (12), Hay et al (13), and
belief that DTC was a fatal disease unless treated early with
Brown et al (14) reported a significant increase in second
combined surgery plus RAI, the endpoint of treatment was to
malignancies among patients treated with radiation or RAI
render children free from disease. This resulted in a high
(relative risk, 1.16; P .05) particularly among younger pa-
survival rate but an unacceptable risk of complications. For that
tients. For that reason, the Guidelines emphasizes preoperative
reason, the American Thyroid Association empaneled the Pe-
and postoperative staging of children and adolescents with
diatric Thyroid Cancer Guideline Committee to develop man-
thyroid cancer so that a well-planned treatment can be outlined
agement guidelines for children with thyroid cancer. Published
and implemented without reliance on universal RAI therapy.
in 2015, the Guidelines stratify children into three groups based
The Guidelines now stratifies children and adolescents into
on the risk of recurrence and propose different treatments for
three groups based on their risk for recurrence: low risk, inter-
children at low, intermediate, and high risk for recurrence (4).
mediate risk, and high risk (4). Patients at low risk for recur-
These Guidelines are used throughout the following case dis-
rence are generally managed with surgery but defer RAI
cussions to illustrate their application to clinical management.
therapy for only those few who have persistent or progressive
disease over time. By so doing, most low-risk children will not
SIGNIFICANCE OF THE CLINICAL PROBLEM receive RAI therapy.
According to the Surveillance, Epidemiology, and End Results Another change in the approach is management of cervical
(SEER) data base, DTC represents only 1.4% of all pediatric lymph nodes. Previous surgery usually included palpation and
malignancies, but the incidence is increasing and among berry picking of suspicious lymph nodes at the time of
15-19-year-olds, thyroid cancer is the eighth most frequently thyroidectomy. This resulted in a higher rate of recurrence
diagnosed cancer and the second most common cancer among when compared with compartment focused central neck dissec-
girls (5-8). Adolescents have a 10-fold greater incidence than tion. For that reason, the Guidelines recommend central neck
younger children and there is a female:male preponderance dissection for children and adolescents with suspected or
(5:1) during adolescence that is not seen in young children (5, known lymph node involvement and suggest that prophylactic
7-11). The most common presentation for DTC in children is central neck dissection may be considered for others with large
that of a thyroid nodule. However, papillary thyroid cancer tumors or tumors showing angio-lymphatic invasion (4).

The Endocrine Society. Downloaded from press.endocrine.org by [${individualUser.displayName}] on 12 January 2017. at 12:19 For personal use only. No other uses without permission. . All rights reserved.
ENDO 2016 PEDIATRIC ENDOCRINOLOGY 251

Postoperative staging is also stratified according to the risk DTC should be similarly treated and has provided the oppor-
of recurrence. Patients at low risk for recurrence may be staged tunity to broaden the scope of acceptable therapy in an attempt
with simply a suppressed serum thyroglobulin (Tg) level, to provide aggressive therapy when warranted, and less aggres-
whereas those at intermediate and certainly high risk can be sive therapy for children who are unlikely to benefit from
staged with TSH-stimulated Tg and whole-body radioactive aggressive treatment.
iodine scan (WB-RAI). Neck ultrasound (US) or hybrid imag-
ing (SPECT-CT) may be used to further define the anatomic LEARNING OBJECTIVES
location of RAI uptake. As a result of participating in this session, learners should be
The Guidelines also differs in the use of WB-RAI scan for able to:
followup. Previous recommendations relied on the WB-RAI to Describe the most common type and presentation of
detect residual or recurrent disease but this subjected the pa- DTC in children.
tient to scheduled periods of hypothyroidism in preparation for List at least two major controversial areas in the
scanning and is not the most sensitive method to detect residual contemporary approach and treatment of DTC in
or recurrent disease. Neck US and serum Tg measurements children.
seem to be more sensitive and less invasive. For that reason, Describe the limitations of using serum Tg as a
WB-RAI scan is no longer recommended for annual followup biomarker for DTC in children.
and has been replaced by serial US and Tg measures. WB-RAI
scan is still important for children with pulmonary metastases
or increasing Tg when neck US and computerized tomography STRATEGIES FOR DIAGNOSIS, THERAPY,
(CT) or other imaging fails to provide anatomic localization. AND/OR MANAGEMENT
Thyroid Nodules
Last, but equally important, previous recommendations at-
One of the most common presentations for DTC in children is
tempted to render all children with DTC free from disease but
detection of a thyroid nodule and in fact, the probability that a
this is difficult and perhaps impossible for some children with
thyroid nodule will prove to be malignant is much greater
pulmonary metastases. Following surgery and RAI, LaQuaglia
(approximately 26%) among children (17, 18). Previous ap-
et al (15) reported that 65% of young patients with pulmonary
proaches to thyroid nodules in children recommended removal
metastases developed stable but persistent disease and none
of all nodules due to this high risk for malignancy. We have
died. More recently, Biko, et al (16) showed a continual de-
learned, however, that almost all benign lesions remain benign
cline in serum thyroglobulin levels (Tg) that continued for
and that a combination of US features and FNA cytology
years after RAI therapy was discontinued. For this reason, the
provides excellent predictive value in distinguishing benign
Guidelines recommend that disease progression should be
and malignant lesions. Recent series using US guidance have
documented prior to additional therapy.
shown a very low incidence of false-negative FNA in children.
It is clear from this discussion that we must better distin-
Overall accuracy of FNA in children is reported as high as
guish those children who are likely to have recurrent disease
93.7% (19) and 90.4% (20). Based on these data, the 2015
and to treat them appropriately; to distinguish those children
Guidelines suggests that benign nodules do not require removal
with minimal risk of recurrence and to avoid exposing them to
as long as followup is available. Lesions that enlarge over time
potentially harmful treatment with RAI; to improve our knowl-
should be removed. However, one may consider removal based
edge of serum Tg and long-term changes in Tg in young
on patient and/or parental desire.
patients with DTC; and to develop clear and appropriate end-
In children, size of the nodule alone does not predict malignant
points for therapy. The Guidelines from the American Thyroid
histology (21). Lyshchik et al (22) examined thyroid lesions in
Association are the first attempt to achieve these goals.
children with US and power-doppler and showed that malignant
and benign lesions had similar size (13.8 8.8 and 14.8 10.7
BARRIERS TO OPTIMAL PRACTICE mm, respectively). Drozd et al (23) showed that small PTCs that
There are no prospective, placebo-controlled, double-blind developed following radiation exposure are less likely to show
studies comparing treatments for children with DTC. Published typical malignant features on US. Irregular margins were more
data are from retrospective cohorts and are potentially subject common in large lesions and were associated with younger age;
to investigator bias or nonrandom assignment to various treat- however, this feature is not adequate to identify all PTC in chil-
ment groups. Further limiting the development of treatment dren, and irregular borders were not seen in 21.7% of PTC.
recommendations is the fact that most published series of DTC Based on these data, the Guidelines recommends that all
in children averaged only 10 years of followup. This has made thyroid nodules in children should be evaluated by US and that
it difficult to determine whether any treatment results in im- FNA should be performed on lesions with suspicious features
proved recurrence risk, mortality, or complication rates. Nev- (such as calcification, irregular borders, subcapsular location,
ertheless, retrospective analysis of therapeutic options has led and central blood flow) or lesions that grow over time.
to a reconsideration of the former concept that all children with Preoperative FNA of a hyper-functioning nodule is not typi-

The Endocrine Society. Downloaded from press.endocrine.org by [${individualUser.displayName}] on 12 January 2017. at 12:19 For personal use only. No other uses without permission. . All rights reserved.
252 ENDO 2016 MEET-THE-PROFESSOR CLINICAL CASE MANAGEMENT

cally warranted in children as all hyper-functioning nodules all children and adolescents with preoperative evidence of
should be removed. There is approximately a 5% risk for multifocal disease and/or regional metastasis should un-
malignancy among hyper-functioning nodules in children so dergo a total thyroidecotmy and a therapeutic level VI
that neck US to evaluate the cervical nodes and removal of all dissection. There are two reports in children showing that this
hyper-functioning nodules is recommended. combination is associated with DFS as high as 95% at 5 and 10
Cytologically benign lesions should be followed with serial years (40, 41). Although this approach may not improve over-
US. The appearance of concerning US features or growth over all survival, it does seem to decrease the need for second
time should prompt removal. However, repeat FNA prior to surgical procedures and increased DFS. In addition, level VI
removal will allow appropriate preoperative staging and defini- dissection my increase the efficacy of RAI on distant metasta-
tive surgery for those lesions that are suspicious or malignant sis when progression is associated with decreased survival
on FNA. Fortunately, there are no data to suggest an increase (36-38, 42, 43). For all other children, one must weigh the
in disease-specific mortality for patients with delayed diagnosis increased risk of surgical complications associated with pro-
and removal of what is ultimately shown to be PTC. phylactic level VI with the potential benefit of decreasing
persistent and/or recurrent disease. The extent of initial sur-
Differentiated Thyroid Cancers gery seems to have the greatest effect on improving long-
The overwhelming majority of thyroid cancers in children are term DFS (13, 37).
PTC followed by follicular (FTC) and even more rarely, med- RAI is warranted for the treatment of children with
ullary thyroid cancer (10, 24-26) There are important clinical, distant metastases as long as these remain iodine-avid. Ad-
molecular, and pathological differences in DTC among chil- ditional RAI therapy should be given as long as prior therapy
dren when compared with adults that prompted the American was documented to benefit (reduced tumor burden). The dose
Thyroid Association to empanel a task force to develop pedi- of RAI should be individualized based on body weight or body
atric treatment guidelines. When controlled for histology and weight plus extent of disease. The use of RAI for other patients
tumor size, children with PTC are more likely to have regional should be individualized. In general, patients with bulky neck
lymph node involvement, extrathyroidal extension, and distant disease are likely to benefit from RAI but only after surgical
pulmonary metastasis than are adults. Despite extensive dis- removal of as much tumor as can be safely resected (24).
ease, children are less likely to die from disease (2% cause- Patients with disease confined to the thyroid gland are not
specific mortality in recent series) than are adults (8, 13, 26, likely to benefit from RAI therapy at the outset but could be
27), and many children with pulmonary metastases (30 45%) expectantly followed for future use of RAI if indicated (9, 40).
develop persistent, albeit stable disease following radioactive Follow-on therapy should include T4 suppression and rou-
iodine (RAI) therapy (28, 29). Progression-free survival in tine surveillance to detect future recurrence. Empiric treat-
children with persistent DTC is also more favorable than in ment of low-level Tg without structural evidence of disease
adults. is not warranted unless serial Tg measures increase.
All children newly diagnosed with PTC or suspected Much is known about serum Tg in adults with DTC but
PTC require preoperative evaluation to optimize surgical extrapolating this to children may be problematic. Robbins, et
outcome and further treatment planning. In all cases, a al (44) showed in adults that rh-TSH stimulated serum Tg
comprehensive neck US should be obtained by an experienced levels correlate with tumor histology, extent, and location of
examiner to interrogate the contralateral thyroid lobe and the disease. Mazzaferri et al (45) showed that recombinant thyro-
central and lateral neck lymph node basins, as it may identify tropin (rhTSH) stimulated serum Tg 2 ng/mL had a sensi-
disease not otherwise appreciated on physical examination tivity of 100%, a negative predictive value of 100%, and a
(30-33). false-positive rate of only 9% for detecting persistent or recur-
For most children, total thyroidectomy is recommended. rent DTC in adults.
The rationale for this recommendation is based on multiple Unfortunately, serum Tg levels were not reported in the vast
studies showing an increased incidence of bilateral and majority of studies on young patients with DTC. One pediatric
multifocal disease, 30 and 57% respectively, as well as a higher study did correlate serum Tg with response to therapy (46).
risk of recurrence and subsequent second procedures when less Children who eventually entered remission had lower initial
than total thyroidectomy is performed (13, 34-39). In long-term stimulated Tg (median, 1.8; maximum 91 ng/mL) than did
analysis, bilateral resection compared with unilateral resection children who failed to enter remission (median, 13.0; maxi-
has been shown to decrease the local recurrence from 35 6% mum 329 ng/mL), but the absolute levels of serum Tg were
after 40 years of followup (13). higher than levels reported in adults. Furthermore, Biko, et al
Many children with PTC have loco-regional cervical metas- (16) reported serum Tg levels for 20 children with persistent
tasis at the time of diagnosis. Similar to adults, there is clear DTC. The patients were treated with total thyroidectomy and
evidence showing decreased disease-free survival (DFS) and RAI ablation (100 MBq/kg, 2.7 mCi/kg) at intervals of 5-12
increased disease progression related to persistent or recurrent months. After five or six courses of RAI and a total average
loco-regional disease. For that reason, we recommend that accumulated dose of 24.2 GBq (654 mCi), RAI therapy was

The Endocrine Society. Downloaded from press.endocrine.org by [${individualUser.displayName}] on 12 January 2017. at 12:19 For personal use only. No other uses without permission. . All rights reserved.
ENDO 2016 PEDIATRIC ENDOCRINOLOGY 253

discontinued. At that time, the median serum Tg was 56 ug/L 4-fold if there are other family members with DTC (47).
(56 ng/mL). The patients were followed for an additional 10 The risk of malignancy is related to the serum TSH level in
years without additional RAI therapy. Serum Tg showed a children as it is in most adult studies and serum TSH 2.5
continuous decline of approximately 35% per year and at the mIU/L predicts malignancy in children (48). However, the
end of followup, 16/20 patients (80%) had serum Tg 10 ug/L EPIC study, performed in adults, suggests that the risk for
( 10 ng/mL). The rate of decline in serum Tg while receiv- thyroid cancer may actually be inversely related to the TSH
ing RAI treatment was similar to the rate of decline during levels in adults (odds ratio, 0.56) (49). How or if these data
the 10 years after discontinuation of RAI therapy, suggest- relate to children with PTC is unclear. Despite that contro-
ing that there might be long-term effects of RAI on DTC in versy, recent data suggest that 5% of hyperfunctioning nodules
children. in children may be malignant so that a suppressed TSH does
Based on these data we suggest that surveillance for not exclude malignancy (50). For these reasons, the American
recurrent DTC should be lifelong in children given that Thyroid Association Management Guidelines for Children with
some will develop recurrent disease as long as 20-30 years Thyroid Nodules and Differentiated Thyroid Cancer (hereafter
after initial therapy. However, treatment based on serum referred to as the Guidelines) recommend that all hyperfunctioning
Tg alone, without anatomic evidence of disease, should be nodules in children should be removed.
considered only if serum Tg is increasing over time. Thyroid and neck US should be performed to interrogate the
gland as well as regional lymph nodes. The probability of
malignancy is increased for children with suspicious lymph
MAIN CONCLUSIONS
nodes (51).
Twenty-six percent of thyroid nodules in children are
Sonographic features of a thyroid nodule in a child have
malignant.
been helpful in predicting malignancy (microcalcifications,
The most common DTC in children is PTC.
hypoechoic pattern, intranodular vascularity, subcapsular loca-
Treatment for PTC must be individualized to reduce the
tion); however, none provide sufficient predictive value to
risk of recurrence and also limit adverse effects from
reliably exclude malignancy. For that reason, most thyroid
therapy.
nodules in children require FNA. FNA is highly sensitive and
Most children with PTC will benefit from total
specific for PTC in children and adolescents and recent studies
thyroidectomy and central compartment lymph node
by Mussa et al (51) report overall accuracy of 91%, sensitivity
dissection.
of 100%, and specificity of 88%.
Those with metastases will benefit from RAI therapy but
An area of concern is the FNA showing follicular lesion of
RAI should be judiciously used in others only after due
uncertain significance or atypia of uncertain significance. In a
consideration of the risks and benefits.
recent study, 40% of FNA with atypia were malignant in
Followup should be life long.
children as were 100% of those with suspicious for follicular
After initial treatment, additional therapy for a low-level
neoplasm (52). Several studies in adults suggest that molecu-
Tg should only be performed with anatomic proof of
lar analysis of these indeterminate lesions may be helpful but
disease and or increasing Tg levels.
there are limited data in children. A recent study by Buryk et al
(53) reported 9/9 (100%) nodules with BRAF, RAS, RET/PTC
CASES AND DISCUSSION or PAX8/PPAR were malignant. However, the number of
Case 1 nodules so far reported in children is small and the Guidelines
A 13-year-old male presents with a 2.3-cm thyroid mass. committee was therefore unable to recommend molecular stud-
1. Which tests should be performed? ies for indeterminate cytology in children. It seems prudent to
2. If this is cytologically benign, is there any treatment? treat nodules with proven mutations as malignant but the com-
3. If this is cytologically benign, what followup is mittee recommends that all nodules with indeterminate cytol-
warranted? ogy in children should be removed due to the high probability
4. If this is cytologically benign, what is the probability of of malignancy.
sampling error, wrong diagnosis, and ultimately a The FNA in this case was consistent with a follicular neo-
pathological diagnosis of malignant thyroid nodule? plasm which could be a benign follicular adenoma, follicular
5. Would the outcome be different if the diagnosis is variant of PTC, or FTC. Although FTC is uncommon in chil-
ultimately shown to be PTC but delayed several months dren, it is a diagnosis that requires surgical pathology to detect
during which the lesion is followed by US? capsular invasion. FTCs are generally subdivided into those
with minimal invasion and those that are widely invasive.
Discussion Because FTC spreads by hematogenous routes rather than
The typical approach to a thyroid nodule begins with family lymphatics (as is seen for PTC) RAI is generally recommended
history, determination of serum TSH, and thyroid US followed for all but FTC with minimal invasion. However, a recent study
by FNA. The risk of malignancy is increased by approximately by Enomoto et al (54) reported on 20 children with FTC. There

The Endocrine Society. Downloaded from press.endocrine.org by [${individualUser.displayName}] on 12 January 2017. at 12:19 For personal use only. No other uses without permission. . All rights reserved.
254 ENDO 2016 MEET-THE-PROFESSOR CLINICAL CASE MANAGEMENT

were 16 minimally invasive and only four widely invasive laryngeal nerve and/or superior parathyroid glands, might be
tumors, but vascular or lymphatic invasion was seen in 9/20. considered in an effort to decrease the risk of permanent
Recurrence was seen in three, all of which were minimally damage to these structures.
invasive, but all three had vascular invasion suggesting that The Guidelines recommend central compartment lymph
minimally invasive FTC with vascular invasion might require node dissection to reduce recurrence risk in young patients
more aggressive therapy. Overall, however, 30-year disease- with PTC. Lateral compartment dissection would be recom-
specific survival (100%) and DFS (62.8%) were comparable to mended if extensive disease is found in the central compart-
PTC. ment or if there is evidence for disease in the lateral compart-
ment. The operative risks are increased after standardized
Case 2 dissection but minimal if the procedure is performed by a
A 6-and-7/12-year-old-male presents with a pea-sized swelling high-volume thyroid surgeon.
in the right lower neck. In this case, disease was found to be superficially invading
Observation was recommended by his pediatrician but the the trachea in the midline and the right paratracheal area.
lesion enlarged, US showed metastatic lymphadenopathy and Soft-tissue involvement of tumor involving the right recurrent
FNA of the lymph node was consistent with PTC. There was laryngeal nerve at the recurrent laryngeal nerve inlet required
no family history of thyroid cancer and no other risk factors. extensive microdissection to spare these arborized branches.
1. What preoperative staging should be performed? He was classified postoperatively as T3N1bM1 (presumed
2. Why is total thyroidectomy recommended over
based on CT finding).
lobectomy for children with PTC?
Postoperative staging is recommended in the Guidelines to
3. When is central neck dissection recommended for
identify children with low, intermediate, and high risk for
children with PTC?
recurrence. In this case, with T3N1bPresumed M1 disease, the
4. When is RAI therapy indicated for children with PTC?
patient is categorized as high risk for recurrence and the Guide-
5. What is the purpose of restaging the patient after
lines recommend postoperative stimulated-Tg and 123Iodine
initial and any subsequent treatment?
WB-RAI scan. In this case, laboratory findings included:
TSH, 119 U/mL
Discussion Tg, 142 ng/mL (neg Ab)
This presentation is the second most common presentation of
PTC in children: a palpable lymph node without palpable Unlike in adults, less is known about the prognostic value of
abnormality of the thyroid gland. The majority, but not all PTC serum Tg levels in children.
in children and adolescents, will have lymph node involve- Robbins, et al (44) showed in adults that rh-TSH stimulated
ment. serum Tg levels correlate with tumor histology, extent and
In all cases of PTC in children, preoperative staging is location of disease. Mazzaferri et al (45) showed that rhTSH-
suggested to identify additional unsuspected neck involvement stimulated serum Tg 2 ng/mL had a sensitivity of 100%, a
or pulmonary metastases. Thyroid and neck US, chest x-ray or negative predictive value of 100%, and a false-positive rate of
CT, and serum Tg with Tg antibody are required. In this case, only 9% for detecting persistent or recurrent DTC in adults.
a small nodule was noted in the lung field on CT but the chest Unfortunately, serum Tg levels were not reported in the vast
radiograph was negative. Laboratory studies revealed: majority of studies on young patients with DTC. One pediatric
TSH, 4.41 U/mL (0.50-5.50 U/mL) study did correlate serum Tg with response to therapy (46).
FT4, 1.1 ng/dL (0.9-1.8 ng/dL) Children who eventually entered remission had lower initial
Tg, 393.0 ng/mL stimulated Tg (median, 1.8; maximum, 91 ng/mL) than did
Tg antibody 20 IU/mL children who failed to enter remission (median, 13.0; maxi-
Papendieck et al (48) showed that in children, TSH 2.5 mum, 329 ng/mL) but the absolute levels of serum Tg were
U/mL is commonly associated with malignancy of thyroid higher than levels reported in adults.
nodules. In this case, the Tg value of 142 ng/mL strongly suggests
Due to the presence of lateral neck disease (initial FNA) this residual disease of sufficient importance to require additional
child had total thyroidectomy, central (bilateral level VI), and therapy. However, the diagnostic 123I scan will help determine
bilateral lateral (levels II-V) neck dissection. whether additional therapy is best provided by repeat neck
Total thyroidectomy is recommended for all PTC in young dissection (macroscopic disease remaining in the neck) or RAI
patients with the exception of small, incidentally discovered therapy (pulmonary metastases). In this case, the WB-RAI scan
lesions. The Guidelines suggest that in patients with a small showed some uptake in the neck and lungs for which he
unilateral tumor confined to the thyroid gland, a near-total received 60 mCi 131I.
thyroidectomy, whereby a small amount of thyroid tissue The Guidelines recommend RAI therapy to treat patients
(12%) is left in place at the entry point of the recurrent with distant metastases and for those at high risk for recurrence

The Endocrine Society. Downloaded from press.endocrine.org by [${individualUser.displayName}] on 12 January 2017. at 12:19 For personal use only. No other uses without permission. . All rights reserved.
ENDO 2016 PEDIATRIC ENDOCRINOLOGY 255

(presentation with bulky neck disease or high-risk histology; ie, A decision to treat with additional surgery or RAI should be
extensive vascular/lymphatic invasion). based on evidence of progressive disease such as increasing
If RAI is given, there are no standardized doses but most serum Tg or identifiable disease with US or other imaging.
experts treat using either 1-1.5 mCi/kg body weight or a There is little utility for positron emission tomography scan in
weight-adjusted adult dose (ie, the dose that would be given to children and young patients who have PTC given that it is very
an adult with similar disease patient weight in kg/70 kg). rare for PTC in this age group to dedifferentiate and there are
Whether to prepare the patient with thyroid hormone with- many false-positive positron emission tomography scans due to
drawal (2 wk is usually sufficient for children) or rhTSH reactive adenopathy.
stimulation is debated. Although rhTSH is not United States
Food and Drug Administrationapproved for children, Luster REFERENCES
et al (55) published a large series of rhTSH administration in 1. Hughes B. Papilliferous carcinoma of the thyroid gland. Br Med J.
children. Safety was excellent, but efficacy was not evaluated. 1920;1(3089):362-363.
2. Schreiner BF, Murphy WT. Malignant neoplasms of the thyroid gland.
They examined 181 doses of rhTSH, of which 92% were the Ann Surg. 1934;99(1):116-125.
typical adult dose (0.9 mg 2) and 34% also had mini-thyroid 3. Seidlin SM, Marinelli LD, Oshry E. Radioactive iodine therapy; Effect on
hormone withdrawal (for 7 d). Almost all (98%) achieved a functioning metastases of adenocarcinoma of the thyroid. J Am Med Assoc.
1946;132(14):838-847.
peak TSH greater than 25 mU/L but higher body mass index
4. Francis GL, Waguespack SG, Bauer AJ, et al. Management guidelines for
was associated with lower peak TSH. In 88% there were no children with thyroid nodules and differentiated thyroid cancer. Thyroid.
adverse events, but nausea was seen in 5%, vomiting in 3%, 2015;25(7):716-759.
5. Wu XC, Chen VW, Steele B, et al. Cancer incidence in adolescents and
and serious nausea or vomiting in five cases.
young adults in the United States, 1992-1997. J Adolesc Health.
Postoperative therapy would include thyroid hormone sup- 2003;32(6):405-415.
pression. Followup would generally involve serum Tg and 6. American Cancer Society. Cancer facts, figures 2009. Atlanta: American
anti-Tg antibody on a quarterly basis, and neck US at 6 months Cancer Society, 2009.
7. Horner MJ, Ries LAG, Krapcho M, et al. SEER Cancer Statistics Review,
and then at least annually thereafter. It is not yet clear when 1975-2006. National Cancer Institute. Accessed from: http://seer.cancer.
this level of surveillance should be relaxed. That would largely gov/csr/1975_2006 on March 12, 2009.
depend on the Tg and US findings over time. Routine repeated 8. Hogan AR, Zhuge Y, Perez EA, Koniaris LG, Lew JI, Sola JE. Pediatric
thyroid carcinoma: Incidence and outcomes in 1753 patients. J Surg Res.
WB-RAI is no longer recommended for followup given that 2009;156(1):167-172.
the sensitivity is less than that of the combined Tg and US 9. Waguespack S, Wells S, Ross J, Bleyer A. Thyroid cancer. In: Bleyer A,
surveillance. OLeary M, Barr R, Ries L, eds. Cancer epidemiology in older adolescents
and young adults 15 to 29 years of age, including SEER incidence and
In this case, the postoperative suppressed serum Tg is con-
survival 1975-2000. Bethesda, MD: National Cancer Institute, 2006;143-
sistently 1.5 ng/mL. In adults, this is suspicious for persistent 154.
disease, but in children we know little about serum Tg levels 10. Demidchik YE, Saenko VA, Yamashita S. Childhood thyroid cancer in
over time. This probably represents persistent disease but as Belarus, Russia, and Ukraine after Chernobyl and at present. Arq Bras
Endocrinol Metabol. 2007;51(5):748-762.
discussed previously, Biko et al (16) found that much higher 11. Spoudeas HA, ed. Paediatric endocrine tumours. West Sussex, United
Tg levels have declined in children over time without addi- Kingdom: Novo Nordisk Ltd, 2005.
tional therapy. We do not know the magnitude of disease 12. Rubino C, de Vathaire F, Dottorini ME, et al. Second primary malignan-
cies in thyroid cancer patients. Br J Cancer. 2003;89(9):1638-1644.
burden that can be tolerated for decades in children. 13. Hay ID, Gonzalez-Losada T, Reinalda MS, Honetschlager JA, Richards
The Guidelines recommends that for this child who has ML, Thompson GB. Long-term outcome in 215 children and adolescents
received therapeutic 131I, there may be a role for a follow-up with papillary thyroid cancer treated during 1940 through 2008. World
J Surg. 2010;34(6):1192-1202.
WB-RAI scan. Children with known iodine-avid metastases
14. Brown AP, Chen J, Hitchcock YJ, Szabo A, Shrieve DC, Tward JD. The
based upon a prior post-treatment scan are the most likely to risk of second primary malignancies up to three decades after the treatment
benefit from subsequent WB-RAI scan. Ideally, the WB-RAI of differentiated thyroid cancer. J Clin Endocrinol Metab. 2008;93(2):504-
scan should be performed only after a significant period of time 515.
15. La Quaglia MP, Black T, Holcomb GW 3rd, et al. Differentiated thyroid
has elapsed to assess the response from the last activity of cancer: Clinical characteristics, treatment, and outcome in patients under
therapeutic 131I (typically 1-2 y), recognizing that clinical re- 21 years of age who present with distant metastases. A report from the
sponse can continue for years. Once a diagnostic whole body Surgical Discipline Committee of the Childrens Cancer Group. J Pediatr
Surg. 2000;35(6):955-959; discussion 960.
scan is negative, there is no utility in repeating the procedure 16. Biko J, Reiners C, Kreissl MC, Verburg FA, Demidchik Y, Drozd V.
unless disease is clinically suspected. The patient could also be Favourable course of disease after incomplete remission on (131)I therapy
interrogated for neck disease with US and for pulmonary dis- in children with pulmonary metastases of papillary thyroid carcinoma: 10
years follow-up. Eur J Nucl Med Mol Imaging. 2011;38(4):651-655.
ease with CT scan. Anatomically visible disease should be
17. Niedziela M. Pathogenesis, diagnosis and management of thyroid nodules
treated based on location and iodine avidity (SPECT-CT). in children. Endocr Relat Cancer. 2006;13(2):427-453.
There is little value in assessing stimulated serum Tg in this 18. Gharib H, Papini E, Valcavi R, et al. American Association of Clinical
case as the unstimulated Tg is detectable indicating disease and Endocrinologists and Associazione Medici Endocrinologi medical guide-
lines for clinical practice for the diagnosis and management of thyroid
the magnitude of the stimulated Tg is not in and of itself nodules. Endocr Pract. 2006;12(1):63-102.
adequate to determine whether additional therapy is warranted. 19. Izquierdo R, Shankar R, Kort K, Khurana K. Ultrasound-guided fine-

The Endocrine Society. Downloaded from press.endocrine.org by [${individualUser.displayName}] on 12 January 2017. at 12:19 For personal use only. No other uses without permission. . All rights reserved.
256 ENDO 2016 MEET-THE-PROFESSOR CLINICAL CASE MANAGEMENT

needle aspiration in the management of thyroid nodules in children and 38. Popovtzer A, Shpitzer T, Bahar G, Feinmesser R, Segal K. Thyroid cancer
adolescents. Thyroid. 2009;19(7):703-705. in children: Management and outcome experience of a referral center.
20. Corrias A, Einaudi S, Chiorboli E, et al. Accuracy of fine needle aspiration Otolaryngol Head Neck Surg. 2006;135(4):581-584.
biopsy of thyroid nodules in detecting malignancy in childhood: Compari- 39. Giuffrida D, Scollo C, Pellegriti G, et al. Differentiated thyroid cancer in
son with conventional clinical, laboratory, and imaging approaches. J Clin children and adolescents. J Endocrinol Invest. 2002;25(1):18-24.
Endocrinol Metab. 2001;86(10):4644-4648. 40. Jarzab B, Handkiewicz-Junak D. Differentiated thyroid cancer in children
21. McHenry CR, Huh ES, Machekano RN. Is nodule size an independent and adults: Same or distinct disease? Hormones (Athens). 2007;6(3):200-
predictor of thyroid malignancy? Surgery. 2008;144(6):1062-1068; discus- 209.
sion 1068-1069. 41. Savio R, Gosnell J, Palazzo FF, et al. The role of a more extensive surgical
22. Lyshchik A, Drozd V, Demidchik Y, Reiners C. Diagnosis of thyroid approach in the initial multimodality management of papillary thyroid
cancer in children: Value of gray-scale and power doppler US. Radiology. cancer in children. J Pediatr Surg. 2005;40(11):1696-1700.
2005;235(2):604-613. 42. Feinmesser R, Lubin E, Segal K, Noyek A. Carcinoma of the thyroid in
23. Drozd VM, Lushchik ML, Polyanskaya ON, et al. The usual childrenA review. J Pediatr Endocrinol Metab. 1997;10(6):561-568.
ultrasonographic features of thyroid cancer are less frequent in small 43. Schlumberger M, De Vathaire F, Travagli JP, et al. Differentiated thyroid
tumors that develop after a long latent period after the Chernobyl radiation carcinoma in childhood: Long term follow-up of 72 patients. J Clin
release accident. Thyroid. 2009;19(7):725-734. Endocrinol Metab. 1987;65(6):1088-1094.
24. Halac I, Zimmerman D. Thyroid nodules and cancers in children. 44. Robbins RJ, Srivastava S, Shaha A, et al. Factors influencing the basal and
Endocrinol Metab Clin North Am. 2005;34(3):725-744, x. recombinant human thyrotropin-stimulated serum thyroglobulin in patients
25. Harness JK, Thompson NW, McLeod MK, Pasieka JL, Fukuuchi A. with metastatic thyroid carcinoma. J Clin Endocrinol Metab. 2004;89(12):
Differentiated thyroid carcinoma in children and adolescents. World 6010-6016.
J Surg. 1992;16(4):547-553; discussion 553-554. 45. Mazzaferri EL, Robbins RJ, Spencer CA, et al. A consensus report of the
26. Demidchik YE, Demidchik EP, Reiners C, et al. Comprehensive clinical role of serum thyroglobulin as a monitoring method for low-risk patients
with papillary thyroid carcinoma. J Clin Endocrinol Metab. 2003;88(4):
assessment of 740 cases of surgically treated thyroid cancer in children of
1433-1441.
Belarus. Ann Surg. 2006;243(4):525-532.
46. Hanscheid H, Verburg FA, Biko J, et al. Success of the postoperative 131I
27. Chow S, Law S, Mendenhall W, et al. Differentiated thyroid carcinoma in
therapy in young Belarusian patients with differentiated thyroid cancer
childhood and adolescenceClinical course and role of radioiodine.
after Chernobyl depends on the radiation absorbed dose to the blood and
Pediatr Blood Cancer. 2004;42:176-183.
the thyroglobulin level. Eur J Nucl Med Mol Imaging. 2011;38(7):1296-
28. Hay ID, Hutchinson ME, Gonzalez-Losada T, et al. Papillary thyroid
1302.
microcarcinoma: A study of 900 cases observed in a 60-year period.
47. Mihailovic J, Nikoletic K, Srbovan D. Recurrent disease in juvenile dif-
Surgery. 2008;144(6):980-987; discussion 987-988.
ferentiated thyroid carcinoma: prognostic factors, treatments, and out-
29. Vassilopoulou-Sellin R, Klein MJ, Smith TH, et al. Pulmonary metastases
comes. J Nucl Med. 2014;55(5):710-717.
in children and young adults with differentiated thyroid cancer. Cancer.
48. Papendieck P, Gruneiro-Papendieck L, Venara M, et al. Differentiated
1993;71(4):1348-1352.
thyroid cancer in children: Prevalence and predictors in a large cohort with
30. Kouvaraki MA, Shapiro SE, Fornage BD, et al. Role of preoperative thyroid nodules followed prospectively. J Pediatr. 2015;167(1):199-201.
ultrasonography in the surgical management of patients with thyroid can- 49. Rinaldi S, Plummer M, Biessy C, et al. Thyroid-stimulating hormone,
cer. Surgery. 2003;134(6):946-954; discussion 954-955. thyroglobulin, and thyroid hormones and risk of differentiated thyroid
31. Solorzano CC, Carneiro DM, Ramirez M, Lee TM, Irvin GL 3rd. Surgeon- carcinoma: The EPIC study. J Natl Cancer Inst. 2014;106(6):dju097.
performed ultrasound in the management of thyroid malignancy. Am Surg. 50. Jatana KR, Zimmerman D. Pediatric thyroid nodules and malignancy.
2004;70(7):576-580; discussion 580-582. Otolaryngol Clin North Am. 2015;48(1):47-58.
32. Gonzalez HE, Cruz F, OBrien A, et al. Impact of preoperative 51. Mussa A, De Andrea M, Motta M, Mormile A, Palestini N, Corrias A.
ultrasonographic staging of the neck in papillary thyroid carcinoma. Arch Predictors of malignancy in children with thyroid nodules. J Pediatr.
Otolaryngol Head Neck Surg. 2007;133(12):1258-1262. 2015;167(4):886-92 e1.
33. Stulak JM, Grant CS, Farley DR, et al. Value of preoperative ultrasonog- 52. Gupta A, Ly S, Castroneves LA, et al. A standardized assessment of
raphy in the surgical management of initial and reoperative papillary thyroid nodules in children confirms higher cancer prevalence than in
thyroid cancer. Arch Surg. 2006;141(5):489-494; discussion 494-496. adults. J Clin Endocrinol Metab. 2013;98(8):3238-3245.
34. Bargren AE, Meyer-Rochow GY, Delbridge LW, Sidhu SB, Chen H. 53. Buryk MA, Simons JP, Picarsic J, et al. Can malignant thyroid nodules be
Outcomes of surgically managed pediatric thyroid cancer. J Surg Res. distinguished from benign thyroid nodules in children and adolescents by
2009;156(1):70-73. clinical characteristics? A review of 89 pediatric patients with thyroid
35. Grigsby PW, Gal-or A, Michalski JM, Doherty GM. Childhood and ado- nodules. Thyroid. 2015;25(4):392-400.
lescent thyroid carcinoma. Cancer. 2002;95(4):724-729. 54. Enomoto K, Enomoto Y, Uchino S, Yamashita H, Noguchi S. Follicular
36. Handkiewicz-Junak D, Wloch J, Roskosz J, et al. Total thyroidectomy and thyroid cancer in children and adolescents: clinicopathologic features,
adjuvant radioiodine treatment independently decrease locoregional recur- long-term survival, and risk factors for recurrence. Endocr J. 2013;60(5):
rence risk in childhood and adolescent differentiated thyroid cancer. J Nucl 629-635.
Med. 2007;48(6):879-888. 55. Luster M, Handkiewicz-Junak D, Grossi A, et al. Recombinant thyrotropin
37. Jarzab B, Handkiewicz Junak D, Wloch J, et al. Multivariate analysis of use in children and adolescents with differentiated thyroid cancer: A
prognostic factors for differentiated thyroid carcinoma in children. Eur multicenter retrospective study. J Clin Endocrinol Metab. 2009;94(10):
J Nucl Med. 2000;27(7):833-841. 3948-3953.

The Endocrine Society. Downloaded from press.endocrine.org by [${individualUser.displayName}] on 12 January 2017. at 12:19 For personal use only. No other uses without permission. . All rights reserved.
ENDO 2016 PEDIATRIC ENDOCRINOLOGY 257

Hypoparathyroidism in Children

M08 SIGNIFICANCE OF THE CLINICAL PROBLEM


Presented, April 1 4, 2016 Hypocalcemia in childhood, although rare, is a relatively com-
mon cause of pediatric endocrinology consultations in children.
Although the differential diagnosis for hypocalcemia is exten-
Rachel I. Gafni, MD. Skeletal Clinical Studies Section, sive, it is important to consider parathyroid disorders in the
National Institute of Dental and Craniofacial Research, differential. Unlike adults, in whom hypoparathyroidism is
National Institutes of Health, Bethesda, Maryland 20892, most often a sequela of neck surgery, hypoparathyroidism in
E-mail: gafnir@nidcr.nih.gov children can be due to a wide variety of transient, genetic, and
acquired disorders. Making the correct diagnosis is essential
for developing a treatment plan, screening for associated ab-
INTRODUCTION
normalities and morbidities, and providing genetic counseling
Historical Overview
to the family.
In 1862, Sir Richard Owen described a small gland attached to
the thyroid (Owens gland) found while dissecting a rhinoceros
carcass. Despite this earlier report, it was not until 1880 when BARRIERS TO OPTIMAL PRACTICE
anatomist Ivar Sandstrom identified four glands of unknown Signs and symptoms of hypoparathyroidism are often
function attached to the thyroid in several different animal nonspecific, leading to a delay in recognition and
species and named them glandulae parathyroidae. Ten to fif- diagnosis.
teen years later, an association between tetany and thyroidec- Collection of urine samples essential in the evaluation
tomy was established and by 1900 it was determined that the of hypoparathyroidismis often difficult in young
tetany was, in fact, due to the loss of the parathyroids. After children or children with cognitive delay.
much debate, it was postulated that hypocalcemia was the Current therapies for hypoparathyroidism are suboptimal
cause of the tetany as it could be reversed with calcium and often associated with comorbidities.
therapy. A turning point occurred in 1925 when Dr James Genetic testing to determine the etiology of
Betram Collip, a member of the group that had earlier isolated hypoparathyroidism is costly and, in many cases, not
insulin, used a potent parathyroid extract to normalize blood commercially available.
calcium levels in parathyroidectomized dogs, establishing the
parathyroid gland as a direct regulator of calcium metabolism. LEARNING OBJECTIVES
Over the next 25 years, the pathophysiology of PTH excess and As a result of participating in this session, learners should be
deficiency became better defined. Numerous causes of able to:
syndromic and nonsyndromic hypoparathyroidism presenting Recognize the signs and symptoms of
in childhood were described over the years, heralded by the hypoparathyroidism and hypocalcemia.
observation of congenital absence of parathyroids and the thy- Identify the most common causes of hypoparathyroidism
mus by Sedlackova in 1955, Lobdell in 1959, and DiGeorge in and understand the underlying pathophysiology.
1965. In 1959, Dr Gerald Aurbach isolated PTH, followed by a Formulate an approach to evaluate the patient with
long and fruitful collaboration with Dr John Potts, resulting in hypocalcemia.
the development of a PTH immunoassay, a better understand- Develop a treatment and monitoring plan, keeping in
ing of PTHs physiologic activities mediated by cyclic AMP mind the potential comorbidities that may be associated
generation, and decoding of the hormone sequence in 1970. with the underlying disease and its therapy.
Identification of the PTH receptor and its coupling with Gs by
Dr Harald Juppner and colleagues soon followed. Resistance to STRATEGIES FOR DIAGNOSIS, THERAPY,
PTH (psuedohypoparathyroidism) was later recognized as be- AND/OR MANAGEMENT
ing due to abnormalities in GNAS, the gene encoding Gs, Background
rather than a mutation in the PTH receptor itself. In 1992, the At approximately the fifth week of gestation, the parathyroid
calcium-sensing receptor (CaSR) was discovered by Dr Ed- glands develop from the third and fourth pharyngeal pouches,
ward Brown, providing crucial insight into the regulation of with part of the third pouch also giving rise to the thymus. In
PTH secretion by the parathyroids as well as calcium handling utero, both PTH production by the fetal parathyroids and
by the kidney. With increasing knowledge of PTH physiology, PTHrP produced by the placenta play important roles in the
numerous genetic causes of hypoparathyroidism have been regulation of circulating calcium levels and the maintenance of
identified (Table 1), which will hopefully lead to the develop- maternal calcium transport across the placenta, respectively
ment of targeted therapies. (1). After birth, however, calcium metabolism is primarily

The Endocrine Society. Downloaded from press.endocrine.org by [${individualUser.displayName}] on 12 January 2017. at 12:19 For personal use only. No other uses without permission. . All rights reserved.
258 ENDO 2016 MEET-THE-PROFESSOR CLINICAL CASE MANAGEMENT

TABLE 1: Genetic Causes of Hypoparathyroidism


Inheritance Locus Gene Associated Abnormalities

Disorders of Parathyroid Gland Formation


Chromosome 22q11.2 deletion (DiGeorge/VCFS) Sporadic or AD 22q11.21-q11.23 TBX1 Thymic hypoplasia w/immune deficiency, conotruncal
DiGeorge/VCFS Complex 2 10p13-p14 defects, cleft palate, dysmorphic facies
Hypoparathyroidism, deafness, renal syndrome (HDR) AD 10p14 GATA3 Deafness, renal dysplasia
Hypoparathyroidsim-retardation-dysmorphism syndrome AR or AD 1q42.3 TBCE Growth retardation, developmental delay,
(Sanjad-Sakati syndrome/Kenny-Caffey Syndrome) microcephaly, microphthalmia, small hands/feet
Kenny Caffey Syndrome 2 AD 11q12 FAM111A Medullary stenosis, otic abnormalities
Mitochondrial Disease variable mtDNA MTP Encephalopathy, eye disease, heart block
Kearns-Sayre mtDNA Anemia, pancreatic dysfunction
Pearson Marrow- Pancreas syndrome mt tRNA Myopathy, encephalopathy, acidosis, stroke
MELAS 2p23 Hypoglycemia, hypotonia
LCHAD
Familial isolated hypoparathyroidism AR XR 6p24.2 Xq26-27 GCMB
Disorder of Parathyroid Gland Secretion
PTH gene mutations AR or AD 11p15.3 PTH
Autosomal Dominant Hypocalcemia (activating)
Type 1 AD 3q13.3-21 CaSR Variable degree of severity, hypomagnesemia,
Type 2 19p13.3 GNA11 hypercalciuria
Parathyroid Gland Destruction
Autoimmune polyendocrinopathy-candidiasis- AR 21q22.3 AIRE Adrenal insufficiency, mucocutaneous candidiasis,
ectodermal dystrophy syndrome malabsorption, vitiligo, alopecia, hepatitis,
pernicious anemia, hypogonadism, asplenia
Resistance to Parathyroid Hormone
Pseudohypoparathyroidism Ia AD 20q13.2-13.3 (mat) GNAS Albrights Hereditary Osteodystrophy (AHO), obesity,
other hormone resistance
Pseudopseudohypoparathyroidism AD 20q13.2-13.3 (pat) GNAS AHO without obesity or hormone resistance
Pseudohypoparathyroidism Ib Sporadic or AD 20q13.3 (pat) GNAS Mutation in differentially methylated region upstream
STX16 of Gs promoter, non-dysmorphic, may have TSH
resistance

regulated by PTH. PTH is translated as a prepro-peptide that is enhances gastrointestinal absorption of calcium and phospho-
cleaved and secreted into the circulation as an intact 84-amino- rus and also promotes osteoclastic bone resorption acting
acid peptide, with the biological activity residing in the first 34 through RANK ligand.
amino acids. Its effects occur through binding to the Calcium is critical for numerous physiologic processes, act-
PTH/PTH-rP receptor, a G protein coupled receptor. Binding ing both as an intracellular second messenger and as an impor-
to the receptor results in activation of the Gs pathway and tant extracellular cation at the neuromuscular junction. De-
generation of its second messenger, cAMP. creased calcium concentrations can lead to prolongation of the
Under normal conditions, ionized blood calcium levels are QT interval and neuromuscular excitability. Common symp-
maintained within a narrow normal range of approximately toms associated with hypocalcemia include paresthesias and
1.121.32 mmol/L (4.5-5.3 mg/dL), with greater levels in neo- muscle cramps. Severe hypocalcemia can be associated with
nates and infants. This tight regulation of circulating calcium is
tetany, seizures, and laryngospasm. Classic findings on physi-
controlled through constant adjustments of PTH secretion,
cal examination may include a positive Chvosteks sign
1,25-OH2 Vitamin D (1,25D) production, and renal handling of
(twitching of the circumoral and orbicularis oculi when the
calcium. Ambient calcium levels are sensed by the calcium-
facial nerve is tapped) and/or Trousseaus sign (carpopedal
sensing receptor (CaSR) located on the parathyroid and renal
spasm when a sphygmomanometer is inflated above the sys-
tubule cells (2). In cases of hypercalcemia, stimulation of the
tolic blood pressure for 3 minutes).
CaSR results in decreased renal calcium reabsorption and sup-
pressed PTH release. Conversely, hypocalcemia turns off the
CaSR, promoting renal reabsorption of calcium and stimulating Differential Diagnosis
PTH secretion. At the bone, PTH stimulates the differentiation Hypoparathyroidism is a disorder characterized by hypocalce-
of osteoclasts, promoting the release of calcium from bone into mia and hyperphosphatemia due to inadequate supply or effec-
the circulation. In the kidney, PTH plays several roles: 1) PTH tiveness of PTH. Although the most common cause of hypo-
acts at the distal renal tubule to promote reabsorption of cal- parathyroidism in adults is parathyroid damage or removal
cium; 2) PTH acts at the proximal tubule to promote phospha- during neck surgery, hypoparathyroidism in children is usually
turia; and 3) PTH acts at the proximal renal tubule to induce due to a genetic condition (3). These genetic disorders are
1--hydroxylase, thus increasing production of 1,25D which numerous and varied, affecting parathyroid gland development,

The Endocrine Society. Downloaded from press.endocrine.org by [${individualUser.displayName}] on 12 January 2017. at 12:19 For personal use only. No other uses without permission. . All rights reserved.
ENDO 2016 PEDIATRIC ENDOCRINOLOGY 259

PTH synthesis or secretion, autoimmune destruction, or PTH severe progressive osseous heteroplasia (11). Patients with
resistance (Table 1). PTH secretion may also be impaired in PHP type 1b, due to upstream mutations causing imprinting
states of hypo- or hypermagesemia (4). defects in GNAS, rather than mutation in GNAS itself, have
The most common genetic condition associated with hypo- PTH resistance and, often TSH resistance but a normal pheno-
parathyroidism is the Chromosome 22q11.2 deletion syndrome type. The etiology of PHP type 2 is unclear; however, it has
(DiGeorge/Velocardiofacial/Conotruncal anomaly face syn- been suggested that the PTH resistance is associated with
drome), with an incidence of approximately 1 in 4000 live births. Vitamin D deficiency that reverses with Vitamin D replace-
It is a contiguous deletion syndrome typically resulting in a loss of
ment (12). It is important to distinguish PHP from the high
30-40 genes on one copy of chromosome 22. The phenotype is
PTH, hypocalcemia, and hyperphosphatemia seen in children
broad and highly variable with classic features including hypo-
with renal failure or those who have received treatment with
parathyroidism, thymic hypoplasia, conotruncal cardiac anoma-
phosphate enemas.
lies, cleft palate, dysmorphic facial features, and cognitive
impairment. Hypoparathyroidism may be transient, intermit-
tent, or permanent, with a reported frequency of 40 70% (5). Diagnostic Evaluation
Another less common but important genetic cause of hypo- When hypoparathyroidism is suspected, a systematic evalu-
parathyroidism is autosomal dominant hypocalcemia (ADH) ation is crucial. As always, a careful history and physical
due to activating mutations of the CaSR (type 1) (6) or its with special attention to family history, comorbidities, and
downstream G protein subunit 11 (type 2) (7), which create identification of dysmorphic features is important. Once
an altered setpoint for PTH secretion, tricking the parathy- hypocalcemia is confirmed either with measurement of the
roid into believing that the circulating calcium is higher than it ionized calcium or correcting for albumin, blood levels of
actually is. This condition is associated with more significant intact PTH, phosphorus, magnesium, creatinine, 25-OH-
hypercalciuria and hypomagnesemia due to compounded CaSR Vitamin D, and 1,25-OH2-Vitamin D should be obtained
mishandling of calcium and magnesium by the kidney. while the calcium level is abnormal (13). Age-appropriate
Acquired hypoparathyroidism can be seen in children with normal ranges should be used for interpretation of labora-
the autoimmune-polyendocrinopathy-candidiasis-ectodermal
tory tests, especially serum phosphorus, which is signifi-
dystrophy syndrome (due to homozygous mutations in AIRE),
cantly higher in neonates and gradually decreases during
in which hypoparathyroidism typically develops within the
childhood and adolescents until finally reaching the adult
first decade of life and is often associated with candidiasis and
normal range. Skeletal radiographs may be useful for iden-
other autoimmune conditions, most commonly primary adrenal
insufficiency (8). Other less common disorders include tifying brachydactyly, osteoma cutis, medullary stenosis, or
hypoparathyroidism-deafness-renal syndrome due to mutations in other skeletal abnormalities seen with syndromic forms of
GATA3, hypoparathyroidism-retardation-dysmorphism syndrome hypoparathyroidism. When an autoimmune cause is sus-
due to mutations in TBCE, disorders of PTH production (eg, PTH pected, periodic screening for other autoimmune diseases,
gene mutation, GCMB mutation), and rare mitochondrial disor- particularly adrenal insufficiency is important. Urine creati-
ders (eg, Kearns-Sayre, MELAS) (9). Infiltrative diseases or con- nine and calcium should be measured; 24-hour collections
ditions associated with iron or copper overload can also lead to are ideal. Again, age-specific urine calcium/creatinine norms
parathyroid dysfunction. As in adults, hypoparathyroidism can should be applied and 24-hour collections should be cor-
occur following neck surgery, particularly during thyroidectomy. rected for body weight (normal calcium excretion 4
When PTH is elevated with concomitant hypocalcemia and mg/kg/d). Patients with PHP typically have low or undetectable
hyperphosphatemia, a diagnosis of pseudohypoparathyroidism urinary calcium excretion as the distal renal tubule responds
(PHP) with decreased PTH action should be considered. PHP appropriately to the elevated levels of PTH. In contrast, urinary
type 1a is due to inactivating mutations in the maternal GNAS calcium levels can be inappropriately normal or frankly el-
gene, an imprinted gene that is biallelically expressed in most
evated in patients with hypoparathyroidism, particularly in pa-
tissues but only expressed by the maternal allele in the proxi-
tients with activating mutations of the CaSR. In cases of
mal renal tubule, thyroid, pituitary, and gonad (10). The absent
hypercalciuria, renal ultrasound is recommended to evaluate
GNAS expression in the proximal tubule leads to reduced
for nephrocalcinosis (14). Calculation of the fractional excre-
1--hydroxylation of 25-hydroxy vitamin D and suppressed phos-
phate excretion, resulting in hypocalcemia, hyperphosphatemia, tion of magnesium may be indicated in patients suspected of
and an appropriately elevated PTH. Resistance to TSH, LH/FSH, having a primary magnesium disorder causing impaired PTH
and GHRH is also often observed. The 50% GNAS expression in secretion. However, one must be aware that magnesium is also
the distal tubules prevents hypercalciuria; reduced GNAS expres- a ligand for the CaSR; thus, patients with activating mutations
sion in other tissues, including the skeleton, causes the frequently also have hypomagnesemia and hypermagnesuria.
Albrights hereditary osteodystrophy phenotype. Heterotopic Genetic testing is commercially available for many of the
ossification may be present ranging from mild osteoma cutis to genetic causes of hypoparathyroidism.

The Endocrine Society. Downloaded from press.endocrine.org by [${individualUser.displayName}] on 12 January 2017. at 12:19 For personal use only. No other uses without permission. . All rights reserved.
260 ENDO 2016 MEET-THE-PROFESSOR CLINICAL CASE MANAGEMENT

Management Questions
For patients with acute life-threatening symptoms of hypocal- 1. What is the most likely cause of her hypocalcemia?
cemia, IV calcium gluconate is the treatment of choice; this 2. What other hormone resistance syndromes might you
often must be followed with a continuous calcium infusion for see in this patient?
several hours until the patient has stabilized and can tolerate 3. What is the preferred treatment for this patient?
oral medication. In patients with hypocalcemia, elevated
PTH and low 25-OH-Vitamin D, it is important to correct
Answers
vitamin D deficiency before confirming a diagnosis of PHP.
1. This patient most likely has pseudohypoparathyroidism
PHP is treated with calcitriol (or alphacalcidiol or high dose
type 1b due to a methylation defect in GNAS. These
ergo/cholecalciferol) and calcium to normalize serum calcium
patients have biochemical PTH resistance but lack the
and PTH and avoid hyperparathyroid bone disease. Hypercalciuria
phenotype of Albrights hereditary osteodystrophy seen
in PHP rarely occurs except with overtreatment. Calcium
in PHP type 1a. However, this could be PHP type 2,
supplements, given in divided doses, ideally with food, and
which could resolve with correction of her vitamin D
calcitriol (or high dose ergo/cholecalciferol) are also used for
deficiency. Thus, her vitamin D should be repleted prior
hypoparathyroidism, although serum calcium should be tar-
to confirming the diagnosis.
geted to the low-normal range to avoid exacerbation of
2. In PHP 1a there is often resistance to several other
hypercalciuria and nephrocalcinosis, which can lead to renal
hormones that act through G protein coupled receptors
insufficiency. Increasing fluid intake and reducing dietary so-
dium may reduce the risk of renal calcification. When urinary including LH, FSH, TSH, and GHRH. In contrast, PHP
calcium excretion is extremely elevated, some physicians pre- type 1b is generally only sometimes associated with
scribe thiazides to increase renal calcium reabsorption; how- mild TSH resistance.
ever, this therapy can lead to hypokalemia and its effectiveness 3. In PHP 1b, resistance to PTH is primarily limited to the
in reducing renal morbidity in hypoparathyroidism has not proximal renal tubules, leading to impaired 1--
been established. Frequent monitoring of blood and urine as hydroxylation of 25-OH-Vitamin D (and inadequate
well as renal ultrasonography is recommended. My practice is 1,25-OH2-Vitamin D) and impaired renal phosphate
to also monitor urine citrate and treat low urine citrate with excretion. Unlike other forms of hypoparathyroidism, in
potassium citrate as hypocitraturia is a known risk factor for PHP, PTH is functional in the distal tubules and tubular
nephrocalcinosis and nephrolithiasis; again, this treatment has reabsorption of calcium is normal. Thus, patients with
not be rigorously studied in hypoparathyroidism. Subcutaneous PHP do not typically experience hypercalciuria and
PTH 1-84 has recently been approved as a treatment for adults nephrocalcinosis as in other forms of
with hypoparathyroidism not due to mutations in the CaSR. hypoparathyroidism. Therefore, the goal of therapy in
However, it contains a black box warning describing a poten- PHP is to increase the blood calcium levels sufficiently
tial risk of osteosarcoma, particularly in those with open to normalize the PTH levels and prevent
growth plates and is therefore contraindicated in children. Fur- hyperparathyroid bone disease. This is accomplished
ther drug development and long-term safety studies are needed. with 1--hydroxylated vitamin D analogs such as
calcitriol and oral calcium.
MAIN CONCLUSIONS
The differential diagnosis of hypoparathyroidism in children is Case 2
lengthy. However, a careful history and physical examination A 6-year-old boy is being evaluated for a learning disability as
combined with focused studies looking for associated features he has been noted to have difficulty concentrating in school.
will usually narrow the differential so that the appropriate Growth and development have been otherwise normal and his
treatment and genetic counseling can be applied. Therapy physical examination is unremarkable. Laboratory tests reveal
should be tailored to the underlying disease, recognizing the a calcium level, 6.8 mg/dL (8.5-10.5 mg/dL), phosphorus, 8
potential complications and comorbidities. mg/dL (3.5-6 mg/dL), normal 25-OH-Vitamin D, and an intact
PTH level of 13 pg/mL (12-65 pg/mL). On family history, you
learn that his father has a history of kidney stones and gets
CASES
frequent cramping in his hands; In addition, he had two broth-
Case 1
A 12-year-old girl presents to her primary care physician after an ers who died in infancy from a seizure disorder.
episode of tetany in her hands and feet. Lab tests show calcium,
5.4 mg/dL (8.5-10.6); phosphorus, 7.1 mg/dL (3.5-6); intact PTH, Questions
403 pg/mL (12-65); and 25-OH-Vitamin D, 11 ng/mL (25-80). 1. What is the most likely cause of hypoparathyroidism
Growth and development has been normal and her physical ex- and what is the inheritance pattern of this disorder?
amination is remarkable only for a Chvosteks sign. 2. Why is the PTH level in the normal range?

The Endocrine Society. Downloaded from press.endocrine.org by [${individualUser.displayName}] on 12 January 2017. at 12:19 For personal use only. No other uses without permission. . All rights reserved.
ENDO 2016 PEDIATRIC ENDOCRINOLOGY 261

3. What other tests would you like to see? What is the Disclaimer: The opinions expressed reflect those of the author
recommended treatment? and not of the National Institutes of Health or the Unites States
federal government.
Answers
1. This child and his father have autosomal dominant REFERENCES
1. Simmonds CS, Kovacs CS. Role of parathyroid hormone (PTH) and
hypocalcemia due to an activating mutation of the PTH-related protein (PTHrP) in regulating mineral homeostasis during
CaSR. Delayed diagnosis is not uncommon and the fetal development. Crit Rev Eukaryot Gene Expr. 2010;20:235-273.
severity of symptoms can be variable, even within 2. Brown EM. Role of the calcium-sensing receptor in extracellular calcium
homeostasis. Best Pract Res Clin Endocrinol Metab. 2013;27:333-343.
families carrying the same mutation. In fact, this 3. Gafni RI, Levine MA. Genetic causes of hypoparathyroidism. In: Naveh-
father was diagnosed at the same time as his son. Many T, ed. Molecular biology of the parathyroid. New York: Kluwer
Although this child never had a hypocalcemic Academic/Plenum Publishers, 2005;159-178.
4. Knoers NV. Inherited forms of renal hypomagnesemia: An update. Pediatr
seizure, this is the most likely cause of death in his Nephrol. 2009;24:697-705.
two brothers. 5. McDonald-McGinn DM, Sullivan KE. Chromosome 22q11.2 deletion syn-
2. Activating mutations of the CaSR alter the calcium set drome (DiGeorge syndrome/velocardiofacial syndrome). Medicine (Balti-
more). 2011;90:1-18.
point for PTH secretion by the parathyroid glands. 6. Baron J, Winer KK, Yanovski JA, et al. Mutations in the Ca(2)-sensing
Thus, whereas the parathyroid glands may be receptor gene cause autosomal dominant and sporadic hypoparathyroid-
underactive, they are otherwise normal and do contain ism. Hum Mol Genet. 1996;5:601-606.
7. Nesbit MA, Hannan FM, Howles SA, et al. Mutations affecting G-protein
the machinery necessary to produce and secrete PTH. subunit 11 in hypercalcemia and hypocalcemia. N Engl J Med. 2013;368:
Thus, some patients with activating mutations of the 2476-2486.
CaSR may have detectable PTH levels, albeit not high 8. Husebye ES, Perheentupa J, Rautemaa R, Kampe O. Clinical manifesta-
tions and management of patients with autoimmune polyendocrine syn-
enough to normalize blood calcium. drome type I. J Intern Med. 2009;265:514-529.
3. Patients with ADH often have inappropriately normal 9. Grigorieva IV, Thakker RV. Transcription factors in parathyroid develop-
or frankly elevated urinary calcium excretion, even ment: Lessons from hypoparathyroid disorders. Ann N Y Acad Sci.
2011;1237:24-38.
prior to therapy, placing them at increased risk for 10. Levine MA. An update on the clinical and molecular characteristics of
nephrocalcinosis, nephrolithiasis, and renal pseudohypoparathyroidism. Curr Opin Endocrinol Diabetes Obes.
insufficiency. A renal ultrasound should be obtained 2012;19:443-451.
11. Adegbite NS, Xu M, Kaplan FS, Shore EM, Pignolo RJ. Diagnostic and
and the patient should be started on calcitriol and mutational spectrum of progressive osseous heteroplasia (POH) and other
calcium supplements in divided doses. Magnesium forms of GNAS-based heterotopic ossification. Am J Med Genet A.
levels should also be checked as these patients 2008;146a:1788-1796.
12. Lemos MC, Thakker RV. GNAS mutations in pseudohypoparathyroidism
frequently require magnesium supplementation. type 1a and related disorders. Hum Mutat. 2015;36(1):11-19.
Blood calcium levels should be targeted at or slightly 13. Shaw N. A practical approach to hypocalcaemia in children. Endocr Dev.
below the lower limit of the normal range, just high 2009;16:73-92.
14. Boyce AM, Shawker TH, Hill SC, et al. Ultrasound is superior to com-
enough so that the patient is not having significant puted tomography for assessment of medullary nephrocalcinosis in hypo-
symptoms of hypocalcemia. parathyroidism. J Clin Endocrinol Metab. 2013;98:989-994.

The Endocrine Society. Downloaded from press.endocrine.org by [${individualUser.displayName}] on 12 January 2017. at 12:19 For personal use only. No other uses without permission. . All rights reserved.
262 ENDO 2016 MEET-THE-PROFESSOR CLINICAL CASE MANAGEMENT

MODY and Other Monogenic Forms of Diabetes

M42 NDM
Presented, April 1 4, 2016 NDM was considered to be a rare entity (1:500 000 live
births), primarily due to defects in genes regulating the
coordinated developmental formation and function of beta
Mark A. Sperling, MD. Department of Pediatrics, cells (4). A transient form, remitting at approximately 3
Childrens Hospital of Pittsburgh, University of Pittsburgh months but relapsing again to diabetes in the pubertal period
School of Medicine, Pittsburgh, Pennsylvania 15224; and was found to be due to an imprinted gene on chromosome
Department of Pediatrics, Icahn School of Medicine at 6q24. With the increased sophistication of molecular biol-
Mount Sinai, New York 10029, E-mail: masp@pitt.edu ogy and gene sequencing, approximately 20 genes have been
identified and can be determined via targeted next-
INTRODUCTION generation sequencing (NGS); several demonstrate remit-
Historical Overview relapse patterns (5, 6). The predominant genes causing
Maturity-onset diabetes of the young (MODY) was so named NDM are 6q24, the KATP genes and the insulin gene; rarer
by Fajans, who recognized a form of mild diabetes respon- entities present with distinct patterns of developmental de-
sive to tolbutamide, similar to maturity-onset diabetes mellitus fects, sometimes as named syndromes subsequently detailed.
(DM), which we now know as type 2 diabetes mellitus Homozygous defects in the glucokinase enzyme, pdx1, and
(T2DM) (1, 2). This entity segregated in some families and hnf1 cause NDM, whereas their heterozygous forms cause
patients were younger, lean not obese, and often were asymp- MODY. The discovery that high-dose oral sulfonylurea
tomatic despite abnormal oral glucose tolerance tests (1960s). (SU) drugs can overcome the defective insulin secretion in
Hence, he named the condition maturity-onset diabetes of KATP gene mutations has revolutionized the management of
childhood or of the young. such patients and represents a triumph of research (7). In-
Contemporaneously, Tattersall in the United Kingdom re- deed, most of the information presented and discussed here
ported a mild form of DM with autosomal-dominant inheri- was completely unknown as recently as 20 years ago and
tance; together with Fajans, he called it MODY: maturity-onset has come to the fore in the last 5-10 years (13, 57).
diabetes of youth. He found varying degrees of impaired insu-
lin secretion in these patients. SIGNIFICANCE OF THE CLINICAL PROBLEM
The molecular-genetic basis of MODY was discovered as Patients with MODY constitute approximately 23% of all pa-
being due to transcription factors for insulin or the enzyme tients diagnosed with diabetes and depending on age at discovery
glucokinase between 1992 and 1997; the number refers to the are misdiagnosed as having type 1 diabetes mellitus (T1DM) and
sequence of discovery (1). inappropriately treated with insulin; T2DM and treated with insu-
1996: Bell found MODY 1 as due to hnf4 (hepatic lin sensitizers, DPP4 inhibitors, GLP-1 agonist or other agents; or
nuclear factor) MODY 3 as due to hnf1. gestational diabetes with postpregnancy monitoring. Glucokinase
1996: Froguel and Hattersley described abnormalities in mutations (MODY2) affect approximately 1:1000 individuals (8)
the enzyme glucokinase, causing familial diabetes in and treatment with any agent is generally not necessary given that
1992; however, the molecular basis for this form of micro- and macrovascular complications do not occur even after
MODY was discovered in 1996 and named MODY 2. 50 years as subsequently detailed (9). Treatment may be appropri-
1997: Stoffers and Habener discovered insulin promoter ate for MODY2 during pregnancy to avoid macrosomia in the
factor 1(ipf-1) also known as pancreatic-duodenal fetus. MODY3 and MODY1 generally respond to SU agents
homeobox-1(pdx-1) as the cause of MODY4. initially, although they require monitoring because they lose insu-
1997: Horikawa and Bell showed mutations in hnf1 lin secreting capacity in time (6). Misdiagnosis is accompanied by
associated with DM and renal cysts and called it
inappropriate genetic counseling; as an autosomal-dominant trait,
MODY5.
50% of all children are likely to be affected, whereas with T1DM
MODY 6 is due to mutations in NEUROD1, another
the risk is 510%. It is conceivable that an affected parent or
transcription factor.
sibling likewise has been misdiagnosed, inappropriately treated,
Online Mendelian Inheritance in Man (OMIM) now lists 11 and given incorrect genetic counseling. MODY1-3 constitute ap-
forms of MODY; Bonnefond and Froguel list 13 forms (3) by proximately 85% of all known forms of MODY, so genetic
also including mutations in abcc8 (SUR1) and kcnj11(Kir6.2), screening for these three forms would seem to be warranted in
which are the ATP- regulated potassium channel (KATP) genes those who have a strong family history of diabetes, a milder
governing insulin secretion, usually associated with neonatal degree of hyperglycemia, and especially if they are negative for
diabetes mellitus (NDM). autoantibodies to islet-cell components. Most reports now list

The Endocrine Society. Downloaded from press.endocrine.org by [${individualUser.displayName}] on 12 January 2017. at 12:19 For personal use only. No other uses without permission. . All rights reserved.
ENDO 2016 PEDIATRIC ENDOCRINOLOGY 263

NDM as occurring 1:100 000 200 000 births; in regions of creased urination. Some affected infants with NDM escape
high consanguinity, the overall incidence is as high as detection until they present with ketoacidosis.
1:20 000 40 000 (10). For NDM, a correct diagnosis confirmed A second barrier is the reluctance or difficulty in perform-
by molecular typing may permit treatment with oral agents that ing the appropriate genetic test. Some insurance companies
restore endogenous insulin secretion, improve metabolic control, are averse to paying for molecular diagnostic testing in
and avoid painful injections or the use of insulin delivery devices cases of suspected MODY. Yet, a simple calculation sug-
such as pumps (7). Future pregnancies can be monitored by NGS gests that such testing would be cost effective with substan-
performed on fetal DNA in maternal blood beyond the first weeks tial cost savings if the patient is erroneously labeled as
of gestation; recent reports suggest that early diagnosis of some having T1DM and treated with insulin instead of oral SU
forms of syndromic DNA occur before the manifestations found medications. This also holds true for newborns diagnosed
in typical cases, suggesting potential avoidance of these compli- with NDM; here, molecular diagnostics is essential to define the
cations by earlier treatment eg, developmental delay, epilepsy, defect and treat appropriately with oral SU agents if indicated.
neonatal diabetes (DEND syndrome) (5, 11). Finally, the signifi- Again, if the cause is a defect in the KATP channel amenable to
cance of these clinical problems are magnified well beyond their treatment with oral agents, cost saving and better metabolic con-
known incidence; increasingly, it is recognized that mutations in trol would result. As costs of genetic testing decrease and aware-
genes that cause MODY and NDM are implicated in genome- ness of the genetic entities increase this barrier may diminish and
wide association studies of typical T2DM, emphasizing that hopefully disappear (16).
T2DM is the result of insulin-secretory defects that impair the
ability to respond to insulin resistance by sufficiently increasing LEARNING OBJECTIVES
insulin secretion (12). As a result of participating in this session, learners should be
able to:
BARRIERS TO OPTIMAL PRACTICE Distinguish between T1DM, T2DM, and monogenic
There are two major barriers to optimal practice. First, many forms of the disease complex.
in practices and even in academic medical centers are un- Recognize the diagnostic, therapeutic, and prognostic
aware or less aware of the entity of MODY, considered rare. significance of establishing the correct molecular
Patients with MODY are generally lean, have a family diagnosis.
history of diabetes in two to three previous generations, Assess the contribution of genetic defects to the global
onset occurs before age 35 years, manifestations are mild, burden of DM.
and affected patients may be asymptomatic (MODY2) or
have only mild symptoms of increased urination (MODY 1 STRATEGIES FOR DIAGNOSIS, THERAPY,
and 3). Given this scenario, many would suspect T1DM but AND/OR MANAGEMENT
antibodies to islet-cell components are absent, and C-peptide The American Diabetes Association lists three types of diabe-
measurements reveal considerable -cell reserve (C-peptide tes (17). T1DM is considered an autoimmune disease by virtue
0.8 ng/mL), which also manifests as a prolonged honey- of its associations with thyroid, gastrointestinal, adrenal, and
moon phase (13). It is not uncommon for hyperglycemia to cutaneous (vitiligo) immune syndromes and its predominant
be discovered during a routine blood or urine laboratory gene associations (HLA complex, Variable Number of Tandem
test; this is most true in patients with MODY2, especially Repeats [VNTR] in the insulin gene, AutoImmune Regulator
with healthy young women during pregnancy undergoing [aire], protein tyrosine phosphatase, non-receptor type 22
routine glucose tolerance screening tests at the beginning of [ptpn22], Cytotoxic T-Cell Lymphocyte Antigen 4 [ctla4]), all
the third trimester. In NDM, routine ultrasound may reveal important immune regulators. Typically, patients with T1DM
intra-uterine growth retardation and babies are born small are young (25 y at diagnosis). T2DM is associated with
for gestational age. This is due to insulin deficiency given insulin resistance in the context of relative insulin deficiency;
that insulin (along with IGF-I) is a major regulator of fetal obesity is the major cause of insulin resistance and Genome-
growth. Thus, infants with hyperinsulinism typically display Wide Association Studies (GWAS) studies link several genes
macrosomia at birth, similar to infants born to diabetic associated with MODY or NDM. Gestational diabetes is the
mothers whose hyperglycemia induces higher insulin secre- third category. Monogenic forms of diabetes are listed under
tion in the fetus in response to the nutrient mixture trans- the category of Other. In children/adolescents/young adults
ferred from the mother (14, 15). By contrast, a fetus with presenting with suspected T1DM, negative autoantibody deter-
defects in insulin secretion is small at birth; the more severe minations together with C-peptide greater than 0.8 ng/mL oc-
the insulin-secretory defect, the more severe is the degree of cur in approximately 15% and among these approximately
weight deficit expressed as the z score for weight at the 10 15% will have MODY, especially if family history is
corrected gestational age. Investigation of subsequent failure positive in two to three generations (13). Molecular screening
to thrive in such infants reveals hyperglycemia and in- could be restricted to MODY 1, 2, and 3, which together

The Endocrine Society. Downloaded from press.endocrine.org by [${individualUser.displayName}] on 12 January 2017. at 12:19 For personal use only. No other uses without permission. . All rights reserved.
264 ENDO 2016 MEET-THE-PROFESSOR CLINICAL CASE MANAGEMENT

account for approximately 85% of all forms of MODY; no Blood glucose, 252 mg/dL; insulin, 29 uU/mL, C-
specific treatment is indicated for MODY2, the commonest peptide, 3.58 ng/mL.
form found in children and otherwise-healthy, nonobese preg- Family History: father diagnosed at age 29 years with
nant women. For MODY 1 and 3, SU medication should be T2DM and fathers two sisters diagnosed with DM: one
tried initially as most MODY3 and many patients with with T1DM and other with T2DM.
MODY1 respond. Genetic counseling must stress the Father treated with metformin and glipizide and runs
autosomal-dominant nature, so that each subsequent offspring HbA1c of 6-6.5%. There are no complications after 30
(or sibling) has a 50% chance of being affected. Likewise, years of DM.
parents previously diagnosed as having T2DM should be Father recalls that his father died from leukemia but
checked for likely having the same mutation, with adjustment was found to have high blood sugar during treatment
of therapy. (Figure 1).
In summary, patients with MODY differ from T1DM by: MODY 3 is due to numerous different mutations in the HNF1
Not developing ketoacidosis. gene and is generally considered to be the most common form of
Lacking common islet-cell antibodies (insulin, GADA, MODY; HNF1 is expressed in liver and -cells and interacts
IA-2A, ICA). with HNF4, thus having a relationship to MODY1. Progressive
Displaying an autosomal dominant inheritance pattern. decline of insulin secretion occurs with progressive age so that the
Onset up to 35 years of age. ability to respond to SU drugs is lost, insulin may be required, and
They differ from T2DM by: vascular complications of DM occur.
Being generally lean. Another common cause of MODY is due to mutations in the
Having normal insulin sensitivity. enzyme glucokinase so that there is decreased insulin secretory
response to normal glucose concentrations, but a normal
These characteristics are illustrated in the following Case 1 history: insulin-secretory response at higher glucose concentrations.
This results in mild diabetes that may be discovered at routine
Case 1 testing even in young children or during pregnancy; family
A 15-year-old previously healthy white male, presented with history may be positive. In our own series at a tertiary medical
hyperglycemia discovered during a routine physical examina- center for children, MODY2 was more common than MODY3.
tion. Due to strong family history of diabetes, the family In a recent report, patients with MODY2 followed for close
requested a screening test for him. A brother, now aged 17 to 50 years had no increased incidence of micro- or
years, was diagnosed at age 9 years with T1DM; a sister now macrovascular complications when compared with a normal
aged 13 years was diagnosed at age 8 years with T1DM. Both nondiabetic, age-matched control population (9). Therefore,
are on insulin. His random metered blood glucose was 230 treatment with glucose-lowering agents is not indicated and
mg/dL and urine was 1 for glucose. He denied any symptoms likely would not change the transient postprandial hyperglyce-
of DM such as weight loss, polyuria, polydipsia, nocturia, or mia. Some have recommended treatment of women who have
enuresis. Review of systems was otherwise negative. Past MODY2 with insulin during pregnancy to limit hyperglycemia
medical history is non-contributory. Physical examination; that could promote increased insulin secretion and hence, fetal
healthy appearing, robust adolescent. macrosomia. It is not clear that this treatment is effective, nor

Figure 1. Family tree for Case 1. Each sibling plus father and both aunts have 779C>T resulting in change from
Thr260Met associated with MODY3. All siblings switched to glipizide with reduction of HbA1c to near normal.

The Endocrine Society. Downloaded from press.endocrine.org by [${individualUser.displayName}] on 12 January 2017. at 12:19 For personal use only. No other uses without permission. . All rights reserved.
ENDO 2016 PEDIATRIC ENDOCRINOLOGY 265

TABLE 1.
Classification of MODY
Type Gene Chromosome Treatment Distinguishing Features
MODY 1: HNF4alfa 20q12-q13.1 INSULIN/SU Hypoglycemia in newborn
MODY 2: Glucokinase 7p15-p13 EXERCISE/DIET HbA1c rarely 7.2%; no complications
MODY 3: HNF1alfa 12q24.2 INSULIN/SU Most common form in adults
MODY 4: PDX-1/IPF-1 13q12.2 INSULIN Onset may be later; T2DM pattern
MODY 5: HNF1beta 17q12 INSULIN/SU Renal Cysts; Genital abnormalities
MODY 6: NeuroD1-beta2 2q32 INSULIN Rare

MODY 7-13 RARE AND EACH IS 1% OF MODY (OMIM; Bonnefond A et al. Cell Metab 2015;3:357).
MODY 1, 2, and 3 together constitute 85% of all known MODY syndromes; if MODY5 is included, almost 90% of all MODY syn-
dromes are defined. MODY2 is stable, rarely requires treatment except during pregnancy to protect the fetus from hyperglycemia, and
has an excellent prognosis for avoidance of vascular complications. MODY1 and 3 frequently respond to sulfonylurea drugs (SU) ini-
tially but may progress to insulin dependence and risk for development of vascular complications dependent on metabolic control.
CRITERIA FOR DIAGNOSING MODY
1. Hyperglycemia usually diagnosed before age 25 yr in at least 1 and ideally 2 family members.
2. Autosomal dominant inheritance, with a vertical transmission of diabetes through 3 or more generations, and a similar phenotype
shared by diabetic family members.
3. Absence of insulin therapy at least 5 yr after diagnosis or significant C-peptide levels even in a patient on insulin treatment (Some
may be inappropriately diagnosed asT1DM and treated with insulin or T2DM and treated with a variety of oral agents).
4. Insulin levels are often in the normal range, although inappropriately low for the degree of hyperglycemia, suggesting a primary defect
in -cell function.
Fajans S et al. NEJM. 2001;345;971-980.

indicated, given that 50% of fetuses are unaffected by the same improved metabolic control; and genetic counseling is essential
mutation and hence would respond to maternal hyperglycemia to transmit the risk of recurrence in future children or the
by increased insulin secretion; the 50% affected by the same existence of the same entity in parents or siblings misdiagnosed
mutation require the higher glucose to respond with normal as having T1DM or T2DM and treated with the wrong medi-
insulin secretion to avoid growth retardation. In one report, the cation (eg, insulin for suspected T1DM; metformin, GLP-1
prevalence of MODY2 was estimated at 1:1000 (0.1%) of the agonists, or DPP4 inhibitors for T2DM) or any medication
population including pregnant women (8). when none is indicated as in the case of MODY2.
MODY1 due to HNF4 mutations is much less common A common question is the cost effectiveness of performing
than MODY2 and MODY3 but is similar to MODY3 in pro- molecular testing, and the denial of payment for the costs by
gressive loss of responsiveness to oral agents, requirement for health insurance. In the case of a confirmed diagnosis, the cost
insulin, and risk for complications. effectiveness is clear because the costs of monthly insulin at
MODY4 due to mutations in PDX1 (pancreas duodenum
current prices far outweighs the cost of a molecular diagnosis
homeobox1) are rare with no specific diagnostic clues.
targeted to the three major forms (MODY 1, 2, and 3), which
MODY5 is of interest because the mutation in HNF1 is
together constitute approximately 85% of all forms. A formal cost
associated with renal cysts, vaginal/uterine malformations, ab-
analysis has been investigated and reported (16). Table 1 summa-
normal liver function, and nondiabetic renal disease. The renal
rizes the common causes, responsible gene defects, and their
disease often precedes the manifestations of diabetes; thus,
chromosomal location, treatment, and distinguishing features of
referral from a renal clinic because of the discovery of glucos-
MODY. The criteria for diagnosing MODY are also listed.
uria is not unusual.
All other forms of MODY, including mutations in the insu-
lin gene itself, the SU receptor SUR1 (ABCC8), and inward NDM
rectifying potassium channel Kir6.2(KCNJ11) are rare, but Diabetes diagnosed in the first 6 months of life is termed NDM
have been reported as causes of MODY (3, 6). and is due to several different mutations in genes regulating the
Those working in this field believe that in the circumstance formation or function of the -cell (Table 2). T1DM with
of strong family history of young onset, mild, nonketotic hy- markers of autoimmunity does not occur during the first 6
perglycemia, negative antibodies, and lean body habitus, mo- months of life. Nevertheless, some extend the period at diag-
lecular diagnostic testing is justified because the prognosis is nosis to 9-12 months, although T1DM may exist by this time.
considerably better; oral agents such as the SU drugs induce Three major forms of presentation are characteristic. The first
endogenous insulin secretion into the portal vein with resultant is characterized by moderate to severe intra-uterine growth

The Endocrine Society. Downloaded from press.endocrine.org by [${individualUser.displayName}] on 12 January 2017. at 12:19 For personal use only. No other uses without permission. . All rights reserved.
266 ENDO 2016 MEET-THE-PROFESSOR CLINICAL CASE MANAGEMENT

TABLE 2.
Neonatal Diabetes Mellitus (NDM)
45% Transient (TNDM) 45% Permanent (PNDM) 10% Syndrome-Rare
TNDM1 70% of TNDM Involves 6q24 EIF2AK3-Spondyloepiphyseal dysplasia, renal
anomalies
Disturbance of Imprinted Genes PLAGL 50% KCNJ11 DEND Syndrome FOXP3-IPEX Immunodysregulation, Polyendocrinopathy,
(ZAC) and HYMAI caused by Enteropathy, X-linked
a) paternal uniparental disomy (sporadic) 30% Insulin Gene Mutation
b) paternal chromosome 6 duplication
(dominant transmission)
c) relaxation of imprinting maternal
hypomethylation - sporadic or
mutation in the transcription
factor ZFP57
TNDM2 15% ABCC8 DEND GLIS3-Hypothyroid, hepatic fibrosis, glaucoma, cystic
kidneys, devltal. delay
3% GCK homozygous PTF1-Pancreatic and cerebellar agenesis
15% of TNDM ABCC8 (SUR1) 2% Others : Homozygous mutations RFX6. Digestive system defects known as Mitchell-Riley
Mutations in PDX-1 and HNF1B PDX-1 Syndrome
homozygous HNF1B
10% of TNDM KCNJ11 (KIR6.2) Heterozygous mutations in GCK, NEUROG3 with congenital malabsorptive diarrhea
Mutation PDX-1, HNF1B Cause MODY2, and enteroendocrine cell dysgenesis
MODY4, MODY5, respectively
5% of TNDM Insulin Gene DEND Developmental Delay, GATA6 with pancreatic agenesis and cardiac
Mutation Epilepsy, Neonatal Diabetes malformations

retardation, and followed by failure to thrive and the discovery Genetic analysis showed uniparental disomy of fathers
of hyperglycemia and glucosuria. Treatment with insulin re- chromosome 6.
sults in remarkable catch-up growth, and remission at approxi- Diagnosis: classical transient form of NDM.
mately 3-4 months followed by recurrence in at least one half
Most cases of transient NDM (70%) are due to uniparental
later during life. During the period of remission, insulin secre-
disomy of the fathers chromosome 6, paternal duplication of
tion seems to be normal. Case 2 is representative.
the 6q24 region, or an abnormal methylation of the maternal
6q24. The involved genes are listed in Table 2 and details
Case 2
Born to a healthy, 30-year-old mother at 35 and 5/7 found in the references. The remaining third of transient cases
weeks gestation. are due to activating mutations in the genes of the ATP-
Birth weight, 2045 g-IUGR (Intra Uterine Growth regulated potassium channel, namely ABCC8 (SUR1) or
Retardation) diagnosed by ultrasound in utero. KCNJ11 (Kir6.2) and occasionally other genes as listed.
Respiratory distress and cyanosis. Apgar score 8/8. A second form is permanent from the outset without a remis-
Recovered with O2 sion phase; it has later onset and variable manifestations including
Developed hyperglycemia in first 36 hours of life diabetic ketoacisosis, and in severe cases developmental delay and
epilepsy, so-called developmental delay epilepsy neonatal diabe-
Persistence required insulin. Magnetic resonance imag-

tes (DEND). The most common forms are due to activating
ing showed pancreas.
mutations of KCNJ11, insulin and ABCC8 genes, which are more
Insulin carefully titrated against blood glucose 0.01 u/kg.
severe than those causing transient NDM. Severity is determined
By age 48 days, the weight was 2.82 kg and the baby

was breastfeeding. by the site of the mutation and the proportion of abnormal vs
Insulin, 0.03 u/hr via pump using 1:10 dilution of fast- normal components in each of the four subunits of the KATP
acting insulin was continued. channels. Other rarer causes of permanent diabetes are listed in
Parents were trained to monitor and use insulin pump at Table 2 and illustrated in Case 3.
age 80 days taking 0.5 u/kg/d 45% basal. Of note, most mutations in the KCNJ11 are amenable to
correction of insulin secretion by SU therapy at higher doses
Length, 54.5 cm. Weight, 4.7 kg. No dysmorphic
than used in adults. Titration of these higher doses should be
features.
performed in a hospital setting under supervision of trained
Insulin discontinued at age 4 months. personnel.

The Endocrine Society. Downloaded from press.endocrine.org by [${individualUser.displayName}] on 12 January 2017. at 12:19 For personal use only. No other uses without permission. . All rights reserved.
ENDO 2016 PEDIATRIC ENDOCRINOLOGY 267

Case 3 genes responsible for the formation of the entero-insulin


Born full term, with an unremarkable prenatal course, axis. Treatment with insulin results in catch-up growth;
and birth weight, of 5 lbs, 8 oz. almost one half have transient diabetes that remits, but may
At 3 weeks of age, the baby started having episodes of recur during periods of stress (eg, puberty or pregnancy)
vomiting and multiple formula changes. while about half have permanent diabetes from the outset.
At age 9 weeks the baby had increased episodes of Depending on the severity of the mutation, presentation may
vomiting, decreased by-mouth intake, lethargy, and
be considerably delayed so that relatives considered to have
apparent respiratory distress.
T2DM actually have the same mutation as their offspring
Presented in shock, and diabetic keto-acidosis (DKA) as with NDM. Indeed, genetic studies in patients considered to
manifested by the following laboratory values: have classical T2DM demonstrate associations with the
Glucose, 1007 mg/dl; pH, 6.9; K, 6.9 meq/L; Na, 166 same genes that cause MODY including KCNJ11, HNF1,
meq/L, BUN, 62 mg/dL and creatinine, 1.3 mg/dL. Base HNF1, HNF4, and others.
deficit, 29, HbA1c, 15.8%, glycated hemoglobin
(HbA1c), 11% (post transfusion). Other Rare Single Gene Defects Associated With
Initially intubated, mechanically ventilated for 3 days, Diabetes
Rx for DKA and hypernatremic dehydration. Wolfram Syndrome: Diabetes Insipidus, Diabetes
Maintained on insulin continuous subcutaneous insulin infu- Mellitus, Optic Atrophy, Deafness
Wolfram syndrome is an autosomal-recessive disorder charac-
sion (CSII) requiring approximately 0.8 units/kg/d of insulin.
terized by several associated abnormalities that tend to develop
KCNJ11 gene sequence analysis: heterozygous for
in a particular sequence over time. This disorder was consid-
known mutation R201C detected in exon 2 of KCNJ11
ered rare with an estimated population prevalence of 1:770 000
gene. Once the mutation was identified, the baby was
births and a carrier frequency of 1 in 354 people. However,
admitted to hospital for transition and was successfully
recent estimates suggest it may be more common than previ-
transitioned to oral SU therapy.
ously thought (18). Optic atrophy closely follows or precedes
the onset of diabetes. In one series, the median age of onset of
SYNDROMIC FORMS OF NEONATAL diabetes was 6 years, followed by progressive optic atrophy at
DIABETES MELLITUS 11 years with near total loss of vision during the following
A third form of NDM involves several syndromes including decade. Central diabetes insipidus is the next most common
NDM as well as other multisystem manifestations (Table 2) manifestation, followed by sensorineural deafness in the sec-
for which the responsible gene has been identified. The ond decade. By the third decade, patients may have inconti-
frequency of these syndromes varies in different popula- nence and evidence of neurologic degeneration with cerebellar
tions. For example, the Rallison-Walcott syndrome due to ataxia and myoclonus by the fourth decade. A wide arc of
mutations in the EIF2AK3 gene is more common in areas of neurological manifestations includes autonomic neuropathy,
consanguinity in which the incidence of NDM was calcu- loss of sense of taste and smell, hemiparesis, myoclonus, and
lated to be 1:21 000, or approximately 5-10 times more absent or reduced reflexes. Magnetic resonance imaging dem-
common than in more heterogeneous populations (5, 10). As onstrates brain atrophy. Psychiatric illness characterized by
targeted sequencing technologies improve, permitting timely dementia and short-term memory loss and suicidal ideation are
and cost-efficient identification of genetic mutations associ- common later in the evolution of this syndrome. Presumed
ated with NDM, diagnosis is established earlier, before carriers of this mutation have been reported to be predisposed
some of the recognized complications of a syndrome be- to psychiatric illness. Affected individuals have insulin defi-
come manifest, allowing earlier intervention. An example is ciency requiring insulin for treatment. The diabetes insipidus
the earlier recognition of mutations known to cause DEND, responds to desmopressin. Some patients also benefit from
in which earlier treatment with SU drugs mitigates and hearing aids for their deafness. Gonadal atrophy is also com-
improves the neurological and psychomotor function in pa- mon later in males, although some females have been reported
tients with potassium channel defects (5, 11). to have successful pregnancies.
In summary, the key common features of NDM mellitus Heterozygous carriers of this mutation have an increased
are intrauterine growth retardation with low birth weight and frequency of hearing loss and diabetes. The function of the
length, reflecting the role of insulin as an in-utero growth WFS1 remains unknown. There are some similarities between
factor. The postnatal course is variable, with mild or severe Wolfram syndrome and mitochondrial diseases, such as
glucosuria and polyuria, dehydration, failure to thrive, keto- Lebers hereditary optic neuropathy (LHON) and mitochon-
acidosis, and low insulin levels in blood. Some have drial encephalomyelopathy, lactic acidosis, and stroke-like epi-
dysmorphic features including psychomotor retardation, sodes (MELAS). Although diabetes does occur in MELAS, and
muscle weakness, epilepsy (KATP defects) or other optic atrophy is a feature of LHON, these syndromes are
syndromic features with specific features due to mutations in clearly due to mitochondrial genome mutations as discussed in

The Endocrine Society. Downloaded from press.endocrine.org by [${individualUser.displayName}] on 12 January 2017. at 12:19 For personal use only. No other uses without permission. . All rights reserved.
268 ENDO 2016 MEET-THE-PROFESSOR CLINICAL CASE MANAGEMENT

the section on maternally inherited diabetes and deafness thiamine supplements may become ineffective, so all patients require
(MIDD) below. Examination of mitochondrial DNA in patients insulin therapy with regular blood transfusions in adulthood (20).
with classic Wolfram shows no evidence for specific mutations
or general mutations or deletions. MAIN CONCLUSIONS
Monogenic forms of diabetes may often be misdiagnosed as
MIDD T1DM or T2DM, treated with inappropriate therapy and
MIDD is associated with the 3243AG mtDNA point muta- provided with inaccurate genetic information. As recogni-
tion, a condition that may affect up to 1% of patients with tion of these entities becomes more widely accepted, and
diabetes (19). These gene mutations are nearly always inherited molecular diagnosis becomes part of routine investigation,
from the mother, given that mtDNA is present in oocytes but not these conditions will be increasingly diagnosed.
in spermatozoa. Therefore, relatives on the maternal side may also The same genes that cause these monogenic forms are
have manifestations of mitochondrial gene defects, of which dia- increasingly recognized as contributing to the epidemic of
betes is the third most common systemic manifestation after car- T2DM, linked with obesity, which has unmasked the pres-
diac conduction defects and cardiomyopathy. Other systemic ence of these genes in the population.
manifestations include short stature, pigmentary retinopathy, lactic Sound clinical practice should include careful family his-
acidosis, glomerulopathy with focal segmental glomerulosclerosis, tory, measurement of autoantibodies to confirm T1DM, and
and strokes with cerebellar and cerebral atrophy. Because the consideration of genetic causes in atypical cases.
3243AG mutation results in diminished ATP production, tissues
with high energy turnover, such as pancreatic islets and the co- CASES FOR DISCUSSION
chlear stria vascularis, are affected, explaining the presence of Case 4
deafness and diabetes. The diabetes may be variable in its sever- Two young children are brought to the emergency depart-
ity, presenting as diabetic ketoacidosis in young patients and ment in winter for suspected influenza. Routine physical
therefore being misdiagnosed as T1DM. Alternatively, milder examination is unremarkable and laboratory tests reveal
forms may present later in life and be diagnosed as T2DM. LHON hyperglycemia and modest elevations in HbA1c. These re-
and MELAS may be part of the same familial spectrum and are sults and family history are shown in Figure 2.
due to the same gene mutation. Some patients with MIDD have What is the most likely diagnosis?
been reported to have classic islet cell antibodies, which may What else should be performed?
represent coincident autoimmune mechanisms or represent sec- What treatment should be ordered?
ondary response to pancreatic -cell destruction related to the
mitochondrial gene mutation and its potential effects on inducing
apoptosis within the islets.
The prognosis for MIDD is determined by the associated
systemic manifestations, including cardiac failure and central
nervous system dysfunction, and the diabetes requires insulin
in those with younger severe onset but may be managed con-
servatively, at least at first, in older patients presenting with a
pattern suggestive of T2DM. Genetic counseling is important
in clarifying the maternal inheritance of this entity to affected
family members (19).

Thiamine-Responsive Megaloblastic Anemia


Thiamine-responsive megaloblastic anemia (TRMA) and dia-
betes are due to a defect in the thiamine transporter SLC19A2
(20). TRMA, also known as Rogers syndrome, combines megalo-
Figure 2.
blastic anemia, diabetes, and sensorineural deafness, which respond
by variable degrees to thiamine replacement. The SLC19A2 gene Case 5
encodes a transmembrane protein that acts as the thiamine transporter A mother apologizes for late arrival for a diabetes appointment
and has homology to reduced folate carrier proteins. Cardiac anoma- for her son; she was delayed in the renal clinic with her other
lies and abnormalities of the optic nerves and retina may occur son. Family history is shown in Figure 3.
occasionally in the TRMA syndrome. Markers of autoimmunity are What is the most likely diagnosis?
absent in this autosomal-recessive form of diabetes. Pharmacologic Which test should be performed?
doses of thiamine may reverse anemia and diabetes early in the What treatment should be provided?
course of the disease; however, sometime during and after puberty, What genetic counseling should be provided?

The Endocrine Society. Downloaded from press.endocrine.org by [${individualUser.displayName}] on 12 January 2017. at 12:19 For personal use only. No other uses without permission. . All rights reserved.
ENDO 2016 PEDIATRIC ENDOCRINOLOGY 269

monogenic diabetes in children and adolescents. Pediatr Diabetes.


2014;15(Suppl 20):47-64.
7. Pearson ER, Flechtner I, Njlstad PR, et al. Switching from insulin to oral
sulfonylureas in patients with diabetes due to Kir6.2 mutations. N Engl
J Med. 2006;355(5):467-477.
8. Chakera AJ, Steele AM, Gloyn AL, et al. Recognition and management of
individuals with hyperglycemia because of a heterozygous glucokinase
mutation. Diabetes Care. 2015;38(7):1383-1392.
9. Steele AM, Shields BM, Wensley KJ, Colclough K, Ellard S, Hat-
tersley AT. Prevalence of vascular complications among patients with
glucokinase mutations and prolonged, mild hyperglycemia. JAMA.
2014;311(3):279-286.
10. Habeb AM, Al-Magamsi MS, Eid IM, et al. Incidence, genetics, and
clinical phenotype of permanent neonatal diabetes mellitus in northwest
Saudi Arabia. Pediatr Diabetes. 2012;13(6):499-505.
11. Beltrand J, Elie C, Busiah K, et al. Sulfonylurea therapy benefits
Figure 3. neurological and psychomotor functions in patients with neonatal
diabetes owing to potassium channel mutations. Diabetes Care.
2015;38(11):2033-2041.
12. Florez JC. Clinical review: The genetics of type 2 diabetes: A realistic
Case 6 appraisal in 2008. J Clin Endocrinol Metab. 2008;93(12):4633-4642.
A newborn baby considered to be IUGR is found to have 13. Pihoker C, Gilliam LK, Ellard S, et al. Prevalence, characteristics and
clinical diagnosis of maturity onset diabetes of the young due to mutations
hyperglycemia with glucosuria. Diabetes is diagnosed and the
in HNF1A, HNF4A, and glucokinase: Results from the SEARCH for
baby is started on insulin. Molecular diagnostics confirms Diabetes in Youth. J Clin Endocrinol Metab. 2013;98(10):4055-4062.
NDM due to an insulin gene mutation. 14. Menon RK, Cohen RM, Sperling MA, Cutfield WS, Mimouni F, Khoury
JC. Transplacental passage of insulin in pregnant women with insulin-
True or false: The parents should be told this is transitory
dependent diabetes mellitus. Its role in fetal macrosomia. N Engl J Med.
and the baby switched to oral SU therapy. 1990;323(5):309-315.
15. Sperling MA. ATP-sensitive potassium channelsNeonatal diabetes mel-
litus and beyond. N Engl J Med. 2006;355(5):507-510.
REFERENCES 16. Naylor RN, John PM, Winn AN, et al. Cost-effectiveness of MODY
1. Fajans SS, Bell GI, Polonsky KS. Molecular mechanisms and clinical genetic testing: Translating genomic advances into practical health appli-
pathophysiology of maturity-onset diabetes of the young. N Engl J Med. cations. Diabetes Care. 2014;37(1):202-209.
2001;345(13):971-980. 17. Classification and diagnosis of diabetes. Diabetes Care. 2015;38(Suppl):S8S16.
2. Fajans SS, Bell GI. MODY: History, genetics, pathophysiology, and clini- 18. Blanco-Aguirre ME, la Parra DR, Tapia-Garcia H, et al. Identification of
cal decision making. Diabetes Care. 2011;34(8):1878-1884. unsuspected Wolfram syndrome cases through clinical assessment and
3. Bonnefond A, Froguel P. Rare and common genetic events in type 2 WFS1 gene screening in type 1 diabetes mellitus patients. Gene.
diabetes: What should biologists know? Cell Metab. 2015;21(3):357-368. 2015;566(1):63-67.
4. von Muhlendahl KE, Herkenhoff H. Long-term course of neonatal diabe- 19. Murphy R, Turnbull DM, Walker M, Hattersley AT. Clinical features,
tes. N Engl J Med. 1995;333(11):704-708. diagnosis and management of maternally inherited diabetes and deafness
5. De Franco E, Flanagan SE, Houghton JA, et al. The effect of early, (MIDD) associated with the 3243AG mitochondrial point mutation. Diabet
comprehensive genomic testing on clinical care in neonatal diabetes: An Med. 2008;25(4):383-399.
international cohort study. Lancet. 2015;386(9997):957-963. 20. Shaw-Smith C, Flanagan SE, Patch AM, et al. Recessive SLC19A2 muta-
6. Rubio-Cabezas O, Hattersley AT, Njlstad PR, et al. ISPAD Clinical tions are a cause of neonatal diabetes mellitus in thiamine-responsive
Practice Consensus Guidelines 2014. The diagnosis and management of megaloblastic anaemia. Pediatr Diabetes. 2012;13(4):314-321.

The Endocrine Society. Downloaded from press.endocrine.org by [${individualUser.displayName}] on 12 January 2017. at 12:19 For personal use only. No other uses without permission. . All rights reserved.
270 ENDO 2016 MEET-THE-PROFESSOR CLINICAL CASE MANAGEMENT

Disorders of Sexual Differentiation in Newborns,


Infants, and Children

M45 about genital appearance, difficulties with a legal sex assign-


Presented, April 1 4, 2016 ment on the birth certificate, and substantial efforts to exhibit
maleness (7). A reflection about current practice within the
historical context can be very informative and further changes
Nils Krone, MD, FRCPCH. Academic Unit of Child in the approach to individuals with DSD are foreseeable.
Health, Department of Oncology and Metabolism,
University of Sheffield, Sheffield S10 2TH, United
SIGNIFICANCE OF THE CLINICAL PROBLEM
Kingdom, E-mail: n.krone@sheffield.ac.uk
DSDs represent an umbrella term for a wide and diverse range
of conditions. Thus, DSDs present with diverse clinical fea-
INTRODUCTION tures and pathophysiology. When including all changes, the
Historical Overview birth prevalence of individuals born with atypical genitalia
The idea of a biological continuum of sex rather than the might be as high as 1 in 300 live births. However, the preva-
traditional binary concept seems to gather increasing pace, lence of DSD conditions requiring further expert examination
making it into the news in recent years and also being high- based on genital ambiguity has been estimated to be approxi-
lighted as a feature in leading scientific journals (1). This idea mately 1 in 5000 live births (8). Even this lower figure trans-
is, however, at least more than 20 years old (2) and is relevant lates to approximately 65 000 affected individuals in the USA
to the care provision for individuals with disorders (or differ- and approximately 100 000 individuals in the European Union.
ences) of sexual differentiation (DSDs). The diversity of all conditions classified as DSD, however,
In the Western world, the existence of individuals who were requires highly specialized and personalized care provision for
perceived as neither female nor male has captured the imagi- individuals when medical health care provision is required and
nation of humans at least since antiquity. Several ancient re- desired. Within the last decade and in the years to come, care
ports exist and apparently the attitude toward hermaphrodites
provision in DSD has seen an overhaul in several aspects
changed also during those ancient times from regarding such
ranging from scientific advances in almost all aspects over a
individuals as monsters who were often killed to the view that
technological revolution of the diagnostic pathway to the ques-
affected individuals represent an amusing quirk of nature (3).
tioning about the requirement of care provision. These devel-
Over the course of the 17th to early 20th centuries, individu-
opments have led to improvements, but also highlighted mul-
als with ambiguous internal and external genitalia were re-
tiple dilemmas, uncertainties, and barriers to optimal practice.
garded as cases of mistaken sex. The reality, however, chal-
lenged the ideal of two sexes and raised questions about what it
meant to be female or male (4). BARRIERS TO OPTIMAL PRACTICE
Medical progress in the 20th century led to the develop- Most recommendations on the management of DSD lack a
ment of surgical reconstructive procedures, during times when solid evidence basis. Diagnostic approaches to define specific
understanding of the underlying pathophysiology was limited. conditions have significantly improved. During the recent de-
Interestingly, one of the first comprehensive publications by cade, data on long-term outcomes are emerging, which are
Hugh H. Young has been relatively open minded and prag- often far from optimal (9). This has led to significant reevalu-
matic rather than dogmatic or judgmental (5), despite the fact ation of the management requirements across all specialties
that he laid the foundations for surgical interventions. A key involved in the care pathway.
driver in the clinical management has been the idea of the A first key issue in the care of individuals with DSD is often
environment over biology and that gender identity might fol- sex assignment and registration of individuals with a specific
low the sex of assignment and rearing more closely than the sex with the authorities in most countries around the globe.
underlying biology. This dogmatic view as been challenged This readily illustrates traditional views and raises the question
over recent decades and one should be aware that different why daily life requires disclosure about an individuals sex.
paradigmatic periods have heavily influenced management of It is now common consensus that appropriate management
individuals with DSD (6). of DSD is multidisciplinary and requires a high level of inte-
The translation and commentary to the first detailed medical gration and interaction between the involved specialists. This,
description of congenital adrenal hyperplasia (CAH) highlights however, must be seen in the context of different health care
similar issues for affected individuals today as 150 years ago systems and possibilities to implement desirable health care
including the genital ambiguity at birth, social pressure follow- structures including networks providing expertise to care of
ing reversed sex assignment in childhood, adult embarrassment individuals with rare conditions such as DSD across regions.

The Endocrine Society. Downloaded from press.endocrine.org by [${individualUser.displayName}] on 12 January 2017. at 12:19 For personal use only. No other uses without permission. . All rights reserved.
ENDO 2016 PEDIATRIC ENDOCRINOLOGY 271

LEARNING OBJECTIVES audit clinical practice and outcomes in DSD. The classification
The goal of this this session is that the learners should of the Chicago DSD Consensus meeting (10) provides a ratio-
be able to: nal and well-structured guidance to the further medical diag-
Recognize and classify DSDs nostic pathway despite the heavy weighting of the underlying
Understand the genetic and hormonal basis for the more karyotype (Table 1). With the use of novel technologies in the
common DSDs diagnostic pathway, an informed and differentiated parallel
Recognize the lack of evidence of previous and also rather than stepwise diagnostic approach is increasingly imple-
recently changed practice mented. A thorough evaluation to identify the type of DSD is
Understand the role and limitations of novel diagnostic warranted because this information is essential for personalized
pathways and methods management.
Recognize that not all DSDs merit therapy or require a
management plan for long-term management.
Clinical Diagnosis
In addition to a standard medical history and physical
STRATEGIES FOR DIAGNOSIS, THERAPY, examination, employing standardized scoring systems de-
AND/OR MANAGEMENT spite their recognized limitations can be useful in clinical
The complexity of the different aspects involving diagnosis, practice. The Prader genital classification can be commonly
therapy, and management of individuals with DSD as well as used for 46,XX individuals, who are mostly patients with
the rapid technological advances requires a multidisciplinary virilizing forms of CAH. A more detailed and clinically
approach to provide an optimal care provision. Key to optimiz- useful system is the extragenital masculinization score rep-
ing outcomes is the standardized and structured clinical assess- resenting an easy tool for the clinical assessment of indi-
ment and the development of a diagnostic and management viduals with under-masculinization (8). The documentation
plan covering all aspects. Only such an approach will enable to of the anogenital distance seems to be a good correlate of

TABLE 1. Classification of DSD Guided by Genetic Differences


46,XX DSD 46,XY DSD Sex Chromosome DSD
Disorders of gonadal development: Disorders of gonadal development 46,XX/46,XY (chimeric, ovotesticular DSD)
Ovotesticular DSD Complete gonadal dysgenesis 45,X/46,XY (MGD, ovotesticular DSD)
Testicular DSD Partial gonadal dysgenesis

Gonadal dysgenesis Gonadal regression

Ovotesticular DSD

Disorders of androgen excess Disorders in androgen synthesis 47,XXY (Klinefelter syndrome/variants)


CAH (mutations in CYP21A2, CLAH and CAH (e.g. mutations in StAR, 45,X (Turner syndrome and variants)
CYP11B1, POR, HSD3B2) CYP11A1, HSD3B2, CYP17A1, POR)
Aromatase deficiency Isolated androgen biosynthesis deficiencies
(mutations in CY5B, HSD17B3, SRD5A2)
Maternal luteoma Luteinizing hormone receptor defects
(e.g. Leydig cell hypoplasia, aplasia)
Exogenous exposure, iatrogenic
Disorders in androgen action Other chromosomal rearrangements
Complete and partial androgen insensitivity
syndrome
Unclassified conditions Unclassified conditions
Cloacal exstrophy Persistent Mllerian Duct Syndrome
(mutations in AMH and AMHR)
Vaginal atresia Hypospadias, unknown aetiology

MURCS, Mllerian, renal, Epispadias


cervicothoracic somite
abnormalities
Other complex syndromes Other complex syndromes

CYP21A2: 21-hydroxylase, CYP11B1: 11-hydroxylase, POR: P450 oxidoreductase, HSD3B2: 3--hydroxysteroid dehydrogenase,
StAR: steroidogenic acute regulatory protein, CYP11A1: P450 side chain cleavage enzyme, CYP17A1: 17-hydroxylase, CY5B: cyto-
chrome B5, HSD17B3: 17-hydroxysteroid dehydrogenase, SRD5A2: 5-reductase type 2, AMH: anti-Mllerian hormone, AMHR:
anti-Mllerian hormone receptor.

The Endocrine Society. Downloaded from press.endocrine.org by [${individualUser.displayName}] on 12 January 2017. at 12:19 For personal use only. No other uses without permission. . All rights reserved.
272 ENDO 2016 MEET-THE-PROFESSOR CLINICAL CASE MANAGEMENT

androgen effects during embryogenesis. of mini-puberty can provide additional insights into gonadal
Special attention should be paid toward additional anomalies function and can potentially avoid the use of more invasive
of other organ systems given that 25% of patients registered stimulation tests.
with the I-DSD registry showed an involvement of other organ
systems (11). Genetic Evaluation
Given that the adrenal and gonad have the same develop- Key to the overall diagnostic planning is the rapid determina-
mental origins, a child with DSD is at increased risk to suffer tion of the karyotype. Depending on the suspected underlying
from adrenocortical insufficiency. However, the degree of the condition further genetic analysis has a key role in the diagnos-
specific genital ambiguity does not necessarily correlate with tic process. The use of comparative genomic hybridization
the degree of impaired adrenocortical function. (CGH) arrays can provide significant insights into the etiology
The evaluation of the internal genitalia provides useful in- when complex rearrangements are expected. However, the suc-
formation toward a clinical diagnosis. An ultrasound investiga- cess rate finding genetic anomalies is significantly higher in
tion represents a powerful test and should be performed by an individuals with additional anomalies and remains low in indi-
experienced ultrasonographer. A disclosure of the absence of viduals with isolated DSD (13). The strategies for molecular
anatomical structure must be handled with caution and requires genetic analysis are currently in flux with the more widespread
informing parents about the potential limitations of ultrasound implementation and availability of NGS approaches.
investigations. A common recommendation is the use of mag- In our clinical pathway, we are using targeted analysis
netic resonance imaging. However, as this technique generally mainly for the molecular genetic analysis of CYP21A2 defi-
requires general anesthesia in young children, and in our clini- ciency, employing direct sequencing after PCR amplification
cal practice we often perform an examination under anesthesia and multiplex ligation probe amplification. A variety of other
if a further investigation of the internal anatomy is required. conditions (46,XX and 46,XY DSD) can be comprehensively
analyzed using a targeted NGS DSD panel containing 32
Endocrine Assessment genes, which can lead to faster results than a candidate gene
The role of the endocrine evaluation in the diagnostic process approach and the parallel analysis of multiple samples on a
of DSD is changing. Traditionally the hormonal evaluation has single test platform.
provided the diagnostic basis for further molecular genetic Recent developments in the United Kingdom will allow to
analysis to confirm the diagnosis. Comprehensive hormonal for whole-genome sequencing of patients with DSD within the
evaluation is nowadays increasingly a part of the overall national 100 000 genomes project (http://www.genomicsengland.
phenotyping of individuals with DSD, which can provide in- co.uk). Such an approach might help to establish the genetic
formation on the function of endocrine organs and can be used diagnosis in individuals with DSD of unknown etiology. This,
to monitor potentially required therapeutic interventions. however, will have a significant number of potential ethical impli-
A key question in newborns, infants, and childrenquestion cations and the necessity to demonstrate causality of novel vari-
in newborns, infants, and children with DSD is the adreno- ants. Exome sequencing seems to provide additional insights into
cortical function requiring assessment of patients glucocor- the etiology of 46,XY DSD (14); however, currently in a signifi-
ticoid and mineralocorticoid status. A plasma and/or urinary cant number of patients a definite diagnosis is still not achieved. It
steroid profile will commonly provide the differential diag- is foreseeable that the molecular diagnosis of unsolved aetiologies
nosis in most patients with various forms of CAH and other leading to DSD might require novel diagnostic strategies. One can
conditions affecting the adrenal gland. Deficiencies of anticipate the widespread implementation and use of NGS meth-
17-beta-hydroxysteroid dehydrogenase type 3 (HSD17B3) ods and genomic medicine in clinical routine.
and 5-alpha-reductase type 2 (SRD5A2) are more complex The rapidly changing landscape of genetic and genomic
to diagnose in the newborn. SRD5A2 deficiency can be analytical approaches makes the input of a genetic expert into
easily diagnosed without the need for invasive tests such as these analytical pathways vital; diagnostic planning and strate-
the human chorionic gonadotropin (HCG) test from 3 gies should be developed in close liaison with expert clinical
months of age analyzing the ratio of 5- to 5- reduced geneticists. It will, however, require basic understanding of
steroid metabolites from a spot urine (12). genomic medicine by all members of the wider multidisci-
In our clinical practice, we are increasingly using a parallel plinary team (MDT).
approach in most patients with conditions other than CAH
using a clinically accredited next-generation sequencing (NGS) Management
DSD panel for targeted gene analysis. Such an approach can The holistic management of individuals with DSD and their
lead to earlier specific diagnosis relevant to treatment or families can have significant challenges. The conflict between
nontreatment decisions (Figure 1). In such an approach, bio- the individual right of the child of physical integrity and self
chemical data are used in a complementary fashion providing determination, and the parental right to care for their child in
information on gonadal function rather than stratifying the their perceived best way remains a key ethical dilemma in the
diagnostic pathway. Studying gonadal function during the time care provision of patients with DSD.

The Endocrine Society. Downloaded from press.endocrine.org by [${individualUser.displayName}] on 12 January 2017. at 12:19 For personal use only. No other uses without permission. . All rights reserved.
ENDO 2016 PEDIATRIC ENDOCRINOLOGY 273

Figure 1. Algorithm for the medical evaluation of DSD. The clinical diagnostic process will be influenced by the availability
of analytical technology and funding streams by national health care providers and national health services. Ideally, steroid
hormones from either blood and/or urine should be analyzed as comprehensive profiles using specific immunoassays (ideally
after extraction), liquid chromatographytandem mass spectrometry, or GC/MS, which will be readily lead to diagnosis of
also rarer forms of CAH. The biochemical analysis of 17-hydroxysteroid dehydrogenase type 3 and 5-reductase type 2
deficiency can be challenging without the use of stimuation tests. 5-reductase type 2 deficiency can be easily diagnosed from
a urinary steroid metabolite profile after the age of 3 months. Adapted from Hiort et al (18).

Meeting the right balance between over- and Sex assignment should be based on available evidence on
undermedicalization seems to be of utmost importance in the long-term outcomes with an emphasis of future quality of life,
current medical and societal context. Patient education and likely sex role, risk of sex dysphoria, and fertility. There
empowerment are a key requisite to understand different remains a clear association between the external appearance of
choices. However, they should not lead to shifting the sole the genitalia and the choice of sex assignment. However, there
responsibility of decision making toward the parents. are clear changes over time in practice with an increased
Depending on the condition, patients will need different likelihood of affected infants with 46,XY karyotype being
intervals of follow-up; some individuals may not need to be raised as boys (15).
seen regularly until close to the start of puberty unless there 46,XX newborns and infants with virilizing forms of CAH
are newly emerging problems or the request by the patient are most commonly raised as females and outcomes with re-
and the family. gard to sex role and low risk of sex dysphoria are encouraging
(16). Commonly 46,XY individuals with complete androgen
Sex Assignment insensitivity and complete gonadal dysgenesis are assigned
The current requirement in most societies to assign a certain female. Even in the severely undermasculinized 46,XY new-
social sex at a specific age can put extra pressure on families born and infants with deficiencies of HSD17B3 or SRD5A2
and health care professionals. Within a multicultural envi- male sex assignment might be favored. This practice, however,
ronment beliefs and traditions play a significant role, which has a higher uncertainty than in the above conditions. Further-
must be carefully addressed. Such societal constrains must, more, changes in clinical practice to assign 46,XY with partial
however, not lead to irreversible interventions. androgen insensitivity and other undefined conditions of

The Endocrine Society. Downloaded from press.endocrine.org by [${individualUser.displayName}] on 12 January 2017. at 12:19 For personal use only. No other uses without permission. . All rights reserved.
274 ENDO 2016 MEET-THE-PROFESSOR CLINICAL CASE MANAGEMENT

undermasculinization as males are not without new therapeutic discussion and the development of individual management
challenges in the future. plans and strategies based on the available evidence. All local
activities are aligned with national and international endeavors
Medical around the British Society for Paediatric Endocrinology &
Most individuals with DSD will require hormonal replacement Diabetes (BSPED) DSD special interest group, the I-DSD
or treatment during their life. As emphasized above, it is vital registry (www.i-dsd.org) and the European Union Cooperation
to rule out potentially life-threatening adrenal insufficiency and in Science and Technology (EU COST) action DSDnet
to establish the requirement of glucocorticoid and mineralocor- (www.dsdnet.eu).
ticoid replacement. Critical voices rate hormone treatment in- This approach follows the philosophy that management of
ducing irreversible effects ethically similar to surgical interven- individuals with DSD should be performed in or in close
tion. Of note, there are only a limited number of conditions, liaison with expert centers following national and international
where this holds true. The requirement of hormonal therapy standards, which must allow a personalized approach to the
needs to be personalized. This should be based on the best individual. The embedding into regional or national, and inter-
available evidence. The decision of hormonal treatment inter- national expert networks to discuss management of patients
ventions should be a MDT decision taking the results of the with other colleagues in cases in which evidence is lacking
multidisciplinary assessment into account. A clear treatment seems to be essential to improve outcomes. To create the
plan with future goals and milestones should be discussed with evidence basis in this field of rare conditions the registration of
the family and revisited when these milestones are reached. patients into registries and audit of clinical practice and out-
comes is paramount to improving health care provision.
Surgery
The role of surgery in the management of DSD has fundamen- CASES WITH QUESTIONS
tally changed in recent years. There is an increasing notion that The cases must be seen within the context of care provision within
irreversible interventions should be performed with caution. the settings at Birmingham Childrens Hospital and Sheffield
However, this again will heavily depend on the underlying Childrens Hospital. They are not meant to provide a dogmatic
etiology and the certainty of potential outcomes. Slight modi- care pathway for individuals with similar presentations.
fications to traditional pathways can reduce the frequency of
certain procedures. An example is the decline of clitoral reduc-
Case 1
tion procedures in patients with CAH in our center by not A newborn with ambiguous genitalia is referred to DSD MDT
opting for an intervention during infancy, but delaying procedures clinic. The baby has ovoid palpable structures in the labial like
to approximately 18 months of age. Overall, some individuals structure and a small phallus. A low random blood glucose
might well benefit from corrective surgery, whereas others might concentration has been reported in the history.
not benefit from a surgical intervention. The physical and psycho- 1. What are the next diagnostic steps?
logical consequences of late interventions after an individual can A. Karyotype
provide full informed consent remain largely unclear. Parents B. LH, FSH
should be provided with very realistic expectations and should C. Random cortisol
receive clear information on future outcomes and be aware of lack D. Steroid hormone profile
of evidence basis if no evidence exists. E. Other factors
The risk of germ cell tumors is highly dependent on the 2. What is the potential strategies to manage the patient?
underlying etiology and emphasizes the importance of estab- A. Urgent gender assignment as female
lishing a specific molecular diagnosis (10, 17). Ideally a poten- B. Urgent gender assignment as male if karyotype
tially required gonadectomy is delayed until the patient can 46,XY
provide fully informed consent without putting the individual C. Never assign a gender until the child can assent/
at risk of developing a malignancy. A gonadectomy is rarely consent
required in early childhood, but the risk of carcinoma in situ D. Avoid hormonal and surgical treatment
and tumors can rapidly increase in later childhood. This needs E. E. Consider hormonal and surgical treatment
to be managed on an individual basis and is determined by the
underlying condition and the associated malignancy risk.
Case 2
A 2.5-year-old girl is referred with significant clitoromegaly
MAIN CONCLUSIONS and query single opening. The mother is aware about the
The clinical DSD MDT at Birmingham Childrens and Shef- atypical external genitalia and thought that the girl might have
field Childrens Hospital include a broad range of specialists the same condition as the girls maternal uncle, who underwent
running monthly MDT clinics with the presence of all mem- a sex reassignment from female to male approximately 20
bers of the team. Vital to these clinics are a preclinical case years ago. The girl has a 46,XY karyotype, hormonal analysis

The Endocrine Society. Downloaded from press.endocrine.org by [${individualUser.displayName}] on 12 January 2017. at 12:19 For personal use only. No other uses without permission. . All rights reserved.
ENDO 2016 PEDIATRIC ENDOCRINOLOGY 275

rules out SRD5A2 deficiency, and genetic analysis confirms mas, and uncertainties of future gender identity. The common
HSD17B3 deficiency. Reportedly, the grandfather is adamant feedback from all family members was that everyone despite
that this is his little grandson and shall follow the same path different opinions wants only the best for the child.
as the uncle. The rest of the family is unclear how to proceed. Working together with the family, a clear management plan
1. For this child, which of the following factors are important in was developed agreeing on potential medical interventions and
deciding whether to (manage) them as a male or female? psychological reassessment around the time of puberty.
A. Family pressures
B. Karyotype Case 3
C. Hormonal profile This case illustrates the role of a molecular genetic diagnosis in
D. Future fertility the personalized management of individuals with DSD. After
E. Other factors implementing a novel NGS based DSD panel at our clinical
genetics units, all three sisters underwent testing for 32 genes
Case 3 associated with DSD. All three sisters were found to have a
A 5-year-old girl referred with hernia. Her cousin has had hemizygous frameshift mutation in the androgen receptor asso-
the same problem, has been diagnosed with 46,XY DSD ciated with complete androgen insensitivity syndrome (CAIS)
with a homozygous HSD17B3 mutation, and had some de- explaining the presentation with 46,XY DSD in this branch of
gree of virilization at the onset of puberty. The cousin the family in contrast with the causative HSD17B3 mutation in
underwent gonadectomy. Our patient has also a 46,XY the other branch of the family. Thus, a certain diagnosis could
karyotype, no signs of virilization, blood tests and a urinary be provided allowing to inform the family about a relatively
steroid profile do not provide a clear diagnosis. She is certain future prognosis. The family has agreed to defer the
heterozygous for the same HSD17B3 mutation. Subse- discussion about potential gonadectomy until the affected indi-
quently, her two younger sisters are also diagnosed with vidual can provide full informed consent.
46,XY DSD, one is a heterozygous carrier for the HSD17B3
mutation, the other sister has no HSD17B3 variant. The REFERENCES
family is very worried about the potential risk of virilization 1. Ainsworth C. Sex redefined. Nature. 2015;518:288-291.
2. Fausto-Sterling A. The Five Sexes. Sciences (New York). 2000;40(4):
during puberty and is vehemently pushing for immediate
18-23.
gonadectomy despite not having a clear diagnosis. 3. Androutsos G. Hermaphroditism in Greek and Roman antiquity. Hor-
1. Which of the following is the best next step in this mones (Athens). 2006;5:214-217.
patient? 4. Reis E. Impossible hermaphrodites: Intersex in America, 1620 1960. J Am
Hist. 2005;92:411-441.
A. Proceed with gonadectomy to avoid virilization 5. Young HH. Genital Abnormalities, Hermaphrodites and Related Adrenal
B. Glucocorticoid treatment Disease. Baltimore, MD: Williams & Wilkins; 1937.
C. GnRH-analog treatment 6. Telles-Silveira M, Knobloch F, Kater CE. Management framework para-
digms for disorders of sex development. Arch Endocrinol Metab. 2015;59:
D. Additional diagnostic tests 383-390.
E. No interventions required 7. Delle Piane L, Rinaudo PF, Miller WL. 150 years of congenital adrenal
hyperplasia: translation and commentary of De Crecchios classic paper
from 1865. Endocrinology. 2015;156:1210-1217.
DISCUSSION OF CASES AND ANSWERS 8. Ahmed SF, Achermann JC, Arlt W, et al. Society for Endocrinology UK
Case 1 guidance on the initial evaluation of an infant or an adolescent with a
suspected disorder of sex development (Revised 2015). Clin Endocrinol
This case deals with the diagnostic workup of a newborn with
(Oxf). 2015:10.1111/cen.12857. [Epub ahead of print]
DSD as well as the therapeutic intervention. The initial analysis 9. Kohler B, Kleinemeier E, Lux A, Hiort O, Gruters A, Thyen U. Satisfac-
ruled out adrenal insufficiency. The karyotype was 46,XY. tion with genital surgery and sexual life of adults with XY disorders of sex
Further biochemical tests were not conclusive. However, mo- development: results from the German clinical evaluation study. J Clin
Endocrinol Metab. 2012;97:577-588.
lecular genetic analysis established the diagnosis of SRD5A2 10. Lee PA, Houk CP, Ahmed SF, Hughes IA. Consensus statement on
deficiency rapidly. Male sex was assigned and treatment with management of intersex disorders. International Consensus Conference on
Adractim (DHT) 2.5% initiated. After a good phallic growth Intersex. Pediatrics. 2006;118:e488-e500.
11. Cox K, Bryce J, Jiang J, et al. Novel associations in disorders of sex
male reconstructive surgery (3 stages) was performed. development: findings from the I-DSD Registry. J Clin Endocrinol Metab.
2014;99:E348-E355.
Case 2 12. Krone N, Hughes BA, Lavery GG, Stewart PM, Arlt W, Shackleton CH.
Gas chromatography/mass spectrometry (GC/MS) remains a pre-eminent
This case highlights potential uncertainties and difficulties in
discovery tool in clinical steroid investigations even in the era of fast
the management of patients even if a clear molecular genetic liquid chromatography tandem mass spectrometry (LC/MS/MS). J Steroid
diagnosis has been established. Multiple psychological assess- Biochem Mol Biol. 2010;121:496-504.
13. Ledig S, Hiort O, Scherer G, et al. Array-CGH analysis in patients with
ments over the period of 2 years did indicate a female gender
syndromic and non-syndromic XY gonadal dysgenesis: evaluation of array
identity. The MDT has had multiple meetings with the wider CGH as diagnostic tool and search for new candidate loci. Hum Reprod.
family, without the patient, to explain the diagnosis, the dilem- 2010;25:2637-2646.

The Endocrine Society. Downloaded from press.endocrine.org by [${individualUser.displayName}] on 12 January 2017. at 12:19 For personal use only. No other uses without permission. . All rights reserved.
276 ENDO 2016 MEET-THE-PROFESSOR CLINICAL CASE MANAGEMENT

14. Baxter RM, Arboleda VA, Lee H, et al. Exome sequencing for the in chromosomal females with congenital adrenal hyperplasia. Arch Sex
diagnosis of 46,XY disorders of sex development. J Clin Endocrinol Behav. 2005;34:389-397.
Metab. 2015;100:E333-E344. 17. Jrgensen A, Lindhardt Johansen M, Juul A, et al. Pathogenesis of
15. Kolesinska Z, Ahmed SF, Niedziela M, et al. Changes over time in sex germ cell neoplasia in testicular dysgenesis and disorders of sex devel-
assignment for disorders of sex development. Pediatrics. 2014;134: opment. Semin Cell Dev Biol. 2015;45:124-137.
e710-e715. 18. Hiort O, Birnbaum W, Marshall L, et al. Management of disorders of sex
16. Dessens AB, Slijper FM, Drop SL. Gender dysphoria and gender change development. Nat Rev Endocrinol. 2014;10:520-529.

The Endocrine Society. Downloaded from press.endocrine.org by [${individualUser.displayName}] on 12 January 2017. at 12:19 For personal use only. No other uses without permission. . All rights reserved.
ENDO 2016 PEDIATRIC ENDOCRINOLOGY 277

Endocrine Effects of Cancer Treatment

M60 (components of) the metabolic syndrome (5, 8, 10, 1214).


Presented, April 1 4, 2016 During the last two decades more research data have become
available on the endocrine and metabolic sequelae. However,
many of these articles are published outside the endocrine field
Sebastian Neggers, MD, PhD. Department of Medicine in oncology journals and are therefore not well known among
and Endocrinology, Erasmus University Medical Center, the endocrine community.
Rotterdam, Netherlands, E-mail:
s.neggers@erasmusmc.nl BARRIERS TO OPTIMAL PRACTICE
To date, recommendations on the management of endocrine or
INTRODUCTION metabolic late effects are continent or country specific. Most of
Historical Overview them are based on retrospective cohort studies and the endo-
Cancer survivorship has increased since the late 1960s and crine tests used are often not well defined or suboptimal per-
1970s due to the use of combination chemotherapy. Improve- formed. Moreover in the current adult CCS population the
ment in surgery, stem-cell transplantation, radiotherapy, sup- endocrine late effects are due to treatment regiments from at
portive care, and better stratification of therapy also played an least two decades ago, and within one decade we might en-
important role (1). However, cancer survivorship is not equally counter new endocrine late effects due to new treatment regi-
distributed over the different age groups. To date in the United ments with biologicals.
States, 80 90% of the cancer survivors had their tumor before For the management of endocrine or metabolic late effects,
the age of 20 years. With contemporary therapies for pediatric knowledge of the original diagnosis and treatment regiments of
malignancies, long-term survival into adulthood is expected for the malignancy are essential. Prevalence of different endocrine
more than 80% of children (2, 3). Although being cured from and metabolic conditions differ from the normal population.
their pediatric malignancy, survivors frequently have sequelae However, some CCSs forget to inform the doctor about their
that are referred to as late effects. medical history and many do not know all the treatment details.
In the 1970s the first studies on late effects after treatment of To diagnose the late effect, medical history is useful, but for
intracranial tumors were published. One of the fields pioneers treatment and future perspective of the disease this might be
was Steve Shalet, an endocrinologist working in the United essential. Therefore, collaboration with the pediatric oncologist
Kingdom (4). To date we know that 62% (confidence interval, or pediatric endocrinologist is necessary and given that 39% of
59.5 64.6%) of the childhood cancer survivors (CSSs) have the CCSs have multiple conditions a multidisciplinary ap-
late effects of the endocrine or reproductive system (5), impli- proach is recommendable.
cating the necessity for proper transition from pediatric care to
adult care. LEARNING OBJECTIVES
As a result of participating in this session, learners should be
SIGNIFICANCE OF THE CLINICAL PROBLEM able to:
Five years after treatment, the standardized mortality ratio Recognize the association between the management of
(SMR) of CCS has increased, with an overall SMR of 8.4 and pediatric cancer and subsequent endocrine disorders and
the cause specific SMR is 15.2 for malignancy, 7.0 for cardiac, metabolic disorders.
8.8 for pulmonary, and 2.6 for other causes (6). CCSs are prone Understand the unique aspects in the evaluation of
to developing chronic health conditions. More than 60% of cancer survivors for endocrine and metabolic disorders.
CSSs have at least one chronic condition, 39% have multiple Identify the potential risk of standard hormone
conditions, and 27.5% have a severe or life-threatening condi- replacement strategies in a pediatric cancer survivor.
tion (7). Endocrine and metabolic late effects are among the
most commonly observed sequelae after childhood cancer STRATEGIES FOR DIAGNOSIS, THERAPY,
(8 10). CCSs have a need for medical and endocrine attention, AND/OR MANAGEMENT
particularly as their numbers are increasing. In 2005, approxi- Background
mately 328 652 CCSs were living in the United States and 27% The latency period between endocrinopathies and exposure to
of them were age 40 years or older (11). cancer treatments varies. The kind of endocrinopathy that can
The endocrine and metabolic late effects can occur through- develop depends on the treatment but also on the susceptibility
out the endocrine system (12) depending on the treatment a of the individual. Importantly, the treatment of some of these
survivor received. The most frequent are pituitary deficiencies, endocrinopathies will lead to an additional risk in the develop-
gonadal dysfunction, primary hypothyroidism, diabetes, and ment of secondary cancers (15).

The Endocrine Society. Downloaded from press.endocrine.org by [${individualUser.displayName}] on 12 January 2017. at 12:19 For personal use only. No other uses without permission. . All rights reserved.
278 ENDO 2016 MEET-THE-PROFESSOR CLINICAL CASE MANAGEMENT

Hypopituitarism, like GH deficiency and hypogonadotropic age of 7 years, she was treated for Hodgkins lymphoma with
hypogonadism, are mainly caused by cranial or total-body chest mantle field radiation and Mustargen Oncovin Procarbazine
irradiation and sometimes by surgery or the tumor localization Prednisone (alkylating agent). At the age of 13 years, she had her
(5). When these hormones are substituted, the possible benefi- menarche. Two years later, started with OCP due to an irregular
cial effects are similar to noncancer survivors; however, data in cycle. The OCP was discontinued 6 months ago. No spontaneous
cancer survivors is scarce (5). menses have occurred. Her gonadotropins are measured and levels
Substitution of GH in survivors has similar benefits to are within the postmenopausal range.
noncancer survivors but cancer survivors are susceptible to die To address this patients amenorrhea, which of the following
from malignancies (6). To date, this increased risk for neo- is the best therapeutic option?
plasms with GH treatment was not observed anymore after A. Transdermal estrogen
longer followup in the same cohort that originally reported B. Oral ethinyl estradiol and prometrium
higher risks for neoplasms (16). The treatment of hypogonad- C. OCP
ism in females is different. Females that are treated with chest D. Cyclic progesterone
irradiation have an increased risk of developing breast cancer E. Long-term estrogen replacement is contraindicated
as a secondary tumor, for example after the treatment of Hodg-
kins lymphoma (15). Survivors of Hodgkins lymphoma with-
Case 2
out any gonadal function have a lower risk of developing breast A 40-year-old man with a BMI of 25 kg/m2, waist circumfer-
cancer than survivors with residual ovarian function (15). This ence 92 cm, arterial hypertension (140/95 mm Hg), and
causes an ongoing discussion if one should substitute young Triglycerides 150 mg/dL. At the age of 7 years, he was
women with ovarian failure after chest and high-dose abdomi-
treated for a neuroblastoma. Treatment consisted of surgery,
nal irradiation.
abdominal radiation (30 Gy), and chemotherapy.
Components of the metabolic syndrome are increased in
According to The National Cholesterol Education Programs
cancer survivors (5, 8, 10, 17). Obesity assessed by waist/hip
Adult Treatment Panel III criteria, does this patient have a
ratio or body mass index (BMI) is grossly underestimated (5,
metabolic syndrome?
8) and therefore the presence of metabolic syndrome probably
A. Yes
is as well. Many studies have tried to determine the cause of
B. No
the increased obesity but so far have not found a relation
C. Maybe
between treatment or genetics and obesity. The only treatment
that caused obesity was cranial irradiation (5). Radiation ap-
plied to the abdomen caused an increased risk for diabetes Case 3
mellitus and for the metabolic syndrome (13, 14). Treatment of A 20-year-old man with multiple pituitary deficiencies is
the metabolic problems in survivors are not different from treated with recombinant GH, thyroxin, hydrocortisone, and T
nonsurvivors, but again there are very few data in survivors. replacement. Eight years prior, he was treated for medulloblas-
toma with surgery, craniospinal irradiation (50 Gy), and addi-
tional chemotherapy. His last brain magnetic resonance image
Diagnostic Approach
The diagnosis of hypogonadism and adult GH deficiency in showed a new lesion in the region of the original medulloblas-
cancer survivors is no different from noncancer survivors and toma, and the patient is wondering if his GH has caused this.
diagnostic guidelines (18, 19). Be aware that pituitary function Which of the following is the most appropriate response?
might decrease further when time progresses after radiotherapy A. Yes, GH treatment is associated with recurrence of
(5) and that other pituitary deficiencies must be substituted medulloblastoma.
before dynamic testing of GH. So regular followup of these B. Yes, but the new lesion is likely not a
patients is recommended. medulloblastoma.
Metabolic sequelae diagnosis is the same as in nonsurvivors; C. No, the level of craniospinal irradiation was below the
however, there are some pitfalls, given that BMI and hip-waist risk threshold.
ratio underestimate fat mass (8). To assess fat mass in every D. No, there was no risk in GH therapy in this regard.
cancer survivor is not feasible; therefore, waist circumference
(WC) is the best alternative. However, careful attention is DISCUSSION
needed after abdominal radiotherapy and surgery given that Case 1
with these treatment WC might be especially affected. This young female was treated with a high dose of alkylating
chemotherapy and that probably caused her premature ovarian
CASES WITH QUESTIONS failure. Due to her young age the choice of treatment would
Case 1 have been oral estrogen-progestin. Nevertheless, this will not
A 17-year-old female is evaluated with irregular menstrual cycles work as a contraceptive, although pregnancy chances are very
after discontinuation of her oral contraceptive pill (OCP). At the limited. The problem is that this patient underwent chest

The Endocrine Society. Downloaded from press.endocrine.org by [${individualUser.displayName}] on 12 January 2017. at 12:19 For personal use only. No other uses without permission. . All rights reserved.
ENDO 2016 PEDIATRIC ENDOCRINOLOGY 279

mantle field radiation that increases the risk for breast cancer related late effects observed in survivors of childhood neoplasia. Curr
Opin Endocrinol Diabetes Obes. 2014;21(1):71-76.
comparable to a BCRA2 mutation. Therefore, you must discuss
6. Mertens AC, Liu Q, Neglia JP, et al. Cause-specific late mortality among
this problem with the patient and point out the risks of any 5-year survivors of childhood cancer: The Childhood Cancer Survivor
estrogen substitution. Study. J Natl Cancer Inst. 2008;100(19):1368-1379.
7. Oeffinger KC, Mertens AC, Sklar CA, et al. Chronic health conditions in
adult survivors of childhood cancer. N Engl J Med. 2006;355(15):1572-1582.
Case 2 8. Blijdorp K, van den Heuvel-Eibrink MM, Pieters R, et al. Obesity is
This patient does not seem to have overweight or obesity with underestimated using body mass index and waist-hip ratio in long-term
a BMI of 25 kg/m2 and waist circumference 92 cm. He has adult survivors of childhood cancer. PloS One. 2012;7(8):e43269.
9. Diller L, Chow EJ, Gurney JG, et al. Chronic disease in the Childhood
only two of five National Cholesterol Education Programs
Cancer Survivor Study cohort: A review of published findings. J Clin
Adult Treatment Panel III criteria. Therefore, we cannot diag- Oncol. 2009;27(14):2339-2355.
nose him with the metabolic syndrome. However, patients after 10. van Waas M, Neggers SJ, Pieters R, van den Heuvel-Eibrink MM. Com-
abdominal radiation have a lower WC, and BMI or WC grossly ponents of the metabolic syndrome in 500 adult long-term survivors of
childhood cancer. Ann Oncol. 2010;21(5):1121-1126.
underestimate fat mass in CCS. 11. Mariotto AB, Rowland JH, Yabroff KR, et al. Long-term survivors of
childhood cancers in the United States. Cancer Epidemiol Biomarkers
Case 3 Prev. 2009;18(4):1033-1040.
12. Brignardello E, Felicetti F, Castiglione A, et al. Endocrine health condi-
This patient has been treated with GH substitution due to
tions in adult survivors of childhood cancer: The need for specialized
(pan)hypopituitarism. Nine years earlier he was irradiated for a adult-focused follow-up clinics. Eur J Endocrinol. 2013;168(3):465-472.
medulloblastoma. To date, a new neoplasm was observed on 13. de Vathaire F, El-Fayech C, Ben Ayed FF, et al. Radiation dose to the
the magnetic resonance image. Given that there is a 9-year gap pancreas and risk of diabetes mellitus in childhood cancer survivors: A
retrospective cohort study. Lancet Oncol. 2012;13(10):1002-1010.
between the original neoplasm and the current one it is more 14. van Waas M, Neggers SJ, Raat H, van Rij CM, Pieters R, van den
likely to be a secondary neoplasm. The diagnosis of this second Heuvel-Eibrink MM. Abdominal radiotherapy: A major determinant of
neoplasm is not certain but given that he has been irradiated, a metabolic syndrome in nephroblastoma and neuroblastoma survivors. PloS
One. 2012;7(12):e52237.
meningioma is one of the most probable diagnoses. From the
15. De Bruin ML, Sparidans J, vant Veer MB, et al. Breast cancer risk in
most recent data on GH treatment in CCS, no increased risk is female survivors of Hodgkins lymphoma: Lower risk after smaller radia-
found in the development of a secondary neoplasm or recur- tion volumes. J Clin Oncol. 2009;27(26):4239-4246.
rence of the primary tumor. 16. Patterson BC, Chen Y, Sklar CA, et al. Growth hormone exposure as a risk
factor for the development of subsequent neoplasms of the central nervous
system: A report from the childhood cancer survivor study. J Clin
REFERENCES Endocrinol Metab. 2014;99(6):2030-2037.
1. Meadows AT. Pediatric cancer survivors: Past history and future chal- 17. van Waas M, Neggers SJ, van der Lelij AJ, Pieters R, van den Heuvel-
lenges. Curr Probl Cancer. 2003;27(3):112-126. Eibrink MM. The metabolic syndrome in adult survivors of childhood
2. Jemal A, Siegel R, Xu J, Ward E. Cancer statistics, 2010. CA Cancer cancer, a review. J Pediatr Hematol Oncol. 2010;32(3):171-179.
J Clin. 2010;60(5):277-300. 18. Bhasin S, Cunningham GR, Hayes FJ, et al. Testosterone therapy in men
3. Ries LAG, Eisner MP, Kosary CL. SEER Cancer Statistics Review, with androgen deficiency syndromes: An Endocrine Society clinical prac-
1975-2002. National Cancer Institute: Bethesda, 2005. tice guideline. J Clin Endocrinol Metab. 2010;95(6):2536-2559.
4. Shalet SM, Beardwell CG, Morris-Jones PH, Pearson D. Pituitary 19. Molitch ME, Clemmons DR, Malozowski S, Merriam GR, Vance ML,
function after treatment of intracranial tumours in children. Lancet. Endocrine S. Evaluation and treatment of adult growth hormone
1975;2(7925):104-107. deficiency: An Endocrine Society clinical practice guideline. J Clin
5. Chemaitilly W, Hudson MM. Update on endocrine and metabolic therapy- Endocrinol Metab. 2011;96(6):1587-1609.

The Endocrine Society. Downloaded from press.endocrine.org by [${individualUser.displayName}] on 12 January 2017. at 12:19 For personal use only. No other uses without permission. . All rights reserved.
280 ENDO 2016 MEET-THE-PROFESSOR CLINICAL CASE MANAGEMENT

Puberty Disorders in Girls

M30 Understand the complexity of diagnosing and treating


Presented, April 1 4, 2016 adolescents with delayed pubertal development.
Determine appropriate diagnostic and effective
therapeutic interventions in adolescent girls with
Sally Radovick, MD. Rutgers Robert Wood Johnson hyperandrogenism.
Medical School, New Brunswick, New Jersey 08901,
E-mail: s.radovick@rutgers.edu
STRATEGIES FOR MANAGEMENT OF
PRECOCIOUS PUBERTY
SIGNIFICANCE OF THE CLINICAL PROBLEM Background
Puberty is the developmental process that culminates in repro- Normal pubertal development in girls follows an ordered se-
ductive capability. It is initiated by the release of GnRH from quence for breast and pubic hair development, beginning with
specialized neurons of the hypothalamus to stimulate a hor- thelarche and progressing as described by Tanner. Puberty
monal cascade resulting in gonadal activation and secretion of begins when GnRH neurons of the hypothalamus secrete
sex steroids. The age of pubertal onset in girls may be younger GnRH in a pulsatile manner. Pulsatile GnRH causes pituitary
than in previous decades, emphasizing the gaps in knowledge gonadotrophs to release LH and FSH. LH induces the produc-
about pubertal onset and factors that modulate it. Pubertal tion of androstenedione in the ovarian thecal cells, whereas
delay or permanent hypogonadism are difficult to differentiate FSH induces the aromatase enzyme in follicular cells to syn-
due to the spectrum of disorders and the lack of definitive thesize estradiol. The increase in serum estradiol causes breast
biochemical criteria. Abnormalities in pubertal development tissue to enlarge and become palpable. Precocious puberty is
may manifest with significant consequences in later life, in- defined as breast budding younger than 2.5 SDs from the mean
cluding adolescent antecedents of polycystic ovarian syndrome or younger than 8 years of age (1). A controversial cross-
(PCOS). This review describes the state of the debate on the sectional study of 17 000 girls from the Pediatric Research in
age of pubertal initiation, the relationship of childhood obesity Office Settings network, using visual inspection of girls seen
to metabolic consequences in adult life, and the uncertainties for regular office visits, reported that 6.5% of white girls and
associated with the diagnosis of hypogonadotropic hypogonad- 27.2% of Black girls had breast or pubic hair development
ism in adolescence. before the age of 8 years. The authors concluded that applica-
tion of the traditional definitions of precocious puberty would
BARRIERS TO OPTIMAL PRACTICE result in a high proportion of potentially normal girls undergo-
Recent studies suggest that the average age of pubertal onset is ing extensive and expensive testing for precocious puberty (2).
decreasing in American girls, sparking controversy in defining Subsequent studies substantiated the ethnic differences in the
the age at which puberty is considered precocious. This con- trend toward early breast development. A panel convened to
troversy results in significant implications for the diagnosis, as analyze secular trends in pubertal timing; however, a younger
well as extensive and expensive testing for precocious puberty average age of pubertal onset was not proposed because the
and consideration of therapy. Adolescence is a time of transi- analyzed studies had disparate data on the timing of breast
tion from the prepubertal state to true puberty; however, there budding (3).
are no diagnostic tests to mark the transition. Hence the ability
to differentiate constitutional delay of puberty from permanent Precocious Puberty
hypogonadotropic hypogonadism is difficult. Obesity in ado- When breast or sexual pubic hair development begins before
lescents may result in metabolic syndrome in adulthood and the age of 8.0 years or menses begin before the age of 9.5
optimum therapeutic intervention is not known. Further, debate years, puberty is traditionally considered precocious. It should
persists on whether adolescent PCOS exists, how to define it, be kept in mind that breast development during the seventh
whether to treat it, and what therapy may be efficacious. year is within normal limits in ethnic minority girls. In addi-
tion, presexual pubic hair (stage II) may be normal in 6- and
LEARNING OBJECTIVES 7-year-old ethnic minority girls. Puberty can occur prematurely
As a result of participating in these sessions, learners will be as an extreme variation of normal, because of a disturbance in
able to: the hypothalamic-pituitary-gonadal (HPG) axis or because of a
Understand the role of the hypothalamic-pituitary- disturbance outside the HPG axis. It is important to distinguish
gonadal axis on the onset of puberty. between true precocious puberty and pseudoprecocious pu-
Define the parameters for evaluation and treatment of a berty. True precocious puberty is gonadotropin dependent.
girl with sexual precocity. Pseudoprecocious puberty is gonadotropin independent. Ap-

The Endocrine Society. Downloaded from press.endocrine.org by [${individualUser.displayName}] on 12 January 2017. at 12:19 For personal use only. No other uses without permission. . All rights reserved.
ENDO 2016 PEDIATRIC ENDOCRINOLOGY 281

proximately 95% of true precocity in girls is idiopathic and Management


may be attributed to premature triggering of the normal puber- The goals in management are to rule out an organic disorder
tal mechanism (4). Precocity in the 6 8-year age range usually that requires treatment and to ascertain whether sexual precoc-
is not rapidly progressive and most commonly seems to be due ity is either compromising height potential or resulting in
to excessive adiposity. Any type of intracranial disturbance can important secondary psychological disturbances in the child.
cause true isosexual precocity. True puberty may begin after The situation of a girl presenting with the onset of breast
correction of virilizing or feminizing disorders that have ad- development or pubic hair between 6 and 8 years of age
vanced the bone age to 10 12 years. Rare causes of true sex warrants special consideration. Breast development between 7
precocity include mutations that enhance kisspeptin signaling, and 8 years of age is normal in Blacks and Hispanics. How-
inactivating mutation in a GnRH release-inhibiting signaling ever, pubertal development in girls in the 6 8 year age range
molecule MKRN3, maternal uniparental disomy of chromo- may be associated with pathology, with excessive adiposity
some 14, and hyperglycinemia (1). being the major consideration in most. Many 6 8-year-old
girls with central precocious puberty have slowly progressive
precocity, with a normal timing of menarche, and are at low
Diagnosis risk of short adult stature. Most such girls do not require GnRH
In the history, the physician should inquire about the possibility of
agonist therapy to preserve adult stature. Thus, a less compre-
exposure or access to exogenous steroids in the form of unusual
hensive investigation may be warranted in selected girls pre-
creams, pills, or diet. In the examination, the physician should
senting with thelarche between 6 and 8 years of age. For most
search for nevi, acanthosis nigricans, signs that might suggest
of these girls, a complete history and physical examination,
intracranial or abdominal-pelvic disease, and inspect the external
including obesity evaluation and a bone age determination may
genitalia. The childs height and weight should be recorded, the be all that is needed, along with careful longitudinal followup.
growth curve examined, and the body mass index (BMI) percen- However, 6 8-year-old girls with a suggestion of rapidly pro-
tile plotted. A bone age determination is indicated to screen for gressive feminization, neurologic symptoms, linear growth ac-
whether appreciable hormone excess exists only if the history and celeration, or significant bone age advancement should be more
examination are unremarkable. If the skeletal age is not abnor- completely evaluated. The mismatch between physical, hor-
mally advanced relative to height age, it is likely that the present- monal, and psychological development may cause behavior
ing symptom is a variant of normal, which requires no treatment. changes ranging from social withdrawal to aggression or sexu-
To confirm the diagnosis, the child must be reevaluated after 3 6 ality. When central precocity is accompanied by documented
months. If more than one sign of precocious puberty is present or progression of pubertal development that accelerates growth
develops or if the growth is accelerated, a more-extensive workup and compromises normal height potential, GnRH agonist treat-
is indicated. Bone age advancement that is currently or becomes ment is indicated. Documentation typically requires 3 6
disproportionate to height (as indicated by compromised height months but may be unnecessary if puberty is substantially
potential or bone age 20% greater than height age) is an indica- advanced clinically and hormonally on presentation. The
tion for a more-extensive investigation to determine the cause of down-regulating effect of GnRH agonists on pituitary gonado-
the precocity. The laboratory investigation of premature pubertal tropin release inhibits gonadotropin secretion within 1 month.
development requires determinations of sex steroids, LH, and Treatment is adequate if the estradiol and baseline LH levels
FSH by assays of high sensitivity: at least 10 pg/mL for estradiol, become prepubertal or LH is below 4.0 U/L (1 h) or 6.6 U/L (2
10 ng/dL for T, 5 mcg/dL for dehydroepiandrosterone sulfate h) after GnRH agonist 1 month after institution of therapy.
(DHEAS), and 0.2 U/L for LH and FSH. Measurement of T and Arrest of breast development and the pubertal growth spurt
estradiol require assays of high sensitivity and specificity such as become apparent by 3 6 months. Long-term safety data remain
postchromatographic RIA or tandem mass spectrometry. Prepu- incomplete, but current studies following subjects into young
bertal estradiol levels are normally less than 10 pg/mL, and pre- adulthood are reassuring (1, 4).
pubertal T less than 20 ng/dL. Examination of the vaginal mucosa
for estrogen effect is a more-sensitive indicator of the presence of Hypogonadotropic Hypogonadism
early puberty than is an estradiol blood level because it represents Congenital gonadotropin deficiency can occur in association
the integrated effect of estrogen. Early morning basal LH greater with cerebral, hypothalamic, or pituitary dysfunction or as an
than 0.6 is diagnostic of central precocious puberty in girls. A isolated defect. Congenital hypothalamic dysfunction may be
post-GnRH peak LH greater than 6.9 U/L has been reported to be associated with other neurologic or endocrine dysfunction,
92% sensitive and 100% specific, and a post-GnRH agonist peak such as in the Prader-Willi syndrome or the Laurence-Moon-
LH greater than 4.0 5.0 U/L has been reported to beat least 90% Biedl syndrome. Congenital hypogonadotropic hypogonadism
accurate for the diagnosis of central precocious puberty (5, 6). may result from mutations in genes responsible for GnRH
Magnetic resonance imaging (MRI) of the hypothalamic-pituitary neuronal migration, the anosmin-signaling pathway, gene ex-
area is indicated in progressive true sexual precocity, especially pression, or protein section. Currently, mutations in approxi-
those less than 6 years old or those at risk of organic causes. mately 25 genes have been implicated in the etiology of

The Endocrine Society. Downloaded from press.endocrine.org by [${individualUser.displayName}] on 12 January 2017. at 12:19 For personal use only. No other uses without permission. . All rights reserved.
282 ENDO 2016 MEET-THE-PROFESSOR CLINICAL CASE MANAGEMENT

hypogonadotropic hypogonadism. GnRH-receptor mutations estrogen exposure but does not mean that it is current. Deter-
account for approximately half of autosomal-recessively inher- mination of serum estradiol is the simplest test, but diurnal and
ited cases of isolated normosmic gonadotropin deficiency. The cyclic variations must be taken into account. A progestin with-
degree of hypogonadism is variable. Loss-of-function muta- drawal test is often helpful. A female who does not experience
tions of GnRH and in signaling systems that modulate GnRH progestin withdrawal bleeding probably has an ambient estra-
release (kisspeptin/GPR54, neurokinin B/TAC3R) are rare diol level of less than approximately 40 pg/mL. If bleeding
causes. Gonadotropin gene mutations are also very rare. The does not occur in response to this maneuver, the integrity of the
hypogonadism of most subjects with neurokinin B/TAC3R uterus can be demonstrated by eliciting withdrawal bleeding
mutations is reversed by sex-steroid therapy, which suggests after a 3-week course of estrogen-progestin, most conveniently
that this signaling system is important for the initiation of administered in the form of birth-control pills. Pelvic ultra-
puberty, but not its maintenance. Acquired gonadotropin defi- sound may demonstrate hypoplastic ovaries, endometrial hypo-
ciency can be a consequence of tumors, trauma, autoimmune plasia or disorders, or polycystic ovaries. Magnetic resonance
hypophysitis, degenerative disorders involving the hypothala- imaging of the hypothalamic-pituitary area is important in the
mus and pituitary irradiation, chemotherapy, or chronic illness. workup of gonadotropin deficiency, hyperprolactinemia, and
Pituitary adenoma, craniopharyngioma, and dysgerminoma are hypothalamic anovulation.
the most common neuroendocrine neoplasms responsible for
hypopituitarism in children (1, 7). Anorexia nervosa is a com-
mon cause of hypogonadotropinism in teenagers (8). Hypothy- Therapy
For prolactinomas, dopaminergic treatment is the initial
roidism and hyperprolactinemia are potentially reversible
causes of gonadotropin deficiency. treatment of choice and other tumors require surgery and/or
radiotherapy. Underlying disorders must be treated appropri-
ately. Anorexia nervosa is best managed by an experienced
Diagnosis multidisciplinary team. In patients in whom short stature is
Investigation for hypogonadism should begin when puberty is
an important concern, growth potential must be considered
delayed or does not progress normally. Delayed puberty is
before undertaking estrogen replacement. Transdermal es-
indicated by lack of thelarche by the chronologic or bone age
tradiol is a convenient, physiologic form of therapy that
of 13 years. Abnormal progression of puberty is indicated by
seems to have long-term health advantages over commonly
failure of menses to occur within 4.5 years of the onset of
used oral estrogens, it is recommended to start transdermal
puberty or if secondary amenorrhea or oligomenorrhea has
feminization with 25 mcg daily for 1 week per month, and
persisted for 1 year. A family history of delayed puberty is
escalating at 6-month intervals to an adult dose at 3 years. For
compatible with the delay being constitutional rather than hav-
girls with hypogonadism and an intact uterus, cyclic progestin
ing an organic basis. The laboratory workup depends on the
should be added after 2 years of estrogen therapy or when
degree of estrogenization, as initially assessed from the stage of
bleeding begins to occur at unpredictable times. A simple
breast development: it includes a bone age radiograph in ado-
regimen is to use 100 mg of micronized progesterone at bed-
lescents who are not sexually mature and generally begins with
time for 714 days during the second to third week of estrogen
a chronic disease panel, and determination of gonadotropins,
therapy. Once optimal height is achieved, most patients prefer
estradiol, and T level. FSH elevation suggests primary ovarian
failure. Chromosome abnormalities are ordinarily the first con- to switch to birth control pills. Hypogonadotropic patients can
sideration given that the most common cause is Turner syn- achieve ovulation with gonadotropin therapy and GnRH defi-
drome and its variants. Those individuals with primary ovarian ciency can be successfully treated by pulsatile GnRH (1, 9).
failure that is not due to Turner syndrome and its variants
should be investigated for the fragile X premutation. In early Adolescent Hyperandrogenism
puberty, the gonadotropins may not be elevated until central Adolescent hyperandrogenism is usually due to PCOS, but the
nervous system maturation has been reached as indicated by a differential diagnosis includes other ovarian or adrenal disorders,
bone age of approximately 10 11 years. If FSH is not elevated abnormal peripheral formation of androgen, and drugs. Several
and bone age has reached 11 years, in a prepubertal girl international conferences have developed somewhat different but
without a growth disorder, the differential diagnosis becomes overlapping diagnostic criteria for adult women (10 12). Criteria
either constitutional delay of puberty or isolated gonadotropin for PCOS diagnosis have been additionally controversial in ado-
deficiency. The distinction of constitutional delay from isolated lescents because menstrual cycles, hyperandrogenic features,
gonadotropin deficiency may be difficult given that LH levels polycystic ovary morphology, and insulin resistance have special
in hypogonadotropic patients often overlap those of pre- and characteristics during this developmental stage. Endocrine Society
midpubertal normal children. The single most useful test is the guidelines suggest that the diagnosis in adolescents be limited to
LH level in response to GnRH agonist to at 3 4 hours (6, 9). those with persistent hyperandrogenic menstrual anovulation. Per-
The assessment of an adolescents degree of estrogenization is sistence of symptoms for 12 years has been suggested to avoid
often difficult. Breast development indicates that there has been overdiagnosis; however, this may lead to delay in therapy. Hirsut-

The Endocrine Society. Downloaded from press.endocrine.org by [${individualUser.displayName}] on 12 January 2017. at 12:19 For personal use only. No other uses without permission. . All rights reserved.
ENDO 2016 PEDIATRIC ENDOCRINOLOGY 283

ism is graded according to the Ferriman-Gallwey system, which diabetes and cardiovascular disease. Virilizing congenital adrenal
quantitates the extent of hair growth in androgen-sensitive areas. hyperplasia frequently causes ovarian hyperandrogenism. Ste-
A score of 8 or more is abnormal. Menstrual irregularity is present roidogenic blocks in ovarian steroid synthetic pathways, such as
in approximately two thirds of PCOS cases. The distinction be- caused by 3-HSD or aromatase deficiency17-ketosteroid reduc-
tween PCOS and physiologic anovulation is often delayed in tase deficiency can cause hyperandrogenism. Extreme insulin resis-
adolescents because of the broad normal range of menstrual cycle tance, acromegaly, Cushings syndrome, and prolactin excess are
variation. PCOS is a state of relative, not absolute, infertility in accompanied by PCOS. Valproic acid causes hyperandrogenism and
which ovulation is unpredictable. Hence, contraception is recom- polycystic ovaries, and an association of epilepsy itself with PCOS is
mended for sexually active adolescents. Obesity, present in ap- possible. Functional ovarian hyperandrogenism may also result from
proximately one half of patients with PCOS, often is the initial an ovotesticular disorder of sex development. An extremely high
complaint. Normal-weight young women with PCOS are reported level of hCG due to tumor or a virilizing tumor are rare causes of
to have a body fat content that is 50% greater than normal. hyperandrogenism. A rapid pace of development or progression of
Acanthosis nigricans, a manifestation of insulin resistance, may be hyperandrogenic signs of hirsutism raise concern for an androgen-
the presenting complaint of patients with PCOS (1, 10 12). secreting neoplasm; however, tumors can have indolent presentations.
Adrenal hyperandrogenism can on rare occasions arise from other
rare congenital disorders of adrenal steroid action or metabolism, such
Diagnosis
Serum T is the single most important androgen to evaluate. as glucocorticoid resistance, apparent cortisone reductase deficiency,
Serum free T is approximately 50% more sensitive in detecting and apparent sulfotransferase deficiency. Drug-induced masculiniza-
excessive androgen production because hyperandrogenic tion in adolescence should be considered especially in athletes (13).
women have a relatively low level of SHBG. There are many
pitfalls in T assays and direct assays of the free T concentration Management
are inaccurate and should be avoided. Functional ovarian Management is individualized according to symptoms and pa-
hyperandrogenism can be documented by the GnRH agonist tient goals. Given that PCOS is associated with the early
and human chorionic gonadotropin (hCG) tests. Most patients development of type 2 diabetes mellitus and metabolic syn-
with PCOS show an increased 17-hydroxyprogesterone respon- drome, a fasting lipid panel and oral glucose tolerance test are
siveness without evidence of a steroidogenic block. The dexa- recommended and screening for sleep-disordered breathing.
methasone androgen-suppression test may be used to document Given that PCOS is closely related to parental metabolic syn-
the increase in ovarian androgens after suppression of adrenal drome, a similar evaluation of primary relatives should be
function by glucocorticoid administration. It shows elevated T considered. Women with PCOS are at increased risk of mood
postdexamethasone in 80% of patients with PCOS. Ovarian function and anxiety disorders and depression and should be screened
tests are normal in 15% of PCOS patients; the androgen excess seems for these disorders. Cosmetic measures are the cornerstone of
due to isolated functional adrenal hyperandrogenism or obesity. A care for hirsutism. Oral contraceptive pills (OCPs) are the
polycystic ovary, although not completely specific for PCOS, is found first-line endocrine treatment for women with the dermatologic
in approximately two thirds of adolescents with PCOS. A normal or menstrual abnormalities of PCOS. They act by suppressing
ultrasound examination does not exclude PCOS. An ovary with at serum androgens, particularly free T, mainly by inhibiting
least 10 small antral follicles (29 mm) in the maximum plane or ovarian function. They also raise SHBG and modestly lower
greater than 10.8 cc in volume (in the absence of a follicle 10 mm) DHEA sulfate levels. It is advisable to recheck patients after 3
in an adolescent seems equivalent to the international consensus months of therapy to assess the efficacy of treatment and
definition of the adult polycystic ovary. Endocrine screening is indi- normalization of androgen levels. OCP treatment should be
cated to rule out pregnancy, hyperprolactinemia, to screen for continued until the patient is gynecologically mature (5 y
nonclassic congenital adrenal hyperplasia with DHEAS and AM postmenarcheal) or has lost a substantial amount of excess
17-hydroxyprogesterone, and assessment for Cushings syndrome, weight. At that point, withholding treatment for a few months
thyroid dysfunction, or acromegaly. Pelvic ultrasound is also use- to allow recovery of suppression of pituitary-gonadal function
ful in ruling out tumor and disorders of sexual differentiation. An and to ascertain whether the menstrual abnormality persists is
anti-Mullerian hormone elevation of 2-fold or more suggests advisable. The addition of antiandrogens generally yields im-
PCOS associated with the increased number of growing follicles. provement in hirsutism. Spironolactone in high dosage is the
LH may be increased in PCOS and is associated with the degree safest potent antiandrogen. The recommended starting dose is
of hyperandrogenism. Insulin resistance occurs with approxi- 100 mg twice a day until the maximal effect has been achieved
mately half of obese adolescents with PCOS. Metabolic syndrome and then attempting to reduce the dose to 50 mg twice a day for
results from the interaction of insulin resistance with obesity and maintenance therapy. Insulin-lowering treatments, from weight
age. It is associated with abdominal obesity, elevated blood pres- loss to drug treatment improve hyperandrogenism. Metformin
sure, serum triglycerides, glucose, and lowered high-density lipo- therapy should start with 500 mg daily of the extended-release
protein (HDL). It poses a 2-fold increased risk for sleep- form accompanying a meal, with an increase in the dose by 500
disordered breathing in adolescence, as well as long-term risks for mg per week to a maximal dose of 2000 mg, usually divided

The Endocrine Society. Downloaded from press.endocrine.org by [${individualUser.displayName}] on 12 January 2017. at 12:19 For personal use only. No other uses without permission. . All rights reserved.
284 ENDO 2016 MEET-THE-PROFESSOR CLINICAL CASE MANAGEMENT

twice a day as tolerated. It is advisable to obtain a comprehen- bimanual examination. Her complete blood count, blood chem-
sive metabolic panel at baseline to confirm normal hepatic and istry panel, and beta subunit of human chorionic gonadotropin
renal function. Bariatric surgery has led to improvement in are all normal. Her TSH and free T4 were low and she was
androgen levels and menses in the vast majority, but improve- placed on levothyroxine.
ments in hirsutism and ovulation have been inconsistent in 1. Which of the following is the best next diagnostic step in
adults (1). this patients evaluation?
A. Progesterone challenge test
MAIN CONCLUSIONS B. LH, FSH, E, prolactin
The diagnosis of pubertal disorders requires expert knowledge C. Free T and DHEAS
about the normal physiology associated with pubertal onset and D. MRI of brain and pituitary gland
the mechanism by which disorders are mediated. The diagnosis E. Karyotype
of sexual precocity in the 6 -8-year-old is controversial with
new data from population-based studies; however, no guide- Case 3
lines exist for the diagnostic and therapeutic interventions. A 16-year-old girl is seen for evaluation of hirsutism and
These patients require careful followup. Diagnosis of amenor- obesity. She has had repeated sports-related injuries. She had
rhea and hyperandrogenism in the adolescent is complicated at menarche at 12 years of age and has three or four menstrual
this developmental stage and the associated psychological im- periods per year. OCPs were recommended by her primary medi-
plications add further challenges. In both cases, timely diagno- cal doctor but they were declined by her mother. Both her mother
sis of the etiology and institution of appropriate therapy is and father have type 2 diabetes. Her BMI is 32 kg/m2, and her
indicated. blood pressure is 135/75 mm Hg. Her Feriman Gallway score is
12 and she has acanthosis nigricans. Her serum total T level is
elevated at 0.7 ng/mL, her plasma high-density lipoprotein choles-
CASES WITH QUESTIONS
terol (HDL-C) level is 28 mg/dL, and her triglyceride level is
Case 1
140 mg/dL. Her serum glucose level 2 hours after the
A 7-year-old Latina presents after her mother noticed breast
ingestion of 75 g of dextrose is 128 mg/dL.
development approximately 6 months ago. She had a normal
1. Which of the following is the best next diagnostic step in
prenatal course, was born appropriate for gestational age, and
this patients evaluation?
reached normal developmental milestones. Her mother is con-
A. Pelvic ultrasound
cerned that she is hanging around girls in the fourth grade.
B. Serum LH level
Mom had menarche at 11 12. Her height is at the 75th percen-
C. Adrenal androgens (DHEAS), AM 17-hydroxy-
tile for age, her weight is at the 90th percentile for age; her
progesterone
midparental height is at the 50th percentile. Her examination
D. All of the above
was unremarkable except for Tanner II breast development
without pubic hair. She had a normal neurologic examination. E. B and C
Laboratory studies show a morning LH, 2.2 U/L; FSH 0.2 2. In addition to diet and exercise, which of the following
U/L; estradiol, 40 pg/mL. Her bone age is 8 years. treatments would you suggest at this time?
Which of the following would be the best next step in this A. Diet and exercise
childs evaluation? B. Metformin
A. Brain MRI C. Antiandrogen (ie, spironolactone)
B. Pelvic ultrasound D. A and B
C. Reassurance and followup in 3 6 months
D. GnRH agonist stimulation test DISCUSSION OF CASES AND ANSWERS
Case 1
Case 2 Recent studies suggest that the average age of pubertal onset is
A 16-year-old girl presents for evaluation of amenorrhea. She decreasing in American girls, sparking controversy in defining
reports her menarche was at the age of 15 years and she has the age at which puberty is considered precocious. Studies have
had only one bleeding episode. Since she started running for also shown ethnic differences in the trend toward early breast
exercise and sport at the age of 14 years, she has lost approxi- development, especially among African-American and Latina
mately 5 lb. She runs 35 miles per day. She denies galactor- girls. It has been suggested that obesity may play a role in the
rhea, headache, or visual changes. She has experienced some possible early onset of puberty in girls, as girls with early onset
ankle pain for the last few weeks. Her mother had menarche at of breast budding have higher BMI scores than age-matched
16 years and she has an aunt with infertility. On physical girls without budding. Significant implications arise regarding
examination, her BMI is 20 kg/m2. Her physical examination this diagnosis, including extensive and expensive testing for
reveals Tanner III breasts, stage II pubic hair, and a normal precocious puberty and consideration of therapy. The only

The Endocrine Society. Downloaded from press.endocrine.org by [${individualUser.displayName}] on 12 January 2017. at 12:19 For personal use only. No other uses without permission. . All rights reserved.
ENDO 2016 PEDIATRIC ENDOCRINOLOGY 285

permanent physical complication of true isosexual precocity is rhea have a low serum estradiol and low or low-to-normal
short adult height. Excessive sex hormone production causes levels of LH and FSH, with a preserved gonadotropin response
early maturation of the epiphyses resulting in their premature to GnRH or GnRH analog. Estradiol assays can be limited by
closure. Approximately 50% of girls with this disorder reach an poor assay sensitivity, variation among assays, and the fact that
adult height of 5359 inches and the remainder are over 60 a measurement reflects a single time point. Short-term admin-
inches tall. This young girl has no clinical evidence of progres- istration of medroxyprogesterone acetate (10 mg for 10 d) may
sive puberty; hence, close followup is indicated. However, be useful in the evaluation; the onset of normal menstrual
6 8-year-old girls with a suggestion of rapidly progressive or bleeding after cessation of this drug (usually within 1 to 3
excessive androgenization or feminization, neurologic symp- days) suggests estrogen sufficiency. MRI of the brain is not
toms, linear growth acceleration, or significant bone age ad- routinely needed in patients with hypothalamic hypogonadism.
vancement should be more completely evaluated. Ultrasonog- It is often clinically challenging to differentiate adolescents
raphy is indicated to screen for abdominal or pelvic masses with CDP from those with isolated hypogonadotropic hypogo-
when progressive feminizing or virilizing disorders are sus- nadism (IHH), which usually is a permanent condition. Distin-
pected. MRI of the hypothalamic-pituitary area is indicated in guishing between these conditions is especially difficult be-
progressive true sexual precocity, especially those less than 6 cause adolescents with both etiologies are often prepubertal or
years old or those at risk of organic causes by virtue of their in early puberty and have low levels of gonadotropins. LH and
underling condition or neurologic symptoms and signs. If the FSH levels are low in CDP because the hypothalamic-pituitary-
precocious development continues over the next 3 4 months a gonadal axis has not yet matured to secrete pubertal levels of
GnRH test may be considered, especially if GnRH agonist GnRH and levels are low in IHH because of a lack of GnRH
therapy is being considered. A post-GnRH peak LH greater secretion or action. There are clinical features that can poten-
than 6.9 U/L has been reported to be 92% sensitive and 100% tially distinguish CDP from IHH, although these are usually
specific, whereas a post-GnRH agonist peak LH greater than not diagnostic. A family history of delayed puberty is strongly
4.0 5.0 U/L has been reported to be at least 90% accurate for suggestive of CDP although individuals with CDP are also
the diagnosis of central precocious puberty. The GnRH agonist seen among pedigrees with IHH. An important consideration in
test also permits assessment of the ovarian gonadotropin- our patient is that 20-40% of patients with IHH have evidence
responsiveness: an estradiol peak at least 34 pg/mL is approxi-
of initial but then stalled pubertal development, and 10% may
mately 90% sensitive and at least 60 pg/mL 95% specific for
have sustained reversal of IHH in adulthood. Thus, in many
puberty. FSH levels are not as helpful diagnostically given that
cases, routine initial clinical evaluation cannot distinguish CDP
prepubertal values overlap considerably with pubertal ones and
from IHH with certainty. Basal gonadotropins have limited
they may be elevated in premature thelarche.
discriminatory ability and unfortunately, significant variability
in response to GnRH and GnRH agonist testing has not led to
Case 2 an inability to distinguish accurately between the two groups.
This represents a complex case with delayed pubertal develop-
GnRH agonists seem to offer better discriminatory value than
ment in a female athlete, a family history of delayed puberty,
GnRH stimulation testing. There are no studies in female
and weight loss. Appropriate initial testing was performed,
adolescents looking at the diagnostic utility of inhibin B. De-
including a complete blood count and blood chemistry panel to
spite significant progress made in identifying genes that
rule out chronic illness, pregnancy test beta subunit of hCG,
cause IHH, mutations in known genes explaining CDP are
TSH, and free T4 (to rule out both primary and central hypo-
unknown. Thus, considering the limitations in our under-
thyroidism). LH, FSH, and prolactin should also be performed.
standing of the genetic basis of IHH and CDP, routine
Free T and DHEAS may also be performed if clinically indi-
testing is not recommended.
cated. It should be recognized that the results of thyroid func-
tion tests in patients with eating disorders can resemble the
pattern in those with central hypothyroidism. Management
Functional hypothalamic amenorrhea is a condition charac- Initial management should include an evaluation of her ankle
terized by the absence of menses due to the suppression of the pain considering the risk for stress fractures along with a
hypothalamic-pituitary-ovarian axis, in which no anatomical or recommendation to decrease her level of activity. Appropriate
organic disease is identified. In adolescents, this condition may nutritional counseling should be suggested. A bone densitom-
be difficult to differentiate from constitutional delay of puberty etry study is warranted. Although oral contraceptive (OC) pills
(CDP) during the initial postmenarchal years. Hypothalamic are commonly prescribed to improve skeletal status, several
amenorrhea reflects a state of estrogen deficiency, which may studies suggest that this intervention did not lead to a signifi-
compromise peak bone mass attained in young women. The cant increase in bone density. An adequate intake of calcium
term female athlete triad refers to the interplay between low (1300 mg/d of elemental calcium) and vitamin D (400 1000
energy availability (with or without an eating disorder), amen- IU/d) is recommended, although appropriate supplementation
orrhea, and osteoporosis. Patients with hypothalamic amenor- doses are debated.

The Endocrine Society. Downloaded from press.endocrine.org by [${individualUser.displayName}] on 12 January 2017. at 12:19 For personal use only. No other uses without permission. . All rights reserved.
286 ENDO 2016 MEET-THE-PROFESSOR CLINICAL CASE MANAGEMENT

Case 3 approved by the United States Food and Drug Administration


The patient presents with obesity, hirsutism, irregular menses, in children or for the treatment of PCOS, it is commonly used
and hyperlipidemia. Her parents have type 2 diabetes mellitus. for this purpose.
She fits the criteria for polycystic ovary syndrome (PCOS).
Pelvic ultrasound is useful in ruling out tumor and disorders of ANSWERS
sexual differentiation, although both are unlikely due to the Case 1: C
time course of the disorder and the T level. Screening is Case 2: B
indicated to rule out nonclassic congenital adrenal hyperplasia Case 3.1: E
with DHEAS and AM 17-hydroxyprogesterone. Serum LH Case 3.2: D
may be increased in PCOS and is associated with the degree of
hyperandrogenism. An elevated serum LH level may be a REFERENCES
useful therapeutic follow-up measure; however, a normal level 1. Rosenfield RL, Cooke DW, Radovick S. The ovary and female maturation.
does not exclude PCOS. In: Sperling M, editor. 4th, ed. Pediatric Endocrinology. 3rd . Philadelphia,
PA: Elsevier, 2008.
An estrogen-progestin OC, with or without an antiandrogen
2. Herman-Giddens ME, Kaplowitz PB, Wasserman R. Navigating the recent
such as spironolactone, has been the mainstay of therapy. This articles on girls puberty in Pediatrics: what do we know and where do we
approach is effective in achieving the traditional treatment go from here? Pediatrics. 2004;113:911-917.
goals in the polycystic ovary syndrome, which include amelio- 3. Parent AS, Teilmann G, Juul A, Skakkebaek NE, Toppari J, Bourguignon
JP. The timing of normal puberty and the age limits of sexual precocity:
rating the effects of androgen excess and restoring regular
Variations around the world, secular trends, and changes after migration.
menses, preventing endometrial hyperplasia. However, her Endocr Rev. 2003;24:668-693.
mother is adamant that her daughter not use OCPs. 4. Nakamoto JM, Franklin SL, Geffner ME. Puberty. In: Kappy MS, Allen
Although she currently does not have type 2 diabetes melli- DB, Geffner ME, eds. Pediatric Practice: Endocrinology, 2nd ed. New
York, NY: McGraw-Hill; 2013. Accessed November 13, 2015 from:
tus, she is at high risk. In addition to obesity, she has signs of
http://accesspediatrics.mhmedical.com.ezp.welch.jhmi.edu/content.
insulin resistance, which include a low serum HDL-C and a aspx?bookid1082&Sectionid61462284.
high triglyceride level. Although her glucose tolerance is cur- 5. de Vries L, Horev G, Schwartz M, Phillip M. Ultrasonographic and
rently normal, treatment with metformin is reasonable, and a clinical parameters for early differentiation between precocious puberty
and premature thelarche. Eur J Endocrinol. 2006;154:891-898.
weight-loss diet and exercise are also encouraged. Metformins 6. Rosenfield RL, Bordini B, Yu C. Comparison of detection of normal
mechanism of action is to inhibit hepatic glucose production, puberty in girls by a hormonal sleep test and a gonadotropin-releasing
and to some degree increase the sensitivity of peripheral tissues hormone agonist test. J Clin Endocrinol Metab 2013;98:1591-1601.
to insulin. The increase in insulin sensitivity has also been 7. Sedlmeyer IL, Palmert MR. Delayed puberty: Analysis of a large case series
from an academic center. J Clin Endocrinol Metab. 2002;87:1613-1620.
shown in nondiabetic women with PCOS. In women with 8. Yager J, Andersen AE. Clinical practice. Anorexia nervosa. N Engl J Med.
PCOS, treatment with metformin may decrease circulating an- 2005;353:1481-1488.
drogen levels, thereby improving menstrual cyclicity and po- 9. Palmert MR, Dunkel L. Clinical practice. Delayed puberty. N Engl J Med.
tentially improve hirsutism. With long-term use it has been 2012;366(5):443-453.
10. Rotterdam ESHRE/ASRM-Sponsored PCOS Consensus Workshop Group.
shown to retard or prevent progression to type 2 diabetes in Revised 2003 consensus on diagnostic criteria and long-term health risks
patients with impaired glucose tolerance. When metformin is related to polycystic ovary syndrome. Fertil Steril. 2004;81:19-25.
prescribed, advice about a weight-loss diet and an exercise 11. Azziz R, Carmina E, Dewailly D, et al. The Androgen Excess and PCOS
Society criteria for the polycystic ovary syndrome: The complete task
program should also be suggested. When initiating metformin,
force report. Fertil Steril. 2009;91:456-488.
OC agents and antiandrogens are not prescribed as they may 12. Johnson T, Kaplan L, Ouyang P, Rizza R. National Institutes of Health
affect menstruation or serum androgen levels and confound the evidence-based methodology workshop on polycystic ovary syndrome
assessment of therapeutic benefits of metformin. After 6 9 (PCOS). (http://prevention.nih.gov/workshops/2012/pcos/default.aspx).
NIH EbMW Report. Vol.1. Bethesda, MD: National Institutes of
months of treatment, the efficacy of metformin is assessed. If
Health, 2013:1-14.
hirsutism remains troublesome, an OC agent, antiandrogen, or 13. Rosenfield RL. Clinical review: Identifying children at risk for polycystic
both could be added to metformin. Although metformin is not ovary syndrome. J Clin Endocrinol Metab. 2007;92:787-796.

The Endocrine Society. Downloaded from press.endocrine.org by [${individualUser.displayName}] on 12 January 2017. at 12:19 For personal use only. No other uses without permission. . All rights reserved.
ENDO 2016 PEDIATRIC ENDOCRINOLOGY 287

Bone Fragility in Children: When to Worry and What To Do

M29 SIGNIFICANCE OF THE CLINICAL PROBLEM


Presented, April 1 4, 2016 Threats to bone health are increasingly a pediatric concern.
Genetic or acquired disorders can compromise gains in bone
quantity and quality leading to osteoporosis early in life (10).
Laura K. Bachrach, MD. Stanford University School Recurrent fractures in otherwise healthy youth may also indi-
of Medicine, Stanford, California 94035-5208, E-mail: cate underlying bone fragility (11). The stakes are high to
lkbach@stanford.edu address these early threats to bone health, because the founda-
tion for bone health is established during the first 2 decades (2).
Failure to optimize bone mass, quality, and architecture can
HISTORICAL OVERVIEW
result in an increased lifetime risk for osteoporosis. Genetic
In the mid-1970s, Dr Charles E Dent recognized that senile
factors determine an estimated 70% of the variability in PBM
osteoporosis is a pediatric disease, underscoring the impor-
but reaching ones genetic potential requires optimal modifi-
tance of optimizing peak bone mass (PBM) to help prevent
able factors (2).
osteoporosis in later life (1). An estimated 60% of the risk of
osteoporosis can be attributed to variability in PBM with dif-
fering rates of subsequent bone loss accounting for the remain- BARRIERS TO OPTIMAL PRACTICE
der. There has been considerable progress since then in identi- Despite advances in the diagnosis and treatment of pediatric
fying determinants of and threats to bone health in children and osteoporosis, important limitations persist. The goal of clinical
teens (2). The gains in bone mass and geometry associated with practice is to prevent fractures in at risk patients before they
growth and puberty have been characterized (3). Peak rates of occur and to reduce the likelihood of their recurrence. It re-
bone mineral accrual have been shown to lag behind peak mains challenging to predict which children will fracture. The
growth velocity by several months (4). In randomized con- surrogate measures of bone health (densitometry, bone mark-
trolled trials in healthy youth, calcium supplementation (5) and ers) used to assess fracture risk in adults are far more challeng-
increased weight bearing activity (6) have been shown to ing to interpret in children. Treatment options for children are
stimulate gains in bone mineral with the response influenced by limited as well. The pharmacologic agents that have proven
baseline calcium intake and pubertal stage. safe and effective to treat osteoporosis in adults have not been
A greater focus on early bone fragility also evolved in recent fully tested in children. Without randomized, controlled trials
decades. Clinicians encountered many challenges in adapting using clinical endpoints (fractures and bone pain), there is no
the diagnostic tools and therapies used in older adults to man- consensus about the optimal drug, dose, or treatment duration
age pediatric osteoporosis. The availability of dual energy for pediatric use. The limitations of current pharmacologic
x-ray absorptiometry after 1987 brought with it the promise of therapy make it more important to accurately predict who will
a safe, precise, noninvasive means to identify children most at fracture and who might recover spontaneously.
risk for fragility fractures. Initial studies with dual energy x-ray
absorptiometry (DXA) were hampered by a paucity of pediatric LEARNING OBJECTIVES
reference data and by the challenges of interpreting bone min- As a result of participating in this session, learners should be
eral density (BMD) in growing children. BMD is an areal able to:
measurement strongly influenced by bone size with apparently Identify the indications for bone densitometry in children
lower values in smaller individuals. Estimates of volumetric and teens.
BMD were developed as one approach to address this limita- Evaluate children who have had a low trauma fracture.
tion (7). Early DXA software reported BMD results in terms of Explain the criteria for the diagnosis of osteoporosis in
t-scores (comparing results to the means for healthy adults) pediatrics.
rather than or in addition to z-scores (SD scores for age and Discuss the benefits and limitations of pharmacologic
sex). This led to the erroneous diagnosis of osteoporosis in agents for osteoporosis in pediatrics.
younger patients who had not yet achieved PBM (8). Develop-
ing appropriate supportive and pharmacologic therapy for STRATEGIES FOR DIAGNOSIS AND
young patients with bone fragility has also proved challenging. MANAGEMENT OF BONE FRAGILITY
Bisphosphonate therapy was initially used for children with Conditions Linked to Bone Fragility
osteogenesis imperfecta with a dose extrapolated from that A number of genetic and acquired disorders have been linked
used for adult patients with Pagets disease (9). Drug therapy to increased bone fragility (Table 1). Bone health is threatened
for primary and secondary osteoporosis in children continues to in several of these conditions by reduced mobility, chronic
evolve. inflammation, undernutrition, and exposure to osteotoxic medi-

The Endocrine Society. Downloaded from press.endocrine.org by [${individualUser.displayName}] on 12 January 2017. at 12:19 For personal use only. No other uses without permission. . All rights reserved.
288 ENDO 2016 MEET-THE-PROFESSOR CLINICAL CASE MANAGEMENT

TABLE 1. Disorders Associated With Low Bone Mass T4 and TSH if hyperthyroidism is suspected, IGF-1 if GH
and Fractures in Childhood deficiency is suspected, and a urinary-free cortisol or midnight
Primary Bone Disorders salivary cortisol in the rare situation that glucocorticoid excess
Idiopathic juvenile osteoporosis is suspected. Genetic testing for osteogenesis imperfecta may
Osteogenesis imperfecta be warranted if no other cause for the fragility fractures is
Disorders Secondary to identified even if stigmata such as blue sclera or dentinogenesis
Inflammation imperfecta are absent.
Inflammatory bowel disease
Juvenile idiopathic arthritis Surrogate Measures of Bone Strength
Cystic fibrosis Bone turnover markers and bone densitometry have been used
Immobilization successfully in adults to identify older patients at greatest risk
Cerebral palsy and to consider drug therapy before a first fracture occurs
Myopathies (primary prevention). These surrogate measures must be inter-
Epidermolysis bullosa preted with caution in younger patients. Serum and urine bio-
Endocrine disturbance chemical markers of bone turnover are affected by myriad
Turner syndrome factors including age, pubertal stage, nutrition, growth velocity,
Anorexia nervosa time of day, and day-to-day variability, and a single value
Cancer and therapies with adverse effects on bone cannot be used to diagnose osteoporosis (12).
Acute lymphoblastic leukemia Bone densitometry is often a useful part of a comprehensive
After chemotherapy for childhood cancer skeletal assessment. The indications for performing bone den-
Following transplantation sitometry in pediatric patients have been outlined by the Pedi-
Hematologic disorders atric Position Development Conference (PDC), a panel of pe-
Thalassemia diatric bone experts (13). The anteroposterior (AP) lumbar
spine (L1-4) and total body less head are the preferred skeletal
sites for measurement in most children. For children ages 0 5
years, the spine bone mineral content (BMC) and BMD can be
cations or radiation. The likelihood of a fragility fracture for
measured; whole-body measurements are feasible only for
individual patients with any of these disorders is influenced by
those ages 3 or older. Additional regions of interest using DXA
age at onset, disease severity, medications, and genetic factors.
are recommended in special cases. Scans of the lateral distal
For this reason, clinical judgement is required to determine the
femur can be valuable in patients with immobilization disor-
extent of evaluation and treatment. Recurrent fractures in oth-
ders and in those with contractures who cannot be positioned
erwise healthy youth may also be an indication of reduced
properly for spine or whole-body scans (14). This site is also
skeletal health; when compared with controls without a history
helpful in assessing patients with immobilization disorders be-
of fracture, children who have had a broken bone are more
likely to have lower bone mineral, increased body fat, less cause bone fragility is greatest in the lower extremities (15).
physical activity, and siblings with a history of fracture (11). The distal radius can be measured in patients who exceed the
weight limit for the equipment. DXA scans should avoid areas
with metal implants, contractures, or vertebrae where fractures
Evaluation
Children with disorders linked to bone fragility warrant a have occurred. DXA results should be compared with norms
comprehensive assessment to identify any modifiable risk fac- collected from similar DXA equipment and corrected for age,
tors (2). In addition, any patient who has sustained a vertebral sex, race, and height using the robust norms available to gen-
or peripheral fracture after minimal or no trauma warrants a erate a z-score (16). T-scores, an expression of the SDs from
thorough investigation to identify potential cause. This assess- the mean for healthy young adult means, should not be used in
ment includes a review of illnesses, medication use, physical patients under age 18 (13). The PDC guidelines recommends
activity, calcium, vitamin D, and other nutrient intake, and a that the first densitometry examination be performed for the
history of fractures in the patient and relatives. It is helpful to conditions listed in Table 1 when the patient might benefit from
ask about hip fractures in the elderly and recurrent fractures in intervention and when the densitometry results would influence
any relative. management (13). Repeat scans (to monitory disease progres-
Laboratory evaluation includes serum calcium, phosphorus, sion or response to therapy) should be done more often than
magnesium, alkaline phosphatase, 25 OH vitamin D (25 OHD), every 6 12 months.
parathyroid hormone, blood urea nitrogen, creatinine and celiac Newer 3-dimensional densitometry methods [quantitative com-
screen. Urinary calcium to creatinine ratio should be measured, puted tomography (QCT), peripheral QCT (pQCT), magnetic
ideally on a 24-hour collection, to rule out hypercalciuria. resonance imaging (MRI)] offer valuable insights into volumetric
Other tests to consider include sex steroids in pubertal patients, bone density and bone micro- and macroarchitecture. At present,

The Endocrine Society. Downloaded from press.endocrine.org by [${individualUser.displayName}] on 12 January 2017. at 12:19 For personal use only. No other uses without permission. . All rights reserved.
ENDO 2016 PEDIATRIC ENDOCRINOLOGY 289

they remain primarily research tools due to a lack of standardized used bisphosphonate in pediatrics in doses ranging from 4 9
scanning protocols and normative data. mg/kgy. Zoledronic acid (ZA) is 100 200 times more potent
than pamidronate, can be infused more rapidly, and has a longer
Diagnosis of Osteoporosis duration of action (19). In case series, ZA given every 3 or 6
Despite advances in pediatric densitometry, the diagnosis of months in a dose of 0.1 mg/kg y has been shown to increase
osteoporosis in a pediatric patient cannot be made on the basis spine aBMD in children with type I osteogenesis imperfecta (19).
of BMD alone (13). The PDC guidelines established the fol- Variable efficacy has been reported with oral bisphosphonates
lowing criteria for the diagnosis of pediatric osteoporosis: (alendronate, risedronate) in reducing fractures (17). The maximal
One or more vertebral fractures occurring without high benefits from pamidronate therapy in children are seen after 2 4
energy trauma. Measuring BMD can add to the years, but low-dose maintenance bisphosphonate therapy may be
assessment of these patients but is not required as a needed until growth is complete in patients with persistent risk
diagnostic criterion. OR factors for osteoporosis such as those with OI (17).
Low bone density (BMC or areal BMD z-scores 2) Minor adverse reactions are common after the initial dose
and a significant fracture history (2 or more long bone of bisphosphonates including fatigue, fever, nausea, and
fractures before age 10 or 3 or more long bone fractures muscle aches. To date, no atypical femur fractures or avas-
before age 19). cular necrosis of the jaw have been observed in pediatric
patients. A concern for potential teratogenic effects has been
Management. General Measures raised since small amounts of bisphosphonates are released
Prevention and treatment of bone fragility must address all from bone for years and readily cross the placenta but this
potential threats to skeletal health (17). Calcium from diet and risk appears low (20).
supplements should total 1300 mg/d for ages 9 18 and daily
vitamin D should be a minimum of 600 IU after the first year
of life. Adequate total body stores of vitamin D are reflected in MAIN CONCLUSIONS
a serum 25 OHD concentration above 20 30 ng/mL. High- Several genetic and acquired disorders in children and teens
impact activities should be avoided in children with bone can compromise gains in bone mass, quality, and geometry
fragility, but modified activity as tolerated is important to resulting in clinical bone fragility (low trauma fractures of
counteract the adverse effect of immobility. Replacement of spine or long bones). Bone densitometry is part of a compre-
sex steroids and GH for those with documented deficiencies is hensive skeletal evaluation, but the diagnosis of pediatric os-
also appropriate. teoporosis is based upon fracture history. Low trauma vertebral
fractures alone or recurrent long bone fractures in combination
Pharmacologic Therapy for Osteoporosis with low bone mineral for age are required. Initial management
When low trauma fractures occur despite these general mea- includes addressing all risk factors (undernutrition, immobility,
sures, the addition of pharmacologic agents should be consid- hormone deficiencies or excess, and controlling the underlying
ered. An anabolic agent would be ideal, because pediatric disease). Pharmacologic therapy is generally reserved for pa-
osteoporosis often results from a failure to acquire bone rather tients with a history of fragility fracture. Bisphosphonates have
than accelerated loss. However, synthetic parathyroid hormone been used most extensively, although there remains no consen-
has a black box warning against its use in patients with open sus on optimal agent, dose, or duration.
growth plates. This limits pharmacologic therapy primarily to
antiresorptive agents. Sex hormone therapy (SHT) is indicated
CASES WITH DISCUSSION
for those with hypogonadism. However, the efficacy of SHT to
Case 1. You are asked to evaluate a 15-year-old female cross-
improve BMD or reduce fractures in exercise associated amen-
country runner with secondary amenorrhea for 6 months. She
orrhea or eating disorders remains uncertain. In light of this,
has had 1 stress fracture of the foot 4 months ago. Her mother
the American College of Sports Medicine has recommended
(aged 48) has osteoporosis by DXA, but family history is
that oral contraceptives be considered in amenorrheic athletes
otherwise negative. She has a BMI of 15 and Tanner 5 pubertal
only after age 16 and if BMD is decreasing despite adequate
development.
nutrition and weight gain (18).
Bisphosphonates have been used to treat osteogenesis
imperfecta (OI) and a variety of disorders causing secondary Management Questions
osteoporosis for more than 3 decades (17). Cochrane re- Would you order a DXA and why (or why not)?
What other tests would you order?
views have found available evidence to be insufficient to
What treatment(s) would you suggest?
support the routine use of bisphosphonates but compassion-
ate use is reasonable for those with fragility fractures. There Case 2. You are referred a 10-year-old nonambulatory boy with
is no consensus about the optimal agent, dose, and duration cerebral palsy and seizures (on Dilantin). He has no history of
of therapy (17). Pamidronate has been the most extensively fractures. His parents are requesting a DXA and treatment.

The Endocrine Society. Downloaded from press.endocrine.org by [${individualUser.displayName}] on 12 January 2017. at 12:19 For personal use only. No other uses without permission. . All rights reserved.
290 ENDO 2016 MEET-THE-PROFESSOR CLINICAL CASE MANAGEMENT

Management Questions low. If you do a DXA, the most informative region of


Would you order a DXA and why (or why not)? interest would be lateral distal femur. Immobilized youth are
If you order a DXA, what skeletal site(s) would you most likely to fracture in the lower extremity, and BMD at
scan and why? this site is more strongly correlated with fracture risk than
What other tests would you order? lumbar spine BMD. A complete evaluation would include
What treatment(s) would you suggest? review of overall nutrition, calcium, vitamin D intake, and
Case 3. You are asked to consult on a 9-year-old boy currently serum 25 OHD. Therapy would include optimizing nutrition
hospitalized for a flare of a multisystem inflammatory disease and physical therapy regimens.
involving heart, brain, and muscles. He has been treated with Case 3. Back pain in the setting of chronic inflammatory
high-dose glucocorticoids for 2 years. He has been complain- disease and glucocorticoid therapy raises the concern for
ing of back pain for the 4 weeks. His height is on 10th vertebral fracture. The first diagnostic step would be a lat-
percentile and BMI on 90th percentile. eral radiograph of thoracolumbar spine. This study is or-
dered and reveals several vertebral compression fractures
Management Questions and washed out bones. These findings establish the diag-
Would you order a DXA and why (or why not)? nosis of osteoporosis but a DXA could serve as baseline
What other tests would you order? before treatment. Any vertebrae with fractures should be
What treatment(s) would you suggest? excluded from analysis. In addition to general measures,
bisphosphonate therapy should be offered to potentially re-
Case 4. You see an apparently healthy 7-year-old boy for
duce bone pain and future fracture risk.
frequent fractures while rough housing with siblings at
Case 4. This history of recurrent fractures starting before age
home. At age 3, he fractured his right forearm; at age 4, his left
5 warrants evaluation. This could include a DXA of spine and
radius; at age 6, his right radius; and his right tibia last month.
whole body (if this can be done without sedation) and a lateral
He has no previous history of hospitalizations, chronic illness,
thoracolumbar spine x-ray to rule out vertebral fracture. Con-
or medications. He has no blue sclera, hyperextensibility, or
sider measuring serum 25 OHD, celiac screen, CBC, ESR,
abnormal dentition on examination.
IGF-1, TFTs, urine calcium to creatinine ratio, and possibly
test for osteogenesis imperfecta. This can be performed in
Management Questions several commercial labs with 35 mL of EDTA blood in
Would you order a DXA and why (or why not)? children.
What other tests would you order?
What treatment(s) would you suggest?
REFERENCES
1. Dent CE. Osteoporosis in childhood. Postgrad Med J. 1977;53:450 457.
DISCUSSION OF CASES 2. Rizzoli R, Bianchi ML, Garabedian M, McKay HA, Moreno LA. Maxi-
Case 1. This teen likely has exercise-associated secondary mizing bone mineral mass gain during growth for the prevention of
fractures in adolescents and the elderly. Bone. 2010;46:294-305.
amenorrhea (with perhaps an element of disordered eating). 3. Bonjour JP, Theintz G, Buchs B, Slosman D, Rizzoli R. Critical years and
Her foot fracture is likely a stress fracture, which is an overuse stages of puberty for spinal and femoral bone mass accumulation during
rather than fragility injury. The family history of osteoporosis adolescence. J Clin Endocrinol Metab. 1991;73:555-563.
4. Bailey DA, McKay HA, Mirwald RI, Crocker PR, Faulkner RAA. Six-
is limited to moms diagnosis of osteoporosis based on DXA year longitudinal study of the relationship of physical activity to bone
findings. An important question is how would DXA findings (if mineral accrual in growth children: The University of Saskatchewan Bone
ordered) change management? If bone density is low, will this Mineral Accrual Study. J Bone Miner Res. 1999;14: 1672-1679.
5. Winzenberg T, Shaw K, Fryer J, Jones G. Effects of calcium supplemen-
motivate the patient to work on nutrition? If normal, will she be tation on bone density in healthy children: meta-analysis of randomised
less likely to adhere to suggestions regarding activity and diet? controlled trials. Brit Med J. 2006;333:775.
Exercise-associated amenorrhea is a threat to bone health likely 6. Tan VPS, Macdonald HM, Kim S, et al. Influence of physical activity on
bone strength in children and adolescents: a systematic review and narra-
due to the combination of both low sex steroids and energy tive synthesis. J Bone Miner Res. 2014;29:2161-2181.
deficits. Other tests to consider include FSH (rule out ovarian 7. Carter DR, Bouxsein ML, Marcus R. New approaches for interpreting
insufficiency), prolactin, pregnancy test, celiac screen, com- projected bone densitometry data. J Bone Miner Res. 1992;7:137-145.
8. Gafni RI, Baron J. Overdiagnosis of osteoporosis in children due to
plete blood county, erythrocyte sedimentation rate, thyroid
misinterpretation of dual energy x-ray absorptiometry (DEXA). J Pediatr.
function tets, and 25OH vitamin D. The teen should be given 2004;144:253-257.
nutrition counseling and encouraged to reduce her activity and 9. Glorieux FH, Bishop N, Plotkin H, Chabot G, Lanoue G, Travers RT.
Cyclic administration of pamidronate in children with severe osteogenesis
increase her caloric intake. Sex steroid replacement with oral
imperfecta. N Engl J Med. 1998;339:947-952.
contraceptives is not recommended as a first line treatment. 10. Boyce AM, Gafni RI. Approach to the child with fractures. J Clin
Case 2. This child is at risk for bone fragility because of Endocrinol Metab. 2011;96:1943-1952.
immobilization and anticonvulsant therapy. In the absence 11. Goulding A, Jones IE, Taylor RW, et al. More broken bones: a 4-year
double cohort study of young girls with and without distal forearm frac-
of previous fracture, a DXA is not clearly indicated because tures. J Bone Miner Res. 2000;15:2011-1019.
the results will not determine therapy. Bone mass will be 12. Rauchenzauner M, Schmid A, Heinz-Erian P, et al. Sex- and age-

The Endocrine Society. Downloaded from press.endocrine.org by [${individualUser.displayName}] on 12 January 2017. at 12:19 For personal use only. No other uses without permission. . All rights reserved.
ENDO 2016 PEDIATRIC ENDOCRINOLOGY 291

specific reference curves for serum markers of bone turnover in healthy bone mineral content and areal bone mineral density according to
children from 2 months to 18 years. J Clin Endocrinol Metab. age and sex for Black and Non-Black children: Results of the Bone
2007;92:443-449. Mineral Density in Childhood Study. J Clin Endocrinol Metab.
13. Gordon CM, Leonard MB, Zemel BS. 2013 Pediatric Position Develop- 2011;96:3160-3169.
ment Conference: executive summary and reflections. J Clin Densitom. 17. Bachrach LK. Diagnosis and treatment of pediatric osteoporosis. Curr
2014;17:219-224. Opin Endocrinol Diabetes Obes. 2014;21:454-460.
18. Nattiv A, Loucks AB, Manore MM et al. American College of Sports
14. Zemel BS, Stallings VA, Leonard MB, et al. Revised pediatric reference
Medicine Position Stand: the female athlete triad. Med Sci Sport Exerc.
data for the lateral distal femur measured by hologic discovery/delphi
2007;39:1867-1882.
dual-energy x-ray absorptiometry. J Clin Densitom. 2009;12:207-218. 19. Ooi HL, Briody J, Biggin A, et al. Intravenous zoledronic acid given
15. Henderson RC, Berglund LM, May R, et al. The relationship between every 6 months in childhood osteoporosis. Horm Res Paediatr.
fractures and DXA measures of BMD in the distal femur of children and 2013;80:179-184.
adolescents with cerebral palsy or muscular dystrophy. J Bone Miner Res. 20. Stathopoulos IP, Liakou CG, Katsalira A, et al. The use of bisphosphonates in
2010;25:520-526. women prior to or during pregnancy and lactation. Hormones (Athens).
16. Zemel BS, Kalkwarf KJ, Gilsanz V, et al. Revised reference curves for 2011;10:280-291.

The Endocrine Society. Downloaded from press.endocrine.org by [${individualUser.displayName}] on 12 January 2017. at 12:19 For personal use only. No other uses without permission. . All rights reserved.
292 ENDO 2016 MEET-THE-PROFESSOR CLINICAL CASE MANAGEMENT

Diabetes Insipidus in Children

M50 Both making the diagnosis of CDI and establishing its etiol-
Presented, April 1 4, 2016 ogy present challenges for the practitioner.

2) BARRIERS TO OPTIMAL PRACTICE


Craig A. Alter, MD. Childrens Hospital of Philadelphia,
Challenges in the workup of a child with newly diagnosed
Perelman School of Medicine, University of Pennsylvania,
CDI include how frequently to obtain an magnetic resonance
Philadelphia, Pennsylvania 19104, E-mail: alterc@email.
imaging (MRI) of the brain as well as when is a biopsy
chop.edu
needed of a lesion in the pituitary region. Treatment can be
with intranasal desmopressin acetate, oral desmopressin ac-
INTRODUCTION etate, or a combination but choosing the appropriate therapy
Historical Perspective. Diabetes Insipidus can be a challenge.
The first descriptions of diabetes as a disease presenting with
polyuria and thirst can be traced to ancient times; however, it
3) LEARNING OBJECTIVES
was not until the 18th century that the distinction was made
As a result of participating in this session, learners will be
between diabetes mellitus and diabetes insipidus (1). In 1769,
able to:
the Scottish physician William Cullen (1710 1790) first called
1) Determine when a water deprivation is appropriate to
attention to diabetic (polyuric) urine that was insipid in taste
diagnose CDI.
and added the descriptive adjective mellitus to the disease
2) Develop a diagnostic approach to determining the cause
described by Thomas Willis in 1670s (2).
of the CDI.
Prior to the 20th century, it was generally believed that
3) Understand advantages and disadvantages of oral vs
diabetes insipidus was a renal disease. This theory changed in
intranasal desmopressin acetate therapy.
1912 due to clinical observations which suggested an associa-
tion between hypophysis and diabetes insipidus. In 1913, it was
determined that the extract of the posterior lobe of the pituitary 4) STRATEGIES FOR DIAGNOSIS, THERAPY,
was an effective treatment for diabetes insipidus. By the 1920s, AND/OR MANAGEMENT
the available evidence was conclusive enough to define diabe- When a child presents with excessive thirst and urination, or
tes insipidus as a disorder of the pituitary and label it a newly developed nocturia, the endocrinologist and/or primary
hypopituitary syndrome (2). It was not until the 1950s that it care practitioner first considers if the presentation is related to
was established that the posterior lobe hormones are secreted hyperglycemia. When diabetes mellitus has been ruled out,
in the hypothalamus. It was also during this time period that the diabetes insipidus must be considered (1, 2).
antidiuretic hormone was isolated and synthesized (1). These There are several possible ways to establish the diagnosis
significant discoveries paved the way for the current knowl- of diabetes insipidus in a child with overt polyuria, polydip-
edge and treatment of diabetes insipidus. sia, and nocturia. Fluid intake and output may be above
2L/meter2/d. A formal water deprivation may not be needed
REFERENCES as in some patients, diagnostic criteria are met without the
1. Lindholm J. Diabetes insipidus: historical aspects. Pituitary. 2004;7(1):33-38. study.
2. Eknoyan G. A History of diabetes insipidus: paving the road to internal The diagnosis of diabetes insipidus classically is made by a
water balance. Am J Kidney Dis. 2010;56(6):1175-1183. single serum and urine sample showing a serum sodium of
more than 300 mOsm/kg with urine osmolality less than 600
1) SIGNIFICANCE OF THE CLINICAL mOsm/kg. A water deprivation can be avoided if a first morn-
PROBLEM ing serum and urine show these findings. The fasting study is
Central diabetes insipidus (CDI) in children is an uncommon followed by administering desmopressin acetate (0.1 0.4 mcg
condition, which presents challenges both in establishing the by sc injection). Long-standing primary polydipsia will blunt
diagnosis as well as determining the etiology. In some children, the ability of the kidneys to reach maximal concentrating abil-
the diagnosis of CDI can sometimes be easy to establish, and in ity with desmopressin acetate.
others may require a water deprivation study. After the diag- In addition, at the session I will discuss whether having an
nosis of CDI is established, it is imperative to consider the MRI showing an absent posterior pituitary T1-bright spot with
etiology. Causes of CDI include hypophysitis, Langerhan cell or without a thickened pituitary infundibular stalk has value in
histiocytosis, central nervous system (CNS) germinoma, vascu- supporting a diagnosis of diabetes insipidus (3, 4).
lar abnormalities, familial (genetic), post-CNS surgery, and Once the diagnosis is established of CDI, it is imperative to
from CNS malformations. investigate the potential CNS cause.

The Endocrine Society. Downloaded from press.endocrine.org by [${individualUser.displayName}] on 12 January 2017. at 12:19 For personal use only. No other uses without permission. . All rights reserved.
ENDO 2016 PEDIATRIC ENDOCRINOLOGY 293

In 2000, Maghnie et al did a retrospective analysis of 79 Answer: A


children with a diagnosis of CDI. About half the children had 2) A child with polyuria undergoes a water deprivation test
idiopathic CDI, presumed to be hypophysitis (3). A thickened and the results suggest CDI. On examination, he has a
stalk occurred in 37% implied an infiltrative disorder. In their papular rash. Which of the following diagnoses is sug-
study, still idiopathic CDI accounted for 62%, histiocytosis in gested by these finding?
17%, and germinoma in 17% of cases. Germinoma represent a) Congenital diabetes insipidus, X-linked
7.8% of pediatric brain tumors and are often radiosensitive for b) Congenital diabetes insipidus, autosomal dominant
successful therapy, although early diagnosis is important for c) Langerhan cell histiocytosis
treatment success. d) Germinoma
A more recent study in August, 2015 by Werny et al at
Seattle Childrens Hospital described a retrospective study Answer: C, Langerhan cell histiocytosis
of 147 children with diabetes insipidus (5). They found a 3) A 14-year-old child with known diabetes insipidus due to
lower incidence of 12% idiopathic and also found the im- hypophysitis is on oral desmopressin acetate, 600 mcg
portance of assessing the thickness of the pituitary in- daily. Poor growth is noted, and she is determined to
fundibular stalk. have GH deficiency. She is poorly compliant with the GH
Findings from these studies and others will be presented at therapy although seems to take well her desmopressin
the session as well as a discussion to help explain the differ- acetate.
ence in their findings as compared with that of Maghnie et al in She reports that her desmopressin acetate requirement has de-
2000 (6 10). creased to just 200 mcg a day with standard daily breakthrough.
In terms of therapy for diabetes insipidus, the choice of Which of the follow most likely accounts for this finding?
desmopressin acetate oral vs intranasal should be individual- a) Poor compliance of the GH
ized for each family. A discussion of the advantages of each b) Adrenal insufficiency
including the ease of administration will be reviewed. c) Hypocalcemia
d) Central hypothyroidism

5) MAIN CONCLUSIONS:
Answer: B adrenal insufficiency
1) The diagnosis CDI may not require a full water
deprivation study.
2) If the pituitary stalk is thickened, frequent MRI with 7) DISCUSSION OF CASES AND ANSWERS
contrast will need to be performed such as every 3 Question 1
The absence of the posterior pituitary bright spot is seen in
months.
approximately 90% of children with CDI, but its presence does
3) In choosing desmopressin acetate therapy, one may
not point to a specific etiology of the CDI. Anyone with CDI
need to consider various options including oral
needs to have an investigation for anterior pituitary hormone
desmopressin acetate, intranasal desmopressin acetate,
deficiencies. Although some studies suggest an association
or even a combination.
with germinoma and anterior pituitary hormone deficiencies,
more evidence suggest that there is an association of a thick-
6) CASES: ened pituitary stalk and malignancy as well as a thickened stalk
A 14-year-old boy presents with polyuria and polydipsia of 6 and anterior pituitary hormone deficiencies, however, not an
months of duration. After a water restriction, he is concluded to association between malignancy and anterior pituitary hormone
have CDI. An MRI of the pituitary shows a thickened stalk and deficiencies in a child with CDI.
an absent posterior pituitary bright spot. Additional studies to search for the potential germinoma
include an human chorionic gonadotropin (HCG) and an
Questions: -fetoprotein.
1) What is the implication of an absent posterior pituitary
bright spot? Question 2
a) It suggests CDI but does not give information as to The history should include looking for any CNS risk factors
the specific cause such as headaches or visual complaints. On examination, he
b) It suggests CDI as well as multiple pituitary hormone had an unusual papular popular erythematous rash on his
deficiencies groin and wrists. Dermatology consulted and determined by
c) It suggests CDI and increases the risk that a CNS biopsy that he had Langerhan cell histiocytosis. The water
tumor is present deprivation study was obviated by the initial fasting labs
d) It suggests CDI and implies frequent MRI will be which showed hyperosmolality and urine hypoosmolality. A
needed serum osmolality should be measured.

The Endocrine Society. Downloaded from press.endocrine.org by [${individualUser.displayName}] on 12 January 2017. at 12:19 For personal use only. No other uses without permission. . All rights reserved.
294 ENDO 2016 MEET-THE-PROFESSOR CLINICAL CASE MANAGEMENT

Langerhan cell histiocytosis can present with bone findings, 4. Alter CA, Bilaniuk LT. Utility of magnetic resonance imaging in the
rash, or with signs of CDI. In any child with potential DI, a evaluation of the child with central diabetes insipidus. J Pediatr
Endocrinol. 2002;15(suppl 2):681-687.
careful examination of the skin is therefore required. 5. Werny D, Elfers C, Perez FA, Pihoker C, Roth CL. Pediatric central DI:
brain malformations are common and few patients have idiopathic disease.
Question 3 J Clin Endocrinol Metab. 2015;100(8):3074-3080.
Any child with hypophysitis can develop additional pituitary 6. Kilday JP, Laughlin S, Urbach S, Bouffet E, Bartels U. Diabetes insipidus
in pediatric germinomas of the suprasellar region: characteristic features
hormone deficiencies. She already has confirmed GH defi-
and significance of the pituitary bright spot. J Neuro Oncol.
ciency, hence monitoring for central hypothyroidism and cen- 2015;121(1):167-175.
tral adrenal insufficiency are needed at least yearly. Because 7. Di Iorgi N, Allegri AE, Napoli F, et al. Central diabetes insipidus in
cortisol promotes free water clearance, someone with DI who children and young adults: etiological diagnosis and long-term outcome of
develops adrenal insufficiency may experience a decline in idiopathic cases. J Clin Endocrinol Metab. 2014;99(4):1264-1272.
8. Maghnie M, Arico M, Villa A, Genovese E, Beluffi G, Severi F. MR of the
desmopressin acetate requirements until glucocorticoid re-
hypothalamic-pituitary axis in Langerhans cell histiocytosis. Amer
placement is initiated. J Neuroradiol. 1992;13(5):1365-1271.
9. Maghnie M, Villa A, Arico M, et al. Correlation between magnetic
REFERENCES resonance imaging of posterior pituitary and neurohypophyseal func-
1. Ghirardello S, Garre ML, Rossi A, Maghnie M. The diagnosis of tion in children with diabetes insipidus. J Clin Endocrinol Metab.
children with central diabetes insipidus. J Pediatr Endocrinol. 1992;74(4):795-800.
2007;20(3):359-375. 10. Perrotta S, Di Iorgi N, Ragione FD, et al. Early-onset central diabetes
2. Maghnie M. Diabetes insipidus. Horm Res. 2003;59(suppl 1):42-54. insipidus is associated with de novo arginine vasopressin-neurophysin
3. Maghnie M, Cosi G, Genovese E, et al. Central diabetes insipidus in II or Wolfram syndrome 1 gene mutations. European J Endocrinol.
children and young adults. N Engl J Med. 2000;343:998-1007. 1015;172(4):461-472.

The Endocrine Society. Downloaded from press.endocrine.org by [${individualUser.displayName}] on 12 January 2017. at 12:19 For personal use only. No other uses without permission. . All rights reserved.
REPRODUCTIVE
ENDOCRINOLOGY

The Endocrine Society. Downloaded from press.endocrine.org by [${individualUser.displayName}] on 12 January 2017. at 12:19 For personal use only. No other uses without permission. . All rights reserved.
296 ENDO 2016 MEET-THE-PROFESSOR CLINICAL CASE MANAGEMENT

State-of-the-Art: Use of Hormones in Transgender Individuals

CMF5 More widely recognized area of endocrine practice;


Presented, April 1 4, 2016 however, not fully supported by the endocrine
establishment.
Lack of structured curriculum in medical training,
Benjamin Boh, DO, MS and Joshua D. Safer, MD, including in medical school, residency, and fellowship.
FACP. Department of Endocrinology, Diabetes and Historically transgender care not covered by health
Metabolism, Dartmouth-Hitchcock Medical insurance.
Center/Dartmouth Medical School, Lebanon, New Paucity of research regarding long-term health outcome
Hampshire 03756, E-mail: benjamin.boh@hitchcock.org; data, particularly in the United States.
and Department of Medicine, Section of Endocrinology, General lack of awareness regarding treatment of
Boston Medical Center/Boston University School of transgender adolescents.
Medicine, Boston, Massachusetts 02118, E-mail:
jsafer@bu.edu
LEARNING OBJECTIVES
As a results or participating in this session, learners should be
INTRODUCTION able to:
Historical Overview
Understand the organic, durable nature of gender identity
Transgender people are individuals whose gender identity does
and the implication for treatment strategy.
not match their sexual anatomy. Some transgender individuals
Understand the role of the endocrinologist in providing
suffer gender dysphoria, which the Diagnostic and Statistical
well-informed, compassionate care for transgender
Manual of Mental Disorders 5th Edition (1) defines as distress
individuals.
caused by that incongruence. Transgender individuals are a
Describe the treatment strategies and potential regimens
diverse group and not all individuals seek medical transition.
for both transgender adolescents and adults.
However, in endocrine practice most patients the endocrinolo-
Understand the importance of fertility preservation for
gist will encounter are seeking hormonal treatment.
transgender individuals.
Sex steroids were isolated and characterized in the 1920s
Discuss strategies to minimize complications of cross-
and early 1930s, with clinical use starting in the 1940s. A
sex hormone administration.
physician by the name of Michael Dillon was the first docu-
mented case of a natal female taking testosterone (1939) and
transitioning successfully to live as a male. In the 1940s, there STRATEGIES FOR DIAGNOSIS, THERAPY,
were very few physicians prescribing hormone therapy (HT) for AND/OR MANAGEMENT
transgender individuals. In contrast with current strategy, which is Diagnosis of transgender identity is straightforward for most
informed by recognition of the durable biological nature of adults. Most individuals who recognize that there is a choice
gender identity, treatment at that time was centered around will be able to determine a gender identity without much doubt.
conversion therapy, the ineffective attempt to change an indi- Whether and to what degree a given individual with a
viduals gender identity and/or sexual orientation. transgender identity wants to address the congruence is a very
An American endocrinologist by the name of Harry Benja- personal decision relating to many other factors in life.
min was a leading advocate for hormone use in individuals To avoid a rare instance of a psychiatric condition con-
with gender dysphoria and gender incongruence. In 1966 he founding the situation to such a degree that sexual identity is
published The Transsexual Phenomenon, which documented not clear, a mental health provider is solicited to confirm the
nearly 20 years of experience working with transgender indi- absence of a confounder. A transgender individual should be
viduals. In his work with large numbers of transgender pa- under the care of individuals with expertise in the relevant
tients, he identified that conversion therapy was not only mental health, endocrinology, and primary care.
ineffective but also harmful. Alternatively, he reported that Because some children with gender incongruence will go on
cross-sex HT relieved gender dysphoria. In the 1970s Dr Ben- to live as adults who are not transgender, diagnostic accuracy is
jamin helped form an international organization for providers paramount. Diagnosis is undertaken with a multidisciplinary
of transgender care, now known as The World Professional team that includes expertise in mental health, endocrinology,
Association of Transgender Health (WPATH). and primary care. The issue is problematic because individuals
who wish to avail themselves of transgender treatment will find
BARRIERS TO CLINICAL PRACTICE it easier at a younger age; prior to the need to reverse opposite
Lack of knowledge about transgender individuals and sex characteristics developed in puberty. A paradigm to ad-
health concerns. dress the contradiction is to use GnRH analogs when puberty

The Endocrine Society. Downloaded from press.endocrine.org by [${individualUser.displayName}] on 12 January 2017. at 12:19 For personal use only. No other uses without permission. . All rights reserved.
ENDO 2016 REPRODUCTIVE ENDOCRINOLOGY 297

begins to delay puberty until the long-term treatment plan can TABLE 1. Pubertal Suppression for Transgender
be established with confidence. Adolescents
Drug Dose
Treatment Leoprolide depot, monthly formulation 15 mg im every mo
Hormone treatment of transgender individuals is straightfor- Leuprolide depot, 3-mo formulation 30 mg im every 3 mo
ward following conventional hormone paradigms with the an- Histrelin implant 50 mg (lasting 24 y)
ticipated concerns and effects that are seen when using the
same hormones for other purposes. Adapted from the 2009 Endocrine Society Guidelines (4).
Typically, for transgender men (female to male), hormone
treatment consists of testosterone to bring the serum testoster-
one from the female range to the male range. The doses Usual monitoring includes serum testosterone (to
required are similar to those used for treatment of hypogonadal determine success of therapy), hematocrit, and
males in general. Small case series demonstrate stable testos- occasional lipid profile.
terone levels on sc injection at doses of 50-100 mg per week, Malignancy screening must include all body parts present
which were well tolerated by patients. regardless of whether they are associated with one sex or
Typically, for transgender women (male to female), hor- another (eg, do not forget Pap test and mammogram in
mone treatment consists of an antiandrogen to lower testoster- transgender men with cervix, or residual breast tissue,
one levels (if testis are present) and estrogen supplementation respectively).
(which may also act to suppress the endogenous axis and
decrease testosterone levels). Transgender Women
The antiandrogen of choice in the United States has been Antiandrogen therapy of any sort (transgender drugs,
spironolactone because of its low cost and its long-term safety oral birth control [for birth control, acne, or hirsutism])
profile arising from its 50-year history as a potassium-sparing may result in decreased libido.
diuretic to treat hypertension. Higher doses are used than are Usual monitoring includes serum testosterone (to
required for blood pressure control (in divided doses if needed determine success of therapy), estrogen level (estradiol),
for patient to tolerate). GnRH agonists can seem attractive prolactin, potassium, and lipid profile.
because their mechanism of action seems straightforward. The Malignancy screening should include all body parts
progestin, cyproterone acetate, has been used in Europe for (including remaining prostate tissue, there is insufficient
many years. However, it carries a risk of hepatotoxicity and is data to recommend routine mammography for all
currently not approved for use in the United States. transgender women).
Multiple estrogen options exist. For ease of use (as widely Tables 1, 2, 3, 4, 5, 6, and 7 are updates adapted from the 2009
available oral agents), the two most popular are premarin Endocrine Society Guidelines.
(conjugated estrogens obtained from pregnant mares urine)
and estradiol (a pure laboratory-produced product). The latter
CASES
is cheapest and does not involve captive animals; thus, generic Case 1
estradiol is preferred by these authors. Anticipate doses double A 16-year-old patient with female sexual anatomy presents for endo-
to quadruple those for postmenopausal hormone replacement crine consultation accompanied by her parents. Parents are concerned
therapy (ie, 1-2 mg). The doses sometimes need to be higher by recent deepening of voice and worsening of acne. The adolescent
yet for individuals with testis present (up to 4 or 8 mg/d) to has been dressing in male attire for several years and prefers
suppress testosterone into the female range. There is some stereotypical male activities. Her academic performance has been
belief that thrombosis risk might be mitigated with lower doses deteriorating and she is withdrawing socially and at home.
or the use of parenteral/topical products to avoid the first-pass
metabolism and stimulation of clotting factors in the liver. TABLE 2. Masculinizing Hormones for Induction of
Puberty Transgender Male Adolescents
Concerns Hormone Dose
Transgender Men
Testosterone cypionate or enanthate 20100 mg sc or
The biggest concerns for testosterone therapy are an
im every wk
increase in hematocrit, increased blood pressure, and a
Topical testosterone gel, 1.62% 20.25 mg/d
shift in lipid profile. Long-term databases are needed to Topical testosterone gel, 2% 1020 mg/d
assess the risk of coronary disease in transgender men.
Androgen therapy is also associated with increased sleep Adapted from the 2009 Endocrine Society Guidelines (4).
apnea.

The Endocrine Society. Downloaded from press.endocrine.org by [${individualUser.displayName}] on 12 January 2017. at 12:19 For personal use only. No other uses without permission. . All rights reserved.
298 ENDO 2016 MEET-THE-PROFESSOR CLINICAL CASE MANAGEMENT

TABLE 3. Follow-Up Protocol During Suppression of TABLE 5. Feminizing Hormones for Transgender
Puberty Women
Every 3 mo Hormone Dose
Anthro pometry: height, weight, sitting height, Tanner stages Oral estradiol 210 mg/d
Laboratory: LH, FSH, estradiol/testosterone Estradiol patch 0.10.4 mg twice per wk
Every year Parental estradiol (valerate) 510 mg sc or im every wk
Laboratory: renal and liver function, lipids, glucose, insulin, Anti-androgen: spironolactone 100200 mg/d
glycosylated hemoglobin Gonadal suppression: leuprolide 3.75 mg im once/mo
Bone density using dual-energy x-ray absorptiometry depot
Bone age on x-ray of the left hand
Adapted from the 2009 Endocrine Society Guidelines (4).
Adapted from the 2009 Endocrine Society Guidelines (4).

role and he desires to continue HT. His current hormone


How do you address this patient and what are you goals for regimen consists of testosterone cypionate, 200 mg im every 2
the first office visit? weeks. He complains of pruritus during warm showers and
frequently feels flushed. Laboratory results reveal a hematocrit
of 56%.
Discussion What treatment related condition has he experienced and
The primary concern in this case is illicit testosterone use, what changes do you recommend?
which can occur in the transgender population and is often
motivated by the arduous process of qualifying for HT or lack
of access to providers. However, the first priority is the need to Discussion
develop a good rapport with the patient, which is particularly Testosterone stimulates erythropoiesis. Androgen therapy is
important when working with the adolescent population. We likely to raise hematocrit but usually within the normal range.
recommend starting every new patient encounter with an in- However, androgen therapy can unmask polycythemia and can
quiry into preferred pronouns and names. Once you have a exacerbate concern in patients with borderline high hematocrit
built a rapport, you can inquire into previous or current testos- levels. Although for most hypogonadal males in general and
terone use. Polycystic ovarian syndrome is an alternative ex- transgender males specifically, 50 100 mg of injectable testos-
planation for virilization. Less likely possibilities include viril-
izing tumors, Cushings syndrome, and late-onset congenital TABLE 6. Monitoring of Transgender Women on HT
adrenal hyperplasia. In this case, the patient admitted to surrep- Evaluate patient every 3 months in the first year and then 12
titious use of injectable testosterone. The risks of unmonitored times/y afterward to monitor for appropriate signs of
testosterone use were explained and he was referred to a mental feminization and for development of adverse reactions.
health professional for help in establishing a multidisciplinary Measure serum testosterone and estradiol every 3 mo.
approach to the diagnosis and treatment. Target serum testosterone levels should be 100 ng/dL.
Serum estradiol should not exceed the peak physiological
Case 2 range for young healthy females, with ideal levels 200
A 26-year-old transgender male patient has completed 1 year pg/mL.
of testosterone therapy with marked improvement in his well- Doses of estrogen should be adjusted according to the
serum levels of estradiol.
being and social function. He is successfully living in the male
For individuals on spironolactone, potassium should be
monitored every 3 mo initially in the first year, or at least
with dose changes.
TABLE 4. Masculinizing Hormones for Transgender Routine cancer screening is recommended as in
Men nontransgender individuals (breasts, colon, prostate).
Drug Dose Consider BMD testing at baseline if risk factors for
osteoporotic fracture are present (eg, previous fracture,
Testosterone cypionate or enanthate 50100 mg sc or im
family history, glucocorticoid use, prolonged
injection every wk
hypogonadism). In individuals at low risk, screening for
Testosterone gel, 1.65% 40.581 mg/d osteoporosis should be conducted at age 60 y and in
Testosterone gel, 2% 4070 mg/d those who are not compliant with HT.
Testosterone undecanoate 1000 mg every 1012 wk
Abbreviation: BMD, bone mineral density.
Adapted from the 2009 Endocrine Society Guidelines (4). Adapted from the 2009 Endocrine Society Guidelines (4).

The Endocrine Society. Downloaded from press.endocrine.org by [${individualUser.displayName}] on 12 January 2017. at 12:19 For personal use only. No other uses without permission. . All rights reserved.
ENDO 2016 REPRODUCTIVE ENDOCRINOLOGY 299

TABLE 7. Monitoring of Transgender Men on HT Would you prescribe estrogen to this patient? What would
Evaluate patient every 3 mo in the first year and then 12 times/y be your reasoning?
to monitor for appropriate signs of virilization and for
development of adverse reactions.
Discussion
Measure serum testosterone every 3 mo until levels are in the
normal physiological male range:
A mainstay of transgender hormone treatment is estrogen for
For testosterone enanthate/cypionate injections, some measure
which there is a clear thrombosis risk for ethinyl estradiol and
peaks and troughs and some measure midway between a likely risk for the less-thrombogenic 17-estradiol. Although
injections. any other thrombosis risk, like immobility, might have been
For parenteral testosterone undecanoate, testosterone should be felt to be an absolute contraindication to estrogen therapy in the
measured just before the next injection. past, greater use has made it apparent that the risk may be
For transdermal testosterone, the testosterone level can be sufficiently modest that a conservative approach could include
measured at any time after 1 wk. treatment. There has been at least one proposal for treatment
Measure estradiol levels during the first 6 months of testosterone along with parallel prophylaxis (such as aspirin or low-dose
treatment or as needed.
warfarin), although there are no data to demonstrate effective-
Measure hematocrit (or hemoglobin) at baseline and every 3 mo
for the first year; then 12 times/y. Monitor weight, blood ness of such an approach. The patient was quite insistent and
pressure, lipids, fasting blood sugar, and/or hemoglobin A1c demonstrated good comprehension of the relative risk. Low-
at regular visits as needed. dose estradiol was given in addition to spironolactone. The
Consider BMD testing at baseline if risk factors for osteoporotic patient was quite pleased with the physical changes and came
fracture are present. In individuals at low risk, screening for out as transgender. She has been taking a cross-sex regimen for
osteoporosis might be conducted at age 60 y and in those
several years without incident.
who are not compliant with HT.
If cervical tissue is present, pap smears as indicated for other
population groups. Case 4
If breast tissue is present, mammograms as indicated for other A 38-year-old natal female with a male gender identity
populations. (transgender male) has completed 18 years of testosterone
therapy and is successfully living as a male. He has a female
Abbreviation: BMD, bone mineral density. partner of 1 year and they come to you to discuss future
Adapted from the 2009 Endocrine Society Guidelines (4). fertility options. What are their options for fertility? What
are the options for transgender individuals seeking fertility
or fertility preservation?
terone administered every week should be safe, monitoring of
hematocrit is essential. In patients with erythrocytosis, tobacco Discussion
use and sleep apnea should be excluded as contributors. The Historically, fertility preservation in the transgender patient
elevated hematocrit can be mitigated with lower doses of testos- has not been viewed as a high-priority issue. However,
terone as long as levels appropriate for bone health and cessation given that individuals transition at younger ages the need for
of menses are maintained. Whether topical testosterone is genu- fertility planning and preservation has never been greater.
inely less prone to erythropoiesis or is simply less efficient with Fertility plans and the possibility of permanent infertility
resulting lower testosterone levels remains unclear. When the must be discussed prior to beginning HT and again prior to
testosterone regimen is modest but the hematocrit levels remain reproductive organ removal. The patient in the above case had
high, hematological evaluation may be considered to exclude the not yet undergone hysterectomy/oophorectomy. In a case series
possibility of an underlying primary polycythemia. of 41 transgender males who underwent total hysterectomy/
oophorectomy, the uteri of transgender men were significantly
Case 3 smaller than cisgender women. The effect of long-term testos-
A 50-year-old individual with male sexual anatomy presents terone on the adenexa and uterus are unknown and therefore
reporting being a transgender female but being denied hormone one might consider periodic transvaginal ultrasound and Pap
treatment because it is too dangerous. The patient had not testing, at least until fertility planning is finalized. Transvaginal
identified a female sex identity prior to this year but now states ultrasound can be performed 57 years after testosterone initia-
that she has always felt female and recent publicity helped her tion and periodically thereafter, based on risk factors. Patients
recognize her own incongruence. Unfortunately, she was in a who are not able to adhere to routine gynecologic screening
severe accident 1 year ago from which she has been left nearly should be encouraged to consider total hysterectomy and
quadriplegic with only modest use of her extremities (sufficient oophorectomy.
to manipulate her automatic wheelchair and to transfer on her The patient and his partner in this case decided upon oocyte
own). Although hospitalized, she had a deep venous thrombo- retrieval and in vitro fertilization of the transgender partners
sis diagnosed and treated with warfarin. oocytes with donor sperm. Menses resumed 3 months after

The Endocrine Society. Downloaded from press.endocrine.org by [${individualUser.displayName}] on 12 January 2017. at 12:19 For personal use only. No other uses without permission. . All rights reserved.
300 ENDO 2016 MEET-THE-PROFESSOR CLINICAL CASE MANAGEMENT

testosterone was discontinued. Anti-Mullerian hormone level tration: A clinical case series of subcutaneous testosterone use in twenty-
two transgender males. Program of the 97th Annual Meeting of the
returned at 3.5 and antral follicle count was high at 18. Gonad-
Endocrine Society, San Diego, CA, March 7-9, 2015.
otropins were not elevated and therefore the plan was to begin 3. Cohen-Kettenis PT, van Goozen SH. Pubertal delay as an aid in diagnosis
a regimen of exogenous gonadotropins for follicle stimulation and treatment of a transsexual adolescent. Eur Child Adolesc Psychiatry.
1998;7:246-248.
on day 2 of his menstrual cycle. Ultimately, the female partner 4. Cohen-Kettenis PT, Schagen SE, Steensma TD, de Vries AL, Delemarre-
plans to be the gestational carrier for her partners embryo(s). van de Waal HA. Puberty suppression in a gender-dysphoric adolescent: A
Other fertility options for transgender men consist of oocyte 22-year follow-up. Arch Sex Behav. 2011;40(4):843-847.
5. Hembree WC, Cohen-Kettenis P, Delemarre-van de Waal HA, et al.
cryopreservation, sperm donation, adoption, pregnancy, and Endocrine treatment of transsexual persons: An Endocrine Society Clinical
ovarian tissue cryopreservation. Ovarian cryopreservation at Practice Guideline. J Clin Endocrinol Metab. 2009;94:3132-3154.
this time is an experimental process. In vitro fertilization can 6. Mahesh VB. Hirsutism, virilism, polycystic ovarian disease, and the
steroid-gonadotropin-feedback system: A career retrospective. Am J
be accompanied by preimplantation genetic screening, thereby Physiol Endocrinol Metab. 2012;302(1):E4-E18.
reducing the risk of aneuploidy, particularly in patients above 7. Murad MH, Elamin MB, Garcia MZ, et al. Hormonal therapy and sex
the age of 30 years. reassignment: A systematic review and meta-analysis of quality of life and
psychosocial outcomes. Clin Endocrinol (Oxf). 2010;72(2):214-231.
Fertility preservation in transgender women is a simpler 8. OHanlan KA, Dibble SL, Young-Spint M. Total laparoscopic hysterectomy for
process, consisting of sperm cryopreservation prior to hormone female-to-male transsexuals. Obstet Gynecol. 2007;110(5):1096-1101.
initiation. Cryopreserved sperm can be used for intrauterine or 9. Saraswat A, Weinand JD, Safer JD. Evidence supporting the biologic
nature of gender identity. Endocr Pract. 2015;21(2):199-204.
intracervical insemination of a female partner or a gestational 10. Wallien MS, Cohen-Kettenis PT. Psychosexual outcome of gender-
carrier. dysphoric children. J Am Acad Child Adolesc Psychiatry. 2008;47(12):
1413-1423.
11. Weinand JD, Safer JD. Hormone therapy in transgender adults is safe with
REFERENCES provider supervision; A review of hormone therapy sequelae for
1. Diagnostic and Statistical Manual of Mental Disorders, Fifth Edition. transgender individuals. J Clin Transl Endocr. 2015;2:55-60.
Washington DC: American Psychiatric Association, 2013. 12. Wren B. Early physical intervention for young people with atypical gender
2. Boh B, Turco JH, and Comi RJ. New and improved testosterone adminis- identity development. Clin Child Psychol Psychiatry. 2000;5:220-231.

The Endocrine Society. Downloaded from press.endocrine.org by [${individualUser.displayName}] on 12 January 2017. at 12:19 For personal use only. No other uses without permission. . All rights reserved.
ENDO 2016 REPRODUCTIVE ENDOCRINOLOGY 301

Female Sexual Dysfunction: Do Hormones Help?

M07 vaginal estrogen therapy (3). In the United States, from the age
Presented, April 1 4, 2016 of 18 years, the prevalence of any form of FSD associated with
distress is in the order of 10 15% (4). Oddly enough, women
continue to be sexually active even when they find sexual
Susan R. Davis, MBBS, FRACP, PhD. School of Public activity dissatisfying and despite increasing dyspareunia after
Health and Preventive Medicine, Monash University, menopause (5). This is often to protect the relationship and
Melbourne, Victoria 3004, Australia, E-mail: domestic harmony.
susan.davis@monash.edu

BARRIERS TO OPTIMAL PRACTICE


INTRODUCTION Barriers to the best care of women with FSD in the context of
Historical Overview endocrinology include:
Hysteria, an affliction of women, has been reported since the Among healthcare providers:
fourth century BC. Over centuries the recommended treatment
was intercourse, or for unmarried women, vigorous vulval Not understanding what constitutes sexual function and
massage. The prescription of this remedy has been documented dysfunction
across centuries. In the 17th century, the celebrated Dutch Not recognizing the effect of endocrine conditions on

physician Pieter van Foreest recommended genital massage, by female sexual function
a physician or midwife, until the afflicted woman be aroused to Not accepting that all women have a right to sexual

paroxysm, to alleviate the hysterical illness. Science came to pleasure


the fore when Alfred Kinsey published Sexual Behavior in the Ageism, with respect to this issue, is widespread among

Human Female in 1953, in which the anatomy, physiology, and clinicians


psychology of female sexuality was discussed. The role of Not feeling comfortable to talk to female patients about

hormones and female sexual function has only received atten- sexual function
tion in recent years. In a paper titled, Hormone therapy for Not knowing what management options are available

sexual dysfunction. The only true aphrodisiac, Karpas and Many women report having to speak to several doctors

Greenblatt (1) wrote, Human sexual behavior is extremely before they feel one will listen to their concerns.
complex indeed. The authors think the time is ripe for an Among women:
assessment of how hormones modify sexual responsiveness. In
Being embarrassed to broach sexual issues with their

particular, they hope this article will help to dispel the notion
endocrinologist
current in many circles that hormonal therapy is of little value.
Poor understanding of sexual function
Nonetheless, the understanding of the role of hormones in
Not knowing that management options may be available
female sexual dysfunction (FSD) is still evolving.
Women feel that there is a stigma to wanting to enjoy sex, and
it can take substantial courage for a woman to talk to her
SIGNIFICANCE OF THE CLINICAL PROBLEM specialist about her sexual concerns. Standard sex education
The World Health Organization (WHO) describes sexual perpetuates this problem: standard sex education teaches about
health as a state of physical, emotional, mental and social ovulation and contraception, but the clitoris, a vital female
well-being related to sexuality; it is not merely the absence of sexual organ, is rarely (if ever) mentioned and certainly absent
disease, dysfunction or infirmity. The WHO also reminds us from most teaching diagrams.
of important items:
The focus of this activity is on the endocrine aspects of FSD,
Sexuality is experienced and expressed in thoughts, fantasies, not the psychosocial issues.
desires, beliefs, attitudes, values, behaviors, practices, roles and Various efforts have been made to classify FSD, including
relationships. Although sexuality can include all of these di- the classifications in the Diagnostic Statistical Manual of Men-
mensions, not all of them are always experienced or expressed. tal Disorders (DSM)-IV (6) and subsequently, the DSM-V (7).
FSD is present across the adult life span and across cultures. These are not necessarily helpful for clinical practice. It is
The most commonly reported sexual problems in women in- universally accepted that for a sexual problem to be a disor-
clude loss of desire, lack of pleasure, and global dissatisfaction. der the affected woman needs to be concerned about the
Dyspareunia is highly prevalent and undertreated in postmeno- problem. The term distress is often used by US researchers in
pausal women (2). In a recent study of Australian women age this context, however many women feel that the term distress is
6579 years, one of three partnered women reported dyspareu- somewhat excessive. The DSM-IV classifications include
nia in the preceding month, yet less than 7% were using hypoactive sexual desire disorder (HSDD; low desire with

The Endocrine Society. Downloaded from press.endocrine.org by [${individualUser.displayName}] on 12 January 2017. at 12:19 For personal use only. No other uses without permission. . All rights reserved.
302 ENDO 2016 MEET-THE-PROFESSOR CLINICAL CASE MANAGEMENT

distress), arousal disorders, orgasm disorders, and sexual pain Estrogen


disorders. There is substantial overlap and interaction between Available data do not support estrogen deficiency as a leading
these. Furthermore, sexual function is not static such that a cause of FSD. However, estrogen deficiency causes VVA that
woman may have primarily lowered desire at one time and results in dyspareunia and avoidance conditioning. Unlike
poor arousal at another. The more recent DSM-V has combined other menopausal symptoms that diminish with time, VVA
desire and arousal as a desire-arousal disorder. This new clas- worsens with time since menopause. HSDD is more prevalent
sification is based on selected expert opinion, not quantitative following surgical menopause than natural menopause.
data. It is also highly proscriptive, requiring at least four major
symptoms (an arbitrary number) and mostly for at least 75% of
Testosterone
the time (this percentage threshold, based on expert opinion,
Observational studies in premenopausal and postmenopausal
and also arbitrary). It also refers to receptivity to a partner, but
women provide evidence that low T is associated with lower
not every woman has a partner. Similar metrics have been
self-reported sexual function (10). There is no cut-off level for
applied to orgasmic disorder in the DSM-V. There is no vali-
dated questionnaire for desire-arousal disorder; therefore, there any androgen that predicts FSD.
is no research tool that can be applied to this diagnosis. The
DSM-V classification of Genito-Pelvic Pain/Penetration Disor- Other Hormones
der describes fear and pain related to intercourse, but allows for Although implicated as having a role in female sexual function,
no insight into cause. Ongoing practice and research in this there is no evidence to support the use of oxytocin for the
area continues to adhere to the DSM-IV classifications. treatment of FSD. Similarly, evidence for a therapeutic role for
progesterone is also lacking.
LEARNING OBJECTIVES
As a result of participating in this session, learners should be When to Consider FSD in Endocrine Practice
able to: Endocrinologists should be alert to conditions associated with
Understand the range of sexual difficulties that affect diminished androgen production in women. The most common
women cause of low androgen production is the physiological decline
Feel more confident about talking to your female with age, which commences in the mid-reproductive years
patients about their sexual wellbeing (11). Other common and important causes include surgical
Ensure that appropriate postmenopausal patients are menopause, premature ovarian insufficiency, hypopituitarism
treated for vulvo-vaginal atrophy (VVA)
causing hypogonadism, and adrenal insufficiency. Polycystic
Be ready to appropriately refer or treat women with
ovary syndrome, obesity, and the metabolic syndrome may be
sexual dysfunction
associated with FSD but data are limited. There is a strong
association between diabetes mellitus and FSD in women (12).
STRATEGIES FOR DIAGNOSIS, THERAPY, Hyperprolactinemia, causing ovulation suppression, will lower
AND/OR MANAGEMENT estrogen and T levels and treating hyperprolactinemia may
Hormones and FSD improve FSD (13). The combined oral contraceptive (COC)
Innate sexual behavior emanates from complex interactions
blocks ovarian androgen production and increases SHBG lev-
between sex steroids, other hormones, their receptors, neu-
els, hence lowering free T. Some women may experience
rotransmitters within the brain, and a range of brain neuronal
substantial loss of the libido with the COC. Evidence that
networks. There is no single biological test to diagnose FSD. A
COCs containing antiandrogenic progestins have a more ad-
diagnosis of estrogen deficiency predicts VVA and dyspareu-
verse effect is lacking. Oral postmenopausal estrogen therapy
nia. Brain imaging studies of otherwise-healthy young pre-
also increases SHBG and may cause lowering of libido in some
menopausal women with and without HSDD suggest that for
such women, HSDD may arise from overactivation of neuronal women. Selective serotonin reuptake inhibitors and some selec-
networks in the brain that impede spontaneous or response tive noradrenaline reuptake inhibitors are associated with FSD
desire (ie, women being subconsciously hyper-vigilant as to in approximately 56% of treated women.
their response rather than allowing themselves to respond spon- It is critical to determine the patients primary concern, and
taneously) (8). why she is presenting at this time. Is the condition of recent
In postmenopausal women there is less limbic brain activity onset, long standing or life long? Is it situational (limited to
on exposure to erotic visual stimuli compared with premeno- certain types of stimulation, situations, or partners) or general-
pausal women (8, 9). Combined estrogen plus T therapy (but ized? It is important to understand each womans expectations
not estrogen alone) restores limbic activation following visual of any available therapy. Women will usually have sought
sexual stimulation in postmenopausal women to a degree simi- information from a variety of sources before speaking to their
lar to that seen in premenopausal women (9). health care provider.

The Endocrine Society. Downloaded from press.endocrine.org by [${individualUser.displayName}] on 12 January 2017. at 12:19 For personal use only. No other uses without permission. . All rights reserved.
ENDO 2016 REPRODUCTIVE ENDOCRINOLOGY 303

Assessing your patient: suggestions to start the conversation: Investigations


Are you presently sexually active? If the answer is no, General biochemical investigations are as indicated by history
Is this of concern to you? and examination. Women presenting with fatigue should have
Are you having any sexual concerns or difficulties at iron deficiency, hypothyroidism, and glucose intolerance ex-
this time? cluded. An androgen profile, with an SHBG level and free T
Have you noticed any change in your sexual interest? (calculated) is only indicated if treatment with T is considered.
If so, Is this a concern for you? This is not a diagnostic test.
Are you having difficulty with vaginal lubrication or
discomfort with sexual activity? SHBG
Do you have any concerns about your sexual wellbeing? SHBG is essential if T therapy is to be prescribed. Low SHBG
To optimize the care of women with FSD clinicians must be will result in rapid cellular uptake of T, and dosing should be
receptive, nonjudgemental, empathetic, and allow time for judicious. If SHBG is high (above the upper limit of the assay)
medical and psychosocial history taking (Table 1). A medical then efficacy of T therapy is unlikely. If a woman has a high
examination is undertaken as indicated: including genital and SHBG and is on oral estrogen the first step would be to stop the
pelvic examination, particularly for loss of sensitivity or pain oral estrogen therapy and review. One must wait approximately
disorders. 12 weeks for the SHBG to normalize and then up to another 12
weeks to see if the normalized SHBG is associated with im-
proved sexual wellbeing. Total and free T levels in the high-
TABLE 1. Information to collect from patient history
normal range suggest the womans problem is most unlikely to
Factors associated with FSD be related to androgen insufficiency and T therapy is usually
Higher education level contraindicated.
Poor health
Being in a significant relationship
Management
Daily/weekly urinary incontinence Women with estrogen deficiency should be prescribed vaginal
Depression estrogen therapy for prevention/treatment of VVA as low-dose
Past emotional and/or sexual abuse
vaginal estradiol/estriol. When vaginal estrogen is absolutely
When taking a history address the following:
contraindicated, vaginal moisturizers provide an effective
Context
nonhormonal option, although cost may limit use for some
Lack of privacy
women. Like estrogen, these must be used continuously. The
Safety
use of a vaginal moisturizer may not eliminate the need for a
Emotional rapport
vaginal lubricant during intercourse. Lubricants are effective
Culture/religious beliefs
for dyspareunia but offer no benefits beyond this.
Lack of appropriate stimuli
Individual and partners understanding of anatomy and
sexuality Testosterone Therapy
Lack of adequate sexual stimulation
The updated Endocrine Society Guideline for Androgens in
Partners sexual function
Women states:
Partners health Evidence supports the short-term efficacy and safety of high
Biological/hormonal physiologic doses of T (T) treatment of postmenopausal women
Amenorrhoea, menopause sexual dysfunction due to HSDD. Importantly, endogenous T
Medications (eg, antidepressants, oral contraceptive pill, levels did not predict response to therapy. At present, physio-
anti-androgen therapy) logical T preparations are not available in many countries
Depression including the United States, and long-term safety data are
Concurrent medical conditions lacking. We recommend that any woman receiving T therapy
Fatigue be monitored for signs and symptoms of androgen excess. (14)
Interpersonal
Relationship issues
Transdermal Testosterone Therapy
Lack of emotional intimacy
Transdermal T therapy has been shown to be effective for the
Intrapersonal
treatment of HSDD in randomized controlled trials of pre-
Body Image/self esteem
menopausal women age 35 46 years, and in naturally and
Trauma (sexual, physical, medical)
surgically postmenopausal women, both using and not using
Negative emotions (anxiety, fear, shame, guilt)
concurrent estrogen therapy. Small studies have shown that
Gender identify
transdermal T therapy is effective for FSD in women with
Concurrent mental health issues (eg, depression)
hypopituitism (14) and women with antidepressant-associated

The Endocrine Society. Downloaded from press.endocrine.org by [${individualUser.displayName}] on 12 January 2017. at 12:19 For personal use only. No other uses without permission. . All rights reserved.
304 ENDO 2016 MEET-THE-PROFESSOR CLINICAL CASE MANAGEMENT

FSD (15). The Endocrine Society guideline emphasizes that other women may have inappropriate activation of brain net-
initiation of T should be considered a trial, blood levels must works that inhibit sexual responsivity when exposed to sex
be monitored, and treatment ceased if there is lack of efficacy such that they may respond better to T 5-HT1A receptor
by 6 months (14). agonist. These concepts assume female sexual function is al-
ways responsive as opposed to intrinsic. It also assumes that
Tibolone these are the valid causes of FSD, that women continuously
Tibolone is a synthetic steroid, commonly prescribed to man- decrease into one specific category and that a clinician can
age menopause. It has a weak androgenic action and may determine which women have which problem. Interestingly,
improve sexual function, particularly sexual desire and arousal, both approaches use T as a therapy. Phase III trials of these
to a greater extent than traditional menopausal hormone new treatment approaches are under way.
therapy (MHT). It reverses vaginal atrophy and improves dys-
pareunia and urinary symptoms. MAIN CONCLUSIONS
FSD is a common problem with multifaceted aetiologies that
Dehydroepiandrosterone requires nonhormonal and hormonal assessment. Systemic T,
A meta-analysis of systemic dehydroepiandrosterone (DHEA) the most researched therapy for FSD, is effective for the treat-
for women with normal adrenal function reported no benefit o ment of HSDD and probably desire/arousal disorder and has a
for FSD (16). A Cochrane review reported a very small, but good safety profile. Despite the extensive evidence to support
statistically significant benefit, of a magnitude considered not short-term efficacy and safety of T for women, no T prepara-
clinically meaningful (17). A meta-analysis of DHEA for tion has been approved and pharmaceutical companies see little
women with adrenal insufficiency (primary and/or secondary) future in pursuing this field of drug development. Hence, in
found no statistically significant effect of DHEA on libido or most countries, physicians must prescribe male formulations or
sexual satisfaction (18) and a single study found no benefit for compounded T therapy.
women with hypopituitarism (19). DHEA is not recommended
for the treatment of FSD. Daily intravaginal DHEA has been CASES WITH QUESTIONS
reported to improve vaginal cytology and sexual function in Case 1
women with dyspareunia. One study of intravaginal DHEA A 52-year-old woman presents with low libido. Her last normal
administered 3 times per week showed no efficacy over pla- menstrual period was 18 months ago. She has vasomotor symp-
cebo. Intravaginal DHEA warrants further research. toms and commenced conjugated estrogens and medroxypro-
gesterone acetate but ceased because of sore breasts. She again
Flibanserin has hot flashes, irritability, vaginal dryness and dyspareunia,
Flibanserin has recently been approved for the treatment of disturbed sleep, and fatigue. She has a supportive husband,
HSDD in premenopausal women. Flibanserin acts as an agonist three children, is a nonsmoker, social drinker, and works in
at postsynaptic serotonin (5-HT1A) receptors, is an antagonist administration. No relevant family history. She is stressed at
at 5-HT2A receptors, and a very weak partial agonist on dopa- work. Her breast cancer risk is low.
mine D receptors in the brain (4). The most frequent adverse Which of the following is the next best step in her manage-
effects have been dizziness, nausea, fatigue, and somnolence. ment?
In addition syncope has been reported, resulting in strict pre- A. Refer for sexual counseling
scription guidelines for this drug. B. Prescribe a compounded T cream
C. Prescribe vaginal estrogen
Phosphodiesterase Type 5 Inhibitors D. Prescribe systemic estrogen progestin therapy
Phosphodiesterase type 5 inhibitors (PDE5I) may have a role in
the treatment of women with antidepressant-associated arousal Case 2
disorder as well as women with neurodegenerative diseases A 50-year-old woman presents with diminished libido and
leading to FSD. The available data does not conclusively sup- difficulty reaching orgasm since she had surgery 2 years ago, at
port these drugs as a standard treatment option for all women which time she had a resection of a right ovarian cyst and
affected with female sexual arousal disorder (FSAD), which is bilateral salpingectomy. Given that that time she has experi-
not simply a disorder of genital engorgement. enced amenorrhea and vasomotor symptoms (VMS). She also
has chronic fatigue. She reports poor memory and diminished
Combination Therapy organizational skills. She is married, has adult children, and
It has been proposed that some women with HSDD/arousal manages the family dairy farm. She was started on tibolone 18
disorder have blunted sensitivity to sexual cues and might best months prior, but has yet to notice any substantial improvement
be treated with a combination of T and a PDE5I. Conversely, in her symptoms.

The Endocrine Society. Downloaded from press.endocrine.org by [${individualUser.displayName}] on 12 January 2017. at 12:19 For personal use only. No other uses without permission. . All rights reserved.
ENDO 2016 REPRODUCTIVE ENDOCRINOLOGY 305

Which of the following is the next best step in her manage- 2. Gartoulla P, Worsley R, Bell RJ, Davis SR. Moderate to severe vasomotor
and sexual symptoms remain problematic for women aged 60 to 65 years.
ment?
Menopause. 2015;22:694-701.
A. Continue tibolone and explain that more time is often 3. Zeleke BM, Bell RJ, Billah B, Davis SR. Vasomotor and sexual symptoms
needed for a response in older Australian women: A cross-sectional Study [published online
October 9, 2015]. Fertil Steril. doi: 10.1016/j.fertnstert.2015.09.017.
B. Continue tibolone and refer for sexual counseling
4. Shifren JL, Monz BU, Russo PA, Segreti A, Johannes CB. Sexual prob-
C. Tell her symptoms are gynecological and that lems and distress in United States women: Prevalence and correlates.
hormones will not help her Obstet Gynecol. 2008;112:970-978.
D. Switch her to an alternative hormone therapy regimen 5. Davison SL, Bell RJ, LaChina M, Holden SL, Davis SR. The relationship
between self-reported sexual satisfaction and general well-being in
that includes T women. J Sex Med. 2009;6:2690-2697.
E. Discontinue tibolone and begin therapy with DHEA 6. Diagnostic and statistical manual of mental disorders. 4th ed. Washington
DC: American Psychiatric Press, 1994.
7. Diagnostic and statistical manual of mental disorders. 5th ed. Arlington,
DISCUSSION AND ANSWERS VA: American Psychiatric Publishing, 2013.
8. Jeong GW, Park K, Youn G, et al. Assessment of cerebrocortical regions
Case 1
associated with sexual arousal in premenopausal and menopausal women
This woman has classic estrogen deficiency symptoms. T by using BOLD-based functional MRI. J Sex Med. 2005;2:645-651.
therapy may help her but prescription is premature. Treating 9. Archer JS, Love-Geffen TE, Herbst-Damm KL, Swinney DA, Chang JR.
her estrogen deficiency should improve her sleep, mood, and Effect of estradiol versus estradiol and testosterone on brain-activation
patterns in postmenopausal women. Menopause. 2006;13:528-537.
hot flashes, and her vaginal dryness and dyspareunia should 10. Davis SR, Wahlin-Jacobsen S. Testosterone in women-the clinical signifi-
resolve. Thus, the first step would be menopausal hormone cance. Lancet Diabetes Endocrinol. 2015;3(12):980-992.
therapy and review after 6 8 weeks. If she then has persistent 11. Davison SL, Bell R, Donath S, Montalto JG, Davis SR. Androgen levels in
adult females: Changes with age, menopause, and oophorectomy. J Clin
libido loss and seeks treatment for this T can be considered. Endocrinol Metab. 2005;90:3847-3853.
12. Pontiroli AE, Cortelazzi D, Morabito A. Female sexual dysfunction and
Case 2 diabetes: A systematic review and meta-analysis. J Sex Med. 2013;10:1044-
1051.
This lady has become peri-/postmenopausal. Her symptoms are 13. Kalkavoura CS, Michopoulos I, Arvanitakis P, et al. Effects of cabergoline
hormonal but tibolone is clearly not effectively alleviating her on hyperprolactinemia, psychopathology, and sexual functioning in
estrogen deficiency symptoms. DHEA is not an effective alter- schizophrenic patients. Exp Clin Psychopharmacol. 2013;21:332-341.
14. Wierman ME, Arlt W, Basson R, et al. Androgen therapy in women: A
native for either her vasomotor symptoms or sexual problems. reappraisal: An endocrine society clinical practice guideline. J Clin
She needs to have her menopausal symptoms treated with Endocrinol Metab. 2014;99:3489-3510.
estrogen plus a progestogen, and a trial of T therapy. A sen- 15. Fooladi E, Bell RJ, Jane F, Robinson PJ, Kulkarni J, Davis SR. Testoster-
one improves antidepressant-emergent loss of libido in women: Findings
sible approach would be the prescription of a combined
from a randomized, double-blind, placebo-controlled trial. J Sex Med.
estrogen-progestin transdermal patch and a trial of transdermal 2014;11:831-839.
T at a dose appropriate for a woman. Her blood T levels should 16. Elraiyah T, Sonbol MB, Wang Z, et al. Clinical review: The benefits and
harms of systemic dehydroepiandrosterone (DHEA) in postmenopausal
be checked to ensure she is not exceeding the recommended
women with normal adrenal function: A systematic review and meta-
dose in approximately 3 weeks and she should be reviewed in analysis. J Clin Endocrinol Metab. 2014;99:3536-3542.
12 weeks. She should be advised that because you have treated 17. Scheffers CS, Armstrong S, Cantineau AE, Farquhar C, Jordan V.
her with a T therapy appropriate for a woman it is to be Dehydroepiandrosterone for women in the peri- or postmenopausal phase.
Cochrane Database Syst Rev. 2015;1:CD011066.
expected that she may not experience an improvement in 18. Alkatib AA, Cosma M, Elamin MB, et al. A systematic review and
sexual wellbeing for 4 8 weeks. meta-analysis of randomized placebo-controlled trials of DHEA treatment
effects on quality of life in women with adrenal insufficiency. J Clin
Endocrinol Metab. 2009;94:3676-3681.
REFERENCES 19. Johannsson G, Burman P, Wiren L, et al. Low dose dehydroepiandrosterone
1. Greenblatt RB, Karpas A. Hormone therapy for sexual dysfunction. The affects behavior in hypopituitary androgen-deficient women: A placebo-
only true aphrodisiac. Postgrad Med. 1983;74:78-80, 84-89. controlled trial. J Clin Endocrinol Metab. 2002;87:2046.

The Endocrine Society. Downloaded from press.endocrine.org by [${individualUser.displayName}] on 12 January 2017. at 12:19 For personal use only. No other uses without permission. . All rights reserved.
306 ENDO 2016 MEET-THE-PROFESSOR CLINICAL CASE MANAGEMENT

Polycystic Ovarian Syndrome

M19 polycystic ovaries on ultrasound but no evidence of ovula-


Presented, April 1 4, 2016 tory abnormalities or hyperandrogenism, the prevalence is
12%20%. Whichever definition is used, PCOS is one of the
most common endocrine disorders. Although often initially
Margaret E. Wierman, MD. University of Colorado evaluated for amenorrhea in adolescence or for infertility in
School of Medicine, Aurora, CO 80045 and Research young adult years, women with PCOS have issues across the
Service Veterans Affairs Medical Center, Denver, lifespan that need to be addressed. These may include:
Colorado 80220, E-mail: margaret.wierman@ucdenver.edu symptoms of androgen excess such as hirsutism, acne, alo-
pecia, metabolic abnormalities such as abnormal weight
gain, obesity, insulin resistance, increased risk of gestational
HISTORICAL OVERVIEW
diabetes and type 2 diabetes, metabolic syndrome, and other
Descriptions of women with signs and symptoms suggestive
abnormalities such as mood disorders, obstructive sleep ap-
of polycystic ovarian syndrome (PCOS) are present in an-
nea (OSA), nonalcoholic fatty liver disease (NAFLD), and
cient texts from the time of Hippocrates (460 377 BC). The
endometrial cancer. PCOS is an oligogenic/polygenic disor-
prevalence is similar across many populations suggesting
der with incomplete penetrance with associated epigenetic
PCOS emerged before racial diversity. Despite association
modification and environmental factors leading to difficul-
with decreased fecundity, PCOS may have persisted because
ties in the defining the genetic mechanisms. Multiple Ge-
of association with increased body weight with fat stores
nome Wide Association Study (GWAS) studies have been
available for times of limited caloric availability, ability to
recently published and may result in a better understanding
space timing of births, and perhaps delayed menopause.
of the pathogenesis of the disorder; however, the functional
PCOS was described as the Stein-Leventhal syndrome based
relevance of the long list of candidate loci is still lacking
on the observations of these investigators on the ovarian
and does not yet inform clinical practice.
morphology.
In 1990, an leave as is standard abbreviation for national
institutes of health consensus conference defined PCOS as BARRIERS TO OPTIMAL PRACTICE
biochemical and/or clinical hyperandrogenism with oligo- These include heterogeneity of the patients and definitions
anovulation after excluding other disorders (including of PCOS based upon consensus expert opinion, recent prac-
Cushings syndrome, androgen producing ovarian, or adre- tice to diagnose all obese women as having PCOS, lack of
nal tumor or exogenous androgen excess from drugs as understanding of the mechanisms underlying the disorder,
well as other causes of ovulatory dysfunction including lack of clinician knowledge about the comorbid conditions
hyperprolactinemia, thyroid disorders, or pregnancy). In to recognize and address, and inadequate information con-
2003, the Rotterdam consensus altered the diagnostic crite- cerning optimal treatment strategies across the lifespan.
ria to include 2 of 3 of the following: clinical and/or bio-
chemical hyperandrogenism, oligo-anovulation, and poly- LEARNING OBJECTIVES
cystic ovaries on ultrasound (again after exclusion of other As a result of participating in this session, learners should be
endocrinopathies). The NIH consensus definition is the bet- able to:
ter predictor of future metabolic disorders. The Rotterdam Understand the different consensus criteria for a
consensus definition is useful for predicting infertility and diagnosis of PCOS.
issues with induction of ovulation. The Androgen Excess Discuss the associated hormonal, metabolic and other
and PCOS Society suggests a modified definition which disorders that accompany PCOS.
includes the use of ultrasound abnormalities, but not the Outline the treatment strategies for women with PCOS
inclusion of ovarian morphologic changes alone without across the lifespan.
evidence of hyperandrogenism or oligo-anovulation. The List the other causes of hyperandrogenic anovulation
variability in the definitions and expert opinion reflects the other than PCOS and how to exclude them.
heterogeneity in the clinical presentation of the disease and
the underlying molecular mechanisms.
STRATEGIES FOR DIAGNOSIS THERAPY
AND/OR MANAGEMENT
SIGNIFICANCE OF THE CLINICAL PROBLEM The diagnosis of PCOS should be considered using the NIH
The prevalence of PCOS is 6%10% of reproductive aged and/or Rotterdam criteria. Exclusion of other endocrine dis-
women using the NIH consensus definition. If one uses the orders should be performed before assigning the diagnosis
Rotterdam criteria, which allows inclusion of women with of PCOS to the patient. Other causes of anovulation includ-

The Endocrine Society. Downloaded from press.endocrine.org by [${individualUser.displayName}] on 12 January 2017. at 12:19 For personal use only. No other uses without permission. . All rights reserved.
ENDO 2016 REPRODUCTIVE ENDOCRINOLOGY 307

ing hypothalamic amenorrhea, hyperprolactinemia, thyroid hair on upper lip, chin, but no male pattern balding. She has
dysfunction, generalized obesity, and pregnancy should had difficulty losing weight with diet and lifestyle intervention.
be excluded. Other causes of clinical or biochemical In the last year, she has had 3 periods and another with
hyperandrogenism should be excluded, including congenital progestin withdrawal (medroxyprogesterone 5 mg for 10 d).
hyperplasia, Cushings syndrome, ovarian or adrenal tumor, On examination, she has an anabolic appearance, BMI of 29
or exogenous exposure to androgens. kg/m2, excess hair on lip chin, neck, no temporal recession or
Treatment options in adolescence and adulthood when clitoromegaly, and grade II acne. On timed bloods days 15
pregnancy is not desired include diet and lifestyle modifica- after an induced menses, her LH is 15 mU/mL, FSH 5 mU/mL,
tion to prevent weight gain and/or induce weight loss, sup- testosterone 70 ng/dL (nl 40), this is a standard ab-
pression of androgens to help with acne and hirsutism and breviation dehydroepiandrostenedione sulfate DHEA-S 400
regularization of menses with a regimen that includes: an ng/mL (nl 350), and this is all commonly used medical
antiandrogen (spironolactone in United States) with an oral terminology and abbreviations does not need further clarifica-
contraceptive (OCP) with constant but less androgenic pro- tion AIC 5.5 (nl 4.0 6.0).
gestin (see slides). Metformin used twice daily before meals After exclusion of other endocrine disorders, which of the
as an insulin sensitizer helps insulin resistance may mitigate following define the NIH consensus criteria for a diagnosis of
weight gain and can improve the number of ovulations per PCOS?
year but is inadequate as the only therapeutic intervention. A) Hi LH to FSH ratio, clinical or biochemical
For those women desiring pregnancy, studies suggest that hyperandrogenism, and multiple cysts on ovaries by
letrozole, an aromatase inhibitor, is superior to clomiphene ultrasound (US).
citrate, especially in PCOS women who are obese. Both clomi- B) Abnormal weight gain, clinical and/or biochemical
phene citrate and letrozole are superior to metformin for induc- hyperandrogenism, and multiples cysts on ovaries by
tion of ovulation and resultant live births. Metformin has a role US.
in treatment of prediabetes and this is a standard abbreviation C) Clinical and/or biochemical hyperandrogenism and
for type 2 diabetes mellitus would keep DM2 across the lifes- oligo-anovulation.
pan. In addition to the reproductive and metabolic issues, D) Multiple cysts on ovaries and clinical and/or
women with PCOS are at higher risk of mood disorders, OSA, biochemical hyperandrogenism.
NAFLD, and endometrial cancer at an early age (if regular
endometrial shedding is not ensured). It is critical for the Case 2
provider caring for these women to identify and treat such A 28-year-old woman with a history of irregular periods,
comorbid conditions. Diet and lifestyle are critical components hirsutism, and progressive weight gain would like to get
of their care across the lifespan. pregnant and wants to know the best treatment options.
PCOS was diagnosed at age 18 years, and she has been
intermittently prescribed OCPs and spironolactone for hir-
MAIN CONCLUSIONS sutism. She weighed 140 lb (63.6 kg) in high school and 160
PCOS is one of the most common endocrine disorders,
lb (72.7 kg) in college. Her current weight is 180 lb (81.8
which results in comorbid complications in adolescence,
kg), and height is 63 in (160 cm) (BMI 31.9 kg/m2).
adulthood, with hirsutism, acne, alopecia, infertility, abnor- Blood pressure is 130/80 mm Hg. A urine pregnancy test is
mal weight gain and obesity, risk of gestational diabetes, negative. She began metformin 6 months ago, and her peri-
DM2, metabolic syndrome, and increase in cardiovascular ods have been more regular for the last 3 months.
risk markers. Increased early endometrial cancer can be Which of the following treatments would be most effective
prevented with use of OCPs or cyclic progestins to ensure in induction of ovulation?
endometrial shedding. Treatment of hyperandrogenism with A) Letrozole
OCPs and antiandrogens as well as, treatment of insulin B) Clomiphene citrate
resistance with metformin and diet and lifestyle are critical C) Human menopausal gonadotropins
components of the care of women with PCOS. Identification D) Cabergoline
of other issues, including DM2, metabolic syndrome, mood E) Progesterone suppositories
disorders, OSA, and NAFLD, should be addressed and
treated. PCOS is a disorder that requires intervention across
Case 3
the lifespan. A 52-year-old woman presents with worsening hirsutism
since menopause 2 years earlier. Her menarche was at age
CASES 11 years, and she noticed the onset of hirsutism and acne in
Case 1 adolescence. OCPs were prescribed to treat irregular menses
A 19-year-old woman presents with hirsutism, acne, and ir- in her twenties. She required clomiphene citrate for induc-
regular menses since menarche at age 11. She has increasing tion of ovulation to conceive her children. She has gained

The Endocrine Society. Downloaded from press.endocrine.org by [${individualUser.displayName}] on 12 January 2017. at 12:19 For personal use only. No other uses without permission. . All rights reserved.
308 ENDO 2016 MEET-THE-PROFESSOR CLINICAL CASE MANAGEMENT

weight over the years and her BMI is now 32 kg/m2. She has licles responding to the gonadotropins and excess estrogen,
noticed increasing hair growth on her breasts, chin, face, and poor endometrial receptivity, and poor rates of ovulation, preg-
upper abdomen since menopause. nancy, or live births. Clomiphene citrate (answer B), a selective
Laboratory test results: estrogen modulator that activated the central axis to increase
LH 32 mIU/mL (32 IU/L). FSH production, has been the mainstay of ovulation induction
FSH 34 mIU/mL (34 IU/L). and in studies has been shown to be superior to metformin
Testosterone 120 ng/dL (4.2 nmol/L). targeting the insulin resistance. However, its efficacy was in
DHEA-S 280 mg/mL (7.6 mol/L). the range of 23% live births, and obese women had a high rate
Which of the following is the most likely diagnosis? of resistance to clomiphene. Recent studies of the Reproductive
Network showed that use of a second generation aromatase
A) Adrenal virilizing tumor
inhibitor, letrozole (correct answer A), was superior in induc-
B) Hyperthecosis ovarii
tion of ovulation and live births compared with clomiphene,
C) Granulosa tumor of the ovary
especially in the obese subjects. There was a nonsignificant
D) Obesity-induced hyperandrogenism
increase in congenital abnormalities in the letrozole group but
E) Sertoli-Leydig cell tumor of the ovary
less multiple live births and overall good side effect profile.
Cabergoline (answer D) is a dopamine agonist used to treat
DISCUSSION OF CASES AND ANSWERS hyperprolactinemia. Cyclic progestin (answer E) may be used
Case 1. Take Home Point: To Understand the Different to shed the endometrial lining in women who cannot take
Criteria for a Diagnosis of PCOS OCPs, but it has no effect to improve rates of ovulation induc-
The definition of PCOS is based on consensus conference tion as a primary therapy.
guidelines rather than on prospective data. In 1990, an NIH Our understanding of the genetic underpinnings of PCOS
consensus conference defined PCOS as biochemical and/or has been frustrated by the fact that it often results in infer-
clinical hyperandrogenism and oligo-anovulation (answer C) tility, so large family studies were underpowered to identify
after excluding other disorders including Cushings syndrome, etiologic factors. Recently, 4 large GWAS studies have
androgen producing ovarian, or adrenal tumor or exogenous identified SNPs in regions of chromosomes that localize to
androgen excess from drugs, as well as other causes of ovula- genes that control steroidogenesis, production, or action of
tory dysfunction, including hyperprolactinemia, thyroid disor- gonadotropins especially FSH and the FSH receptor and
ders, or pregnancy. In 2003, the Rotterdam consensus altered many unknown genes that remain to be studied for their
the diagnostic criteria to include 2 of 3 of the following clinical functional role. The future may hold better understanding of
and/or biochemical hyperandrogenism, oligo-anovulation, and the diversity of the genetic underpinnings and future strate-
polycystic ovaries on ultrasound either by number of follicles gies to personalize therapies for our patients.
or more recently by ovarian volume, after exclusion of other
endocrinopathies. The NIH consensus definition predicts best Case 3. Take Home Point: To Understand the
the reproductive and metabolic disorders, including infertility Differential Diagnosis of Hyperandrogenism in the
and T2DM. The Rotterdam consensus definition is useful for Postmenopausal Woman and How to Evaluate and
predicting infertility and issues with induction of ovulation. Treat
The Androgen Excess and PCOS Society definition adds the Hyperthecosis (correct answer B) occurs when high gonad-
use of ultrasound abnormalities to the NIH criteria but not the otropin levels drive androgen production from the theca
inclusion of ovarian morphologic changes without evidence of cells. Whether women with hyperthecosis always have un-
hyperandrogenism or oligo-anovulation suggested by the Rot- derlying PCOS has not been clarified. An adrenal virilizing
terdam criteria. The other answers A, B, and D include com- tumor (answer A) would present with a rapid onset, severe
ponents of the Rotterdam criteria. Early menarche, obesity, or symptoms and signs, and usually high DHEA-S levels. A
high LH/FSH days 15 of the cycle are often observed but are granulosa tumor of the ovary (answer C) would present with
not a part of the diagnostic criteria. The variable diagnostic high estrogen levels and endometrial hyperplasia. Obesity
criteria may impact on clinical decision making and evaluation (answer D) can cause hirsutism because adipose tissue can
of future studies of treatment interventions. have increased 5-reductase activity, as well as local
aromatase activity, which can cause androgenic and estro-
Case 2. Take Home Point: To Understand the Current genic effects. However, it does not usually result in viriliza-
Data Concerning Best Treatment Options of Induction tion. Weight loss and suppression of androgens are the goals
of Ovulation in Women With PCOS and Infertility when treating with obesity-induced hyperandrogenism.
Induction of ovulation in women with PCOS is difficult be- Ovarian tumors that cause hyperandrogenism include
cause of the multiple ovarian cysts that can respond to gonad- Sertoli-Leydig cell tumors (answer E), arrhenoblastomas, or
otropins (answer C) human menopausal gonadotroprins (HMG) hilus cell tumors that secrete high large amounts of testos-
and/or recombinant FSH with hyperstimulation multiple fol- terone resulting in levels in the male normal range (240

The Endocrine Society. Downloaded from press.endocrine.org by [${individualUser.displayName}] on 12 January 2017. at 12:19 For personal use only. No other uses without permission. . All rights reserved.
ENDO 2016 REPRODUCTIVE ENDOCRINOLOGY 309

ng/dL [8.3 nmol/L]) and cause virilization. Because these 3. Dumesic DA, Oberfild SE, Stener-Vitorin E, Marshall JC, Laven JS,
Legro RS. Scientific statement on the diagnostic criteria, epidemiology,
tumors are rare, more common causes of postmenopausal
pathophysiology and molecular genetics of polycystic ovary syndrome.
hirsutism and virilization should be considered first. Endocr Rev. 2015;36:487-525.
4. McAllister JM, Legro RS, Modi BP, Strauss JF 3rd. Functional genom-
ics of PCOS: from GWAS to molecular mechanisms. Trends Endocrinol
REFERENCES Metab. 2015;26:118-124.
1. Azziz R, Dumesic DA, Goodarzi MO. Polycystic ovary syndrome: an 5. Alpanes M, Gonzalez-Casbas JM, Sanchez J, Pian H, Escobar-Morreale
ancient disorder? Fertil Steril. 2011;95:1544-1548. HF. RoManagement of postmenopausal virilization. J Clin Endocrinol
2. Legro RS, Arslanian SA, Ehrman DA, et al. Diagnosis and treatment of Metab. 2012;97(8):2584-2588.
polycystic ovary syndrome: an Endocrine Society clinical practice 6. Rothman MS, Wierman ME. How should postmenopausal androgen
guideline J Clin Endocrinol Metab. 2013:98:4565-4592. excess be evaluated? Clin Endocrinol (Oxf). 2011;75(2):160-164.

The Endocrine Society. Downloaded from press.endocrine.org by [${individualUser.displayName}] on 12 January 2017. at 12:19 For personal use only. No other uses without permission. . All rights reserved.
310 ENDO 2016 MEET-THE-PROFESSOR CLINICAL CASE MANAGEMENT

Bioidentical Hormone Replacement

M27 ing Center, which lead to more than 700 illnesses and at least 64
Presented, April 1 4, 2016 documented deaths. In November 2013, under President Obama,
Congress passed the Drug Quality and Safety Act of 2013
(DQSA), which strengthened the FDAs authority to regulate
JoAnn V. Pinkerton, MD. Division of Midlife Health, compounded pharmaceuticals under Title 1 (the Compounding
University of Virginia Health System, Charlottesville, Quality Act [CQA]) (4).
Virginia 22903; and North American Menopause Society, Key components of the DQSA include that drugs could not
Cleveland, Ohio 44124, E-mail: jvp9u@virginia.edu be compounded that the FDA considers unsafe or ineffective,
are new drugs, and are manufactured in large quantities of
INTRODUCTION approved drugs unless a drug shortage occurs, or are complex
Historical Overview medications or dosage forms. Compounded drugs must include
The U.S. Food and Drug Administration (FDA) became con- components of FDA-approved drugs or Active Pharmaceutical
cerned that pharmacies were producing medications beyond Ingredients (APIs) or bulk drug substances for which a U.S.
traditional compounding in the 1990s and issued initially a Pharmacopoeia or National Formulary monograph exists or are
Compliance Policy guide in 1992, which detailed plans for included on a forthcoming FDA list. Most importantly, com-
enforcing the Federal Food, Drug, and Cosmetic Act (FDCA) pounded drugs were to be packaged correctly and accurately
requirements against compounded drugs. This was withdrawn labeled with strength, quality, and purity of the drug. Com-
after the FDA Modernization Act (FDAMA) in 1997, which pounded pharmacists/pharmacies were barred from making false
codified the FDAs enforcement authority over compounding or misleading claims about the drugs (5) Compounding pharma-
under the FDCA as 503A, which placed limits on the phar- cies can be accredited by the Pharmacy Compounding Accredita-
macies of state compounded drug sale but exempted licensed tion Board, (PCAB) (6) a service of the Accreditation Commis-
pharmacists engaged in traditional compounding from good sion for Health Care. Accredited pharmacies can be found on their
manufacturing procedures (GMP), federal labeling standards; website, PCAB.org. The new DQSA will hopefully improve the
and new drug administration requirements, including safety safety of compounded hormones, particularly for outsourcing
and efficacy studies, if their compounded drugs included the pharmacies, although still not held to the same rigorous testing,
use of bulk substances or active product ingredients for which safety, and efficacy standards of FDA-approved hormones (7).
a U.S. Pharmacopoeia or National Formula monograph existed
and were not too complex to be produce in the pharmacy
setting. If requirements were met, the pharmacists were not SIGNIFICANCE OF THE CLINICAL PROBLEM
required to register with the FDA, undergo routine inspection, FDA is concerned that the claims for safety, effectiveness, and
or report adverse events. Limitations placed on compounding superiority of compounded BHT are not founded and that
pharmacists were that they could not advertise or solicit pre- patients as well as health care professionals may be misled.
scriptions for compounded drugs, regularly reproduce copies of Compounded drugs are not reviewed by the FDA for safety and
FDA-approved drugs, or compound drugs the FDA classified effectiveness. Concerns regarding the use of compounded BHT
as unsafe (1). The 503A clarified the FDAs authority over include inadequate evidence of efficacy and safety, variable
compounding but oversight was by the state pharmacy boards. purity and potency, and insufficient labeling. The Endocrine
Challenges to the prohibitions on advertising and solicitation Society, North American Menopause Society, American Con-
by the compounding industry occurred over the next years with gress of Obstetricians and Gynecologists, American Society for
varying responses and disparate ruling, which led to the FDA Reproductive Medicine, International Menopause Society, and
having different levels of enforcement throughout the country the FDA recommend against compounded bioidentical hor-
(2). At a Senate hearing on compounded drugs in 2007, con- mone therapy (CBHT) use by anyone without a medical con-
cern was expressed about increased use of compounded dition preventing them from using FDA-approved hormone
bioidentical hormone therapy (CBHT) and misleading about therapy (HT). Compounding errors may result in over- or
postulated benefits as being advertised as superior to FDA- underdosing, which may increase the risk of adverse effects.
approved therapies while lacking in scientific evidence or Health care providers should determine for each individual
warnings about risks (3). The FDA in 2007 issued warning patient whether HT is indicated for menopausal symptom relief
letters to seven pharmacies and developed a campaign to make or other reasons such as prevention of bone loss or quality of
the public aware of misperceptions about compounded BHT. life. If patients request compounded non-FDA-approved hor-
Risk became evident in 2012, when the multistate meningi- mone drugs, the provider should be prepared to determine
tis outbreak occurred from contaminated methyl prednisone whether an FDA-approved BHT will meet their needs or
acetate injections compounded at the New England Compound- whether compounded drugs are the best option for their spe-

The Endocrine Society. Downloaded from press.endocrine.org by [${individualUser.displayName}] on 12 January 2017. at 12:19 For personal use only. No other uses without permission. . All rights reserved.
ENDO 2016 REPRODUCTIVE ENDOCRINOLOGY 311

cific medical needs. Salivary testing is not recommended for suppositories, or troches and can consist of individual or com-
monitoring hormone levels because hormone levels in saliva do bined hormones.
not accurately reflect the amount of hormones a woman has in Postulated safety of compounded and FDA-approved BHT
her body. is based on studies of transdermal therapies, which seem to
have less clotting effect because they avoid the first-pass effect
through the liver. In addition, compared with synthetic proges-
BARRIERS TO OPTIMAL PRACTICE
tins, micronized progesterone has been shown to have fewer
Myths and celebrity endorsements abound about safety
negative effects on lipids, sleep, mood, and breast density,
and efficacy of compounded hormone therapies.
breast tenderness, and possibly breast cancer risk when com-
Compounded HTs have same risks as FDA-approved
bined with estrogen. There remains no available FDA-
therapies with unique risks related to compounding,
approved bioidentical T therapy for women due to modest
including over- or underdosing or presence of
effects on hypoactive sexual desire and concerns regarding risk
contaminants and lack of warnings.
of heart disease or breast cancer.
Many patients are vulnerable due to misinformation and
Many women consider CBHT options to be more safer
thus require more education in the office.
because they are marketed as natural with unsubstantiated
safety and efficacy claims in the media, Internet, with celebrity
LEARNING OBJECTIVES promotion of CBHT as safer and superior than FDA-approved
As a result of participating in this session, learners should be hormone therapies. These compounded HTs are often custom
able to: compounded and include multiple hormones. Salivary testing is
Define bioidentical HT. often recommended, although without proven efficacy.
Understand the potential concerns regarding compounded Dosing of CBHT depends on expert opinions as there is a
HT. paucity of pharmokinetic data on the products. Two commonly
Recognize differences between compounded and FDA- prescribed examples are BiEst and TriEst, which can be com-
approved bioidentical HT products. pounded into capsules, creams, gels, and troches. BiEst is
Recommend an approach to managing patients who usually in an 80:20 estriol-to-estradiol ratio and TriEst in an
request bioidentical HT. 80:10:10 estriol-to-estradiol-to-estrone ratio. Estriol is consid-
ered an investigational product by the FDA as it is not ap-
STRATEGIES FOR DIAGNOSIS, THERAPY, proved in the United States and made primarily during preg-
AND/OR MANAGEMENT (8 10) nancy. It is a weak estrogen but can stimulate the uterus if
The term bioidentical menopausal hormones refers to exog- dosed high enough. Estriol, unlike estrone, does not convert to
enous hormones biochemically similar to those produced estradiol, binds very weakly to estrogen receptors, and is rap-
within the body including 17-estradiol (predominant estrogen idly excreted. It is available in Europe as an intravaginal
before menopause), estrone (predominant estrogen after meno- estriol, which has been shown to treat urinary incontinence,
pause), estriol (from placenta), progesterone (ovaries, placenta, urogenital atrophy, and recurrent urinary tract infections in
and adrenal glands), T (ovaries and adrenal glands), and their postmenopausal women. The Wiley Protocol is another type of
conjugates. These are primarily derived from soy and yam CBHT, which is described by its founder as a biomimetic
precursors and must be chemically processed to be absorbed by multiphasic hormone replacement with hormones derived from
the human body. plant sources dosed to mimic the natural hormone blood levels
Bioidentical hormones are available by prescription both as found in a normal menstrual cycle in a healthy young woman
FDA-approved medications and as non-FDA-approved com- (age 20 y) and recommended for perimenopausal women. For
pounded therapies. Compounded drugs are customized and postmenopausal women, there is another regimen called the
prepared by a physician or pharmacist after receipt of a pre- Lunar Wiley Protocol, which follows the cycles of the moon.
scription primarily for those who are precluded from using a
similar approved FDA-approved product (allergic reaction) or MAIN CONCLUSIONS
who need an approved drug in an unavailable formulation. Evidence is lacking about superiority or safety for CBHT.
Prescription FDA-approved systemic BHT is available as es- FDA-approved products tested in large randomized clinical
tradiol (oral or transdermal as patch, gel, lotion, mist, and trials with an appropriate label (package insert) are recom-
vaginal ring with low-dose vaginal estradiol for vulvovaginal mended over custom-compounded HTs for most women (Table
atrophy available as cream, or tablet) and micronized proges- 1). Salivary testing has limitations due to intra- and
terone (oral or vaginal cream or suppository). Compounded interindividual variability and is not recommended for deter-
bioidentical hormone therapies (CBHT) are prepared according mining or following dosing decisions.
to a providers prescription into transdermal products such as Health care providers should consider compounded
gels, creams, lotions, sublingual tablets, subdermal implants, bioidentical hormones only for those women who cannot tol-

The Endocrine Society. Downloaded from press.endocrine.org by [${individualUser.displayName}] on 12 January 2017. at 12:19 For personal use only. No other uses without permission. . All rights reserved.
312 ENDO 2016 MEET-THE-PROFESSOR CLINICAL CASE MANAGEMENT

TABLE 1. Endocrine Society/American Congress 2. If you decide to prescribe the compounded preparation,
of Obstetrics and Gynecology (ACOG)/North which of the following would be the best marker to guide
American Menopause Society (NAMS)/International dosing and the relative composition of each component?
Menopause Society (IMS) Do Not Recommend A. Salivary hormone levels
Custom-Compounded HT B. Plasma hormone levels
Concerns C. Relief of clinical symptoms
Lack of approval by any regulatory agency D. Changes in lipid panel
Lack safety and efficacy data; no formal testing E. Performance on sequential neurocognitive testing
Absence of regulatory oversight in manufacturing Correct answer: C
Quality, purity, and batch-to-batch consistency concerns
False sense of an improved safety profile Case 2
A 65-year-old white female requests a second opinion on her
current treatment with compounded HT. The patient provides
limited information but tells you she had a treated breast cancer
erate an FDA-approved product due to intolerance or allergy 10 years before. Through review of electronic records, you are
(such as peanut allergy given that the FDA-approved micron- able to determine that she was diagnosed with breast cancer at
ized progesterone is in peanut oil), for those who need a lower age 55 years for which she underwent a lumpectomy followed
dose than FDA-approved commercially available doses or a by radiation therapy and chemotherapy for a 4-cm, grade 3,
different dosing regimen or formulation, or for whom a pro-
12-nodes-positive estrogen receptor positive (ER)/progesterone
vider decides that the compounded product best serves a spe-
receptor positive (PR), HER (herceptin receptor) negative duc-
cific womens medical needs.
tal cancer. Two years after her breast cancer, her homeopathic
There are potential medicolegal risks for those who pre-
doctor started her on Triest (compounded HT cream with
scribe CBHT if an adverse event occurs due to incorrect dosing
estradiol, estrone, and estriol) and told her it would prevent
or contaminants associated with compounding process occurs.
recurrent breast cancer. Two years ago, he switched her to the
Prescribers should document the specific reasons that a non-
Wiley Protocol (patented as a Biomimetic Hormone Restora-
FDA-approved therapy is being recommended and that discus-
tion Therapy consisting of an estradiol and progesterone
sion has included unique risks of compounding.
cream, which varies throughout a 28-day cycle and is dosed to
mimic the natural hormones produced by a womans body
CASES WITH QUESTIONS when in her prime at age 20 y).
Case 1 You discuss with her the pros and cons for continuing HT at
A 53-year-old white female (with a uterus) is seen to discuss all given her prior breast cancer history. You then discuss
treatment for her bothersome menopausal symptoms of hot flashes specifics of her compounded hormone replacement regimen
(7-10/d) with soaking sweats at night. She is considering her (matching hormone levels of a 20-year-old) and discuss the
compounding therapists suggestion that she use a mixed formu- alternative of FDA-approved estradiol and progesterone
lation transdermal cream of Triest (estradiol, estrone, and estriol) therapy.
with progesterone, T, and dehydroepiandrosterone (DHEA). Her
3. Considering this patients breast cancer history, which of
mother had postmenopausal breast cancer and osteoporosis and
the following best describes the comparison of the two
there is a family history of Alzheimers as well. She is fearful of
treatment strategies?
traditional HT because of the potential risk of heart disease,
A. Compounded preparation is less likely to cause a
dementia and breast cancer. During the appointment, you discuss
recurrence
pros and cons of HT and the difference between FDA-approved
B. Estradiol and progesterone is less likely to cause a
and compounded HT. You recommend the FDA-approved option
recurrence
of transdermal estradiol as patch, gel, lotion, or spray and oral
C. The two treatments are equivalent in the risk of
micronized progesterone. The patient tries these for 3 months, but
causing a recurrence
returns dissatisfied and convinced she wants to try transdermal
Correct answer: C
Triest cream with added T, progesterone, and DHEA like her
friend.
1. Which one of the following should serve as the best guide DISCUSSION OF CASES AND ANSWERS
for general HT dosing decisions? Case 1
A. Salivary hormone testing This patient presents with early menopausal vasomotor symp-
B. Blood hormone levels toms that are bothersome to her. The first step in the discussion
C. Relief of clinical symptoms is to identify reasons why she would be a candidate for therapy
D. Bone density changes and then discuss various options, which should include discus-
E. Patient preference of dose sion of alternatives to HT. We often review non-medication

The Endocrine Society. Downloaded from press.endocrine.org by [${individualUser.displayName}] on 12 January 2017. at 12:19 For personal use only. No other uses without permission. . All rights reserved.
ENDO 2016 REPRODUCTIVE ENDOCRINOLOGY 313

options that have been shown in randomized clinical trials to be ability and lack of correlation with tissue levels. Bone density
effective for moderate to severe hot flashesthese include testing is not done routinely at menopause unless women have
cognitive behavioral therapy and hypnosis. Discussion then risks for osteoporosis. Calcium in diet or supplement at 1200
moves to review non-hormonal medical therapies, which in- mg is recommended during the menopausal transition to pre-
clude FDA-approved low-dose paroxetine salt and off-label use vent bone loss. Due to concerns about risks of longer-term use
of SSRIs, SSNRs, gabapentin, and antihypertension patch of HT, the lowest dose to meet treatment goals is used. Patient
clonidine. After review of non-hormonal therapies discussion preference of dosing may not provide best balance of treatment
moves to risks and benefits of HTs for relief of her hot effect compared with potential risks and thus clinical response
flashes including the 2007 Rossouw reanalysis of the Wom- drives dosing decisions regardless of whether the patient is on
ens Health Initiative study (WHI) with less risk for women FDA-approved BHT or non-FDA-approved compounded
under 60 years and within 10 years of menopause. Potential therapy.
risks of breast cancer, blood clots, stroke, gallstones, heart Correct answer is C: Relief of clinical symptoms should be
disease, and breast and uterine cancer are discussed. Com- used to determine dosing decisions.
mon adverse effects of unscheduled bleeding and breast Although this patient might be a candidate for the tissue
tenderness for women with a uterus on combined selective estrogen complex (conjugated estrogen and estrogen
estrogen/progestogen therapy are reviewed. agonist/antagonist bazedoxifene) due to neutral (similar to pla-
After discussion, she is most interested in HT. Current cebo) effects on bleeding, breast tenderness, and breast density,
recommendation would be the lowest effective dose for the that choice would not be bioidentical and thus would not be
shortest duration that meets treatment goals. Given that she offered at this time.
desires BHT, the recommendation would be for an FDA- If she has an identified reason for compounded therapy such
approved HT, which would include an estrogen (oral or trans- as peanut allergy (micronized progesterone is compounded in
dermal estradiol) as well as protection against endometrial peanut oil in FDA-approved therapy), then one would consider
cancer by a systemic progestogen (bioidentical micronized pro- either non-bioidentical synthetic progestin therapies or com-
gesterone). Dosing often starts with lowest effective dose, pounded progesterone therapy in soy or olive oil. Patient pref-
which might be a 0.025 mg estradiol patch with 100 mg of oral erence for compounding should not drive the discussion to
micronized progesterone taken continuously or cyclically last compounded therapies without a documented discussion of
14 days of cycle. Although hormone levels are sometimes unique risks of compounded therapies including that estriol is a
drawn to document menopausal status, dosing is usually ad- non-FDA-approved hormone and that potential exists for pres-
justed to relief of clinical symptoms. Hormone levels are re- ence of undesirable additives or preservatives and for under- or
served for challenging clinical cases where relief of symptoms overdosing. Compounded hormone therapies are not monitored
is not occurring. There is no evidence that salivary testing by the FDA for purity or dose standardization. There is no
provides information to adjust dosing due to individual vari- requirement for a package insert or boxed warning, which is

TABLE 2. Compounded HT (Traditional or Outsourcing) Compared With FDA-Approved Therapies


Traditional Registered Outsourcing FDA-Approved
Regulation Compounders Facilities Therapies
Cannot make drugs the FDA considers
unsafe or ineffective
Cannot make drugs that are complex or that
use complex dosage forms
Cannot introduce new drugs without
premarket review or filing an NDA
Cannot reproduce large quantities of FDA-
approved drugs
Cannot make false or misleading claims on
labels or in ads
Must conduct clinical trials for safety and
efficacy
Concerns about under/overdosing

Abbreviation: NDA, new drug administration.


Differences in Federal Regulations for Drugs Made by Traditional Compounders, Registered Outsourcing Facilities, and Commercial Manu-
facturers. Update on medical and regulatory issues pertaining to compounded and FDA-approved drugs, including HT. Pinkerton and Pickar (7).

The Endocrine Society. Downloaded from press.endocrine.org by [${individualUser.displayName}] on 12 January 2017. at 12:19 For personal use only. No other uses without permission. . All rights reserved.
314 ENDO 2016 MEET-THE-PROFESSOR CLINICAL CASE MANAGEMENT

present on all FDA-approved BHT products, which provides in- approved oral progesterone. The progesterone molecule is large
formation regarding potential risks, warnings, and contraindica- and is not well absorbed through the skin. Thus, the concern
tions. The products have not undergone FDA-approved rigorous always exists when transdermal estrogen/progesterone
studies evaluating efficacy or potential risk and there are no creams are used that there is inadequate protection of the
pharmacokinetic data available for CBHT formulations. endometrium. Due to this concern, we recommend the addi-
tion of oral progesterone in addition to transdermal proges-
Case 2 terone to protect the endometrium.
This patient has entered back into traditional health care after Age-related concerns about HT include the unclear risks in
an absence of 10 years. She has unique risks for taking any general of continuing systemic HT at age 65 years. The
type of HT due to her prior node-positive estrogen-sensitive 2007 Rossouw reanalysis suggested less risk at less than 60
large breast cancer. Breast cancer recurrence has a long tail years of age or within 10 years of menopause. For this
and thus the fact that she is without evidence of disease at 10 patient, unique risks include both dosing (high blood levels
years is not reassuring about her potential risk of recurrence. with Wiley Protocol) and her duration of HT of at least 8
Many women with this type of cancer would be on either years. Lastly, it is important to educate and document the
tamoxifen or an aromatase inhibitor to prevent recurrence or discussion about the differences between FDA-approved
new cancers. Her first therapy, Triest, would have given her BHT compared with less-regulated and untested com-
less risk as this is usually a more standard postmenopausal pounded hormone therapies, particularly if CBHT is pre-
dosing regimen. Her current regimen, Wiley Protocol for scribed, due to potential medicolegal risks.
young women, is currently providing her with a multiphasic
estradiol and progesterone cream to match the hormonal levels REFERENCES
and rhythm of a 20-year-old. This is one time when blood 1. Federal Food, Drug, and Cosmetic Act, 21 USC 301, (2006).
2. Guidance for FDA Staff and Industry. Compliance Policy Guides Manual.
levels might be helpful to provide information to the patient Sec. 460.200 Pharmacy Compounding. US Food and Drug Administration
about her risks. Indeed, levels were drawn for this patient, website. Available at: http://www.fda.gov/ohrms/dockets/98fr/02d-
which showed markedly elevated levels of estradiol, estrone, 0242_gdl0001.pdf. Accessed November 30, 2015.
3. Hearing Before the Senate Special Committee on Aging, 110th Cong, 1st
and T.
Sess (April 19, 2007) (testimony of Steven K. Galson, MD, director,
Discussion with this patient should include her risks of CDER, FDA).
recurrent or new breast cancer, her age-related risks of using 4. Drug Quality and Security Act, Pub L No. 113-154, 127 Stat 587 (2013).
5. US Department of Health and Human Services, Food and Drug Administra-
HT at 65 years of age, the unique risks of compounding over
tion, Center for Drug Evaluation and Research. Guidance: Pharmacy com-
FDA-approved therapies and the unique blood levels seen with pounding of human drug products under Section 503A of the Federal Food,
the Wiley Protocol. The first part of the discussion would Drug, and Cosmetic Act. Available at: http://www.fda.gov/downloads/Drugs/
include potential risks of systemic HT which could stimulate GuidanceComplianceRegulatoryInformation/Guidances/UCM377052.pdf.
Accessed November 30, 2015.
the breast, an important considerations given that she had a 6. Pharmacy Compounding Accreditation Board: For Pharmacists. Available
prior estrogen receptor-positive, node-positive breast cancer. at: http://achc.org/pcab. Accessed November 30, 2015.
Although there has been much discussion that estriol protects 7. Pinkerton JV, Pickar JH. Update on medical and regulatory issues pertain-
ing to compounded and FDA-approved drugs, including hormone therapy
against breast cancer, in reality, estriol is a weak estrogen. If it [published online September 25, 2015]. Menopause. doi: 10.1097/GME.
is dosed high enough to be effective by itself, it has been 0000000000000523.
shown to stimulate the endometrium. Less is known about its 8. Pinkerton JV, Santoro N. Compounded bioidentical hormone therapy:
identifying use trends and knowledge gaps among US women. Menopause.
direct effect on breast cancer risk. There is no data showing 2015;22(9):926-936.
that compounded CBHT made with estriol is safer than FDA- 9. Pinkerton JV. What are the concerns about custom-compounded
approved estradiol/progesterone therapy. Both types of thera- bioidentical hormone therapy? Menopause. 2014;21(12):1298-1300.
10. Pinkerton JV. Think twice before prescribing custom-compounded bioidentical
pies have the potential to stimulate the breast and are worri-
hormone therapy. J Womens Health (Larchmt). 2014;23(8):631-3.
some for this patient in light of her prior estrogen-sensitive 11. Bhavnani BR, Stanczyk FZ. Misconception and concerns about bioidenti-
breast cancer. cal hormones used for custom-compounded hormone therapy. J Clin
Studies have shown that progesterone has less negative effect Endocrinol Metab. 2012;97(3):756-759.
12. Rossouw JE, Prentice RL, Manson JE, et al. Postmenopausal hormone therapy
on breast density and possibly breast cancer when combined with and risk of cardiovascular disease by age and years since menopause. JAMA.
estrogen; however, clinical trials have included government- 2007;4;297(13):1465-1477. Erratum in: JAMA. 2008;299(12):142626.

The Endocrine Society. Downloaded from press.endocrine.org by [${individualUser.displayName}] on 12 January 2017. at 12:19 For personal use only. No other uses without permission. . All rights reserved.
ENDO 2016 REPRODUCTIVE ENDOCRINOLOGY 315

Testosterone Replacement Therapy in Men

M40 inappropriately in men with minimal or atypical clinical mani-


Presented, April 1 4, 2016 festations of androgen deficiency and/or at most a single low T
level.
Once a diagnosis of hypogonadism is confirmed, it is impor-
Alvin M. Matsumoto, MD. Geriatric Research, tant to measure serum gonadotropin levels to determine
Education and Clinical Center, VA Puget Sound Health whether primary, secondary, or combined primary and second-
Care System, Department of Medicine, University of ary hypogonadism is present and furthermore, to distinguish
Washington School of Medicine, Seattle Washington, between organic vs functional etiologies of hypogonadism that
98108, E-mail: alvin.matsumoto@va.gov may affect the decision to treat with T or alter management
strategies. Unfortunately, gonadotropin levels are often not
INTRODUCTION measured (4), and potentially treatable or reversible functional
Historical Overview (1) causes of hypogonadism are often not considered (5).
In antiquity, animal testes were prescribed to treat symptoms of Given that the recent availability of more acceptable trans-
androgen deficiency. In 1889, CE Brown-Sequard reported dermal T formulations, launching of direct-to-consumer dis-
rejuvenating effects after self-administering aqueous extracts of ease (low T) awareness campaigns, increased recognition
animal testes, resulting in accelerated use of organotherapy. that low T levels occur frequently in common clinical condi-
In 1935, E Laqueur identified the potent androgen, and- tions (eg, aging, diabetes and obesity) and increased serum T
rosten-17-ol-3-one, from bull testes and coined the name testing, the number and rate of T prescriptions have increased
testosterone (testo for testis; ster for sterol; and one for ke- dramatically, particularly in the Unites States (2).
tone), and shortly thereafter, testosterone (T) was synthesized Increased T use has occurred despite the increased realiza-
by A Butenandt and G Hanisch and also by L Ruzicka and A tion of the challenges in making an accurate clinical and
Wettstein. biochemical diagnosis of hypogonadism and the unknown
The initial clinical descriptions of Klinefelter syndrome in long-term benefits and risks of T treatment on clinically mean-
1942 and Kallmann syndrome in 1944, highlighted conditions ingful outcomes, particularly in men with nonclassical causes
associated with severe androgen deficiency that could be of androgen deficiency, such as in aging men.
treated with T.
In the mid-1950s, longer-acting esters, such as T enanthate
and cypionate, became available and remained the mainstay of BARRIERS TO OPTIMAL PRACTICE
T replacement for next half a century. Potential barriers and challenges to optimal management of
In the late 1970s, an orally effective formulation, T men with hypogonadism may occur because:
undecanoate, became available and popular to replace T in a A diagnosis of hypogonadism is not firmly established
number of countries outside the United States. and confirmed prior to initiating T replacement therapy.
In the 1990s, the first transdermal T formulation, a scrotal T Functional causes of secondary or combined
patch, followed by a nonscrotal T patch became available. In hypogonadism that are potentially reversible or treatable
2000, the first T gel became available for the treatment of without T treatment are not considered.
hypogonadism, followed by a number of gels and solutions of Patient-centered goals of care and expectations are not
various T concentrations. Over the last 15 years, T gels have established prior to starting T replacement therapy.
rapidly become the most commonly prescribed formulations in Monitoring of efficacy and safety of T therapy is
the United States and other countries (2). inadequate.
In 2004, a longer-acting injectable formulation of T Long-term, randomized, controlled trials of T therapy on
undecanoate became available and has become a very popular clinically meaningful outcomes are lacking, limiting the
formulation for T replacement. It has only been available in the treatment discussions with patients to shorter-term benefits
United States since 2014. and risks and to the uncertainty of cardiovascular disease
(CVD) and prostate cancer risks of T treatment.
SIGNIFICANCE OF THE CLINICAL PROBLEM
T replacement therapy is indicated to treat male hypogonadism, LEARNING OBJECTIVES
a diagnosis that should only be made in men with clinical As a result of participating in this session, learners will be
manifestations (symptoms and signs) of androgen deficiency as able to:
well as consistently low serum T levels (on at least two occa- Improve the accuracy of diagnosing male hypogonadism to
sions) (3). Unfortunately, T treatment is too often initiated avoid potential overdiagnosis and overtreatment with T.

The Endocrine Society. Downloaded from press.endocrine.org by [${individualUser.displayName}] on 12 January 2017. at 12:19 For personal use only. No other uses without permission. . All rights reserved.
316 ENDO 2016 MEET-THE-PROFESSOR CLINICAL CASE MANAGEMENT

Identify functional causes of secondary hypogonadism may ameliorate symptoms of low vitality or energy
that may be reversible or treatable without T treatment. (fatigue), low libido, erectile dysfunction (ED), and/or
Integrate consideration of the contribution of depressed mood.
comorbidities and/or medications to clinical Evaluate Patients for an Organic vs Functional
manifestations attributable to androgen deficiency during Etiology of Hypogonadism (5)
T replacement. Most causes of primary hypogonadism are organic (due
Identify key elements of monitoring during T to a congenital, destructive or degenerative disorder of
replacement therapy. the testis), but causes of secondary hypogonadism may
be organic or functional.
Organic causes of secondary hypogonadism include
STRATEGIES FOR THERAPY AND
congenital, destructive, or infiltrative disorders of the
MANAGEMENT OF HYPOGONADISM hypothalamus or pituitary gland, resulting in permanent
Confirm the Diagnosis of Hypogonadism (3, 5) hypogonadism.
Consider the potential contribution of comorbidities or Functional causes of secondary hypogonadism that are
medications vs androgen deficiency on nonspecific
potentially reversible or treatable include suppression of
symptoms and signs of hypogonadism.
gonadotropin production by high prolactin levels, certain
Ask whether the patient has suffered a recent illness,
medications, obesity, nutritional deficiency, excessive
nutritional deprivation, or has recently used a medication
exercise, chronic comorbid illness, and alcohol abuse
(eg, opioids or glucocorticoids) that could transiently
(Table 1).
suppress T levels.
Measure serum T levels on at least two occasions, in the Considerations Prior to Staring T Treatment (5)
morning and fasting if possible, to make sure that T is Once clinical hypogonadism is diagnosed, the following issues
consistently low. should be considered before T treatment is initiated:
If a condition associated with altered SHBG levels (eg,
obesity) is present or suspected, measure a free T (using
an accurate assay, eg, calculated free T or free T by Severity of Clinical and Biochemical Androgen
equilibrium dialysis) Deficiency
Patients with more severe clinical and biochemical hypogonad-
Measure Gonadotropins ism are more likely to be treated with and benefit from T
LH and FSH levels should be measured to determine
therapy than those with few or isolated symptoms, low-normal
whether hypogonadism is due to a primary testicular
or slightly low total T and free T levels, and midnormal LH
disorder or secondary to a hypothalamic or pituitary
disorder, given that the latter may have management and FSH levels.
implications in addition to T treatment (3, 5).
Assess the Contribution of Comorbid Illness, Contribution of Comorbidities and Medications to
Depression, and/or Medications on Clinical Clinical Manifestations
Manifestations (5) Treatment of comorbidities (eg, depression) or discontinuation of
Treatment of the associated comorbid illness or medications (eg, antihypertensive medications) that might contrib-
depression, or discontinuation of offending medications ute to symptoms should be considered before starting T treatment.

Table 1. Organic and Functional Causes of Secondary Hypogonadism


Organic Causes Functional Causes
Idiopathic hypogonadotropic hypogonadism, Kallmann syndrome Hyperprolactinemia, prolactinoma
Hemochromatosis Opioids
Hypopituitarism (infiltrative, destructive, infectious, vascular or Glucocorticoids, Cushings syndrome
traumatic disease, radiation or surgery, hypophysitis)
Pituitary or hypothalamic tumor Anabolic steroids, progestins, estrogens
Pituitary stalk section or disease Morbid obesity, type 2 diabetes, sleep apnea
Nutritional deficiency, excessive exercise
Chronic systemic illness
Organ failure (renal, liver, heart, lung)
Alcohol abuse
Aging?

Including combined primary and secondary hypogonadism with a hormonal pattern consistent with secondary hypogonadism.

The Endocrine Society. Downloaded from press.endocrine.org by [${individualUser.displayName}] on 12 January 2017. at 12:19 For personal use only. No other uses without permission. . All rights reserved.
ENDO 2016 REPRODUCTIVE ENDOCRINOLOGY 317

Treatment of Potentially Reversible Functional Causes of Contraindications and Precautions to T treatment


Hypogonadism Despite inadequate and conflicting studies, the US Food and
Examples include: Drug Administration issued an advisory in 2014 that patients
Discontinuing medications that cause hyperprolactinemia should be warned of the possibility of CVD risk associated
or dopamine agonist treatment of hyperprolactinemia. with T treatment. CVD is not a contraindication to T treatment.
Tapering off or discontinuing opioids, glucocorticoids, However, in general, it is prudent not start T treatment (or any
central nervous systemactive medications, or progestins. new therapies) in a patient who has suffered a major (myocar-
Correction of nutritional deficiency, or weight reduction dial infarction, stroke, or deep vein thrombosis/pulmonary em-
program or surgery for morbid obesity. bolism) or other (unstable angina or transient ischemic attack)
In many instances, functional causes cannot be treated or CVD event in the preceding 6 months (Table 3) (5).
managed within a reasonable timeframe (eg, chronic
opioid dependency) so that T treatment should be
considered. Choice of T Formulations (5)
The choice is based both on pharmacological considerations
and patient preferences (eg, related to convenience and cost)
Potential Clinical Benefits and Risks of T treatment (Table 4) (5).
T treatment has the potential to significantly improve symptoms of
androgen deficiency and quality of life in hypogonadal men.
The most common short-term risk of T treatment is erythro- Monitor Efficacy and Safety During T therapy (5)
Monitoring should occur shortly after starting T therapy, ie,
cytosis. Other risks are relatively uncommon. As a conse-
within the first 6 months, and then yearly during T treatment.
quence of increased monitoring for prostate cancer (eg, digital
rectal examination [DRE] and prostate-specific antigen [PSA]
levels) during T therapy, there is an increased likelihood of Efficacy
detecting an abnormality during monitoring that may result in a Monitored by self-reported improvement of symptoms
prostate biopsy. and measurement of serum T levels.
Table 2 (5). If no clinical improvement or worsening of symptoms,
consider whether compliance, proper use, comorbid
illness, or change in medications might contribute, and
Discuss Patient-Centered Goals of Therapy measure serum T levels.
Identification of patient-specific goals and expectations are Ask men on intramuscular (IM) T injections whether
particularly important in men with borderline hypogonadism, there is a noticeable worsening of symptoms prior to
ie, men with limited clinical manifestations of androgen defi- next injection.
ciency (eg, isolated ED), and low total T but low-normal to Because T absorption and levels are variable and
slightly low free T levels. unpredictable, T levels should be measured on at least
Often, there are other causes for symptoms and alternative one or two occasions during T gel treatment to assess
treatments to T therapy (eg, phosphodiesterase type 5 inhibitors adequacy of T replacement (6).
for ED) and it is reasonable not to treat these men with T initially.
If T treatment is considered, a suggested approach is to identify
Risk
specific treatment goals, discuss alternative treatment options, Hematocrit should be monitored initially at 3-6 months
lower the patients expectation of improvement with T treatment, and then yearly. If significant erythrocytosis (eg,
treat with T for limited prescribed period of time (eg, 6 mo), and hematocrit 54%) occurs, T treatment should be
discontinue T treatment if there is no clinical response. stopped, T levels should be measured and an evaluation
for conditions that may cause hypoxemia (eg, obstructive
sleep apnea [OSA] or chronic lung disease) should be
Table 2. Potential Benefits and Risks of T Treatment performed. If T levels are found to be high or if a
hypoxemic condition is identified and treated
Benefits Risks
appropriately, it is possible to consider restarting T
Increased sexual function and Erythrocytosis treatment at a lower dosage.
activity The effect of T administration on the natural history of
Improved energy and vitality Formulation-related adverse benign prostatic hyperplasia (BPH) and prostate cancer
effect
is not known. Because of concerns related to the
Improved mood Increased prostate biopsy
potential risk of T therapy on these common age-
(related to monitoring)
associated conditions, The Endocrine Society Clinical
Increased bone mineral density Gynecomastia (uncommon)
Practice Guidelines recommend monitoring for lower
Increased muscle strength Increased obstructive sleep
urinary tract symptoms (LUTS) and prostate cancer (3)
apnea (rare)
(Table 5) (5).

The Endocrine Society. Downloaded from press.endocrine.org by [${individualUser.displayName}] on 12 January 2017. at 12:19 For personal use only. No other uses without permission. . All rights reserved.
318 ENDO 2016 MEET-THE-PROFESSOR CLINICAL CASE MANAGEMENT

Table 3. Contraindications to T Treatment


Contraindications Concern
Active or metastatic prostate cancer Stimulation of cancer growth
ER-positive breast cancer Stimulation of cancer growth
Precautions Concern
Prostate nodule/induration or high PSA Increased prostate cancer risk
High hematocrit or poorly-controlled hypoxic condition Increased risk of erythrocytosis; CV risk?
Severe, untreated OSA Increased OSA; cardiovascular risk?
Severe LUTS Increased risk of urinary retention
Severe or poorly controlled CHF or edematous condition Worsening of CHF or edema from fluid retention

Abbreviations: CHF, congestive heart failure; ER, estrogen receptor; PSA, prostate-specific antigen; OSA, obstructive sleep apnea;
LUTS, lower urinary tract symptoms, CV, cardiovascular.

MAIN CONCLUSIONS free T by equilibrium dialysis of 45 pg/mL (SLIGHTLY


Prior to initiating T replacement therapy, it is important to LOW); LH, 3.3 IU/L (NORMAL); FSH, 4.0 IU/L (NORMAL);
confirm the diagnosis of hypogonadism to avoid overdiagnosis and normal serum prolactin and transferrin saturation.
and potential overtreatment with T and to identify functional What is the next best step in managing this patient?
causes of secondary hypogonadism that may be treatable or A. Start T replacement because he has hypogonadism.
managed without T therapy. B. Focus management on weight loss, exercise, and
During T therapy, the contribution of comorbidities and glucose control.
medications to the clinical manifestations of androgen defi- C. No further intervention is necessary because he does
ciency should be considered in assessing an individuals re- not have hypogonadism.
sponse to T treatment. D. Discontinue metoprolol.
Given the unknown risks of cardiovascular disease and pros- E. Order a sella magnetic resonance imaging to rule out a
tate cancer with T treatment, monitoring of both efficacy and pituitary tumor.
safety is essential during T replacement therapy, particularly in
older hypogonadal men. Case 2
Mr. M. is a 34-year-old moderate obese caucasian man with
CASES WITH QUESTIONS recently diagnosed XXY-Klinefelter syndrome who was started
Case 1 on 1% T gel, 5.0 g daily 3 months ago and returns for a
Mr. O. is an obese, 54-year-old caucasian man with type 2 diabe- follow-up visit.
tes mellitus, coronary artery disease without cardiovascular (CV) Prior to starting T gel, his laboratory tests on two separate
events or angina, and hypertension, who complains of a 5-year occasions revealed a total T, 131 and 144 ng/dL (VERY LOW);
history of ED and 1-year history of loss of libido and very low free T by equilibrium dialysis, 21 pg/mL (VERY LOW); LH, 35
energy. He is referred for treatment of hypogonadism by his IU/L (VERY HIGH); and FSH, 56 IU/L (VERY HIGH).
primary care physician, who found that the patient had a low Since starting T gel, he feels great and notes a marked
serum T level. increase in energy, libido, motivation, and socialization, and
He denies recent illness or change in medications but he has reduction in irritability and anxiety (more even keeled), but
gained 20 lb in the last 8 months. no change in always-normal sexually aroused erections or
Examination is remarkable for an obese man in good sexual activity. He denied acne, greasy skin, new hair growth,
spirits; blood pressure, 124/80 mm Hg; pulse, 50 beats/min; or change in muscle bulk or strength. He also denied an
weight, 255 lb; ht, 58; body mass index (BMI), 39 kg/m2; increase size or tenderness of gynecomastia bilaterally, daytime
normal axillary, chest, and pubic hair; no kyphosis; 3 cm somnolence, or change in urination.
palpable breast tissue bilaterally; testes, 25 mL bilaterally; Examination is remarkable for moderate obese man with notably
normal muscle strength and gait. more spontaneous conservation and good mood; blood pressure,
Medications include metformin, metoprolol, hydrochlo- 118/83 mm Hg; pulse, 62 beats/min; ht, 60; wt, 239 lbs; BMI, 32.4
rothiazide, and lisinopril. kg/m2; no acne, skin irritation, or hair growth at gel application site;
Laboratory tests obtained by his primary care physician sparse axillary, chest, and pubic hair unchanged; no kyphosis; 6-cm
revealed a total T, 230 ng/dL (LOW) and LH, 2.9 IU/L nontender gynecomastia bilaterally unchanged; normal lungs, heart,
(NORMAL); hemoglobin A1c, 8.0%; and TSH, 2.9 IU/L. You and abdomen; nontender firm 2-mL testes bilaterally; no edema;
repeat fasting, morning labs that reveal a total T, 225 ng/dL; normal muscle bulk and strength.

The Endocrine Society. Downloaded from press.endocrine.org by [${individualUser.displayName}] on 12 January 2017. at 12:19 For personal use only. No other uses without permission. . All rights reserved.
ENDO 2016 REPRODUCTIVE ENDOCRINOLOGY 319

Table 4. T Formulations
Formulation Advantages Disadvantages
Intramuscular (IM)
T enanathate or Long-standing use IM injections every 1-2 wk
cypionate
Effective Symptom fluctuations
Reliable Higher T levels and more erythrocytosis vs
transdermal T
Inexpensive (self-injection)
Some flexibility in dose
T undecanoate Fewer injections Larger needle and volume (3-4 mL) injections in
clinic
REMS needed in the U.S. for potential immediate
post-injection pulmonary oil microembolism
Transdermal Effective Daily application
T gel (1%) Relatively steady T levels (T gels and solution) More variable T absorption and lower T levels vs
im T
T gel (1.62%) Circadian T variation (T patch) Dose titration, 2 patches often needed
T gel (2%) Ease and convenience Skin irritation/rash (T patch gel or solution),
dryness or stickiness; odor; poor adhesion of T
patch with excessive hair or sweating
T solution (2%) Less erythrocytosis vs im T injections Contact transfer of T (gels and solutions)
T patch (adhesive) Expensive
Transbuccal Relatively steady T levels Twice daily application
T tablet Only single dosage
Gum irritation, poorly tolerated initially
Expensive
Implanted pellet Relatively steady T levels over 3-6 mo Surgical implantation
T pellet Large No. of pellets
Extrusion, bleeding and infection
Not removable, if adverse effect of T
Infrequent use
Orala Ease (oral) 2-3 daily with meals
T undecanoate Variable T absorption, lower T levels and variable
clinical response vs other T formulations
High DHT levels (clinical significance?)
Nasal No injections 3 daily in each nostril
T nasal spray T levels highly variable
Nasal irritation

Abbreviation: REMS, Risk Evaluation and Mitigation Strategy.


a
Not available in United States.

Medications include T gel and ibuprofen. E. Switch to a 2% T gel or solution


Repeat laboratory tests on T treatment, fasting and the morn-
ing of his follow-up visit: hematocrit, 36% (LOW); total T, 270
ng/mL (LOW), SHBG, 30 nmol/L (NORMAL); calculated free DISCUSSION OF CASES
T, 48 pg/mL (LOW). Case 1
What recommendations would you make regarding T Correct answer: B. Focus management on weight loss, exer-
treatment? cise, and glucose control.
A. Switch to T injections to achieve higher T levels The patient has mild secondary hypogonadism with symp-
B. Apply T gel over legs to enhance T absorption toms but no objective signs of androgen deficiency, low total T
C. Increase T gel dose to 10 g daily on two occasions, slightly low free T, and normal gonadotropin
D. Continue the current dose of T gel levels. Therefore, answer C is incorrect.

The Endocrine Society. Downloaded from press.endocrine.org by [${individualUser.displayName}] on 12 January 2017. at 12:19 For personal use only. No other uses without permission. . All rights reserved.
320 ENDO 2016 MEET-THE-PROFESSOR CLINICAL CASE MANAGEMENT

Table 5. Monitoring During T Treatment.5


Efficacy Monitoring Frequency Management
Clinical improvement Baseline, 3-6 mo, then yearly If no clinical improvement or worsening
Self-reported symptoms and of symptoms by 6 mo or physical
physical examination manifestations by 1 y
Proper administration and compliance
Contribution of
comorbidities/medications
T level at nadir after IM T or any
time after other T formulations, and
if T levels low, increase dosage
In the absence of these explanations,
consider stopping T treatment
Serum T level Baseline, 3-6 months, then yearly Adjust T dosage to achieve mid-normal to
depending on clinical change low-normal (in older men) T level
Bone mineral density Baseline, then 1-2 y depending If osteoporosis is present at baseline,
DXA scan on other fracture risks calcium, vitamin D and
conventional osteoporosis treatment
(eg, bisphosphonates)
Safety Monitoring Frequency Management
Hematocrit Baseline, 3-6 mo, then yearly If hematocrit 54%, stop T
T level at nadir after im T or any
time after other T formulations, and
if levels high, decrease T dosage
Evaluate for hypoxic conditions, treat
appropriately and consider
restarting T at a reduced dosage
LUTS, PSA, and DRE Baseline, 3-6 mo, then as per If IPSS 19, PSA increase 1.4 in
LUTS by IPSS or self-reported standard of care any 12-mo period, or abnormal
DRE (new nodule or induration),
consider appropriate treatment (eg,
-blocker for LUTS) and/or
urological consultation
Gynecomastia Baseline, 3-6 months, then yearly If new or worsening symptomatic
Self-reported breast pain, tenderness gynecomastia, consider reducing T
or enlargement and examination dosage
Induction or worsening of OSA Baseline, 3-6 months, then yearly If new or worsening symptomatic OSA,
Daytime somnolence, witnessed consider sleep study and institution
apnea, snoring of CPAP or BiPAP therapy or
adjustment of CPAP or BiPAP,
respectively
Formulation-related adverse reaction Baseline, 3-6 months, then yearly If adverse effect, consider switching T
formulation

Abbreviations: DXA, dual energy x-ray absorptiometry; IPSS, International Prostate Symptom Score; IM, intramuscular; PSA,
prostate-specific antigen; DRE, digital rectal examination; LUTS, lower urinary tract symptoms; CPAP, continuous positive airway
pressure; BiPAP, bilevel positive airway pressure.

By history, he did not have a recent illness or medication symptoms, suggesting the possibility that weight loss could
change that could have suppressed his gonadotropins or T levels. improve his symptoms, gonadotropin suppression, and low
However, he gained weight recently. His BMI is now T levels, as well as other aspects of his health. So, the best
close to the morbid obesity range and blood glucose is answer is B. Focus management on weight loss, exercise,
poorly controlled. Therefore, the most likely working diag- and glucose control. In fact, some men who are told that low
nosis is functional secondary hypogonadism induced by T is caused by obesity become motivated to lose weight.
severe obesity. The patients recent weight gain is approxi- Although T treatment increases lean mass and decreases fat
mately associated temporally with onset of his most recent mass in hypogonadal men, the amount of fat loss is usually not

The Endocrine Society. Downloaded from press.endocrine.org by [${individualUser.displayName}] on 12 January 2017. at 12:19 For personal use only. No other uses without permission. . All rights reserved.
ENDO 2016 REPRODUCTIVE ENDOCRINOLOGY 321

reflected in large losses in body weight. An uncontrolled study mood and motivation that were his stated goals of therapy. He
suggested that chronic T treatment in obese men resulted in did not feel that low libido was a problem for him.
progressive weight loss. So, it is possible that T treatment During T gel therapy, day-to-day serum T levels are
induces weight loss in obese hypogonadal men, perhaps highly variable and unpredictable. Therefore, decisions to
through its effects on improving mood, motivation, and/or change T gel dose (answer C) or transdermal T formulation
activity. Because weight loss and maintenance require sus- (answer E), or to switch to another route of T delivery
tained lifestyle modifications, an initial trial of lifestyle modi- (answer A) should not be made based on a single T level,
fication to achieve weight loss, increase activity, and improve particularly given that he has had a good clinical response to
blood glucose control (answer B) is preferable to T treatment his current T gel regimen. Therefore, answer D is correct.
(answer A) as an initial intervention. The T gel formulation used is absorbed better when applied
The use of metoprolol (and/or other antihypertensive medi- over both shoulders and upper arms than over both legs; so
cations) may contribute to some of his sexual symptoms and answer B is incorrect.
low vitality. However, he has taken metoprolol for over 10
years and his symptoms only developed more recently. Also,
REFERENCES
the patient is adequately -blocked and his angina has been 1. Nieschlag E, Nieschlag S. Testosterone deficiency: A historical perspec-
well controlled on metoprolol, so discontinuing metoprolol tive. Asian J Androl. 2014;16:161-168.
(answer D) is not advisable. 2. Layton JB, Li D, Meier CR, et al. Testosterone lab testing and initiation in
Hyperprolactinemia and iron overload were ruled out. Because the United Kingdom and the United States, 2000 to 2011. J Clin
Endocrinol Metab. 2014;99:835-842.
there is a high likelihood of severe obesity-induced secondary 3. Bhasin S, Cunningham GR, Hayes FJ, et al. Testosterone therapy in men
hypogonadism and free T levels were only slightly low in the with androgen deficiency syndromes: An Endocrine Society Clinical Prac-
presence of normal gonadotropin levels, a sella magnetic reso- tice Guideline. J Clin Endocrinol Metab. 2010;95:2536-2559.
nance imaging (answer E) is not needed to rule out structural 4. Muram D, Zhang X, Cui Z, Matsumoto AM. Use of hormone testing for
the diagnosis and evaluation of male hypogonadism and monitoring of
causes of secondary hypogonadism, such as a pituitary tumor. testosterone therapy: Application of hormone testing guideline recommen-
dations in clinical practice. J Sex Med. 2015;12:1886-1894.
Case 2 5. Matsumoto AM. Testosterone administration in older men. Endocrinol
Metab Clin N Am. 2013;42:271-286.
Correct answer: D. Continue the current dose of T gel.
6. Swerdloff RS, Pak Y, Wang C, et al. Serum testosterone (T) level variabil-
The patient experience substantial short-term improvements ity in T gel-treated older hypogonadal men: Treatment monitoring impli-
in his symptoms of androgen deficiency, in particular energy, cations. J Clin Endocrinol Metab. 2015;100:3280-3287.

The Endocrine Society. Downloaded from press.endocrine.org by [${individualUser.displayName}] on 12 January 2017. at 12:19 For personal use only. No other uses without permission. . All rights reserved.
322 ENDO 2016 MEET-THE-PROFESSOR CLINICAL CASE MANAGEMENT

Care of the Adult Woman with Turner Syndrome

M58 Considering/establishing the diagnosis of TS


Presented, April 1 4, 2016 Continuing care after transition from the pediatric
endocrinologist
Care of the previously diagnosed TS women who has
Amanda Vincent, MBBS, B Med Sci, FRACP, PhD. been lost to followup.
Department of Endocrinology, Monash Medical Centre,
Monash Health, Clayton, Victoria 3168, Australia, E-mail: BARRIERS TO OPTIMAL PRACTICE
amanda.vincent@monash.edu Lack of awareness or experience with TS may lead to
delay in diagnosis and/or inadequate screening for
INTRODUCTION complications.
Historical Overview The range of health issues associated with TS means a
In 1938, the November issue of the Endocrine Societys jour- multidisciplinary team approach may be required.
nal, Endocrinology, published an article by Dr Henry Turner Long-term estrogen replacement therapy is necessary but
(1) titled, A Syndrome of Infantilism, Congenital Webbed compliance can be difficult to achieve. There is a lack of
Neck and Cubitus Valgus. A description of similar physical high-quality evidence to guide choice of therapy.
characteristics in female patients was published in the German The pathophysiology, natural history, and optimal
literature by O Ullrich in 1930 (20), hence the alternative name management of many of the health conditions associated
of Ullrich-Turner syndrome. However, definitive evidence for with TS remain unclear.
an X-chromosomal abnormality as the underlying cause of this
syndrome was not established until 1959 (2). LEARNING OBJECTIVES
As a result of participating in this session, learners should be
SIGNIFICANCE OF THE PROBLEM able to:
Turner syndrome (TS) is the most common chromosomal Recognize the clinical features of TS.
abnormality in females, affecting approximately 1/2000 live Consider the diagnosis of TS in the adult woman as part of
female births (3). It is the result of complete or partial X- the differential diagnosis of primary ovarian insufficiency.
chromosomal monosomy (either absent or structurally abnor- Apply the clinical practice guidelines for TS.
mal) in a phenotypic female. The characteristic clinical features Recognize the advantages and disadvantages of different
are short stature and gonadal failure, although multiple organ estrogen replacement preparations.
systems may be involved and the phenotype may vary. Women Understand the issues surrounding fertility in women
with TS are at risk of multiple medical problems (Table 1) and with TS.
TS is associated with a 2-4-fold increase in overall mortality
(4). Both diagnostic delay and underdiagnosis are significant STRATEGIES FOR DIAGNOSIS, THERAPY,
issues. A Danish study reported that the observed number of AND/OR MANAGEMENT
TS diagnoses was 50% of expected cases, assuming a preva- Diagnosis
lence rate of 1/2000 (3). Although usually diagnosed prena- The diagnosis should be considered in any female with:
tally, at birth or in childhood, up to 38% of women are Unexplained growth failure/short stature
diagnosed in adulthood (3). Primary amenorrhea, premature Pubertal delay
ovarian insufficiency (POI) and infertility, potential presen- Features of the TS phenotype (Table 1) and/or
tations of TS, may prompt a referral to the adult endocri- Primary or secondary amenorrhea with elevated FSH
nologist. Although intensive medical followup occurs in concentrations (6).
childhood, care of adult women with TS is suboptimal. Loss Diagnosis is made on the basis of karyotype (which should
to followup following the transition to adult care, lack of include probing for Y chromosome material). The absent sex
regular medical care, and inadequate complication screening chromosome may be an X or Y chromosome and may be
has been reported (5). Failure to screen/identify and manage absent in all cells (45X) or in only some cells resulting in
the health consequences of TS may contribute to increased mosaicism (eg, 45X/46XX). There may be complete or partial
morbidity/mortality. International guidelines (6) advocate a loss of the X chromosome (short or long arm) or structural
multidisciplinary approach to care of patients with TS. Thus, abnormalities (isochrome or ring chromosome). The presence
the challenges the adult endocrinologist may face in relation of Y chromosome material may lead to the development of
to TS include: gonadoblastoma and referral for consideration of gonadectomy

The Endocrine Society. Downloaded from press.endocrine.org by [${individualUser.displayName}] on 12 January 2017. at 12:19 For personal use only. No other uses without permission. . All rights reserved.
ENDO 2016 REPRODUTIVE ENDOCRINOLOGY 323

Table 1. Clinical Manifestations of TS Table 1. Clinical Manifestations of TS (Continued)


Feature Frequency (%) Feature Frequency (%)
Skeletal Skin
Short stature 100 Increased nevi 25
Osteoporosis 50 Neurocognitive
Cubitus valgus 45 Nonverbal neuropsychological deficits 7080
High arched palate 35 Visual-spatial organisation, executive
Short fourth metacarpal 35 function, social cognition, problem
Scoliosis 1020 solving, motor skills
Shield chest 60 Attention deficit disorder 24
Reproductive Behaviors 7080
Spontaneous puberty/menarche 30 Emotional immaturity, unassertiveness,
shyness, overcompliance, social
Primary/premature ovarian insufficiency 90
isolation, reduced self esteem,
Infertility 9598 obsessive behaviors, social anxiety
Gonadoblastoma 330
Cardiovascular Abbreviations: RR, relative risk; NASH, nonalcoholic
Bicuspid aortic valve 1530 steatohepatitis.
Coarctation of the aorta 17 Data derived from Freriks et al (5), Bondy (6), and Mortensen et
Other congenital heart defect: 14 al (9).
Atrial/ventricular septal defect
Aortic dilatation 50
Aortic dissection 2 is warranted. The phenotype varies with the karyotype; con-
Hypertension 2158 genital cardiac abnormalities are associated with the 45X
Ischemic heart disease RR 2.0 karyotype, whereas type 2 diabetes mellitus is associated with
Electrocardiogram abnormalities: 21 the isochrome Xq karyotype (7).
Sinus tachycardia, Right axis deviation,
QT prolongation, T wave abnormalities
Management
Endocrine
There is limited evidence based data to guide management and
Autoimmune hypothyroidism 2334
most recommendations are based on expert opinion (6, 8).
Autoimmune hyperthyroidism 2.5
Management of TS adults (Table 2) includes:
Impaired glucose intolerance 2578
Complication screening and treatment
Type 2 diabetes mellitus 1843
Lifestyle counseling (for general wellbeing and disease
Type 1 diabetes mellitus 0.55
prevention)
Dyslipidemia 50
Estrogen therapy (ET) until the usual age of menopause
Obesity 20 (higher?)
Psychosocial support
Gastrointestinal
Education
Elevated liver enzymes 2080
NASH/cirrhosis 30
Coeliac disease 46 Lifestyle Counseling
Inflammatory bowel disease 3 Lifestyle counseling regarding diet, weight management, ces-
Lymphatic obstruction sation of smoking, and exercise is essential for prevention/
Webbed neck 25 management of obesity, hypertension, metabolic syndrome,
Swollen hands/feet 20 osteoporosis, and cardiovascular disease. Low physical fitness
Low posterior hairline 40 and sedentary lifestyle contribute to the increased risk of
Spoon shaped nails 12 obesity.
Renal
Structural renal tract abnormalities 3040 Complication Screening
Ears Complication screening is summarized in Table 2. Cardiovas-
Otitis media 6080 cular disease is the main cause of excess mortality in women
Sensorineural hearing loss 5090 with TS (4). Hypertension, affecting most TS women, is a risk
Eyes factor for aortic dissection/dilatation, stroke, and ischemic
Acuity deficit/strabsimus 2535 heart disease. Hypertension should be treated vigorously ac-
(Table continues) cording to age-specific normal ranges, with systolic blood pres-
sure (BP) 120 mm Hg for individuals with bicuspid aortic

The Endocrine Society. Downloaded from press.endocrine.org by [${individualUser.displayName}] on 12 January 2017. at 12:19 For personal use only. No other uses without permission. . All rights reserved.
324 ENDO 2016 MEET-THE-PROFESSOR CLINICAL CASE MANAGEMENT

Table 2. Summary of TS Management Protocol at Monash Health


Screening Frequency

Diagnosis/ 2-5-
TS Comorbidity Clinical Management Investigation Initial Visit Annual Yearly Prepregnancy

CVD Treat CVD risk factors ECG, ECHO/cardiac if aortic if N


ECHO/cardiac MRI root 3 cm
Cardiologist review if cardiac if cardiac
abnormality abnormality
Hypertension Strict BP control. Systolic BP BP
120 mmg Hg if BAV
Renal Treat urinary tract infections, FBE, U&E, eGFR
hypertension
Urinalysis if structural
abnormalities
Renal US
Thyroid Treat thyroid disorders TFTs
Thyroid antibodies
Hearing Hearing loss underreported Audiology if abnormal if normal
Metabolic/Obesity Maintain healthy weight range Weight/BMI
(BMI 25 kg/m2)
Treat hyperlipidemia, diabetes Fasting lipids
mellitus
FBG or HbA1c
OGTT if abnormal
Osteoporosis Optimise calcium intake, Vitamin D CPM, Vitamin D,
and exercise, HRT until 50 y PTH
DXA
Gastro-intestinal Refer if persistent/progressive LFTs
increase in liver enzymes
Coeliac antibodies
Gynaecological Pelvic/vaginal US
Institute HRT and continue until Pap smear if sexually
50 y. Transdermal HRT if active
hypertensive or abnormal LFTs
discuss contraception/fertility
options
Breast examination Mammogram if 45 y
Psychological Assessment of overall function
Referral to support groups and
psychologist
Vocational guidance

Abbreviations: BAV, bicuspid aortic valve; BMI, body mass index; CPM, serum calcium/phosphate/magnesium; CVD; cardiovascu-
lar disease; ECG, electrocardiogram; ECHO, echocardiogram; eGFR, estimated glomerular filtration rate; FBE, full blood examination;
FBG, fasting blood glucose; HbA1c, hemoglobin A1c; HRT, hormone replacement therapy; LFT, liver function test; N, normal;
OGTT, oral glucose tolerance test; TFT, thyroid function test; U&E, electrolytes.
Based on Bondy (6) and Conway et al (8).

valve (8). Nocturnal hypertension is often observed in TS significant aortic dilatation, defined by aortic size index (as-
individuals and 24-hour ambulatory BP monitoring may be cending aortic diameter indexed for body surface area [BSA])
required to optimize control. The incidence of aortic dissection greater than 2.0 cm/m2 or absolute ascending aortic diameter
is 100 times the general female population with an earlier peak greater than 3 cm, require more frequent cardiology review
incidence in the third to fifth decades of life. Risk factors for (10). However, aortic dissection may occur with aortic diam-
aortic dissection include: left-sided cardiac abnormalities (in- eters considered normal for women with TS. There is a lack
cluding bicuspid aortic valve, coarctation), hypertension, aortic of data to guide the timing of surgical intervention; however,
dilatation, 45X karyotype and pregnancy (9). Transthoracic individuals with an aortic size index 2.5 cm/m2 or absolute
echocardiography is routinely used to monitor cardiac status; ascending aorta diameter 3.5 cm are considered at greatest risk
however, cardiac magnetic resonance imaging (MRI) is supe- of dissection and should be considered for prophylactic cardiac
rior in the assessment of aortic dilatation (9). TS women with surgery (10). Review by a cardiologist is essential where any

The Endocrine Society. Downloaded from press.endocrine.org by [${individualUser.displayName}] on 12 January 2017. at 12:19 For personal use only. No other uses without permission. . All rights reserved.
ENDO 2016 REPRODUTIVE ENDOCRINOLOGY 325

cardiac abnormalities, cardiac symptoms or multiple risk fac- increased muscle mass, increased high-density lipoprotein cho-
tors for ischemic heart disease exist and prior to pregnancy. lesterol, reduced hyperinsulinemia, and improved vascular
Type 2 diabetes mellitus is more common (relative risk, 4.4) function (9). EPT should be continued to the age of usual
in TS women. Both beta-cell dysfunction and obesity-related menopause ( 51 y); continuation after this date would depend
insulin resistance are implicated in the pathophysiology of on the individual womans risk/benefit analysis. However, a
impaired glucose tolerance. significant proportion of TS women do not maintain EPT
Elevated liver enzymes are common in TS with a 5-fold following pubertal induction. Counseling and education is nec-
increase in the risk of cirrhosis. Postulated causes of hepatic essary to increase compliance with EPT.
dysfunction are:
Congenital intrahepatic vascular abnormalities, and/or Psychosocial and Cognitive Difficulties
Nonalcoholic fatty liver disease related to the metabolic Psychosocial and cognitive difficulties associated with TS are
syndrome/obesity (11). often under-recognized or obscured by good verbal skills. Phy-
The reported relative risk of fracture is 1.25-2.16 in TS women, sicians should be aware of the negative psychological effect of
predominately affecting the forearm (12). Contributing factors infertility and POI. Psychological care is essential and should
include: estrogen deficiency, skeletal dysplasia related to SHOX be directed at identifying issues (women may be reticent to
gene deletion, small bones with altered geometry, cortical bone volunteer concerns), reinforcing and supporting the individuals self-
deficits, vitamin D deficiency, and increased falls risk due to esteem and assisting women with TS to remain in the main-
visual-spatial difficulties, decreased muscle tone/physical activity, stream of social, educational, and occupational life.
impaired hearing and coordination/balance problems (12). Moni-
toring of bone mineral density (BMD) by dual x-ray absorp- MAIN CONCLUSIONS
tiometry (DXA) is necessary. However, DXA underestimates TS, affecting approximately 1/2000 females, results from com-
areal BMD (ie, reads low) in the setting of small bones with plete or partial X chromosomal monosomy. Although short
height less than 150 cm (12). Calcium intake and vitamin D stature and gonadal failure are characteristic, the range of
should be optimized. ET is effective in increasing BMD; how- phenotypes may vary leading to under-diagnosis or delayed
ever, data regarding fracture prevention is lacking. Other diagnosis. Multiple organ systems may be affected in TS and
agents (eg, bisphosphonate/denosumab) should be considered lifelong complication screening is necessary. Physicians also
if a minimal trauma fracture occurs or if there is significant must be aware of the psychosocial risks associated with TS.
decline in BMD while on adequate ET, although data regarding Although intensive medical followup occurs during childhood,
the use of these agents in women with TS is limited. followup of adolescents/women with TS is frequently inad-
equate. ET may be required for pubertal induction or in women
ET with POI and should be continued until the age of usual
ET may be required for pubertal induction and in women who menopause. Most women with TS are infertile, requiring donor
develop POI (6). Pubertal induction is usually commenced at oocyte to achieve pregnancy. However, pregnancy in TS
age 8-10 years coincident with GH therapy, thus optimizing the women is associated with significantly increased risks and
effect of GH and allowing breast development to occur at the requires careful evaluation and counseling before proceeding.
same pace as it would in a non TS adolescent (13). It may also
be necessary in the older adolescent presenting with primary CASES WITH QUESTIONS
amenorrhea. Gradually increasing doses of estrogen (oral or Case 1
transdermal) are used with the aim of completing feminization A.B., a 36-year-old married teacher, is referred to you having 8
over a 2-4-year period. Doses are adjusted according to GH use months of amenorrhea following cessation of the combined
and clinical response (Tanner stage breast development, bone oral contraceptive (OC) pill (COCP). She describes some va-
age, and uterine growth). A progestogen is added once uterine somotor symptoms during this time although these are not
growth has occurred and the endometrial thickness is greater especially troubling. She is sexually active. Menarche occurred
than 4 mm or after the first vaginal bleed. Abnormal breast at age 13 years with regular menses prior to commencing the
development (tubular breasts) may occur with too-high/too- COCP 10 years ago. She is Gravida 2 Para 2 (children age 11
rapid estrogen dose introduction/escalation and synthetic/too- and 13 y) with no difficulties with conception/pregnancies. Her
early progestogen introduction as the postulated causes. Most mothers age at menopause was 50 years and her sister, age 39
women with TS require long-term estrogen and progestogen years, has regular menses. Her history is unremarkable except
therapy (EPT), although there is a lack of long term follow-up for low vitamin D levels. Her only medication is a vitamin D
data. Studies of EPT in TS women have demonstrated im- supplement. She has never smoked and does not drink alcohol.
provement in aspects of cognitive function (executive ability, She has a family history of hyperlipidemia. Her mother is 165
memory, and motor functions), maintenance of BMD, protec- cm and father 182 cm tall. Physical examination: normal ex-
tive effect on liver function, reduced visceral adipose tissue, cept for height 138.5 cm (weight 46 kg). Investigations per-

The Endocrine Society. Downloaded from press.endocrine.org by [${individualUser.displayName}] on 12 January 2017. at 12:19 For personal use only. No other uses without permission. . All rights reserved.
326 ENDO 2016 MEET-THE-PROFESSOR CLINICAL CASE MANAGEMENT

formed by her general practitioner: Negative pregnancy test; C. Advise her that women with TS should not become
FSH, 129 IU; Estradiol 73 pmol/L; Vaginal ultrasound (US): pregnant
normal size uterus with two follicles visible in the left ovary D. Organize a preconception/pregnancy TS evaluation
and right ovary not visualized.
Repeat FSH (89 IU) and estradiol (40 pmol/L) levels with
DISCUSSION OF CASES AND ANSWERS
karyotype 45X/47XXX confirms the diagnosis of POI second-
Case 1
ary to mosaic TS. TS evaluation (Table 2): Normal This case illustrates the diagnosis of TS in adulthood. Diagnos-
echocardiogram/DXA; horseshoe kidney on renal US; normal
tic criteria for POI are: More than 4 months of amenorrhea
laboratory investigations except TSH, 5.5 mU/L (FT4, 14
associated with menopausal-range FSH levels on two occa-
pmol/L); positive thyroglobulin and thyroid peroxidase anti-
sions at least 1 month apart with exclusion of secondary causes
bodies and 25 hydroxy-vitamin D, 40 nmol/L.
of amenorrhea (14). Her short stature raises the question of TS
1. Which of the following is the most appropriate next step
as the cause of her POI (6). A mosaic karyotype of 45X/
in the management of her POI?
47XXX is more likely to be associated with spontaneous men-
A. 0.625 mg conjugated equine estrogen plus 10 mg
arche and pregnancy (15). Management of TS includes com-
cyclical medroxy-progesterone acetate
plication screening and treatment (Table 2), institution of ET
B. Transdermal patch containing 50 mcg estradiol and
and psychosocial support (6).
continuous 140 mcg norethisterone
C. COCP
D. T therapy Question 1: Answer C
E. 2 mg oral estradiol plus oral cyclical micronized A.B. requires ET until the age of usual menopause. As she is
progesterone sexually active and does not want further pregnancies, the
F. Venlafaxine COCP is the most suitable form of ET. Up to 10% of women
Two years later, at her annual review, A.B. tells you that her may spontaneously conceive following a diagnosis of POI (14).
BP has been elevated when checked by her general practitio- The optimal ET for adult women with TS is not established
ner. She is divorced from her husband and has no current and ET should be individualized. Factors to be considered
sexual partner. BP today: 140/90 mmHg. include:
2. Which of the following is the most appropriate next step Need for contraception
in her management? Uterus present: use combined EPT
A. Commence a calcium antagonist for her hypertension Patient preference
B. Cease all hormone therapy (HT) Presence of comorbidities:
C. COCP
Hypertension: avoid COCP. Transdermal estrogen may

D. Transdermal 100 mcg estradiol and oral cyclical mi-
be preferable
cronized progesterone
Osteoporosis: may require higher dose of estrogen
E. 0.625 mg conjugated equine estrogen plus 5 mg
Use transdermal estrogen if obesity/metabolic syndrome/
continuous medroxy-progesterone acetate
elevated liver enzymes/thrombosis risk factors
Symptoms of vulvovaginal atrophy: consider addition of
Case 2
vaginal estrogen preparation
C.D. is a 23-year-old recently married childcare worker with
Androgen concentrations are reduced in women with TS and a
TS who initially presented with short stature and was diag-
small pilot study (16) noted improved lipid profile, bone den-
nosed with 45X TS at age 6 years. She received GH treatment
sity, body composition, quality of life, and aspects of cognitive
for 7 years and pubertal induction with increasing doses of
function with oral T addition to EPT compared with placebo.
estrogen from age 11 years. She has been taking COCP since
Further research is required.
age 16 years. Complications associated with her TS include:
hypothyroidism controlled on 100 mcg T4, vitamin D defi-
ciency for which she takes a vitamin D supplement, and mild Question 2: Answer D
hearing loss. During her consultation as part of her usual TS She no longer requires contraception and the COCP may el-
review, she says that she and her husband would like children. evate BP. She needs to continue ET and transdermal estrogen
She has a 28-year-old sister without TS who has 2 children is suggested for hypertensive women (8). If her BP remains
Height, 154 cm; weight, 54 kg; BP, 134/85 mm Hg. Cardio- elevated then she requires antihypertensive treatment. There is
vascular examination is normal. no evidence to support specific antihypertensive agents; how-
1. Which of the following is the most appropriate next step ever, -blockers or angiotensin receptor antagonists have theo-
in her management? retical advantages in regard to aortic dilatation/dissection
A. Begin home ovulation testing prophylaxis, -blockers are useful for sinus tachycardia and
B. Organize a semen analysis for her husband vasodilators may exacerbate edema (9).

The Endocrine Society. Downloaded from press.endocrine.org by [${individualUser.displayName}] on 12 January 2017. at 12:19 For personal use only. No other uses without permission. . All rights reserved.
ENDO 2016 REPRODUTIVE ENDOCRINOLOGY 327

Estradiol has been advocated as the preferred estrogen with karyotypes and are associated with an increased risk of fetal
avoidance of conjugated equine estrogens (17) . An estrogen loss (30 45%) although the rate of birth defects seems to be
dose adequate for symptom relief (if present) and prevention of similar to the general population (6, 15).
complications is necessary and limited evidence suggests a Options regarding oocyte/ovarian cryopreservation from girls/
dose of 2-4 mg oral or 100-200 mcg transdermal estradiol is adolescents with TS is evolving but not routinely offered/
required especially to maintain BMD (6). The effect of HT on performed.
glucose tolerance is unclear. HT has no harmful effect on liver
function in TS and may be beneficial (11). REFERENCES
1. Turner HH. A syndrome of infantalism, congenital webbed neck and
cubitas valgus. Endocrinology. 1938;23(5):566-574.
Case 2 2. Ford CE, Jones KW, Polani PE, De almeida JC, Briggs JH. A sex-
Question 1: Answer D chromosome anomaly in a case of gonadal dysgenesis (Turners syn-
Counseling regarding fertility and pregnancy is essential for all drome). Lancet. 1959;1(7075):711-713.
3. Stochholm K, Juul S, Juel K, et al. Prevalence, incidence, diagnostic delay,
girls/women with TS. Most TS women are infertile and preg- and mortality in Turner syndrome. J Clin Endocrinol Metab. 2006;91(10):
nancy can only be achieved through assisted reproductive tech- 3897-3902.
nology with donor oocyte/embryo. Pregnancy in TS women is 4. Schoemaker MJ, Swerdlow AJ, Higgins CD, et al. Mortality in women
with turner syndrome in Great Britain: A national cohort study. J Clin
considered high risk, requiring counseling, prepregnancy Endocrinol Metab. 2008;93(12):4735-4742.
evaluation (Table 2), and review by a cardiologist experienced 5. Freriks K, Timmermans J, Beerendonk CC, et al. Standardized multidisci-
plinary evaluation yields significant previously undiagnosed morbidity in
in managing TS women (6, 15, 18). However, prepregnancy
adult women with Turner syndrome. J Clin Endocrinol Metab. 2011;96(9):
screening is often not performed or is incomplete, which may E1517-E1526.
contribute to poorer outcomes. A study of 93 French women 6. Bondy CA. Care of girls and women with Turner syndrome: A guideline
of the Turner Syndrome Study Group. J Clin Endocrinol Metab. 2007;
with TS and pregnancy following oocyte donation reported 92(1):10-25.
that only 37.8% of women were prescreened with cardiac 7. Bakalov VK, Cheng C, Zhou J, Bondy CA. X-chromosome gene dosage
imaging (19). and the risk of diabetes in Turner syndrome. J Clin Endocrinol Metab.
2009;94(9):3289-3296.
Obstetric complications include an increased risk of miscar- 8. Conway GS, Band M, Doyle J, Davies MC. How do you monitor the
riage, maternal cardiovascular complications (pre-eclampsia, patient with Turners syndrome in adulthood? Clin Endocrinol. 2010;73(6):
postpartum hemorrhage aortic dilatation/dissection), gesta- 696-699.
9. Mortensen KH, Andersen NH, Gravholt CH. Cardiovascular phenotype in
tional diabetes, and cephalopelvic disproportion requiring cae- Turner syndromeintegrating cardiology, genetics, and endocrinology.
sarean section (6). Aortic dissection may occur in up to 2% of Endocr Rev. 2012;33(5):677-714.
pregnant women with TS (compared with general maternal 10. Matura LA, Ho VB, Rosing DR, Bondy CA. Aortic dilatation and dissec-
tion in Turner syndrome. Circulation. 2007;116(15):1663-1670.
mortality of 1/10000) with an 86% maternal mortality rate. 11. Roulot D. Liver involvement in Turner syndrome. Liver Int. 2013;33(1):
Cardiac MRI should therefore be performed before pregnancy 24-30.
is attempted and any significant cardiac valve abnormality, 12. Bakalov VK, Bondy CA. Fracture risk and bone mineral density in Turner
syndrome. Rev Endocr Metab Disord. 2008;9(2):145-151.
coarctation or aortic size index greater than 2.0 cm/m2 has been 13. Ross JL, Quigley CA, Cao D, et al. Growth hormone plus childhood
proposed as an absolute contraindication to pregnancy (18). low-dose estrogen in Turners syndrome. New Engl J Med. 2011;364(13):
1230-1242.
Single embryo transfer only is recommended with significantly
14. Nelson LM. Clinical practice. Primary ovarian insufficiency. New Engl
increased risks with twin pregnancies. Women with TS require J Med. 2009;360:606-614.
close surveillance during pregnancy including treatment of 15. Bondy C. Pregnancy and cardiovascular risk for women with Turner
syndrome. Womens Health (Lond Engl). 2014;10(4):469-476.
hypertension and periodic echocardiography/MRI with cardi-
16. Zuckerman-Levin N, Frolova-Bishara T, Militianu D, et al. Androgen
ologist review (18). Hypertensive disorders and aortic dissec- replacement therapy in Turner syndrome: A pilot study. J Clin Endocrinol
tion are more prevalent with oocyte donor pregnancies but may Metab. 2009;94(12):4820-4827.
17. Trolle C, Hjerrild B, Cleemann L, et al. Sex hormone replacement in
be declining due to better cardiovascular screening and single Turner syndrome. Endocrine. 2012;41(2):200-219.
embryo transfer (15). Surrogacy or adoption are alternatives for 18. Practice Committee of American Society For Reproductive Medicine.
having a family when pregnancy is not advised. Increased maternal cardiovascular mortality associated with pregnancy in
women with Turner syndrome. Fertil Steril. 2012;97(2):282-284.
TS adolescents/women with spontaneous menarche/menstrual 19. Chevalier N, Letur H, Lelannou D, et al. Materno-fetal cardiovascular
cycles should receive counseling regarding contraception/timing of complications in Turner syndrome after oocyte donation: Insufficient
pregnancy and increased risk of POI including a discussion of prepregnancy screening and pregnancy follow-up are associated with poor
outcome. J Clin Endocrinol Metab. 2011;96(2):E260-E267.
oocyte/embryo cryopreservation. Spontaneous pregnancies (2 8% 20. Ullrich O. Uber typische kombinationsbilder multipler Abartung.
of women with TS) usually occur in women with mosaic Zeitschrift fur Kinderheilkunde. 1930;49: 271-276.

The Endocrine Society. Downloaded from press.endocrine.org by [${individualUser.displayName}] on 12 January 2017. at 12:19 For personal use only. No other uses without permission. . All rights reserved.
The Endocrine Society. Downloaded from press.endocrine.org by [${individualUser.displayName}] on 12 January 2017. at 12:19 For personal use only. No other uses without permission. . All rights reserved.
THYROID/HPT AXIS

The Endocrine Society. Downloaded from press.endocrine.org by [${individualUser.displayName}] on 12 January 2017. at 12:19 For personal use only. No other uses without permission. . All rights reserved.
330 ENDO 2016 MEET-THE-PROFESSOR CLINICAL CASE MANAGEMENT

Thyroid Function Tests That Do Not Make Sense

CMF3 Recognize when serum TSH and thyroid hormone levels


Presented, April 1 4, 2016 are incongruent with each other and with the clinical
status of the patient.
Understand how thyroid function tests can be difficult to
Virginia D. Sarapura, MD; Jacqueline Jonklaas, MD, interpret when there are ongoing changes in thyroid status.
PhD. University of ColoradoDenver, Denver, Colorado Describe the effect that illness and hospitalization can
80309, E-mail: virginia.sarpura@ucdenver.edu; and have on thyroid function tests.
Georgetown University Medical Center, Washington, District Develop a strategy for determining whether thyroid
of Columbia 20007, E-mail: jonklaaj@georgetown.edu function test inconsistencies are due to a complex, dynamic
clinical situation; nonthyroidal illness; inaccurate
INTRODUCTION measurement of TSH or thyroid hormones; protein-binding
Historical Overview alterations; confounding medications; or other issues.
Laboratory assessment using thyroid function tests has im-
proved over the years, with the ability to measure TSH greatly
STRATEGIES FOR DIAGNOSIS, THERAPY,
adding to our ability to diagnose and treat thyroid disease.
AND/OR MANAGEMENT
More sensitive TSH assays have allowed clinicians to distin-
Techniques for measuring TSH concentrations are generally
guish degrees of hyperthyroidism and TSH suppression. Cur-
accurate but can be affected by antibodies that cause TSH
rent immunoassays used to measure thyroid hormones have
levels to be falsely elevated (1, 2). Heterophilic antibodies
several limitations that may be circumvented by measuring
against glycoprotein hormones that cause falsely high TSH
thyroid hormones using tandem mass spectrometry methods.
values during laboratory testing have been described. Often
These advances in our methodology must be combined with
such patients have a history of exposure to animals in their jobs
thorough clinical investigation.
or home environment. The true TSH value can be obtained via
laboratory analysis by using nonimmune mouse IgG or appro-
SIGNIFICANCE OF THE CLINICAL PROBLEM priate blocking antibodies. Macro-TSH is a situation in which
The thyroid status of a patient can be assessed by examining the TSH combines with an immunoglobulin and produces a com-
clinical presentation of the patient and by comparing this with a plex that cannot be filtered by the kidneys (3, 4). This complex
biochemical laboratory assessment of the patient, aided where accumulates in the serum and remains immunoactive so that it
necessary by additional diagnostic tests. When there is discor- can be measured in TSH assays but is biologically inactive. In
dance between the clinical and biochemical assessment of the all these situations a clinically euthyroid patient may be sus-
patient, careful consideration of potential interfering factors is pected of having hypothyroidism due to the elevated TSH.
necessary. Additional testing and repeat thyroid testing may be TSH values may also be elevated in other clinical situations
necessary. Failure to be able to appreciate the true clinical status such as resumption of levothyroxine after a period of nonad-
of the patient may result in necessary treatment being withheld or
herence, pituitary resistance to thyroid hormone, or TSH-
unnecessary treatment being given.
secreting pituitary adenomas (2, 5, 6). Routine TSH immunoas-
says measure TSH concentrations. Assays for the bioactivity of
BARRIERS TO OPTIMAL PRACTICE TSH can also be performed using cell culture systems. If serum
The complexity of the hypothalamic pituitary thyroid axis, TSH concentration is directly related to bioactivity, these two
particularly where there is a fluctuating thyroid status. measurements should be concordant. However, under certain
Adaptations in the thyroid axis during illness and circumstances such as aging, they may become discordant and
starvation. TSH concentration may be higher than its bioactivity (7). TSH
Limitations in the free thyroid hormone immunoassays levels may also be falsely lowered by several medications such
that are used to measure free thyroid hormones, as dopamine, high dose steroids, or chronic opioid use. Further-
particularly in specific situations such as alterations in more, serum TSH can lag behind the clinical picture, such as
protein binding. when a patient treated for Graves disease with radioiodine
The presence of antibodies that can interfere with the therapy has developed low thyroid hormone concentrations but
measurement of either TSH or thyroid hormones. concurrently still has a low TSH value (8).
Thyroid hormone assays should be accurate and reflect the
LEARNING OBJECTIVES concentration of hormone in the sample. Barriers to accurate
As a result of participating in this session learners should be measurement using immunoassays can include changes in
able to: binding proteins, presence of heterophilic antibodies, and con-

The Endocrine Society. Downloaded from press.endocrine.org by [${individualUser.displayName}] on 12 January 2017. at 12:19 For personal use only. No other uses without permission. . All rights reserved.
ENDO 2016 THYROID/HPT AXIS 331

centration of nonesterified free fatty acids. Such factors may increased T3 catabolism due to changes in the deiodinases in
account for the method-dependent variation documented in peripheral tissues. The usual differential diagnosis, when con-
thyroid hormone measurements made during such physiologic and fronted with this spectrum of abnormalities, is central hypothy-
medical conditions as pregnancy, renal failure, nonthyroidal ill- roidism. The latter benefits from treatment with levothyroxine,
ness, and genetic abnormalities in binding proteins (1, 2, 5, 9). whereas nonthyroidal illness does not.
Discrepancies between thyroid hormone assays can be demon-
strated by examining the correlation between the results obtained
MAIN CONCLUSIONS
when different assays are used to measure the thyroid hormone
The first task when confronted with a patient with confusing
levels in the same sample. Another means of judging the validity
thyroid function tests is to accurately assess the patients clini-
of thyroid hormone measurements is to examine the relationship
cal situation to the extent possible. Sometimes this is difficult
between thyroid hormone concentration and the logarithmically
to achieve because of concurrent illnesses or other complicat-
transformed TSH value. This relationship generally follows an
ing factors. The constellation of data should then be consid-
inverse linear relationship. However, this relationship is quite
ered, with an effort to determine which piece of data is the
complex and can potentially be affected by age and sex (10).
outlier or is inaccurate. Sometimes following the patient and
The performance of thyroid hormone assays is of impor-
the laboratory tests over time may resolve the discrepancies.
tance across the whole range of values. However, assay perfor-
Sometimes additional testing or specialized testing may pro-
mance may be particularly important at the low and high values
vide insight and resolve the clinical uncertainty.
for thyroid hormones, as it is at these two extremes that
presence of thyroid disorders is more likely. Erroneous values
for thyroid hormones may provide incongruent laboratory val- CASES
ues that prevent the correct and timely diagnosis of hypothy- Case 1
roidism or hyperthyroidism. This could add additional com- A 21-year old female presented to an emergency department
plexity in cases that are already difficult or challenging based with anxiety. Her thyroid gland was of normal size and texture
on their clinical complexity. Measurement by tandem mass without evidence of palpable nodules. She was given a diagno-
spectrometry is a technique that is accurate, precise, and more sis of Graves disease based on elevated free thyroid hormone
specific than immunoassays (11, 12). When coupled with levels despite the fact that her serum TSH and total thyroid
physical separation methods it permits the reliable measure- hormones were normal. She was discharged from the emer-
ment of free thyroid hormone in any of the conditions that may gency department with a prescription for methimazole. She
result in changes in binding protein concentrations (11). These was seen by a series of endocrinologists and treated with
situations could include, for example, pregnancy and nonthyroidal various doses of methimazole. Her free thyroid hormone levels
illness (13, 14). remained elevated and her anxiety levels fluctuated. An endo-
Illness and malnutrition are situations in which patients crinologist referred her for thyroidectomy with nonadherence
with normal thyroid function may experience progressive to her medication being mentioned in the referral. The surgeon
depression of the thyroid axis, such that circulating T3 questioned the need for thyroidectomy. Representative labora-
concentration decreases, followed by declines in serum tory tests preceding her surgical referral are shown in Table 1.
TSH, and then declines in FT4 (13). There are changes in Which of the following tests is most likely to provide insight
thyroid hormone binding and decreased T4 to T3 conversion into this patients thyroid status?
due to the effects of endogenous circulating free fatty acids, A. A magnetic resonance image of the pituitary
cortisol, cytokines, and various other drugs. There may also be B. A detailed documentation of family history

Table 1. Data for Case 1


Date TSH (0.4-4.5 mIU/L) FT4 (0.82-1.77 ng/dL) T4 (4.5-12 mcg/dL) FT3 (nl 2.3-5.0 pg/mL) T3 (71-180 ng/dL) MMI
9/14 0.8 1.79 182 MMI
11/14 1.1 1.9 178 MMI
1/15 0.656 2.18 5.5 188 ?
2/15 0.56 2.01 213 ?
2/15 1.25
4/15 0.616 2.38 6.1 9.1 184 Off MMI
6/15 0.636 2.27 5.9 6.7 167 Off MMI
7/15 0.805 2.63 7.0 202

Abbreviation: MMI, methimazole.

The Endocrine Society. Downloaded from press.endocrine.org by [${individualUser.displayName}] on 12 January 2017. at 12:19 For personal use only. No other uses without permission. . All rights reserved.
332 ENDO 2016 MEET-THE-PROFESSOR CLINICAL CASE MANAGEMENT

C. A radioactive iodine uptake and scan Case 4


D. Measurement of thyroid peroxidase antibodies A 60-year-old female with a history of vitiligo and Graves
E. Measurement of free thyroid hormones by tandem mass disease was treated with radioactive iodine many years ago,
spectrometry and subsequently became hypothyroid. She was stable on treat-
ment with levothyroxine 125 mcg daily. She then presented in
Case 2 September 2013 with a low TSH of 0.18 mIU/L and a FT4 of
A 61-year-old male underwent a nephrectomy for renal cell 2.76 ng/dL, and the dose of levothyroxine was decreased to
carcinoma. His postoperative course was complicated and in- 100 mcg daily. Follow-up tests were as shown in Table 2 .
cluded development of hospital-acquired pneumonia. His After her labs in May 2015, she was referred to the Endocrine
medications in the hospital included iv antibiotics, sc Clinic and was seen in August 2015. The patient reported
enoxaparin, and oral oxycodone. Thyroid function tests were feeling well, she had no complaints. She had a normal pulse of
68 beats per minute, no tremor, normal deep tendon reflexes,
obtained when the patient developed some agitation. The thy-
and except for being overweight her examination was normal.
roid function tests showed the following: FT4, 3.1 ng/dL (39.9
Medications included levothyroxine 100 mcg daily and multi-
pmol/L); TSH, 4.14 mIU/L; total T3, 91 ng/dL (1.4 nmol/L);
vitamins.
and rT3, 39 ng/dL (0.6 nmol/L). The patients agitation re-
1. What could be causing simultaneously high TSH and FT4?
solved once treatment was initiated for the patients pneumo-
2. Could the FT4 be high due to recurrence of Graves
nia. The patient had no prior history of, or current symptoms
disease?
of, hyperthyroidism. His physical examination revealed a
3. Could the patient have a falsely high FT4?
normal-size thyroid without palpable nodules. There were no
4. Could the TSH be high due to under-replaced
stigmata of thyrotoxicosis. Repeated laboratory assessment of
hypothyroidism?
his thyroid function was as follows: FT4, 3.6 ng/dL (46.3
5. Could the TSH be falsely high?
pmol/L); TSH, 5.74 mIU/L; total T3, 85 ng/dL (1.3 nmol/L);
and total T4, 9.3 g/dL (119.7 nmol/L).
Which of the following diagnoses would best explain the DISCUSSION OF CASES AND ANSWERS
patients abnormal thyroid test results? Case 1
This is a case in which the patient had a normal serum TSH, a
A. Nonthyroidal illness
normal thyroid examination, and some symptoms that overlap
B. Opiate suppression of the pituitary gland
with hyperthyroidism. Based on high free thyroid hormone
C. Artifactual increase in measured serum FT4
concentrations, she was treated with methimazole for several
concentration
years. The patient had never had a low TSH documented.
D. Inhibition of the type 2 deiodinase
Either her TSH is being erroneously measured as normal, or
E. Recovery from nonthyroidal illness
her free thyroid hormone levels are artifactually high. Mea-
surement of free thyroid hormone levels by tandem mass
Case 3 spectrometry demonstrated normal values. This suggests
45-year-old female presents with complaints of fatigue and weight that some substance, such as interfering antibodies, are caus-
gain. Several relatives had been diagnosed with Graves disease or ing false elevation of the patients free thyroid hormones.
hypothyroidism. She had a TSH of 15 mIU/L (normal, 0.34-6
mIU/L), and was started on levothyroxine, 125 mcg daily by her
Case 2
primary care physician. Her TSH remained above normal and the This patient has no history of thyroid disorders, is clinically
dose of levothyroxine was gradually increased to 200 mcg daily. euthyroid, and has a normal TSH. However, he has an elevated
On this dose, the TSH level was 14 mIU/L. A levothyroxine FT4 level. This pattern of abnormalities is not consistent with
absorption test was performed and this was normal. The patient the changes in thyroid function associated with nonthyroidal
was then referred to the Endocrine Clinic. She reported no signifi- illness given that the T3 and TSH levels are not low. Further-
cant change in her symptoms. She had a pulse of 92 beats per more, the FT4 is actually elevated. Several medications affect
minute and seemed anxious. Test showed a TSH of 16 mIU/L,
high FT4 of 3 ng/dL (normal, 0.5-1.9 ng/dL) and negative TPO
antibodies. Prolactin and pituitary MRI were normal, and Table 2. Data for Case 4
levothyroxine was discontinued. A month later, the FT4 normal- TSH FT4
Date 0.34-6 0.5-1.9 Levothyroxine
ized and the TSH level remained high.
Normal Ranges mIU/L ng/dL Daily Dose, mcg
1. What are the potential causes of a high TSH and
9/2013 0.18 2.76 125
normal or high FT4?
3/2014 2.03 2.53 100
2. Could the TSH be high due to subclinical
5/2015 7.29 2.53 100
hypothyroidism?
8/2015 9.44 2.27 100
3. Could the TSH be falsely high?

The Endocrine Society. Downloaded from press.endocrine.org by [${individualUser.displayName}] on 12 January 2017. at 12:19 For personal use only. No other uses without permission. . All rights reserved.
ENDO 2016 THYROID/HPT AXIS 333

thyroid function in vitro. Intravenously or subcutaneously ad- with levothyroxine absorption; however, she was taking multivi-
ministered heparin can falsely elevate measured FT4 values. tamins 8-12 hours apart, and was lacking any signs or symptoms
The mechanism is thought to involve nonesterified fatty acids. of hypothyroidism. Further investigation showed that the hetero-
Heparin mobilizes lipoprotein lipase from the endothelium in phil anti-mouse antibodies were negative, and macroTSH was
vivo. During sample storage or incubation, the lipoprotein suspected to be the cause for the abnormal TSH.
lipase then acts to generate nonesterified fatty acids from
triglycerides in the sample. The nonesterified fatty acids then REFERENCES
displace T4 binding from T4-binding globulin, resulting in an in 1. Despres N, Grant AM. Antibody interference in thyroid assays: A potential
vitro increase in FT4. for clinical misinformation. Clin Chem. 1998;44(3):440-454.
2. Koulouri O, Moran C, Halsall D, Chatterjee K, Gurnell M. Pitfalls in the
measurement and interpretation of thyroid function tests. Best Pract Res
Case 3 Clin Endocrinol Metab. 2013;27(6):745-762.
This patient had a high TSH and a strong family history of 3. Loh TP, Kao SL, Halsall DJ, et al. Macro-thyrotropin: A case report and
autoimmune thyroid disease, with nonspecific symptoms con- review of literature. J Clin Endocrinol Metab. 2012;97(6):1823-1828.
4. Mendoza H, Connacher A, Srivastava R. Unexplained high thyroid stimu-
sistent with hypothyroidism, and the high TSH level prompted lating hormone: A BIG problem. BMJ case reports. 2009;10.1136/bcr.
initiating treatment with levothyroxine. However, her TSH 01.2009
remained high and her symptoms did not improve. She was not 5. Gurnell M, Halsall DJ, Chatterjee VK. What should be done when thyroid
function tests do not make sense? Clin Endocrinol (Oxf). 2011;74(6):673-
taking any medications that could have interfered with absorp-
678.
tion, and a levothyroxine absorption test was normal. There 6. England ML, Hershman JM. Serum TSH concentration as an aid to
was no evidence of a TSH-secreting pituitary adenoma and no monitoring compliance with thyroid hormone therapy in hypothyroidism.
family history of thyroid hormone resistance. After increasing Am J Med Sci. 1986;292(5):264-266.
7. Estrada JM, Soldin D, Buckey TM, Burman KD, Soldin OP. Thyrotro-
the dose of levothyroxine, not only did her TSH not improve,
pin isoforms: Implications for thyrotropin analysis and clinical prac-
but she developed anxiety and her FT4 level became high. tice. Thyroid. 2014;24(3):411-423.
Further investigation revealed that the cause of the high TSH 8. Uy HL, Reasner CA, Samuels MH. Pattern of recovery of the
was interference with the TSH assay due to the presence of hypothalamic-pituitary-thyroid axis following radioactive iodine therapy in
patients with Graves disease. Am J Med. 1995;99(2):173-179.
heterophil anti-mouse antibodies. 9. Klee GG. Interferences in hormone immunoassays. Clin Lab Med.
2004;24(1):1-18.
Case 4 10. Hadlow NC, Rothacker KM, Wardrop R, Brown SJ, Lim EM, Walsh JP.
The relationship between TSH and free T(4) in a large population is
This patient has a history of Graves disease and has postabla- complex and nonlinear and differs by age and sex. J Clin Endocrinol
tive hypothyroidism. Two years before presentation, she had a Metab. 2013;98(7):2936-2943.
low TSH with a similar FT4, so a TSH-secreting pituitary 11. van Deventer HE, Mendu DR, Remaley AT, Soldin SJ. Inverse log-linear
adenoma or thyroid hormone resistance are not likely to have relationship between thyroid-stimulating hormone and free thyroxine mea-
sured by direct analog immunoassay and tandem mass spectrometry. Clin
developed within this timeframe. She has no symptoms or Chem. 2011;57(1):122-127.
signs of hyperthyroidism, or any evidence of a goiter, to sug- 12. Soldin OP, Soldin SJ. Thyroid hormone testing by tandem mass spectrom-
gest recurrent Graves disease. Total T4 and T3 resin uptake etry. Clin Biochem. 2011;44(1):89-94.
13. Van den Berghe G. Non-thyroidal illness in the ICU: A syndrome with
were midnormal, so the high FT4 was attributed to assay
different faces. Thyroid. 2014;24(10):1456-1465.
interference, as in Case 1. The high TSH could have been 14. Lee RH, Spencer CA, Mestman JH, et al. Free T4 immunoassays are
attributed to under-replaced hypothyroidism, or interference flawed during pregnancy. Am J Obstet Gynecol. 2009;200(3):260.e1 e6.

The Endocrine Society. Downloaded from press.endocrine.org by [${individualUser.displayName}] on 12 January 2017. at 12:19 For personal use only. No other uses without permission. . All rights reserved.
334 ENDO 2016 MEET-THE-PROFESSOR CLINICAL CASE MANAGEMENT

Thyroid Cancer: Advanced Cases

CMF9 BARRIERS TO OPTIMAL PRACTICE


Presented, April 1 4, 2016 Advanced thyroid cancer cases are uncommon compared
patients with low or intermediate thyroid cancer, making
it more difficult for general practitioners to develop
Mona Sabra, MD; Bryan Haugen, MD. Memorial Sloan expertise in their care.
Kettering Cancer Center, New York City, New York Management of advanced thyroid cancer requires the
10065, E-mail: sabram@mskcc.org; Department of participation of a multidisciplinary group of specialists
Medicine, University of Colorado Medical School, Aurora, commonly available in referral centers.
Colorado 80045, E-mail: bryan.haugen@ucdenver.edu With the advent of new effective therapies for the
management of these advances cases, endocrinologists
INTRODUCTION and oncologists are still working on defining the effect
Historical Overview of these agents on the patients disease course, overall
For more than 70 years, radioactive iodine (RAI) therapy has survival, and quality of life.
been the main treatment for advanced thyroid cancer after
surgical resection. It was Sam Siedlin, MD, who, in 1943, LEARNING OBJECTIVES
treated a patient with widely metastatic follicular thyroid can- As a result of participating in this session, learners should be
cer with repeat activities of RAI and demonstrated complete able to:
remission of the tumor. This opened the door to the wide- Recognize the importance of a multidisciplinary
spread use of RAI for the treatment of metastatic thyroid approach to the management of the advanced thyroid
cancer. In 1980s to late 2000s, ample data demonstrated that cases.
overall survival of patients with metastatic thyroid cancer was Define the role and limitations of repeat-RAI therapies in
significantly different depending on their presenting clinico- the management of metastatic thyroid cancer.
pathologic features and their tumors ability to respond to RAI Define the role and toxicities of localized therapies for
(2, 3). Concomitantly, molecular studies demonstrating the specific cases of progressive metastatic thyroid cancer.
importance of driver mutations of the MAPK kinase, PIK3CA Define the indications, toxicities and role of novel
pathways in determining a given tumors differentiation and molecular targeted therapies in the management of
ability to produce thyroglobulin and to concentrate RAI (4). thyroid cancer.
Recently, there have been strong interests in the use of
molecular-targeted therapies, which block various aspects of
STRATEGIES FOR DIAGNOSIS, THERAPY,
the MAPK/PIK3CA pathways and vascular endothelial sys-
AND/OR MANAGEMENT
tems, for the management of patients with metastatic RAI
At the outset, it is important to note that the current approach to
refractory progressive thyroid cancer.
the management of advanced thyroid cancer cases is limited to
those patients presenting with tumors of follicular origin, ex-
SIGNIFICANCE OF THE CLINICAL PROBLEM cluding those with anaplastic thyroid cancer and medullary
Advanced thyroid cancer cases, including those presenting with thyroid cancer.
locally advanced disease or distant metastatic thyroid cancer, Advanced thyroid cancer cases have varied clinical pre-
compromises 10 15% of all thyroid cancer cases (5). Only 4% sentation that ranges from those with isolated locally inva-
of patients with thyroid cancer present with distant metastasis sive thyroid cancer, those with widely metastatic thyroid
at thyroid cancer diagnosis. Given the relatively uncommon cancer with or without locally invasive disease, and with or
presentation, many general endocrinologists do not see enough without critical organ involvement (eg, spine metastasis
of these advanced cases to develop expertise. In addition, the with cord compression, bone metastasis with pathologic
management of these cases requires the participation of a fracture, brain metastasis).
multidisciplinary group of specialists including endocrinolo- For patients with the locally advanced thyroid cancer, it is
gists, radiologists, nuclear medicine specialists, surgeons, ra- important to involve a group of specialists (endocrinologists,
diation oncologists, and medical oncologists, for the complex radiation oncologist and surgeons) early on in the treatment
evaluation and management of treatment-related issues that decision making, to determine the best strategy to deliver local
arise when dealing with these patients. Furthermore, with the disease control with the least morbidity and long-term effects
advent of novel therapies, it is important to establish specific on the patients quality of life.
criteria to determine which patients are likely to benefit for For patients presenting with critical organ involvement, it is
treatment with these systemic therapies. important to offer local control of the disease at the critical

The Endocrine Society. Downloaded from press.endocrine.org by [${individualUser.displayName}] on 12 January 2017. at 12:19 For personal use only. No other uses without permission. . All rights reserved.
ENDO 2016 THYROID/HPT AXIS 335

organ prior to treating the patient as a whole. Thus, airway 1. The metastatic disease does not concentrate RAI at the
involvement (trachea, larynx, major bronchi) should be ad- first diagnostic or therapeutic whole-body scan.
dressed in priority. Patients with bone metastasis with impend- 2. The metastatic disease loses the ability to concentrate
ing fracture or pathologic fracture should undergo embolization iodine after initial evidence of RAI avidity.
followed by surgical resection and internal fixation radiation 3. The metastatic disease shows structural and/or
therapy prior to proceeding with thyroidectomy and RAI biochemical disease progression within 6 12 months
therapy. Those presenting with large brain metastasis would after RAI therapy.
benefit from surgery and/or radiation therapy to the brain prior 4. The metastatic disease shows RAI avid in some but
to thyroidectomy. Similarly, patients presenting with base of not most foci. This is usually presents in patients with
the skull metastasis or spina l metastasis with cord compression FDG-PET-avid metastatic foci that coexists with more
or who may be at risk for cord compression with prolonged RAI-avid foci.
hypothyroidism and RAI therapy should be evaluated by neu- In patients with progressive metastatic thyroid cancer, the treat-
rosurgery and radiation oncology prior to considering RAI ment options are not limited to repeat-RAI therapies. As such,
therapy. in some instances, patients present with isolated recurrent or
Total thyroidectomy followed by high-dose RAI therapy progressive disease in the neck with stable distal metastasis.
remains the most effective treatment option for patients pre- These patients can be treated with surgical resection of the
senting with distant metastatic thyroid cancer. Having said that, neck disease with the understanding that the surgery is not
that treatment strategy is not curative and most patients with curative but will reduces the overall burden of cancer and
distant metastatic thyroid cancer ultimately demonstrate struc- protect against loco-regional disease progression and associ-
tural disease progression during their followup. The estimated ated potential quality-of-life consequences with continued dis-
overall 10, 15, and 25-year survival of patients with distant ease progression. Alternatively, ethanol ablation may be used
metastatic thyroid cancer are 42, 33, and 29%, respectively (2). to treat selected cases of small progressive nodal metastasis in
Patients with RAI avid disease, particularly in younger patients patients who are not surgical candidates. Patients with isolated
(age 40 y at distant metastasis diagnosis), those with distant metastasis or with rapid progression in a single meta-
subcentimeter pulmonary metastases, and those with non- static focus can be treated with metastasectomy and/or radia-
fluorodeoxyglucosepositron emission tomography (FDG- tion therapy and/or embolization for local control. The treat-
PET) avid metastatic lesions have been shown to have a sur- ment goals in those instances are:
vival benefit over those with RAI refractory, FDG-PET avid Disease control, thus prolonging progression-free
disease with macronodular lung involvement, and those with survival (PFS) and delaying the use of molecular
osseous metastasis (2, 3, 6). Despite being an effective therapy, targeted therapies.
repeat RAI therapy is not curative in patients with distant Symptomatic relief (pain for bone metastasis, hemoptysis
metastases with more than half of the patients demonstrating for lung metastasis with bronchial involvement).
structural disease progression on followup (7). Prevention for critical organ injury (such as cord
RAI therapy is still offered at least once at the diagnosis of compression, postobstructive pneumoniae).
distant metastasis from thyroid cancer. It is important to estab- When proceeding with local treatment modalities, the benefits
lish a treatment response to that initial therapy using RECIST- of the chosen intervention must be weighed carefully against
like criteria prior to offering repeat RAI therapy, recognizing procedural risks and the overall morbidity to the patient. Pa-
that most patients will not respond to RAI. Here, it is important tients who exhibit multiple sites of RAI-refractory (RAI-R)
to ensure that prior therapies were conducted with appropriate progressive metastatic cancer may be more appropriate for
TSH stimulation and low-iodine-diet preparation without acci- systemic therapy.
dental iodine contamination. Not all patients with RAI-R metastatic thyroid cancer
For those patients with significant tumor shrinkage and/or benefit from systemic therapy. Patients with stable disease
significant decrease in thyroglobulin, it is reasonable to repeat burden and those with slowly progressive disease may not
RAI therapy in 1218 months after the initial treatment to benefit from further therapy until they demonstrate rapid
maximize the treatment benefit. structural progression.
It is crucial to identify those patients with metastatic thyroid Patients who have clearly progressive, RAI-R differentiated
cancer who are resistant to conventional RAI therapies early thyroid cancer (DTC) can be considered for systemic targeted
on, thus limiting the risks of morbidity from elevated cumula- therapy, especially if the disease is larger volume, rapidly
tive RAI activities and offering them alternative potentially progressive, and/or symptomatic. It can be quite difficult to
more effective treatment options. The just-published 2015 determine the optimal time to start systemic targeted therapy.
American Thyroid Association guidelines for the management Careful informed consent and engagement of the patient in the
of thyroid nodules and thyroid cancer propose the following decision-making process is important. Options for systemic
definition for RAI-refractory metastatic thyroid cancer (5): targeted therapy are U.S. Food and Drug Administration (FDA)

The Endocrine Society. Downloaded from press.endocrine.org by [${individualUser.displayName}] on 12 January 2017. at 12:19 For personal use only. No other uses without permission. . All rights reserved.
336 ENDO 2016 MEET-THE-PROFESSOR CLINICAL CASE MANAGEMENT

approved drugs (currently lenvatinib and sorafenib), entry into tology of the dominant left thyroid nodule was interpreted as
clinical trials, and off-label use of directed therapies approved atypical, cytology of the right dominant node interpreted as
for other cancers that have shown some benefit in RAI-R DTC benign. PET scan shows an additional 3.6-cm PET-avid right
in smaller trials. Use of FDA-approved drugs should be ex- iliac metastasis (SUV 7) with cortical thinning. She was treated
plored first unless there are specific circumstances that warrant with complete resection of the humerus metastasis with nega-
consideration of the other approaches. Genetic approaches to tive margin and internal fixation followed by total thyroidec-
precision oncology looking for actionable targets (ie, tomy. The final pathology showed a 1.5-cm encapsulated
BRAFV600E, Alk fusions, etc.) will likely be standard ap- poorly differentiated thyroid cancer of follicular phenotype
proaches in the future. with extensive vascular invasion. Her 6 weeks postoperative
Sorafenib was shown in a placebo-controlled trial to result thyroglobulin level was 23, 450 ng/mL with negative
in a 12% partial response rate (30% tumor shrinkage) and an antithyroglobulin antibody and TSH 0.02 mIU/L.
increased PFS of 5 months compared with placebo (8). What is the next step in her management?
Lenvatinib was shown in a similarly designed placebo- A. RAI after appropriate thyroid hormone withdrawal.
controlled trial to result in a 63% partial response rate and an B. RAI after recombinant human TSH (rhTSH)
increased PFS of nearly 15 months compared with placebo (9). stimulation.
The patient populations were different in the two groups, so C. Systemic therapy with multitarget tyrosine kinase
direct comparison of the response rates and PFS should be inhibitor.
D. Intravenous bisphosphonate.
interpreted with caution. Toxicities and the need for dose
E. A localized therapy with either surgery or external
reductions were common in both studies, which is why these
beam irradiation.
drugs should generally not be used in patients with relatively
low-volume disease that is asymptomatic, even with some slow Case 2
growth. A 56-year-old male owner of a floor refinishing business was
Patients who fail FDA-approved therapies should next be diagnosed in 2009 with a 3.8-cm Hurthle cell carcinoma (HCC)
considered for clinical trials. If a patient cannot enter a and lung nodules (biopsy HCC). Following a thyroidectomy
clinical trial, off-label therapy with targeted therapies show- and central neck dissection (0/8 positive nodes, gross
ing some benefit in Phase II clinical trials in RAI-R DTC extrathyroid invasion, T4aN0M1), he received 200-mCi 131I
may be considered (pazopanib, sunitinib, axitinib, vandetanib). with uptake in the neck, but no uptake in the lung lesions. This
If a patient progresses on one targeted therapy, another targeted was followed by 62 Gy of external beam radiation (EBRT) to
therapy can be considered given that each therapy has a some- his thyroid bed region. PET/CT revealed PET-avid lung lesions
what different target profile (5). and small PET-avid lesions in the bone (L2, L inferior is-
chium). He was asymptomatic.
What is the best approach to his management?
MAIN CONCLUSIONS
A. More radioiodine
A multidisciplinary approach is needed for the optimal
B. External radiation of the lung and bone lesions.
management of advanced thyroid cancer cases.
C. Close monitoring on TSH suppression therapy,
RAI therapy is rarely curative in patients with advanced
consider systemic bone-directed therapy.
thyroid cancer. It is crucial to define the tumor ability to
D. Start systemic therapy (kinase inhibitor).
concentrate RAI before repeat RAI therapy.
E. Enter him into a clinical trial.
Local treatment modalities can provide symptomatic
relief and delay the use of molecular targeted therapies He was monitored every 4 months over the next few years on
in a subset of patients with progressive advanced thyroid iv bisphosphonates and had continued PET-positive disease
cancer. progression, with new lesions in the bone and growing lesions
Novel molecular therapies can provide an effective in the lung.
alternative for properly selected patients with rapidly What is the best approach to his management?
progressive RAI refractory advanced thyroid cancer. A. More radioiodine
B. External radiation of the lung and bone lesions.
C. Continued close monitoring on TSH suppression
CASES WITH QUESTIONS therapy and systemic bone-directed therapy.
Case 1 D. Start systemic therapy (kinase inhibitor).
A 58-year-old nurse presents with severe left humerus pain E. Enter him into a clinical trial.
after helping a patient. X-ray of the left humerus shows a
5.7-cm lytic lesion in the left humerus with pathologic fracture. DISCUSSION OF CASES AND ANSWERS
Biopsy is compatible with metastatic follicular thyroid cancer. Case 1
Neck ultrasound showed bilateral thyroid nodules and inde- This is a classic case of distant metastasis from follicular
terminate right neck nodes. Fine needle aspiration biospy cy- thyroid cancer. Follicular thyroid cancers with extensive vas-

The Endocrine Society. Downloaded from press.endocrine.org by [${individualUser.displayName}] on 12 January 2017. at 12:19 For personal use only. No other uses without permission. . All rights reserved.
ENDO 2016 THYROID/HPT AXIS 337

cular invasion are at high risk for distant metastasis. They are or threatening lesions. Systemic bisphosphonates (generally
often associated with RAS tumor mutations, produce a lot of zoledronic acid iv, 4 mg every 13 mon or denosumab sq, 120
thyroglobulin, and are more likely to respond to RAI then mg every mo) should be considered for patients with bone
BRAF-mutated tumors. It was important to get a bone biopsy metastases. Once the disease becomes clearly progressive and
at diagnosis to rule out other nonthyroid-related malignancies. larger volume or symptomatic where directed therapy cannot
The next step in her management is to consider either help, systemic therapy generally starting with the FDA-
surgery and/or radiation therapy for the weight-bearing me- approved lenvatinib or sorafenib, should be considered. Clini-
tastasis at risk for fracture (given associated cortical thin- cal trials or off-label kinase inhibitor therapy can be considered
ning). The decision to proceed with either therapy includes for those patients for whomapproved therapies (progression or
the involvement of the orthopedic surgeon, radiation oncolo- significant toxicity) fail. Careful attention to and prevention of
gist, and nuclear medicine. The hip metastasis is likely to adverse effects to the systemic therapy is important. It is also
swell with prolonged hypothyroidism, rhTSH, and RAI important to inform patients that these drugs are almost never
therapy, putting the patient at risk for fracture. If a fracture curative, but can prolong PFS in some patients.
is not felt to be imminent and the patient is not a candidate
for internal fixation, we often treat these lesions with radia- REFERENCES
1. Siegel E. The beginnings of radioiodine therapy of metastatic thyroid
tion therapy prior to RAI therapy.
carcinoma: a memoir of Samuel M. Seidlin, M. D. (1895-1955) and his
Once the localized therapy is completed consideration celebrated patient. Cancer Biother Radiopharm. 1999;14:71-79.
should be made for both RAI scanning and RAI therapy 2. Durante C, Haddy N, Baudin E, et al. Long-term outcome of 444 patients
with distant metastases from papillary and follicular thyroid carcinoma:
under hypothyroid conditions and after appropriate low- Benefits and limits of radioiodine therapy. J Clin Endocrinol Metab.
iodine preparation. rhTSH use is not FDA approved for the 2006;91:2892-2899.
management of thyroid cancer. Thyroglobulin level is ex- 3. Robbins RJ, Wan Q, Grewal RK, et al. Real-time prognosis for metastatic
thyroid carcinoma based on 2-[18F]fluoro-2-deoxy-D-glucose-positron emis-
pected to decrease dramatically after RAI therapy. The bone sion tomography scanning. J Clin Endocrinol Metab. 2006;91:498-505.
lesions may remain stable. The soft-tissue component may 4. Carneiro RM, Carneiro BA, Agulnik M, Kopp PA, Giles FJ. Targeted
shrink in response to RAI. therapies in advanced differentiated thyroid cancer. Cancer Treat Rev.
2015;41:690-698.
In the long run, the patient would benefit from iv 5. Haugen BR, Alexander EK, Bible KC, et al. 2015 American Thyroid
bisphosphonate or rank ligand inhibitor therapy to prevent Association Management Guidelines for adult patients with thyroid nod-
skeletal-related events for bone metastasis related to ules and differentiated thyroid cancer. Thyroid. 2015 [Epub ahead of print].
6. Chopra S, Garg A, Ballal S, Bal CS. Lung metastases from differentiated
thyroid cancer. thyroid carcinoma: Prognostic factors related to remission and disease-free
survival. Clin Endocrinol. 2015;82:445-452.
7. Sabra MM, Dominguez JM, Grewal RK, et al. Clinical outcomes and
Case 2
molecular profile of differentiated thyroid cancers with radioiodine-avid
Patients with Hurthle cell carcinoma are often quite PET posi- distant metastases. J Clin Endocrinol Metab. 2013;98:829-836.
tive and radioiodine is of limited benefit for metastatic disease, 8. Brose MS, Nutting CM, Jarzab B, et al. Sorafenib in radioactive iodine-
although one good attempt at RAI therapy is reasonable. Pa- refractory, locally advanced or metastatic differentiated thyroid cancer: A
randomised, double-blind, phase 3 trial. Lancet. 2014;384:319-328.
tients who are RAI refractory can be followed closely with 9. Schlumberger M, Tahara M, Wirth LJ, et al. Lenvatinib versus placebo in
directed therapy (surgery, EBRT, thermal ablation) for painful radioiodine-refractory thyroid cancer. N Engl J Med. 2015;372:621-630.

The Endocrine Society. Downloaded from press.endocrine.org by [${individualUser.displayName}] on 12 January 2017. at 12:19 For personal use only. No other uses without permission. . All rights reserved.
338 ENDO 2016 MEET-THE-PROFESSOR CLINICAL CASE MANAGEMENT

Nonthyroidal Illness Syndrome: To Treat or Not?

CMF12 tify those patients who would likely benefit from treatment. A
Presented, April 1 4, 2016 delay in diagnosis and treatment of primary thyroid disease can
result in adverse outcomes. Special populations have been
identified that may possibly benefit from thyroid hormone
Gregory A. Brent, MD; Alan P. Farwell, MD, treatment, such as those with cardiac disease.
FACE. Department of Medicine, VA Greater Los
Angeles Healthcare System and David Geffen School of
BARRIERS TO OPTIMAL PRACTICE
Medicine at University of CaliforniaLos Angeles, Los Difficulty in interpreting thyroid function studies and
Angeles, California 90073, E-mail: identifying the usual clinical features of primary thyroid
gbrent@mednet.ucla.edu; Section of Endocrinology, disease in patients who are hospitalized.
Diabetes and Nutrition, Boston University School of Identifying those patients with thyroid function changes,
Medicine, Boston Medical Center, Boston, Massachusetts in the setting of nonthyroidal illness, who would
01583, E-mail: alan.farwell@bmc.org potentially benefit from thyroid hormone therapy.

INTRODUCTION LEARNING OBJECTIVES


Historical Overview As a result of participating in this session, learners should be
Several studies in the 1960s identified alterations in thyroid
able to:
hormone binding to proteins in serum taken from patients with Describe the changes that occur in thyroid function
illness (1). A later clinical study identified low serum concen- studies with illness.
trations of T3 in a group of hospitalized clinically euthyroid Identify the clinical features that favor a diagnosis of
patients with a range of illnesses (2). The high prevalence of primary thyroid disease in the setting of illness.
these thyroid function test changes in illness, termed the Identify the clinical conditions associated with the
nonthyroidal illness syndrome or euthyroid sick syndrome, and thyroid function changes of nonthyroidal illness that may
association with mortality led to intensive studies on the benefit from treatment.
mechanism and significance of these changes. Elevated
cytokines, altered thyroid hormone metabolism, inhibitors of
thyroid hormone binding to serum proteins, and altered TSH STRATEGIES FOR DIAGNOSIS, THERAPY,
glycosylation, were all thought to contribute to the observed AND/OR MANAGEMENT
changes (3). Whether these thyroid hormone abnormalities Overview
It has long been recognized that a range of conditions, includ-
were a physiologic adaptation to illness or a pathologic change
ing illness, surgery, trauma, and fasting, are associated with
that would benefit from treatment is a topic that is still debated,
alterations of thyroid function tests, generally in a predictable
with most believing that the alteration in thyroid studies with
pattern (6 9) (Figure 1). The first test to become abnormal is
illness were protective and helped to reduce catabolism (3). A
the serum T3 concentration, which is reduced within just a few
small randomized study of patients in the intensive care unit
hours after acute illness or surgery. Rather than being con-
(ICU) with low serum T4 concentrations in 1986 identified no
verted to T3, T4 is preferentially converted to reverse T3 (rT3),
change in outcome as a result of levothyroxine treatment (4).
which can be directly measured in the serum. With more severe
Subsequent similar small trials of treatment with T4 or T3 did
and prolonged illness, especially patients requiring intensive
not show a concrete benefit, although some continued to advo-
care, the serum T4 concentration is also reduced. The magnitude
cate for the value of treatment (5).
of reduction in serum T4 concentration is associated with in-
creased mortality. Despite the reduced serum T4 and T3 con-
SIGNIFICANCE OF THE CLINICAL PROBLEM centration, most patients have a normal range-serum TSH con-
Thyroid function changes associated with nonthyroidal illness centration, which is what distinguishes the nonthyroidal illness
are seen in most hospitalized patients and, in the ICU, mortality syndrome from primary hypothyroidism. The profile, however,
is correlated with the magnitude of the reduction in serum T4 could also represent central hypothyroidism: reduced serum T4
concentration. The essential question is whether the changes concentration with an inadequate TSH response. Early studies
seen in the sick euthyroid syndrome are physiologic or patho- focused on estimates of the free fraction of thyroid hormone to
logic, and has resulted in the controversy of the role, if any, of determine whether the free T4 is truly reduced or if reduced
treating the disorder with thyroid hormone. The influence of binding proteins and augmented release of binding inhibitors
illness and administered medications in the ICU setting can results in a normal free fraction of T4, despite low total T4.
make it difficult to diagnose primary thyroid disease and iden- These studies, reporting a normal free fraction of T4, supported

The Endocrine Society. Downloaded from press.endocrine.org by [${individualUser.displayName}] on 12 January 2017. at 12:19 For personal use only. No other uses without permission. . All rights reserved.
ENDO 2016 THYROID/HPT AXIS 339

FIGURE 1. Changes in serum thyroid hormone concentrations with progressive illness and recovery.

that this condition should not be treated with thyroid hormone Factors Affecting TSH Levels
(3). The association of the profile of low serum T4 and T3 Serum TSH levels usually remain normal during the early
concentrations with poor outcome, and the potential that the phases of acute illness. However, with illness progression, TSH
TSH response may be blunted, has led many to consider the steadily decreases as a result of multiple factors that have a
potential value of treatment with thyroid hormone, although direct inhibitory effect on TSH secretion. These factors include
most have concluded that it is not indicated (510). the use of dopamine and glucocorticoids in the critically ill. As
noted above, D2 activity has been reported to be increased in
Mechanisms the pituitary in the nonthyroidal illness syndrome, increasing
Factors Affecting T3 Levels local production of T3 and decreasing TSH synthesis. Further-
Recent progress in animal and cellular models of nonthyroidal more, increased production of thyroid hormone metabolites
illness have provided insights into the reason that a normal such as 3,5,3-triiodothyroacetic acid (Triac) during acute ill-
TSH is observed, despite reduced serum T3 concentration ness also have a direct inhibitory effect on TSH synthesis.
(9 11). Initially, the decrease in serum T3 in nonthyroidal There is ample evidence of decreased TRH production and
illness was ascribed completely to a decrease in type 1 secretion leading to a decrease in TSH in the euthyroid sick
5-deiodinase (D1) activity, mainly due to findings in studies of syndrome. Leptin has been reported to directly regulate TRH
rodent models of illness. However, recent studies have recog- production and serum leptin levels decrease during fasting, as
nized the importance of the type 2 5-deioidinase (D2) in well as in elderly patients with the euthyroid sick syndrome,
contributing to serum T3 in humans. In contrast with rodents, in leading to subsequent decrease in TSH levels. Increased hypo-
which most T3 is generated from conversion of T4 to T3 by D1, thalamic D2 activity, as well as increased Triac production,
in humans D2 makes the greatest contribution to serum T3. D1 have also been reported to directly decrease TSH production,
is directly regulated by T3 and low levels of T3 are associated similar to what has been observed in the pituitary. Some inves-
with reduced D1, and high levels of T3 with high levels of D1.
tigators have hypothesized a central role for decreased TRH
In contrast, D2 activity is inversely related to serum T4
production in the euthyroid sick syndrome leading to proposal
concentration. Higher levels of T4 reduce D2 activity and
to use TRH as a therapeutic intervention. Indeed, a trial of
lower levels of serum T4 augment D2 activity. Most active
exogenous TRH administration in conjunction with GH-
T3 in the hypothalamus and pituitary, which regulates TRH
releasing petide-2 in 14 patients with prolonged critical illness
and TSH, is generated by D2 from local conversion of T4 to
restored the alterations seen in serum thyroid hormone param-
T3. As T4 decreases in nonthyroidal illness, D2 in the hypo-
eters, with subsequent improvement in metabolic parameters
thalamus and pituitary becomes more active, and there is
such as protein degradation (12).
more T4 to T3 conversion, usually resulting in normal TRH
and TSH. This has been shown in a number of in vitro and
animal models. MAIN CONCLUSIONS
Other factors that play a role in the decreased T3 levels in Describe the Changes that Occur in Thyroid Function
nonthyroidal illness include the increased activity of type 3 Studies with Illness
5-deiodinase (D3) activity in liver and inflammatory cells, that Serum T3 concentration is reduced first in illness and is ob-
inactivates T3 by conversion to rT3, as well as increases in served within a few hours of fasting, illness, or surgery (low
nondeiodinative pathways, such as sulfoconjugation and ala- T3). Serum T4 concentration decreases with more severe illness
nine side chain deamination/decarboxylation. Thus, it has be- and is associated with a poor outcome (low T4). Serum rT3
come clear that there are many pathways that contribute to concentration increases with reduced T3 production. Serum
reduced T3 levels early in the euthyroid sick syndrome. TSH remains in the normal range, sometimes below the lower

The Endocrine Society. Downloaded from press.endocrine.org by [${individualUser.displayName}] on 12 January 2017. at 12:19 For personal use only. No other uses without permission. . All rights reserved.
340 ENDO 2016 MEET-THE-PROFESSOR CLINICAL CASE MANAGEMENT

reference range with severe illness. Rarely, such as in the hypothyroidism, those patients with hypothermia and bradycar-
setting of acute psychiatric illness, serum T4 concentration can dia, should be strongly considered for empiric T4 therapy.
be elevated. Recovery from illness is associated with normal-
ization of serum T4 and serum T3 concentration, usually with a Identify the Clinical Conditions Associated with the
transient increase in serum TSH, often as high as 10 20 Thyroid Function Changes of Nonthyroidal Illness that
mIU/L, generally normalizing in 4 6 weeks. May Benefit from Treatment
A summary of clinical trials of thyroid hormone therapy in
Identify the Clinical Features that Favor a Diagnosis of nonthyroidal illness syndrome is provided (Table 1). A pos-
Primary Thyroid Disease in the Setting of Illness sible benefit to therapy is limited to premature infants, cardiac
Awareness of the sequential changes in serum thyroid hormone donors, and congestive heart failure.
parameters in nonthyroidal illness will allow perturbations due
to intrinsic thyroid dysfunction to be a bit more clear. For
example, an increased TSH early in illness is more consistent CASES
with pre-existing thyroid disease as opposed to later in the Case 1
recovery phase of a prolonged illness. A high-normal serum T3 A 68-year-old man with schizophrenia, hyperlipidemia, and
concentration in the setting of a low TSH in any patient with type 2 diabetes mellitus is found unresponsive in the field and
acute illness should raise concern about hyperthyroidism. Prior taken to the emergency department. He had been receiving
history of thyroid disease, neck surgery, or neck radiation prednisone for a myopathy for the past month. In the emer-
should be obtained, when possible, and would increase the gency department he was found to have a systolic blood pres-
likelihood of primary thyroid disease. Prior thyroid testing, sure (BP) in the 60s, and his BP increased only slightly with
especially TSH and anti-thyroid peroxidase antibodies (TPO fluid resuscitation. His temperature was 99.5F, pulse 68 bpm,
Ab), should be reviewed. Further, anti-TPO testing during the and oxygen saturation was reduced at 90%. He was intubated
evaluation of abnormal thyroid tests may be helpful to support and begun on mechanical ventilation. He required dopamine to
a diagnosis of underlying Hashimotos thyroiditis. In illness maintain a normal systolic BP. His evaluation was significant
that is suspected to be complicated by underlying primary for a white blood cell (WBC) count of 23 000 and a large

TABLE 1. Results of Trials of Thyroid Hormone Therapy in Nonthyroidal Illness Syndrome


Illness Results of Treatment Trial
Starvation/undernutrition Treatment with T3 results in increased protein breakdown and increased nitrogen excretion in fasting
normal and obese patients
General ICU patients No benefit of LT4 on general medical patients, patients with acute renal failure, or those undergoing
renal transplantation
No benefit of T3 in burn patients
Premature infants No benefit of LT4 on developmental indices of premature infants at 2628 wk of gestation
Possible beneficial effect of LT4 on infants of 2526 wk of gestation, but possible deleterious effects
on infants of 2730 wk of gestation
No benefit of T3
Meta-analysis shows no significant effects of thyroid hormone treatment of premature infants
Patients undergoing cardiac Small studies suggest improved hemodynamic variables with T3
surgical procedures Large trials show no benefit in outcome of T3 in adult patients undergoing cardiac bypass
Possible improvement in hemodynamic variables and hospital stay with T3 in children undergoing
cardiac surgical procedures
Cardiac donors Variable results of the effects of T3 in preserving function of normal hearts
in brain-dead cardiac donors before transplantation
Possible benefits of T3 in improving function of impaired hearts before transplantation, potentially
increasing the pool of organs available for transplantation
Consensus conference recommend the use of T3 as part of the hormonal resuscitation in donors
whose cardiac ejection fraction is 45%
Congestive heart failure Small, uncontrolled study suggested that short-term LT4 therapy increased cardiac output and
functional capacity and decreased systemic vascular resistance
Improved hemodynamic variables and neurohumoral profiles with short-term iv T3 infusion, possibly
necessitating supraphysiologic concentrations

Adapted from Lee et al (9).

The Endocrine Society. Downloaded from press.endocrine.org by [${individualUser.displayName}] on 12 January 2017. at 12:19 For personal use only. No other uses without permission. . All rights reserved.
ENDO 2016 THYROID/HPT AXIS 341

amount of bacteria and WBCs in the urine. His blood cultures range, 80 180 ng/dL), TSH 0.03 mIU/L (normal range,
and urine were ultimately positive for E. coli. He was treated 0.353.5 mIU/L).
with broad-spectrum antibiotics and stress dose glucocorticoids. What is the next appropriate step in management?
Initial thyroid function studies: TSH, 3.42 mIU/L (normal A. Begin treatment with iv liothyronine.
range, 0.5 4.8 mIU/L); free T4, 0.61 ng/dL (normal range, B. Begin treatment with combined levothyroxine and
0.89 1.76 ng/dL); T3, 21 ng/dL (normal range, 80 180 ng/dL). liothyronine.
TSH 1 year prior to hospitalization was 1.3 mIU/L. C. Begin treatment with TRH.
The patient was extubated on hospital day 2, tapered off D. Measure anti-TPO antibodies.
his dopamine drip, and maintained a normal range systolic
BP. He continued on antibiotic therapy. On hospital day 3 Case 4
he became more responsive and by hospital day 4, he was A 54-year-old woman with a past medical history of hyperten-
transferred to the general medical ward. He received antibi- sion was diagnosed with a cancerous mass after a recent
otics with continued clinical improvement, became afebrile, colonoscopy. She was admitted to the surgical service for an
and his WBC normalized. His thyroid function studies on elective resection of the colonic mass, which was uneventful.
hospital day 8 were TSH, 17.34 mIU/L; free T4, 1.35 ng/dL; She was kept NPO postoperatively and because of sluggish
and T3, 69 ng/dL. return of bowel function, thyroid function tests were obtained
What is the next appropriate step in management? on the third postoperative day: FT4, 0.9 ng/dL (normal range,
A. Begin treatment with levothyroxine. 0.71.5 ng/dL); total T3, 70 ng/dL (normal range, 80 180 ng/dL);
B. Begin treatment with combination levothyroxine and TSH, 0.36 mIU/L (normal range, 0.353.5 mIU/L). There is no
liothyronine. prior history of thyroid dysfunction and the thyroid gland was
C. Measure anti-thyroid peroxidase (TPO) antibodies. normal by palpation.
D. Repeat thyroid function studies in 4 6 weeks. What is the next appropriate step in management?
A. Begin treatment with iv liothyronine.
Case 2 B. Evaluate for parenteral nutrition.
A 45-year-old man with a past medical history of asthma C. Measure anti-TPO Ab.
requiring periodic admissions over the last few years was D. Repeat thyroid function studies in 4 6 weeks.
admitted with an asthma exacerbation and treated with iv E. Options B and D.
solumedrol and inhaled bronchodilators. He had been in his
usual state of health prior to developing an upper respiratory DISCUSSION OF CASES AND ANSWERS
illness 2 days before admission. He responded to therapy with
an improvement in wheezing and discharge to home was Case 1
planned. On the third hospital day, he developed new-onset Case 1 is a patient in septic shock due to E.Coli bacteremia
atrial fibrillation and thyroid function tests were obtained: Free associated with a urinary tract infection. The patient is
Thyroxine Index (FT4I), 8.0 (normal range, 312); TSH, 0.1 critically ill, requiring intubation and dopamine for support.
mIU/L (normal range, 0.353.5 mIU/L). He has no prior thy- Thyroid function studies show a reduced serum free T4 and
roid tests to compare. Physical examination revealed a pal- total T3 concentration. The TSH is normal range, potentially
pable thyroid gland that was not enlarged. inappropriately normal for the low serum T4 and T3 concen-
What is the next appropriate step in management? tration. The administration of dopamine and stress dose
A. Begin treatment with antithyroid drugs. glucocorticoids could suppress a TSH that would otherwise be
B. Measure a total T3 and thyroid antibodies, TPO, and increased and suggest primary hypothyroidism. This patient,
thyroid-stimulating immunoglobulins. however, had a normal serum TSH 1 year ago, and does not
C. Perform a thyroid scan and 24-hour radioactive iodine have a reduced body temperature. His pulse rate, however, was
uptake. low. The patient improved clinically and follow-up thyroid
D. Repeat thyroid function studies in 4 6 weeks. studies on hospital day 8 show an increase in free T4 and T3
concentration, but the TSH is elevated to 17.34, a range usually
Case 3 associated with primary hypothyroidism. This patient, how-
A 68-year-old female with an extensive cardiac history and ever, is improving clinically and has had an increase in his T4
chronic obstructive pulmonary disease collapsed and was ad- and T3 concentrations. This pattern is most consistent with
mitted to the hospital following resuscitation after a cardiac recovery from nonthyroidal illness syndrome and does not need
arrest. She was intubated and started on vasopressors, including to be treated. If the primary factor influencing the elevation in
dopamine, and also received high doses of iv solumedrol. After TSH was release of suppression from the previous dopamine
2 weeks, she remained comatose and required vasopressors to and glucocorticoid therapy, it would not be expected that the
maintain a normal BP. Thyroid function tests were obtained: serum T4 concentration would increase. An anti-TPO Ab could
FT4I, 1.5 (normal range, 312), total T3 50 ng/dL (normal be measured, but his TSH was normal 1 year ago and it would

The Endocrine Society. Downloaded from press.endocrine.org by [${individualUser.displayName}] on 12 January 2017. at 12:19 For personal use only. No other uses without permission. . All rights reserved.
342 ENDO 2016 MEET-THE-PROFESSOR CLINICAL CASE MANAGEMENT

not likely change management at this time. If the TSH remains Case 4
elevated in 4 6 weeks, the possibility of primary hypothyroid- Case 4 represents a clear example of the nonthyroidal illness
ism would need to be considered. syndrome being an adaptive, rather than a pathologic process.
The changes in thyroid hormone levels in starvation or malnu-
Case 2 trition seem to be an adaptive response to a reduction in
Case 2 highlights the effect of drugs on thyroid hormone catabolic processes and preservation of total body protein
parameters. The most likely cause of the low-but-detectable stores. Caloric restriction is known to be a potent inhibitory
factor of D1 activity, with a reduction in serum T3 levels and
TSH is a direct inhibitory effect of the high-dose iv glucocor-
an increase in serum rT3 levels, within 24 hours of a fast. A
ticoid on TSH secretion at the pituitary levels. The difference
reduction in D1 activity, coupled with an increase in D3 activ-
between a low-but-detectable TSH and one that is undetectable
ity and decrease in TSH secretion, in part from suppressed
is the reason for development of the sensitive TSH assays in
leptin secretion and reduction in thyroid hormone uptake, all
the 1980s. With the third-generation ultrasensitive TSH assay
seem to play a role in the thyroid hormone alterations seen in
in use today, the vast majority of cases of true hyperthyroidism fasting or malnutrition. Studies have shown that early nutri-
will have undetectable TSH values whereas patients with a low tional support in postoperative patients can prevent develop-
TSH in the setting of nonthyroidal illness should have a detect- ment of the euthyroid sick syndrome, suggesting again that
able TSH. Indeed, a landmark study in 1987 showed that less acute fasting-induced the euthyroid sick syndrome is an adap-
than 1% of hospitalized patients with a low-but-detectable tive and potentially beneficial mechanism (14).
serum TSH have intrinsic thyroid dysfunction (13). Hyperthy-
roidism is in the differential of the causes of new-onset AF, but REFERENCES
his pulmonary disease, as well as possible underlying cardiac 1. Oppenheimer JH, Squef R, Surks MI, Hauer H. Binding of thyroxine by
disease in a 45-year-old male is more likely the cause. To sort serum proteins evaluated by equilibrum dialysis and electrophoretic
techniques. alterations in nonthyroidal illness. J Clin Invest. 1963;42:
this out, measuring a total T3 would be diagnostic: it would be 1769-1782.
expected to be low normal to frankly low in the setting of 2. Bermudez F, Surks MI, Oppenheimer JH. High incidence of decreased
nonthyroidal illness. A high normal or elevated T3 would serum triiodothyronine concentration in patients with nonthyroidal disease.
J Clin Endocrinol Metab. 1975;41:27-40.
prompt further evaluation with radioiodine uptake and likely 3. Chopra IJ. Clinical review 86: Euthyroid sick syndrome: Is it a misnomer?
treatment with antithyroid drugs. J Clin Endocrinol Metab. 1997;82:329-334.
4. Brent GA, Hershman JM. Thyroxine therapy in patients with severe
nonthyroidal illnesses and low serum thyroxine concentration. J Clin
Case 3 Endocrinol Metab. 1986;63:1-8.
Case 3 illustrates the effects of multiorgan failure on thyroid- 5. De Groot LJ. Dangerous dogmas in medicine: The nonthyroidal illness
syndrome. J Clin Endocrinol Metab. 1999;84:151-164.
hormone parameters. This critically ill patient exhibits low 6. Langton JE, Brent GA. Nonthyroidal illness syndrome: Evaluation of
serum values in T4, T3, and TSH and the mortality in this thyroid function in sick patients. Endocrinol Metab Clin North Am.
situation approaches 85%. This is also the situation in which 2002;31:159-172.
7. Farwell AP. Nonthyroidal illness syndrome. Curr Opin Endocrinol Diabe-
others have advocated treatment with thyroid hormone, tes Obes. 2013;20:478-484.
whether T4, T3, or a combination (5). The rationale is that, in 8. Fliers E, Bianco AC, Langouche L, Boelen A. Thyroid function in criti-
addition to the physiologic reasons for the decrease in thyroid cally ill patients. Lancet Diabetes Endocrinol. 2015;3:816-825.
9. Lee S, Farwell AF. Sick euthyroid syndrome. Compr Physiol. In press.
function (ie, to decrease catabolism and conserve energy), there 10. Kaptein EM, Beale E, Chan LS. Thyroid hormone therapy for obesity and
are likely pathologic reasons such as the use of steroids and nonthyroidal illnesses: A systematic review. J Clin Endocrinol Metab.
2009;94:3663-3675.
vasopressors contributing to the reduction in serum TSH levels.
11. Gereben B, Zavacki AM, Ribich S, et al. Cellular and molecular basis
Indeed, this picture most likely represents the effects of severe of deiodinase-regulated thyroid hormone signaling. Endocr Rev.
nonthyroidal illness and associated treatments, on hypotha- 2008;29:898-938.
12. Van den Berghe G, Wouters P, Weekers F, et al. Reactivation of pituitary
lamic and pituitary function. As such, some investigators have
hormone release and metabolic improvement by infusion of growth hormone-
focused on addressing the pituitary hypofunction by advoca- releasing peptide and thyrotropin-releasing hormone in patients with pro-
ting the use the hypothalamic peptides TRH and GH-releasing tracted critical illness. J Clin Endocrinol Metab. 1999;84:1311-1323.
13. Spencer C, Eigen A, Shen D, Duda M, Qualls S, Weiss S, Nicoloff J.
petide-2 as opposed to thyroid hormone (12). However, TRH is Specificity of sensitive assays of thyrotropin (TSH) used to screen for
currently unavailable in the United States. In any event, there thyroid disease in hospitalized patients. Clin Chem. 1987;33:1391-1396.
are multiple studies in the literature that demonstrate that the 14. Langouche L, Vander Perre S, Marques M, et al. Impact of early nutrient
restriction during critical illness on the nonthyroidal illness syndrome and
use of thyroid hormone treatment in this setting has no effect its relation with outcome: A randomized, controlled clinical study. J Clin
on outcomes (4, 6 9). Endocrinol Metab. 2013;98:1006-1013.

The Endocrine Society. Downloaded from press.endocrine.org by [${individualUser.displayName}] on 12 January 2017. at 12:19 For personal use only. No other uses without permission. . All rights reserved.
ENDO 2016 THYROID/HPT AXIS 343

Thyroid Disease in Pregnancy

M20 of adverse obstetric and fetal events. During the last 15 years,
Presented, April 1 4, 2016 many studies have examined possible associations between
maternal subclinical hypothyroidism and adverse pregnancy
outcomes. Although many studies suggest that untreated sub-
Elizabeth N. Pearce, MD, MSc. Boston University clinical hypothyroidism is associated with poor obstetric and
School of Medicine, Boston Medical Center, Boston, child neurodevelopmental outcomes, data are conflicting, and
Massachusetts 02118, E-mail: elizabeth.pearce@bmc.org screening and treatment recommendations remain controver-
sial. Thyroid autoimmunity, even in the absence of thyroid
dysfunction, has been linked to increased risk for miscarriage
INTRODUCTION
Historical Overview and premature delivery. Finally, pregnant women have in-
The ancient Egyptians were aware that anterior neck enlarge- creased dietary iodine requirements, and mild-to-moderate io-
ment frequently occurred in pregnant women. Soranus of dine deficiency in pregnancy remains widespread globally,
Ephesus described neck swelling in women after pregnancy in including in developed nations such as the United States.
the second century AD. Following these observations, the field
was relatively quiescent for centuries. By the early 20th cen- BARRIERS TO OPTIMAL PRACTICE
tury it was known that basal metabolic rate increased in preg- Trimester-specific, assay-specific reference ranges are
nancy, and that goiter and thyroid hyperplasia were frequently not available for most thyroid function assays.
observed. Once it could be measured, protein-bound iodine was There are few prospective, randomized trials to inform
found to double in the first 12 weeks of gestation, and it was current practice recommendations.
demonstrated by the early 1960s that this was due, at least in Currently published guidelines are not in agreement,
part, to increased T4 binding globulin (TBG) levels. The first especially regarding recommendations for thyroid
use of thiouracil to treat hyperthyroidism in pregnancy was function screening in pregnancy.
documented by Edwin Astwood in 1944 (1). Teratogenic ef-
fects of methimazole (MMI) were first reported by Samuel LEARNING OBJECTIVES
Milham and William Elledge in 1972 (2). An association be- As a result of participating in this session, learners will be able to:
tween hypothyroidism and miscarriage risk was described by Describe the physiologic changes in thyroid function
King and Herring in 1939 (3). In the 1950s Evelyn Man (4) which occur during gestation.
reported developmental deficits in the children of severely Explain how to recognize and treat thyroid dysfunction
hypothyroid mothers; in 1999 adverse child developmental in pregnant women.
effects of milder hypothyroidism were described by James Understand current areas of controversy regarding
Haddow and colleagues (5), and adverse cognitive effects of diagnosis and therapy.
maternal hypothyroxinemia were described by VJ Pop and
colleagues (6). The necessity for increased levothyroxine (LT4)
dosing for hypothyroid women in pregnancy was first demon- STRATEGIES FOR DIAGNOSIS, THERAPY,
strated by Susan Mandel and colleagues in 1990 (7). Also in AND/OR MANAGEMENT
Physiologic Thyroid Changes in Pregnancy
1990, Alex Stagnaro-Green and colleagues (8) were the first to
Pregnancy induces multiple changes to thyroid physiology.
report associations between thyroperoxidase (TPO) antibody
Therefore, thyroid function tests in euthyroid pregnant women
positivity and miscarriage risk.
may differ from those of euthyroid nonpregnant adults. Starting
in early pregnancy, high levels of circulating estrogen decrease
SIGNIFICANCE OF THE CLINICAL PROBLEM catabolism of TBG, leading to increased circulating TBG levels,
Thyroid disorders are prevalent among pregnant women. Two which remain elevated until delivery. This increases circulating
to three percent of pregnant women have some degree of levels of total T3 and T4 approximately 1.5-fold. In addition, in
hypothyroidism at the time of routine screening, of whom early gestation the thyroid is stimulated not only by TSH, but by
0.3-0.5% will have overt hypothyroidism and 2-2.5% subclini- human chorionic gonadotropin (hCG), which also binds to and
cal hypothyroidism. Hyperthyroidism occurs in approximately stimulates the TSH receptor. The production of hCG peaks at 9-11
0.1-0.4% of pregnancies. Approximately 10 15% of pregnant weeks gestation; levels then decline until approximately 20
women are antithyroid antibody positive. Strategies for the weeks gestation and remain stable for the remainder of the preg-
diagnosis of thyroid dysfunction differ in pregnancy compared nancy. The thyrotrophic activity of hCG results in decreased
with those for nonpregnant women. Untreated overt maternal serum TSH concentrations in the first trimester (9), with a reduc-
hyperthyroidism and hypothyroidism clearly increase the risk tion in both the lower (decreased by 0.1-0.2 mIU/L) and the

The Endocrine Society. Downloaded from press.endocrine.org by [${individualUser.displayName}] on 12 January 2017. at 12:19 For personal use only. No other uses without permission. . All rights reserved.
344 ENDO 2016 MEET-THE-PROFESSOR CLINICAL CASE MANAGEMENT

upper limit of serum TSH (decreased by 1.0 mIU/L), relative to Women with a serum TSH greater than 10 mIU/L should
the nonpregnant TSH reference range. Where available, assay- always be treated in pregnancy. The goal of therapy is to
specific and trimester-specific reference intervals should be used maintain serum TSH less than 2.5 mIU/L. Thyroid hormone
for all thyroid-function tests in pregnant women. requirements increase starting early in pregnancy. To maintain
normal thyroid function, most pregnant women require an
Iodine Requirements in Pregnancy increase in LT4 from their prepregnancy doses. This increased
Adequate dietary iodine intake is required for the production of requirement for LT4 starts at approximately 4-6 weeks gesta-
thyroid hormone. Because of increased thyroid hormone pro- tion. LT4 requirements gradually increase through 16-20 weeks
duction, increased renal iodine excretion, and fetal iodine re- of pregnancy and then plateau until delivery (19). LT4 dose
quirements, dietary iodine requirements are higher in preg- increases, when necessary, should be made as soon as possible
nancy than they are for nonpregnant adults (10). Severe iodine after pregnancy is confirmed. One strategy to achieve this in
deficiency is associated with poor obstetric outcomes including women who are euthyroid on LT4 prepregnancy is to increase
spontaneous abortion, prematurity, and stillbirth (11). Severe LT4 by two additional tablets weekly (nine tablets per week
iodine deficiency is also associated with adverse effects on the instead of seven; 29% increase) as soon as pregnancy is diag-
fetus including congenital anomalies, decreased intelligence, nosed (20). Another option is to empirically increase the dos-
and cretinism. Mild-to-moderate iodine deficiency is less well age of daily LT4 by approximately 2530% as early as possible
studied, but has been associated with decreases in offspring in pregnancy. In pregnant patients with treated hypothyroidism,
neurocognitive function (12). serum TSH should be monitored approximately every 4 weeks
Pregnant and lactating women should ingest approximately during the first half of pregnancy, given that further LT4 dose
250 g iodine daily. The American Thyroid Association, En- adjustments are often required. Maternal TSH should also be
docrine Society, and other groups have recommended 150 mcg checked at least once during the second half of gestation.
daily iodine supplementation for all women who are pregnant, Following delivery, maternal LT4 dosing should be reduced to
planning a pregnancy, or breastfeeding (13, 14). This should be prepregnancy levels, and a serum TSH assessed at approxi-
provided in the form of potassium iodide. mately 6 weeks postpartum.

Hypothyroidism Maternal Hypothyroxinemia


Untreated overt maternal hypothyroidism has been associated Isolated maternal hypothyroxinemia is usually defined as an
with increased risk of premature delivery, gestational hyperten- FT4 level in the lowest 2.5% of values in a given study
sion, low birth weight, and fetal death, as well as decreased population in association with normal serum TSH. Maternal
offspring intelligence. Results of studies describing the risks of hypothyroxinemia has been associated with adverse child cog-
maternal subclinical hypothyroidism have been inconsistent. nitive outcomes in observational studies, (21) but the CATS
However, in at least some studies, subclinical hypothyroidism study, the only randomized controlled trial to date, did not
has been associated with adverse obstetric outcomes including demonstrate a treatment benefit from LT4 (18). Given the
placental abruption, miscarriage, and preterm delivery (15, 16). uncertainty about FT4 thresholds for diagnosis in individual
Subclinical hypothyroidism may also be associated with patients, the lack of clear treatment targets, and the lack of
neurocognitive deficits in the developing fetus (5). The Con- positive interventional studies, treatment of isolated maternal
trolled Antenatal Screening Study (CATS) trial failed to hypothyroxinemia is not currently recommended (13).
demonstrate that LT4 treatment of women with mild hypothy-
roidism improves child cognitive outcomes (17). A similar Thyroid Autoimmunity
National Institutes of Healthfunded multicenter trial is cur- TPO Ab or antithyroglobulin thyroid autoantibodies (Tg Ab)
rently close to completion in the United States, but results are are present in 10 15% of unselected pregnant women. Risks
not yet available. A single clinical trial did demonstrate a for both miscarriage and premature delivery seem to be in-
reduction in a composite adverse obstetric outcome endpoint creased in women with detectable antithyroid antibodies (22).
among TPO-positive women with serum TSH greater than 2.5 One randomized controlled trial has demonstrated that the
mIU/L who were randomly assigned to receive LT4 starting at treatment of TPO Abpositive women with LT4 may decrease
approximately 9 weeks gestation (18). In the absence of risks for both miscarriage and premature delivery (23). How-
interventional studies showing a clear benefit of LT4 treatment ever, this finding has not to date been confirmed in other
for maternal subclinical hypothyroidism, current guidelines are studies. Current guidelines do not advocate treatment of
inconsistent. The Endocrine Society guideline recommends antibody-positive, euthyroid pregnant women (13, 14). Two
treatment of subclinical hypothyroidism whereas the American randomized controlled trials which could lead to changes in
Thyroid Association guideline states that there is not enough treatment recommendations are currently ongoing. The TAB-
evidence to recommend for or against treatment in subclinically LET trial in the United Kingdom is randomizing euthyroid,
hypothyroid antithyroperoxidase thyroid autoantibody (TPO Ab) TPO Abpositive women with a history of infertility or mis-
negative women (13, 14). carriage to LT4 vs placebo to assess effects on live birth rates.

The Endocrine Society. Downloaded from press.endocrine.org by [${individualUser.displayName}] on 12 January 2017. at 12:19 For personal use only. No other uses without permission. . All rights reserved.
ENDO 2016 THYROID/HPT AXIS 345

The T4Life trial in The Netherlands is examining the effects of tive safety and efficacy of these different approaches has not
LT4 treatment in euthyroid pregnant women with a history of been prospectively compared. When Graves disease is diag-
recurrent miscarriage; the primary outcome is live birth rate. nosed prior to pregnancy, consideration may also be given to
definitive treatment with radioactive iodine or thyroidectomy
Hyperthyroidism pregestation to preclude the need for ATD in pregnancy alto-
In early pregnancy, the most common causes of a suppressed gether. In all hyperthyroid patients, thyroid function tests
serum TSH with elevated peripheral thyroid hormone values should be monitored closely throughout gestation. Graves dis-
are Graves disease and gestational thyrotoxicosis (24). Less ease frequently improves in late pregnancy, and ATD can be
frequent causes are toxic multinodular goiter, toxic adenoma, discontinued in 20 30% of women, although the hyperthyroid-
and factitious thyrotoxicosis. A careful history and physical ism may recur postpartum.
examination are essential for determining the etiology of thy- In women with active Graves disease or women with a
rotoxicosis. Hyperthyroid symptoms preceding pregnancy, or a history of Graves who have undergone radioactive iodine
family or personal history of autoimmune thyroid dysfunction ablation or thyroidectomy, maternal serum TRAb should be
favor a diagnosis of Graves disease. When the cause of hyper- measured by 20-24 weeks gestation to help determine the risk
thyroidism is unclear, measurement of thyroid receptor anti-
for neonatal hyperthyroidism. A TRAb level more than three
bodies (TRAb) is frequently helpful.
times the upper limit of normal is an indication for close
Gestational hyperthyroidism, which occurs in 13% of preg-
followup and continued maternal ATD therapy, as risk to the
nancies, is transient and limited to the first half of pregnancy. It
fetus is increased.
is associated with markedly elevated serum hCG values, and
Thyroidectomy can be considered in women with
often occurs in the setting of hyperemesis gravidarum (severe
contraindications to both ATDs. If surgery is required, it is
nausea and vomiting in early pregnancy with 5% weight loss,
most safely performed in the second trimester. Beta blockers
dehydration, and ketonuria). Treatment of women with gesta-
tional hyperthyroidism is directed at alleviation of nausea and such as propranolol may be used in pregnancy for the short-
dehydration; antithyroid drugs (ATDs) are not suggested. term control of hyperadrenergic symptoms.
Poor control of overt hyperthyroidism due to Graves dis-
ease or toxic nodular disease is associated with miscarriages, Screening
premature delivery, pregnancy-induced hypertension, low birth The question of whether to screen for thyroid dysfunction in
weight, intrauterine growth restriction, stillbirth, thyroid storm, asymptomatic pregnant women has been extremely controver-
and maternal congestive heart failure. ATDs are the mainstay sial. Current guidelines include conflicting recommendations
of treatment for hyperthyroidism in pregnancy. Both MMI and regarding thyroid function testing in pregnancy, with the
porpylthiouracil (PTU) cross the placenta. Therefore, to avoid American College of Obstetrics and Gynecology recommend-
causing fetal hypothyroidism or goiter, the goal of ATD ing against testing in the absence of symptoms or known
therapy in pregnancy is to use the lowest possible dose to thyroid dysfunction (27) the American Thyroid Association
maintain maternal serum FT4 values at or just above the upper advocating for a relatively broad case-finding approach (15),
limit of normal. It has been known for many years that expo- and the Endocrine Society panel unable to reach consensus
sure to MMI is associated with an embryopathy that includes regarding universal screening vs case finding (16).
choanal or esophageal atresia and dysmorphic faces. Recently,
it has been determined that PTU exposure is also associated
MAIN CONCLUSIONS
with congenital defectsprimarily abnormalities of face and
Trimester-specific reference intervals should be used for
neck and urinary tractalthough the PTU-associated abnor-
the interpretation of thyroid function tests in pregnant
malities are generally considered less severe than those associ-
women.
ated with MMI (25). Due to case reports of PTU-associated
fulminant hepatic failure, PTU is now regarded as the second- Pregnant and lactating women should ingest
line agent in most settings (26). If ATD therapy is needed approximately 250 g iodine daily.
during the first trimester, PTU is preferred over MMI because LT4 doses must be increased from preconception levels
the risk for severe birth defects is lower, but changing back to in most hypothyroid pregnant women, beginning very
MMI after the first trimester is recommended. In women early in gestation.
treated with MMI prior to pregnancy, if the likelihood of In women with overt hyperthyroidism due to Graves
remission seems to be high (for example, there is no large disease, the smallest possible dose of antithyroid drug
goiter, TSH is normal on ATD, there has been a relatively long that decreases maternal free T4 to just above the upper
duration of ATD use, and/or the TRAb titers are low), consid- limit of the reference range should be employed. Both
eration may be given to stopping MMI rather than substituting PTU and MMI cross the placenta, and both are
PTU when pregnancy is diagnosed. If this is attempted, there associated with congenital defects, although MMI-
should be very close monitoring of thyroid function. The rela- associated embryopathy is more severe. When ATDs

The Endocrine Society. Downloaded from press.endocrine.org by [${individualUser.displayName}] on 12 January 2017. at 12:19 For personal use only. No other uses without permission. . All rights reserved.
346 ENDO 2016 MEET-THE-PROFESSOR CLINICAL CASE MANAGEMENT

are needed in the first trimester of pregnancy, PTU is thyroid gland) biochemical state. Mild hyperthyroidism due to
preferred. hCG effects in the first trimester is physiologic and resolves
spontaneously.
CASES
Case 1 Case 2
A 32-year-old healthy woman presents for evaluation follow- The same patient now presents for a follow-up visit at 21
ing a missed menstrual cycle. She believes she may be preg- weeks gestation. She is feeling well. Her serum TSH on LT4 is
nant. She is feeling well. Her medical history is notable for 0.96 mIU/L.
Graves hyperthyroidism, which was treated with radioactive
iodine ablation 2 years ago. She is currently treated with LT4, Question
112 g daily, and her most recent TSH concentration (mea- What additional testing should be performed?
sured 4 mo ago) was 1.3 mIU/L (normal, 0.5-4.5 mIU/L). A. No additional testing
Testing confirms the patient is pregnant, and you estimate B. Fetal TSH by cordocentesis
she is currently at 9 weeks gestation. She has continued to take C. Maternal TRAb titer
her LT4 daily without any missed doses. D. Maternal FT4
E. Maternal TPO Ab titer
Question
Which statement most likely describes her current thyroid
Answer
status, and the most appropriate recommendation? Correct answer: C
A. The patient is likely euthyroid, and no intervention is In women with a history of Graves disease who have under-
required
gone radioactive iodine ablation or thyroidectomy, TRAb may
B. The patient is likely hypothyroid, and her LT4 dose
still be present, and maternal thyroid function cannot be used as an
should be increased
index of Graves disease activity. Therefore, maternal serum
C. The patient is likely hypothyroid, although no LT4 dose
TRAb should be measured by 20-24 weeks gestation to help
adjustment should be made because this represents
determine the risk for fetal and neonatal hyperthyroidism. A
normal gestational physiology that will normalize
TRAb level more than three times the upper limit of normal is an
D. The patient is likely hyperthyroid, and her LT4 dose
indication for close followup becuase risk to the fetus is increased.
should be decreased
Cordocentesis carries a risk of fetal loss and therefore is only
E. The patient is likely hyperthyroid, although no LT4
rarely used for testing fetal thyroid function. LT4 is dosed to
dose adjustment should be made because this represents
normalize maternal TSH; measuring maternal FT4 would not
normal gestational physiology which will normalize.
change management. The serum TPO antibody would likely be
positive in the setting of the history of Graves disease, but TPO
Answer Ab testing would not be clinically useful in this setting.
Correct answer: B
On average, a 40% increase in maternal LT4 is required
Case 3
during pregnancy to maintain euthyroidism. This increased
A 29-year-old patient presents at 10 weeks gestation with the
demand occurs early in pregnancy, often before the preg-
following thyroid function tests: serum TSH 0.01 mIU/L;
nancy is diagnosed. If the thyroid is functional, increased
FT4 index, 26 (trimester-specific reference range, 8.5-19); total
pituitary demand stimulates increased thyroid hormone pro-
T3, 280 ng/dL (nonpregnancy reference range, 63-180 ng/dL).
duction seamlessly, and no biochemical abnormality will
She has no previous history of thyroid disease. She is feeling
occur. In patients with thyroid dysfunction (prior surgery,
well apart from nausea and emesis intermittently throughout
radioactive iodine ablation, or Hashimotos disease), the
the day for the past 2 weeks. On examination her thyroid is
thyroid cannot adapt to pituitary stimulation and an in-
normal in size without nodules or tenderness. She has no
creased dose of exogenous LT4 must be administered. This
ophthalmopathy.
patient has no functional thyroid tissue because of prior
radioactive iodine treatment. Although she was euthyroid
prior to conception, it is likely she is presently hypothyroid Question
and her LT4 dose must be increased to compensate for Which of the following is the next best step?
greater demand. Normally, a 30% dose increase is provided, A. Start MMI
with repeat testing recommended in 2-4 weeks. B. Start PTU
If this patient had no history of thyroid disease, testing at 9 C. Obtain a radioactive iodine uptake and scan
weeks gestation would likely demonstrate a euthyroid, or D. Repeat the thyroid function tests in 2 weeks
perhaps a mildly hyperthyroid (due to hCG stimulation of the E. Thyroid ultrasound

The Endocrine Society. Downloaded from press.endocrine.org by [${individualUser.displayName}] on 12 January 2017. at 12:19 For personal use only. No other uses without permission. . All rights reserved.
ENDO 2016 THYROID/HPT AXIS 347

Answer 11. World Health Organization, United Nations Childrens Fund, International
Correct answer: D Council for the Control of Iodine Deficiency Disorders. Assessment of
iodine deficiency disorders and monitoring their elimination. 3rd ed. Ge-
This patient most likely has gestational thyrotoxicosis due to neva: WHO, 2007.
hCG stimulation of the thyroidal TSH receptor. The presence 12. Bath SC, Steer CD, Golding J, Emmett P, Rayman MP. Effect of inad-
of nausea and vomiting and the absence of stigmata of Graves equate iodine status in UK pregnant women on cognitive outcomes in their
children: Results from the Avon Longitudinal Study of Parents and Chil-
disease support this diagnosis. However, Graves hyperthyroid-
dren (ALSPAC). Lancet. 2013;382(9889):331-337.
ism can also present in the first trimester. Radioactive iodine 13. Stagnaro-Green A, Abalovich M, Alexander E, et al. Guidelines of the
scanning is contraindicated in pregnancy, and thyroid ultra- American Thyroid Association for the diagnosis and management of thy-
sound will not help to distinguish between gestational thyro- roid disease during pregnancy and postpartum. Thyroid. 2011;21(10):1081-
1125.
toxicosis and Graves disease. Obtaining a TRAb and/or TPO
14. De Groot L, Abalovich M, Alexander EK, et al. Management of thyroid
Ab level might be helpful in determining the etiology of the dysfunction during pregnancy and postpartum: an Endocrine Society clini-
thyrotoxicosis, but those options were not provided. ATDs are cal practice guideline. J Clin Endocrinol Metab. 2012;97(8):2543-2565.
not indicated for the treatment of gestational thyrotoxicosis. In 15. Casey BM, Dashe JS, Wells CE, et al. Subclinical hypothyroidism and
pregnancy outcomes. Obstet Gynecol. 2005;105:239-245.
this patient in whom the etiology of hyperthyroidism is not 16. Allan WC, Haddow JE, Palomaki GE, et al. Maternal thyroid deficiency
currently clear, and who does not have highly elevated thyroid and pregnancy complications: Implications for population screening.
hormone levels, it is reasonable to provide supportive care and J Med Screen. 2000;7:127-130.
to repeat the thyroid function tests in 2 weeks. Serum hCG 17. Lazarus JH, Bestwick JP, Channon S, et al. Antenatal thyroid screening
and childhood cognitive function. N Engl J Med. 2012;366:493-501.
levels peak at 8-10 weeks gestation, so if this is gestational 18. Negro R, Schwartz A, Gismondi R, Tinelli A, Mangieri T, Stagnaro-Green
thyrotoxicosis it is anticipated that thyroid function will start to A. Universal screening versus case finding for detection and treatment of
improve by 12 weeks gestation. Worsened hyperthyroidism thyroid hormonal dysfunction during pregnancy. J Clin Endocrinol Metab.
2010;95:1699-1707.
when thyroid function is repeated in 2 weeks would strongly
19. Alexander EK, Marqusee E, Lawrence J, Jarolim P, Fischer GA, Larsen
suggest the presence of Graves disease. PR. Timing and magnitude of increases in levothyroxine requirements
during pregnancy in women with hypothyroidism. N Engl J Med. 2004;
351:241-249.
REFERENCES 20. Yassa L, Marqusee E, Fawcett R, Alexander, EK. Thyroid hormone early
1. Astwood EB. Thiouracil treatment in hyperthyroidism. J Clin Endocrinol.
adjustment in pregnancy (the THERAPY) trial. J Clin Endocrinol Metab.
1944:4;229-248.
2010;95:3234-3241.
2. Milham S, Elledge W. Maternal methimazole and congenital defects in
21. Henrichs J, Ghassabian A, Peeters RP, Tiemeier H. Maternal hypothyroxinemia
children. Teratology. 1972;5:125-126.
and effects on cognitive functioning in childhood: How and why? Clin
3. King EL, Herring JS. Hypothyroidism in the causation of abortion, espe-
Endocrinol (Oxf). 2013;79(2):152-162.
cially of the missed variety. JAMA. 1939:113(14);1300-1302.
22. Thangaratinam S, Tan A, Knox E, Kilby MD, Franklyn J, Coomarasamy
4. Man EB, Shaver BA Jr, Cooke RE. Studies of children born to women
A. Association between thyroid autoantibodies and miscarriage and
with thyroid disease. Am J Obstet Gynecol. 1958;75(4):728-741.
5. Haddow JE, Palomaki GE, Allan WC, et al. Maternal thyroid deficiency preterm birth: Meta-analysis of evidence. BMJ. 2011;342:d2616.
during pregnancy and subsequent neuropsychological development of the 23. Negro R, Formoso G, Mangieri T, Pezzarossa A, Dazzi D, Hassan H.
child. N Engl J Med. 1999;341:549-555. Levothyroxine treatment in euthyroid pregnant women with autoimmune
6. Pop VJ, Kuijpens JL, van Baar AL, et al. Low maternal free thyroxine thyroid disease: Effects on obstetrical complications. J Clin Endocrinol
concentrations during early pregnancy are associated with impaired psy- Metab. 2006;91(7):2587-2591.
chomotor development in infancy. Clin Endocrinol (Oxf). 1999;50(2):149- 24. Patil-Sisodia K, Mestman JH. Graves hyperthyroidism and pregnancy: A
155 clinical update. Endocr Pract. 2010;16:118-129.
7. Mandel SJ, Larsen Pr, Seely EW, Brent GA, Increased need for thyroxine 25. Andersen SL, Olsen J, Wu CS, Laurberg P. Birth defects after early
during pregnancy in women with primary hypothyroidism. N Engl J Med. pregnancy use of antithyroid drugs: A Danish nationwide study. J Clin
1990;323(2);91-96. Endocrinol Metab. 2013;98(11):4373-4381.
8. Stagnaro-Green A, Roman SH, Cobin RH, el-Harazy E, Alvarez-Marfany 26. Bahn RS, Burch HS, Cooper DS, et al. The role of propylthiouracil in the
M, Davies TF. Detection of at-rist pregnancy by means of highly sensitive management of Graves disease in adults: Report of a meeting jointly
assays for thyroid autoantibodies. JAMA. 1990;264(11):1422-1425. sponsored by the American Thyroid Association and the Food and Drug
9. Glinoer D. The regulation of thyroid function in pregnancy: Pathways of Administration. Thyroid. 2009;19:673-674.
endocrine adaptation from physiology to pathology. Endocr Rev. 1997;18: 27. American College of Obstetrics and Gynecology. ACOG practice bulletin.
404-433. Thyroid disease in pregnancy. Number 37, August 2002. American Col-
10. Yarrington CD, Pearce EN. Dietary iodine in pregnancy and postpartum. lege of Obstetrics and Gynecology. Int J Gynaecol Obstet. 2002;79(2):171-
Clin Obstet Gynecol 2011;54(3):459-470. 180.

The Endocrine Society. Downloaded from press.endocrine.org by [${individualUser.displayName}] on 12 January 2017. at 12:19 For personal use only. No other uses without permission. . All rights reserved.
348 ENDO 2016 MEET-THE-PROFESSOR CLINICAL CASE MANAGEMENT

Graves Orbitopathy

M28 patient and society (4). Therefore, assessing QOL in patients


Presented, April 1 4, 2016 with GO must become part of the clinical practice and also part
of the research outcomes utilized to decide on the effectiveness
of various therapies.
Marius N. Stan, MD. Division of Endocrinology, Epidemiologically, GO is clinically present in approxi-
Diabetes, Metabolism, and Nutrition, Mayo ClinicRochester, mately 25% of patients with GD. Its annual incidence was
Rochester 55905, Minnesota, E-mail: stan.marius@mayo.edu reported at 16/100 000 for women and 2.9/100 000 for men
(5) but it seems to be in decline in recent years. Thus, for
INTRODUCTION many physicians there is a limited exposure to this disease,
Historical Overview which creates uncertainty regarding its management. Most
Graves orbitopathy (GO) (also known as thyroid eye dis- GO cases have a mild course in which managing the risk
ease, Graves ophthalmopathy or thyroid associated factors is the most important therapy, along with local eye
orbitopathy) was described by Robert Graves in 1835 as an protection measures. However, understanding and managing
association between goiter and exophthalmos while around
those risk factors as well as becoming familiar with the local
the same time Caleb H Parry and Karl Adolph von Basedow
eye protection measures are still not employed routinely,
made similar reports. Over the years GO has become the
likely due to the limited exposure mentioned earlier. In
most recognizable abnormality associated with autoimmune
contrast, moderate-to-severe GO has been a frustrating clini-
hyperthyroidism. The assessment of the disease was revolu-
cal entity given the limited clinical response to available
tionized by Francis Felix Rundle, who has theorized the
dynamic behavior of the disease in what is now known as therapeutic agents (mainly glucocorticoids) and the signifi-
the Rundle curve (22). This concept was advanced further cant functional and social impairment associated with this
when it was proposed that the curve represents the changes degree of disease severity. We must work toward better
in both orbital inflammation and disease severity. It thus identification of patients who are likely to respond to these
reflects both an active and inactive phase of the disease agents and avoid exposure to potential serious adverse
whose severity does not quite return to its baseline. Quanti- effects for those unlikely to benefit. Some other im-
fying the activity of the disease has been the Clinical Activ- munomodulators (rituximab, cyclosporine, and tocilizumab)
ity Score (1) that is currently being used as a marker of are being occasionally used and their utilization must be
possible response to anti-inflammatory therapy. Severity of based on a solid understanding of the evidence behind them
the disease has been also quantified during the last decade, as well as their adverse effect profile. Ultimately, sight-
thus allowing for stratification of the interventions. In par- threatening complications, although rare, can lead to perma-
allel with clinical advances describing the disease stages nent visual loss. Therefore early identification and treatment
there have been significant advances in understanding its are essential to prevent or minimize a major functional
pathophysiology. During the last 15 years it has been iden- deficit.
tified the dominant role of autoimmunity toward TSH recep-
tor and the secondary role played by IGF-I receptortargeted
autoimmunity (2). The major cytokines involved in that BARRIERS TO OPTIMAL PRACTICE
process have also been identified and these advances in Several factors relate to the difficulty of adequately managing GO
pathophysiology are now guiding the modern therapeutic cases. First is the rarity of cases with moderate-to-severe or sight-
developments. threatening GO cases. Second is the scarcity of ophthalmologists
with sufficient expertise in GO management, hampering the for-
mation of an essential endocrinology-ophthalmology team, an
SIGNIFICANCE OF THE CLINICAL PROBLEM
approach recommended by both the American Thyroid Associa-
GO has a significant effect on patients wellbeing, as depicted
by a number of studies on patients quality of life (QOL) (3). tion guidelines for management of hyperthyroidism (6) and the
Some of that effect translates into a functional impairment European Group on Graves Orbitopathy (EUGOGO) (7). The
relating to diplopia, photophobia, constant tearing, and other third element is that of outcome assessment: although physicians
similar signs and symptoms that can progress all the way to tend to think in terms of overall GO benefit, all agents available to
loss of vision. Other deficits are related to the social effect of us work for one or another of the various elements of GO but none
the disease induced by the striking change that it induces in works for all of the disease features. That must be well understood
patients appearance. These are now quantified by QOL instru- and communicated to patients to properly define the treatment
ments and have been found to lead to significant costs for expectations.

The Endocrine Society. Downloaded from press.endocrine.org by [${individualUser.displayName}] on 12 January 2017. at 12:19 For personal use only. No other uses without permission. . All rights reserved.
ENDO 2016 THYROID/HPT AXIS 349

LEARNING OBJECTIVES Managing GO


After participation in this session, learners should be able to: Beyond its mildest forms GO management should be pursued
Understand the effect of GO on the therapeutic decisions in a collaborative team with ophthalmology. The EUGOGO
in treating the hyperthyroidism of GD. organization is using the model of combined thyroid eye clin-
Recognize the role of disease activity and severity in GO ics (7) where the patients are seen jointly by the two specialists
therapy selection. and then a management decision is reached in the same man-
ner. For the more severe cases, otorhinolaryngology (ENT) and
Develop a plan of management for patients with GO.
radiation therapy expertise should also be sought. Other exper-
tise might be needed on a case-by-case basis.
STRATEGIES FOR DIAGNOSIS, THERAPY, For all GO patients management should start with an assess-
AND/OR MANAGEMENT ment of their modifiable risk factors.
Natural History Both hyper- and hypothyroidism have been documented
GO is often diagnosed simultaneously with the diagnosis of repeatedly to be associated with development or deteriora-
hyperthyroidism, a scenario that applies from 20% [data from tion of GO (10). Therefore, therapy for GD is the first step.
Olmsted county, Minnesota (5)] to 40% of cases [data from The use of antithyroid drugs and thyroidectomy are neutral
Amsterdam, The Netherlands (8)]. A smaller number of GO toward GO outcome, whereas radioactive iodine (RAI) has
cases (13%) are identified before the thyroid dysfunction. the potential to lead to GO deterioration in patients with
Therefore, most the patients are diagnosed after development active disease (11, 12). Combining RAI with glucocorticoids
of hyperthyroidism, most of them within the first 6 months is able to minimize that negative effect. The dose of
after GD diagnosis. The association of GO with hypothyroid- glucocorticoids most effective in clinical trials has been
ism or with normal thyroid function is also consistently de- 0.4 0.5 mg prednisone/kg of body weight, started 48 72
hours after RAI administration (11). Some case series have
scribed in epidemiological studies with roughly 5% of GO
employed a lower dose of prednisone (0.2 mg/kg) (13) but
cases in each category (9). Once diagnosed the disease follows
that has not yet been verified in a clinical trial. Overall, the
a course that for many years now has been simply described as
data supports the American Thyroid Association Guidelines
Randalls curve with a gradual increase in disease inflamma-
(6) recommending the use of glucocorticoids in patients
tion and subsequent disease severity, followed by a plateau of with mild and active GO who are about to be treated with
both and then a remission of both. Unfortunately, although the RAI. In contrast, for patients with inactive GO the use of
inflammation subsides completely there are often enough re- RAI is unlikely to lead to any deterioration, as documented
sidual changes in disease severity that necessitate targeted be Perros et al (14). Thus, glucocorticoids are not recom-
medical intervention. mended here, except for high-risk cases. The prevention of
hypothyroidism in patients with GD is equally important. It
Diagnosis of GO was demonstrated to be effective in a cohort protocol (15) in
Diagnosis of GO is usually made on clinical grounds when which treating patients with low doses of levothyroxine
evidence of autoimmune thyroid disease is associated with prevented GO development or deterioration, as opposed to
objective changes at the eye level (eg, lid retraction, swelling starting therapy when patients were already hypothyroid.
and redness, conjunctival injection, proptosis, diplopia) com- Smoking cessation is an important element in preventing
bined with subjective complaints (eg, orbital pain; constant and treating GO (6, 7). It is apparent that the smoking-cessation
tearing, paradoxically associated with eye dryness; photopho- clinics are more effective than individual practitioners at this
bia; sensation of foreign body in the eye). If a number of these task and therefore they should be employed. They offer the
benefit of behavioral therapy with psychological counseling
changes are present bilaterally in a patient with hyperthyroid-
and appropriate pharmacotherapy where needed.
ism the diagnosis is secured. The diagnosis is more challenging
The decision to treat and the selection of therapies should be
in euthyroid cases with unilateral eye changes. In this scenario
based on the assessment of disease activity and severity.
orbital imaging with computed tomography or magnetic reso-
nance imaging is indicated to distinguish GO from an infiltra-
Management of Mild GO
tive process (eg, lymphoma), orbital mass lesion (eg, meningi-
After correcting the risk factors the clinician should consider
oma, hemangioma), IgG4-RD inflammation, or other orbital the use of local measures that address the signs and symptoms
abnormality. In euthyroid individuals with a suggestive com- of GO (16). Topical agents are very effective at dealing with
puted tomography imaging the presence of TSH receptor anti- corneal dryness, and thus indirectly with the symptoms of
bodies is extremely helpful for confirming the diagnosis. In the grittiness, excessive watering, and local pain. During daytime
absence of TSH receptor antibodies additional evaluation with we recommend frequent use of artificial tears while at night it
either a period of observation or extraocular muscle biopsy are is best to use gels or ointments that have a longer biological
needed for establishing the diagnosis. action. For patients with severe corneal exposure, taping the

The Endocrine Society. Downloaded from press.endocrine.org by [${individualUser.displayName}] on 12 January 2017. at 12:19 For personal use only. No other uses without permission. . All rights reserved.
350 ENDO 2016 MEET-THE-PROFESSOR CLINICAL CASE MANAGEMENT

eyelids shut at night or using a moisture chamber can provide Management of Sight-Threatening GO
significant relief next morning. Eyeglasses with lateral shield The conditions qualified as such are DON, globe subluxation,
or goggles can relieve the eye discomfort in dusty or windy and corneal ulceration. All require immediate attention with the
environments and sunglasses can eliminate photophobia and last two being exclusively the prerogative of ophthalmology.
minimize the perceived social effect of the disease. The peri- For DON the therapy should start with high-dose iv
ocular swelling might be improved by having patients sleep glucocorticoids every other day for 1 week and could be
with the head of the bed elevated, although this measure is not repeated after a break of one week. It is expected to restore
well tolerated by some patients. The addition of selenium has normal vision in approximately 40% of individuals. Poor re-
been demonstrated to be beneficial in these mild GO cases. It sponse or deterioration should lead to orbital decompression.
has been used as selenium selenite 100 mg twice daily for 6 Ultimately, all therapies should be assessed based on
months (17) and found to improve the overall ophthalmic score changes to patients QOL besides the objective changes noted
and the GO-specific QOL measurements. The results are likely by the physician to the eyes and orbits.
to relate to the antioxidant properties of selenium and trans-
lated mainly in improvement in soft-tissue changes and de-
crease in eyelid aperture in the treated patients. The drug was
MAIN CONCLUSIONS
well tolerated but questions remain whether similar benefits Correction of risk factors should be the first step in
would be found in a selenium-replete population such as those managing GO.
in North America, given that the study was carried out in Only active GO cases should be considered for
European populations with marginal selenium levels. anti-inflammatory therapy.
The intensity of the therapy should match the
Management of Moderate-to-Severe GO disease severity.
Active moderate-to-severe GO usually requires immune- For active moderate-to-severe and sight-threatening
modulatory therapy if the disease has a significant effect on GO, iv glucocorticoids are the first therapeutic
the patients QOL. The most established intervention for choice.
this group is iv methylprednisolone. The standard regimen
for the drug is to administer 500 mg weekly for 6 weeks CASES
followed by another six-weekly infusion of 250 mg, for a Case 1
total of 4.5 g of methylprednisolone (7). This is more effec- A 39-year-old woman diagnosed with GD decides to undergo
tive than oral steroids and the results are usually noticeable RAI therapy. She is minimally symptomatic with increased
within few days. Unfortunately, there are significant mor- gastrointestinal motility and heat intolerance. In addition, she
bidities that can develop with this therapy (6.5%) and very has noted suboptimal control of her type 2 DM, which until
rarely, patients have died (0.6%) in connection with iv recently responded well to lifestyle modification. She was re-
glucocorticoid therapy (18). Therefore, these patients should cently started on metformin. She has no other comorbidities
be screened for preexistent liver dysfunction, chronic infec- and is not a smoker. Her biggest concern is the possibility of
tions, psychiatric illnesses, severe hypertension, cardiovas- GO development post-RAI.
cular insufficiency, and poorly controlled diabetes mellitus
(DM). A single daily dose of methylprednisolone should not
exceed 750 mg and total therapy dose should be less than Examination
8 g. Few other management regimens have been found to be Body mass index, 27 kg/m2; she has a symmetrically enlarged
somewhat efficacious and they could be considered on a thyroid (20 g); mild resting tachycardia (90/min) with regular
case-by-case scenario: 1) combination oral glucocorticoids rhythm and rate; there is no orbitopathy or dermopathy.
plus orbital radiation therapy, and 2) combination oral Which of the following therapeutic options is documented to
glucocorticoids plus cyclosporine therapy. Option 1 is most prevent development of GO and would be appropriate for this
beneficial in patients with motility dysfunction whereas op- patient?
tion 2 could be considered for its steroid-sparing benefit. A. Selenium therapy starting with RAI.
The use of rituximab has been proposed as well for these B. Low-dose prednisone therapy (0.2 mg/kg) starting with
patients after a number of encouraging case series. Unfortu- RAI.
nately, the randomized clinical trials have failed to provide a C. Early initiation of LT4 (2 6 wk after RAI) for
consistent beneficial result (19, 20). Therefore, its clinical prevention of hypothyroidism.
use should be carefully considered and definitely avoided in D. Methimazole therapy for 2 months after RAI.
patients at high risk for or with impending dysthyroid optic E. Change metformin to a thiazolidinedione (TZD) prior
neuropathy (DON). to RAI.

The Endocrine Society. Downloaded from press.endocrine.org by [${individualUser.displayName}] on 12 January 2017. at 12:19 For personal use only. No other uses without permission. . All rights reserved.
ENDO 2016 THYROID/HPT AXIS 351

Case 2 is 80% (21) and it is better tolerated than oral steroids. Atten-
A 61-year-old man is referred for progressive GO of 1 years tion must be paid to proper patient selection (eg, no liver,
duration. He had GD with hyperthyroidism and for treatment cardiovascular, or psychiatric comorbidities). Given that this
underwent thyroidectomy 2 years ago. He has been replaced patient does not have significant diplopia there is no role for
with LT4 and has been euthyroid since. Mild GO developed 1 orbital radiation, with or without steroids. The use of rituximab
year ago and despite stopping smoking his disease has pro- is controversial. One of the randomized clinical trials found it
gressed. His main complaint is the local discomfort with dry- effective in GO (20) whereas the other one did not find any
ness, tearing, and photophobia and has big a concern for sight benefit by comparison with placebo, in particular, in cases of
loss in the future. There are no other comorbidities. disease duration of 1 year or longer (19). The use of cyclospo-
rine combined with prednisone has been more effective than
Examination either agent alone but it remains a second-choice therapy,
Body mass index, 25 kg/m2; severe soft tissue periorbital favored in cases in which a steroid-sparing approach is needed.
edema and chemosis (Clinical Activity Score 6), propto- Selenium is an effective therapy for mild GO cases. This case
sis of 21 mm bilaterally, 1 mm lagophthalmos right and is more severe and unlikely that selenium would have a signifi-
none left; no diplopia and normal neuro-ophthalmic param- cant effect on disease status.
eters (visual acuity and color plates). There is mild der-
mopathy over pretibial area. REFERENCES
Which of the following is the best therapeutic option for this 1. Mourits MP, Prummel MF, Wiersinga WM, Koornneef L. Clinical activity
score as a guide in the management of patients with Graves ophthalmopa-
patient? thy. Clin Endocrinol (Oxf). 1997;47:9-14.
A. Orbital radiation 2. Bahn RS. Current insights into the pathogenesis of Graves ophthalmopa-
B. Rituximab thy. Horm Metab Res. 2015;47:773-778.
3. Terwee C, Wakelkamp I, Tan S, Dekker F, Prummel MF, Wiersinga W.
C. Intervenous methylprednisolone
Long-term effects of Graves ophthalmopathy on health-related quality of
D. Cyclosporine life. Eur J Endocrinol. 2002;146:751-757.
E. Selenium 4. Ponto KA, Merkesdal S, Hommel G, Pitz S, Pfeiffer N, Kahaly GJ. Public
health relevance of Graves orbitopathy. J Clin Endocrinol Metab.
2013;98:145-152.
DISCUSSION OF CASES AND ANSWERS 5. Bartley GB. The epidemiologic characteristics and clinical course of oph-
Case 1 thalmopathy associated with autoimmune thyroid disease in Olmsted
County, Minnesota. Trans Am Ophthalmol Soc. 1994;92:477-588.
This case describes a patient with low risk for GO development 6. Bahn Chair RS, Burch HB, Cooper DS, et al. Hyperthyroidism and other
who will receive RAI therapy. Selenium has never been tested for causes of thyrotoxicosis: Management guidelines of the American Thyroid
this purpose. The use of steroids is not advised in these low risk Association and American Association of Clinical Endocrinologists. Thy-
roid. 2011;21:593-646.
cases based on American Thyroid Association guidelines (6) but it
7. Bartalena L, Baldeschi L, Dickinson AJ, et al. Consensus statement of the
also increases the risk of further deterioration of her DM while European group on Graves orbitopathy (EUGOGO) on management of
likely being ineffective at the low dose proposed. Methimazole Graves orbitopathy. Thyroid. 2008;18:333-346.
therapy initiated very early after RAI (3 days) will decrease the 8. Wiersinga WM, Smit T, van der Gaag R, Koornneef L. Temporal relation-
ship between onset of Graves ophthalmopathy and onset of thyroidal
efficacy of RAI. Furthermore this option is likely to lead to Graves disease. J Endocrinol Invest. 1988;11:615-619.
hypothyroidism if continued for 2 months and thus increase the 9. Bartley GB, Fatourechi V, Kadrmas EF, et al. Clinical features of
risk for GO. TZDs might improve DM control but it will take few Graves ophthalmopathy in an incidence cohort. Am J Ophthalmol.
1996;121:284-290.
weeks for that and it is likely that her DM will improve anyway
10. Prummel MF, Wiersinga WM, Mourits MP, Koornneef L, Berghout A,
by controlling her thyroid dysfunction. In contrast, TZDs are van der Gaag R. Effect of abnormal thyroid function on the severity of
known for stimulating adipogenesis and that effect was speculated Graves ophthalmopathy. Arch Intern Med. 1990;150:1098-1101.
to potentially lead to GO deterioration. There is no evidence that 11. Bartalena L, Marcocci C, Bogazzi F, et al. Relation between therapy for
hyperthyroidism and the course of Graves ophthalmopathy. N Engl
these agents might have GO prevention benefit. Development of J Med. 1998;338:73-78.
biochemical hypothyroidism has been associated with develop- 12. Traisk F, Tallstedt L, Abraham-Nordling M, et al. Thyroid-associated
ment or deterioration of GO (10). In studies where this has been ophthalmopathy after treatment for Graves hyperthyroidism with antithy-
roid drugs or iodine-131. J Clin Endocrinol Metab. 2009;94:3700-3707.
prevented aggressively (15) there has been a noted benefit in 13. Lai A, Sassi L, Compri E, et al. Lower dose prednisone prevents
preventing GO or its deterioration. radioiodine-associated exacerbation of initially mild or absent graves
orbitopathy: A retrospective cohort study. J Clin Endocrinol Metab.
2010;95:1333-1337.
Case 2 14. Perros P, Kendall-Taylor P, Neoh C, Frewin S, Dickinson J. A prospective
This case describes a man with moderate-to-severe GO who study of the effects of radioiodine therapy for hyperthyroidism in patients
has progressive disease despite correction of modifiable risk with minimally active graves ophthalmopathy. J Clin Endocrinol Metab.
2005;90:5321-5323.
factors. This is a perfect case for use of immune-modulatory
15. Tallstedt L, Lundell G, Blomgren H, Bring J. Does early administration of
therapy. The agent that has been found effective on a consistent thyroxine reduce the development of Graves ophthalmopathy after radio-
basis has been iv methylprednisolone. Expected response rate iodine treatment? Eur J Endocrinol. 1994;130:494-497.

The Endocrine Society. Downloaded from press.endocrine.org by [${individualUser.displayName}] on 12 January 2017. at 12:19 For personal use only. No other uses without permission. . All rights reserved.
352 ENDO 2016 MEET-THE-PROFESSOR CLINICAL CASE MANAGEMENT

16. Stan MN, Garrity JA, Bahn RS. The evaluation and treatment of graves 20. Salvi M, Vannucchi G, Curro N, et al. Efficacy of B-cell targeted therapy
ophthalmopathy. Med Clin North Am. 2012;96:311-328. with rituximab in patients with active moderate to severe Graves
17. Marcocci C, Kahaly GJ, Krassas GE, et al. Selenium and the course of orbitopathy: A randomized controlled study. J Clin Endocrinol Metab.
mild Graves orbitopathy. N Engl J Med. 2011;364:1920-1931. 2015;100:422-431.
18. Zang S, Ponto KA, Kahaly GJ. Clinical review: Intravenous glucocorticoids 21. Kahaly GJ, Pitz S, Hommel G, Dittmar M. Randomized, single blind trial
for Graves orbitopathy: Efficacy and morbidity. J Clin Endocrinol Metab. of intravenous versus oral steroid monotherapy in Graves orbitopathy.
2011;96:320-332. J Clin Endocrinol Metab. 2005;90:5234-5240.
19. Stan MN, Garrity JA, Carranza Leon BG, Prabin T, Bradley EA, Bahn RS. 22. Rundle FF, Wilson CW. Development and course of exophthalmos and
Randomized controlled trial of rituximab in patients with Graves ophthalmoplegia in Graves disease with special reference to the effect of
orbitopathy. J Clin Endocrinol Metab. 2015;100:432-441. thyroidectomy. Clin Sci. 1945;5:177-194.

The Endocrine Society. Downloaded from press.endocrine.org by [${individualUser.displayName}] on 12 January 2017. at 12:19 For personal use only. No other uses without permission. . All rights reserved.
ENDO 2016 THYROID/HPT AXIS 353

Thyrotoxicosis: When Antithyroid Drugs Fail

M41 The most important barrier to optimal practice in patients


Presented, April 1 4, 2016 failing ATD therapy involves the optimal selection and moni-
toring of non-ATD therapy.

Henry B. Burch, MD. Walter Reed National Military


LEARNING OBJECTIVES
Medical Center, Bethesda, Maryland 20889,
As a result of participating in this session, learners should be
E-mail: henry.b.burch.mil@mail.mil
able to:
Determine the etiology of ATD failure.
INTRODUCTION Gain familiarity with additional pharmacological
Historical Overview measures for controlling thyrotoxicosis.
The thionamide antithyroid drugs (ATDs) methimazole (MMI), Rapidly prepare a thyrotoxic patient with ATD failure
and propylthiouracil (PTU) were developed in the 1940s and for emergent thyroid or nonthyroid surgery.
early 1950s, and primarily act to inhibit new thyroid hormone
synthesis. The mechanism involves interference with thyroid
STRATEGIES FOR DIAGNOSIS AND
peroxidase (TPO) mediated iodination of tyrosine residues
MANAGEMENT OF ATD FAILURE
within thyroglobulin. This occurs through iodination and me-
Determining the Etiology of ATD Failure
tabolism of the ATDs themselves, thus diverting oxidized io-
Misdiagnosis
dine away from thyroid hormone synthesis. The drugs may also Occasionally, patients are initially referred to an endocrinolo-
interfere with the intramolecular-coupling process within gist after having been started on ATDs by primary care physi-
thyroglobulin, in which iodotyrosine residues are linked to cians for thyrotoxicosis without an adequate diagnostic evalu-
form the iodothyronines T4 and T3. In addition, PTU, but not ation to distinguish destructive thyroiditis from Graves disease
MMI, can block peripheral conversion of T4 to T3. The (GD). TSH-receptor antibody testing can be performed in this
greatest limitation of ATDs is drug-associated adverse ef- circumstance and is probably preferred over a radioactive io-
fects that often lead to altered adherence or contraindication dine (RAI) uptake study, which requires discontinuation of the
for further use. ATD. In addition, a thyroid ultrasound will assist in the dis-
tinction between a diffuse goiter due to GD and a multinodular
SIGNIFICANCE OF THE CLINICAL PROBLEM goiter. Doppler flow, which can distinguish untreated GD from
Given that the goal of ATD therapy is a restoration of euthy- destructive thyroiditis, is less useful in patients already ren-
roidism, ATD failure is defined as a failure to achieve this goal. dered euthyroid on ATDs.
The causes of ATD failure are highly varied and complex, and
include misdiagnosis, inadequate dosing, and an inability to Inadequate Dosing
continue ATDs due to adverse effects. Misdiagnosis involves A rough guideline for the initial dosing of ATD therapy with
an inappropriate use of ATDs in patients who actually have MMI is shown below:
destructive thyroiditis or factitious thyrotoxicosis. Inadequate Free T4 initial MMI dosing
dosing includes both underprescribing of ATDs by the patients 1-1.5 times upper limit of normal (ULN), 5-10 mg daily
physician and nonadherence to therapy by the patient. Inad- 1.5-2 times ULN, 10-20 mg daily
equate dosing leads to ATD failure due to an incomplete 2-3 times ULN, 30-40 mg daily
blockade of new thyroid hormone synthesis. Adverse effects Although the toxicity of MMI is dose related, patients
represent the most significant limitation of ATDs, occurring with severe hyperthyroidism often require higher-than-
in up to 15% of treated patients. Although not all adverse usual doses of MMI, such as 30-40 mg twice daily, and
effects are contraindications to further therapy with ATDs, in most circumstances the negative consequences of
most adverse effects disrupt a patients adherence to varying inadequately treated thyrotoxicosis outweigh the
degrees. increased risk of adverse effects associated with higher
doses.
BARRIERS TO OPTIMAL PRACTICE
Radioiodine and thyroidectomy represent viable alternatives in Nonadherence
patients for whom ATD therapy has failed. However, some Many patients initially thought to be resistant to ATDs are
patients either decline radioiodine or have contraindications to occasionally if not frequently nonadherent with therapy. This is
its use (eg, pregnancy and breast feeding), and thyroidectomy particularly true in the case of PTU, for which an initial
requires control of thyrotoxicosis prior to surgical intervention. three-times-daily administration is recommended. In one study,

The Endocrine Society. Downloaded from press.endocrine.org by [${individualUser.displayName}] on 12 January 2017. at 12:19 For personal use only. No other uses without permission. . All rights reserved.
354 ENDO 2016 MEET-THE-PROFESSOR CLINICAL CASE MANAGEMENT

six of nine patients thought to be resistant to PTU had low young children at home and future pregnancies).
plasma levels of this drug shortly after they had reported taking Started on MMI with improvement, dose adjusted. ALT
this medication (1). Given that MMI is preferred over PTU and increased to 5 times upper limit of normal. MMI stopped.
can be given once daily, simplification of a patients regimen is Patient still refused consideration of RAI therapy.
one technique for improving adherence as is frequent followup, How would you manage this patient at this time?
with appropriate questioning of patients with poor control de-
spite apparently adequate dosing of ATDs. Case 2
A 24-year-old woman with irritability, palpitations, heat intol-
Adverse Effects Preventing Further ATD Usage erance, and weight loss occurring 3 months postpartum.
Whereas some adverse effects related to ATDs are mild (such Physical examination
as pruritus, which is amenable to antihistamine therapy), oth- Pulse: 76 BPM
ers, such as moderate-to-severe hepatocellular damage, agranu- Eyes: no orbitopathy
locytosis, and vasculitis typically lead to a search for non-ATD Neck: diffuse, nontender thyroid enlargement
therapy rather than risk recurrence of these adverse effects. Cardiovascular: no murmur
Laboratory tests
Non-ATD Medical Therapy for Hyperthyroidism Free T4, 2.90 ng/dL (0.79-2.35 ng/dL)

Additional pharmacological agents that have been applied TSH 0.015 mU/L

to the treatment of hyperthyroidism include lithium (2), Total T3, 230 ng/dL (80-200 ng/dL)
cholestyramine, and other binding resins (3), corticosteroids Thyroid stimulating immunoglobulins, 410% (0 139%)
(4), oral cholecystographic agents (5), and potassium iodide Course: patient elected to receive ATDs
(6, 7). Gradually worsening thyrotoxicosis despite dose adjustment
to as high as MMI, 30 mg twice a day. After 5 months of ATDs
Rapid Preparation of a Thyrotoxic Patient for Thyroid she developed elevations of liver associated enzymes (2-3
or Nonthyroidal Surgery times upper limit of normal). RAI planned but patient learned
Hyperthyroid patients who are unable to take ATDs but require
she was pregnant (8 months after last delivery).
rapid preparation for either thyroid or nonthyroidal surgery
How would you manage this patient?
have been treated with the following regimen at the authors
center: Propranolol, 60 mg orally twice daily; dexamethasone,
2 mg iv four times daily; cholestyramine, 4 g orally four times REFERENCES
1. Cooper DS. Propylthiouracil levels in hyperthyroid patients unresponsive to
daily; and saturated solution of potassium iodide, 2 drops orally large doses. Evidence of poor patient compliance. Ann Intern Med.
three times daily (8, 9). 1985;102(3):328-331.
2. Kristensen O, Andersen HH, Pallisgaard G. Lithium carbonate in the treat-
ment of thyrotoxicosis. A controlled trial. Lancet. 1976;1(7960):603-605.
CASES 3. Solomon BL, Wartofsky L, Burman KD. Adjunctive cholestyramine
Case 1 therapy for thyrotoxicosis. Clin Endocrinol (Oxf). 1993;38(1):39-43.
4. Chopra IJ, Williams DE, Orgiazzi J, Solomon DH. Opposite effects of
A 23-year old woman with GD presents with weight loss and
dexamethasone on serum concentrations of 3,3,5-triiodothyronine (reverse
palpitations over the preceding 2-3 months. Past medical his- T3) and 3,35-triiodothyronine (T3). J Clin Endocrinol Metab. 1975;41(5):
tory: negative Social history: no tobacco 911-920.
Physical examination 5. Tyer NM, Kim TY, Martinez DS. Review of oral cholecystographic agents
for the management of hyperthyroidism. Endocr Pract. 2014;20(10):1084-
Pulse: 120 BPM 1092.
Eyes: clinical activity scale-0, no proptosis, normal 6. Uchida T, Goto H, Kasai T, et al. Therapeutic effectiveness of potassium
extraocular movements iodine in drug-nave patients with Graves disease: A single-center experi-
ence. Endocrine. 2014;47(2):506-511.
Thyroid: diffuse goiter with bruit
7. Okamura K, Sato K, Fujikawa M, Bandai S, Ikenoue H, Kitazono T.
Extremities: tremor Remission after potassium iodide therapy in patients with Graves hyper-
Laboratory tests: Free T4, 2.47 (0.8-1.8 ng/dL); TSH, thyroidism exhibiting thionamide-associated side effects. J Clin Endocrinol
0.001 mU/L Metab. 2014;99(11):3995-4002.
8. Langley RW, Burch HB. Perioperative management of the thyrotoxic pa-
Aspartate aminotransferase (AST), 15; alanine tient. Endocrinol Metab Clin North Am. 2003;32(2):519-534.
aminotransferase (ALT), 19; alkaline phosphatase, 90 IU/L 9. Warnock AL, Cooper DS, Burch HB. Life-threatening thyrotoxicosis. Thy-
roid storm and adverse effects of antithyroid drugs. Endocrine and meta-
Discussed treatment options: patient preferred ATDs (would bolic medical emergencies. 2014 ed. Washington, DC: Endocrine Press.
not consider RAI therapy due to concerns regarding effect on 2014;110-126.

The Endocrine Society. Downloaded from press.endocrine.org by [${individualUser.displayName}] on 12 January 2017. at 12:19 For personal use only. No other uses without permission. . All rights reserved.
ENDO 2016 THYROID/HPT AXIS 355

Hypothyroidism in the Elderly

M43 BARRIERS TO OPTIMAL PRACTICE


Presented, April 1 4, 2016 Identification of hypothyroidism requires a careful history for
both classical symptoms found at all ages and nonclassical
symptoms specific to the elderly. Interpretation of thyroid func-
Anne R. Cappola, MD, ScM. Division of Endocrinology, tion testing during hospitalization for acute illness may be
Diabetes, and Metabolism, Perelman School of Medicine difficult. Once recognized, the starting dose of levothyroxine,
at the University of Pennsylvania, Philadelphia, frequency of monitoring, and target TSH may differ from
Pennsylvania 19104, E-mail: acappola@mail.med.upenn.edu younger individuals. Over-replacement with levothyroxine,
which has a narrow therapeutic window, should be avoided.
INTRODUCTION Subclinical hypothyroidism is identified more frequently than
Historical Overview (1, 2) overt hypothyroidism, and the risks and benefits of treatment of
The first suggestion that hypothyroidism was due to a defi- subclinical hypothyroidism have not been established in the
ciency in the thyroid gland occurred in the mid 1800s when elderly. A change in the reference range for older people may
two physicians independently reported sporadic cretinism in be required, which would have implications for the timing of
children without a goiter at a meeting of the Royal Medical and initiation of levothyroxine therapy and the treatment goals after
Chirurgical Society. In 1873, Sir William Gull presented a initiation.
report to the Society on five adult women with appearances
similar to cretinous children, and William Ord later reported on
LEARNING OBJECTIVES
five more adult women in 1877, coining the term myxoedema
As a result of participating in this session, learners should be
(mucinous edema), referring to the skin of these women at
able to:
autopsy. Initially attempted therapies included stimulants such
Recognize the signs and symptoms of hypothyroidism in
as arsenic, nitroglycerine, strychinine, and quinine, and the
the elderly.
plant extract jaborandi or its derived alkaloid pilocarpine.
Describe the evaluation and treatment of older patients
In 1883, E. Theodor Kocher independently coined the term
with hypothyroidism, both in ambulatory and
cachexia struipriva to describe a degenerative condition as-
hospitalized settings.
sociated with removal of the thyroid gland. He received the
Nobel Prize in Physiology and Medicine in 1909, the only Summarize current evidence regarding treatment of
awarded for work on the thyroid. Later in 1883, Sir Andrew subclinical hypothyroidism in the elderly.
Clark proposed that absence or probably complete degenera-
tion of the thyroid body was the common link between cre- STRATEGIES FOR DIAGNOSIS, THERAPY,
tinism, myxoedema, and the post-thyroidectomy syndrome. A AND/OR MANAGEMENT
committee was assembled of members of the Royal and Medical Signs and Symptoms of Hypothyroidism in the Elderly
and Chirurgical Society to investigate this observation. In 1888, The major symptoms of hypothyroidism in the elderly are
they issued a report in support of this hypothesis, but it was not neuromuscular, gastrointestinal, psychiatric, and cardiovascu-
until George Murray injected sheeps thyroid into a patient with lar. These can be insidious in onset. Cognitive and functional
myxoedema in 1891 that replacement therapy was considered. T4 decline are late manifestations of hypothyroidism. A study of
was first isolated in 1914 and subsequently synthesized by 54 overtly hypothyroid young (mean age, 41 y) and 67 elderly
Harington and Barger in 1928, followed by identification and (mean age, 79 y) overtly hypothyroid patients with similar
synthesis of T3 by Gross and Pitt-Rivers in 1954. duration of hypothyroidism and degree of TSH elevation ex-
amined the frequency of 24 signs and symptoms of hypothy-
SIGNIFICANCE OF THE CLINICAL PROBLEM roidism (3). Fatigue and weakness were found in more than
Overt hypothyroidism occurs in 12% of people age 65 years and 50% of the elderly patients, but no single sign or symptom was
older, and if unrecognized, can result in serious morbidity and pathognomonic for hypothyroidism (Table 1). Nonclassical
even death. The lack of specificity of symptoms and frequency symptoms of hypothyroidism were found more commonly in
of comorbid conditions presents special challenges to diagnosis the elderly groupreduced hearing, anorexia, pallor, weight
and management of hypothyroidism in older age groups. loss, and disorientation compared with the young group.
Whether mild elevations of TSH found in subclinical hypothy- There were, however, fewer clinical signs in the elderly pa-
roidism, which is present in approximately 10% of the elderly, tients than in the young patients (mean, 6.6 vs 9.3 signs).
represent thyroid disease or simply age-associated adaptive Hypercholesterolemia, hyponatremia, and macrocytic ane-
change is an area of active investigation. mia may also be present. More serious cardiac manifestations

The Endocrine Society. Downloaded from press.endocrine.org by [${individualUser.displayName}] on 12 January 2017. at 12:19 For personal use only. No other uses without permission. . All rights reserved.
356 ENDO 2016 MEET-THE-PROFESSOR CLINICAL CASE MANAGEMENT

TABLE 1. Clinical Features of Overt Hypothyroidism in TABLE 2. Causes of Hypothyroidism in the Elderly
Elderly vs. Young Patients Population
Elderly, Young, Primary hypothyroidism
Symptoms and Signs 70 Years (%) 55 Years (%) Chronic autoimmune thyroiditis (Hashimotos thyroiditis)
Fatigue 68 83 Radiation
131
Weakness 53 67 I therapy for hyperthyroidism
Mental slowness 45 48 Radiation therapy for head and neck cancer
Drowsiness 40 43 Surgical thyroidectomy
Cold intolerance 35 65 Drugs
Dry skin 35 45 Iodine-containing drugs: amiodarone, iodine and iodine-
Constipation 33 41 containing compounds
Reduced hearing 32 25 Antithyroid drugs (propythiouracil, methimazole)
Depression 28 52 Other drugs that decrease thyroid hormone secretion
Hoarseness 28 29 (lithium, aminoglutethimide, interferon , interleukin 2)
Edema 27 43 Tyrosine kinase inhibitors
Anorexia 27 13 Central hypothyroidism
Pallor 27 18 Hypothalamic tumors or infiltrative lesions
Weight gain 24 59 Pituitary tumors or infiltrative lesions
Hypoactive reflexes 24 31 Pituitary surgery
Muscle cramps 20 55 Radiation
Paresthesias 18 61 Drugs that decrease TSH secretion (bexarotene, octreotide)
Snoring 18 22
Ataxia 14 33
Weight loss 14 4 Management of Overt Hypothyroidism in Outpatients
Hair loss 12 28 Levothyroxine therapy can be initiated immediately after ob-
Bradycardia 12 19 taining a TSH concentration that shows significant elevation, at
Buzzing 11 26 20 mIU/L or higher. However, if the TSH concentration is less
Disorientation 9 0 than 20 mIU/L and the free T4 is normal, repeat evaluation 2 4
weeks later may be considered prior to levothyroxine initiation.
The starting dose of levothyroxine should be tailored to the
include pericardial effusion or heart failure. The effects of age, severity of hypothyroidism, and cardiac status of the
hypothyroidism are reversible with treatment. patient. Dessicated thyroid preparations or T3 should not be
used for thyroid hormone replacement in older people due to
Diagnosis of Hypothyroidism in the Elderly risk of adverse cardiac effects from supraphysiologic T3 con-
The most common form of hypothyroidism is primary hypo- centrations. In elderly patients with underlying cardiovascular
thyroidism, defined as a high TSH with low free T4 concentra- disease, it is generally recommended to start low, go slow.
tions. Rarely, central hypothyroidism may be found in conjunc- The rationale is to diminish risk of exacerbating angina or
tion with pituitiary or hypothalamic dysfunction, or bexarotene tachyarrthmias, with a tradeoff of a longer duration of under-
or octreotide use. In central hypothyroidism, TSH may be low, replacement and more frequent testing. One trial in patients
normal, or slightly high, with low free T4 concentrations. free of cardiovascular disease comparing an initial dose of 25
Measurement of T3 concentrations is not helpful in diagnosing g and gradual titration to an initial dose of 1.6 g/kg per day
hypothyroidism. Measurement of antithyroid antibodies is not showed faster normalization of thyroid function in the 1.6
required for diagnosis. g/kg per day group with no difference in symptoms (5).
Autoimmune thyroid dysfunction is the most common etiol- Though the number of patients age 65 years and older was
ogy of hypothyroidism in the elderly, particularly among small in this study, it provides evidence to support the safety of
women. Additional etiologies are listed in Table 2 [reprinted initiating a full replacement dose in healthy elderly patients
from Cappola (4)]. with known low cardiovascular risk.
The clinical context must be considered when evaluating With age, lean body mass and clearance of T4 decrease,
thyroid function tests. Otherwise, levothyroxine may be pre- resulting in an increase in half-life from 7 to 9 days in those
scribed as long-term therapy for a condition that spontaneously age 80 years and older. In patients with a functioning thyroid,
resolves. Transient hypothyroidism may occur during recovery there is a compensatory decline in T4 secretion, to maintain the
from thyroiditis or following an iodine load. Similarly, elevated euthyroid state. There are two implications of these age-
TSH with low free T4 may be found during recovery from associated changes. First, the time to reach steady state is
acute, nonthyroidal illness, with subsequent normalization. longer, and 6 8 weeks may be required between dose changes

The Endocrine Society. Downloaded from press.endocrine.org by [${individualUser.displayName}] on 12 January 2017. at 12:19 For personal use only. No other uses without permission. . All rights reserved.
ENDO 2016 THYROID/HPT AXIS 357

before TSH testing. Second, the ultimate dose requirement may sedatives should be avoided. Patients should be monitored
be lower than 1.6 g/kg per day. closely for neurobehavioral decompensation including erratic
The target TSH range in the elderly patient with hypothy- behavior, headaches, ataxia, nystagmus, and muscle spasms.
roidism has been debated, with recent guidelines recommend- Elderly inpatients who are taking levothyroxine at the time
ing a target TSH of 4 6 mIU/L in those who are over age of hospital admission should continue outpatient dosing. Ad-
70 80 years (6). Once TSH concentrations stabilize, TSH justment or initiation of levothyroxine should only be per-
monitoring need only be performed annually. formed in inpatients if the testing is grossly abnormal.
Stabilization of dosing may be more difficult in the older
population due to problems with adherence or concomitant
Subclinical Hypothyroidism in the Elderly
medications or conditions that affect levothyroxine dosing.
Subclinical hypothyroidism is defined as a high TSH with a
Medications that decrease levothyroxine absorption in-
normal free T4 concentration. The prevalence of subclinical
clude calcium supplements, cholestyramine, colestipol, aluminum
hypothyroidism increases from 4% in the general population to
hydroxide, sevelamer, ferrous sulfate, sucralfate, and raloxifene.
Separation by 4 hours from levothyroxine dosing is recom- 10% of those age 65 years and older. Most older individuals
mended. Decreased acidity from proton pump inhibitors and with subclinical hypothyroidism have TSH concentrations in
small intestinal disease also diminish levothyroxine absorption. the 4 7-mIU/L range, with over 50% in this range reverting to
Phenobarbitol, carbamazepine, phenytoin, and rifampin, in- euthyroidism on subsequent testing (8). Subclinical hypothy-
crease levothyroxine metabolism, and estrogens and hepatitis roidism with TSH of 7 mIU/L has not been associated with
increase T4 binding globulin, resulting in the need for higher any adverse outcomes. In meta-analyses of observational stud-
doses of levothyroxine. Initiation of androgens, liver failure, ies, subclinical hypothyroidism with a TSH of 10 mIU/L or
nephrotic syndrome, and severe systemic illness can result in higher has been associated with increased cardiovascular mor-
the need for lowering the dose. tality, coronary heart disease, and heart failure, and of 79.9
Iatrogenic hyperthyroidism should be avoided, due to risks mIU/L with increased cardiovascular mortality (9 10). These
of atrial fibrillation and bone loss. It has been reported that up associations do not differ by age. Subclinical hypothyroidism
to 40% of older individuals taking thyroid hormone replace- has not been associated with fractures or dementia.
ment have a TSH 0.5 mIU/L (7). Careful initial titration TSH testing should be repeated 13 months after initial
and subsequent monitoring are required to reduce the risk of
testing demonstrates subclinical hypothyroidism. If the repeat
over-replacement.
TSH is 7 mIU/L or higher, initiation of levothyroxine therapy
at 25 g daily could be considered, with careful monitoring to
Management of Overt Hypothyrioidism in Inpatients
prevent iatrogenic hyperthyroidism.
Myxedema coma occurs almost exclusively in older patients.
The usual presentation is longstanding untreated hypothyroid-
ism with an intervening precipitant such as infection, cold MAIN CONCLUSIONS
exposure, alcoholism, or use of narcotics, sedatives, or antipsy- Elderly patients with any of the classic symptoms of hypothyroidism,
chotic medication. Myxedema coma is characterized by pro- including fatigue and weakness, as well as nonclassical symptoms
found metabolic slowing, including hypothermia, bradycardia, such anorexia and weight loss, should be evaluated for hypo-
and hypoventilation. Clinical status may dictate initiation of thyroidism with a TSH test. Overt hypothyroidism should be
thyroid hormone replacement based on historical and clinical treated with levothyroxine therapy, with an initial dose ranging
data, before thyroid function test results are available. In the from 25 g to near-full replacement dose (1.6 g/kg/d) in
absence of thyroid function testing data, inappropriate admin- outpatients, depending on the underlying health, particularly
istration of large doses of thyroid hormone in a critically ill, cardiac status, of the patient. Concomitant medications or con-
euthyroid patient must be carefully weighed against delayed
ditions that affect levothyroxine absorption or metabolism
therapy in a profoundly hypothyroid patient. Treatment of
should be assessed. The interval between dose adjustments
precipitating factors and supportive therapy for associated met-
should be 6 8 weeks and TSH goals should be relaxed. Myx-
abolic disturbances should be initiated in parallel with initiation
edema coma is rare, but life threatening, and requires treatment
of thyroid hormone replacement. Impaired adrenal reserve may
be present. Serum should be collected for cortisol measurement of precipitating conditions, and initiation of hydrocortisone and
and hydrocortisone should be initiated prior to thyroid hormone a loading dose of levothyroxine. Subclinical hypothyroidism is
initiation, with cessation of hydrocortisone if the cortisol level common in elderly patients, although the benefits of treatment
is adequate. Levothyroxine 200 400 g should be initiated, are unproven and risks of iatrogenic hyperthyroidism of con-
followed by 75100 g daily. T3 therapy is generally discour- cern. Levothyroxine treatment should be considered for persis-
aged in older patients, due to concerns about acute cardiac tent subclinical hypothyroidism with TSH concentrations of 7
effects. The dosage of all medications should be carefully mIU/L or higher, although watchful waiting is also a reason-
considered in light of the profoundly slowed metabolism, and able approach.

The Endocrine Society. Downloaded from press.endocrine.org by [${individualUser.displayName}] on 12 January 2017. at 12:19 For personal use only. No other uses without permission. . All rights reserved.
358 ENDO 2016 MEET-THE-PROFESSOR CLINICAL CASE MANAGEMENT

CASES DISCUSSION OF CASES AND ANSWERS


Case 1 Case 1
A 77-year-old woman presents to your office with fatigue, The patient has symptoms and a physical examination that are
weakness, and weight loss of 4 kg from a baseline weight of 68 typical of an elderly patient with hypothyroidism, and testing
kg. Her laboratory testing shows a TSH of 58 mIU/L and free unequivocally shows overt hypothyroidism. The TSH concen-
T4 is below the limit of detection. She has a history of glau- tration is very elevated with concordant very low free T4
coma and a myocardial infarction 4 years earlier for which she concentrations. Answer C is correct. In light of her history of
had a stent placed. She denies angina. Her current medications coronary artery disease, initiation of 25 g with gradual titra-
are aspirin, 81 mg; metoprolol; atorvastatin; and latanoprost tion based on TSH levels is appropriate. Waiting and repeating
ophthalmic solution. On examination, her blood pressure is testing could be detrimental in this older patient (Answer A).
148/92 mm Hg, heart rate is 54 bpm, and weight is 64 kg. She Answer B is incorrect due to the timing of repeat laboratory
has periorbital edema, her thyroid is not enlarged, and she has testing. Therapy with T3 is not indicated (Answer D), and 75
a delayed relaxation phase of her deep tendon reflexes. g is too high a starting dose in this patient (Answer E).
Which of the following is the best next step(s) in her man-
agement? Case 2
A. No therapy indicated at this time. Repeat TSH and free This elderly woman has subclinical hypothyroidism. She has
T4 in 3 months depression as an alternative explanation for her symptoms
B. Initiate levothyroxine, 25 g daily and recheck TSH in along with a TSH that is below 7 mIU/L. Answer A is correct.
2 weeks Treatment for depression and watchful waiting are indicated.
C. Initiate levothyroxine, 25 g daily and recheck TSH in Initiation of thyroid hormone replacement therapy is not indi-
6 weeks cated (Answers B through E).
D. Initiate levothyroxine, 25 g daily and liothyronine 5
mcg daily and recheck TSH in 6 weeks REFERENCES
E. Initiate levothyroxine, 75 g daily and recheck TSH in 1. Sawin CT. Introduction: Defining myxoedema and its cause. In: The
6 weeks Clinical Society of London Report on Myxoedema, Boston, 1888 The
Francis A. Countway Library of Medicine. Science History Publications,
Canton, MA; 1991, 1-14.
Case 2 2. http://www.thyroid.org/about-american-thyroid-association/clark-t-sawin-
An 85-year-old woman presents to your office with depression, history-resource-center/thyroid-history-timeline. Accessed December 7,
fatigue, and weight loss. Her laboratory testing shows a TSH of 2015.
3. Doucet J, Trivalle C, Chassagne P, et al. Does age play a role in clinical
5.2 mIU/L with a free T4 of 1.09 ng/dL (0.821.77 ng/dL). She presentation of hypothyroidism? J Am Geriatr Soc. 1994;42(9):984-986.
has a history of hypertension, arthritis, and vertigo. Her current 4. Cappola AR. Thyroid Diseases. In: Halter J, Ouslander J, Studenski S,
medications are aspirin, 81 mg; and hydrocholorothiazide. On High K, Asthana S, Ritchie C, Supiano M, eds. Hazzards geriatric medi-
cine and gerontology. 7th ed. New York: McGraw-Hill Education, 2016.
physical examination, her blood pressure is 148/92 mm Hg, 5. Roos A, Linn-Rasker SP, van Domburg RT, Tijssen JP, Berghout A. The
heart rate is 62 bpm, and weight is 64 kg. She has mild starting dose of levothyroxine in primary hypothyroidism treatment: A
periorbital edema, her thyroid is not enlarged, and her deep prospective, randomized, double-blind trial. Arch Intern Med. 2005;165(15):
1714-1720.
tendon reflexes are diminished.
6. Jonklaas J, Bianco AC, Bauer AJ, et al. Guidelines for the treatment of
Which of the following is the best next step(s) in her man- hypothyroidism: prepared by the American Thyroid Association task force
agement? on thyroid hormone replacement. Thyroid. 2014;24(12):1670-1751.
A. No therapy indicated at this time. Repeat TSH and free 7. Somwaru LL, Arnold AM, Joshi N, Fried LP, Cappola AR. High fre-
quency of and factors associated with thyroid hormone over-replacement
T4 in 3 months and under-replacement in men and women aged 65 and over. J Clin
B. Initiate levothyroxine, 25 g daily and recheck TSH in Endocrinol Metab. 2009;94:1342-1345.
2 weeks 8. Somwaru LL, Somwaru LL, Arnold AM, Cappola AR. The natural history
of subclinical hypothyroidism in the elderly: The Cardiovascular Health
C. Initiate levothyroxine, 25 g daily and recheck TSH in Study. J Clin Endocrinol Metab. 2012;97(6):1962-1969.
6 weeks 9. Gencer B, Collet TH, Virgini V, et al. Subclinical thyroid dysfunction and
D. Initiate levothyroxine, 25 g daily and liothyronine 5 the risk of heart failure events: an individual participant data analysis from
6 prospective cohorts. Circulation. 2012;126(9):1040-1049.
mcg daily and recheck TSH in 6 weeks
10. Rodondi N, den Elzen WP, Bauer DC, et al. Subclinical hypothyroidism
E. Initiate levothyroxine 75 g daily and recheck TSH in and the risk of coronary heart disease and mortality. JAMA. 2010;304(12):
6 weeks 1365-1374.

The Endocrine Society. Downloaded from press.endocrine.org by [${individualUser.displayName}] on 12 January 2017. at 12:19 For personal use only. No other uses without permission. . All rights reserved.
ENDO 2016 THYROID/HPT AXIS 359

Levothyroxine Therapy: When Outcomes Are Less Than Optimal -


Optimizing Patient Care

M59 and abnormal thyroid function outcomes despite/or on alarm-


Presented, April 1 4, 2016 ingly high doses of LT4. This alerts the clinician that safe and
effective restoration of the euthyroid state has not been achieved
and barriers to achieving this goal are present. To provide accurate
James V. Hennessey, MD. Division of Endocrinology, and cost-effective care, physicians must identify and systemati-
Harvard Medical School, Boston, Massachusetts 02115, cally address issues as transparent but difficult to document as
E-mail: jhenness@bidmc.harvard.edu compliance, less obvious such as interactions with other drugs or
meals, and as diagnostically challenging as the identification of
INTRODUCTION variation in LT4 losses or absorption.
Historical Overview
A rich history of physician intervention in thyroid dysfunction LEARNING OBJECTIVES
dates back more than two millennia. Thyroid ingestion from As a result of participating in this session, the learner will be
animal sources had been used for centuries but was finally able to:
scientifically described and documented in Europe more than Articulate the pitfalls encountered in current practice of
130 years ago. Since these initial reports, there has been con- thyroid hormone replacement.
tinuous documentation of outcomes, refinement of hormone Outline successful strategies to identify the causes of
preparation production, and updating of recommendations for less than optimal thyroid status.
the most effective and safe use of these hormones for the relief Implement successful interventions to achieve improved
of symptoms of hypothyroidism (1). Initially, thyroid extract outcomes.
preparations containing both levothyroxine LT4 and LT3 were
the only preparations available, issues with the pharmacokinet-
ics of extract preparations and clinical management (2) lead the CASES AND DISCUSSION
Case 1
clinical community to replace these medications in general
A 65-year-old woman is seen because of a 3-month history
usage by LT4. Current guidelines do not endorse extract use,
of increasing fatigue, constipation, cold intolerance, and muscle
given that controlled studies do not clearly document enhanced
aches. She has been treated for hyperlipidemia with Atorvastatin,
objective outcomes when compared with LT4 monotherapy.
hypertension with hydrochlorothiazide. Lab shows TSH,
Continuous progress has been made in the administration of
11.8 mU/L (1), FT4, 0.6 ng/dL (2), thyroid peroxidase antibody
LT4 for the replacement of thyroid function (1).
(TPO-ab) positive.
1. Does LT4 treatment reliably restore euthyroidism?
SIGNIFICANCE OF THE CLINICAL PROBLEM
Although seemingly simple, the achievement of optimal clini-
cal outcomes with LT4 is quite variable. Both overdosage and Discussion
underdosage with LT4 have clinical ramifications that are at Several studies document less-than-optimal outcomes with LT4
odds with the expected restoration of euthyroidism and resolu- therapy in one third to one half of subjects treated with LT4.
tion of symptoms. Investigation into the root causes of this Consequences of suboptimal LT4 therapy may include cardio-
variability serves to teach us numerous lessons when faced vascular and orthopedic complications (3). Elderly subjects
with the patient with less-than-optimal outcomes after the ini- studied prospectively while on LT4 for hypothyroidism demon-
tiation of LT4. We typically assume compliance when assured strated an increased risk of new-onset thyrotoxicosis in just
by the patient and often seek additional sophisticated and under 10% of newly treated subjects and approximately 6% of
potentially expensive evaluations when dose increases fail to those continuing chronic LT4 treatment (4). Older women,
accomplish therapeutic goals in patients who continue to main- more likely to be started on LT4, represent most of those
tain cooperation with ingestion protocols. A systematic ap- becoming thyrotoxic (4). Thyrotoxicosis was observed in 1.5%
proach to this problem can be developed and should become of the longitudinal cohort overall [12 of the 22 subjects devel-
evident upon review of the evidence. oping thyrotoxicosis were on LT4 (4)] a 12-fold increased risk
vs no LT4 treatment.
BARRIERS TO OPTIMAL CARE
The list of clinical conditions and day-to-day issues that influ- Case 2
ence successful ingestion of a daily medication is long and A 41-year-old man presents for fatigue, facial puffiness, and
growing. Patients present with persistent ongoing symptoms bloating. His TSH is 21.88 mU/L (normal range, 0.4 4.12

The Endocrine Society. Downloaded from press.endocrine.org by [${individualUser.displayName}] on 12 January 2017. at 12:19 For personal use only. No other uses without permission. . All rights reserved.
360 ENDO 2016 MEET-THE-PROFESSOR CLINICAL CASE MANAGEMENT

mU/L), FT4, 0.6 ng/dL (normal range, 0.8-1.8 ng/dL), TPO-ab TABLE 1.
is positive at high titer. A decision is made to treat with LT4. Increased Metabolism
Impaired absorption (Clearance)
2. How much will restore a euthyroid state?
Aluminum Hydroxide Antacids Axitinib, Bosutinib, Crizotinib
Discussion Calcium Carbonate, Sucralfate Dasatinib, Erlotinib, Imitinib
Over the years, the mean doses of LT4 per kilogram have Binding Resins, Ciprofloxacin Lapatinib, Nilotinib,
decreased from the 2.0 mcg/kg range to 1.6-1.7 mcg/kg daily Pazopanib
(5, 6). Older individuals generally require less LT4, and HPLC Sodium Polystyrene Sulfonate Sorafenib, Sunitinib,
Vandetanib
standardization of LT4 products accounts for this decline (5, 6).
Colesevlam, Didanosine Barbiturates, Carbamazepine
Recent guidelines rely on the previously determined 1.6 1.7
Chromium Picolinate, Phenytoin
mcg/kg/d dosing as a good starting point (7). Case 2 goes on to Sevelamer
reveal poor control of thyroid function despite ever-increasing Magnesium Citrate, FeSO4 Mixed Mechanism
doses of LT4, which prompts the clinician to seek reasonable Lanthanum, Orlistat, Rifampin
explanations for this malabsorption of LT4. The first of these Raloxifene
is patient adherence with the treatment regimen (8). Up to 33%
of patients report being nonadherent for such reasons as they Epocrates Drug Resource Accessed 31 May 2013 @ 22:00.
doubt the efficacy of treatment, they are interested in saving
money, they have experienced relief of symptoms, or simply
because they forget to take their medications (8). Other reasons
reported include patients reluctance to follow medical advice, Medication Interactions
not following the directions on their prescriptions, and not A long list of drug interactions impair the absorption, increase the
understanding that continuous LT4 treatment for hypothyroid- metabolism (clearance), or have mixed mechanisms of disturbing
ism is required (9, 10). An additional factor involved in nearly the effectiveness of LT4 therapy. The top on the list of recognized
one third of persistent medication noncompliance include agents causing interference are iron, calcium, proton pump inhibi-
changes in the color or shape of medications as would be seen tors, and estrogens whereas statins, at least in one study, seemed to
with generic substitution (11). Steps should be taken to im- result in lower TSH values, and histamine blockers and the clini-
prove patient adherence including patient education about is- cal use of glucocorticoids seemed to have a neutral effect on the
sues involved in discontinuation of treatment, avoidance of outcomes of LT4 therapy (13).
unnecessary changes in treatment regimens, and incorporating
pharmacy follow-up services (8). Strategies for improving out-
Loss of Thyroid Hormone
comes include the use of a daily pill box (end of the week 7,
An alternative scenario of Case 2 finds our patient to have
pills for daily dosing), setting up a routine: eg, pills next to
negative antithyroid antibodies and LT4 treatment is found to
toothbrush or alarm clock, reminders on cell phones or
be less than optimal. The symptoms of facial puffiness and
watches, or posting notes on the refrigerator. Finally, weekly (7
weight gain are easily attributed to the unresolved hypothyroid-
times the daily dose) administration should be considered (8).
ism so dose escalation is usually routine. This pattern fits a
Case 3 seldom-recognized source of pseudomalabsorption, which
A 43-year-old woman diagnosed 3 years ago with hypothyroid- has been described for over 40 years (14). A loss of serum
ism with a TSH of 28 mU/L (1), FT4 of 0.5 ng/dL (2), proteins including thyroid-binding globulin through the kid-
TPO-ab, 438 (). Her TSH values over time vary, resulting in neys results in substantial loss of T4 in patients with nephrosis.
progressive LT4 dose changes. Factors other than noncompli- Hypothyroidism has been noted in patients with significant
ance may result in apparent malabsorption including medica- nephrotic range proteinuria, which is frequently associated with
tion or dietary interference with absorption. Examples of this failure of typical doses of LT4 to render and maintain patients
effect include the loss of suppressive LT4 effectiveness when in a euthyroid state. After considering the timing of LT4 inges-
comparing fasting and meal time ingestion of the LT4. The tion in relation to with meals or other medications, this easily-
effect of a United States Food and Drug Administration Stan- identified condition can be identified with an inexpensive and
dard Breakfast on LT4 absorption was demonstrated to be up to universally available assessment: urine protein. Resolution of
a 40% reduction in the absorption of the LT4 dose. Further the hypothyroidism has been documented when patients with
information on the effect of timing of LT4 administration reversible nephrotic syndrome were treated, indicating that the
reports higher TSH values when LT4 is ingested at bedtime or hypothyroidism resulted from the thyroid hormone losses in the
with breakfast compared with fasting for 60 minutes prior to urine (14, 15). This reason for excessive LT4 dosage offers a
breakfast (12). Another study of this problem confirmed these likely seldom-encountered but very important condition that
findings by demonstrating higher TSH values in those ingesting should be ruled out before embarking on an extensive and
their LT4 with breakfast instead of after an overnight fast. potentially expensive malabsorption workup.

The Endocrine Society. Downloaded from press.endocrine.org by [${individualUser.displayName}] on 12 January 2017. at 12:19 For personal use only. No other uses without permission. . All rights reserved.
ENDO 2016 THYROID/HPT AXIS 361

Pregnancy and LT4 without the prescribers knowledge and more than a quarter
The effect of pregnancy on LT4 effectiveness is well known indicated significant effect on the patients lives. Several re-
and has resulted in guideline recommendations that treated ports from pediatric endocrinologists complained of a loss of
hypothyroid patients (receiving LT4) who are newly pregnant the euthyroid state during a critical period of treatment for
should independently increase their dose of LT4 by 2530% congenital hypothyroidism (19).
upon a missed menstrual cycle or positive home pregnancy test The effect of diagnosis and intervention on clinical out-
and notify their caregiver promptly (16). Lower preconception comes in those with suboptimal LT4 outcomes indicates that in
TSH values (within the nonpregnant reference range) reduce 17 500 LT4-treated hypothyroid subjects, 1% were identified as
the risk of TSH elevation during the first trimester (16). having potential suboptimal outcomes on the basis of taking
greater than 225 mcg/d. Over 35% of these cases had no known
Gastrointestinal Disease or Surgery and LT4 explanation for such excessive doses while over 20% were
Gastrointestinal disease or surgery have been noted to have an found to have gastric parietal cell antibodies or medication
effect through the mechanism of malabsorption of LT4 or interference. Noncompliance was thought to account for more
maldissolution of LT4 tablets. As such, the presence of celiac than 15% of cases and celiac was identified in just under 5% of
disease, pancreatic insufficiency, obstructive liver disease, cir- patients (20). During a 2-year follow-up evaluation, significant
rhosis of the liver, and small bowel bacterial overgrowth have improvement in the mcg/kg dosing was noted among those
been reported to adversely influence success in LT4 treatment where medication interference had been identified. Smaller
(17). Maldissolution due to decreased HCL secretion is seen in improvements were also noted in those with gastric antibodies,
H. pylori infection and atrophic gastritis (17). celiac, and those with unknown etiology (better compliance?)
but among those thought to be overtly noncompliant, the
Adverse Drug Events mcg/kg dose disappointingly increased (20)!
Adequate patient monitoring can lead to fewer adverse drug
events (ADE) attributed to LT4 as demonstrated by a review of MAIN CONCLUSIONS
363 LT4-treated patients, 56% of whom were considered to be In summary, patients selected for LT4 therapy should have
guideline compliant in their followup. ADEs occurred in 1% of adequate initial dosing based on weight. Risk of adverse out-
well-monitored patients and 6% of poorly monitored patients. comes may be predicted by sex and age. Once LT4 therapy has
ADEs included depressive symptoms returning, unstable an- been started, poor control should lead the clinician to consider
gina, new-onset atrial fibrillation, and palpitations. After re- issues with compliance. Intervention with monitoring and com-
view most were classified as preventable (18). munication seem to enhance outcomes. A careful review of
history to identify interference by food intake and the co-
Case 4 ingestion of medications has demonstrated improvement in the
Our final case concerns a 55-year-old man with a 20-year precision of the LT4 treatment. It is reassuring that the duration
history of hypothyroidism presenting for his annual examina- of therapy seems to play a role in the accomplishment of better
tion. He has had no change in his nonthyroid medications but control but should this fail, unusual sources of LT4 malabsorp-
has a new complaint, a loss of energy. The patient has been tion or loss such as through the kidneys may be reversible with
successfully treated with 150 mcg/d LT4, for many years TSH, specific treatment. A more recently identified potential source
0.4-2.5 mU/L. The current test results show TSH level to be of less-than-optimal outcome may include the discovery of a
8.1 mU/L. What may have happened here? substitution of LT4 preparations with different appearance,
dissolution, and absorption characteristics.
Discussion
A pharmacovigilance survey of practicing endocrinologists REFERENCES
1. Hennessey JV. Historical and current perspective in the use of thyroid
controlled by eliminating reports issued while know factors extracts for the treatment of hypothyroidism. Endocr Pract. 2015;21(10):
interfering with LT4 outcomes were present reported ADEs in 1161-1170.
nearly 200 individuals (19). Before the reported adverse event, 2. Jackson IM, Cobb WE. Why does anyone still use desiccated thyroid
USP? Am J Med. 1978;64(2):284-288.
TSH values were in the expected range for patients being
3. Flynn RW, Bonellie SR, Jung RT, MacDonald TM, Morris AD, Leese GP.
treated for predominantly hypothyroidism. TSH values were Serum thyroid-stimulating hormone concentration and morbidity from
distributed very differently at the time of the event with evi- cardiovascular disease and fractures in patients on long-term thyroxine
therapy. J Clin Endocrinol Metab. 2010;95(1):186-193.
dence of both over- and under-dosage documented. Nearly
4. Mammen JS, McGready J, Oxman R, Chia CW, Ladenson PW, Simonsick
90% of these reports were associated with changes in the EM. Thyroid hormone therapy and risk of thyrotoxicosis in community-
source of LT4 supplied to the patient (19). Most of these resident older adults: Findings from the Baltimore Longitudinal Study of
changes were from name brand LT4 products to generic prepa- Aging. Thyroid. 2015;25(9):979-986.
5. Hennessey JV, Evaul JE, Tseng YC, Burman KD, Wartofsky L. L-
rations (potentially different shape and color). Nearly 92% thyroxine dosage: A reevaluation of therapy with contemporary prepara-
indicated that the patient had been switched at the pharmacy tions. Ann Intern Med. 1986;105:11-15.

The Endocrine Society. Downloaded from press.endocrine.org by [${individualUser.displayName}] on 12 January 2017. at 12:19 For personal use only. No other uses without permission. . All rights reserved.
362 ENDO 2016 MEET-THE-PROFESSOR CLINICAL CASE MANAGEMENT

6. Fish LH, Schwartz HL, Cavanaugh J, Steffes MW, Bantle JP, Op- 14. Fonseca V, Thomas M, Katrak A, Sweny P, Moorhead JF. Can urinary
penheimer JH. Replacement dose, metabolism, and bioavailability of thyroid hormone loss cause hypothyroidism? Lancet. 1991;338(8765):475-
levothyroxine in the treatment of hypothyroidism. Role of triiodothyronine 476.
in pituitary feedback in humans. N Engl J Med. 1987;316:764-770. 15. Benvenga S, Vita R, Di Bari F, Fallahi P, Antonelli A. Do not forget
7. Garber JR, Cobin RH, Gharib H, et al. Clinical practice guidelines for nephrotic syndrome as a cause of increased requirement of levothyroxine
hypothyroidism in adults: Cosponsored by the American Association of replacement therapy. Eur Thyroid J. 2015;4(2):138-142.
Clinical Endocrinologists and the American Thyroid Association. Thyroid. 16. Stagnaro-Green A, Abalovich M, Alexander E, et al. Guidelines of the
2012;22(12):1200-1235. American Thyroid Association for the diagnosis and management of thy-
8. Lips DJ, van Reisen MT, Voigt V, Venekamp W. Diagnosis and treatment roid disease during pregnancy and postpartum. Thyroid. 2011;21(10):1081-
1125.
of levothyroxine pseudomalabsorption. Neth J Med. 2004;62(4):114-118.
17. Checchi S, Montanaro A, Pasqui L, et al. L-thyroxine requirement in
9. Vaisman F, Coeli CM, Ward LS, et al. How good is the levothyroxine
patients with autoimmune hypothyroidism and parietal cell antibodies.
replacement in primary hypothyroidism patients in Brazil? Data of a
J Clin Endocrinol Metab. 2008;93(2):465-469.
multicentre study. J Endocrinol Invest. 2013;36(7):485-488.
18. Stelfox HT, Ahmed SB, Fiskio J, Bates DW. An evaluation of the ad-
10. Bagattoli RM, et al. Estudo de Adesao ao Tratamento do Hipotiroidismo. equacy of outpatient monitoring of thyroid replacement therapy. J Eval
Arq Braz Endocrinol Metab. 2000;44(2):483-487. Clin Pract. 2004;10(4):525-530.
11. Kesselheim AS, Choudhry NK, Avorn J. Burden of changes in generic pill 19. Hennessey JV, Malabanan AO, Haugen BR, Levy EG. Adverse event report-
appearance. Ann Intern Med. 2014;161(11):840. ing in patients treated with levothyroxine: Results of the pharmacovigilance
12. Bach-Huynh TG, Nayak B, Loh J, Soldin S, Jonklaas J. Timing of task force survey of the American Thyroid Association, American Association
levothyroxine administration affects serum thyrotropin concentration. of Clinical Endocrinologists, and the Endocrine Society. Endocr Pract.
J Clin Endocrinol Metab. 2009;94(10):3905-3912. 2010;16(3):357-370.
13. Irving SA, Vadiveloo T, Leese GP. Drugs that interact with levothyroxine: 20. Robertson HM, Narayanaswamy AK, Pereira O, et al. Factors contributing
An observational study from the Thyroid Epidemiology, Audit and Re- to high levothyroxine doses in primary hypothyroidism: An interventional
search Study (TEARS). Clin Endocrinol (Oxf). 2015;82(1):136-141. audit of a large community database. Thyroid. 2014;24(12):1765-1771.

The Endocrine Society. Downloaded from press.endocrine.org by [${individualUser.displayName}] on 12 January 2017. at 12:19 For personal use only. No other uses without permission. . All rights reserved.

You might also like